Você está na página 1de 327

SASDH Niterói

Psicólogo

Aspectos Gerais – Elaboração de informes psicológicos (de acordo com legislação em vigor no
Conselho Federal de Psicologia). ............................................................................................................ 1
Ética profissional. .............................................................................................................................. 11
Resoluções do CFP (Resolução CFP nº 13/2018, CFP nº 09/2018, CFP N.º 001/2018). .................. 17
O Estatuto da Criança e do Adolescente e a proteção integral à Infância e à Juventude. Lei nº 8.069/90
- Estatuto da Criança e do Adolescente e suas alterações; ................................................................... 28
Crianças e adolescentes em situação de acolhimento institucional e familiar. .................................. 83
Orientações Técnicas: Serviços de Acolhimento para Crianças e Adolescentes. Ministério do
Desenvolvimento Social e Combate à Fome, Conselho Nacional de Assistência Social e Conselho
Nacional dos Direitos da Criança e do Adolescente. Aolescentes em conflito com a lei e as medidas
socioeducativas. ................................................................................................................................... 122
Lei nº 12.594/12 – SINASE. ............................................................................................................ 137
O Estatuto do Idoso (Lei nº 10.741, de 1º outubro de 2003) e Política Nacional do Idoso (Lei nº 8.842,
de 4 de janeiro de 1994). ..................................................................................................................... 163
Violência intrafamiliar: conceito, diagnóstico e intervenção. ............................................................ 189
Técnicas de Resolução de Conflitos e Comunicação Não Violenta. ............................................... 202
Grupo social e familiar: influência da família e da sociedade no desenvolvimento da personalidade e
modalidades de intervenção. Norma, disciplina e poder. ..................................................................... 210
Psicologia, Direitos Humanos e Democracia. .................................................................................. 226
Psicologia social. ............................................................................................................................ 239
Sociedade e processos de exclusão e segregação. ........................................................................ 248
Criminalização e produção de subjetividades. ................................................................................ 251
Drogas, assistência e cidadania. ..................................................................................................... 258
Sexualidade, gênero e identidade. .................................................................................................. 269
Qualidade de vida na velhice. ......................................................................................................... 285
Desenvolvimento humano: fases, influências. ................................................................................. 288
O papel do psicólogo em equipe multidisciplinar. ............................................................................ 322

Apostila gerada especialmente para: Betânia Mueller 023.712.580-32


. 1
Candidatos ao Concurso Público,
O Instituto Maximize Educação disponibiliza o e-mail professores@maxieduca.com.br para dúvidas
relacionadas ao conteúdo desta apostila como forma de auxiliá-los nos estudos para um bom
desempenho na prova.
As dúvidas serão encaminhadas para os professores responsáveis pela matéria, portanto, ao entrar
em contato, informe:
- Apostila (concurso e cargo);
- Disciplina (matéria);
- Número da página onde se encontra a dúvida; e
- Qual a dúvida.
Caso existam dúvidas em disciplinas diferentes, por favor, encaminhá-las em e-mails separados. O
professor terá até cinco dias úteis para respondê-la.
Bons estudos!

Apostila gerada especialmente para: Betânia Mueller 023.712.580-32


. 2
Aspectos Gerais – Elaboração de informes psicológicos (de acordo com
legislação em vigor no Conselho Federal de Psicologia).

Caro(a) candidato(a), antes de iniciar nosso estudo, queremos nos colocar à sua disposição, durante
todo o prazo do concurso para auxiliá-lo em suas dúvidas e receber suas sugestões. Muito zelo e técnica
foram empregados na edição desta obra. No entanto, podem ocorrer erros de digitação ou dúvida
conceitual. Em qualquer situação, solicitamos a comunicação ao nosso serviço de atendimento ao cliente
para que possamos esclarecê-lo. Entre em contato conosco pelo e-mail: professores@maxieduca.com.br

Elaboração de parecer e laudo psicológico;

Um laudo pericial psicológico deve estar pautado em aspectos éticos, legais e psicológicos que
permitam informações que auxiliem o Juiz e contribua para uma justiça.
O laudo pericial consiste em um documento que será elaborado pelo perito ao final de um processo
de avaliação.
Este documento é parecido com o formato de um laudo psicológico em uma avaliação clínica, porém
diferenciando-se em algumas peculiaridades.
De forma resumida, este laudo pericial será composto pelos dados de identificação do avaliando, pelos
métodos e procedimentos utilizados pelo perito, seus achados e discussão sobre os mesmos e, por fim,
por uma breve conclusão.
Apesar de ser considerado um meio de prova, o laudo pericial não se constitui em uma verdade
absoluta e, consequentemente, é passível de crítica e questionamento.
Sob o ponto de vista legal, esta é uma das principais questões que o perito deverá atentar-se.
Conforme os itens contidos no artigo 2º do CEPP, é vedado ao psicólogo: (Alíneas)
g) Emitir documentos sem fundamentação e qualidade técnico-científica;
h) Interferir na validade e fidedignidade de instrumentos e técnicas psicológicas, adulterar resultados
ou fazer declarações falsas.
Estas são algumas das observações dispostas no Código de Ética do Psicólogo, juntamente com o
contido no artigo 147 do Código de Processo Civil que influenciarão de forma direta na elaboração e
disposição de um laudo psicológico jurídico ou forense.
No laudo psicológico jurídico ou forense deverá constar dados extremamente objetivos e com alto grau
de precisão e clareza na discussão de seus achados, fundamentados teoricamente para que se possa
justificar a conclusão e principalmente evitar possíveis sansões administrativas ao profissional, em caso
de não observação destas considerações legais e deontológicas.
Outra importante consideração a ser feita ao redigir o laudo diz respeito ao conteúdo apresentado
neste documento.
O que é possível revelar em um laudo psicológico forense? Como fica a questão do sigilo profissional
do Psicólogo?
O sigilo das informações deve ser observado nas comunicações orais ou escritas com outros
profissionais, com a imprensa ou autoridades.
São quatro tipos de situações, nas quais este sigilo poderá ser quebrado:
a) pelo próprio consentimento;
b) dever legal, a fim de evitar a propagação de moléstias;
c) em risco de suicídio;
e) em justas causas, cujo significado prático versa sobre situações em que o sigilo deve ser sacrificado
em benefício de outro direito como por exemplo, a vida ou a saúde de outra pessoa ou da sociedade.
Uma solução recomendada pelos especialistas da área, em que o texto apresentado no laudo é se
limitar às questões pertinentes a pergunta formulada pelo juiz ou pelos advogados.
Assim, a presença de todo conteúdo que será apresentado no laudo será justificado pela necessidade
de responder (e somente responder) a questão inicialmente requerida.

RESOLUÇÃO CFP N.º 007/2003

Institui o Manual de Elaboração de Documentos Escritos produzidos pelo psicólogo, decorrentes de


avaliação psicológica e revoga a Resolução CFP º 17/2002.

Apostila gerada especialmente para: Betânia Mueller 023.712.580-32


. 1
O CONSELHO FEDERAL DE PSICOLOGIA, no uso de suas atribuições legais e regimentais, que lhe
são conferidas pela Lei nº 5.766, de 20 de dezembro de 1971;

CONSIDERANDO que o psicólogo, no seu exercício profissional, tem sido solicitado a apresentar
informações documentais com objetivos diversos;

CONSIDERANDO a necessidade de referências para subsidiar o psicólogo na produção qualificada


de documentos escritos decorrentes de avaliação psicológica;

CONSIDERANDO a frequência com que representações éticas são desencadeadas a partir de queixas
que colocam em questão a qualidade dos documentos escritos, decorrentes de avaliação psicológica,
produzidos pelos psicólogos;

CONSIDERANDO os princípios éticos fundamentais que norteiam a atividade profissional do psicólogo


e os dispositivos sobre avaliação psicológica contidos no Código de Ética Profissional do Psicólogo;

CONSIDERANDO as implicações sociais decorrentes da finalidade do uso dos documentos escritos


pelos psicólogos a partir de avaliações psicológicas;

CONSIDERANDO as propostas encaminhadas no I FORUM NACIONAL DE AVALIAÇÃO


PSICOLÓGICA, ocorrido em dezembro de 2000;

CONSIDERANDO a deliberação da Assembleia das Políticas Administrativas e Financeiras, em


reunião realizada em 14 de dezembro de 2002, para tratar da revisão do Manual de Elaboração de
Documentos produzidos pelos psicólogos, decorrentes de avaliações psicológicas;

CONSIDERANDO a decisão deste Plenário em sessão realizada no dia 14 de junho de 2003,

RESOLVE:

Art. 1º - Instituir o Manual de Elaboração de Documentos Escritos, produzidos por psicólogos,


decorrentes de avaliações psicológicas.

Art. 2º - O Manual de Elaboração de Documentos Escritos, referido no artigo anterior, dispõe sobre os
seguintes itens:
I. Princípios norteadores;
II. Modalidades de documentos;
III. Conceito / finalidade / estrutura;
IV. Validade dos documentos;
V. Guarda dos documentos.

Art. 3º - Toda e qualquer comunicação por escrito decorrente de avaliação psicológica deverá seguir
as diretrizes descritas neste manual.
Parágrafo único – A não observância da presente norma constitui falta ético disciplinar, passível de
capitulação nos dispositivos referentes ao exercício profissional do Código de Ética Profissional do
Psicólogo, sem prejuízo de outros que possam ser arguidos.

Art. 4º - Esta resolução entrará em vigor na data de sua publicação.

Art. 5º - Revogam-se as disposições em contrário.

Brasília, 14 de junho de 2003.

ODAIR FURTADO
Conselheiro Presidente

Apostila gerada especialmente para: Betânia Mueller 023.712.580-32


. 2
MANUAL DE ELABORAÇÃO DE DOCUMENTOS DECORRENTES DE AVALIAÇÕES
PSICOLÓGICAS CONSIDERAÇÕES INICIAIS

A avaliação psicológica é entendida como o processo técnico-científico de coleta de dados, estudos e


interpretação de informações a respeito dos fenômenos psicológicos, que são resultantes da relação do
indivíduo com a sociedade, utilizando-se, para tanto, de estratégias psicológicas – métodos, técnicas e
instrumentos. Os resultados das avaliações devem considerar e analisar os condicionantes históricos e
sociais e seus efeitos no psiquismo, com a finalidade de servirem como instrumentos para atuar não
somente sobre o indivíduo, mas na modificação desses condicionantes que operam desde a formulação
da demanda até a conclusão do processo de avaliação psicológica.

O presente Manual tem como objetivos orientar o profissional psicólogo na confecção de documentos
decorrentes das avaliações psicológicas e fornecer os subsídios éticos e técnicos necessários para a
elaboração qualificada da comunicação escrita.
As modalidades de documentos aqui apresentadas foram sugeridas durante o I FÓRUM NACIONAL
DE AVALIAÇÃO PSICOLÓGICA, ocorrido em dezembro de 2000.

Este Manual compreende os seguintes itens:


I. Princípios norteadores da elaboração documental;
II. Modalidades de documentos;
III. Conceito / finalidade / estrutura;
IV. Validade dos documentos;
V. Guarda dos documentos.

I - PRINCÍPIOS NORTEADORES NA ELABORAÇÃO DE DOCUMENTOS

O psicólogo, na elaboração de seus documentos, deverá adotar como princípios norteadores as


técnicas da linguagem escrita e os princípios éticos, técnicos e científicos da profissão.

1 – PRINCÍPIOS TÉCNICOS DA LINGUAGEM ESCRITA

O documento deve, na linguagem escrita, apresentar uma redação bem estruturada e definida,
expressando o que se quer comunicar. Deve ter uma ordenação que possibilite a compreensão por quem
o lê, o que é fornecido pela estrutura, composição de parágrafos ou frases, além da correção gramatical.
O emprego de frases e termos deve ser compatível com as expressões próprias da linguagem
profissional, garantindo a precisão da comunicação, evitando a diversidade de significações da linguagem
popular, considerando a quem o documento será destinado.
A comunicação deve ainda apresentar como qualidades: a clareza, a concisão e a harmonia. A clareza
se traduz, na estrutura frasal, pela sequência ou ordenamento adequado dos conteúdos, pela explicitação
da natureza e função de cada parte na construção do todo. A concisão se verifica no emprego da
linguagem adequada, da palavra exata e necessária. Essa “economia verbal” requer do psicólogo a
atenção para o equilíbrio que evite uma redação lacônica ou o exagero de uma redação prolixa.
Finalmente, a harmonia se traduz na correlação adequada das frases, no aspecto sonoro e na ausência
de cacofonias.

2 – PRINCÍPIOS ÉTICOS E TÉCNICOS

2.1.Princípios Éticos
Na elaboração de DOCUMENTO, o psicólogo baseará suas informações na observância dos princípios
e dispositivos do Código de Ética Profissional do Psicólogo.
Enfatizamos aqui os cuidados em relação aos deveres do psicólogo nas suas relações com a pessoa
atendida, ao sigilo profissional, às relações com a justiça e ao alcance das informações - identificando
riscos e compromissos em relação à utilização das informações presentes nos documentos em sua
dimensão de relações de poder.
Torna-se imperativo a recusa, sob toda e qualquer condição, do uso dos instrumentos, técnicas
psicológicas e da experiência profissional da Psicologia na sustentação de modelos institucionais e
ideológicos de perpetuação da segregação aos diferentes modos de subjetivação. Sempre que o trabalho
exigir, sugere-se uma intervenção sobre a própria demanda e a construção de um projeto de trabalho que
aponte para a reformulação dos condicionantes que provoquem o sofrimento psíquico, a violação dos

Apostila gerada especialmente para: Betânia Mueller 023.712.580-32


. 3
direitos humanos e a manutenção das estruturas de poder que sustentam condições de dominação e
segregação.
Deve-se realizar uma prestação de serviço responsável pela execução de um trabalho de qualidade
cujos princípios éticos sustentam o compromisso social da Psicologia.
Dessa forma, a demanda, tal como é formulada, deve ser compreendida como efeito de uma situação
de grande complexidade.

2.2. Princípios Técnicos


O processo de avaliação psicológica deve considerar que os objetos deste procedimento (as questões
de ordem psicológica) têm determinações históricas, sociais, econômicas e políticas, sendo as mesmas
elementos constitutivos no processo de subjetivação.

O DOCUMENTO, portanto, deve considerar a natureza dinâmica, não definitiva e não cristalizada do
seu objeto de estudo.
Os psicólogos, ao produzirem documentos escritos, devem se basear exclusivamente nos
instrumentais técnicos (entrevistas, testes, observações, dinâmicas de grupo, escuta, intervenções
verbais) que se configuram como métodos e técnicas psicológicas para a coleta de dados, estudos e
interpretações de informações a respeito da pessoa ou grupo atendidos, bem como sobre outros materiais
e grupo atendidos e sobre outros materiais e documentos produzidos anteriormente e pertinentes à
matéria em questão. Esses instrumentais técnicos devem obedecer às condições mínimas requeridas de
qualidade e de uso, devendo ser adequados ao que se propõem a investigar.
A linguagem nos documentos deve ser precisa, clara, inteligível e concisa, ou seja, deve-se restringir
pontualmente às informações que se fizerem necessárias, recusando qualquer tipo de consideração que
não tenha relação com a finalidade do documento específico.
Deve-se rubricar as laudas, desde a primeira até a penúltima, considerando que a última estará
assinada, em toda e qualquer modalidade de documento.

II - MODALIDADES DE DOCUMENTOS

1. Declaração *
2. Atestado psicológico
3. Relatório / laudo psicológico
4. Parecer psicológico *
* A Declaração e o Parecer psicológico não são documentos decorrentes da avaliação Psicológica,
embora muitas vezes apareçam desta forma. Por isso consideramos importante constarem deste manual
afim de que sejam diferenciados.

III - CONCEITO / FINALIDADE / ESTRUTURA

1 – DECLARAÇÃO

1.1. Conceito e finalidade da declaração


É um documento que visa a informar a ocorrência de fatos ou situações objetivas relacionados ao
atendimento psicológico, com a finalidade de declarar:
a) Comparecimentos do atendido e/ou do seu acompanhante, quando necessário;
b) Acompanhamento psicológico do atendido;
c) Informações sobre as condições do atendimento (tempo de acompanhamento, dias ou horários).

Neste documento não deve ser feito o registro de sintomas, situações ou estados psicológicos.

1.2. Estrutura da declaração


a) Ser emitida em papel timbrado ou apresentar na subscrição do documento o carimbo, em que conste
nome e sobrenome do psicólogo, acrescido de sua inscrição profissional (“Nome do psicólogo / N.º da
inscrição”).
b) A declaração deve expor:
- Registro do nome e sobrenome do solicitante;
- Finalidade do documento (por exemplo, para fins de comprovação);
- Registro de informações solicitadas em relação ao atendimento (por exemplo: se faz
acompanhamento psicológico, em quais dias, qual horário);

Apostila gerada especialmente para: Betânia Mueller 023.712.580-32


. 4
- Registro do local e data da expedição da declaração;
- Registro do nome completo do psicólogo, sua inscrição no CRP e/ou carimbo com as mesmas
informações.
Assinatura do psicólogo acima de sua identificação ou do carimbo.

2 – ATESTADO PSICOLÓGICO

2.1. Conceito e finalidade do atestado


É um documento expedido pelo psicólogo que certifica uma determinada situação ou estado
psicológico, tendo como finalidade afirmar sobre as condições psicológicas de quem, por requerimento,
o solicita, com fins de:
a) Justificar faltas e/ou impedimentos do solicitante;
b) Justificar estar apto ou não para atividades específicas, após realização de um processo de
avaliação psicológica, dentro do rigor técnico e ético que subscreve esta Resolução;
c) Solicitar afastamento e/ou dispensa do solicitante, subsidiado na afirmação atestada do fato, em
acordo com o disposto na Resolução CFP nº 015/96.

2.2. Estrutura do atestado


A formulação do atestado deve restringir-se à informação solicitada pelo requerente, contendo
expressamente o fato constatado. Embora seja um documento simples, deve cumprir algumas
formalidades:
a) Ser emitido em papel timbrado ou apresentar na subscrição do documento o carimbo, em que conste
o nome e sobrenome do psicólogo, acrescido de sua inscrição profissional (“Nome do psicólogo / N.º da
inscrição”).
b) O atestado deve expor:
- Registro do nome e sobrenome do cliente;
- Finalidade do documento;
- Registro da informação do sintoma, situação ou condições psicológicas que justifiquem o
atendimento, afastamento ou falta – podendo ser registrado sob o indicativo do código da Classificação
Internacional de Doenças em vigor;
- Registro do local e data da expedição do atestado;
- Registro do nome completo do psicólogo, sua inscrição no CRP e/ou carimbo com as mesmas
informações;
- Assinatura do psicólogo acima de sua identificação ou do carimbo.

Os registros deverão estar transcritos de forma corrida, ou seja, separados apenas pela pontuação,
sem parágrafos, evitando, com isso, riscos de adulterações. No caso em que seja necessária a utilização
de parágrafos, o psicólogo deverá preencher esses espaços com traços.
O atestado emitido com a finalidade expressa no item 2.1, alínea b, deverá guardar relatório
correspondente ao processo de avaliação psicológica realizado, nos arquivos profissionais do psicólogo,
pelo prazo estipulado nesta resolução, item V.

3 – RELATÓRIO PSICOLÓGICO

3.1. Conceito e finalidade do relatório ou laudo psicológico


O relatório ou laudo psicológico é uma apresentação descritiva acerca de situações e/ou condições
psicológicas e suas determinações históricas, sociais, políticas e culturais, pesquisadas no processo de
avaliação psicológica. Como todo DOCUMENTO, deve ser subsidiado em dados colhidos e analisados,
à luz de um instrumental técnico (entrevistas, dinâmicas, testes psicológicos, observação, exame
psíquico, intervenção verbal), consubstanciado em referencial técnico-filosófico e científico adotado pelo
psicólogo.
A finalidade do relatório psicológico será a de apresentar os procedimentos e conclusões gerados pelo
processo da avaliação psicológica, relatando sobre o encaminhamento, as intervenções, o diagnóstico, o
prognóstico e evolução do caso, orientação e sugestão de projeto terapêutico, bem como, caso
necessário, solicitação de acompanhamento psicológico, limitando-se a fornecer somente as informações
necessárias relacionadas à demanda, solicitação ou petição.

3.2. Estrutura

Apostila gerada especialmente para: Betânia Mueller 023.712.580-32


. 5
O relatório psicológico é uma peça de natureza e valor científicos, devendo conter narrativa detalhada
e didática, com clareza, precisão e harmonia, tornando-se acessível e compreensível ao destinatário. Os
termos técnicos devem, portanto, estar acompanhados das explicações e/ou conceituação retiradas dos
fundamentos teórico-filosóficos que os sustentam.
O relatório psicológico deve conter, no mínimo, 5 (cinco) itens: identificação, descrição da demanda,
procedimento, análise e conclusão.

1.Identificação
2.Descrição da demanda
3. Procedimento
4. Análise
5. Conclusão

3.2.1. Identificação
É a parte superior do primeiro tópico do documento com a finalidade de identificar:
O autor/relator – quem elabora;
O interessado – quem solicita;
O assunto/finalidade – qual a razão/finalidade.

No identificador AUTOR/RELATOR, deverá ser colocado o(s) nome(s) do(s) psicólogo(s) que
realizará(ão) a avaliação, com a(s) respectiva(s) inscrição(ões) no Conselho Regional.
No identificador INTERESSADO, o psicólogo indicará o nome do autor do pedido (se a solicitação foi
da Justiça, se foi de empresas, entidades ou do cliente).
No identificador ASSUNTO, o psicólogo indicará a razão, o motivo do pedido (se para
acompanhamento psicológico, prorrogação de prazo para acompanhamento ou outras razões pertinentes
a uma avaliação psicológica).

3.2.2. Descrição da demanda


Esta parte é destinada à narração das informações referentes à problemática apresentada e dos
motivos, razões e expectativas que produziram o pedido do documento.
Nesta parte, deve-se apresentar a análise que se faz da demanda de forma a justificar o procedimento
adotado.

3.2.3. Procedimento
A descrição do procedimento apresentará os recursos e instrumentos técnicos utilizados para coletar
as informações (número de encontros, pessoas ouvidas etc) à luz do referencial teórico-filosófico que os
embasa. O procedimento adotado deve ser pertinente para avaliar a complexidade do que está sendo
demandado.

3.2.4. Análise
É a parte do documento na qual o psicólogo faz uma exposição descritiva de forma metódica, objetiva
e fiel dos dados colhidos e das situações vividas relacionados à demanda em sua complexidade. Como
apresentado nos princípios técnicos, “O processo de avaliação psicológica deve considerar que os objetos
deste procedimento (as questões de ordem psicológica) têm determinações históricas, sociais,
econômicas e políticas, sendo as mesmas elementos constitutivos no processo de subjetivação. O
DOCUMENTO, portanto, deve considerar a natureza dinâmica, não definitiva e não cristalizada do seu
objeto de estudo”.
Nessa exposição, deve-se respeitar a fundamentação teórica que sustenta o instrumental técnico
utilizado, bem como princípios éticos e as questões relativas ao sigilo das informações. Somente deve
ser relatado o que for necessário para o esclarecimento do encaminhamento, como disposto no Código
de Ética Profissional do Psicólogo.
O psicólogo, ainda nesta parte, não deve fazer afirmações sem sustentação em fatos e/ou teorias,
devendo ter linguagem precisa, especialmente quando se referir a dados de natureza subjetiva,
expressando-se de maneira clara e exata.

3.2.4. Conclusão
Na conclusão do documento, o psicólogo vai expor o resultado e/ou considerações a respeito de sua
investigação a partir das referências que subsidiaram o trabalho. As considerações geradas pelo processo

Apostila gerada especialmente para: Betânia Mueller 023.712.580-32


. 6
de avaliação psicológica devem transmitir ao solicitante a análise da demanda em sua complexidade e
do processo de avaliação psicológica como um todo.
Vale ressaltar a importância de sugestões e projetos de trabalho que contemplem a complexidade das
variáveis envolvidas durante todo o processo.
Após a narração conclusiva, o documento é encerrado, com indicação do local, data de emissão,
assinatura do psicólogo e o seu número de inscrição no CRP.

4 – PARECER

4.1. Conceito e finalidade do parecer


Parecer é um documento fundamentado e resumido sobre uma questão focal do campo psicológico
cujo resultado pode ser indicativo ou conclusivo.
O parecer tem como finalidade apresentar resposta esclarecedora, no campo do conhecimento
psicológico, através de uma avaliação especializada, de uma “questão-problema”, visando a dirimir
dúvidas que estão interferindo na decisão, sendo, portanto, uma resposta a uma consulta, que exige de
quem responde competência no assunto.

4.2. Estrutura
O psicólogo parecerista deve fazer a análise do problema apresentado, destacando os aspectos
relevantes e opinar a respeito, considerando os quesitos apontados e com fundamento em referencial
teórico-científico.
Havendo quesitos, o psicólogo deve respondê-los de forma sintética e convincente, não deixando
nenhum quesito sem resposta. Quando não houver dados para a resposta ou quando o psicólogo não
puder ser categórico, deve-se utilizar a expressão “sem elementos de convicção”. Se o quesito estiver
mal formulado, pode-se afirmar “prejudicado”, “sem elementos” ou “aguarda evolução”.

O parecer é composto de 4 (quatro) itens:


1. Identificação
2. Exposição de motivos
3. Análise
4. Conclusão

4.2.1. Identificação
Consiste em identificar o nome do parecerista e sua titulação, o nome do autor da solicitação e sua
titulação.

4.2.2. Exposição de Motivos


Destina-se à transcrição do objetivo da consulta e dos quesitos ou à apresentação das dúvidas
levantadas pelo solicitante. Deve-se apresentar a questão em tese, não sendo necessária, portanto, a
descrição detalhada dos procedimentos, como os dados colhidos ou o nome dos envolvidos.

4.2.3. Análise
A discussão do PARECER PSICOLÓGICO se constitui na análise minuciosa da questão explanada e
argumentada com base nos fundamentos necessários existentes, seja na ética, na técnica ou no corpo
conceitual da ciência psicológica. Nesta parte, deve respeitar as normas de referências de trabalhos
científicos para suas citações e informações.

4.2.4. Conclusão
Na parte final, o psicólogo apresentará seu posicionamento, respondendo à questão levantada. Em
seguida, informa o local e data em que foi elaborado e assina o documento.

V – VALIDADE DOS CONTEÚDOS DOS DOCUMENTOS

O prazo de validade do conteúdo dos documentos escritos, decorrentes das avaliações psicológicas,
deverá considerar a legislação vigente nos casos já definidos. Não havendo definição legal, o psicólogo,
onde for possível, indicará o prazo de validade do conteúdo emitido no documento em função das
características avaliadas, das informações obtidas e dos objetivos da avaliação.
Ao definir o prazo, o psicólogo deve dispor dos fundamentos para a indicação, devendo apresentá-los
sempre que solicitado.

Apostila gerada especialmente para: Betânia Mueller 023.712.580-32


. 7
VI - GUARDA DOS DOCUMENTOS E CONDIÇÕES DE GUARDA

Os documentos escritos decorrentes de avaliação psicológica, bem como todo o material que os
fundamentou, deverão ser guardados pelo prazo mínimo de 5 anos, observando-se a responsabilidade
por eles tanto do psicólogo quanto da instituição em que ocorreu a avaliação psicológica.
Esse prazo poderá ser ampliado nos casos previstos em lei, por determinação judicial, ou ainda em
casos específicos em que seja necessária a manutenção da guarda por maior tempo.
Em caso de extinção de serviço psicológico, o destino dos documentos deverá seguir as orientações
definidas no Código de Ética do Psicólogo.

Questões

01. (DPE/AM - Analista Social de Defensoria - FCC/2018). Segundo a Resolução CFP n° 007/2003
e o Manual de Elaboração de Documentos Escritos,
(A) o parecer é composto de quatro elementos básicos: identificação, exposição de motivos, análise e
conclusão.
(B) o parecer é um documento de avaliação psicológica que exige a realização de um psicodiagnóstico
detalhado para se obter respostas precisas no campo do conhecimento psicológico.
(C) são três as modalidades de documentos decorrentes da avaliação psicológica: a declaração, o
laudo e o parecer.
(D) nos documentos que embasam as atividades em equipe multiprofissional, o psicólogo registrará
todas as informações que tenha obtido de seus pacientes.
(E) a elaboração do laudo deve contemplar apenas três itens em sua estrutura: descrição da demanda,
análise e conclusão.

02. (EBSERH - Psicólogo - IBFC/2017). De acordo com o Manual de elaboração de documentos


escritos produzidos pelo psicólogo decorrentes de avaliação psicológica, assinale a alternativa incorreta
a respeito da avaliação psicológica.
(A) A avaliação é entendida como o processo técnico-científico de coleta de dados, estudos e
interpretação de informações a respeito dos fenômenos psicológicos, que são resultantes da relação do
indivíduo com a sociedade
(B) O resultado da avaliação tem com finalidade servir como instrumentos para atuar exclusivamente
sobre o indivíduo
(C) Para realizar uma avaliação, é necessária a utilização de estratégias psicológicas como métodos,
técnicas e instrumentos
(D) Os resultados das avaliações devem considerar e analisar os condicionantes históricos e sociais e
seus efeitos no psiquismo
(E) O processo de avaliação pode atuar na modificação de condicionantes que operam desde a
formulação da demanda até a conclusão do processo de avaliação psicológica.

03. (TRE/SP - Analista Judiciário - FCC/2017). A Resolução CFP nº 007/2003 instituiu o Manual de
Elaboração de Documentos Escritos produzidos pelos psicólogos a partir de avaliações psicológicas. O
relatório ou o laudo psicológico configuram-se em documento
(A) resumido que visa apresentar o diagnóstico do avaliado, sendo obrigatório constar a Classificação
Internacional de Doença – CID10.
(B) apresentado de forma descritiva acerca de situações e/ou condições psicológicas e suas
determinações históricas, sociais, políticas e culturais, pesquisadas no processo de avaliação psicológica.
(C) com escrita simples, cujos registros deverão estar transcritos de forma corrida, ou seja, separados
apenas pela pontuação, sem parágrafos, evitando, com isso, riscos de adulterações. No caso em que
seja necessária a utilização de parágrafos, o psicólogo deverá preencher esses espaços com traços.
(D) que tem por finalidade informar sobre as condições psicológicas de quem, por requerimento, o
solicita, com fins de justificar estar apto ou não para atividades específicas.
(E) que tem como finalidade apresentar resposta esclarecedora de uma questão-problema, com base
no campo de conhecimento psicológico e em avaliação especializada.

04. (TRE/SP - Analista Judiciário - FCC/2017). O Manual de Elaboração de Documentos Escritos


produzidos pelo psicólogo, decorrentes de avaliação psicológica, Resolução CFP nº 007/2003, dispõe
que o relatório psicológico deve
(A) conter, no mínimo, quatro itens: identificação, descrição da demanda, procedimento e conclusão.

Apostila gerada especialmente para: Betânia Mueller 023.712.580-32


. 8
(B) apresentar, na descrição da demanda, a análise que se faz da solicitação que justifica o
procedimento adotado.
(C) ter, na identificação, quem elabora, quem solicita a avaliação e os instrumentos utilizados.
(D) conter, no procedimento, uma exposição descritiva de forma metódica, objetiva e fiel dos dados
coletados e das situações vividas relacionados à demanda em sua complexidade.
(E) apresentar, na conclusão, que se limite às afirmações baseadas em teorias não em fatos.

05. (Pref. de Teresina/PI - Técnico de Nível Superior - FCC/2016). Tendo por base a Resolução
CFP no 007/2003 que instituiu o Manual de Elaboração de Documentos Escritos produzidos pelo
psicólogo, decorrentes de avaliação psicológica, uma diferença importante entre o laudo e o parecer
psicológicos é que o parecer é
(A) um documento reflexivo sobre a saúde mental do indivíduo, visando a aproximá-lo de uma
psicoterapia.
(B) a expressão de um posicionamento sobre a história de vida do indivíduo, visando a auxiliá-lo na
busca por autoconhecimento e melhora do autoconceito.
(C) uma resposta a uma consulta, a uma “questão-problema”, visando a dirimir dúvidas que estão
interferindo na decisão.
(D) uma peça de teor jurídico, visando instrumentalizar os advogados das partes a dialogarem com o
psicólogo.
(E) o modo como o psicólogo esclarece suas dúvidas sobre o caso, visando o bem estar do indivíduo.

06. (CFP - Psicologia do Trânsito - Quadrix). O Manual de Elaboração de Documentos Escritos


produzidos pelo psicólogo é estruturado a partir da resolução CFP n° 007/2003. Sobre este manual,
assinale a alternativa incorreta.
(A) O relatório psicológico apresenta as demandas, procedimentos, métodos e conclusões gerados
pelo processo da avaliação psicológica, podendo descrever aspectos de diagnóstico e prognóstico
psicológico.
(B) O "atestado psicológico" permite ao profissional psicólogo realizar registro de informações sobre
sintomas, situação e condição psicológica, podendo ser usado o registro do Código Internacional de
Doença (CID).
(C) No "Parecer" não é permitido apresentar informações de caráter indicativo ou conclusivo do cliente
ou paciente atendido.
(D) O documento do tipo "Declaração" é destinado principalmente à prestação de informações sobre
as condições do atendimento (tempo de acompanhamento, dias ou horários).
(E) O Parecer psicológico não pode ser um documento decorrente de um processo de avaliação
Psicológica.

07. (UEG - Analista de Gestão Administrativa - FUNIVERSA/2015). A avaliação psicológica é


entendida como o processo técnico-científico de coleta de dados, estudos e interpretação de informações
a respeito dos fenômenos psicológicos, que são resultantes da relação do indivíduo com a sociedade,
utilizando-se, para tanto, de estratégias psicológicas - métodos, técnicas e instrumentos. Os resultados
das avaliações devem considerar e analisar os condicionantes históricos e sociais e seus efeitos no
psiquismo, com a finalidade de servirem como instrumentos para atuar não somente sobre o indivíduo,
mas na modificação desses condicionantes que operam desde a formulação da demanda até a conclusão
do processo de avaliação psicológica.

Resolução CFP n.º 007/2003 que institui o Manual de Elaboração de Documentos Escritos.

De acordo com a Resolução CFP n.º 007/2003, que institui o manual de elaboração de documentos
escritos produzidos pelo psicólogo, são modalidades de documentos decorrentes de avaliação
psicológica:
(A) declaração; atestado psicológico; relatório psicológico; e parecer psicológico.
(B) atestado psicológico; relatório/laudo psicológico; e parecer psicológico.
(C) declaração; atestado psicológico; e relatório/laudo psicológico.
(D) relatório/laudo psicológico; e parecer psicológico.
(E) atestado psicológico; e relatório/laudo psicológico.

Apostila gerada especialmente para: Betânia Mueller 023.712.580-32


. 9
Gabarito

01.A / 02.B / 03.B / 04.B / 05.C / 06.C / 07.E

Comentários
01. Resposta: A
RESOLUÇÃO CFP N.º 007/2003
O parecer é composto de 4 (quatro) itens:
1. Identificação
2. Exposição de motivos
3. Análise
4. Conclusão

02. Resposta: B
RESOLUÇÃO CFP N.º 007/2003
Os resultados das avaliações devem considerar e analisar os condicionantes históricos e sociais e
seus efeitos no psiquismo, com a finalidade de servirem como instrumentos para atuar não somente sobre
o indivíduo, mas na modificação desses condicionantes que operam desde a formulação da demanda até
a conclusão do processo de avaliação psicológica.

03. Resposta: B
RESOLUÇÃO CFP N.º 007/2003
O relatório ou laudo psicológico é uma apresentação descritiva acerca de situações e/ou condições
psicológicas e suas determinações históricas, sociais, políticas e culturais, pesquisadas no processo de
avaliação psicológica.

04. Resposta: B
RESOLUÇÃO CFP N.º 007/2003
3.2.2. Descrição da demanda
Esta parte é destinada à narração das informações referentes à problemática apresentada e dos
motivos, razões e expectativas que produziram o pedido do documento.
Nesta parte, deve-se apresentar a análise que se faz da demanda de forma a justificar o procedimento
adotado.

05. Resposta: C
RESOLUÇÃO CFP N.º 007/2003
O parecer tem como finalidade apresentar resposta esclarecedora, no campo do conhecimento
psicológico, através de uma avaliação especializada, de uma “questão problema”, visando a dirimir
dúvidas que estão interferindo na decisão, sendo, portanto, uma resposta a uma consulta, que exige de
quem responde competência no assunto.

06. Resposta: C
RESOLUÇÃO CFP N.º 007/2003
O parecer tem como finalidade apresentar resposta esclarecedora, no campo do conhecimento
psicológico, através de uma avaliação especializada, de uma “questão problema”, visando a dirimir
dúvidas que estão interferindo na decisão, sendo, portanto, uma resposta a uma consulta, que exige de
quem responde competência no assunto.

07. Resposta: E
RESOLUÇÃO CFP N.º 007/2003
II - MODALIDADES DE DOCUMENTOS
1. Declaração *
2. Atestado psicológico
3. Relatório / laudo psicológico
4. Parecer psicológico *
* A Declaração e o Parecer psicológico não são documentos decorrentes da avaliação Psicológica,
embora muitas vezes apareçam desta forma. Por isso consideramos importante constarem deste manual
afim de que sejam diferenciados.

Apostila gerada especialmente para: Betânia Mueller 023.712.580-32


. 10
Ética profissional.

Código de Ética Profissional do Psicólogo1

RESOLUÇÃO CFP Nº 010/05

Aprova o Código de Ética Profissional do Psicólogo.

O CONSELHO FEDERAL DE PSICOLOGIA, no uso de suas atribuições legais e regimentais, que lhe
são conferidas pela Lei no 5.766, de 20 de dezembro de 1971;

CONSIDERANDO o disposto no Art. 6º, letra “e”, da Lei no 5.766 de 20/12/1971, e o Art. 6º, inciso VII,
do Decreto nº 79.822 de 17/6/1977;

CONSIDERANDO o disposto na Constituição Federal de 1988, conhecida como Constituição Cidadã,


que consolida o Estado Democrático de Direito e legislações dela decorrentes;

CONSIDERANDO decisão deste Plenário em reunião realizada no dia 21 de julho de 2005;

RESOLVE:

Art. 1º - Aprovar o Código de Ética Profissional do Psicólogo.

Art. 2º - A presente Resolução entrará em vigor no dia 27 de agosto de 2005.

Art. 3º - Revogam-se as disposições em contrário, em especial a Resolução CFP n º 002/87.

Brasília, 21 de julho de 2005.

Apresentação

Toda profissão define-se a partir de um corpo de práticas que busca atender demandas sociais,
norteado por elevados padrões técnicos e pela existência de normas éticas que garantam a adequada
relação de cada profissional com seus pares e com a sociedade como um todo.
Um Código de Ética profissional, ao estabelecer padrões esperados quanto às práticas referendadas
pela respectiva categoria profissional e pela sociedade, procura fomentar a autorreflexão exigida de cada
indivíduo acerca da sua práxis, de modo a responsabilizá-lo, pessoal e coletivamente, por ações e suas
consequências no exercício profissional. A missão primordial de um código de ética profissional não é de
normatizar a natureza técnica do trabalho, e, sim, a de assegurar, dentro de valores relevantes para a
sociedade e para as práticas desenvolvidas, um padrão de conduta que fortaleça o reconhecimento social
daquela categoria. Códigos de Ética expressam sempre uma concepção de homem e de sociedade que
determina a direção das relações entre os indivíduos. Traduzem-se em princípios e normas que devem
se pautar pelo respeito ao sujeito humano e seus direitos fundamentais. Por constituir a expressão de
valores universais, tais como os constantes na Declaração Universal dos Direitos Humanos;
socioculturais, que refletem a realidade do país; e de valores que estruturam uma profissão, um código
de ética não pode ser visto como um conjunto fixo de normas e imutável no tempo. As sociedades mudam,
as profissões transformam-se e isso exige, também, uma reflexão contínua sobre o próprio código de
ética que nos orienta.
A formulação deste Código de Ética, o terceiro da profissão de psicólogo no Brasil, responde ao
contexto organizativo dos psicólogos, ao momento do país e ao estágio de desenvolvimento da Psicologia
enquanto campo científico e profissional. Este Código de Ética dos Psicólogos é reflexo da necessidade,
sentida pela categoria e suas entidades representativas, de atender à evolução do contexto institucional-
legal do país, marcadamente a partir da promulgação da denominada Constituição Cidadã, em 1988, e
das legislações dela decorrentes.

1
https://site.cfp.org.br/wp-content/uploads/2012/07/codigo-de-etica-psicologia.pdf

Apostila gerada especialmente para: Betânia Mueller 023.712.580-32


. 11
Consoante com a conjuntura democrática vigente, o presente Código foi construído a partir de múltiplos
espaços de discussão sobre a ética da profissão, suas responsabilidades e compromissos com a
promoção da cidadania. O processo ocorreu ao longo de três anos, em todo o país, com a participação
direta dos psicólogos e aberto à sociedade.
Este Código de Ética pautou-se pelo princípio geral de aproximar-se mais de um instrumento de
reflexão do que de um conjunto de normas a serem seguidas pelo psicólogo. Para tanto, na sua
construção buscou-se:
a. Valorizar os princípios fundamentais como grandes eixos que devem orientar a relação do psicólogo
com a sociedade, a profissão, as entidades profissionais e a ciência, pois esses eixos atravessam todas
as práticas e estas demandam uma contínua reflexão sobre o contexto social e institucional.
b. Abrir espaço para a discussão, pelo psicólogo, dos limites e interseções relativos aos direitos
individuais e coletivos, questão crucial para as relações que estabelece com a sociedade, os colegas de
profissão e os usuários ou beneficiários dos seus serviços.
c. Contemplar a diversidade que configura o exercício da profissão e a crescente inserção do psicólogo
em contextos institucionais e em equipes multiprofissionais.
d. Estimular reflexões que considerem a profissão como um todo e não em suas práticas particulares,
uma vez que os principais dilemas éticos não se restringem a práticas específicas e surgem em quaisquer
contextos de atuação.
Ao aprovar e divulgar o Código de Ética Profissional do Psicólogo, a expectativa é de que ele seja um
instrumento capaz de delinear para a sociedade as responsabilidades e deveres do psicólogo, oferecer
diretrizes para a sua formação e balizar os julgamentos das suas ações, contribuindo para o
fortalecimento e ampliação do significado social da profissão.

Princípios Fundamentais

I. O psicólogo baseará o seu trabalho no respeito e na promoção da liberdade, da dignidade, da


igualdade e da integridade do ser humano, apoiado nos valores que embasam a Declaração Universal
dos Direitos Humanos.
II. O psicólogo trabalhará visando promover a saúde e a qualidade de vida das pessoas e das
coletividades e contribuirá para a eliminação de quaisquer formas de negligência, discriminação,
exploração, violência, crueldade e opressão.
III. O psicólogo atuará com responsabilidade social, analisando crítica e historicamente a realidade
política, econômica, social e cultural.
IV. O psicólogo atuará com responsabilidade, por meio do contínuo aprimoramento profissional,
contribuindo para o desenvolvimento da Psicologia como campo científico de conhecimento e de prática.
V. O psicólogo contribuirá para promover a universalização do acesso da população às informações,
ao conhecimento da ciência psicológica, aos serviços e aos padrões éticos da profissão.
VI. O psicólogo zelará para que o exercício profissional seja efetuado com dignidade, rejeitando
situações em que a Psicologia esteja sendo aviltada.
VII. O psicólogo considerará as relações de poder nos contextos em que atua e os impactos dessas
relações sobre as suas atividades profissionais, posicionando-se de forma crítica e em consonância com
os demais princípios deste Código.

Das Responsabilidades do Psicólogo

Art. 1º – São deveres fundamentais dos psicólogos:

a) Conhecer, divulgar, cumprir e fazer cumprir este Código;


b) Assumir responsabilidades profissionais somente por atividades para as quais esteja capacitado
pessoal, teórica e tecnicamente;
c) Prestar serviços psicológicos de qualidade, em condições de trabalho dignas e apropriadas à
natureza desses serviços, utilizando princípios, conhecimentos e técnicas reconhecidamente
fundamentados na ciência psicológica, na ética e na legislação profissional;
d) Prestar serviços profissionais em situações de calamidade pública ou de emergência, sem visar
benefício pessoal;
e) Estabelecer acordos de prestação de serviços que respeitem os direitos do usuário ou beneficiário
de serviços de Psicologia;
f) Fornecer, a quem de direito, na prestação de serviços psicológicos, informações concernentes ao
trabalho a ser realizado e ao seu objetivo profissional;

Apostila gerada especialmente para: Betânia Mueller 023.712.580-32


. 12
g) Informar, a quem de direito, os resultados decorrentes da prestação de serviços psicológicos,
transmitindo somente o que for necessário para a tomada de decisões que afetem o usuário ou
beneficiário;
h) Orientar a quem de direito sobre os encaminhamentos apropriados, a partir da prestação de serviços
psicológicos, e fornecer, sempre que solicitado, os documentos pertinentes ao bom termo do trabalho;
i) Zelar para que a comercialização, aquisição, doação, empréstimo, guarda e forma de divulgação do
material privativo do psicólogo sejam feitas conforme os princípios deste Código;
j) Ter, para com o trabalho dos psicólogos e de outros profissionais, respeito, consideração e
solidariedade, e, quando solicitado, colaborar com estes, salvo impedimento por motivo relevante;
k) Sugerir serviços de outros psicólogos, sempre que, por motivos justificáveis, não puderem ser
continuados pelo profissional que os assumiu inicialmente, fornecendo ao seu substituto as informações
necessárias à continuidade do trabalho;
l) Levar ao conhecimento das instâncias competentes o exercício ilegal ou irregular da profissão,
transgressões a princípios e diretrizes deste Código ou da legislação profissional.

Art. 2º – Ao psicólogo é vedado:


a) Praticar ou ser conivente com quaisquer atos que caracterizem negligência, discriminação,
exploração, violência, crueldade ou opressão;
b) Induzir a convicções políticas, filosóficas, morais, ideológicas, religiosas, de orientação sexual ou a
qualquer tipo de preconceito, quando do exercício de suas funções profissionais;
c) Utilizar ou favorecer o uso de conhecimento e a utilização de práticas psicológicas como
instrumentos de castigo, tortura ou qualquer forma de violência;
d) Acumpliciar-se com pessoas ou organizações que exerçam ou favoreçam o exercício ilegal da
profissão de psicólogo ou de qualquer outra atividade profissional;
e) Ser conivente com erros, faltas éticas, violação de direitos, crimes ou contravenções penais
praticados por psicólogos na prestação de serviços profissionais;
f) Prestar serviços ou vincular o título de psicólogo a serviços de atendimento psicológico cujos
procedimentos, técnicas e meios não estejam regulamentados ou reconhecidos pela profissão;
g) Emitir documentos sem fundamentação e qualidade técnico-científica;
h) Interferir na validade e fidedignidade de instrumentos e técnicas psicológicas, adulterar seus
resultados ou fazer declarações falsas;
i) Induzir qualquer pessoa ou organização a recorrer a seus serviços;
j) Estabelecer com a pessoa atendida, familiar ou terceiro, que tenha vínculo com o atendido, relação
que possa interferir negativamente nos objetivos do serviço prestado;
k) Ser perito, avaliador ou parecerista em situações nas quais seus vínculos pessoais ou profissionais,
atuais ou anteriores, possam afetar a qualidade do trabalho a ser realizado ou a fidelidade aos resultados
da avaliação;
l) Desviar para serviço particular ou de outra instituição, visando benefício próprio, pessoas ou
organizações atendidas por instituição com a qual mantenha qualquer tipo de vínculo profissional;
m) Prestar serviços profissionais a organizações concorrentes de modo que possam resultar em
prejuízo para as partes envolvidas, decorrentes de informações privilegiadas;
n) Prolongar, desnecessariamente, a prestação de serviços profissionais;
o) Pleitear ou receber comissões, empréstimos, doações ou vantagens outras de qualquer espécie,
além dos honorários contratados, assim como intermediar transações financeiras;
p) Receber, pagar remuneração ou porcentagem por encaminhamento de serviços;
q) Realizar diagnósticos, divulgar procedimentos ou apresentar resultados de serviços psicológicos em
meios de comunicação, de forma a expor pessoas, grupos ou organizações.

Art. 3º – O psicólogo, para ingressar, associar-se ou permanecer em uma organização, considerará a


missão, a filosofia, as políticas, as normas e as práticas nela vigentes e sua compatibilidade com os
princípios e regras deste Código.
Parágrafo único: Existindo incompatibilidade, cabe ao psicólogo recusar-se a prestar serviços e, se
pertinente, apresentar denúncia ao órgão competente.

Art. 4º – Ao fixar a remuneração pelo seu trabalho, o psicólogo:

a) Levará em conta a justa retribuição aos serviços prestados e as condições do usuário ou


beneficiário;

Apostila gerada especialmente para: Betânia Mueller 023.712.580-32


. 13
b) Estipulará o valor de acordo com as características da atividade e o comunicará ao usuário ou
beneficiário antes do início do trabalho a ser realizado;
c) Assegurará a qualidade dos serviços oferecidos independentemente do valor acordado.

Art. 5º – O psicólogo, quando participar de greves ou paralisações, garantirá que:

a) As atividades de emergência não sejam interrompidas;


b) Haja prévia comunicação da paralisação aos usuários ou beneficiários dos serviços atingidos pela
mesma.

Art. 6º – O psicólogo, no relacionamento com profissionais não psicólogos:

a) Encaminhará a profissionais ou entidades habilitados e qualificados demandas que extrapolem seu


campo de atuação;
b) Compartilhará somente informações relevantes para qualificar o serviço prestado, resguardando o
caráter confidencial das comunicações, assinalando a responsabilidade, de quem as receber, de
preservar o sigilo.

Art. 7º – O psicólogo poderá intervir na prestação de serviços psicológicos que estejam sendo
efetuados por outro profissional, nas seguintes situações:

a) A pedido do profissional responsável pelo serviço;


b) Em caso de emergência ou risco ao beneficiário ou usuário do serviço, quando dará imediata ciência
ao profissional;
c) Quando informado expressamente, por qualquer uma das partes, da interrupção voluntária e
definitiva do serviço;
d) Quando se tratar de trabalho multiprofissional e a intervenção fizer parte da metodologia adotada.

Art. 8º – Para realizar atendimento não eventual de criança, adolescente ou interdito, o psicólogo
deverá obter autorização de ao menos um de seus responsáveis, observadas as determinações da
legislação vigente:
§1° – No caso de não se apresentar um responsável legal, o atendimento deverá ser efetuado e
comunicado às autoridades competentes;
§2° – O psicólogo responsabilizar-se-á pelos encaminhamentos que se fizerem necessários para
garantir a proteção integral do atendido.

Art. 9º – É dever do psicólogo respeitar o sigilo profissional a fim de proteger, por meio da
confidencialidade, a intimidade das pessoas, grupos ou organizações, a que tenha acesso no exercício
profissional.

Art. 10° – Nas situações em que se configure conflito entre as exigências decorrentes do disposto no
Art. 9º e as afirmações dos princípios fundamentais deste Código, excetuando-se os casos previstos em
lei, o psicólogo poderá decidir pela quebra de sigilo, baseando sua decisão na busca do menor prejuízo.
Parágrafo único – Em caso de quebra do sigilo previsto no caput deste artigo, o psicólogo deverá
restringir-se a prestar as informações estritamente necessárias.

Art. 11° – Quando requisitado a depor em juízo, o psicólogo poderá prestar informações, considerando
o previsto neste Código.

Art. 12° – Nos documentos que embasam as atividades em equipe multiprofissional, o psicólogo
registrará apenas as informações necessárias para o cumprimento dos objetivos do trabalho.

Art. 13° – No atendimento à criança, ao adolescente ou ao interdito, deve ser comunicado aos
responsáveis o estritamente essencial para se promoverem medidas em seu benefício.

Art. 14° – A utilização de quaisquer meios de registro e observação da prática psicológica obedecerá
às normas deste Código e a legislação profissional vigente, devendo o usuário ou beneficiário, desde o
início, ser informado.

Apostila gerada especialmente para: Betânia Mueller 023.712.580-32


. 14
Art. 15° – Em caso de interrupção do trabalho do psicólogo, por quaisquer motivos, ele deverá zelar
pelo destino dos seus arquivos confidenciais.
§ 1° – Em caso de demissão ou exoneração, o psicólogo deverá repassar todo o material ao psicólogo
que vier a substituí-lo, ou lacrá-lo para posterior utilização pelo psicólogo substituto.
§ 2° – Em caso de extinção do serviço de Psicologia, o psicólogo responsável informará ao Conselho
Regional de Psicologia, que providenciará a destinação dos arquivos confidenciais.

Art. 16° – O psicólogo, na realização de estudos, pesquisas e atividades voltadas para a produção de
conhecimento e desenvolvimento de tecnologias:

a) Avaliará os riscos envolvidos, tanto pelos procedimentos, como pela divulgação dos resultados, com
o objetivo de proteger as pessoas, grupos, organizações e comunidades envolvidas;
b) Garantirá o caráter voluntário da participação dos envolvidos, mediante consentimento livre e
esclarecido, salvo nas situações previstas em legislação específica e respeitando os princípios deste
Código;
c) Garantirá o anonimato das pessoas, grupos ou organizações, salvo interesse manifesto destes;
d) Garantirá o acesso das pessoas, grupos ou organizações aos resultados das pesquisas ou estudos,
após seu encerramento, sempre que assim o desejarem.

Art. 17° – Caberá aos psicólogos docentes ou supervisores esclarecer, informar, orientar e exigir dos
estudantes a observância dos princípios e normas contidas neste Código.

Art. 18° – O psicólogo não divulgará, ensinará, cederá, emprestará ou venderá a leigos instrumentos
e técnicas psicológicas que permitam ou facilitem o exercício ilegal da profissão.

Art. 19° – O psicólogo, ao participar de atividade em veículos de comunicação, zelará para que as
informações prestadas disseminem o conhecimento a respeito das atribuições, da base científica e do
papel social da profissão.

Art. 20° – O psicólogo, ao promover publicamente seus serviços, por quaisquer meios, individual ou
coletivamente:

a) Informará o seu nome completo, o CRP e seu número de registro;


b) Fará referência apenas a títulos ou qualificações profissionais que possua;
c) Divulgará somente qualificações, atividades e recursos relativos a técnicas e práticas que estejam
reconhecidas ou regulamentadas pela profissão;
d) Não utilizará o preço do serviço como forma de propaganda;
e) Não fará previsão taxativa de resultados;
f) Não fará autopromoção em detrimento de outros profissionais;
g) Não proporá atividades que sejam atribuições privativas de outras categorias profissionais;
h) Não fará divulgação sensacionalista das atividades profissionais.

Das Disposições Gerais

Art. 21° – As transgressões dos preceitos deste Código constituem infração disciplinar com a aplicação
das seguintes penalidades, na forma dos dispositivos legais ou regimentais:

a) Advertência;
b) Multa;
c) Censura pública;
d) Suspensão do exercício profissional, por até 30 (trinta) dias, ad referendum do Conselho Federal de
Psicologia;
e) Cassação do exercício profissional, ad referendum do Conselho Federal de Psicologia.

Art. 22° – As dúvidas na observância deste Código e os casos omissos serão resolvidos pelos
Conselhos Regionais de Psicologia, ad referendum do Conselho Federal de Psicologia.

Art. 23° – Competirá ao Conselho Federal de Psicologia firmar jurisprudência quanto aos casos
omissos e fazê-la incorporar a este Código.

Apostila gerada especialmente para: Betânia Mueller 023.712.580-32


. 15
Art. 24° – O presente Código poderá ser alterado pelo Conselho Federal de Psicologia, por iniciativa
própria ou da categoria, ouvidos os Conselhos Regionais de Psicologia.

Art. 25°– Este Código entra em vigor em 27 de agosto de 2005.

Questões

01. (UTFPR - Psicólogo - UTFPR/2017). De acordo com o Código de Ética Profissional, ao psicólogo
é vedado:
(A) sugerir serviços de outros psicólogos, sempre que, por motivos justificáveis, não puderem ser
continuados pelo profissional que os assumiu inicialmente.
(B) prestar serviços profissionais em situações de calamidade pública ou de emergência.
(C) fornecer, a quem de direito, na prestação de serviços psicológicos, informações concernentes ao
trabalho a ser realizado e ao seu objetivo profissional.
(D) informar, a quem de direito, os resultados decorrentes da prestação de serviços psicológicos.
(E) induzir a convicções políticas, filosóficas, morais, ideológicas, religiosas ou de orientação sexual,
quando do exercício de suas funções profissionais.

02. (SEDF - Analista de Gestão Educacional - CESPE/2017). Considerando o código de ética


profissional do psicólogo e as perspectivas contemporâneas da atuação do psicólogo escolar, julgue o
item a seguir.
Em situação multiprofissional, não cabe ao psicólogo, em respeito ao código de ética, intervir na
prestação de serviços psicológicos que estejam sendo efetuados por outros profissionais.

( ) Certo ( ) Errado

03. (Pref. de Bela Vista de Minas/MG - Psicólogo - FUNDEP/2017). Sobre os deveres fundamentais
descritos no Art. 1º do Código de Ética Profissional do Psicólogo, é correto afirmar:
(A) O psicólogo deve informar, a quem de direito for, os resultados decorrentes da prestação de
serviços psicológicos, comunicando apenas o que for necessário para a tomada de decisões que afetem
o beneficiário.
(B) O psicólogo deve induzir seus pacientes a convicções de orientação sexual quando do exercício
de suas funções profissionais.
(C) O psicólogo deve pleitear ou receber comissões, doações ou empréstimos além dos honorários
contratados, assim como intermediar transações financeiras.
(D) O psicólogo deve realizar diagnóstico, divulgar procedimentos ou apresentar resultados de serviços
psicológicos em meios de comunicação, de forma a auxiliar grupos e organizações.

04. (SEDF - Professor - Quadrix/2017). Com base no Código de ética profissional do psicólogo, julgue
o item a seguir.
No art. 12, consta que o psicólogo registrará todas as informações necessárias para o cumprimento
dos objetivos do trabalho nos documentos que embasem as atividades em equipe multiprofissional.

( ) Certo ( ) Errado

05. (EBSERH - Psicólogo - IBFC/2017). As transgressões dos preceitos descritos no atual Código de
Ética do Psicólogo constituem infração disciplinar, e são previstas na legislação vigente a aplicação de
sanções e ou penalidades em casos de comprovação de tais transgressões. Assinale a alternativa que
não corresponde a uma penalidade descrita no atual Código de Ética do Psicólogo diante de uma infração
disciplinar cometida pelo profissional.
(A) Cassação do exercício profissional
(B) Multa
(C) Censura pública
(D) Suspensão do exercício profissional por até um ano
(E) Advertência

Gabarito

01.E / 02.errado / 03.A / 04.certo / 05.D

Apostila gerada especialmente para: Betânia Mueller 023.712.580-32


. 16
Comentários

01. Resposta: E
Código de Ética do Psicólogo
Art. 2º – Ao psicólogo é vedado:
( )
b) Induzir a convicções políticas, filosóficas, morais, ideológicas, religiosas, de orientação sexual ou a
qualquer tipo de preconceito, quando do exercício de suas funções profissionais.

02. Resposta: errado


Código de Ética do Psicólogo
Art. 7º – O psicólogo poderá intervir na prestação de serviços psicológicos que estejam sendo
efetuados por outro profissional, nas seguintes situações:
( )
b) Em caso de emergência ou risco ao beneficiário ou usuário do serviço, quando dará imediata ciência
ao profissional;

03. Resposta: A
Código de Ética do Psicólogo
Art. 1º – São deveres fundamentais dos psicólogos:
( )
g) Informar, a quem de direito, os resultados decorrentes da prestação de serviços psicológicos,
transmitindo somente o que for necessário para a tomada de decisões que afetem o usuário ou
beneficiário.

04. Resposta: certo


Código de Ética do Psicólogo
Art. 12 – Nos documentos que embasam as atividades em equipe multiprofissional, o psicólogo
registrará apenas as informações necessárias para o cumprimento dos objetivos do trabalho.

05. Resposta: D
Código de Ética do Psicólogo
Art. 21 – As transgressões dos preceitos deste Código constituem infração disciplinar com a aplicação
das seguintes penalidades, na forma dos dispositivos legais ou regimentais:
( )
d) Suspensão do exercício profissional, por até 30 (trinta) dias, ad referendum do Conselho Federal de
Psicologia.

Resoluções do CFP (Resolução CFP nº 13/2018, CFP nº 09/2018, CFP N.º


001/2018).

RESOLUÇÃO Nº 13, DE 21 DE MAIO DE 20182

Altera o funcionamento e composição da Comissão Nacional de Psicologia na Assistência Social


(CONPAS) no âmbito do Conselho Federal de Psicologia, revoga os arts. 2º, 3º, 4º, 5º e anexos da
Resolução CFP n° 35/2015 e dá outras providências.

O CONSELHO FEDERAL DE PSICOLOGIA, no uso de suas atribuições legais e regimentais, que lhe
são conferidas pela Lei n° 5.766, de 20 de dezembro de 1971,
CONSIDERANDO a importância da Assistência Social como política pública de acesso da população
brasileira aos direitos sociassistenciais;
CONSIDERANDO a instituição da Comissão Nacional de Psicologia na Assistência Social (CONPAS),
pela Resolução nº 35/2015 (art.1º);
CONSIDERANDO a presença das psicólogas e dos psicólogos nas funções essenciais de gestão do
Sistema Único de Assistência Social (SUAS) e na composição obrigatória das equipes da Proteção Social
Básica e
2
https://site.cfp.org.br/wp-content/uploads/2018/05/Resolu%C3%A7%C3%A3o-CONPAS.pdf

Apostila gerada especialmente para: Betânia Mueller 023.712.580-32


. 17
Proteção Social Especial, conforme disposto na Norma Operacional Básica de Recursos Humanos
(NOBRH) do SUAS e na Resolução 17/2011 do Conselho Nacional de Assistência Social (CNAS);
CONSIDERANDO que a política pública de assistência social está em processo de consolidação e
requer a construção de orientações e estratégias de ação e intervenção nesse campo; Resolve:

Art. 1º – A Comissão Nacional de Psicologia na Assistência Social (CONPAS), terá seus membros
nomeados/as por meio de portaria expedida pelo CFP, respeitados os critérios definidos neste ar go.
§ 1º – O número de membros da comissão poderá variar entre 5 (cinco), no mínimo, e 8 (oito), no
máximo.
§ 2º – É atribuição da Plenária do CFP definir a composição da comissão, indicando novos membros
ou subs tuindo os atuais.

Art. 2º – A CONPAS é constituída para contribuir com a atuação profissional da Psicologia no SUAS,
visando à qualidade ética e técnica no exercício profissional, à defesa dos direitos socioassistenciais e à
melhoria das condições e relações do trabalho como estratégia para consolidação da Política Nacional
de Assistência Social (PNAS/2004).

Art. 3º – São atribuições da CONPAS:


I – Articular, no âmbito do Sistema Conselhos de Psicologia, as ações que incidam no tema dos direitos
socioassistenciais;
II – Incentivar e promover reflexões sobre os direitos socioassistenciais e as políticas públicas que os
garantam, considerando aspectos afetos à formação, à prática profissional e à pesquisa, de forma
articulada com as entidades da Psicologia;
III – Promover o diálogo com a totalidade de profissionais que atuam no SUAS, considerando as
deliberações do Congresso Nacional de Psicologia, do Centro de Referências Técnicas em Psicologia e
Política Públicas (CREPOP) e das demais comissões e grupos de trabalho do CFP e dos Conselhos
Regionais de Psicologia (CRPs) e também as demandas advindas dos CRPs e do CFP, com base no
debate das instâncias da política pública de assistência social (controle social) e dos Fóruns de
Trabalhadoras e Trabalhadores do SUAS, nas instâncias nacional, estaduais, regionais e municipais.

Art. 4º – A comissão realizará, anualmente, 2 (duas) reuniões presenciais, que deverá contar com a
participação 1 (um) representante de cada CRP, assim definidas:
I. A primeira reunião será a de Planejamento Geral e deverá garantir a articulação e a pactuação das
ações e assuntos da Assistência Social no âmbito do Sistema Conselhos de Psicologia.
II. A segunda reunião será definida e convocada pela comissão.
III. O custeio das despesas relativas à participação nessas reuniões ocorrerá do seguinte modo:
a) A participação da comissão é de responsabilidade do CFP.
b) A participação de representantes dos CRPs será em conformidade com os critérios de custeio da
Assembleia de Políticas, da Administração e das Finanças (APAF) do Sistema Conselhos de Psicologia.
Parágrafo Único – A comissão poderá convidar pessoas externas para discutir os temas da pauta.

Art. 5º – Compete ao Plenário do Conselho Federal de Psicologia:


I. Estabelecer anualmente a dotação orçamentária específica para a comissão.
II. Aprovar o planejamento geral.

Art. 6º – Revogam-se os arts. 2º, 3º, 4º, 5º e anexos da Resolução CFP n° 35/2015.

Art. 7º – Esta resolução entra em vigor na data de sua publicação.

Rogério Giannini
Conselheiro-Presidente
Conselho Federal de Psicologia

RESOLUÇÃO Nº 9, DE 25 DE ABRIL DE 20183

Estabelece diretrizes para a realização de Avaliação Psicológica no exercício profissional da psicóloga


e do psicólogo, regulamenta o Sistema de Avaliação de Testes

3
https://site.cfp.org.br/wp-content/uploads/2018/04/Resolu%C3%A7%C3%A3o-CFP-n%C2%BA-09-2018-com-anexo.pdf

Apostila gerada especialmente para: Betânia Mueller 023.712.580-32


. 18
Psicológicos - SATEPSI e revoga as Resoluções n° 002/2003, nº 006/2004 e n° 005/2012 e Notas
Técnicas n° 01/2017 e 02/2017.
O CONSELHO FEDERAL DE PSICOLOGIA, no uso das atribuições legais e regimentais que lhe são
conferidas pela Lei nº 5.766, de 20 de dezembro de 1971, e o Decreto nº 53.464, de 21 de janeiro de
1964, e;
CONSIDERANDO que a utilização de métodos e técnicas psicológicas constitui função privativa da
psicóloga e do psicólogo, com base nos objetivos previstos no parágrafo 1º, do art. 13, da Lei nº 4.119,
de 27 de agosto de 1962, e no art. 4º, do Decreto nº 53.464/1964;
CONSIDERANDO a necessidade de estabelecer diretrizes sobre Avaliação Psicológica que possam
orientar o trabalho das psicólogas e dos psicólogos em diferentes contextos de atuação profissional;
CONSIDERANDO a função social do Sistema Conselhos de Psicologia em contribuir para o
aprimoramento da qualidade técnico-científica dos métodos e procedimentos psicológicos;
CONSIDERANDO a garantia do compromisso ético das psicólogas e dos psicólogos na utilização de
testes psicológicos no âmbito profissional;
CONSIDERANDO a demanda social e técnico-científica de construir um sistema contínuo de avaliação
de testes psicológicos compatível com a dinâmica da produção científica e com as necessidades das
profissionais e dos profissionais da Psicologia;
CONSIDERANDO que o Sistema de Avaliação de Testes Psicológicos (SATEPSI) é um sistema
informatizado que tem por objetivo avaliar a qualidade técnico-científica de instrumentos submetidos à
apreciação da Comissão Consultiva em Avaliação Psicológica do Conselho Federal de Psicologia (CFP);
CONSIDERANDO o constante trabalho de aprimoramento e incorporação de melhorias do SATEPSI
sugeridas e debatidas em diferentes fóruns científicos;
CONSIDERANDO a necessidade de tornar público os critérios de análise e o processo de avaliação
de testes psicológicos;
CONSIDERANDO os princípios éticos fundamentais que norteiam a atividade profissional da psicóloga
e do psicólogo, e o inciso IV dos Princípios Fundamentais no Código de Ética Profissional da psicóloga e
do psicólogo, que estabelece a responsabilidade da psicóloga e do psicólogo por seu contínuo
aprimoramento profissional e pelo desenvolvimento da Psicologia como campo científico de
conhecimento e de prática;
CONSIDERANDO a alínea b, do art. 1º do Código de Ética Profissional da psicóloga e do psicólogo
que preconiza que as psicólogas e psicólogos assumam responsabilidades profissionais somente por
atividades para as quais estejam capacitados, pessoal, teórica e tecnicamente, e; CONSIDERANDO a
decisão deste Plenário em 25 de novembro de 2017;

RESOLVE:

DAS DIRETRIZES BÁSICAS PARA A REALIZAÇÃO DE AVALIAÇÃO PSICOLÓGICA NO


EXERCÍCIO PROFISSIONAL DA PSICÓLOGA E DO PSICÓLOGO

Art. 1º - Avaliação Psicológica é definida como um processo estruturado de investigação de fenômenos


psicológicos, composto de métodos, técnicas e instrumentos, com o objetivo de prover informações à
tomada de decisão, no âmbito individual, grupal ou institucional, com base em demandas, condições e
finalidades específicas.
§1 - Os testes psicológicos abarcam também os seguintes instrumentos: escalas, inventários,
questionários e métodos projetivos/expressivos, para fins de padronização desta Resolução e do
SATEPSI.
§2 - A psicóloga e o psicólogo têm a prerrogativa de decidir quais são os métodos, técnicas e
instrumentos empregados na Avaliação Psicológica, desde que devidamente fundamentados na literatura
científica psicológica e nas normativas vigentes do Conselho Federal de Psicologia (CFP).

Art. 2º - Na realização da Avaliação Psicológica, a psicóloga e o psicólogo devem basear sua decisão,
obrigatoriamente, em métodos e/ou técnicas e/ou instrumentos psicológicos reconhecidos cientificamente
para uso na prática profissional da psicóloga e do psicólogo (fontes fundamentais de informação),
podendo, a depender do contexto, recorrer a procedimentos e recursos auxiliares (fontes complementares
de informação).
Consideram-se fontes de informação:
I – Fontes fundamentais:
a) Testes psicológicos aprovados pelo CFP para uso profissional da psicóloga e do psicólogo e/ou;
b) Entrevistas psicológicas, anamnese e/ou;

Apostila gerada especialmente para: Betânia Mueller 023.712.580-32


. 19
c) Protocolos ou registros de observação de comportamentos obtidos individualmente ou por meio de
processo grupal e/ou técnicas de grupo.
II - Fontes complementares:
a) Técnicas e instrumentos não psicológicos que possuam respaldo da literatura científica da área e
que respeitem o Código de Ética e as garantias da legislação da profissão;
b) Documentos técnicos, tais como protocolos ou relatórios de equipes multiprofissionais.
§1 - Será considerada falta ética, conforme disposto na alínea c do Art. 1º e na alínea f do Art. 2º do
Código de Ética Profissional da psicóloga e do psicólogo, a utilização de testes psicológicos com parecer
desfavorável ou que constem na lista de Testes Psicológicos Não Avaliados no site do SATEPSI, salvo
para os casos de pesquisa na forma da legislação vigente e de ensino com objetivo formativo e histórico
na Psicologia.
§2 - Na hipótese de dúvida acerca da classificação do instrumento (teste psicológico ou instrumento
não psicológico), ficam legitimados os Conselhos Regionais de Psicologia (CRPs) a submeter o
respectivo instrumento à Comissão Consultiva em Avaliação Psicológica (CCAP) do CFP para
apreciação.
§3 - A profissional psicóloga e o profissional psicólogo poderão requerer ao CRP a submissão do
instrumento à apreciação da CCAP nos termos do parágrafo §2.

Art. 3º - Documentos decorrentes do processo de Avaliação Psicológica deverão ser elaborados em


conformidade com a(s) resolução(ões) vigente(s) do CFP. É obrigatória a manutenção de todos os
registros dos atendimentos do processo de avaliação psicológica, conforme preconiza a resolução CFP
n° 01/2009.

DA SUBMISSÃO E AVALIAÇÃO DE TESTES AO SISTEMA DE AVALIAÇÃO DE TESTES


PSICOLÓGICOS (SATEPSI)

Art. 4º - Um teste psicológico tem por objetivo identificar, descrever, qualificar e mensurar
características psicológicas, por meio de procedimentos sistemáticos de observação e descrição do
comportamento humano, nas suas diversas formas de expressão, acordados pela comunidade científica.

Art. 5º - Os documentos a seguir são referências para a definição dos conceitos, princípios e
procedimentos de avaliação de instrumentos psicológicos, bem como o detalhamento dos requisitos
estabelecidos nesta Resolução:
I - American Educational Research Association, American Psychological Association & National
Council on Measurement in Education (2014). Standards for Educational and Psychological Testing. New
York: American Educational Research Association;
II - International Testing Comission (2005). Diretrizes para o Uso de Testes: International Test
Commission. (http://www.intestcom.org);
III - International Testing Comission (2005). ITC Guidelines for Translating and Adaptating Tests.
(http://www.intestcom.org);
IV - International Testing Comission (2014). The ITC Guidelines on the Security of Tests, Examinations,
and Other Assessments.
(http://www.intestcom.org);
V - International Testing Comission (2013). ITC Guidelines on Quality Control in Scoring, Test Analysis,
and Reporting of Test Scores. (http://www.intestcom.org);
VI - International Testing Comission (2005). ITC Guidelines on ComputerBased and Internet Delivered
Testing. (http://www.intestcom.org);
VII - CFP (2013). Cartilha de Avaliação Psicológica. (http://site.cfp.org.br/publicacao/cartilha-
avaliacao-psicologica-2013/).

Art. 6º – Os testes psicológicos, para serem reconhecidos para uso profissional de psicólogas e
psicólogos, devem possuir consistência técnico-científica e atender os requisitos mínimos obrigatórios,
listados a seguir:
I - apresentação de fundamentação teórica, com especial ênfase na definição do(s) construto(s),
descrevendo seus aspectos constitutivo e operacional;
II - definição dos objetivos do teste e contexto de aplicação, detalhando a população-alvo;
III - pertinência teórica e qualidade técnica dos estímulos utilizados nos testes;
IV - apresentação de evidências empíricas sobre as características técnicas dos itens do teste, exceto
para os métodos projetivos/expressivos;

Apostila gerada especialmente para: Betânia Mueller 023.712.580-32


. 20
V - apresentação de evidências empíricas de validade e estimativas de precisão das interpretações
para os resultados do teste, caracterizando os procedimentos e os critérios adotados na investigação;
VI - apresentação do sistema de correção e interpretação dos escores, explicitando a lógica que
fundamenta o procedimento, em função do sistema de interpretação adotado, que pode ser:
a) Referenciada à norma, devendo, nesse caso, relatar as características da amostra de normatização
de maneira explícita e exaustiva, preferencialmente comparando com estimativas nacionais,
possibilitando o julgamento do nível de representatividade do grupo de referência usado para a
transformação dos escores.
b) Diferente da interpretação referenciada à norma, devendo, nesse caso, explicar o embasamento
teórico e justificar a lógica do procedimento de interpretação utilizado.
VII - apresentação explícita da aplicação e correção para que haja a garantia da uniformidade dos
procedimentos.
Parágrafo único - Testes psicológicos estrangeiros adaptados para o Brasil devem atender aos incisos
supracitados.
VIII - Atenção aos requisitos explicitados nos artigos 30, 31, 32 e 33.

Art. 7º - O manual do teste psicológico deve atender a todos os incisos do Art. 6° e incluir a ficha síntese
do teste (com objetivo, público-alvo, material, aplicação e correção) e exemplo(s) de utilização,
contemplando a administração, aferição, análise e interpretação dos resultados.

Art. 8º - Os requisitos mínimos obrigatórios são aqueles contidos no Anexo I desta Resolução,
denominado Formulário de Avaliação da Qualidade de Testes Psicológicos.
Parágrafo Único - O Anexo que trata o caput deste Artigo é parte integrante desta Resolução.

Art. 9º - A submissão do teste psicológico para avaliação deverá ser realizada por meio do SATEPSI.
Parágrafo único – A submissão de teste psicológico ao SATEPSI está condicionada à indicação de
responsável técnico com CRP ativo.

Art. 10 - Os testes psicológicos submetidos ao SATEPSI serão avaliados pela CCAP, cuja constituição
e funcionamento seguirá o estabelecido pela Resolução CFP nº 003/2017, ou resoluções que venham a
substituí-la ou alterá-la.

Art. 11 - A tramitação dos testes psicológicos submetidos ao SATEPSI obedecerá às seguintes etapas:
I - Submissão on-line ao SATEPSI;
II - Designação de 2 (dois) pareceristas ad hoc para análise do teste psicológico;
III - Avaliação do teste psicológico por pareceristas;
IV - Análise dos pareceres emitidos e elaboração de relatório conclusivo por membro da CCAP;
V - Apreciação do relatório conclusivo pelo colegiado da CCAP;
VI - Apreciação e decisão pelo Plenário do CFP do relatório da CCAP;
VII - Envio do parecer final do CFP aos requerentes;
VIII - Prazo para interposição de recurso;
IX - Análise do recurso pela CCAP;
X - Apreciação da análise do recurso pelo Plenário do CFP;
XI - Envio do parecer final sobre o recurso aos requerentes.
§ 1º - A designação de pareceristas será feita pela CCAP, considerando a lista de pareceristas ad hoc
vigente à época.
§2º - Quando da análise dos pareceres pelo colegiado da CCAP, esclarecimentos ou informações
complementares poderão ser solicitadas ao responsável técnico do teste psicológico.
§3º - O CFP encaminhará o resultado da avaliação ao requerente, e quando este for desfavorável, o
requerente poderá apresentar recurso por meio do SATEPSI no prazo de até 30 dias, a contar da data de
envio da comunicação do resultado.
§4º - A análise do recurso será realizada pela CCAP na reunião subsequente ao recebimento do
mesmo.
§5º - A avaliação final desfavorável prevalecerá quando, mediante análise do recurso, a avaliação da
CCAP se mantiver, ou quando o recurso não for apresentado no prazo estabelecido.

Art. 12 - Os prazos para cada etapa descrita no Art. 11 desta Resolução são de até:

Apostila gerada especialmente para: Betânia Mueller 023.712.580-32


. 21
I – 30 (trinta) dias, a partir da data de recebimento do teste psicológico completo por meio da plataforma
on-line do SATEPSI, e, se for o caso, do envio de materiais não digitalizáveis, para a designação de 2
(dois) pareceristas ad hoc;
II - 20 (vinte) dias, a partir da data de aceitação da atribuição pelos pareceristas para a emissão dos
pareceres, podendo esse prazo ser prorrogado por igual período, mediante solicitação realizada pelo
parecerista no próprio SATEPSI.
III - 30 (trinta) dias, a partir do recebimento dos pareceres para elaboração de relatório e emissão de
parecer pela CCAP. Nos casos em que houver necessidade de esclarecimentos ou acréscimo de
informações a pedido da CCAP, o prazo de 30 dias será contado a partir do fornecimento destas
informações pelo responsável técnico;
IV - 30 (trinta) dias para emissão e decisão do Plenário do CFP, a partir do relatório final da CCAP.
V - 30 (trinta) dias, a partir da comunicação da decisão do Plenário do CFP para interposição de recurso
pelo requerente;
VI - 30 (trinta) dias a partir do recebimento do recurso, para análise e parecer pela CCAP, quando
houver prazo hábil para análise.
VII - 30 (trinta) dias para emissão e decisão do Plenário do CFP, a partir do relatório final do recurso
pela CCAP.
Parágrafo único - Os prazos previstos no caput deste artigo serão calculados em dias úteis e seguirão
o calendário de Reuniões da CCAP e da Plenária do CFP.

Art. 13 - Os testes psicológicos com parecer final desfavorável do CFP poderão ser reapresentados a
qualquer tempo e seguirão o trâmite previsto no Artigo 12 desta Resolução.

Art. 14 - Os estudos de validade, precisão e normas dos testes psicológicos terão prazo máximo de 15
(quinze) anos, a contar da data da aprovação do teste psicológico pela Plenária do CFP.
§1º - Caso novas versões do teste sejam apresentadas e recebam parecer favorável, versões
anteriores poderão ser utilizadas até o vencimento dos estudos de normatização, validade e precisão.
§2º - Os testes com parecer favorável no SATEPSI com data anterior à publicação desta Resolução
terão sua vigência mantida para os estudos de validade (20 anos) e para normas (15 anos).
§3º - Não sendo apresentada a revisão no prazo estabelecido no caput deste artigo, o teste psicológico
perderá a condição de uso e será excluído da relação de testes com parecer favorável pelo SATEPSI.

Art. 15 - A responsabilidade pela submissão dos estudos de validade, precisão e de atualização de


normas dos testes psicológicos ao SATEPSI, será do responsável técnico pelo teste ou psicóloga ou
psicólogo legalmente constituído.

Art. 16 - Todos os testes psicológicos estão sujeitos ao disposto nesta Resolução, considerando que:
§1o - Os manuais de testes psicológicos devem informar que sua comercialização e seu uso é restrito
a psicólogas e psicólogos, regularmente inscritos no CRP.
§2o - Na comercialização de testes psicológicos, as editoras manterão procedimento de controle, no
qual conste o nome da psicóloga e do psicólogo que os adquiriu, o seu número de inscrição no CRP e
o(s) número(s) de série dos testes adquiridos.

Art. 17 - Os CRPs adotarão as providências para o cumprimento desta Resolução, em suas respectivas
jurisdições, procedendo à orientação, à fiscalização e ao julgamento, podendo:
I - notificar a psicóloga ou psicólogo a respeito de irregularidade, dando prazo para a sua regularização;
II - representar contra profissional ou pessoa jurídica por falta disciplinar;
III - dar conhecimento às autoridades competentes de possíveis irregularidades.
Parágrafo único - Os Conselhos Regionais de Psicologia (CRPs) manterão cadastro atualizado de
pessoas físicas e jurídicas que, em sua jurisdição, comercializem testes psicológicos.

DA SUBMISSÃO AO SATEPSI DE VERSÕES EQUIVALENTES DE TESTES PSICOLÓGICOS


APROVADOS (INFORMATIZADAS E NÃO INFORMATIZADA)

Art. 18 - Será considerada versão equivalente de um teste psicológico aquela com formato diferente
de aplicação descrita na versão aprovada pelo SATEPSI.

Art. 19 - Formato de aplicação diferente daquele descrito no manual do teste aprovado pelo SATEPSI
deverá ser submetido para apreciação da CCAP e terá a seguinte tramitação:

Apostila gerada especialmente para: Betânia Mueller 023.712.580-32


. 22
I - Recepção;
II - Análise;
III - Avaliação;
IV - Comunicação da avaliação aos requerentes, com prazo para recurso;
V - Análise de recurso;
VI - Avaliação final.
§1º - Formato de correção diferente daquele descrito no manual do teste psicológico aprovado pelo
SATEPSI não necessita de nova avaliação, desde que os procedimentos descritos nos seus respectivos
manuais sejam rigorosamente seguidos.
§2º - Compete ao responsável técnico a submissão ao SATEPSI de estudos de equivalência dos
diferentes formatos de aplicação.

Art. 20 - Os procedimentos e prazos para cada etapa descrita no Art. 19 desta Resolução são os
seguintes:
I – O envio deverá ser feito de forma on-line pelo SATEPSI, por meio do preenchimento dos dados de
identificação do teste psicológico e da inserção dos seguintes documentos:
a) arquivo contendo o estudo de equivalência entre os diferentes formatos de aplicação;
b) arquivo digital contendo a versão aprovada do manual;
c) carta de anuência do responsável técnico do teste psicológico aprovado no SATEPSI.
II – No ato do envio, o requerente deverá assinalar a concordância de que o estudo de equivalência
realizado tomou como base o manual da versão aprovada pelo SATEPSI.
III - O material será analisado por 1 (um) parecerista ad hoc, que terá um prazo de 20 dias a partir da
data de aceitação da atribuição para emitir o parecer, podendo esse prazo ser prorrogado por igual
período, mediante solicitação realizada no próprio sistema do SATEPSI.
IV - Após recebimento do parecer, a CCAP terá um prazo de 30 dias para emitir seu relatório
conclusivo, que será enviado para decisão do Plenário do CFP.
V - A avaliação poderá ser favorável quando, por decisão do Plenário do CFP, a versão apresentada
possua evidência favorável quanto à equivalência entre as versões do instrumento, ou desfavorável,
quando, por decisão do Plenário do CFP, a análise indicar divergências significativas entre as versões.
Nesse caso, o parecer deverá apresentar as razões, bem como as orientações para que o problema seja
sanado.
VI - Após o envio da comunicação da avaliação, e nos casos em que ela for desfavorável, o requerente
poderá apresentar recurso no prazo de 30 dias, a contar da data do envio da comunicação do resultado.
VII - A análise do recurso à avaliação desfavorável será realizada pela CCAP, que terá o prazo de 30
dias, a contar da data do recebimento do recurso do requerente.
VIII - A CCAP encaminhará seu parecer para a Plenária do CFP, que fará a deliberação final.
Parágrafo único - Os prazos previstos no caput deste artigo serão calculados em dias úteis e seguirão
o calendário de Reuniões da CCAP e do Plenário do CFP.

DA ATUALIZAÇÃO DE NORMAS DE TESTES PSICOLÓGICOS

Art. 21- Define-se Atualização de Normas o processo de elaboração de novos estudos normativos para
testes psicológicos aprovados e com evidências de validade vigentes.
§1º - Não se trata de atualização de normas o estudo com amostras que possuam características
sóciodemográficas diferentes das especificadas no Manual do teste aprovado pelo SATEPSI.
§2º - Nesse caso, o material deverá ser submetido à nova avaliação pelo SATEPSI, seguindo as
normas desta Resolução, incluindo-se as novas evidências de validade e estudos de precisão.

Art. 22 - O material de atualização de normas deverá considerar os seguintes aspectos:


I - Os resultados deverão ser decorrentes de coleta de dados com nova amostra de participantes, que
contemple um estudo independente da versão aprovada pelo SATEPSI, abarcando os critérios desta
Resolução.
II - Os resultados deverão contemplar, preferencialmente, a representação demográfica de distintas
regiões geopolíticas brasileiras.

Art. 23 – Os procedimentos para atualização das normas terão tramitação interna na CCAP, de acordo
com as seguintes etapas:
I - Recepção;
II - Análise;

Apostila gerada especialmente para: Betânia Mueller 023.712.580-32


. 23
III - Avaliação;
IV - Comunicação da avaliação aos requerentes, com prazo para recurso;
V - Análise de recurso;
VI - Avaliação final.

Art. 24 - Os procedimentos e prazos para cada etapa descrita no Art. 23 desta Resolução são os
seguintes:
I - O envio deverá ser feito on-line pelo site do SATEPSI por meio do preenchimento dos dados de
identificação do teste psicológico e da inserção dos seguintes documentos:
a) estudo que gerou as novas normas, com descrição detalhada dos participantes, do período da coleta
de dados e dos índices de precisão dos escores/indicadores;
b) arquivo digital contendo a versão aprovada do manual;
c) carta de anuência do responsável técnico do teste psicológico aprovado no SATEPSI.
II – O material será analisado pela CCAP, que terá um prazo de 60 dias a partir do recebimento da
solicitação, para encaminhar sua deliberação ao Plenário do CFP.
III - A avaliação poderá ser favorável quando, por decisão do Plenário do CFP, a atualização de normas
contemplar as determinações desta Resolução, ou desfavorável, quando, por decisão do Plenário do
CFP, a análise indicar que a atualização das normas não está em consonância com a referida Resolução.
No caso de parecer desfavorável, deverão ser apresentadas as razões, bem como as orientações para
que o problema seja sanado.
IV - Após o envio da comunicação da avaliação, e nos casos em que ela for desfavorável, o requerente
poderá apresentar recurso no prazo de 30 dias, a contar da data do envio da comunicação do resultado.
V - A análise do recurso à avaliação desfavorável será realizada pela CCAP, que terá o prazo de 30
dias, a contar da data do recebimento do recurso do requerente.
VI - A CCAP encaminhará seu parecer para a Plenária do CFP, que fará a deliberação final.
Parágrafo único - Os prazos previstos no caput deste artigo serão calculados em dias úteis e seguirão
o calendário de Reuniões da CCAP e do Plenário do CFP.

Art. 25 - As normas atualizadas, a partir da data de aprovação, devem ser disponibilizadas juntamente
com o teste psicológico. Cabe aos autores, editores, laboratórios, instituições e responsáveis técnicos do
teste determinarem de que forma tal disponibilização será feita, não podendo este ser utilizado sem a
versão mais atualizada de suas normas aprovadas pelo SATEPSI.
Parágrafo único: A partir da data de aprovação das normas atualizadas, os autores, editoras,
laboratórios e/ou responsáveis técnicos do material terão o prazo de 180 dias para aplicar o disposto no
caput deste artigo.

DA ATUALIZAÇÃO DE ESTUDOS DE VALIDADE DE TESTES PSICOLÓGICOS

Art. 26 - Define-se Atualização de Estudos de Validade o processo de elaboração ou compilação de


novos estudos de evidências de validade que não constem no manual de teste psicológico com parecer
favorável pelo SATEPSI.

Art. 27 - Os procedimentos para atualização de estudos de validade deverão ser submetidos para
apreciação da CCAP, e terá a seguinte tramitação:
I - Recepção;
II - Análise;
III - Avaliação;
IV - Comunicação da avaliação aos requerentes, com prazo para recurso;
V - Análise de recurso;
VI - Avaliação Final.

Art. 28 - Os procedimentos e prazos para cada etapa descrita no Art. 27 desta Resolução são os
seguintes:
I - O envio deverá ser feito on-line pelo site do SATEPSI, por meio do preenchimento dos dados de
identificação do teste psicológico e da inserção dos seguintes documentos:
a) estudos com as novas evidências de validade, contendo a descrição detalhada dos participantes;
b) arquivo digital contendo a versão aprovada do manual;
c) carta de anuência do responsável técnico do teste psicológico aprovado no SATEPSI.

Apostila gerada especialmente para: Betânia Mueller 023.712.580-32


. 24
II - O material será analisado por 1 (um) parecerista ad hoc, que terá um prazo de 20 dias a partir da
data de aceitação da atribuição para emitir o parecer, podendo esse prazo ser prorrogado por igual
período, mediante solicitação realizada no SATEPSI.
III - A CCAP terá um prazo de 30 dias para emitir seu parecer, que será enviado para decisão do
Plenário do CFP.
IV - A avaliação poderá ser favorável quando, por decisão do Plenário do CFP, a versão apresentada
possuir evidência favorável quanto aos estudos de evidência de validade, ou desfavorável, quando, por
decisão do Plenário do CFP, a análise não indicar novos estudos de evidência de validade para o teste.
Nesse caso, o parecer deverá apresentar as razões, bem como as orientações para que o problema seja
sanado.
V - Após o envio da comunicação da avaliação, e nos casos em que ela for desfavorável, o requerente
poderá apresentar recurso no prazo de 30 dias, a contar da data do envio da comunicação do resultado.
VI - A análise do recurso à avaliação desfavorável será realizada pela CCAP, que terá o prazo de 30
dias, a contar da data do recebimento do recurso do requerente.
VII - A CCAP encaminhará seu parecer para a Plenária do CFP, que fará a deliberação final.
Parágrafo único - Os prazos previstos no caput deste artigo serão calculados em dias úteis e seguirão
o calendário de Reuniões da CCAP e do Plenário do Conselho Federal de Psicologia.

Art. 29 - Os novos estudos de validade, a partir da data de aprovação, devem ser disponibilizados,
juntamente com o teste psicológico comercializado. Cabe aos autores, editores, laboratórios e
responsáveis técnicos do teste psicológico determinarem de que forma tal disponibilização será feita, não
podendo este ser comercializado sem a versão mais atualizada dos estudos de validade aprovada pelo
SATEPSI.
Parágrafo único - A partir da data de aprovação dos novos estudos de validade, os autores, editoras,
laboratórios e/ou responsáveis técnicos do material terão o prazo de 180 dias para aplicar o disposto no
caput deste artigo.

JUSTIÇA E PROTEÇÃO DOS DIREITOS HUMANOS NA AVALIAÇÃO PSICOLÓGICA

Art. 30 - Na Avaliação Psicológica, a psicóloga ou psicólogo deverão considerar os princípios e artigos


previstos no Código de Ética Profissional das psicólogas e dos psicólogos, bem como atender aos
requisitos técnicos e científicos definidos nesta Resolução.

Art. 31 - À psicóloga ou ao psicólogo, na produção, validação, tradução, adaptação, normatização,


comercialização e aplicação de testes psicológicos, é vedado:
a) realizar atividades que caracterizem negligência, preconceito, exploração, violência, crueldade ou
opressão;
b) induzir a convicções políticas, filosóficas, morais, ideológicas, religiosas, raciais, de orientação
sexual e identidade de gênero;
c) favorecer o uso de conhecimento da ciência psicológica e normatizar a utilização de práticas
psicológicas como instrumentos de castigo, tortura ou qualquer forma de violência.

Art. 32 - As psicólogas e os psicólogos não poderão elaborar, validar, traduzir, adaptar, normatizar,
comercializar e fomentar instrumentos ou técnicas psicológicas, para criar, manter ou reforçar
preconceitos, estigmas ou estereótipos.

Art. 33 - A psicóloga e o psicólogo, na realização de estudos, pesquisas e atividades voltadas para a


produção de conhecimento e desenvolvimento de tecnologias, atuarão considerando os processos de
desenvolvimento humano, configurações familiares, conjugalidade, sexualidade, orientação sexual,
identidade de gênero, identidade étnico-racial, características das pessoas com deficiência, classe social,
e intimidade como construções sociais, históricas e culturais.

Art. 34 - Casos omissos ou não referidos nesta Resolução serão analisados no âmbito da CCAP e
deliberados pelo Plenário do CFP.

Art. 35 - O descumprimento ao que dispõe a presente Resolução sujeitará o responsável às


penalidades da lei e das Resoluções editadas pelo Conselho Federal de Psicologia.

Apostila gerada especialmente para: Betânia Mueller 023.712.580-32


. 25
Art. 36 - Esta Resolução entrará em vigor na data de sua publicação, revogando-se as Resoluções
CFP nº 002/2003 e 005/2012, as Notas Técnicas n° 01/2017 e n° 02/2017 e disposições em contrário.

Rogério Giannini
Conselheiro Presidente
Conselho Federal de Psicologia

RESOLUÇÃO Nº 1, DE 29 DE JANEIRO DE 20184

Estabelece normas de atuação para as psicólogas e os psicólogos em relação às pessoas transexuais


e travestis.

O CONSELHO FEDERAL DE PSICOLOGIA, no uso de suas atribuições legais e regimentais, que lhe
são conferidas pela Lei n.º 5.766, de 20 de dezembro de 1971, e pelo Decreto nº 79.822, de 17 de junho
de 1977;
CONSIDERANDO os princípios fundamentais previstos no Art. 1º da Constituição Federal de 1988,
que estabelece a dignidade da pessoa humana como fundamento do Estado Democrático de Direito, e o
Art. 5º, que dispõe que “todos são iguais perante a lei, sem distinção de qualquer natureza”;
CONSIDERANDO o Art. 1º da Declaração Universal dos Direitos Humanos, de 10 de dezembro de
1948, o qual enuncia: “todos os seres humanos nascem livres e iguais em dignidade e em direitos.
Dotados de razão e de consciência, devem agir uns para com os outros em espírito de fraternidade”;
CONSIDERANDO os Princípios sobre a aplicação da legislação internacional de direitos humanos em
relação à orientação sexual e identidade de gênero presentes na Convenção de Yogyakarta, de novembro
de 2006;
CONSIDERANDO a Declaração de Durban – Conferência Mundial contra o Racismo, Discriminação
Racial, Xenofobia e Intolerância Correlata –, que reafirma o princípio de igualdade e de não discriminação,
adotada em 8 de setembro de 2001;
CONSIDERANDO a Política Nacional de Saúde Integral de Lésbicas, Gays, Bissexuais, Travestis e
Transexuais, publicada em 2013 pelo Ministério da Saúde;
CONSIDERANDO o Código de Ética Profissional das Psicólogas e dos Psicólogos, editado por meio
da Resolução CFP nº 10/2005, de 21 de julho de 2005;
CONSIDERANDO as expressões e identidades de gênero como possibilidades da existência humana,
as quais não devem ser compreendidas como psicopatologias, transtornos mentais, desvios e/ou
inadequações;
CONSIDERANDO que expressão de gênero refere-se à forma como cada sujeito apresenta-se a partir
do que a cultura estabelece como sendo da ordem do feminino, do masculino ou de outros gêneros;
CONSIDERANDO que identidade de gênero refere-se à experiência interna e individual do gênero de
cada pessoa, que pode ou não corresponder ao sexo atribuído no nascimento, incluindo o senso pessoal
do corpo e outras expressões de gênero;
CONSIDERANDO que cisnormavidade refere-se ao regramento social que reduz a divisão das
pessoas apenas a homens e mulheres, com papéis sociais estabelecidos como naturais, postula a
heterossexualidade como única orientação sexual e considera a conjugalidade apenas entre homens e
mulheres cisgêneros;
CONSIDERANDO a cisnormavidade como discursos e práticas que excluem, patologizam e violentam
pessoas cujas experiências não expressam e/ou não possuem identidade de gênero concordante com
aquela designada no nascimento;
CONSIDERANDO que a autodeterminação constitui-se em um processo que garante a autonomia de
cada sujeito para determinar sua identidade de gênero;
CONSIDERANDO que a estrutura das sociedades ocidentais estabelece padrões de sexualidade e
gênero que permitem preconceitos, discriminações e vulnerabilidades às pessoas transexuais, travestis
e pessoas com outras expressões e identidades de gênero não cisnormavas;

RESOLVE:

Art. 1º - As psicólogas e os psicólogos, em sua prática profissional, atuarão segundo os princípios


éticos da profissão, contribuindo com o seu conhecimento para uma reflexão voltada à eliminação da
transfobia e do preconceito em relação às pessoas transexuais e travestis.

4
https://site.cfp.org.br/wp-content/uploads/2018/01/Resolu%C3%A7%C3%A3o-CFP-01-2018.pdf

Apostila gerada especialmente para: Betânia Mueller 023.712.580-32


. 26
Art. 2º - As psicólogas e os psicólogos, no exercício profissional, não exercerão qualquer ação que
favoreça a discriminação ou preconceito em relação às pessoas transexuais e travestis.

Art. 3º - As psicólogas e os psicólogos, no exercício profissional, não serão coniventes e nem se


omitirão perante a discriminação de pessoas transexuais e travestis.

Art. 4º - As psicólogas e os psicólogos, em sua prática profissional, não se utilizarão de instrumentos


ou técnicas psicológicas para criar, manter ou reforçar preconceitos, estigmas, estereótipos ou
discriminações em relação às pessoas transexuais e travestis.

Art. 5º - As psicólogas e os psicólogos, no exercício de sua prática profissional, não colaborarão com
eventos ou serviços que contribuam para o desenvolvimento de culturas institucionais discriminatórias em
relação às transexualidades e travestilidades.

Art. 6º - As psicólogas e os psicólogos, no âmbito de sua atuação profissional, não parcitiparão de


pronunciamentos, inclusive nos meios de comunicação e internet, que legitimem ou reforcem o
preconceito em relação às pessoas transexuais e travestis.

Art. 7º - As psicólogas e os psicólogos, no exercício profissional, não exercerão qualquer ação que
favoreça a patologização das pessoas transexuais e travestis.
Parágrafo único: As psicólogas e os psicólogos, na sua prática profissional, reconhecerão e legitimarão
a autodeterminação das pessoas transexuais e travestis em relação às suas identidades de gênero.

Art. 8º - É vedado às psicólogas e aos psicólogos, na sua prática profissional, propor, realizar ou
colaborar, sob uma perspectiva patologizante, com eventos ou serviços privados, públicos, institucionais,
comunitários ou promocionais que visem a terapias de conversão, reversão, readequação ou reorientação
de identidade de gênero das pessoas transexuais e travestis.

Art. 9º - Esta Resolução entra em vigor na data de sua publicação.

Questões

01. (IGEPREV/PA - Técnico Previdenciário B - IADES/2018) Segundo a Resolução n° 9/2018 do


Conselho Federal de Psicologia, que estabelece diretrizes para a realização de avaliação psicológica no
exercício profissional de psicólogas e psicólogos, são consideradas(os) fontes fundamentais de
informação
(A) entrevistas psicológicas e registros de observação de comportamentos.
(B) registros de observação de comportamentos e relatórios de equipes multiprofissionais.
(C) testes psicológicos em aprovação pelo Conselho Federal de Psicologia e anamnese.
(D) relatórios de equipes multiprofissionais e testes psicológicos aprovados pelo Conselho Federal de
Psicologia.
(E) anamnese e relatórios de equipes multiprofissionais.

02. (CRP 2º Região - Psicólogo Orientador - Quadrix/2018) Em relação à Resolução n.° 9/2018 do
CFP, que dispõe sobre a realização de avaliação psicológica no exercício profissional do psicólogo e
regulamenta o Sistema de Avaliação de Testes Psicológicos (SATEPSI), assinale a alternativa incorreta.
(A) é facultativo aos Conselhos Regionais de Psicologia manter cadastro atualizado de pessoas físicas
e jurídicas que, em sua jurisdição, comercializem testes psicológicos, uma vez que tal atribuição cabe às
editoras.
(B) Na comercialização de testes psicológicos, as editoras manterão procedimento de controle, no qual
deverá constar o nome do psicólogo que os adquiriu, seu número de inscrição no CRP e o(s) número(s)
de série dos testes adquiridos.
(C) Em caso de irregularidade nos testes psicológicos, é atribuição dos Conselhos Regionais de
Psicologia a notificação do psicólogo a respeito, dando prazo para a regularização.
(D) Cabe aos Conselhos Regionais de Psicologia a representação contra profissional ou pessoa
jurídica por falta disciplinar.
(E) Os Conselhos Regionais de Psicologia deverão dar conhecimento às autoridades competentes de
possíveis irregularidades no uso ou na comercialização dos testes psicológicos considerados como
favoráveis pelo CFP.

Apostila gerada especialmente para: Betânia Mueller 023.712.580-32


. 27
Gabarito

01.A / 02.A

Comentários

01. Resposta: A
Res 9/2018
Art. 2º - Na realização da Avaliação Psicológica, a psicóloga e o psicólogo devem basear sua decisão,
obrigatoriamente, em métodos e/ou técnicas e/ou instrumentos psicológicos reconhecidos cientificamente
para uso na prática profissional da psicóloga e do psicólogo (fontes fundamentais de informação),
podendo, a depender do contexto, recorrer a procedimentos e recursos auxiliares (fontes complementares
de informação).
Consideram-se fontes de informação:
I – Fontes fundamentais:
[ ]
b) Entrevistas psicológicas, anamnese e/ou;
c) Protocolos ou registros de observação de comportamentos obtidos individualmente ou por meio de
processo grupal e/ou técnicas de grupo.

02. Resposta: A
Res 9/2018
Art. 17 - Os CRPs adotarão as providências para o cumprimento desta Resolução, em suas respectivas
jurisdições, procedendo à orientação, à fiscalização e ao julgamento, podendo:
[ ]
Parágrafo único - Os Conselhos Regionais de Psicologia (CRPs) manterão cadastro atualizado de
pessoas físicas e jurídicas que, em sua jurisdição, comercializem testes psicológicos.

O Estatuto da Criança e do Adolescente e a proteção integral à Infância e à


Juventude. Lei nº 8.069/90 - Estatuto da Criança e do Adolescente e suas
alterações;

LEI Nº 8.069, DE 13 DE JULHO DE 19905


DISPÕE SOBRE O ESTATUTO DA CRIANÇA E DO ADOLESCENTE E DÁ OUTRAS
PROVIDÊNCIAS.

O PRESIDENTE DA REPÚBLICA: Faço saber que o Congresso Nacional decreta e eu sanciono a


seguinte Lei:
Título I
Das Disposições Preliminares

Art. 1º Esta Lei dispõe sobre a proteção integral à criança e ao adolescente.

Art. 2º Considera-se criança, para os efeitos desta Lei, a pessoa até doze anos de idade incompletos,
e adolescente aquela entre doze e dezoito anos de idade.
Parágrafo único. Nos casos expressos em lei, aplica-se excepcionalmente este Estatuto às pessoas
entre dezoito e vinte e um anos de idade.

Art. 3º A criança e o adolescente gozam de todos os direitos fundamentais inerentes à pessoa humana,
sem prejuízo da proteção integral de que trata esta Lei, assegurando-se lhes, por lei ou por outros meios,
todas as oportunidades e facilidades, a fim de lhes facultar o desenvolvimento físico, mental, moral,
espiritual e social, em condições de liberdade e de dignidade.
Parágrafo único. Os direitos enunciados nesta Lei aplicam-se a todas as crianças e adolescentes,
sem discriminação de nascimento, situação familiar, idade, sexo, raça, etnia ou cor, religião ou crença,
deficiência, condição pessoal de desenvolvimento e aprendizagem, condição econômica, ambiente
social, região e local de moradia ou outra condição que diferencie as pessoas, as famílias ou a
comunidade em que vivem. (incluído pela Lei nº 13.257, de 2016)
5
Disponível em: http://www.planalto.gov.br/ccivil_03/leis/L8069Compilado.htm - Acesso em 15/10/2018 às 14h00min.

Apostila gerada especialmente para: Betânia Mueller 023.712.580-32


. 28
Art. 4º É dever da família, da comunidade, da sociedade em geral e do poder público assegurar, com
absoluta prioridade, a efetivação dos direitos referentes à vida, à saúde, à alimentação, à educação, ao
esporte, ao lazer, à profissionalização, à cultura, à dignidade, ao respeito, à liberdade e à convivência
familiar e comunitária.
Parágrafo único. A garantia de prioridade compreende:
a) primazia de receber proteção e socorro em quaisquer circunstâncias;
b) precedência de atendimento nos serviços públicos ou de relevância pública;
c) preferência na formulação e na execução das políticas sociais públicas;
d) destinação privilegiada de recursos públicos nas áreas relacionadas com a proteção à infância e à
juventude.

Art. 5º Nenhuma criança ou adolescente será objeto de qualquer forma de negligência, discriminação,
exploração, violência, crueldade e opressão, punido na forma da lei qualquer atentado, por ação ou
omissão, aos seus direitos fundamentais.

Art. 6º Na interpretação desta Lei levar-se-ão em conta os fins sociais a que ela se dirige, as exigências
do bem comum, os direitos e deveres individuais e coletivos, e a condição peculiar da criança e do
adolescente como pessoas em desenvolvimento.

Título II
Dos Direitos Fundamentais
Capítulo I
Do Direito à Vida e à Saúde

Art. 7º A criança e o adolescente têm direito a proteção à vida e à saúde, mediante a efetivação de
políticas sociais públicas que permitam o nascimento e o desenvolvimento sadio e harmonioso, em
condições dignas de existência.

Art. 8o É assegurado a todas as mulheres o acesso aos programas e às políticas de saúde da mulher
e de planejamento reprodutivo e, às gestantes, nutrição adequada, atenção humanizada à gravidez, ao
parto e ao puerpério e atendimento pré-natal, perinatal e pós-natal integral no âmbito do Sistema Único
de Saúde. (Redação dada pela Lei nº 13.257, de 2016)
§ 1o O atendimento pré-natal será realizado por profissionais da atenção primária. (Redação dada pela
Lei nº 13.257, de 2016)
§ 2o Os profissionais de saúde de referência da gestante garantirão sua vinculação, no último trimestre
da gestação, ao estabelecimento em que será realizado o parto, garantido o direito de opção da mulher.
(Redação dada pela Lei nº 13.257, de 2016)
§ 3o Os serviços de saúde onde o parto for realizado assegurarão às mulheres e aos seus filhos recém-
nascidos alta hospitalar responsável e contra referência na atenção primária, bem como o acesso a outros
serviços e a grupos de apoio à amamentação. (Redação dada pela Lei nº 13.257, de 2016)
§ 4o Incumbe ao poder público proporcionar assistência psicológica à gestante e à mãe, no período pré
e pós-natal, inclusive como forma de prevenir ou minorar as consequências do estado puerperal.
§ 5o A assistência referida no § 4o deste artigo deverá ser prestada também a gestantes e mães que
manifestem interesse em entregar seus filhos para adoção, bem como a gestantes e mães que se
encontrem em situação de privação de liberdade. (Redação dada pela Lei nº 13.257, de 2016)
§ 6o A gestante e a parturiente têm direito a 1 (um) acompanhante de sua preferência durante o período
do pré-natal, do trabalho de parto e do pós-parto imediato. (Incluído pela Lei nº 13.257, de 2016)
§ 7o A gestante deverá receber orientação sobre aleitamento materno, alimentação complementar
saudável e crescimento e desenvolvimento infantil, bem como sobre formas de favorecer a criação de
vínculos afetivos e de estimular o desenvolvimento integral da criança. (Incluído pela Lei nº 13.257, de
2016)
§ 8o A gestante tem direito a acompanhamento saudável durante toda a gestação e a parto natural
cuidadoso, estabelecendo-se a aplicação de cesariana e outras intervenções cirúrgicas por motivos
médicos. (Incluído pela Lei nº 13.257, de 2016)
§ 9o A atenção primária à saúde fará a busca ativa da gestante que não iniciar ou que abandonar as
consultas de pré-natal, bem como da puérpera que não comparecer às consultas pós-parto. (Incluído pela
Lei nº 13.257, de 2016)
§ 10. Incumbe ao poder público garantir, à gestante e à mulher com filho na primeira infância que se
encontrem sob custódia em unidade de privação de liberdade, ambiência que atenda às normas sanitárias

Apostila gerada especialmente para: Betânia Mueller 023.712.580-32


. 29
e assistenciais do Sistema Único de Saúde para o acolhimento do filho, em articulação com o sistema de
ensino competente, visando ao desenvolvimento integral da criança. (Incluído pela Lei nº 13.257, de 2016)

Art. 9º O poder público, as instituições e os empregadores propiciarão condições adequadas ao


aleitamento materno, inclusive aos filhos de mães submetidas a medida privativa de liberdade.
§ 1o Os profissionais das unidades primárias de saúde desenvolverão ações sistemáticas, individuais
ou coletivas, visando ao planejamento, à implementação e à avaliação de ações de promoção, proteção
e apoio ao aleitamento materno e à alimentação complementar saudável, de forma contínua. (Incluído
pela Lei nº 13.257, de 2016)
§ 2o Os serviços de unidades de terapia intensiva neonatal deverão dispor de banco de leite humano
ou unidade de coleta de leite humano. (Incluído pela Lei nº 13.257, de 2016)

Art. 10. Os hospitais e demais estabelecimentos de atenção à saúde de gestantes, públicos e


particulares, são obrigados a:
I - manter registro das atividades desenvolvidas, através de prontuários individuais, pelo prazo de
dezoito anos;
II - identificar o recém-nascido mediante o registro de sua impressão plantar e digital e da impressão
digital da mãe, sem prejuízo de outras formas normatizadas pela autoridade administrativa competente;
III - proceder a exames visando ao diagnóstico e terapêutica de anormalidades no metabolismo do
recém-nascido, bem como prestar orientação aos pais;
IV - fornecer declaração de nascimento onde constem necessariamente as intercorrências do parto e
do desenvolvimento do neonato;
V - manter alojamento conjunto, possibilitando ao neonato a permanência junto à mãe.
VI - acompanhar a prática do processo de amamentação, prestando orientações quanto à técnica
adequada, enquanto a mãe permanecer na unidade hospitalar, utilizando o corpo técnico já existente.
(Incluído pela Lei nº 13.436, de 2017)

Art. 11. É assegurado acesso integral às linhas de cuidado voltadas à saúde da criança e do
adolescente, por intermédio do Sistema Único de Saúde, observado o princípio da equidade no acesso a
ações e serviços para promoção, proteção e recuperação da saúde. (Redação dada pela Lei nº 13.257,
de 2016)
§ 1o A criança e o adolescente com deficiência serão atendidos, sem discriminação ou segregação, em
suas necessidades gerais de saúde e específicas de habilitação e reabilitação. (Redação dada pela Lei
nº 13.257, de 2016)
§ 2o Incumbe ao poder público fornecer gratuitamente, àqueles que necessitarem, medicamentos,
órteses, próteses e outras tecnologias assistivas relativas ao tratamento, habilitação ou reabilitação para
crianças e adolescentes, de acordo com as linhas de cuidado voltadas às suas necessidades específicas.
(Redação dada pela Lei nº 13.257, de 2016)
§ 3o Os profissionais que atuam no cuidado diário ou frequente de crianças na primeira infância
receberão formação específica e permanente para a detecção de sinais de risco para o desenvolvimento
psíquico, bem como para o acompanhamento que se fizer necessário. (Incluído pela Lei nº 13.257, de
2016)

Art. 12. Os estabelecimentos de atendimento à saúde, inclusive as unidades neonatais, de terapia


intensiva e de cuidados intermediários, deverão proporcionar condições para a permanência em tempo
integral de um dos pais ou responsável, nos casos de internação de criança ou adolescente. (Redação
dada pela Lei nº 13.257, de 2016)

Art. 13. Os casos de suspeita ou confirmação de castigo físico, de tratamento cruel ou degradante e
de maus-tratos contra criança ou adolescente serão obrigatoriamente comunicados ao Conselho Tutelar
da respectiva localidade, sem prejuízo de outras providências legais.
§ 1o As gestantes ou mães que manifestem interesse em entregar seus filhos para adoção serão
obrigatoriamente encaminhadas, sem constrangimento, à Justiça da Infância e da Juventude. (Incluído
pela Lei nº 13.257, de 2016)
§ 2o Os serviços de saúde em suas diferentes portas de entrada, os serviços de assistência social em
seu componente especializado, o Centro de Referência Especializado de Assistência Social (Creas) e os
demais órgãos do Sistema de Garantia de Direitos da Criança e do Adolescente deverão conferir máxima
prioridade ao atendimento das crianças na faixa etária da primeira infância com suspeita ou confirmação

Apostila gerada especialmente para: Betânia Mueller 023.712.580-32


. 30
de violência de qualquer natureza, formulando projeto terapêutico singular que inclua intervenção em rede
e, se necessário, acompanhamento domiciliar. (Incluído pela Lei nº 13.257, de 2016)

Art. 14. O Sistema Único de Saúde promoverá programas de assistência médica e odontológica para
a prevenção das enfermidades que ordinariamente afetam a população infantil, e campanhas de
educação sanitária para pais, educadores e alunos.
§ 1º É obrigatória a vacinação das crianças nos casos recomendados pelas autoridades sanitárias.
(Renumerado do parágrafo único pela Lei nº 13.257, de 2016)
§ 2º O Sistema Único de Saúde promoverá a atenção à saúde bucal das crianças e das gestantes, de
forma transversal, integral e inter setorial com as demais linhas de cuidado direcionadas à mulher e à
criança. (Incluído pela Lei nº 13.257, de 2016)
§ 3º A atenção odontológica à criança terá função educativa protetiva e será prestada, inicialmente,
antes de o bebê nascer, por meio de aconselhamento pré-natal, e, posteriormente, no sexto e no décimo
segundo anos de vida, com orientações sobre saúde bucal. (Incluído pela Lei nº 13.257, de 2016)
§ 4º A criança com necessidade de cuidados odontológicos especiais será atendida pelo Sistema Único
de Saúde. (Incluído pela Lei nº 13.257, de 2016)
§ 5º É obrigatória a aplicação a todas as crianças, nos seus primeiros dezoito meses de vida, de
protocolo ou outro instrumento construído com a finalidade de facilitar a detecção, em consulta pediátrica
de acompanhamento da criança, de risco para o seu desenvolvimento psíquico. (Incluído pela Lei nº
13.438, de 2017)

Capítulo II
Do Direito à Liberdade, ao Respeito e à Dignidade

Art. 15. A criança e o adolescente têm direito à liberdade, ao respeito e à dignidade como pessoas
humanas em processo de desenvolvimento e como sujeitos de direitos civis, humanos e sociais
garantidos na Constituição e nas leis.

Art. 16. O direito à liberdade compreende os seguintes aspectos:


I - ir, vir e estar nos logradouros públicos e espaços comunitários, ressalvadas as restrições legais;
II - opinião e expressão;
III - crença e culto religioso;
IV - brincar, praticar esportes e divertir-se;
V - participar da vida familiar e comunitária, sem discriminação;
VI - participar da vida política, na forma da lei;
VII - buscar refúgio, auxílio e orientação.

Art. 17. O direito ao respeito consiste na inviolabilidade da integridade física, psíquica e moral da
criança e do adolescente, abrangendo a preservação da imagem, da identidade, da autonomia, dos
valores, ideias e crenças, dos espaços e objetos pessoais.

Art. 18. É dever de todos velar pela dignidade da criança e do adolescente, pondo-os a salvo de
qualquer tratamento desumano, violento, aterrorizante, vexatório ou constrangedor.

Art. 18-A. A criança e o adolescente têm o direito de ser educados e cuidados sem o uso de castigo
físico ou de tratamento cruel ou degradante, como formas de correção, disciplina, educação ou qualquer
outro pretexto, pelos pais, pelos integrantes da família ampliada, pelos responsáveis, pelos agentes
públicos executores de medidas socioeducativas ou por qualquer pessoa encarregada de cuidar deles,
tratá-los, educá-los ou protegê-los.
Parágrafo único. Para os fins desta Lei, considera-se:
I - castigo físico: ação de natureza disciplinar ou punitiva aplicada com o uso da força física sobre a
criança ou o adolescente que resulte em:
a) sofrimento físico; ou
b) lesão;
II - tratamento cruel ou degradante: conduta ou forma cruel de tratamento em relação à criança ou ao
adolescente que:
a) humilhe;
b) ameace gravemente; ou
c) ridicularize.

Apostila gerada especialmente para: Betânia Mueller 023.712.580-32


. 31
Art. 18-B. Os pais, os integrantes da família ampliada, os responsáveis, os agentes públicos executores
de medidas socioeducativas ou qualquer pessoa encarregada de cuidar de crianças e de adolescentes,
tratá-los, educá-los ou protegê-los que utilizarem castigo físico ou tratamento cruel ou degradante como
formas de correção, disciplina, educação ou qualquer outro pretexto estarão sujeitos, sem prejuízo de
outras sanções cabíveis, às seguintes medidas, que serão aplicadas de acordo com a gravidade do caso:
I - encaminhamento a programa oficial ou comunitário de proteção à família;
II - encaminhamento a tratamento psicológico ou psiquiátrico;
III - encaminhamento a cursos ou programas de orientação;
IV - obrigação de encaminhar a criança a tratamento especializado;
V - advertência.
Parágrafo único. As medidas previstas neste artigo serão aplicadas pelo Conselho Tutelar, sem
prejuízo de outras providências legais.
Capítulo III
Do Direito à Convivência Familiar e Comunitária
Seção I
Disposições Gerais

Art. 19. É direito da criança e do adolescente ser criado e educado no seio de sua família e,
excepcionalmente, em família substituta, assegurada a convivência familiar e comunitária, em ambiente
que garanta seu desenvolvimento integral. (Redação dada pela Lei nº 13.257, de 2016)
§ 1º Toda criança ou adolescente que estiver inserido em programa de acolhimento familiar ou
institucional terá sua situação reavaliada, no máximo, a cada 3 (três) meses, devendo a autoridade
judiciária competente, com base em relatório elaborado por equipe interprofissional ou multidisciplinar,
decidir de forma fundamentada pela possibilidade de reintegração familiar ou pela colocação em família
substituta, em quaisquer das modalidades previstas no art. 28 desta Lei. (Redação dada pela Lei nº
13.509, de 2017)
§ 2º A permanência da criança e do adolescente em programa de acolhimento institucional não se
prolongará por mais de 18 (dezoito meses), salvo comprovada necessidade que atenda ao seu superior
interesse, devidamente fundamentada pela autoridade judiciária. (Redação dada pela Lei nº 13.509, de
2017)
§ 3º A manutenção ou a reintegração de criança ou adolescente à sua família terá preferência em
relação a qualquer outra providência, caso em que será esta incluída em serviços e programas de
proteção, apoio e promoção, nos termos do § 1º do art. 23, dos incisos I e IV do caput do art. 101 e dos
incisos I a IV do caput do art. 129 desta Lei. (Redação dada pela Lei nº 13.257, de 2016)
§ 4º Será garantida a convivência da criança e do adolescente com a mãe ou o pai privado de liberdade,
por meio de visitas periódicas promovidas pelo responsável ou, nas hipóteses de acolhimento
institucional, pela entidade responsável, independentemente de autorização judicial.
§ 5º Será garantida a convivência integral da criança com a mãe adolescente que estiver em
acolhimento institucional. (Incluído pela Lei nº 13.509, de 2017)
§ 6º A mãe adolescente será assistida por equipe especializada multidisciplinar. (Incluído pela Lei nº
13.509, de 2017)

Art. 19-A. A gestante ou mãe que manifeste interesse em entregar seu filho para adoção, antes ou
logo após o nascimento, será encaminhada à Justiça da Infância e da Juventude. (Incluído pela Lei nº
13.509, de 2017)
§ 1º A gestante ou mãe será ouvida pela equipe interprofissional da Justiça da Infância e da Juventude,
que apresentará relatório à autoridade judiciária, considerando inclusive os eventuais efeitos do estado
gestacional e puerperal. (Incluído pela Lei nº 13.509, de 2017)
§ 2º De posse do relatório, a autoridade judiciária poderá determinar o encaminhamento da gestante
ou mãe, mediante sua expressa concordância, à rede pública de saúde e assistência social para
atendimento especializado. (Incluído pela Lei nº 13.509, de 2017)
§ 3º A busca à família extensa, conforme definida nos termos do parágrafo único do art. 25 desta Lei,
respeitará o prazo máximo de 90 (noventa) dias, prorrogável por igual período. (Incluído pela Lei nº
13.509, de 2017)
§ 4º Na hipótese de não haver a indicação do genitor e de não existir outro representante da família
extensa apto a receber a guarda, a autoridade judiciária competente deverá decretar a extinção do poder
familiar e determinar a colocação da criança sob a guarda provisória de quem estiver habilitado a adotá-
la ou de entidade que desenvolva programa de acolhimento familiar ou institucional. (Incluído pela Lei nº
13.509, de 2017)

Apostila gerada especialmente para: Betânia Mueller 023.712.580-32


. 32
§ 5º Após o nascimento da criança, a vontade da mãe ou de ambos os genitores, se houver pai registral
ou pai indicado, deve ser manifestada na audiência a que se refere o § 1º do art. 166 desta Lei, garantido
o sigilo sobre a entrega. (Incluído pela Lei nº 13.509, de 2017)
§ 6º (VETADO). (Incluído pela Lei nº 13.509, de 2017)
§ 7º Os detentores da guarda possuem o prazo de 15 (quinze) dias para propor a ação de adoção,
contado do dia seguinte à data do término do estágio de convivência. (Incluído pela Lei nº 13.509, de
2017)
§ 8º Na hipótese de desistência pelos genitores - manifestada em audiência ou perante a equipe
interprofissional - da entrega da criança após o nascimento, a criança será mantida com os genitores, e
será determinado pela Justiça da Infância e da Juventude o acompanhamento familiar pelo prazo de 180
(cento e oitenta) dias. (Incluído pela Lei nº 13.509, de 2017)
§ 9º É garantido à mãe o direito ao sigilo sobre o nascimento, respeitado o disposto no art. 48 desta
Lei. (Incluído pela Lei nº 13.509, de 2017)
§ 10 (VETADO). (Incluído pela Lei nº 13.509, de 2017)

Art. 19-B. A criança e o adolescente em programa de acolhimento institucional ou familiar poderão


participar de programa de apadrinhamento. (Incluído pela Lei nº 13.509, de 2017)
§ 1º O apadrinhamento consiste em estabelecer e proporcionar à criança e ao adolescente vínculos
externos à instituição para fins de convivência familiar e comunitária e colaboração com o seu
desenvolvimento nos aspectos social, moral, físico, cognitivo, educacional e financeiro. (Incluído pela Lei
nº 13.509, de 2017)
§ 2º (VETADO). (Incluído pela Lei nº 13.509, de 2017)
§ 3º Pessoas jurídicas podem apadrinhar criança ou adolescente a fim de colaborar para o seu
desenvolvimento. (Incluído pela Lei nº 13.509, de 2017)
§ 4º O perfil da criança ou do adolescente a ser apadrinhado será definido no âmbito de cada programa
de apadrinhamento, com prioridade para crianças ou adolescentes com remota possibilidade de
reinserção familiar ou colocação em família adotiva. (Incluído pela Lei nº 13.509, de 2017)
§ 5º Os programas ou serviços de apadrinhamento apoiados pela Justiça da Infância e da Juventude
poderão ser executados por órgãos públicos ou por organizações da sociedade civil. (Incluído pela Lei nº
13.509, de 2017)
§ 6º Se ocorrer violação das regras de apadrinhamento, os responsáveis pelo programa e pelos
serviços de acolhimento deverão imediatamente notificar a autoridade judiciária competente. (Incluído
pela Lei nº 13.509, de 2017)

Art. 20. Os filhos, havidos ou não da relação do casamento, ou por adoção, terão os mesmos direitos
e qualificações, proibidas quaisquer designações discriminatórias relativas à filiação.

Art. 21. O poder familiar será exercido, em igualdade de condições, pelo pai e pela mãe, na forma do
que dispuser a legislação civil, assegurado a qualquer deles o direito de, em caso de discordância,
recorrer à autoridade judiciária competente para a solução da divergência.

Art. 22. Aos pais incumbe o dever de sustento, guarda e educação dos filhos menores, cabendo-lhes
ainda, no interesse destes, a obrigação de cumprir e fazer cumprir as determinações judiciais.
Parágrafo único. A mãe e o pai, ou os responsáveis, têm direitos iguais e deveres e responsabilidades
compartilhados no cuidado e na educação da criança, devendo ser resguardado o direito de transmissão
familiar de suas crenças e culturas, assegurados os direitos da criança estabelecidos nesta Lei. (Incluído
pela Lei nº 13.257, de 2016)

Art. 23. A falta ou a carência de recursos materiais não constitui motivo suficiente para a perda ou a
suspensão do poder familiar.
§ 1o Não existindo outro motivo que por si só autorize a decretação da medida, a criança ou o
adolescente será mantido em sua família de origem, a qual deverá obrigatoriamente ser incluída em
serviços e programas oficiais de proteção, apoio e promoção. (Redação dada pela Lei nº 13.257, de 2016)
§ 2º A condenação criminal do pai ou da mãe não implicará a destituição do poder familiar, exceto na
hipótese de condenação por crime doloso sujeito à pena de reclusão contra outrem igualmente titular do
mesmo poder familiar ou contra filho, filha ou outro descendente. (Redação dada pela Lei nº 13.715, de
2018)

Apostila gerada especialmente para: Betânia Mueller 023.712.580-32


. 33
Art. 24. A perda e a suspensão do poder familiar serão decretadas judicialmente, em procedimento
contraditório, nos casos previstos na legislação civil, bem como na hipótese de descumprimento
injustificado dos deveres e obrigações a que alude o art. 22.

Seção II
Da Família Natural

Art. 25. Entende-se por família natural a comunidade formada pelos pais ou qualquer deles e seus
descendentes.
Parágrafo único. Entende-se por família extensa ou ampliada aquela que se estende para além da
unidade pais e filhos ou da unidade do casal, formada por parentes próximos com os quais a criança ou
adolescente convive e mantém vínculos de afinidade e afetividade.

Art. 26. Os filhos havidos fora do casamento poderão ser reconhecidos pelos pais, conjunta ou
separadamente, no próprio termo de nascimento, por testamento, mediante escritura ou outro documento
público, qualquer que seja a origem da filiação.
Parágrafo único. O reconhecimento pode preceder o nascimento do filho ou suceder-lhe ao
falecimento, se deixar descendentes.

Art. 27. O reconhecimento do estado de filiação é direito personalíssimo, indisponível e imprescritível,


podendo ser exercitado contra os pais ou seus herdeiros, sem qualquer restrição, observado o segredo
de Justiça.

Seção III
Da Família Substituta
Subseção I
Disposições Gerais

Art. 28. A colocação em família substituta far-se-á mediante guarda, tutela ou adoção,
independentemente da situação jurídica da criança ou adolescente, nos termos desta Lei.
§ 1º Sempre que possível, a criança ou o adolescente será previamente ouvido por equipe
interprofissional, respeitado seu estágio de desenvolvimento e grau de compreensão sobre as
implicações da medida, e terá sua opinião devidamente considerada
§ 2º Tratando-se de maior de 12 (doze) anos de idade, será necessário seu consentimento, colhido
em audiência.
§ 3º Na apreciação do pedido levar-se-á em conta o grau de parentesco e a relação de afinidade ou
de afetividade, a fim de evitar ou minorar as consequências decorrentes da medida.
§ 4º Os grupos de irmãos serão colocados sob adoção, tutela ou guarda da mesma família substituta,
ressalvada a comprovada existência de risco de abuso ou outra situação que justifique plenamente a
excepcionalidade de solução diversa, procurando-se, em qualquer caso, evitar o rompimento definitivo
dos vínculos fraternais.
§ 5o A colocação da criança ou adolescente em família substituta será precedida de sua preparação
gradativa e acompanhamento posterior, realizados pela equipe interprofissional a serviço da Justiça da
Infância e da Juventude, preferencialmente com o apoio dos técnicos responsáveis pela execução da
política municipal de garantia do direito à convivência familiar.
§ 6ºEm se tratando de criança ou adolescente indígena ou proveniente de comunidade remanescente
de quilombo, é ainda obrigatório.
I - que sejam consideradas e respeitadas sua identidade social e cultural, os seus costumes e
tradições, bem como suas instituições, desde que não sejam incompatíveis com os direitos fundamentais
reconhecidos por esta Lei e pela Constituição Federal;
II - que a colocação familiar ocorra prioritariamente no seio de sua comunidade ou junto a membros
da mesma etnia;
III - a intervenção e oitiva de representantes do órgão federal responsável pela política indigenista, no
caso de crianças e adolescentes indígenas, e de antropólogos, perante a equipe interprofissional ou
multidisciplinar que irá acompanhar o caso.

Art. 29. Não se deferirá colocação em família substituta a pessoa que revele, por qualquer modo,
incompatibilidade com a natureza da medida ou não ofereça ambiente familiar adequado.

Apostila gerada especialmente para: Betânia Mueller 023.712.580-32


. 34
Art. 30. A colocação em família substituta não admitirá transferência da criança ou adolescente a
terceiros ou a entidades governamentais ou não-governamentais, sem autorização judicial.

Art. 31. A colocação em família substituta estrangeira constitui medida excepcional, somente
admissível na modalidade de adoção.

Art. 32. Ao assumir a guarda ou a tutela, o responsável prestará compromisso de bem e fielmente
desempenhar o encargo, mediante termo nos autos.

Subseção II
Da Guarda

Art. 33. A guarda obriga a prestação de assistência material, moral e educacional à criança ou
adolescente, conferindo a seu detentor o direito de opor-se a terceiros, inclusive aos pais.
§ 1º A guarda destina-se a regularizar a posse de fato, podendo ser deferida, liminar ou
incidentalmente, nos procedimentos de tutela e adoção, exceto no de adoção por estrangeiros.
§ 2º Excepcionalmente, deferir-se-á a guarda, fora dos casos de tutela e adoção, para atender a
situações peculiares ou suprir a falta eventual dos pais ou responsável, podendo ser deferido o direito de
representação para a prática de atos determinados.
§ 3º A guarda confere à criança ou adolescente a condição de dependente, para todos os fins e efeitos
de direito, inclusive previdenciários.
§ 4º Salvo expressa e fundamentada determinação em contrário, da autoridade judiciária competente,
ou quando a medida for aplicada em preparação para adoção, o deferimento da guarda de criança ou
adolescente a terceiros não impede o exercício do direito de visitas pelos pais, assim como o dever de
prestar alimentos, que serão objeto de regulamentação específica, a pedido do interessado ou do
Ministério Público.

Art. 34. O poder público estimulará, por meio de assistência jurídica, incentivos fiscais e subsídios, o
acolhimento, sob a forma de guarda, de criança ou adolescente afastado do convívio familiar.
§ 1º A inclusão da criança ou adolescente em programas de acolhimento familiar terá preferência a
seu acolhimento institucional, observado, em qualquer caso, o caráter temporário e excepcional da
medida, nos termos desta Lei.
§ 2º Na hipótese do § 1º deste artigo a pessoa ou casal cadastrado no programa de acolhimento
familiar poderá receber a criança ou adolescente mediante guarda, observado o disposto nos arts. 28 a
33 desta Lei.
§ 3º A União apoiará a implementação de serviços de acolhimento em família acolhedora como política
pública, os quais deverão dispor de equipe que organize o acolhimento temporário de crianças e de
adolescentes em residências de famílias selecionadas, capacitadas e acompanhadas que não estejam
no cadastro de adoção. (Incluído pela Lei nº 13.257, de 2016)
§ 4º Poderão ser utilizados recursos federais, estaduais, distritais e municipais para a manutenção dos
serviços de acolhimento em família acolhedora, facultando-se o repasse de recursos para a própria família
acolhedora. (Incluído pela Lei nº 13.257, de 2016)

Art. 35. A guarda poderá ser revogada a qualquer tempo, mediante ato judicial fundamentado, ouvido
o Ministério Público.
Subseção III
Da Tutela

Art. 36. A tutela será deferida, nos termos da lei civil, a pessoa de até 18 (dezoito) anos incompletos.
Parágrafo único. O deferimento da tutela pressupõe a prévia decretação da perda ou suspensão do
poder familiar e implica necessariamente o dever de guarda.

Art. 37. O tutor nomeado por testamento ou qualquer documento autêntico, conforme previsto no
parágrafo único do art. 1.729 da Lei nº 10.406, de 10 de janeiro de 2002 - Código Civil, deverá, no prazo
de 30 (trinta) dias após a abertura da sucessão, ingressar com pedido destinado ao controle judicial do
ato, observando o procedimento previsto nos arts. 165 a 170 desta Lei.
Parágrafo único. Na apreciação do pedido, serão observados os requisitos previstos nos arts. 28 e 29
desta Lei, somente sendo deferida a tutela à pessoa indicada na disposição de última vontade, se restar

Apostila gerada especialmente para: Betânia Mueller 023.712.580-32


. 35
comprovado que a medida é vantajosa ao tutelando e que não existe outra pessoa em melhores
condições de assumi-la.

Art. 38. Aplica-se à destituição da tutela o disposto no art. 24.

Subseção IV
Da Adoção

Art. 39. A adoção de criança e de adolescente reger-se-á segundo o disposto nesta Lei.
§ 1º A adoção é medida excepcional e irrevogável, à qual se deve recorrer apenas quando esgotados
os recursos de manutenção da criança ou adolescente na família natural ou extensa, na forma do
parágrafo único do art. 25 desta Lei.
§ 2º É vedada a adoção por procuração.
§ 3º Em caso de conflito entre direitos e interesses do adotando e de outras pessoas, inclusive seus
pais biológicos, devem prevalecer os direitos e os interesses do adotando. (Incluído pela Lei nº 13.509,
de 2017)

Art. 40. O adotando deve contar com, no máximo, dezoito anos à data do pedido, salvo se já estiver
sob a guarda ou tutela dos adotantes.

Art. 41. A adoção atribui a condição de filho ao adotado, com os mesmos direitos e deveres, inclusive
sucessórios, desligando-o de qualquer vínculo com pais e parentes, salvo os impedimentos matrimoniais.
§ 1º Se um dos cônjuges ou concubinos adota o filho do outro, mantêm-se os vínculos de filiação entre
o adotado e o cônjuge ou concubino do adotante e os respectivos parentes.
§ 2º É recíproco o direito sucessório entre o adotado, seus descendentes, o adotante, seus
ascendentes, descendentes e colaterais até o 4º grau, observada a ordem de vocação hereditária.

Art. 42. Podem adotar os maiores de 18 (dezoito) anos, independentemente do estado civil.
§ 1º Não podem adotar os ascendentes e os irmãos do adotando.
§ 2º Para adoção conjunta, é indispensável que os adotantes sejam casados civilmente ou mantenham
união estável, comprovada a estabilidade da família.
§ 3º O adotante há de ser, pelo menos, dezesseis anos mais velho do que o adotando.
§ 4º Os divorciados, os judicialmente separados e os ex companheiros podem adotar conjuntamente,
contanto que acordem sobre a guarda e o regime de visitas e desde que o estágio de convivência tenha
sido iniciado na constância do período de convivência e que seja comprovada a existência de vínculos de
afinidade e afetividade com aquele não detentor da guarda, que justifiquem a excepcionalidade da
concessão.
§ 5º Nos casos do § 4º deste artigo, desde que demonstrado efetivo benefício ao adotando, será
assegurada a guarda compartilhada, conforme previsto no art. 1.584 da Lei nº 10.406, de 10 de janeiro
de 2002 - Código Civil
§ 6o A adoção poderá ser deferida ao adotante que, após inequívoca manifestação de vontade, vier a
falecer no curso do procedimento, antes de prolatada a sentença.

Art. 43. A adoção será deferida quando apresentar reais vantagens para o adotando e fundar-se em
motivos legítimos.

Art. 44. Enquanto não der conta de sua administração e saldar o seu alcance, não pode o tutor ou o
curador adotar o pupilo ou o curatelado.

Art. 45. A adoção depende do consentimento dos pais ou do representante legal do adotando.
§ 1º O consentimento será dispensado em relação à criança ou adolescente cujos pais sejam
desconhecidos ou tenham sido destituídos do poder familiar.
§ 2º Em se tratando de adotando maior de doze anos de idade, será também necessário o seu
consentimento.

Art. 46. A adoção será precedida de estágio de convivência com a criança ou adolescente, pelo prazo
máximo de 90 (noventa) dias, observadas a idade da criança ou adolescente e as peculiaridades do caso.
(Redação dada pela Lei nº 13.509, de 2017)

Apostila gerada especialmente para: Betânia Mueller 023.712.580-32


. 36
§ 1º O estágio de convivência poderá ser dispensado se o adotando já estiver sob a tutela ou guarda
legal do adotante durante tempo suficiente para que seja possível avaliar a conveniência da constituição
do vínculo.
§ 2º A simples guarda de fato não autoriza, por si só, a dispensa da realização do estágio de
convivência.
§ 2º-A. O prazo máximo estabelecido no caput deste artigo pode ser prorrogado por até igual período,
mediante decisão fundamentada da autoridade judiciária. (Incluído pela Lei nº 13.509, de 2017)
§ 3º Em caso de adoção por pessoa ou casal residente ou domiciliado fora do País, o estágio de
convivência será de, no mínimo, 30 (trinta) dias e, no máximo, 45 (quarenta e cinco) dias, prorrogável por
até igual período, uma única vez, mediante decisão fundamentada da autoridade judiciária. (Redação
dada pela Lei nº 13.509, de 2017)
§ 3o-A. Ao final do prazo previsto no § 3o deste artigo, deverá ser apresentado laudo fundamentado
pela equipe mencionada no § 4o deste artigo, que recomendará ou não o deferimento da adoção à
autoridade judiciária. (Incluído pela Lei nº 13.509, de 2017)
§ 4º O estágio de convivência será acompanhado pela equipe interprofissional a serviço da Justiça da
Infância e da Juventude, preferencialmente com apoio dos técnicos responsáveis pela execução da
política de garantia do direito à convivência familiar, que apresentarão relatório minucioso acerca da
conveniência do deferimento da medida.
§ 5º O estágio de convivência será cumprido no território nacional, preferencialmente na comarca de
residência da criança ou adolescente, ou, a critério do juiz, em cidade limítrofe, respeitada, em qualquer
hipótese, a competência do juízo da comarca de residência da criança. (Incluído pela Lei nº 13.509, de
2017)

Art. 47. O vínculo da adoção constitui-se por sentença judicial, que será inscrita no registro civil
mediante mandado do qual não se fornecerá certidão.
§ 1º A inscrição consignará o nome dos adotantes como pais, bem como o nome de seus ascendentes.
§ 2º O mandado judicial, que será arquivado, cancelará o registro original do adotado.
§ 3º A pedido do adotante, o novo registro poderá ser lavrado no Cartório do Registro Civil do Município
de sua residência.
§ 4º Nenhuma observação sobre a origem do ato poderá constar nas certidões do registro.
§ 5º A sentença conferirá ao adotado o nome do adotante e, a pedido de qualquer deles, poderá
determinar a modificação do prenome.
§ 6º Caso a modificação de prenome seja requerida pelo adotante, é obrigatória a oitiva do adotando,
observado o disposto nos §§ 1º e 2º do art. 28 desta Lei.
§ 7º A adoção produz seus efeitos a partir do trânsito em julgado da sentença constitutiva, exceto na
hipótese prevista no § 6º do art. 42 desta Lei, caso em que terá força retroativa à data do óbito.
§ 8º O processo relativo à adoção assim como outros a ele relacionados serão mantidos em arquivo,
admitindo-se seu armazenamento em microfilme ou por outros meios, garantida a sua conservação para
consulta a qualquer tempo.
§ 9º Terão prioridade de tramitação os processos de adoção em que o adotando for criança ou
adolescente com deficiência ou com doença crônica.
§ 10. O prazo máximo para conclusão da ação de adoção será de 120 (cento e vinte) dias, prorrogável
uma única vez por igual período, mediante decisão fundamentada da autoridade judiciária. (Incluído pela
Lei nº 13.509, de 2017)

Art. 48. O adotado tem direito de conhecer sua origem biológica, bem como de obter acesso irrestrito
ao processo no qual a medida foi aplicada e seus eventuais incidentes, após completar 18 (dezoito) anos.
Parágrafo único. O acesso ao processo de adoção poderá ser também deferido ao adotado menor de
18 (dezoito) anos, a seu pedido, assegurada orientação e assistência jurídica e psicológica.

Art. 49. A morte dos adotantes não restabelece o poder familiar dos pais naturais.

Art. 50. A autoridade judiciária manterá, em cada comarca ou foro regional, um registro de crianças e
adolescentes em condições de serem adotados e outro de pessoas interessadas na adoção.
§ 1º O deferimento da inscrição dar-se-á após prévia consulta aos órgãos técnicos do juizado, ouvido
o Ministério Público.
§ 2º Não será deferida a inscrição se o interessado não satisfazer os requisitos legais, ou verificada
qualquer das hipóteses previstas no art. 29.

Apostila gerada especialmente para: Betânia Mueller 023.712.580-32


. 37
§ 3º A inscrição de postulantes à adoção será precedida de um período de preparação psicossocial e
jurídica, orientado pela equipe técnica da Justiça da Infância e da Juventude, preferencialmente com
apoio dos técnicos responsáveis pela execução da política municipal de garantia do direito à convivência
familiar.
§ 4º Sempre que possível e recomendável, a preparação referida no § 3º deste artigo incluirá o contato
com crianças e adolescentes em acolhimento familiar ou institucional em condições de serem adotados,
a ser realizado sob a orientação, supervisão e avaliação da equipe técnica da Justiça da Infância e da
Juventude, com apoio dos técnicos responsáveis pelo programa de acolhimento e pela execução da
política municipal de garantia do direito à convivência familiar.
§ 5º Serão criados e implementados cadastros estaduais e nacional de crianças e adolescentes em
condições de serem adotados e de pessoas ou casais habilitados à adoção.
§ 6º Haverá cadastros distintos para pessoas ou casais residentes fora do País, que somente serão
consultados na inexistência de postulantes nacionais habilitados nos cadastros mencionados no § 5º
deste artigo.
§ 7º As autoridades estaduais e federais em matéria de adoção terão acesso integral aos cadastros,
incumbindo-lhes a troca de informações e a cooperação mútua, para melhoria do sistema.
§ 8º A autoridade judiciária providenciará, no prazo de 48 (quarenta e oito) horas, a inscrição das
crianças e adolescentes em condições de serem adotados que não tiveram colocação familiar na comarca
de origem, e das pessoas ou casais que tiveram deferida sua habilitação à adoção nos cadastros estadual
e nacional referidos no § 5º deste artigo, sob pena de responsabilidade.
§ 9º Compete à Autoridade Central Estadual zelar pela manutenção e correta alimentação dos
cadastros, com posterior comunicação à Autoridade Central Federal Brasileira.
§ 10. Consultados os cadastros e verificada a ausência de pretendentes habilitados residentes no País
com perfil compatível e interesse manifesto pela adoção de criança ou adolescente inscrito nos cadastros
existentes, será realizado o encaminhamento da criança ou adolescente à adoção internacional.
(Redação dada pela Lei nº 13.509, de 2017)
§ 11. Enquanto não localizada pessoa ou casal interessado em sua adoção, a criança ou o
adolescente, sempre que possível e recomendável, será colocado sob guarda de família cadastrada em
programa de acolhimento familiar.
§ 12. A alimentação do cadastro e a convocação criteriosa dos postulantes à adoção serão fiscalizadas
pelo Ministério Público.
§ 13. Somente poderá ser deferida adoção em favor de candidato domiciliado no Brasil não cadastrado
previamente nos termos desta Lei quando:
I - se tratar de pedido de adoção unilateral;
II - for formulada por parente com o qual a criança ou adolescente mantenha vínculos de afinidade e
afetividade;
III - oriundo o pedido de quem detém a tutela ou guarda legal de criança maior de 3 (três) anos ou
adolescente, desde que o lapso de tempo de convivência comprove a fixação de laços de afinidade e
afetividade, e não seja constatada a ocorrência de má-fé ou qualquer das situações previstas nos arts.
237 ou 238 desta Lei.
§ 14. Nas hipóteses previstas no § 13 deste artigo, o candidato deverá comprovar, no curso do
procedimento, que preenche os requisitos necessários à adoção, conforme previsto nesta Lei.
§ 15. Será assegurada prioridade no cadastro a pessoas interessadas em adotar criança ou
adolescente com deficiência, com doença crônica ou com necessidades específicas de saúde, além de
grupo de irmãos. (Incluído pela Lei nº 13.509, de 2017)

Art. 51. Considera-se adoção internacional aquela na qual o pretendente possui residência habitual
em país-parte da Convenção de Haia, de 29 de maio de 1993, Relativa à Proteção das Crianças e à
Cooperação em Matéria de Adoção Internacional, promulgada pelo Decreto no3.087, de 21 junho de 1999,
e deseja adotar criança em outro país-parte da Convenção. (Redação dada pela Lei nº 13.509, de 2017)
§ 1o A adoção internacional de criança ou adolescente brasileiro ou domiciliado no Brasil somente terá
lugar quando restar comprovado:
I - que a colocação em família adotiva é a solução adequada ao caso concreto; (Redação dada pela
Lei nº 13.509, de 2017)
II - que foram esgotadas todas as possibilidades de colocação da criança ou adolescente em família
adotiva brasileira, com a comprovação, certificada nos autos, da inexistência de adotantes habilitados
residentes no Brasil com perfil compatível com a criança ou adolescente, após consulta aos cadastros
mencionados nesta Lei; (Redação dada pela Lei nº 13.509, de 2017)

Apostila gerada especialmente para: Betânia Mueller 023.712.580-32


. 38
III - que, em se tratando de adoção de adolescente, este foi consultado, por meios adequados ao seu
estágio de desenvolvimento, e que se encontra preparado para a medida, mediante parecer elaborado
por equipe interprofissional, observado o disposto nos §§ 1º e 2º do art. 28 desta Lei.
§ 2º Os brasileiros residentes no exterior terão preferência aos estrangeiros, nos casos de adoção
internacional de criança ou adolescente brasileiro.
§ 3º A adoção internacional pressupõe a intervenção das Autoridades Centrais Estaduais e Federal
em matéria de adoção internacional.

Art. 52. A adoção internacional observará o procedimento previsto nos arts. 165 a 170 desta Lei, com
as seguintes adaptações:
I - a pessoa ou casal estrangeiro, interessado em adotar criança ou adolescente brasileiro, deverá
formular pedido de habilitação à adoção perante a Autoridade Central em matéria de adoção internacional
no país de acolhida, assim entendido aquele onde está situada sua residência habitual;
II - se a Autoridade Central do país de acolhida considerar que os solicitantes estão habilitados e aptos
para adotar, emitirá um relatório que contenha informações sobre a identidade, a capacidade jurídica e
adequação dos solicitantes para adotar, sua situação pessoal, familiar e médica, seu meio social, os
motivos que os animam e sua aptidão para assumir uma adoção internacional;
III - a Autoridade Central do país de acolhida enviará o relatório à Autoridade Central Estadual, com
cópia para a Autoridade Central Federal Brasileira;
IV - o relatório será instruído com toda a documentação necessária, incluindo estudo psicossocial
elaborado por equipe interprofissional habilitada e cópia autenticada da legislação pertinente,
acompanhada da respectiva prova de vigência;
V - os documentos em língua estrangeira serão devidamente autenticados pela autoridade consular,
observados os tratados e convenções internacionais, e acompanhados da respectiva tradução, por
tradutor público juramentado;
VI - a Autoridade Central Estadual poderá fazer exigências e solicitar complementação sobre o estudo
psicossocial do postulante estrangeiro à adoção, já realizado no país de acolhida;
VII - verificada, após estudo realizado pela Autoridade Central Estadual, a compatibilidade da
legislação estrangeira com a nacional, além do preenchimento por parte dos postulantes à medida dos
requisitos objetivos e subjetivos necessários ao seu deferimento, tanto à luz do que dispõe esta Lei como
da legislação do país de acolhida, será expedido laudo de habilitação à adoção internacional, que terá
validade por, no máximo, 1 (um) ano;
VIII - de posse do laudo de habilitação, o interessado será autorizado a formalizar pedido de adoção
perante o Juízo da Infância e da Juventude do local em que se encontra a criança ou adolescente,
conforme indicação efetuada pela Autoridade Central Estadual.
§ 1o Se a legislação do país de acolhida assim o autorizar, admite-se que os pedidos de habilitação à
adoção internacional sejam intermediados por organismos credenciados.
§ 2o Incumbe à Autoridade Central Federal Brasileira o credenciamento de organismos nacionais e
estrangeiros encarregados de intermediar pedidos de habilitação à adoção internacional, com posterior
comunicação às Autoridades Centrais Estaduais e publicação nos órgãos oficiais de imprensa e em sítio
próprio da internet.
§ 3o Somente será admissível o credenciamento de organismos que:
I - sejam oriundos de países que ratificaram a Convenção de Haia e estejam devidamente
credenciados pela Autoridade Central do país onde estiverem sediados e no país de acolhida do adotando
para atuar em adoção internacional no Brasil;
II - satisfizerem as condições de integridade moral, competência profissional, experiência e
responsabilidade exigidas pelos países respectivos e pela Autoridade Central Federal Brasileira;
III - forem qualificados por seus padrões éticos e sua formação e experiência para atuar na área de
adoção internacional;
IV - cumprirem os requisitos exigidos pelo ordenamento jurídico brasileiro e pelas normas
estabelecidas pela Autoridade Central Federal Brasileira.
§ 4o Os organismos credenciados deverão ainda:
I - perseguir unicamente fins não lucrativos, nas condições e dentro dos limites fixados pelas
autoridades competentes do país onde estiverem sediados, do país de acolhida e pela Autoridade Central
Federal Brasileira;
II - ser dirigidos e administrados por pessoas qualificadas e de reconhecida idoneidade moral, com
comprovada formação ou experiência para atuar na área de adoção internacional, cadastradas pelo
Departamento de Polícia Federal e aprovadas pela Autoridade Central Federal Brasileira, mediante
publicação de portaria do órgão federal competente;

Apostila gerada especialmente para: Betânia Mueller 023.712.580-32


. 39
III - estar submetidos à supervisão das autoridades competentes do país onde estiverem sediados e
no país de acolhida, inclusive quanto à sua composição, funcionamento e situação financeira;
IV - apresentar à Autoridade Central Federal Brasileira, a cada ano, relatório geral das atividades
desenvolvidas, bem como relatório de acompanhamento das adoções internacionais efetuadas no
período, cuja cópia será encaminhada ao Departamento de Polícia Federal;
V - enviar relatório pós-adotivo semestral para a Autoridade Central Estadual, com cópia para a
Autoridade Central Federal Brasileira, pelo período mínimo de 2 (dois) anos. O envio do relatório será
mantido até a juntada de cópia autenticada do registro civil, estabelecendo a cidadania do país de acolhida
para o adotado;
VI - tomar as medidas necessárias para garantir que os adotantes encaminhem à Autoridade Central
Federal Brasileira cópia da certidão de registro de nascimento estrangeira e do certificado de
nacionalidade tão logo lhes sejam concedidos.
§ 5º A não apresentação dos relatórios referidos no § 4o deste artigo pelo organismo credenciado
poderá acarretar a suspensão de seu credenciamento.
§ 6º O credenciamento de organismo nacional ou estrangeiro encarregado de intermediar pedidos de
adoção internacional terá validade de 2 (dois) anos.
§ 7º A renovação do credenciamento poderá ser concedida mediante requerimento protocolado na
Autoridade Central Federal Brasileira nos 60 (sessenta) dias anteriores ao término do respectivo prazo
de validade.
§ 8º Antes de transitada em julgado a decisão que concedeu a adoção internacional, não será permitida
a saída do adotando do território nacional.
§ 9º Transitada em julgado a decisão, a autoridade judiciária determinará a expedição de alvará com
autorização de viagem, bem como para obtenção de passaporte, constando, obrigatoriamente, as
características da criança ou adolescente adotado, como idade, cor, sexo, eventuais sinais ou traços
peculiares, assim como foto recente e a aposição da impressão digital do seu polegar direito, instruindo
o documento com cópia autenticada da decisão e certidão de trânsito em julgado.
§ 10. A Autoridade Central Federal Brasileira poderá, a qualquer momento, solicitar informações sobre
a situação das crianças e adolescentes adotados.
§ 11. A cobrança de valores por parte dos organismos credenciados, que sejam considerados abusivos
pela Autoridade Central Federal Brasileira e que não estejam devidamente comprovados, é causa de seu
descredenciamento.
§ 12. Uma mesma pessoa ou seu cônjuge não podem ser representados por mais de uma entidade
credenciada para atuar na cooperação em adoção internacional.
§ 13. A habilitação de postulante estrangeiro ou domiciliado fora do Brasil terá validade máxima de 1
(um) ano, podendo ser renovada.
§ 14. É vedado o contato direto de representantes de organismos de adoção, nacionais ou
estrangeiros, com dirigentes de programas de acolhimento institucional ou familiar, assim como com
crianças e adolescentes em condições de serem adotados, sem a devida autorização judicial.
§ 15. A Autoridade Central Federal Brasileira poderá limitar ou suspender a concessão de novos
credenciamentos sempre que julgar necessário, mediante ato administrativo fundamentado.

Art. 52-A. É vedado, sob pena de responsabilidade e descredenciamento, o repasse de recursos


provenientes de organismos estrangeiros encarregados de intermediar pedidos de adoção internacional
a organismos nacionais ou a pessoas físicas.
Parágrafo único. Eventuais repasses somente poderão ser efetuados via Fundo dos Direitos da
Criança e do Adolescente e estarão sujeitos às deliberações do respectivo Conselho de Direitos da
Criança e do Adolescente.

Art. 52-B. A adoção por brasileiro residente no exterior em país ratificante da Convenção de Haia, cujo
processo de adoção tenha sido processado em conformidade com a legislação vigente no país de
residência e atendido o disposto na Alínea “c” do Artigo 17 da referida Convenção, será automaticamente
recepcionada com o reingresso no Brasil.
§ 1º Caso não tenha sido atendido o disposto na Alínea “c” do Artigo 17 da Convenção de Haia, deverá
a sentença ser homologada pelo Superior Tribunal de Justiça.
§ 2º O pretendente brasileiro residente no exterior em país não ratificante da Convenção de Haia, uma
vez reingressado no Brasil, deverá requerer a homologação da sentença estrangeira pelo Superior
Tribunal de Justiça.

Apostila gerada especialmente para: Betânia Mueller 023.712.580-32


. 40
Art. 52-C. Nas adoções internacionais, quando o Brasil for o país de acolhida, a decisão da autoridade
competente do país de origem da criança ou do adolescente será conhecida pela Autoridade Central
Estadual que tiver processado o pedido de habilitação dos pais adotivos, que comunicará o fato à
Autoridade Central Federal e determinará as providências necessárias à expedição do Certificado de
Naturalização Provisório.
§ 1º A Autoridade Central Estadual, ouvido o Ministério Público, somente deixará de reconhecer os
efeitos daquela decisão se restar demonstrado que a adoção é manifestamente contrária à ordem pública
ou não atende ao interesse superior da criança ou do adolescente.
§ 2º Na hipótese de não reconhecimento da adoção, prevista no § 1º deste artigo, o Ministério Público
deverá imediatamente requerer o que for de direito para resguardar os interesses da criança ou do
adolescente, comunicando-se as providências à Autoridade Central Estadual, que fará a comunicação à
Autoridade Central Federal Brasileira e à Autoridade Central do país de origem.

Art. 52-D. Nas adoções internacionais, quando o Brasil for o país de acolhida e a adoção não tenha
sido deferida no país de origem porque a sua legislação a delega ao país de acolhida, ou, ainda, na
hipótese de, mesmo com decisão, a criança ou o adolescente ser oriundo de país que não tenha aderido
à Convenção referida, o processo de adoção seguirá as regras da adoção nacional.

Capítulo IV
Do Direito à Educação, à Cultura, ao Esporte e ao Lazer

Art. 53. A criança e o adolescente têm direito à educação, visando ao pleno desenvolvimento de sua
pessoa, preparo para o exercício da cidadania e qualificação para o trabalho, assegurando-se lhes:
I - igualdade de condições para o acesso e permanência na escola;
II - direito de ser respeitado por seus educadores;
III - direito de contestar critérios avaliativos, podendo recorrer às instâncias escolares superiores;
IV - direito de organização e participação em entidades estudantis;
V - acesso à escola pública e gratuita próxima de sua residência.
Parágrafo único. É direito dos pais ou responsáveis ter ciência do processo pedagógico, bem como
participar da definição das propostas educacionais.

Art. 54. É dever do Estado assegurar à criança e ao adolescente:


I - ensino fundamental, obrigatório e gratuito, inclusive para os que a ele não tiveram acesso na idade
própria;
II - progressiva extensão da obrigatoriedade e gratuidade ao ensino médio;
III - atendimento educacional especializado aos portadores de deficiência, preferencialmente na rede
regular de ensino;
IV - atendimento em creche e pré-escola às crianças de zero a cinco anos de idade; (Redação dada
pela Lei nº 13.306, de 2016)
V - acesso aos níveis mais elevados do ensino, da pesquisa e da criação artística, segundo a
capacidade de cada um;
VI - oferta de ensino noturno regular, adequado às condições do adolescente trabalhador;
VII - atendimento no ensino fundamental, através de programas suplementares de material didático-
escolar, transporte, alimentação e assistência à saúde.
§ 1º O acesso ao ensino obrigatório e gratuito é direito público subjetivo.
§ 2º O não oferecimento do ensino obrigatório pelo poder público ou sua oferta irregular importa
responsabilidade da autoridade competente.
§ 3º Compete ao poder público recensear os educandos no ensino fundamental, fazer-lhes a chamada
e zelar, junto aos pais ou responsável, pela frequência à escola.
Art. 55. Os pais ou responsável têm a obrigação de matricular seus filhos ou pupilos na rede regular
de ensino.

Art. 56. Os dirigentes de estabelecimentos de ensino fundamental comunicarão ao Conselho Tutelar


os casos de:
I - maus-tratos envolvendo seus alunos;
II - reiteração de faltas injustificadas e de evasão escolar, esgotados os recursos escolares;
III - elevados níveis de repetência.

Apostila gerada especialmente para: Betânia Mueller 023.712.580-32


. 41
Art. 57. O poder público estimulará pesquisas, experiências e novas propostas relativas a calendário,
seriação, currículo, metodologia, didática e avaliação, com vistas à inserção de crianças e adolescentes
excluídos do ensino fundamental obrigatório.

Art. 58. No processo educacional respeitar-se-ão os valores culturais, artísticos e históricos próprios
do contexto social da criança e do adolescente, garantindo-se a estes a liberdade da criação e o acesso
às fontes de cultura.

Art. 59. Os municípios, com apoio dos estados e da União, estimularão e facilitarão a destinação de
recursos e espaços para programações culturais, esportivas e de lazer voltadas para a infância e a
juventude.

Capítulo V
Do Direito à Profissionalização e à Proteção no Trabalho

Art. 60. É proibido qualquer trabalho a menores de quatorze anos de idade, salvo na condição de
aprendiz.

Art. 61. A proteção ao trabalho dos adolescentes é regulada por legislação especial, sem prejuízo do
disposto nesta Lei

Art. 62. Considera-se aprendizagem a formação técnico-profissional ministrada segundo as diretrizes


e bases da legislação de educação em vigor.

Art. 63. A formação técnico-profissional obedecerá aos seguintes princípios:


I - garantia de acesso e frequência obrigatória ao ensino regular;
II - atividade compatível com o desenvolvimento do adolescente;
III - horário especial para o exercício das atividades.

Art. 64. Ao adolescente até quatorze anos de idade é assegurada bolsa de aprendizagem.

Art. 65. Ao adolescente aprendiz, maior de quatorze anos, são assegurados os direitos trabalhistas e
previdenciários.

Art. 66. Ao adolescente portador de deficiência é assegurado trabalho protegido.

Art. 67. Ao adolescente empregado, aprendiz, em regime familiar de trabalho, aluno de escola técnica,
assistido em entidade governamental ou não-governamental, é vedado trabalho:
I - noturno, realizado entre as vinte e duas horas de um dia e as cinco horas do dia seguinte;
II - perigoso, insalubre ou penoso;
III - realizado em locais prejudiciais à sua formação e ao seu desenvolvimento físico, psíquico, moral
e social;
IV - realizado em horários e locais que não permitam a frequência à escola.

Art. 68. O programa social que tenha por base o trabalho educativo, sob responsabilidade de entidade
governamental ou não-governamental sem fins lucrativos, deverá assegurar ao adolescente que dele
participe condições de capacitação para o exercício de atividade regular remunerada.
§ 1º Entende-se por trabalho educativo a atividade laboral em que as exigências pedagógicas relativas
ao desenvolvimento pessoal e social do educando prevalecem sobre o aspecto produtivo.
§ 2º A remuneração que o adolescente recebe pelo trabalho efetuado ou a participação na venda dos
produtos de seu trabalho não desfigura o caráter educativo.

Art. 69. O adolescente tem direito à profissionalização e à proteção no trabalho, observados os


seguintes aspectos, entre outros:
I - respeito à condição peculiar de pessoa em desenvolvimento;
II - capacitação profissional adequada ao mercado de trabalho.

Apostila gerada especialmente para: Betânia Mueller 023.712.580-32


. 42
Título III
Da Prevenção
Capítulo I
Disposições Gerais

Art. 70. É dever de todos prevenir a ocorrência de ameaça ou violação dos direitos da criança e do
adolescente.

Art. 70-A. A União, os Estados, o Distrito Federal e os Municípios deverão atuar de forma articulada
na elaboração de políticas públicas e na execução de ações destinadas a coibir o uso de castigo físico
ou de tratamento cruel ou degradante e difundir formas não violentas de educação de crianças e de
adolescentes, tendo como principais ações:
I - a promoção de campanhas educativas permanentes para a divulgação do direito da criança e do
adolescente de serem educados e cuidados sem o uso de castigo físico ou de tratamento cruel ou
degradante e dos instrumentos de proteção aos direitos humanos;
II - a integração com os órgãos do Poder Judiciário, do Ministério Público e da Defensoria Pública, com
o Conselho Tutelar, com os Conselhos de Direitos da Criança e do Adolescente e com as entidades não
governamentais que atuam na promoção, proteção e defesa dos direitos da criança e do adolescente;
III - a formação continuada e a capacitação dos profissionais de saúde, educação e assistência social
e dos demais agentes que atuam na promoção, proteção e defesa dos direitos da criança e do
adolescente para o desenvolvimento das competências necessárias à prevenção, à identificação de
evidências, ao diagnóstico e ao enfrentamento de todas as formas de violência contra a criança e o
adolescente;
IV - o apoio e o incentivo às práticas de resolução pacífica de conflitos que envolvam violência contra
a criança e o adolescente;
V - a inclusão, nas políticas públicas, de ações que visem a garantir os direitos da criança e do
adolescente, desde a atenção pré-natal, e de atividades junto aos pais e responsáveis com o objetivo de
promover a informação, a reflexão, o debate e a orientação sobre alternativas ao uso de castigo físico ou
de tratamento cruel ou degradante no processo educativo;
VI - a promoção de espaços inter setoriais locais para a articulação de ações e a elaboração de planos
de atuação conjunta focados nas famílias em situação de violência, com participação de profissionais de
saúde, de assistência social e de educação e de órgãos de promoção, proteção e defesa dos direitos da
criança e do adolescente;
Parágrafo único. As famílias com crianças e adolescentes com deficiência terão prioridade de
atendimento nas ações e políticas públicas de prevenção e proteção.

Art. 70-B.As entidades, públicas e privadas, que atuem nas áreas a que se refere o art. 71, dentre
outras, devem contar, em seus quadros, com pessoas capacitadas a reconhecer e comunicar ao
Conselho Tutelar suspeitas ou casos de maus-tratos praticados contra crianças e adolescentes.
Parágrafo único. São igualmente responsáveis pela comunicação de que trata este artigo, as pessoas
encarregadas, por razão de cargo, função, ofício, ministério, profissão ou ocupação, do cuidado,
assistência ou guarda de crianças e adolescentes, punível, na forma deste Estatuto, o injustificado
retardamento ou omissão, culposos ou dolosos.

Art. 71. A criança e o adolescente têm direito à informação, cultura, lazer, esportes, diversões,
espetáculos e produtos e serviços que respeitem sua condição peculiar de pessoa em desenvolvimento.

Art. 72. As obrigações previstas nesta Lei não excluem da prevenção especial outras decorrentes dos
princípios por ela adotados.

Art. 73. A inobservância das normas de prevenção importará em responsabilidade da pessoa física ou
jurídica, nos termos desta Lei.

Apostila gerada especialmente para: Betânia Mueller 023.712.580-32


. 43
Capítulo II
Da Prevenção Especial
Seção I
Da informação, Cultura, Lazer, Esportes, Diversões e Espetáculos

Art. 74. O poder público, através do órgão competente, regulará as diversões e espetáculos públicos,
informando sobre a natureza deles, as faixas etárias a que não se recomendem, locais e horários em que
sua apresentação se mostre inadequada.
Parágrafo único. Os responsáveis pelas diversões e espetáculos públicos deverão afixar, em lugar
visível e de fácil acesso, à entrada do local de exibição, informação destacada sobre a natureza do
espetáculo e a faixa etária especificada no certificado de classificação.

Art. 75. Toda criança ou adolescente terá acesso às diversões e espetáculos públicos classificados
como adequados à sua faixa etária.
Parágrafo único. As crianças menores de dez anos somente poderão ingressar e permanecer nos
locais de apresentação ou exibição quando acompanhadas dos pais ou responsável.

Art. 76. As emissoras de rádio e televisão somente exibirão, no horário recomendado para o público
infanto juvenil, programas com finalidades educativas, artísticas, culturais e informativas.
Parágrafo único. Nenhum espetáculo será apresentado ou anunciado sem aviso de sua classificação,
antes de sua transmissão, apresentação ou exibição.

Art. 77. Os proprietários, diretores, gerentes e funcionários de empresas que explorem a venda ou
aluguel de fitas de programação em vídeo cuidarão para que não haja venda ou locação em desacordo
com a classificação atribuída pelo órgão competente.
Parágrafo único. As fitas a que alude este artigo deverão exibir, no invólucro, informação sobre a
natureza da obra e a faixa etária a que se destinam.

Art. 78. As revistas e publicações contendo material impróprio ou inadequado a crianças e


adolescentes deverão ser comercializadas em embalagem lacrada, com a advertência de seu conteúdo.
Parágrafo único. As editoras cuidarão para que as capas que contenham mensagens pornográficas
ou obscenas sejam protegidas com embalagem opaca.

Art. 79. As revistas e publicações destinadas ao público infanto-juvenil não poderão conter ilustrações,
fotografias, legendas, crônicas ou anúncios de bebidas alcoólicas, tabaco, armas e munições, e deverão
respeitar os valores éticos e sociais da pessoa e da família.

Art. 80. Os responsáveis por estabelecimentos que explorem comercialmente bilhar, sinuca ou
congênere ou por casas de jogos, assim entendidas as que realizem apostas, ainda que eventualmente,
cuidarão para que não seja permitida a entrada e a permanência de crianças e adolescentes no local,
afixando aviso para orientação do público.

Seção II
Dos Produtos e Serviços

Art. 81. É proibida a venda à criança ou ao adolescente de:


I - armas, munições e explosivos;
II - bebidas alcoólicas;
III - produtos cujos componentes possam causar dependência física ou psíquica ainda que por
utilização indevida;
IV - fogos de estampido e de artifício, exceto aqueles que pelo seu reduzido potencial sejam incapazes
de provocar qualquer dano físico em caso de utilização indevida;
V - revistas e publicações a que alude o art. 78;
VI - bilhetes lotéricos e equivalentes.

Art. 82. É proibida a hospedagem de criança ou adolescente em hotel, motel, pensão ou


estabelecimento congênere, salvo se autorizado ou acompanhado pelos pais ou responsável.

Apostila gerada especialmente para: Betânia Mueller 023.712.580-32


. 44
Seção III
Da Autorização para Viajar

Art. 83. Nenhuma criança poderá viajar para fora da comarca onde reside, desacompanhada dos pais
ou responsável, sem expressa autorização judicial.
§ 1º A autorização não será exigida quando:
a) tratar-se de comarca contígua à da residência da criança, se na mesma unidade da Federação, ou
incluída na mesma região metropolitana;
b) a criança estiver acompanhada:
1) de ascendente ou colateral maior, até o terceiro grau, comprovado documentalmente o parentesco;
2) de pessoa maior, expressamente autorizada pelo pai, mãe ou responsável.
§ 2º A autoridade judiciária poderá, a pedido dos pais ou responsável, conceder autorização válida por
dois anos.

Art. 84. Quando se tratar de viagem ao exterior, a autorização é dispensável, se a criança ou


adolescente:
I - estiver acompanhado de ambos os pais ou responsável;
II - viajar na companhia de um dos pais, autorizado expressamente pelo outro através de documento
com firma reconhecida.

Art. 85. Sem prévia e expressa autorização judicial, nenhuma criança ou adolescente nascido em
território nacional poderá sair do País em companhia de estrangeiro residente ou domiciliado no exterior.

Parte Especial
Título I
Da Política de Atendimento
Capítulo I
Disposições Gerais

Art. 86. A política de atendimento dos direitos da criança e do adolescente far-se-á através de um
conjunto articulado de ações governamentais e não-governamentais, da União, dos estados, do Distrito
Federal e dos municípios.

Art. 87. São linhas de ação da política de atendimento:


I - políticas sociais básicas;
II - serviços, programas, projetos e benefícios de assistência social de garantia de proteção social e
de prevenção e redução de violações de direitos, seus agravamentos ou reincidências; (Redação dada
pela Lei nº 13.257, de 2016)
III - serviços especiais de prevenção e atendimento médico e psicossocial às vítimas de negligência,
maus-tratos, exploração, abuso, crueldade e opressão;
IV - serviço de identificação e localização de pais, responsável, crianças e adolescentes
desaparecidos;
V - proteção jurídico-social por entidades de defesa dos direitos da criança e do adolescente.
VI - políticas e programas destinados a prevenir ou abreviar o período de afastamento do convívio
familiar e a garantir o efetivo exercício do direito à convivência familiar de crianças e adolescentes;
VII - campanhas de estímulo ao acolhimento sob forma de guarda de crianças e adolescentes
afastados do convívio familiar e à adoção, especificamente inter-racial, de crianças maiores ou de
adolescentes, com necessidades específicas de saúde ou com deficiências e de grupos de irmãos.

Art. 88. São diretrizes da política de atendimento:


I - municipalização do atendimento;
II - criação de conselhos municipais, estaduais e nacional dos direitos da criança e do adolescente,
órgãos deliberativos e controladores das ações em todos os níveis, assegurada a participação popular
paritária por meio de organizações representativas, segundo leis federal, estaduais e municipais;
III - criação e manutenção de programas específicos, observada a descentralização político-
administrativa;
IV - manutenção de fundos nacional, estaduais e municipais vinculados aos respectivos conselhos dos
direitos da criança e do adolescente;

Apostila gerada especialmente para: Betânia Mueller 023.712.580-32


. 45
V - integração operacional de órgãos do Judiciário, Ministério Público, Defensoria, Segurança Pública
e Assistência Social, preferencialmente em um mesmo local, para efeito de agilização do atendimento
inicial a adolescente a quem se atribua autoria de ato infracional;
VI - integração operacional de órgãos do Judiciário, Ministério Público, Defensoria, Conselho Tutelar e
encarregados da execução das políticas sociais básicas e de assistência social, para efeito de agilização
do atendimento de crianças e de adolescentes inseridos em programas de acolhimento familiar ou
institucional, com vista na sua rápida reintegração à família de origem ou, se tal solução se mostrar
comprovadamente inviável, sua colocação em família substituta, em quaisquer das modalidades previstas
no art. 28 desta Lei;
VII - mobilização da opinião pública para a indispensável participação dos diversos segmentos da
sociedade.
VIII - especialização e formação continuada dos profissionais que trabalham nas diferentes áreas da
atenção à primeira infância, incluindo os conhecimentos sobre direitos da criança e sobre
desenvolvimento infantil; (Incluído pela Lei nº 13.257, de 2016)
IX - formação profissional com abrangência dos diversos direitos da criança e do adolescente que
favoreça a intersetorialidade no atendimento da criança e do adolescente e seu desenvolvimento integral;
(Incluído pela Lei nº 13.257, de 2016)
X - realização e divulgação de pesquisas sobre desenvolvimento infantil e sobre prevenção da
violência. (Incluído pela Lei nº 13.257, de 2016)

Art. 89. A função de membro do conselho nacional e dos conselhos estaduais e municipais dos direitos
da criança e do adolescente é considerada de interesse público relevante e não será remunerada.

Capítulo II
Das Entidades de Atendimento
Seção I
Disposições Gerais

Art. 90. As entidades de atendimento são responsáveis pela manutenção das próprias unidades, assim
como pelo planejamento e execução de programas de proteção e socioeducativos destinados a crianças
e adolescentes, em regime de:
I - orientação e apoio sócio familiar;
II - apoio socioeducativo em meio aberto;
III - colocação familiar;
IV - acolhimento institucional;
V - prestação de serviços à comunidade;
VI - liberdade assistida;
VII - semiliberdade; e
VIII - internação.
§ 1º As entidades governamentais e não governamentais deverão proceder à inscrição de seus
programas, especificando os regimes de atendimento, na forma definida neste artigo, no Conselho
Municipal dos Direitos da Criança e do Adolescente, o qual manterá registro das inscrições e de suas
alterações, do que fará comunicação ao Conselho Tutelar e à autoridade judiciária.
§ 2º Os recursos destinados à implementação e manutenção dos programas relacionados neste artigo
serão previstos nas dotações orçamentárias dos órgãos públicos encarregados das áreas de Educação,
Saúde e Assistência Social, dentre outros, observando-se o princípio da prioridade absoluta à criança e
ao adolescente preconizado pelo caput do art. 227 da Constituição Federal e pelo caput e parágrafo único
do art. 4º desta Lei.
§ 3º Os programas em execução serão reavaliados pelo Conselho Municipal dos Direitos da Criança
e do Adolescente, no máximo, a cada 2 (dois) anos, constituindo-se critérios para renovação da
autorização de funcionamento:
I - o efetivo respeito às regras e princípios desta Lei, bem como às resoluções relativas à modalidade
de atendimento prestado expedidas pelos Conselhos de Direitos da Criança e do Adolescente, em todos
os níveis;
II - a qualidade e eficiência do trabalho desenvolvido, atestadas pelo Conselho Tutelar, pelo Ministério
Público e pela Justiça da Infância e da Juventude;
III - em se tratando de programas de acolhimento institucional ou familiar, serão considerados os
índices de sucesso na reintegração familiar ou de adaptação à família substituta, conforme o caso.

Apostila gerada especialmente para: Betânia Mueller 023.712.580-32


. 46
Art. 91. As entidades não-governamentais somente poderão funcionar depois de registradas no
Conselho Municipal dos Direitos da Criança e do Adolescente, o qual comunicará o registro ao Conselho
Tutelar e à autoridade judiciária da respectiva localidade.
§ 1º Será negado o registro à entidade que:
a) não ofereça instalações físicas em condições adequadas de habitabilidade, higiene, salubridade e
segurança;
b) não apresente plano de trabalho compatível com os princípios desta Lei;
c) esteja irregularmente constituída;
d) tenha em seus quadros pessoas inidôneas.
e) não se adequar ou deixar de cumprir as resoluções e deliberações relativas à modalidade de
atendimento prestado expedidas pelos Conselhos de Direitos da Criança e do Adolescente, em todos os
níveis.
§ 2º O registro terá validade máxima de 4 (quatro) anos, cabendo ao Conselho Municipal dos Direitos
da Criança e do Adolescente, periodicamente, reavaliar o cabimento de sua renovação, observado o
disposto no § 1º deste artigo.

Art. 92. As entidades que desenvolvam programas de acolhimento familiar ou institucional deverão
adotar os seguintes princípios
I - preservação dos vínculos familiares e promoção da reintegração familiar;
II - integração em família substituta, quando esgotados os recursos de manutenção na família natural
ou extensa;
III - atendimento personalizado e em pequenos grupos;
IV - desenvolvimento de atividades em regime de coeducação;
V - não desmembramento de grupos de irmãos;
VI - evitar, sempre que possível, a transferência para outras entidades de crianças e adolescentes
abrigados;
VII - participação na vida da comunidade local;
VIII - preparação gradativa para o desligamento;
IX - participação de pessoas da comunidade no processo educativo.
§ 1º O dirigente de entidade que desenvolve programa de acolhimento institucional é equiparado ao
guardião, para todos os efeitos de direito.
§ 2º Os dirigentes de entidades que desenvolvem programas de acolhimento familiar ou institucional
remeterão à autoridade judiciária, no máximo a cada 6 (seis) meses, relatório circunstanciado acerca da
situação de cada criança ou adolescente acolhido e sua família, para fins da reavaliação prevista no § 1º
do art. 19 desta Lei.
§ 3º Os entes federados, por intermédio dos Poderes Executivo e Judiciário, promoverão
conjuntamente a permanente qualificação dos profissionais que atuam direta ou indiretamente em
programas de acolhimento institucional e destinados à colocação familiar de crianças e adolescentes,
incluindo membros do Poder Judiciário, Ministério Público e Conselho Tutelar.
§ 4º Salvo determinação em contrário da autoridade judiciária competente, as entidades que
desenvolvem programas de acolhimento familiar ou institucional, se necessário com o auxílio do Conselho
Tutelar e dos órgãos de assistência social, estimularão o contato da criança ou adolescente com seus
pais e parentes, em cumprimento ao disposto nos incisos I e VIII do caput deste artigo.
§ 5º As entidades que desenvolvem programas de acolhimento familiar ou institucional somente
poderão receber recursos públicos se comprovado o atendimento dos princípios, exigências e finalidades
desta Lei.
§ 6º O descumprimento das disposições desta Lei pelo dirigente de entidade que desenvolva
programas de acolhimento familiar ou institucional é causa de sua destituição, sem prejuízo da apuração
de sua responsabilidade administrativa, civil e criminal.
§ 7º Quando se tratar de criança de 0 (zero) a 3 (três) anos em acolhimento institucional, dar-se-á
especial atenção à atuação de educadores de referência estáveis e qualitativamente significativos, às
rotinas específicas e ao atendimento das necessidades básicas, incluindo as de afeto como prioritárias.
(Incluído pela Lei nº 13.257, de 2016)

Art. 93. As entidades que mantenham programa de acolhimento institucional poderão, em caráter
excepcional e de urgência, acolher crianças e adolescentes sem prévia determinação da autoridade
competente, fazendo comunicação do fato em até 24 (vinte e quatro) horas ao Juiz da Infância e da
Juventude, sob pena de responsabilidade.

Apostila gerada especialmente para: Betânia Mueller 023.712.580-32


. 47
Parágrafo único. Recebida a comunicação, a autoridade judiciária, ouvido o Ministério Público e se
necessário com o apoio do Conselho Tutelar local, tomará as medidas necessárias para promover a
imediata reintegração familiar da criança ou do adolescente ou, se por qualquer razão não for isso
possível ou recomendável, para seu encaminhamento a programa de acolhimento familiar, institucional
ou a família substituta, observado o disposto no § 2o do art. 101 desta Lei.

Art. 94. As entidades que desenvolvem programas de internação têm as seguintes obrigações, entre
outras:
I - observar os direitos e garantias de que são titulares os adolescentes;
II - não restringir nenhum direito que não tenha sido objeto de restrição na decisão de internação;
III - oferecer atendimento personalizado, em pequenas unidades e grupos reduzidos;
IV - preservar a identidade e oferecer ambiente de respeito e dignidade ao adolescente;
V - diligenciar no sentido do restabelecimento e da preservação dos vínculos familiares;
VI - comunicar à autoridade judiciária, periodicamente, os casos em que se mostre inviável ou
impossível o reatamento dos vínculos familiares;
VII - oferecer instalações físicas em condições adequadas de habitabilidade, higiene, salubridade e
segurança e os objetos necessários à higiene pessoal;
VIII - oferecer vestuário e alimentação suficientes e adequados à faixa etária dos adolescentes
atendidos;
IX - oferecer cuidados médicos, psicológicos, odontológicos e farmacêuticos;
X - propiciar escolarização e profissionalização;
XI - propiciar atividades culturais, esportivas e de lazer;
XII - propiciar assistência religiosa àqueles que desejarem, de acordo com suas crenças;
XIII - proceder a estudo social e pessoal de cada caso;
XIV - reavaliar periodicamente cada caso, com intervalo máximo de seis meses, dando ciência dos
resultados à autoridade competente;
XV - informar, periodicamente, o adolescente internado sobre sua situação processual;
XVI - comunicar às autoridades competentes todos os casos de adolescentes portadores de moléstias
infectocontagiosas;
XVII - fornecer comprovante de depósito dos pertences dos adolescentes;
XVIII - manter programas destinados ao apoio e acompanhamento de egressos;
XIX - providenciar os documentos necessários ao exercício da cidadania àqueles que não os tiverem;
XX - manter arquivo de anotações onde constem data e circunstâncias do atendimento, nome do
adolescente, seus pais ou responsável, parentes, endereços, sexo, idade, acompanhamento da sua
formação, relação de seus pertences e demais dados que possibilitem sua identificação e a
individualização do atendimento.
§ 1º Aplicam-se, no que couber, as obrigações constantes deste artigo às entidades que mantêm
programas de acolhimento institucional e familiar.
§ 2º No cumprimento das obrigações a que alude este artigo as entidades utilizarão preferencialmente
os recursos da comunidade.

Art. 94-A. As entidades, públicas ou privadas, que abriguem ou recepcionem crianças e adolescentes,
ainda que em caráter temporário, devem ter, em seus quadros, profissionais capacitados a reconhecer e
reportar ao Conselho Tutelar suspeitas ou ocorrências de maus-tratos.

Seção II
Da Fiscalização das Entidades

Art. 95. As entidades governamentais e não-governamentais referidas no art. 90 serão fiscalizadas


pelo Judiciário, pelo Ministério Público e pelos Conselhos Tutelares.

Art. 96. Os planos de aplicação e as prestações de contas serão apresentados ao estado ou ao


município, conforme a origem das dotações orçamentárias.

Art. 97. São medidas aplicáveis às entidades de atendimento que descumprirem obrigação constante
do art. 94, sem prejuízo da responsabilidade civil e criminal de seus dirigentes ou prepostos:
I - às entidades governamentais:
a) advertência;
b) afastamento provisório de seus dirigentes;

Apostila gerada especialmente para: Betânia Mueller 023.712.580-32


. 48
c) afastamento definitivo de seus dirigentes;
d) fechamento de unidade ou interdição de programa.
II - às entidades não-governamentais:
a) advertência;
b) suspensão total ou parcial do repasse de verbas públicas;
c) interdição de unidades ou suspensão de programa;
d) cassação do registro.
§ 1º Em caso de reiteradas infrações cometidas por entidades de atendimento, que coloquem em risco
os direitos assegurados nesta Lei, deverá ser o fato comunicado ao Ministério Público ou representado
perante autoridade judiciária competente para as providências cabíveis, inclusive suspensão das
atividades ou dissolução da entidade.
§ 2º As pessoas jurídicas de direito público e as organizações não governamentais responderão pelos
danos que seus agentes causarem às crianças e aos adolescentes, caracterizado o descumprimento dos
princípios norteadores das atividades de proteção específica.

Título II
Das Medidas de Proteção
Capítulo I
Disposições Gerais

Art. 98. As medidas de proteção à criança e ao adolescente são aplicáveis sempre que os direitos
reconhecidos nesta Lei forem ameaçados ou violados:
I - por ação ou omissão da sociedade ou do Estado;
II - por falta, omissão ou abuso dos pais ou responsável;
III - em razão de sua conduta.

Capítulo II
Das Medidas Específicas de Proteção

Art. 99. As medidas previstas neste Capítulo poderão ser aplicadas isolada ou cumulativamente, bem
como substituídas a qualquer tempo.

Art. 100. Na aplicação das medidas levar-se-ão em conta as necessidades pedagógicas, preferindo-
se aquelas que visem ao fortalecimento dos vínculos familiares e comunitários.
Parágrafo único. São também princípios que regem a aplicação das medidas:
I - condição da criança e do adolescente como sujeitos de direitos: crianças e adolescentes são os
titulares dos direitos previstos nesta e em outras Leis, bem como na Constituição Federal;
II - proteção integral e prioritária: a interpretação e aplicação de toda e qualquer norma contida nesta
Lei deve ser voltada à proteção integral e prioritária dos direitos de que crianças e adolescentes são
titulares;
III - responsabilidade primária e solidária do poder público: a plena efetivação dos direitos assegurados
a crianças e a adolescentes por esta Lei e pela Constituição Federal, salvo nos casos por esta
expressamente ressalvados, é de responsabilidade primária e solidária das 3 (três) esferas de governo,
sem prejuízo da municipalização do atendimento e da possibilidade da execução de programas por
entidades não governamentais;
IV - interesse superior da criança e do adolescente: a intervenção deve atender prioritariamente aos
interesses e direitos da criança e do adolescente, sem prejuízo da consideração que for devida a outros
interesses legítimos no âmbito da pluralidade dos interesses presentes no caso concreto
V - privacidade: a promoção dos direitos e proteção da criança e do adolescente deve ser efetuada no
respeito pela intimidade, direito à imagem e reserva da sua vida privada;
VI - intervenção precoce: a intervenção das autoridades competentes deve ser efetuada logo que a
situação de perigo seja conhecida;
VII - intervenção mínima: a intervenção deve ser exercida exclusivamente pelas autoridades e
instituições cuja ação seja indispensável à efetiva promoção dos direitos e à proteção da criança e do
adolescente;
VIII - proporcionalidade e atualidade: a intervenção deve ser a necessária e adequada à situação de
perigo em que a criança ou o adolescente se encontram no momento em que a decisão é tomada;
IX - responsabilidade parental: a intervenção deve ser efetuada de modo que os pais assumam os
seus deveres para com a criança e o adolescente;

Apostila gerada especialmente para: Betânia Mueller 023.712.580-32


. 49
X - prevalência da família: na promoção de direitos e na proteção da criança e do adolescente deve
ser dada prevalência às medidas que os mantenham ou reintegrem na sua família natural ou extensa ou,
se isso não for possível, que promovam a sua integração em família adotiva; (Redação dada pela Lei nº
13.509, de 2017)
XI - obrigatoriedade da informação: a criança e o adolescente, respeitado seu estágio de
desenvolvimento e capacidade de compreensão, seus pais ou responsável devem ser informados dos
seus direitos, dos motivos que determinaram a intervenção e da forma como esta se processa;
XII - oitiva obrigatória e participação: a criança e o adolescente, em separado ou na companhia dos
pais, de responsável ou de pessoa por si indicada, bem como os seus pais ou responsável, têm direito a
ser ouvidos e a participar nos atos e na definição da medida de promoção dos direitos e de proteção,
sendo sua opinião devidamente considerada pela autoridade judiciária competente, observado o disposto
nos §§ 1º e 2º do art. 28 desta Lei.

Art. 101. Verificada qualquer das hipóteses previstas no art. 98, a autoridade competente poderá
determinar, dentre outras, as seguintes medidas:
I - encaminhamento aos pais ou responsável, mediante termo de responsabilidade;
II - orientação, apoio e acompanhamento temporários;
III - matrícula e frequência obrigatórias em estabelecimento oficial de ensino fundamental;
IV - inclusão em serviços e programas oficiais ou comunitários de proteção, apoio e promoção da
família, da criança e do adolescente; (Redação dada pela Lei nº 13.257, de 2016)
V - requisição de tratamento médico, psicológico ou psiquiátrico, em regime hospitalar ou ambulatorial;
VI - inclusão em programa oficial ou comunitário de auxílio, orientação e tratamento a alcoólatras e
toxicômanos;
VII - acolhimento institucional;
VIII - inclusão em programa de acolhimento familiar;
IX - colocação em família substituta.
§ 1º O acolhimento institucional e o acolhimento familiar são medidas provisórias e excepcionais,
utilizáveis como forma de transição para reintegração familiar ou, não sendo esta possível, para colocação
em família substituta, não implicando privação de liberdade.
§ 2º Sem prejuízo da tomada de medidas emergenciais para proteção de vítimas de violência ou abuso
sexual e das providências a que alude o art. 130 desta Lei, o afastamento da criança ou adolescente do
convívio familiar é de competência exclusiva da autoridade judiciária e importará na deflagração, a pedido
do Ministério Público ou de quem tenha legítimo interesse, de procedimento judicial contencioso, no qual
se garanta aos pais ou ao responsável legal o exercício do contraditório e da ampla defesa.
§ 3º Crianças e adolescentes somente poderão ser encaminhados às instituições que executam
programas de acolhimento institucional, governamentais ou não, por meio de uma Guia de Acolhimento,
expedida pela autoridade judiciária, na qual obrigatoriamente constará, dentre outros
I - sua identificação e a qualificação completa de seus pais ou de seu responsável, se conhecidos;
II - o endereço de residência dos pais ou do responsável, com pontos de referência;
III - os nomes de parentes ou de terceiros interessados em tê-los sob sua guarda;
IV - os motivos da retirada ou da não reintegração ao convívio familiar.
§ 4º Imediatamente após o acolhimento da criança ou do adolescente, a entidade responsável pelo
programa de acolhimento institucional ou familiar elaborará um plano individual de atendimento, visando
à reintegração familiar, ressalvada a existência de ordem escrita e fundamentada em contrário de
autoridade judiciária competente, caso em que também deverá contemplar sua colocação em família
substituta, observadas as regras e princípios desta Lei.
§ 5º O plano individual será elaborado sob a responsabilidade da equipe técnica do respectivo
programa de atendimento e levará em consideração a opinião da criança ou do adolescente e a oitiva dos
pais ou do responsável.
§ 6º Constarão do plano individual, dentre outros:
I - os resultados da avaliação interdisciplinar;
II - os compromissos assumidos pelos pais ou responsável; e
III - a previsão das atividades a serem desenvolvidas com a criança ou com o adolescente acolhido e
seus pais ou responsável, com vista na reintegração familiar ou, caso seja esta vedada por expressa e
fundamentada determinação judicial, as providências a serem tomadas para sua colocação em família
substituta, sob direta supervisão da autoridade judiciária.
§ 7º O acolhimento familiar ou institucional ocorrerá no local mais próximo à residência dos pais ou do
responsável e, como parte do processo de reintegração familiar, sempre que identificada a necessidade,

Apostila gerada especialmente para: Betânia Mueller 023.712.580-32


. 50
a família de origem será incluída em programas oficiais de orientação, de apoio e de promoção social,
sendo facilitado e estimulado o contato com a criança ou com o adolescente acolhido.
§ 8º Verificada a possibilidade de reintegração familiar, o responsável pelo programa de acolhimento
familiar ou institucional fará imediata comunicação à autoridade judiciária, que dará vista ao Ministério
Público, pelo prazo de 5 (cinco) dias, decidindo em igual prazo.
§ 9º Em sendo constatada a impossibilidade de reintegração da criança ou do adolescente à família
de origem, após seu encaminhamento a programas oficiais ou comunitários de orientação, apoio e
promoção social, será enviado relatório fundamentado ao Ministério Público, no qual conste a descrição
pormenorizada das providências tomadas e a expressa recomendação, subscrita pelos técnicos da
entidade ou responsáveis pela execução da política municipal de garantia do direito à convivência familiar,
para a destituição do poder familiar, ou destituição de tutela ou guarda.
§ 10. Recebido o relatório, o Ministério Público terá o prazo de 15 (quinze) dias para o ingresso com a
ação de destituição do poder familiar, salvo se entender necessária a realização de estudos
complementares ou de outras providências indispensáveis ao ajuizamento da demanda. (Redação dada
pela Lei nº 13.509, de 2017)
§ 11. A autoridade judiciária manterá, em cada comarca ou foro regional, um cadastro contendo
informações atualizadas sobre as crianças e adolescentes em regime de acolhimento familiar e
institucional sob sua responsabilidade, com informações pormenorizadas sobre a situação jurídica de
cada um, bem como as providências tomadas para sua reintegração familiar ou colocação em família
substituta, em qualquer das modalidades previstas no art. 28 desta Lei.
§ 12. Terão acesso ao cadastro o Ministério Público, o Conselho Tutelar, o órgão gestor da Assistência
Social e os Conselhos Municipais dos Direitos da Criança e do Adolescente e da Assistência Social, aos
quais incumbe deliberar sobre a implementação de políticas públicas que permitam reduzir o número de
crianças e adolescentes afastados do convívio familiar e abreviar o período de permanência em programa
de acolhimento.

Art. 102. As medidas de proteção de que trata este Capítulo serão acompanhadas da regularização
do registro civil.
§ 1º Verificada a inexistência de registro anterior, o assento de nascimento da criança ou adolescente
será feito à vista dos elementos disponíveis, mediante requisição da autoridade judiciária.
§ 2º Os registros e certidões necessários à regularização de que trata este artigo são isentos de multas,
custas e emolumentos, gozando de absoluta prioridade.
§ 3º Caso ainda não definida a paternidade, será deflagrado procedimento específico destinado à sua
averiguação, conforme previsto pela Lei nº 8.560, de 29 de dezembro de 1992.
§ 4º Nas hipóteses previstas no § 3º deste artigo, é dispensável o ajuizamento de ação de investigação
de paternidade pelo Ministério Público se, após o não comparecimento ou a recusa do suposto pai em
assumir a paternidade a ele atribuída, a criança for encaminhada para adoção.
§ 5º Os registros e certidões necessários à inclusão, a qualquer tempo, do nome do pai no assento de
nascimento são isentos de multas, custas e emolumentos, gozando de absoluta prioridade. (Incluído dada
pela Lei nº 13.257, de 2016)
§ 6º São gratuitas, a qualquer tempo, a averbação requerida do reconhecimento de paternidade no
assento de nascimento e a certidão correspondente. (Incluído dada pela Lei nº 13.257, de 2016)

Título III
Da Prática de Ato Infracional
Capítulo I
Disposições Gerais

Art. 103. Considera-se ato infracional a conduta descrita como crime ou contravenção penal.

Art. 104. São penalmente inimputáveis os menores de dezoito anos, sujeitos às medidas previstas
nesta Lei.
Parágrafo único. Para os efeitos desta Lei, deve ser considerada a idade do adolescente à data do
fato.

Art. 105. Ao ato infracional praticado por criança corresponderão as medidas previstas no art. 101.

Apostila gerada especialmente para: Betânia Mueller 023.712.580-32


. 51
Capítulo II
Dos Direitos Individuais

Art. 106. Nenhum adolescente será privado de sua liberdade senão em flagrante de ato infracional ou
por ordem escrita e fundamentada da autoridade judiciária competente.
Parágrafo único. O adolescente tem direito à identificação dos responsáveis pela sua apreensão,
devendo ser informado acerca de seus direitos.

Art. 107. A apreensão de qualquer adolescente e o local onde se encontra recolhido serão incontinenti
comunicados à autoridade judiciária competente e à família do apreendido ou à pessoa por ele indicada.
Parágrafo único. Examinar-se-á, desde logo e sob pena de responsabilidade, a possibilidade de
liberação imediata.

Art. 108. A internação, antes da sentença, pode ser determinada pelo prazo máximo de quarenta e
cinco dias.
Parágrafo único. A decisão deverá ser fundamentada e basear-se em indícios suficientes de autoria e
materialidade, demonstrada a necessidade imperiosa da medida.

Art. 109. O adolescente civilmente identificado não será submetido a identificação compulsória pelos
órgãos policiais, de proteção e judiciais, salvo para efeito de confrontação, havendo dúvida fundada.

Capítulo III
Das Garantias Processuais

Art. 110. Nenhum adolescente será privado de sua liberdade sem o devido processo legal.

Art. 111. São asseguradas ao adolescente, entre outras, as seguintes garantias:


I - pleno e formal conhecimento da atribuição de ato infracional, mediante citação ou meio equivalente;
II - igualdade na relação processual, podendo confrontar-se com vítimas e testemunhas e produzir
todas as provas necessárias à sua defesa;
III - defesa técnica por advogado;
IV - assistência judiciária gratuita e integral aos necessitados, na forma da lei;
V - direito de ser ouvido pessoalmente pela autoridade competente;
VI - direito de solicitar a presença de seus pais ou responsável em qualquer fase do procedimento.

Capítulo IV
Das Medidas Socioeducativas
Seção I
Disposições Gerais

Art. 112. Verificada a prática de ato infracional, a autoridade competente poderá aplicar ao adolescente
as seguintes medidas:
I - advertência;
II - obrigação de reparar o dano;
III - prestação de serviços à comunidade;
IV - liberdade assistida;
V - inserção em regime de semiliberdade;
VI - internação em estabelecimento educacional;
VII - qualquer uma das previstas no art. 101, I a VI.
§ 1º A medida aplicada ao adolescente levará em conta a sua capacidade de cumpri-la, as
circunstâncias e a gravidade da infração.
§ 2º Em hipótese alguma e sob pretexto algum, será admitida a prestação de trabalho forçado.
§ 3º Os adolescentes portadores de doença ou deficiência mental receberão tratamento individual e
especializado, em local adequado às suas condições.

Art. 113. Aplica-se a este Capítulo o disposto nos arts. 99 e 100.

Apostila gerada especialmente para: Betânia Mueller 023.712.580-32


. 52
Art. 114. A imposição das medidas previstas nos incisos II a VI do art. 112 pressupõe a existência de
provas suficientes da autoria e da materialidade da infração, ressalvada a hipótese de remissão, nos
termos do art. 127.
Parágrafo único. A advertência poderá ser aplicada sempre que houver prova da materialidade e
indícios suficientes da autoria.

Seção II
Da Advertência

Art. 115. A advertência consistirá em admoestação verbal, que será reduzida a termo e assinada.

Seção III
Da Obrigação de Reparar o Dano

Art. 116. Em se tratando de ato infracional com reflexos patrimoniais, a autoridade poderá determinar,
se for o caso, que o adolescente restitua a coisa, promova o ressarcimento do dano, ou, por outra forma,
compense o prejuízo da vítima.
Parágrafo único. Havendo manifesta impossibilidade, a medida poderá ser substituída por outra
adequada.
Seção IV
Da Prestação de Serviços à Comunidade

Art. 117. A prestação de serviços comunitários consiste na realização de tarefas gratuitas de interesse
geral, por período não excedente a seis meses, junto a entidades assistenciais, hospitais, escolas e outros
estabelecimentos congêneres, bem como em programas comunitários ou governamentais.
Parágrafo único. As tarefas serão atribuídas conforme as aptidões do adolescente, devendo ser
cumpridas durante jornada máxima de oito horas semanais, aos sábados, domingos e feriados ou em
dias úteis, de modo a não prejudicar a frequência à escola ou à jornada normal de trabalho.

Seção V
Da Liberdade Assistida

Art. 118. A liberdade assistida será adotada sempre que se afigurar a medida mais adequada para o
fim de acompanhar, auxiliar e orientar o adolescente.
§ 1º A autoridade designará pessoa capacitada para acompanhar o caso, a qual poderá ser
recomendada por entidade ou programa de atendimento.
§ 2º A liberdade assistida será fixada pelo prazo mínimo de seis meses, podendo a qualquer tempo
ser prorrogada, revogada ou substituída por outra medida, ouvido o orientador, o Ministério Público e o
defensor.

Art. 119. Incumbe ao orientador, com o apoio e a supervisão da autoridade competente, a realização
dos seguintes encargos, entre outros:
I - promover socialmente o adolescente e sua família, fornecendo-lhes orientação e inserindo-os, se
necessário, em programa oficial ou comunitário de auxílio e assistência social;
II - supervisionar a frequência e o aproveitamento escolar do adolescente, promovendo, inclusive, sua
matrícula;
III - diligenciar no sentido da profissionalização do adolescente e de sua inserção no mercado de
trabalho;
IV - apresentar relatório do caso
Seção VI
Do Regime de Semiliberdade

Art. 120. O regime de semiliberdade pode ser determinado desde o início, ou como forma de transição
para o meio aberto, possibilitada a realização de atividades externas, independentemente de autorização
judicial.
§ 1º São obrigatórias a escolarização e a profissionalização, devendo, sempre que possível, ser
utilizados os recursos existentes na comunidade.
§ 2º A medida não comporta prazo determinado aplicando-se, no que couber, as disposições relativas
à internação.

Apostila gerada especialmente para: Betânia Mueller 023.712.580-32


. 53
Seção VII
Da Internação

Art. 121. A internação constitui medida privativa da liberdade, sujeita aos princípios de brevidade,
excepcionalidade e respeito à condição peculiar de pessoa em desenvolvimento.
§ 1º Será permitida a realização de atividades externas, a critério da equipe técnica da entidade, salvo
expressa determinação judicial em contrário.
§ 2º A medida não comporta prazo determinado, devendo sua manutenção ser reavaliada, mediante
decisão fundamentada, no máximo a cada seis meses.
§ 3º Em nenhuma hipótese o período máximo de internação excederá a três anos.
§ 4º Atingido o limite estabelecido no parágrafo anterior, o adolescente deverá ser liberado, colocado
em regime de semiliberdade ou de liberdade assistida.
§ 5º A liberação será compulsória aos vinte e um anos de idade.
§ 6º Em qualquer hipótese a desinternação será precedida de autorização judicial, ouvido o Ministério
Público.
§ 7º A determinação judicial mencionada no § 1º poderá ser revista a qualquer tempo pela autoridade
judiciária.

Art. 122. A medida de internação só poderá ser aplicada quando:


I - tratar-se de ato infracional cometido mediante grave ameaça ou violência a pessoa;
II - por reiteração no cometimento de outras infrações graves;
III - por descumprimento reiterado e injustificável da medida anteriormente imposta.
§ 1º O prazo de internação na hipótese do inciso III deste artigo não poderá ser superior a 3 (três)
meses, devendo ser decretada judicialmente após o devido processo legal.
§ 2º Em nenhuma hipótese será aplicada a internação, havendo outra medida adequada.

Art. 123. A internação deverá ser cumprida em entidade exclusiva para adolescentes, em local distinto
daquele destinado ao abrigo, obedecida rigorosa separação por critérios de idade, compleição física e
gravidade da infração.
Parágrafo único. Durante o período de internação, inclusive provisória, serão obrigatórias atividades
pedagógicas.

Art. 124. São direitos do adolescente privado de liberdade, entre outros, os seguintes:
I - entrevistar-se pessoalmente com o representante do Ministério Público;
II - peticionar diretamente a qualquer autoridade;
III - avistar-se reservadamente com seu defensor;
IV - ser informado de sua situação processual, sempre que solicitada;
V - ser tratado com respeito e dignidade;
VI - permanecer internado na mesma localidade ou naquela mais próxima ao domicílio de seus pais
ou responsável;
VII - receber visitas, ao menos, semanalmente;
VIII - corresponder-se com seus familiares e amigos;
IX - ter acesso aos objetos necessários à higiene e asseio pessoal;
X - habitar alojamento em condições adequadas de higiene e salubridade;
XI - receber escolarização e profissionalização;
XII - realizar atividades culturais, esportivas e de lazer:
XIII - ter acesso aos meios de comunicação social;
XIV - receber assistência religiosa, segundo a sua crença, e desde que assim o deseje;
XV - manter a posse de seus objetos pessoais e dispor de local seguro para guardá-los, recebendo
comprovante daqueles porventura depositados em poder da entidade;
XVI - receber, quando de sua desinternação, os documentos pessoais indispensáveis à vida em
sociedade.
§ 1º Em nenhum caso haverá incomunicabilidade.
§ 2º A autoridade judiciária poderá suspender temporariamente a visita, inclusive de pais ou
responsável, se existirem motivos sérios e fundados de sua prejudicialidade aos interesses do
adolescente.
Art. 125. É dever do Estado zelar pela integridade física e mental dos internos, cabendo-lhe adotar as
medidas adequadas de contenção e segurança.

Apostila gerada especialmente para: Betânia Mueller 023.712.580-32


. 54
Capítulo V
Da Remissão

Art. 126. Antes de iniciado o procedimento judicial para apuração de ato infracional, o representante
do Ministério Público poderá conceder a remissão, como forma de exclusão do processo, atendendo às
circunstâncias e consequências do fato, ao contexto social, bem como à personalidade do adolescente e
sua maior ou menor participação no ato infracional.
Parágrafo único. Iniciado o procedimento, a concessão da remissão pela autoridade judiciária
importará na suspensão ou extinção do processo.

Art. 127. A remissão não implica necessariamente o reconhecimento ou comprovação da


responsabilidade, nem prevalece para efeito de antecedentes, podendo incluir eventualmente a aplicação
de qualquer das medidas previstas em lei, exceto a colocação em regime de semiliberdade e a internação.

Art. 128. A medida aplicada por força da remissão poderá ser revista judicialmente, a qualquer tempo,
mediante pedido expresso do adolescente ou de seu representante legal, ou do Ministério Público.

Título IV
Das Medidas Pertinentes aos Pais ou Responsável

Art. 129. São medidas aplicáveis aos pais ou responsável:


I - encaminhamento a serviços e programas oficiais ou comunitários de proteção, apoio e promoção
da família; (Redação dada pela Lei nº 13.257, de 2016)
II - inclusão em programa oficial ou comunitário de auxílio, orientação e tratamento a alcoólatras e
toxicômanos;
III - encaminhamento a tratamento psicológico ou psiquiátrico;
IV - encaminhamento a cursos ou programas de orientação;
V - obrigação de matricular o filho ou pupilo e acompanhar sua frequência e aproveitamento escolar;
VI - obrigação de encaminhar a criança ou adolescente a tratamento especializado;
VII - advertência;
VIII - perda da guarda;
IX - destituição da tutela;
X - suspensão ou destituição do poder familiar.
Parágrafo único. Na aplicação das medidas previstas nos incisos IX e X deste artigo, observar-se-á o
disposto nos arts. 23 e 24.

Art. 130. Verificada a hipótese de maus-tratos, opressão ou abuso sexual impostos pelos pais ou
responsável, a autoridade judiciária poderá determinar, como medida cautelar, o afastamento do agressor
da moradia comum.
Parágrafo único. Da medida cautelar constará, ainda, a fixação provisória dos alimentos de que
necessitem a criança ou o adolescente dependentes do agressor.

Título V
Do Conselho Tutelar
Capítulo I
Disposições Gerais

Art. 131. O Conselho Tutelar é órgão permanente e autônomo, não jurisdicional, encarregado pela
sociedade de zelar pelo cumprimento dos direitos da criança e do adolescente, definidos nesta Lei.

Art. 132. Em cada Município e em cada Região Administrativa do Distrito Federal haverá, no mínimo,
1 (um) Conselho Tutelar como órgão integrante da administração pública local, composto de 5 (cinco)
membros, escolhidos pela população local para mandato de 4 (quatro) anos, permitida 1 (uma)
recondução, mediante novo processo de escolha.

Art. 133. Para a candidatura a membro do Conselho Tutelar, serão exigidos os seguintes requisitos:
I - reconhecida idoneidade moral;
II - idade superior a vinte e um anos;
III - residir no município.

Apostila gerada especialmente para: Betânia Mueller 023.712.580-32


. 55
Art. 134. Lei municipal ou distrital disporá sobre o local, dia e horário de funcionamento do Conselho
Tutelar, inclusive quanto à remuneração dos respectivos membros, aos quais é assegurado o direito a:
I - cobertura previdenciária;
II - gozo de férias anuais remuneradas, acrescidas de 1/3 (um terço) do valor da remuneração mensal;
III - licença-maternidade;
IV - licença-paternidade;
V - gratificação natalina.
Parágrafo único. Constará da lei orçamentária municipal e da do Distrito Federal previsão dos recursos
necessários ao funcionamento do Conselho Tutelar e à remuneração e formação continuada dos
conselheiros tutelares.

Art. 135. O exercício efetivo da função de conselheiro constituirá serviço público relevante e
estabelecerá presunção de idoneidade moral.
Capítulo II
Das Atribuições do Conselho

Art. 136. São atribuições do Conselho Tutelar:


I - atender as crianças e adolescentes nas hipóteses previstas nos arts. 98 e 105, aplicando as medidas
previstas no art. 101, I a VII;
II - atender e aconselhar os pais ou responsável, aplicando as medidas previstas no art. 129, I a VII;
III - promover a execução de suas decisões, podendo para tanto:
a) requisitar serviços públicos nas áreas de saúde, educação, serviço social, previdência, trabalho e
segurança;
b) representar junto à autoridade judiciária nos casos de descumprimento injustificado de suas
deliberações.
IV - encaminhar ao Ministério Público notícia de fato que constitua infração administrativa ou penal
contra os direitos da criança ou adolescente;
V - encaminhar à autoridade judiciária os casos de sua competência;
VI - providenciar a medida estabelecida pela autoridade judiciária, dentre as previstas no art. 101, de I
a VI, para o adolescente autor de ato infracional;
VII - expedir notificações;
VIII - requisitar certidões de nascimento e de óbito de criança ou adolescente quando necessário;
IX - assessorar o Poder Executivo local na elaboração da proposta orçamentária para planos e
programas de atendimento dos direitos da criança e do adolescente;
X - representar, em nome da pessoa e da família, contra a violação dos direitos previstos no art. 220,
§ 3º, inciso II, da Constituição Federal;
XI - representar ao Ministério Público para efeito das ações de perda ou suspensão do poder familiar,
após esgotadas as possibilidades de manutenção da criança ou do adolescente junto à família natural.
XII - promover e incentivar, na comunidade e nos grupos profissionais, ações de divulgação e
treinamento para o reconhecimento de sintomas de maus-tratos em crianças e adolescentes.
Parágrafo único. Se, no exercício de suas atribuições, o Conselho Tutelar entender necessário o
afastamento do convívio familiar, comunicará incontinenti o fato ao Ministério Público, prestando-lhe
informações sobre os motivos de tal entendimento e as providências tomadas para a orientação, o apoio
e a promoção social da família.

Art. 137. As decisões do Conselho Tutelar somente poderão ser revistas pela autoridade judiciária a
pedido de quem tenha legítimo interesse.

Capítulo III
Da Competência

Art. 138. Aplica-se ao Conselho Tutelar a regra de competência constante do art. 147.

Capítulo IV
Da Escolha dos Conselheiros

Art. 139. O processo para a escolha dos membros do Conselho Tutelar será estabelecido em lei
municipal e realizado sob a responsabilidade do Conselho Municipal dos Direitos da Criança e do
Adolescente, e a fiscalização do Ministério Público.

Apostila gerada especialmente para: Betânia Mueller 023.712.580-32


. 56
§ 1º O processo de escolha dos membros do Conselho Tutelar ocorrerá em data unificada em todo o
território nacional a cada 4 (quatro) anos, no primeiro domingo do mês de outubro do ano subsequente
ao da eleição presidencial.
§ 2º A posse dos conselheiros tutelares ocorrerá no dia 10 de janeiro do ano subsequente ao processo
de escolha.
§ 3º No processo de escolha dos membros do Conselho Tutelar, é vedado ao candidato doar, oferecer,
prometer ou entregar ao eleitor bem ou vantagem pessoal de qualquer natureza, inclusive brindes de
pequeno valor.

Capítulo V
Dos Impedimentos

Art. 140. São impedidos de servir no mesmo Conselho marido e mulher, ascendentes e descendentes,
sogro e genro ou nora, irmãos, cunhados, durante o cunhadio, tio e sobrinho, padrasto ou madrasta e
enteado.
Parágrafo único. Estende-se o impedimento do conselheiro, na forma deste artigo, em relação à
autoridade judiciária e ao representante do Ministério Público com atuação na Justiça da Infância e da
Juventude, em exercício na comarca, foro regional ou distrital.

Título VI
Do Acesso à Justiça
Capítulo I
Disposições Gerais

Art. 141. É garantido o acesso de toda criança ou adolescente à Defensoria Pública, ao Ministério
Público e ao Poder Judiciário, por qualquer de seus órgãos.
§ 1º A assistência judiciária gratuita será prestada aos que dela necessitarem, através de defensor
público ou advogado nomeado.
§ 2º As ações judiciais da competência da Justiça da Infância e da Juventude são isentas de custas e
emolumentos, ressalvada a hipótese de litigância de má-fé.

Art. 142. Os menores de dezesseis anos serão representados e os maiores de dezesseis e menores
de vinte e um anos assistidos por seus pais, tutores ou curadores, na forma da legislação civil ou
processual.
Parágrafo único. A autoridade judiciária dará curador especial à criança ou adolescente, sempre que
os interesses destes colidirem com os de seus pais ou responsável, ou quando carecer de representação
ou assistência legal ainda que eventual.

Art. 143. E vedada a divulgação de atos judiciais, policiais e administrativos que digam respeito a
crianças e adolescentes a que se atribua autoria de ato infracional.
Parágrafo único. Qualquer notícia a respeito do fato não poderá identificar a criança ou adolescente,
vedando-se fotografia, referência ao nome, apelido, filiação, parentesco, residência e, inclusive, iniciais
do nome e sobrenome.

Art. 144. A expedição de cópia ou certidão de atos a que se refere o artigo anterior somente será
deferida pela autoridade judiciária competente, se demonstrado o interesse e justificada a finalidade.

Capítulo II
Da Justiça da Infância e da Juventude
Seção I
Disposições Gerais

Art. 145. Os estados e o Distrito Federal poderão criar varas especializadas e exclusivas da infância e
da juventude, cabendo ao Poder Judiciário estabelecer sua proporcionalidade por número de habitantes,
dotá-las de infraestrutura e dispor sobre o atendimento, inclusive em plantões.

Apostila gerada especialmente para: Betânia Mueller 023.712.580-32


. 57
Seção II
Do Juiz

Art. 146. A autoridade a que se refere esta Lei é o Juiz da Infância e da Juventude, ou o juiz que exerce
essa função, na forma da lei de organização judiciária local.

Art. 147. A competência será determinada:


I - pelo domicílio dos pais ou responsável;
II - pelo lugar onde se encontre a criança ou adolescente, à falta dos pais ou responsável.
§ 1º Nos casos de ato infracional, será competente a autoridade do lugar da ação ou omissão,
observadas as regras de conexão, continência e prevenção.
§ 2º A execução das medidas poderá ser delegada à autoridade competente da residência dos pais
ou responsável, ou do local onde sediar-se a entidade que abrigar a criança ou adolescente.
§ 3º Em caso de infração cometida através de transmissão simultânea de rádio ou televisão, que atinja
mais de uma comarca, será competente, para aplicação da penalidade, a autoridade judiciária do local
da sede estadual da emissora ou rede, tendo a sentença eficácia para todas as transmissoras ou
retransmissoras do respectivo estado.

Art. 148. A Justiça da Infância e da Juventude é competente para:


I - conhecer de representações promovidas pelo Ministério Público, para apuração de ato infracional
atribuído a adolescente, aplicando as medidas cabíveis;
II - conceder a remissão, como forma de suspensão ou extinção do processo;
III - conhecer de pedidos de adoção e seus incidentes;
IV - conhecer de ações civis fundadas em interesses individuais, difusos ou coletivos afetos à criança
e ao adolescente, observado o disposto no art. 209;
V - conhecer de ações decorrentes de irregularidades em entidades de atendimento, aplicando as
medidas cabíveis;
VI - aplicar penalidades administrativas nos casos de infrações contra norma de proteção à criança ou
adolescente;
VII - conhecer de casos encaminhados pelo Conselho Tutelar, aplicando as medidas cabíveis.
Parágrafo único. Quando se tratar de criança ou adolescente nas hipóteses do art. 98, é também
competente a Justiça da Infância e da Juventude para o fim de:
a) conhecer de pedidos de guarda e tutela;
b) conhecer de ações de destituição do poder familiar, perda ou modificação da tutela ou guarda;
c) suprir a capacidade ou o consentimento para o casamento;
d) conhecer de pedidos baseados em discordância paterna ou materna, em relação ao exercício do
poder familiar;
e) conceder a emancipação, nos termos da lei civil, quando faltarem os pais;
f) designar curador especial em casos de apresentação de queixa ou representação, ou de outros
procedimentos judiciais ou extrajudiciais em que haja interesses de criança ou adolescente;
g) conhecer de ações de alimentos;
h) determinar o cancelamento, a retificação e o suprimento dos registros de nascimento e óbito.

Art. 149. Compete à autoridade judiciária disciplinar, através de portaria, ou autorizar, mediante alvará:
I - a entrada e permanência de criança ou adolescente, desacompanhado dos pais ou responsável,
em:
a) estádio, ginásio e campo desportivo;
b) bailes ou promoções dançantes;
c) boate ou congêneres;
d) casa que explore comercialmente diversões eletrônicas;
e) estúdios cinematográficos, de teatro, rádio e televisão.
II - a participação de criança e adolescente em:
a) espetáculos públicos e seus ensaios;
b) certames de beleza.
§ 1º Para os fins do disposto neste artigo, a autoridade judiciária levará em conta, dentre outros fatores:
a) os princípios desta Lei;
b) as peculiaridades locais;
c) a existência de instalações adequadas;
d) o tipo de frequência habitual ao local;

Apostila gerada especialmente para: Betânia Mueller 023.712.580-32


. 58
e) a adequação do ambiente a eventual participação ou frequência de crianças e adolescentes;
f) a natureza do espetáculo.
§ 2º As medidas adotadas na conformidade deste artigo deverão ser fundamentadas, caso a caso,
vedadas as determinações de caráter geral.

Seção III
Dos Serviços Auxiliares

Art. 150. Cabe ao Poder Judiciário, na elaboração de sua proposta orçamentária, prever recursos para
manutenção de equipe interprofissional, destinada a assessorar a Justiça da Infância e da Juventude.

Art. 151. Compete à equipe interprofissional dentre outras atribuições que lhe forem reservadas pela
legislação local, fornecer subsídios por escrito, mediante laudos, ou verbalmente, na audiência, e bem
assim desenvolver trabalhos de aconselhamento, orientação, encaminhamento, prevenção e outros, tudo
sob a imediata subordinação à autoridade judiciária, assegurada a livre manifestação do ponto de vista
técnico.
Parágrafo único. Na ausência ou insuficiência de servidores públicos integrantes do Poder Judiciário
responsáveis pela realização dos estudos psicossociais ou de quaisquer outras espécies de avaliações
técnicas exigidas por esta Lei ou por determinação judicial, a autoridade judiciária poderá proceder à
nomeação de perito, nos termos do art. 156 da Lei no 13.105, de 16 de março de 2015 (Código de
Processo Civil). (Incluído pela Lei nº 13.509, de 2017)

Capítulo III
Dos Procedimentos
Seção I
Disposições Gerais

Art. 152. Aos procedimentos regulados nesta Lei aplicam-se subsidiariamente as normas gerais
previstas na legislação processual pertinente.
§ 1º É assegurada, sob pena de responsabilidade, prioridade absoluta na tramitação dos processos e
procedimentos previstos nesta Lei, assim como na execução dos atos e diligências judiciais a eles
referentes.
§ 2º Os prazos estabelecidos nesta Lei e aplicáveis aos seus procedimentos são contados em dias
corridos, excluído o dia do começo e incluído o dia do vencimento, vedado o prazo em dobro para a
Fazenda Pública e o Ministério Público. (Incluído pela Lei nº 13.509, de 2017)

Art. 153. Se a medida judicial a ser adotada não corresponder a procedimento previsto nesta ou em
outra lei, a autoridade judiciária poderá investigar os fatos e ordenar de ofício as providências necessárias,
ouvido o Ministério Público.
Parágrafo único. O disposto neste artigo não se aplica para o fim de afastamento da criança ou do
adolescente de sua família de origem e em outros procedimentos necessariamente contenciosos.

Art. 154. Aplica-se às multas o disposto no art. 214.

Seção II
Da Perda e da Suspensão do Poder Familiar

Art. 155. O procedimento para a perda ou a suspensão do poder familiar terá início por provocação do
Ministério Público ou de quem tenha legítimo interesse.

Art. 156. A petição inicial indicará:


I - a autoridade judiciária a que for dirigida;
II - o nome, o estado civil, a profissão e a residência do requerente e do requerido, dispensada a
qualificação em se tratando de pedido formulado por representante do Ministério Público;
III - a exposição sumária do fato e o pedido;
IV - as provas que serão produzidas, oferecendo, desde logo, o rol de testemunhas e documentos.

Apostila gerada especialmente para: Betânia Mueller 023.712.580-32


. 59
Art. 157. Havendo motivo grave, poderá a autoridade judiciária, ouvido o Ministério Público, decretar a
suspensão do poder familiar, liminar ou incidentalmente, até o julgamento definitivo da causa, ficando a
criança ou adolescente confiado a pessoa idônea, mediante termo de responsabilidade.
§ 1º Recebida a petição inicial, a autoridade judiciária determinará, concomitantemente ao despacho
de citação e independentemente de requerimento do interessado, a realização de estudo social ou perícia
por equipe interprofissional ou multidisciplinar para comprovar a presença de uma das causas de
suspensão ou destituição do poder familiar, ressalvado o disposto no § 10 do art. 101 desta Lei, e
observada a Lei nº 13.431, de 4 de abril de 2017. (Incluído pela Lei nº 13.509, de 2017)
§ 2º Em sendo os pais oriundos de comunidades indígenas, é ainda obrigatória a intervenção, junto à
equipe interprofissional ou multidisciplinar referida no § 1º deste artigo, de representantes do órgão federal
responsável pela política indigenista, observado o disposto no § 6º do art. 28 desta Lei. (Incluído pela Lei
nº 13.509, de 2017)

Art. 158. O requerido será citado para, no prazo de dez dias, oferecer resposta escrita, indicando as
provas a serem produzidas e oferecendo desde logo o rol de testemunhas e documentos.
§ 1º A citação será pessoal, salvo se esgotados todos os meios para sua realização.
§ 2º O requerido privado de liberdade deverá ser citado pessoalmente.
§ 3º Quando, por 2 (duas) vezes, o oficial de justiça houver procurado o citando em seu domicílio ou
residência sem o encontrar, deverá, havendo suspeita de ocultação, informar qualquer pessoa da família
ou, em sua falta, qualquer vizinho do dia útil em que voltará a fim de efetuar a citação, na hora que
designar, nos termos do art. 252 e seguintes da Lei nº 13.105, de 16 de março de 2015 (Código de
Processo Civil). (Incluído pela Lei nº 13.509, de 2017)
§ 4º Na hipótese de os genitores encontrarem-se em local incerto ou não sabido, serão citados por
edital no prazo de 10 (dez) dias, em publicação única, dispensado o envio de ofícios para a localização.
(Incluído pela Lei nº 13.509, de 2017)

Art. 159. Se o requerido não tiver possibilidade de constituir advogado, sem prejuízo do próprio
sustento e de sua família, poderá requerer, em cartório, que lhe seja nomeado dativo, ao qual incumbirá
a apresentação de resposta, contando-se o prazo a partir da intimação do despacho de nomeação.
Parágrafo único. Na hipótese de requerido privado de liberdade, o oficial de justiça deverá perguntar,
no momento da citação pessoal, se deseja que lhe seja nomeado defensor.

Art. 160. Sendo necessário, a autoridade judiciária requisitará de qualquer repartição ou órgão público
a apresentação de documento que interesse à causa, de ofício ou a requerimento das partes ou do
Ministério Público.

Art. 161. Se não for contestado o pedido e tiver sido concluído o estudo social ou a perícia realizada
por equipe interprofissional ou multidisciplinar, a autoridade judiciária dará vista dos autos ao Ministério
Público, por 5 (cinco) dias, salvo quando este for o requerente, e decidirá em igual prazo. (Redação dada
pela Lei nº 13.509, de 2017)
§ 1º A autoridade judiciária, de ofício ou a requerimento das partes ou do Ministério Público,
determinará a oitiva de testemunhas que comprovem a presença de uma das causas de suspensão ou
destituição do poder familiar previstas nos arts. 1.637 e 1.638 da Lei nº 10.406, de 10 de janeiro de 2002
(Código Civil), ou no art. 24 desta Lei. (Redação dada pela Lei nº 13.509, de 2017)
§ 2º (Revogado). (Redação dada pela Lei nº 13.509, de 2017)
§ 3º Se o pedido importar em modificação de guarda, será obrigatória, desde que possível e razoável,
a oitiva da criança ou adolescente, respeitado seu estágio de desenvolvimento e grau de compreensão
sobre as implicações da medida.
§ 4º É obrigatória a oitiva dos pais sempre que eles forem identificados e estiverem em local conhecido,
ressalvados os casos de não comparecimento perante a Justiça quando devidamente citados. (Redação
dada pela Lei nº 13.509, de 2017)
§ 5º Se o pai ou a mãe estiverem privados de liberdade, a autoridade judicial requisitará sua
apresentação para a oitiva

Art. 162. Apresentada a resposta, a autoridade judiciária dará vista dos autos ao Ministério Público,
por cinco dias, salvo quando este for o requerente, designando, desde logo, audiência de instrução e
julgamento.
§ 1º (Revogado). (Redação dada pela Lei nº 13.509, de 2017)

Apostila gerada especialmente para: Betânia Mueller 023.712.580-32


. 60
§ 2º Na audiência, presentes as partes e o Ministério Público, serão ouvidas as testemunhas, colhendo-
se oralmente o parecer técnico, salvo quando apresentado por escrito, manifestando-se sucessivamente
o requerente, o requerido e o Ministério Público, pelo tempo de 20 (vinte) minutos cada um, prorrogável
por mais 10 (dez) minutos. (Redação dada pela Lei nº 13.509, de 2017)
§ 3º A decisão será proferida na audiência, podendo a autoridade judiciária, excepcionalmente,
designar data para sua leitura no prazo máximo de 5 (cinco) dias. (Incluído pela Lei nº 13.509, de 2017)
§ 4º Quando o procedimento de destituição de poder familiar for iniciado pelo Ministério Público, não
haverá necessidade de nomeação de curador especial em favor da criança ou adolescente. (Incluído pela
Lei nº 13.509, de 2017)

Art. 163. O prazo máximo para conclusão do procedimento será de 120 (cento e vinte) dias, e caberá
ao juiz, no caso de notória inviabilidade de manutenção do poder familiar, dirigir esforços para preparar a
criança ou o adolescente com vistas à colocação em família substituta. (Redação dada pela Lei nº 13.509,
de 2017)
Parágrafo único. A sentença que decretar a perda ou a suspensão do poder familiar será averbada à
margem do registro de nascimento da criança ou do adolescente.

Seção III
Da Destituição da Tutela

Art. 164. Na destituição da tutela, observar-se-á o procedimento para a remoção de tutor previsto na
lei processual civil e, no que couber, o disposto na seção anterior.

Seção IV
Da Colocação em Família Substituta

Art. 165. São requisitos para a concessão de pedidos de colocação em família substituta:
I - qualificação completa do requerente e de seu eventual cônjuge, ou companheiro, com expressa
anuência deste;
II - indicação de eventual parentesco do requerente e de seu cônjuge, ou companheiro, com a criança
ou adolescente, especificando se tem ou não parente vivo;
III - qualificação completa da criança ou adolescente e de seus pais, se conhecidos;
IV - indicação do cartório onde foi inscrito nascimento, anexando, se possível, uma cópia da respectiva
certidão;
V - declaração sobre a existência de bens, direitos ou rendimentos relativos à criança ou ao
adolescente.
Parágrafo único. Em se tratando de adoção, observar-se-ão também os requisitos específicos.

Art. 166. Se os pais forem falecidos, tiverem sido destituídos ou suspensos do poder familiar, ou
houverem aderido expressamente ao pedido de colocação em família substituta, este poderá ser
formulado diretamente em cartório, em petição assinada pelos próprios requerentes, dispensada a
assistência de advogado.
§ 1º Na hipótese de concordância dos pais, o juiz: (Redação dada pela Lei nº 13.509, de 2017)
I - na presença do Ministério Público, ouvirá as partes, devidamente assistidas por advogado ou por
defensor público, para verificar sua concordância com a adoção, no prazo máximo de 10 (dez) dias,
contado da data do protocolo da petição ou da entrega da criança em juízo, tomando por termo as
declarações; e (Incluído pela Lei nº 13.509, de 2017)
II - declarará a extinção do poder familiar. (Incluído pela Lei nº 13.509, de 2017)
§ 2º O consentimento dos titulares do poder familiar será precedido de orientações e esclarecimentos
prestados pela equipe interprofissional da Justiça da Infância e da Juventude, em especial, no caso de
adoção, sobre a irrevogabilidade da medida.
§ 3º São garantidos a livre manifestação de vontade dos detentores do poder familiar e o direito ao
sigilo das informações. (Redação dada pela Lei nº 13.509, de 2017)
§ 4º O consentimento prestado por escrito não terá validade se não for ratificado na audiência a que
se refere o § 1º deste artigo. (Redação dada pela Lei nº 13.509, de 2017)
§ 5º O consentimento é retratável até a data da realização da audiência especificada no § 1o deste
artigo, e os pais podem exercer o arrependimento no prazo de 10 (dez) dias, contado da data de prolação
da sentença de extinção do poder familiar. (Redação dada pela Lei nº 13.509, de 2017)
§ 6º O consentimento somente terá valor se for dado após o nascimento da criança.

Apostila gerada especialmente para: Betânia Mueller 023.712.580-32


. 61
§ 7º A família natural e a família substituta receberão a devida orientação por intermédio de equipe
técnica interprofissional a serviço da Justiça da Infância e da Juventude, preferencialmente com apoio
dos técnicos responsáveis pela execução da política municipal de garantia do direito à convivência
familiar. (Redação dada pela Lei nº 13.509, de 2017)

Art. 167. A autoridade judiciária, de ofício ou a requerimento das partes ou do Ministério Público,
determinará a realização de estudo social ou, se possível, perícia por equipe interprofissional, decidindo
sobre a concessão de guarda provisória, bem como, no caso de adoção, sobre o estágio de convivência.
Parágrafo único. Deferida a concessão da guarda provisória ou do estágio de convivência, a criança
ou o adolescente será entregue ao interessado, mediante termo de responsabilidade.

Art. 168. Apresentado o relatório social ou o laudo pericial, e ouvida, sempre que possível, a criança
ou o adolescente, dar-se-á vista dos autos ao Ministério Público, pelo prazo de cinco dias, decidindo a
autoridade judiciária em igual prazo.

Art. 169. Nas hipóteses em que a destituição da tutela, a perda ou a suspensão do poder familiar
constituir pressuposto lógico da medida principal de colocação em família substituta, será observado o
procedimento contraditório previsto nas Seções II e III deste Capítulo.
Parágrafo único. A perda ou a modificação da guarda poderá ser decretada nos mesmos autos do
procedimento, observado o disposto no art. 35.

Art. 170. Concedida a guarda ou a tutela, observar-se-á o disposto no art. 32, e, quanto à adoção, o
contido no art. 47.
Parágrafo único. A colocação de criança ou adolescente sob a guarda de pessoa inscrita em programa
de acolhimento familiar será comunicada pela autoridade judiciária à entidade por este responsável no
prazo máximo de 5 (cinco) dias.

Seção V
Da Apuração de Ato Infracional Atribuído a Adolescente

Art. 171. O adolescente apreendido por força de ordem judicial será, desde logo, encaminhado à
autoridade judiciária.

Art. 172. O adolescente apreendido em flagrante de ato infracional será, desde logo, encaminhado à
autoridade policial competente.
Parágrafo único. Havendo repartição policial especializada para atendimento de adolescente e em se
tratando de ato infracional praticado em coautoria com maior, prevalecerá a atribuição da repartição
especializada, que, após as providências necessárias e conforme o caso, encaminhará o adulto à
repartição policial própria.

Art. 173. Em caso de flagrante de ato infracional cometido mediante violência ou grave ameaça a
pessoa, a autoridade policial, sem prejuízo do disposto nos arts. 106, parágrafo único, e 107, deverá:
I - lavrar auto de apreensão, ouvidos as testemunhas e o adolescente;
II - apreender o produto e os instrumentos da infração;
III - requisitar os exames ou perícias necessários à comprovação da materialidade e autoria da
infração.
Parágrafo único. Nas demais hipóteses de flagrante, a lavratura do auto poderá ser substituída por
boletim de ocorrência circunstanciada.

Art. 174. Comparecendo qualquer dos pais ou responsável, o adolescente será prontamente liberado
pela autoridade policial, sob termo de compromisso e responsabilidade de sua apresentação ao
representante do Ministério Público, no mesmo dia ou, sendo impossível, no primeiro dia útil imediato,
exceto quando, pela gravidade do ato infracional e sua repercussão social, deva o adolescente
permanecer sob internação para garantia de sua segurança pessoal ou manutenção da ordem pública.

Art. 175. Em caso de não liberação, a autoridade policial encaminhará, desde logo, o adolescente ao
representante do Ministério Público, juntamente com cópia do auto de apreensão ou boletim de
ocorrência.

Apostila gerada especialmente para: Betânia Mueller 023.712.580-32


. 62
§ 1º Sendo impossível a apresentação imediata, a autoridade policial encaminhará o adolescente à
entidade de atendimento, que fará a apresentação ao representante do Ministério Público no prazo de
vinte e quatro horas.
§ 2º Nas localidades onde não houver entidade de atendimento, a apresentação far-se-á pela
autoridade policial. À falta de repartição policial especializada, o adolescente aguardará a apresentação
em dependência separada da destinada a maiores, não podendo, em qualquer hipótese, exceder o prazo
referido no parágrafo anterior.

Art. 176. Sendo o adolescente liberado, a autoridade policial encaminhará imediatamente ao


representante do Ministério Público cópia do auto de apreensão ou boletim de ocorrência.

Art. 177. Se, afastada a hipótese de flagrante, houver indícios de participação de adolescente na
prática de ato infracional, a autoridade policial encaminhará ao representante do Ministério Público
relatório das investigações e demais documentos.

Art. 178. O adolescente a quem se atribua autoria de ato infracional não poderá ser conduzido ou
transportado em compartimento fechado de veículo policial, em condições atentatórias à sua dignidade,
ou que impliquem risco à sua integridade física ou mental, sob pena de responsabilidade.

Art. 179. Apresentado o adolescente, o representante do Ministério Público, no mesmo dia e à vista do
auto de apreensão, boletim de ocorrência ou relatório policial, devidamente autuados pelo cartório judicial
e com informação sobre os antecedentes do adolescente, procederá imediata e informalmente à sua
oitiva e, em sendo possível, de seus pais ou responsável, vítima e testemunhas.
Parágrafo único. Em caso de não apresentação, o representante do Ministério Público notificará os
pais ou responsável para apresentação do adolescente, podendo requisitar o concurso das polícias civil
e militar.

Art. 180. Adotadas as providências a que alude o artigo anterior, o representante do Ministério Público
poderá:
I - promover o arquivamento dos autos;
II - conceder a remissão;
III - representar à autoridade judiciária para aplicação de medida socioeducativa.

Art. 181. Promovido o arquivamento dos autos ou concedida a remissão pelo representante do
Ministério Público, mediante termo fundamentado, que conterá o resumo dos fatos, os autos serão
conclusos à autoridade judiciária para homologação.
§ 1º Homologado o arquivamento ou a remissão, a autoridade judiciária determinará, conforme o caso,
o cumprimento da medida.
§ 2º Discordando, a autoridade judiciária fará remessa dos autos ao Procurador-Geral de Justiça,
mediante despacho fundamentado, e este oferecerá representação, designará outro membro do
Ministério Público para apresentá-la, ou ratificará o arquivamento ou a remissão, que só então estará a
autoridade judiciária obrigada a homologar.

Art. 182. Se, por qualquer razão, o representante do Ministério Público não promover o arquivamento
ou conceder a remissão, oferecerá representação à autoridade judiciária, propondo a instauração de
procedimento para aplicação da medida socioeducativa que se afigurar a mais adequada.
§ 1º A representação será oferecida por petição, que conterá o breve resumo dos fatos e a classificação
do ato infracional e, quando necessário, o rol de testemunhas, podendo ser deduzida oralmente, em
sessão diária instalada pela autoridade judiciária.
§ 2º A representação independe de prova pré-constituída da autoria e materialidade.

Art. 183. O prazo máximo e improrrogável para a conclusão do procedimento, estando o adolescente
internado provisoriamente, será de quarenta e cinco dias.

Art. 184. Oferecida a representação, a autoridade judiciária designará audiência de apresentação do


adolescente, decidindo, desde logo, sobre a decretação ou manutenção da internação, observado o
disposto no art. 108 e parágrafo.
§ 1º O adolescente e seus pais ou responsável serão cientificados do teor da representação, e
notificados a comparecer à audiência, acompanhados de advogado.

Apostila gerada especialmente para: Betânia Mueller 023.712.580-32


. 63
§ 2º Se os pais ou responsável não forem localizados, a autoridade judiciária dará curador especial ao
adolescente.
§ 3º Não sendo localizado o adolescente, a autoridade judiciária expedirá mandado de busca e
apreensão, determinando o sobrestamento do feito, até a efetiva apresentação.
§ 4º Estando o adolescente internado, será requisitada a sua apresentação, sem prejuízo da
notificação dos pais ou responsável.

Art. 185. A internação, decretada ou mantida pela autoridade judiciária, não poderá ser cumprida em
estabelecimento prisional.
§ 1º Inexistindo na comarca entidade com as características definidas no art. 123, o adolescente deverá
ser imediatamente transferido para a localidade mais próxima.
§ 2º Sendo impossível a pronta transferência, o adolescente aguardará sua remoção em repartição
policial, desde que em seção isolada dos adultos e com instalações apropriadas, não podendo ultrapassar
o prazo máximo de cinco dias, sob pena de responsabilidade.

Art. 186. Comparecendo o adolescente, seus pais ou responsável, a autoridade judiciária procederá à
oitiva dos mesmos, podendo solicitar opinião de profissional qualificado.
§ 1º Se a autoridade judiciária entender adequada a remissão, ouvirá o representante do Ministério
Público, proferindo decisão.
§ 2º Sendo o fato grave, passível de aplicação de medida de internação ou colocação em regime de
semiliberdade, a autoridade judiciária, verificando que o adolescente não possui advogado constituído,
nomeará defensor, designando, desde logo, audiência em continuação, podendo determinar a realização
de diligências e estudo do caso.
§ 3º O advogado constituído ou o defensor nomeado, no prazo de três dias contado da audiência de
apresentação, oferecerá defesa prévia e rol de testemunhas.
§ 4º Na audiência em continuação, ouvidas as testemunhas arroladas na representação e na defesa
prévia, cumpridas as diligências e juntado o relatório da equipe interprofissional, será dada a palavra ao
representante do Ministério Público e ao defensor, sucessivamente, pelo tempo de vinte minutos para
cada um, prorrogável por mais dez, a critério da autoridade judiciária, que em seguida proferirá decisão.

Art. 187. Se o adolescente, devidamente notificado, não comparecer, injustificadamente à audiência


de apresentação, a autoridade judiciária designará nova data, determinando sua condução coercitiva.

Art. 188. A remissão, como forma de extinção ou suspensão do processo, poderá ser aplicada em
qualquer fase do procedimento, antes da sentença.

Art. 189. A autoridade judiciária não aplicará qualquer medida, desde que reconheça na sentença:
I - estar provada a inexistência do fato;
II - não haver prova da existência do fato;
III - não constituir o fato ato infracional;
IV - não existir prova de ter o adolescente concorrido para o ato infracional.
Parágrafo único. Na hipótese deste artigo, estando o adolescente internado, será imediatamente
colocado em liberdade.

Art. 190. A intimação da sentença que aplicar medida de internação ou regime de semiliberdade será
feita:
I - ao adolescente e ao seu defensor;
II - quando não for encontrado o adolescente, a seus pais ou responsável, sem prejuízo do defensor.
§ 1º Sendo outra a medida aplicada, a intimação far-se-á unicamente na pessoa do defensor.
§ 2º Recaindo a intimação na pessoa do adolescente, deverá este manifestar se deseja ou não recorrer
da sentença.
Seção V-A
(Incluído pela Lei nº 13.441, de 2017)
Da Infiltração de Agentes de Polícia para a Investigação de Crimes contra a Dignidade Sexual de
Criança e de Adolescente

Art. 190-A. A infiltração de agentes de polícia na internet com o fim de investigar os crimes previstos
nos arts. 240, 241, 241-A, 241-B, 241-C e 241-D desta Lei e nos arts. 154-A, 217-A, 218, 218-A e 218-B

Apostila gerada especialmente para: Betânia Mueller 023.712.580-32


. 64
do Decreto-Lei nº 2.848, de 7 de dezembro de 1940 (Código Penal), obedecerá às seguintes regras:
(Incluído pela Lei nº 13.441, de 2017)
I - será precedida de autorização judicial devidamente circunstanciada e fundamentada, que
estabelecerá os limites da infiltração para obtenção de prova, ouvido o Ministério Público; (Incluído pela
Lei nº 13.441, de 2017)
II - dar-se-á mediante requerimento do Ministério Público ou representação de delegado de polícia e
conterá a demonstração de sua necessidade, o alcance das tarefas dos policiais, os nomes ou apelidos
das pessoas investigadas e, quando possível, os dados de conexão ou cadastrais que permitam a
identificação dessas pessoas; (Incluído pela Lei nº 13.441, de 2017)
III - não poderá exceder o prazo de 90 (noventa) dias, sem prejuízo de eventuais renovações, desde
que o total não exceda a 720 (setecentos e vinte) dias e seja demonstrada sua efetiva necessidade, a
critério da autoridade judicial. (Incluído pela Lei nº 13.441, de 2017)
§ 1º A autoridade judicial e o Ministério Público poderão requisitar relatórios parciais da operação de
infiltração antes do término do prazo de que trata o inciso II do § 1º deste artigo. (Incluído pela Lei nº
13.441, de 2017)
§ 2º Para efeitos do disposto no inciso I do § 1º deste artigo, consideram-se: (Incluído pela Lei nº
13.441, de 2017)
I - dados de conexão: informações referentes a hora, data, início, término, duração, endereço de
Protocolo de Internet (IP) utilizado e terminal de origem da conexão. (Incluído pela Lei nº 13.441, de 2017)
II - dados cadastrais: informações referentes a nome e endereço de assinante ou de usuário registrado
ou autenticado para a conexão a quem endereço de IP, identificação de usuário ou código de acesso
tenha sido atribuído no momento da conexão.
§ 3º A infiltração de agentes de polícia na internet não será admitida se a prova puder ser obtida por
outros meios. (Incluído pela Lei nº 13.441, de 2017)

Art. 190-B. As informações da operação de infiltração serão encaminhadas diretamente ao juiz


responsável pela autorização da medida, que zelará por seu sigilo. (Incluído pela Lei nº 13.441, de 2017)
Parágrafo único. Antes da conclusão da operação, o acesso aos autos será reservado ao juiz, ao
Ministério Público e ao delegado de polícia responsável pela operação, com o objetivo de garantir o sigilo
das investigações. (Incluído pela Lei nº 13.441, de 2017)

Art. 190-C. Não comete crime o policial que oculta a sua identidade para, por meio da internet, colher
indícios de autoria e materialidade dos crimes previstos nos arts. 240, 241, 241-A, 241-B, 241-C e 241-D
desta Lei e nos arts. 154-A, 217-A, 218, 218-A e 218-B do Decreto-Lei nº 2.848, de 7 de dezembro de
1940 (Código Penal). (Incluído pela Lei nº 13.441, de 2017)
Parágrafo único. O agente policial infiltrado que deixar de observar a estrita finalidade da investigação
responderá pelos excessos praticados. (Incluído pela Lei nº 13.441, de 2017)

Art. 190-D. Os órgãos de registro e cadastro público poderão incluir nos bancos de dados próprios,
mediante procedimento sigiloso e requisição da autoridade judicial, as informações necessárias à
efetividade da identidade fictícia criada. (Incluído pela Lei nº 13.441, de 2017)
Parágrafo único. O procedimento sigiloso de que trata esta Seção será numerado e tombado em livro
específico. (Incluído pela Lei nº 13.441, de 2017)

Art. 190-E. Concluída a investigação, todos os atos eletrônicos praticados durante a operação deverão
ser registrados, gravados, armazenados e encaminhados ao juiz e ao Ministério Público, juntamente com
relatório circunstanciado. (Incluído pela Lei nº 13.441, de 2017)
Parágrafo único. Os atos eletrônicos registrados citados no caput deste artigo serão reunidos em autos
apartados e apensados ao processo criminal juntamente com o inquérito policial, assegurando-se a
preservação da identidade do agente policial infiltrado e a intimidade das crianças e dos adolescentes
envolvidos. (Incluído pela Lei nº 13.441, de 2017)

Seção VI
Da Apuração de Irregularidades em Entidade de Atendimento

Art. 191. O procedimento de apuração de irregularidades em entidade governamental e não-


governamental terá início mediante portaria da autoridade judiciária ou representação do Ministério
Público ou do Conselho Tutelar, onde conste, necessariamente, resumo dos fatos.

Apostila gerada especialmente para: Betânia Mueller 023.712.580-32


. 65
Parágrafo único. Havendo motivo grave, poderá a autoridade judiciária, ouvido o Ministério Público,
decretar liminarmente o afastamento provisório do dirigente da entidade, mediante decisão
fundamentada.

Art. 192. O dirigente da entidade será citado para, no prazo de dez dias, oferecer resposta escrita,
podendo juntar documentos e indicar as provas a produzir.

Art. 193. Apresentada ou não a resposta, e sendo necessário, a autoridade judiciária designará
audiência de instrução e julgamento, intimando as partes.
§ 1º Salvo manifestação em audiência, as partes e o Ministério Público terão cinco dias para oferecer
alegações finais, decidindo a autoridade judiciária em igual prazo.
§ 2º Em se tratando de afastamento provisório ou definitivo de dirigente de entidade governamental, a
autoridade judiciária oficiará à autoridade administrativa imediatamente superior ao afastado, marcando
prazo para a substituição.
§ 3º Antes de aplicar qualquer das medidas, a autoridade judiciária poderá fixar prazo para a remoção
das irregularidades verificadas. Satisfeitas as exigências, o processo será extinto, sem julgamento de
mérito.
§ 4º A multa e a advertência serão impostas ao dirigente da entidade ou programa de atendimento.

Seção VII
Da Apuração de Infração Administrativa às Normas de Proteção à Criança e ao Adolescente

Art. 194. O procedimento para imposição de penalidade administrativa por infração às normas de
proteção à criança e ao adolescente terá início por representação do Ministério Público, ou do Conselho
Tutelar, ou auto de infração elaborado por servidor efetivo ou voluntário credenciado, e assinado por duas
testemunhas, se possível.
§ 1º No procedimento iniciado com o auto de infração, poderão ser usadas fórmulas impressas,
especificando-se a natureza e as circunstâncias da infração.
§ 2º Sempre que possível, à verificação da infração seguir-se-á a lavratura do auto, certificando-se,
em caso contrário, dos motivos do retardamento.

Art. 195. O requerido terá prazo de dez dias para apresentação de defesa, contado da data da
intimação, que será feita:
I - pelo autuante, no próprio auto, quando este for lavrado na presença do requerido;
II - por oficial de justiça ou funcionário legalmente habilitado, que entregará cópia do auto ou da
representação ao requerido, ou a seu representante legal, lavrando certidão;
III - por via postal, com aviso de recebimento, se não for encontrado o requerido ou seu representante
legal;
IV - por edital, com prazo de trinta dias, se incerto ou não sabido o paradeiro do requerido ou de seu
representante legal.

Art. 196. Não sendo apresentada a defesa no prazo legal, a autoridade judiciária dará vista dos autos
do Ministério Público, por cinco dias, decidindo em igual prazo.

Art. 197. Apresentada a defesa, a autoridade judiciária procederá na conformidade do artigo anterior,
ou, sendo necessário, designará audiência de instrução e julgamento.
Parágrafo único. Colhida a prova oral, manifestar-se-ão sucessivamente o Ministério Público e o
procurador do requerido, pelo tempo de vinte minutos para cada um, prorrogável por mais dez, a critério
da autoridade judiciária, que em seguida proferirá sentença.

Seção VIII
Da Habilitação de Pretendentes à Adoção

Art. 197-A. Os postulantes à adoção, domiciliados no Brasil, apresentarão petição inicial na qual
conste:
I - qualificação completa;
II - dados familiares;
III - cópias autenticadas de certidão de nascimento ou casamento, ou declaração relativa ao período
de união estável;

Apostila gerada especialmente para: Betânia Mueller 023.712.580-32


. 66
IV - cópias da cédula de identidade e inscrição no Cadastro de Pessoas Físicas;
V - comprovante de renda e domicílio;
VI - atestados de sanidade física e mental;
VII - certidão de antecedentes criminais;
VIII - certidão negativa de distribuição cível.

Art. 197-B. A autoridade judiciária, no prazo de 48 (quarenta e oito) horas, dará vista dos autos ao
Ministério Público, que no prazo de 5 (cinco) dias poderá:
I - apresentar quesitos a serem respondidos pela equipe interprofissional encarregada de elaborar o
estudo técnico a que se refere o art. 197-C desta Lei;
II - requerer a designação de audiência para oitiva dos postulantes em juízo e testemunhas;
III - requerer a juntada de documentos complementares e a realização de outras diligências que
entender necessárias.

Art. 197-C. Intervirá no feito, obrigatoriamente, equipe interprofissional a serviço da Justiça da Infância
e da Juventude, que deverá elaborar estudo psicossocial, que conterá subsídios que permitam aferir a
capacidade e o preparo dos postulantes para o exercício de uma paternidade ou maternidade
responsável, à luz dos requisitos e princípios desta Lei.
§ 1º É obrigatória a participação dos postulantes em programa oferecido pela Justiça da Infância e da
Juventude, preferencialmente com apoio dos técnicos responsáveis pela execução da política municipal
de garantia do direito à convivência familiar e dos grupos de apoio à adoção devidamente habilitados
perante a Justiça da Infância e da Juventude, que inclua preparação psicológica, orientação e estímulo à
adoção inter-racial, de crianças ou de adolescentes com deficiência, com doenças crônicas ou com
necessidades específicas de saúde, e de grupos de irmãos. (Redação dada pela Lei nº 13.509, de 2017)
§ 2º Sempre que possível e recomendável, a etapa obrigatória da preparação referida no § 1º deste
artigo incluirá o contato com crianças e adolescentes em regime de acolhimento familiar ou institucional,
a ser realizado sob orientação, supervisão e avaliação da equipe técnica da Justiça da Infância e da
Juventude e dos grupos de apoio à adoção, com apoio dos técnicos responsáveis pelo programa de
acolhimento familiar e institucional e pela execução da política municipal de garantia do direito à
convivência familiar. (Redação dada pela Lei nº 13.509, de 2017)
§ 3º É recomendável que as crianças e os adolescentes acolhidos institucionalmente ou por família
acolhedora sejam preparados por equipe interprofissional antes da inclusão em família adotiva. (Incluído
pela Lei nº 13.509, de 2017)

Art. 197-D. Certificada nos autos a conclusão da participação no programa referido no art. 197-C desta
Lei, a autoridade judiciária, no prazo de 48 (quarenta e oito) horas, decidirá acerca das diligências
requeridas pelo Ministério Público e determinará a juntada do estudo psicossocial, designando, conforme
o caso, audiência de instrução e julgamento.
Parágrafo único. Caso não sejam requeridas diligências, ou sendo essas indeferidas, a autoridade
judiciária determinará a juntada do estudo psicossocial, abrindo a seguir vista dos autos ao Ministério
Público, por 5 (cinco) dias, decidindo em igual prazo.

Art. 197-E. Deferida a habilitação, o postulante será inscrito nos cadastros referidos no art. 50 desta
Lei, sendo a sua convocação para a adoção feita de acordo com ordem cronológica de habilitação e
conforme a disponibilidade de crianças ou adolescentes adotáveis.
§ 1º A ordem cronológica das habilitações somente poderá deixar de ser observada pela autoridade
judiciária nas hipóteses previstas no § 13 do art. 50 desta Lei, quando comprovado ser essa a melhor
solução no interesse do adotando.
§ 2º A habilitação à adoção deverá ser renovada no mínimo trienalmente mediante avaliação por
equipe interprofissional. (Redação dada pela Lei nº 13.509, de 2017)
§ 3º Quando o adotante candidatar-se a uma nova adoção, será dispensável a renovação da
habilitação, bastando a avaliação por equipe interprofissional. (Incluído pela Lei nº 13.509, de 2017)
§ 4º Após 3 (três) recusas injustificadas, pelo habilitado, à adoção de crianças ou adolescentes
indicados dentro do perfil escolhido, haverá reavaliação da habilitação concedida. (Incluído pela Lei nº
13.509, de 2017)
§ 5º A desistência do pretendente em relação à guarda para fins de adoção ou a devolução da criança
ou do adolescente depois do trânsito em julgado da sentença de adoção importará na sua exclusão dos
cadastros de adoção e na vedação de renovação da habilitação, salvo decisão judicial fundamentada,
sem prejuízo das demais sanções previstas na legislação vigente. (Incluído pela Lei nº 13.509, de 2017)

Apostila gerada especialmente para: Betânia Mueller 023.712.580-32


. 67
Art. 197-F. O prazo máximo para conclusão da habilitação à adoção será de 120 (cento e vinte) dias,
prorrogável por igual período, mediante decisão fundamentada da autoridade judiciária. (Incluído pela Lei
nº 13.509, de 2017)

Capítulo IV
Dos Recursos

Art. 198. Nos procedimentos afetos à Justiça da Infância e da Juventude, inclusive os relativos à
execução das medidas socioeducativas, adotar-se-á o sistema recursal da Lei nº 5.869, de 11 de janeiro
de 1973 (Código de Processo Civil), com as seguintes adaptações:
I - os recursos serão interpostos independentemente de preparo;
II - em todos os recursos, salvo nos embargos de declaração, o prazo para o Ministério Público e para
a defesa será sempre de 10 (dez) dias;
III - os recursos terão preferência de julgamento e dispensarão revisor;
IV - (Revogado pela Lei nº 12.010, de 2009)
V - (Revogado pela Lei nº 12.010, de 2009)
VI - (Revogado pela Lei nº 12.010, de 2009)
VII - antes de determinar a remessa dos autos à superior instância, no caso de apelação, ou do
instrumento, no caso de agravo, a autoridade judiciária proferirá despacho fundamentado, mantendo ou
reformando a decisão, no prazo de cinco dias;
VIII - mantida a decisão apelada ou agravada, o escrivão remeterá os autos ou o instrumento à superior
instância dentro de vinte e quatro horas, independentemente de novo pedido do recorrente; se a reformar,
a remessa dos autos dependerá de pedido expresso da parte interessada ou do Ministério Público, no
prazo de cinco dias, contados da intimação.

Art. 199. Contra as decisões proferidas com base no art. 149 caberá recurso de apelação.

Art. 199-A. A sentença que deferir a adoção produz efeito desde logo, embora sujeita a apelação, que
será recebida exclusivamente no efeito devolutivo, salvo se se tratar de adoção internacional ou se houver
perigo de dano irreparável ou de difícil reparação ao adotando.

Art. 199-B. A sentença que destituir ambos ou qualquer dos genitores do poder familiar fica sujeita a
apelação, que deverá ser recebida apenas no efeito devolutivo.

Art. 199-C. Os recursos nos procedimentos de adoção e de destituição de poder familiar, em face da
relevância das questões, serão processados com prioridade absoluta, devendo ser imediatamente
distribuídos, ficando vedado que aguardem, em qualquer situação, oportuna distribuição, e serão
colocados em mesa para julgamento sem revisão e com parecer urgente do Ministério Público.

Art. 199-D. O relator deverá colocar o processo em mesa para julgamento no prazo máximo de 60
(sessenta) dias, contado da sua conclusão.
Parágrafo único. O Ministério Público será intimado da data do julgamento e poderá na sessão, se
entender necessário, apresentar oralmente seu parecer.

Art. 199-E. O Ministério Público poderá requerer a instauração de procedimento para apuração de
responsabilidades se constatar o descumprimento das providências e do prazo previstos nos artigos
anteriores.
Capítulo V
Do Ministério Público

Art. 200. As funções do Ministério Público previstas nesta Lei serão exercidas nos termos da respectiva
lei orgânica.

Art. 201. Compete ao Ministério Público:


I - conceder a remissão como forma de exclusão do processo;
II - promover e acompanhar os procedimentos relativos às infrações atribuídas a adolescentes;
III - promover e acompanhar as ações de alimentos e os procedimentos de suspensão e destituição
do poder familiar, nomeação e remoção de tutores, curadores e guardiães, bem como oficiar em todos os
demais procedimentos da competência da Justiça da Infância e da Juventude;

Apostila gerada especialmente para: Betânia Mueller 023.712.580-32


. 68
IV - promover, de ofício ou por solicitação dos interessados, a especialização e a inscrição de hipoteca
legal e a prestação de contas dos tutores, curadores e quaisquer administradores de bens de crianças e
adolescentes nas hipóteses do art. 98;
V - promover o inquérito civil e a ação civil pública para a proteção dos interesses individuais, difusos
ou coletivos relativos à infância e à adolescência, inclusive os definidos no art. 220, § 3º inciso II, da
Constituição Federal;
VI - instaurar procedimentos administrativos e, para instruí-los:
a) expedir notificações para colher depoimentos ou esclarecimentos e, em caso de não
comparecimento injustificado, requisitar condução coercitiva, inclusive pela polícia civil ou militar;
b) requisitar informações, exames, perícias e documentos de autoridades municipais, estaduais e
federais, da administração direta ou indireta, bem como promover inspeções e diligências investigatórias;
c) requisitar informações e documentos a particulares e instituições privadas;
VII - instaurar sindicâncias, requisitar diligências investigatórias e determinar a instauração de inquérito
policial, para apuração de ilícitos ou infrações às normas de proteção à infância e à juventude;
VIII - zelar pelo efetivo respeito aos direitos e garantias legais assegurados às crianças e adolescentes,
promovendo as medidas judiciais e extrajudiciais cabíveis;
IX - impetrar mandado de segurança, de injunção e habeas corpus, em qualquer juízo, instância ou
tribunal, na defesa dos interesses sociais e individuais indisponíveis afetos à criança e ao adolescente;
X - representar ao juízo visando à aplicação de penalidade por infrações cometidas contra as normas
de proteção à infância e à juventude, sem prejuízo da promoção da responsabilidade civil e penal do
infrator, quando cabível;
XI - inspecionar as entidades públicas e particulares de atendimento e os programas de que trata esta
Lei, adotando de pronto as medidas administrativas ou judiciais necessárias à remoção de irregularidades
porventura verificadas;
XII - requisitar força policial, bem como a colaboração dos serviços médicos, hospitalares,
educacionais e de assistência social, públicos ou privados, para o desempenho de suas atribuições.
§ 1º A legitimação do Ministério Público para as ações cíveis previstas neste artigo não impede a de
terceiros, nas mesmas hipóteses, segundo dispuserem a Constituição e esta Lei.
§ 2º As atribuições constantes deste artigo não excluem outras, desde que compatíveis com a
finalidade do Ministério Público.
§ 3º O representante do Ministério Público, no exercício de suas funções, terá livre acesso a todo local
onde se encontre criança ou adolescente.
§ 4º O representante do Ministério Público será responsável pelo uso indevido das informações e
documentos que requisitar, nas hipóteses legais de sigilo.
§ 5º Para o exercício da atribuição de que trata o inciso VIII deste artigo, poderá o representante do
Ministério Público:
a) reduzir a termo as declarações do reclamante, instaurando o competente procedimento, sob sua
presidência;
b) entender-se diretamente com a pessoa ou autoridade reclamada, em dia, local e horário
previamente notificados ou acertados;
c) efetuar recomendações visando à melhoria dos serviços públicos e de relevância pública afetos à
criança e ao adolescente, fixando prazo razoável para sua perfeita adequação.

Art. 202. Nos processos e procedimentos em que não for parte, atuará obrigatoriamente o Ministério
Público na defesa dos direitos e interesses de que cuida esta Lei, hipótese em que terá vista dos autos
depois das partes, podendo juntar documentos e requerer diligências, usando os recursos cabíveis.

Art. 203. A intimação do Ministério Público, em qualquer caso, será feita pessoalmente.

Art. 204. A falta de intervenção do Ministério Público acarreta a nulidade do feito, que será declarada
de ofício pelo juiz ou a requerimento de qualquer interessado.

Art. 205. As manifestações processuais do representante do Ministério Público deverão ser


fundamentadas.
Capítulo VI
Do Advogado

Art. 206. A criança ou o adolescente, seus pais ou responsável, e qualquer pessoa que tenha legítimo
interesse na solução da lide poderão intervir nos procedimentos de que trata esta Lei, através de

Apostila gerada especialmente para: Betânia Mueller 023.712.580-32


. 69
advogado, o qual será intimado para todos os atos, pessoalmente ou por publicação oficial, respeitado o
segredo de justiça.
Parágrafo único. Será prestada assistência judiciária integral e gratuita àqueles que dela necessitarem.

Art. 207. Nenhum adolescente a quem se atribua a prática de ato infracional, ainda que ausente ou
foragido, será processado sem defensor.
§ 1º Se o adolescente não tiver defensor, ser-lhe-á nomeado pelo juiz, ressalvado o direito de, a todo
tempo, constituir outro de sua preferência.
§ 2º A ausência do defensor não determinará o adiamento de nenhum ato do processo, devendo o juiz
nomear substituto, ainda que provisoriamente, ou para o só efeito do ato.
§ 3º Será dispensada a outorga de mandato, quando se tratar de defensor nomeado ou, sido
constituído, tiver sido indicado por ocasião de ato formal com a presença da autoridade judiciária.

Capítulo VII
Da Proteção Judicial dos Interesses Individuais, Difusos e Coletivos

Art. 208. Regem-se pelas disposições desta Lei as ações de responsabilidade por ofensa aos direitos
assegurados à criança e ao adolescente, referentes ao não oferecimento ou oferta irregular:
I - do ensino obrigatório;
II - de atendimento educacional especializado aos portadores de deficiência;
III - de atendimento em creche e pré-escola às crianças de zero a cinco anos de idade (Redação dada
pela Lei nº 13.306, de 2016).
IV - de ensino noturno regular, adequado às condições do educando;
V - de programas suplementares de oferta de material didático-escolar, transporte e assistência à
saúde do educando do ensino fundamental;
VI - de serviço de assistência social visando à proteção à família, à maternidade, à infância e à
adolescência, bem como ao amparo às crianças e adolescentes que dele necessitem;
VII - de acesso às ações e serviços de saúde;
VIII - de escolarização e profissionalização dos adolescentes privados de liberdade.
IX - de ações, serviços e programas de orientação, apoio e promoção social de famílias e destinados
ao pleno exercício do direito à convivência familiar por crianças e adolescentes.
X - de programas de atendimento para a execução das medidas socioeducativas e aplicação de
medidas de proteção.
XI - de políticas e programas integrados de atendimento à criança e ao adolescente vítima ou
testemunha de violência. (Incluído pela Lei nº 13.431, de 2017)
§ 1º As hipóteses previstas neste artigo não excluem da proteção judicial outros interesses individuais,
difusos ou coletivos, próprios da infância e da adolescência, protegidos pela Constituição e pela Lei.
§ 2º A investigação do desaparecimento de crianças ou adolescentes será realizada imediatamente
após notificação aos órgãos competentes, que deverão comunicar o fato aos portos, aeroportos, Polícia
Rodoviária e companhias de transporte interestaduais e internacionais, fornecendo-lhes todos os dados
necessários à identificação do desaparecido.

Art. 209. As ações previstas neste Capítulo serão propostas no foro do local onde ocorreu ou deva
ocorrer a ação ou omissão, cujo juízo terá competência absoluta para processar a causa, ressalvadas a
competência da Justiça Federal e a competência originária dos tribunais superiores.

Art. 210. Para as ações cíveis fundadas em interesses coletivos ou difusos, consideram-se legitimados
concorrentemente:
I - o Ministério Público;
II - a União, os estados, os municípios, o Distrito Federal e os territórios;
III - as associações legalmente constituídas há pelo menos um ano e que incluam entre seus fins
institucionais a defesa dos interesses e direitos protegidos por esta Lei, dispensada a autorização da
assembleia, se houver prévia autorização estatutária.
§ 1º Admitir-se-á litisconsórcio facultativo entre os Ministérios Públicos da União e dos estados na
defesa dos interesses e direitos de que cuida esta Lei.
§ 2º Em caso de desistência ou abandono da ação por associação legitimada, o Ministério Público ou
outro legitimado poderá assumir a titularidade ativa.

Apostila gerada especialmente para: Betânia Mueller 023.712.580-32


. 70
Art. 211. Os órgãos públicos legitimados poderão tomar dos interessados compromisso de ajustamento
de sua conduta às exigências legais, o qual terá eficácia de título executivo extrajudicial.

Art. 212. Para defesa dos direitos e interesses protegidos por esta Lei, são admissíveis todas as
espécies de ações pertinentes.
§ 1º Aplicam-se às ações previstas neste Capítulo as normas do Código de Processo Civil.
§ 2º Contra atos ilegais ou abusivos de autoridade pública ou agente de pessoa jurídica no exercício
de atribuições do poder público, que lesem direito líquido e certo previsto nesta Lei, caberá ação
mandamental, que se regerá pelas normas da lei do mandado de segurança.

Art. 213. Na ação que tenha por objeto o cumprimento de obrigação de fazer ou não fazer, o juiz
concederá a tutela específica da obrigação ou determinará providências que assegurem o resultado
prático equivalente ao do adimplemento.
§ 1º Sendo relevante o fundamento da demanda e havendo justificado receio de ineficácia do
provimento final, é lícito ao juiz conceder a tutela liminarmente ou após justificação prévia, citando o réu.
§ 2º O juiz poderá, na hipótese do parágrafo anterior ou na sentença, impor multa diária ao réu,
independentemente de pedido do autor, se for suficiente ou compatível com a obrigação, fixando prazo
razoável para o cumprimento do preceito.
§ 3º A multa só será exigível do réu após o trânsito em julgado da sentença favorável ao autor, mas
será devida desde o dia em que se houver configurado o descumprimento.

Art. 214. Os valores das multas reverterão ao fundo gerido pelo Conselho dos Direitos da Criança e
do Adolescente do respectivo município.
§ 1º As multas não recolhidas até trinta dias após o trânsito em julgado da decisão serão exigidas
através de execução promovida pelo Ministério Público, nos mesmos autos, facultada igual iniciativa aos
demais legitimados.
§ 2º Enquanto o fundo não for regulamentado, o dinheiro ficará depositado em estabelecimento oficial
de crédito, em conta com correção monetária.

Art. 215. O juiz poderá conferir efeito suspensivo aos recursos, para evitar dano irreparável à parte.

Art. 216. Transitada em julgado a sentença que impuser condenação ao poder público, o juiz
determinará a remessa de peças à autoridade competente, para apuração da responsabilidade civil e
administrativa do agente a que se atribua a ação ou omissão.

Art. 217. Decorridos sessenta dias do trânsito em julgado da sentença condenatória sem que a
associação autora lhe promova a execução, deverá fazê-lo o Ministério Público, facultada igual iniciativa
aos demais legitimados.

Art. 218. O juiz condenará a associação autora a pagar ao réu os honorários advocatícios arbitrados
na conformidade do § 4º do art. 20 da Lei nº 5.869, de 11 de janeiro de 1973 (Código de Processo Civil),
quando reconhecer que a pretensão é manifestamente infundada.
Parágrafo único. Em caso de litigância de má-fé, a associação autora e os diretores responsáveis pela
propositura da ação serão solidariamente condenados ao décuplo das custas, sem prejuízo de
responsabilidade por perdas e danos.

Art. 219. Nas ações de que trata este Capítulo, não haverá adiantamento de custas, emolumentos,
honorários periciais e quaisquer outras despesas.

Art. 220. Qualquer pessoa poderá e o servidor público deverá provocar a iniciativa do Ministério
Público, prestando-lhe informações sobre fatos que constituam objeto de ação civil, e indicando-lhe os
elementos de convicção.

Art. 221. Se, no exercício de suas funções, os juízos e tribunais tiverem conhecimento de fatos que
possam ensejar a propositura de ação civil, remeterão peças ao Ministério Público para as providências
cabíveis.

Art. 222. Para instruir a petição inicial, o interessado poderá requerer às autoridades competentes as
certidões e informações que julgar necessárias, que serão fornecidas no prazo de quinze dias.

Apostila gerada especialmente para: Betânia Mueller 023.712.580-32


. 71
Art. 223. O Ministério Público poderá instaurar, sob sua presidência, inquérito civil, ou requisitar, de
qualquer pessoa, organismo público ou particular, certidões, informações, exames ou perícias, no prazo
que assinalar, o qual não poderá ser inferior a dez dias úteis.
§ 1º Se o órgão do Ministério Público, esgotadas todas as diligências, se convencer da inexistência de
fundamento para a propositura da ação cível, promoverá o arquivamento dos autos do inquérito civil ou
das peças informativas, fazendo-o fundamentadamente.
§ 2º Os autos do inquérito civil ou as peças de informação arquivados serão remetidos, sob pena de
se incorrer em falta grave, no prazo de três dias, ao Conselho Superior do Ministério Público.
§ 3º Até que seja homologada ou rejeitada a promoção de arquivamento, em sessão do Conselho
Superior do Ministério público, poderão as associações legitimadas apresentar razões escritas ou
documentos, que serão juntados aos autos do inquérito ou anexados às peças de informação.
§ 4º A promoção de arquivamento será submetida a exame e deliberação do Conselho Superior do
Ministério Público, conforme dispuser o seu regimento.
§ 5º Deixando o Conselho Superior de homologar a promoção de arquivamento, designará, desde
logo, outro órgão do Ministério Público para o ajuizamento da ação.

Art. 224. Aplicam-se subsidiariamente, no que couber, as disposições da Lei nº 7.347, de 24 de julho
de 1985.

Título VII
Dos Crimes e Das Infrações Administrativas
Capítulo I
Dos Crimes
Seção I
Disposições Gerais

Art. 225. Este Capítulo dispõe sobre crimes praticados contra a criança e o adolescente, por ação ou
omissão, sem prejuízo do disposto na legislação penal.

Art. 226. Aplicam-se aos crimes definidos nesta Lei as normas da Parte Geral do Código Penal e,
quanto ao processo, as pertinentes ao Código de Processo Penal.

Art. 227. Os crimes definidos nesta Lei são de ação pública incondicionada.

Seção II
Dos Crimes em Espécie

Art. 228. Deixar o encarregado de serviço ou o dirigente de estabelecimento de atenção à saúde de


gestante de manter registro das atividades desenvolvidas, na forma e prazo referidos no art. 10 desta Lei,
bem como de fornecer à parturiente ou a seu responsável, por ocasião da alta médica, declaração de
nascimento, onde constem as intercorrências do parto e do desenvolvimento do neonato:
Pena - detenção de seis meses a dois anos.
Parágrafo único. Se o crime é culposo:
Pena - detenção de dois a seis meses, ou multa.

Art. 229. Deixar o médico, enfermeiro ou dirigente de estabelecimento de atenção à saúde de gestante
de identificar corretamente o neonato e a parturiente, por ocasião do parto, bem como deixar de proceder
aos exames referidos no art. 10 desta Lei:
Pena - detenção de seis meses a dois anos.
Parágrafo único. Se o crime é culposo:
Pena - detenção de dois a seis meses, ou multa.

Art. 230. Privar a criança ou o adolescente de sua liberdade, procedendo à sua apreensão sem estar
em flagrante de ato infracional ou inexistindo ordem escrita da autoridade judiciária competente:
Pena - detenção de seis meses a dois anos.
Parágrafo único. Incide na mesma pena aquele que procede à apreensão sem observância das
formalidades legais.

Apostila gerada especialmente para: Betânia Mueller 023.712.580-32


. 72
Art. 231. Deixar a autoridade policial responsável pela apreensão de criança ou adolescente de fazer
imediata comunicação à autoridade judiciária competente e à família do apreendido ou à pessoa por ele
indicada:
Pena - detenção de seis meses a dois anos.

Art. 232. Submeter criança ou adolescente sob sua autoridade, guarda ou vigilância a vexame ou a
constrangimento:
Pena - detenção de seis meses a dois anos.

Art. 233. Revogado.

Art. 234. Deixar a autoridade competente, sem justa causa, de ordenar a imediata liberação de criança
ou adolescente, tão logo tenha conhecimento da ilegalidade da apreensão:
Pena - detenção de seis meses a dois anos.

Art. 235. Descumprir, injustificadamente, prazo fixado nesta Lei em benefício de adolescente privado
de liberdade:
Pena - detenção de seis meses a dois anos.

Art. 236. Impedir ou embaraçar a ação de autoridade judiciária, membro do Conselho Tutelar ou
representante do Ministério Público no exercício de função prevista nesta Lei:
Pena - detenção de seis meses a dois anos.

Art. 237. Subtrair criança ou adolescente ao poder de quem o tem sob sua guarda em virtude de lei ou
ordem judicial, com o fim de colocação em lar substituto:
Pena - reclusão de dois a seis anos, e multa.

Art. 238. Prometer ou efetivar a entrega de filho ou pupilo a terceiro, mediante paga ou recompensa:
Pena - reclusão de um a quatro anos, e multa.
Parágrafo único. Incide nas mesmas penas quem oferece ou efetiva a paga ou recompensa.

Art. 239. Promover ou auxiliar a efetivação de ato destinado ao envio de criança ou adolescente para
o exterior com inobservância das formalidades legais ou com o fito de obter lucro:
Pena - reclusão de quatro a seis anos, e multa.
Parágrafo único. Se há emprego de violência, grave ameaça ou fraude:
Pena - reclusão, de 6 (seis) a 8 (oito) anos, além da pena correspondente à violência.

Art. 240. Produzir, reproduzir, dirigir, fotografar, filmar ou registrar, por qualquer meio, cena de sexo
explícito ou pornográfica, envolvendo criança ou adolescente:
Pena - reclusão, de 4 (quatro) a 8 (oito) anos, e multa.
§ 1º Incorre nas mesmas penas quem agencia, facilita, recruta, coage, ou de qualquer modo
intermedeia a participação de criança ou adolescente nas cenas referidas no caput deste artigo, ou ainda
quem com esses contracena.
§ 2o Aumenta-se a pena de 1/3 (um terço) se o agente comete o crime:
I - no exercício de cargo ou função pública ou a pretexto de exercê-la;
II - prevalecendo-se de relações domésticas, de coabitação ou de hospitalidade; ou
III - prevalecendo-se de relações de parentesco consanguíneo ou afim até o terceiro grau, ou por
adoção, de tutor, curador, preceptor, empregador da vítima ou de quem, a qualquer outro título, tenha
autoridade sobre ela, ou com seu consentimento.

Art. 241. Vender ou expor à venda fotografia, vídeo ou outro registro que contenha cena de sexo
explícito ou pornográfica envolvendo criança ou adolescente.
Pena - reclusão, de 4 (quatro) a 8 (oito) anos, e multa.

Art. 241-A. Oferecer, trocar, disponibilizar, transmitir, distribuir, publicar ou divulgar por qualquer meio,
inclusive por meio de sistema de informática ou telemático, fotografia, vídeo ou outro registro que
contenha cena de sexo explícito ou pornográfica envolvendo criança ou adolescente:
Pena - reclusão, de 3 (três) a 6 (seis) anos, e multa.
§ 1º Nas mesmas penas incorre quem:

Apostila gerada especialmente para: Betânia Mueller 023.712.580-32


. 73
I - assegura os meios ou serviços para o armazenamento das fotografias, cenas ou imagens de que
trata o caput deste artigo;
II - assegura, por qualquer meio, o acesso por rede de computadores às fotografias, cenas ou imagens
de que trata o caput deste artigo.
§ 2o As condutas tipificadas nos incisos I e II do § 1º deste artigo são puníveis quando o responsável
legal pela prestação do serviço, oficialmente notificado, deixa de desabilitar o acesso ao conteúdo ilícito
de que trata o caput deste artigo.

Art. 241-B. Adquirir, possuir ou armazenar, por qualquer meio, fotografia, vídeo ou outra forma de
registro que contenha cena de sexo explícito ou pornográfica envolvendo criança ou adolescente:
Pena - reclusão, de 1 (um) a 4 (quatro) anos, e multa.
§ 1º A pena é diminuída de 1 (um) a 2/3 (dois terços) se de pequena quantidade o material a que se
refere o caput deste artigo.
§ 2º Não há crime se a posse ou o armazenamento tem a finalidade de comunicar às autoridades
competentes a ocorrência das condutas descritas nos arts. 240, 241, 241-A e 241-C desta Lei, quando a
comunicação for feita por:
I - agente público no exercício de suas funções;
II - membro de entidade, legalmente constituída, que inclua, entre suas finalidades institucionais, o
recebimento, o processamento e o encaminhamento de notícia dos crimes referidos neste parágrafo;
III - representante legal e funcionários responsáveis de provedor de acesso ou serviço prestado por
meio de rede de computadores, até o recebimento do material relativo à notícia feita à autoridade policial,
ao Ministério Público ou ao Poder Judiciário.
§ 3º As pessoas referidas no § 2º deste artigo deverão manter sob sigilo o material ilícito referido.

Art. 241-C. Simular a participação de criança ou adolescente em cena de sexo explícito ou pornográfica
por meio de adulteração, montagem ou modificação de fotografia, vídeo ou qualquer outra forma de
representação visual:
Pena - reclusão, de 1 (um) a 3 (três) anos, e multa.
Parágrafo único. Incorre nas mesmas penas quem vende, expõe à venda, disponibiliza, distribui,
publica ou divulga por qualquer meio, adquire, possui ou armazena o material produzido na forma do
caput deste artigo.

Art. 241-D. Aliciar, assediar, instigar ou constranger, por qualquer meio de comunicação, criança, com
o fim de com ela praticar ato libidinoso.
Pena - reclusão, de 1 (um) a 3 (três) anos, e multa.
Parágrafo único. Nas mesmas penas incorre quem:
I - facilita ou induz o acesso à criança de material contendo cena de sexo explícito ou pornográfica
com o fim de com ela praticar ato libidinoso;
II - pratica as condutas descritas no caput deste artigo com o fim de induzir criança a se exibir de forma
pornográfica ou sexualmente explícita.

Art. 241-E. Para efeito dos crimes previstos nesta Lei, a expressão “cena de sexo explícito ou
pornográfica” compreende qualquer situação que envolva criança ou adolescente em atividades sexuais
explícitas, reais ou simuladas, ou exibição dos órgãos genitais de uma criança ou adolescente para fins
primordialmente sexuais.

Art. 242. Vender, fornecer ainda que gratuitamente ou entregar, de qualquer forma, a criança ou
adolescente arma, munição ou explosivo:
Pena - reclusão, de 3 (três) a 6 (seis) anos.

Art. 243. Vender, fornecer, servir, ministrar ou entregar, ainda que gratuitamente, de qualquer forma,
a criança ou a adolescente, bebida alcoólica ou, sem justa causa, outros produtos cujos componentes
possam causar dependência física ou psíquica:
Pena - detenção de 2 (dois) a 4 (quatro) anos, e multa, se o fato não constitui crime mais grave.

Art. 244. Vender, fornecer ainda que gratuitamente ou entregar, de qualquer forma, a criança ou
adolescente fogos de estampido ou de artifício, exceto aqueles que, pelo seu reduzido potencial, sejam
incapazes de provocar qualquer dano físico em caso de utilização indevida:
Pena - detenção de seis meses a dois anos, e multa.

Apostila gerada especialmente para: Betânia Mueller 023.712.580-32


. 74
Art. 244-A. Submeter criança ou adolescente, como tais definidos no caput do art. 2o desta Lei, à
prostituição ou à exploração sexual
Pena - reclusão de quatro a dez anos e multa, além da perda de bens e valores utilizados na prática
criminosa em favor do Fundo dos Direitos da Criança e do Adolescente da unidade da Federação (Estado
ou Distrito Federal) em que foi cometido o crime, ressalvado o direito de terceiro de boa-fé. (Redação
dada pela Lei nº 13.440, de 2017)
§ 1º Incorrem nas mesmas penas o proprietário, o gerente ou o responsável pelo local em que se
verifique a submissão de criança ou adolescente às práticas referidas no caput deste artigo.
§ 2º Constitui efeito obrigatório da condenação a cassação da licença de localização e de
funcionamento do estabelecimento.

Art. 244-B. Corromper ou facilitar a corrupção de menor de 18 (dezoito) anos, com ele praticando
infração penal ou induzindo-o a praticá-la:
Pena - reclusão, de 1 (um) a 4 (quatro) anos.
§ 1º Incorre nas penas previstas no caput deste artigo quem pratica as condutas ali tipificadas
utilizando-se de quaisquer meios eletrônicos, inclusive salas de bate-papo da internet.
§ 2º As penas previstas no caput deste artigo são aumentadas de um terço no caso de a infração
cometida ou induzida estar incluída no rol do art. 1º da Lei nº 8.072, de 25 de julho de 1990.

Capítulo II
Das Infrações Administrativas

Art. 245. Deixar o médico, professor ou responsável por estabelecimento de atenção à saúde e de
ensino fundamental, pré-escola ou creche, de comunicar à autoridade competente os casos de que tenha
conhecimento, envolvendo suspeita ou confirmação de maus-tratos contra criança ou adolescente:
Pena - multa de três a vinte salários de referência, aplicando-se o dobro em caso de reincidência.

Art. 246. Impedir o responsável ou funcionário de entidade de atendimento o exercício dos direitos
constantes nos incisos II, III, VII, VIII e XI do art. 124 desta Lei:
Pena - multa de três a vinte salários de referência, aplicando-se o dobro em caso de reincidência.

Art. 247. Divulgar, total ou parcialmente, sem autorização devida, por qualquer meio de comunicação,
nome, ato ou documento de procedimento policial, administrativo ou judicial relativo a criança ou
adolescente a que se atribua ato infracional:
Pena - multa de três a vinte salários de referência, aplicando-se o dobro em caso de reincidência.
§ 1º Incorre na mesma pena quem exibe, total ou parcialmente, fotografia de criança ou adolescente
envolvido em ato infracional, ou qualquer ilustração que lhe diga respeito ou se refira a atos que lhe sejam
atribuídos, de forma a permitir sua identificação, direta ou indiretamente.
§ 2º Se o fato for praticado por órgão de imprensa ou emissora de rádio ou televisão, além da pena
prevista neste artigo, a autoridade judiciária poderá determinar a apreensão da publicação ou a
suspensão da programação da emissora até por dois dias, bem como da publicação do periódico até por
dois números. (Expressão declara inconstitucional pela ADIN 869-2)

Art. 248. (Revogado pela Lei nº 13.431, de 2017)

Art. 249. Descumprir, dolosa ou culposamente, os deveres inerentes ao poder familiar ou decorrente
de tutela ou guarda, bem assim determinação da autoridade judiciária ou Conselho Tutelar:
Pena - multa de três a vinte salários de referência, aplicando-se o dobro em caso de reincidência.

Art. 250. Hospedar criança ou adolescente desacompanhado dos pais ou responsável, ou sem
autorização escrita desses ou da autoridade judiciária, em hotel, pensão, motel ou congênere:
Pena - multa.
§ 1º Em caso de reincidência, sem prejuízo da pena de multa, a autoridade judiciária poderá determinar
o fechamento do estabelecimento por até 15 (quinze) dias.
§ 2º Se comprovada a reincidência em período inferior a 30 (trinta) dias, o estabelecimento será
definitivamente fechado e terá sua licença cassada.

Art. 251. Transportar criança ou adolescente, por qualquer meio, com inobservância do disposto nos
arts. 83, 84 e 85 desta Lei:

Apostila gerada especialmente para: Betânia Mueller 023.712.580-32


. 75
Pena - multa de três a vinte salários de referência, aplicando-se o dobro em caso de reincidência.

Art. 252. Deixar o responsável por diversão ou espetáculo público de afixar, em lugar visível e de fácil
acesso, à entrada do local de exibição, informação destacada sobre a natureza da diversão ou espetáculo
e a faixa etária especificada no certificado de classificação:
Pena - multa de três a vinte salários de referência, aplicando-se o dobro em caso de reincidência.

Art. 253. Anunciar peças teatrais, filmes ou quaisquer representações ou espetáculos, sem indicar os
limites de idade a que não se recomendem:
Pena - multa de três a vinte salários de referência, duplicada em caso de reincidência, aplicável,
separadamente, à casa de espetáculo e aos órgãos de divulgação ou publicidade.

Art. 254. Transmitir, através de rádio ou televisão, espetáculo em horário diverso do autorizado ou sem
aviso de sua classificação:
Pena - multa de vinte a cem salários de referência; duplicada em caso de reincidência a autoridade
judiciária poderá determinar a suspensão da programação da emissora por até dois dias.

Art. 255. Exibir filme, trailer, peça, amostra ou congênere classificado pelo órgão competente como
inadequado às crianças ou adolescentes admitidos ao espetáculo:
Pena - multa de vinte a cem salários de referência; na reincidência, a autoridade poderá determinar a
suspensão do espetáculo ou o fechamento do estabelecimento por até quinze dias.

Art. 256. Vender ou locar a criança ou adolescente fita de programação em vídeo, em desacordo com
a classificação atribuída pelo órgão competente:
Pena - multa de três a vinte salários de referência; em caso de reincidência, a autoridade judiciária
poderá determinar o fechamento do estabelecimento por até quinze dias.

Art. 257. Descumprir obrigação constante dos arts. 78 e 79 desta Lei:


Pena - multa de três a vinte salários de referência, duplicando-se a pena em caso de reincidência, sem
prejuízo de apreensão da revista ou publicação.

Art. 258. Deixar o responsável pelo estabelecimento ou o empresário de observar o que dispõe esta
Lei sobre o acesso de criança ou adolescente aos locais de diversão, ou sobre sua participação no
espetáculo:
Pena - multa de três a vinte salários de referência; em caso de reincidência, a autoridade judiciária
poderá determinar o fechamento do estabelecimento por até quinze dias.

Art. 258-A. Deixar a autoridade competente de providenciar a instalação e operacionalização dos


cadastros previstos no art. 50 e no § 11 do art. 101 desta Lei:
Pena - multa de R$ 1.000,00 (mil reais) a R$ 3.000,00 (três mil reais).
Parágrafo único. Incorre nas mesmas penas a autoridade que deixa de efetuar o cadastramento de
crianças e de adolescentes em condições de serem adotadas, de pessoas ou casais habilitados à adoção
e de crianças e adolescentes em regime de acolhimento institucional ou familiar.

Art. 258-B. Deixar o médico, enfermeiro ou dirigente de estabelecimento de atenção à saúde de


gestante de efetuar imediato encaminhamento à autoridade judiciária de caso de que tenha conhecimento
de mãe ou gestante interessada em entregar seu filho para adoção:
Pena - multa de R$ 1.000,00 (mil reais) a R$ 3.000,00 (três mil reais).
Parágrafo único. Incorre na mesma pena o funcionário de programa oficial ou comunitário destinado à
garantia do direito à convivência familiar que deixa de efetuar a comunicação referida no caput deste
artigo.

Art. 258-C. Descumprir a proibição estabelecida no inciso II do art. 81:


Pena - multa de R$ 3.000,00 (três mil reais) a R$ 10.000,00 (dez mil reais);
Medida Administrativa - interdição do estabelecimento comercial até o recolhimento da multa aplicada.

Apostila gerada especialmente para: Betânia Mueller 023.712.580-32


. 76
Disposições Finais e Transitórias

Art. 259. A União, no prazo de noventa dias contados da publicação deste Estatuto, elaborará projeto
de lei dispondo sobre a criação ou adaptação de seus órgãos às diretrizes da política de atendimento
fixadas no art. 88 e ao que estabelece o Título V do Livro II.
Parágrafo único. Compete aos estados e municípios promoverem a adaptação de seus órgãos e
programas às diretrizes e princípios estabelecidos nesta Lei.

Art. 260. Os contribuintes poderão efetuar doações aos Fundos dos Direitos da Criança e do
Adolescente nacional, distrital, estaduais ou municipais, devidamente comprovadas, sendo essas
integralmente deduzidas do imposto de renda, obedecidos os seguintes limites:
I - 1% (um por cento) do imposto sobre a renda devido apurado pelas pessoas jurídicas tributadas com
base no lucro real; e
II - 6% (seis por cento) do imposto sobre a renda apurado pelas pessoas físicas na Declaração de
Ajuste Anual, observado o disposto no art. 22 da Lei no 9.532, de 10 de dezembro de 1997.
§ 1º Revogado.
§ 1º-A. Na definição das prioridades a serem atendidas com os recursos captados pelos fundos
nacional, estaduais e municipais dos direitos da criança e do adolescente, serão consideradas as
disposições do Plano Nacional de Promoção, Proteção e Defesa do Direito de Crianças e Adolescentes
à Convivência Familiar e Comunitária e as do Plano Nacional pela Primeira Infância. (Redação dada pela
Lei nº 13.257, de 2016)
§ 2o Os conselhos nacional, estaduais e municipais dos direitos da criança e do adolescente fixarão
critérios de utilização, por meio de planos de aplicação, das dotações subsidiadas e demais receitas,
aplicando necessariamente percentual para incentivo ao acolhimento, sob a forma de guarda, de crianças
e adolescentes e para programas de atenção integral à primeira infância em áreas de maior carência
socioeconômica e em situações de calamidade. (Redação dada pela Lei nº 13.257, de 2016)
§ 3º O Departamento da Receita Federal, do Ministério da Economia, Fazenda e Planejamento,
regulamentará a comprovação das doações feitas aos fundos, nos termos deste artigo.
§ 4º O Ministério Público determinará em cada comarca a forma de fiscalização da aplicação, pelo
Fundo Municipal dos Direitos da Criança e do Adolescente, dos incentivos fiscais referidos neste artigo.
§ 5º Observado o disposto no § 4º do art. 3º da Lei nº 9.249, de 26 de dezembro de 1995, a dedução
de que trata o inciso I do caput:
I - será considerada isoladamente, não se submetendo a limite em conjunto com outras deduções do
imposto; e
II - não poderá ser computada como despesa operacional na apuração do lucro real.

Art. 260-A. A partir do exercício de 2010, ano-calendário de 2009, a pessoa física poderá optar pela
doação de que trata o inciso II do caput do art. 260 diretamente em sua Declaração de Ajuste Anual.
§ 1º A doação de que trata o caput poderá ser deduzida até os seguintes percentuais aplicados sobre
o imposto apurado na declaração:
I - (Vetado);
II - (Vetado);
III - 3% (três por cento) a partir do exercício de 2012.
§ 2º A dedução de que trata o caput:
I - está sujeita ao limite de 6% (seis por cento) do imposto sobre a renda apurado na declaração de
que trata o inciso II do caput do art. 260;
II - não se aplica à pessoa física que:
a) utilizar o desconto simplificado;
b) apresentar declaração em formulário; ou
c) entregar a declaração fora do prazo;
III - só se aplica às doações em espécie; e
IV - não exclui ou reduz outros benefícios ou deduções em vigor.
§ 3º O pagamento da doação deve ser efetuado até a data de vencimento da primeira quota ou quota
única do imposto, observadas instruções específicas da Secretaria da Receita Federal do Brasil.
§ 4º O não pagamento da doação no prazo estabelecido no § 3o implica a glosa definitiva desta parcela
de dedução, ficando a pessoa física obrigada ao recolhimento da diferença de imposto devido apurado
na Declaração de Ajuste Anual com os acréscimos legais previstos na legislação.
§ 5º A pessoa física poderá deduzir do imposto apurado na Declaração de Ajuste Anual as doações
feitas, no respectivo ano-calendário, aos fundos controlados pelos Conselhos dos Direitos da Criança e

Apostila gerada especialmente para: Betânia Mueller 023.712.580-32


. 77
do Adolescente municipais, distrital, estaduais e nacional concomitantemente com a opção de que trata
o caput, respeitado o limite previsto no inciso II do art. 260.

Art. 260-B. A doação de que trata o inciso I do art. 260 poderá ser deduzida:
I - do imposto devido no trimestre, para as pessoas jurídicas que apuram o imposto trimestralmente; e
II - do imposto devido mensalmente e no ajuste anual, para as pessoas jurídicas que apuram o imposto
anualmente.
Parágrafo único. A doação deverá ser efetuada dentro do período a que se refere a apuração do
imposto.

Art. 260-C. As doações de que trata o art. 260 desta Lei podem ser efetuadas em espécie ou em bens.
Parágrafo único. As doações efetuadas em espécie devem ser depositadas em conta específica, em
instituição financeira pública, vinculadas aos respectivos fundos de que trata o art. 260.

Art. 260-D. Os órgãos responsáveis pela administração das contas dos Fundos dos Direitos da Criança
e do Adolescente nacional, estaduais, distrital e municipais devem emitir recibo em favor do doador,
assinado por pessoa competente e pelo presidente do Conselho correspondente, especificando:
I - número de ordem;
II - nome, Cadastro Nacional da Pessoa Jurídica (CNPJ) e endereço do emitente;
III - nome, CNPJ ou Cadastro de Pessoas Físicas (CPF) do doador;
IV - data da doação e valor efetivamente recebido; e
V - ano-calendário a que se refere a doação.
§ 1o O comprovante de que trata o caput deste artigo pode ser emitido anualmente, desde que
discrimine os valores doados mês a mês.
§ 2o No caso de doação em bens, o comprovante deve conter a identificação dos bens, mediante
descrição em campo próprio ou em relação anexa ao comprovante, informando também se houve
avaliação, o nome, CPF ou CNPJ e endereço dos avaliadores.

Art. 260-E. Na hipótese da doação em bens, o doador deverá:


I - comprovar a propriedade dos bens, mediante documentação hábil;
II - baixar os bens doados na declaração de bens e direitos, quando se tratar de pessoa física, e na
escrituração, no caso de pessoa jurídica; e
III - considerar como valor dos bens doados:
a) para as pessoas físicas, o valor constante da última declaração do imposto de renda, desde que
não exceda o valor de mercado;
b) para as pessoas jurídicas, o valor contábil dos bens.
Parágrafo único. O preço obtido em caso de leilão não será considerado na determinação do valor dos
bens doados, exceto se o leilão for determinado por autoridade judiciária.

Art. 260-F. Os documentos a que se referem os arts. 260-D e 260-E devem ser mantidos pelo
contribuinte por um prazo de 5 (cinco) anos para fins de comprovação da dedução perante a Receita
Federal do Brasil.

Art. 260-G. Os órgãos responsáveis pela administração das contas dos Fundos dos Direitos da Criança
e do Adolescente nacional, estaduais, distrital e municipais devem:
I - manter conta bancária específica destinada exclusivamente a gerir os recursos do Fundo;
II - manter controle das doações recebidas; e
III - informar anualmente à Secretaria da Receita Federal do Brasil as doações recebidas mês a mês,
identificando os seguintes dados por doador:
a) nome, CNPJ ou CPF;
b) valor doado, especificando se a doação foi em espécie ou em bens.

Art. 260-H. Em caso de descumprimento das obrigações previstas no art. 260-G, a Secretaria da
Receita Federal do Brasil dará conhecimento do fato ao Ministério Público.

Art. 260-I. Os Conselhos dos Direitos da Criança e do Adolescente nacional, estaduais, distrital e
municipais divulgarão amplamente à comunidade:
I - o calendário de suas reuniões;
II - as ações prioritárias para aplicação das políticas de atendimento à criança e ao adolescente;

Apostila gerada especialmente para: Betânia Mueller 023.712.580-32


. 78
III - os requisitos para a apresentação de projetos a serem beneficiados com recursos dos Fundos dos
Direitos da Criança e do Adolescente nacional, estaduais, distrital ou municipais;
IV - a relação dos projetos aprovados em cada ano-calendário e o valor dos recursos previstos para
implementação das ações, por projeto;
V - o total dos recursos recebidos e a respectiva destinação, por projeto atendido, inclusive com
cadastramento na base de dados do Sistema de Informações sobre a Infância e a Adolescência; e
VI - a avaliação dos resultados dos projetos beneficiados com recursos dos Fundos dos Direitos da
Criança e do Adolescente nacional, estaduais, distrital e municipais.

Art. 260-J. O Ministério Público determinará, em cada Comarca, a forma de fiscalização da aplicação
dos incentivos fiscais referidos no art. 260 desta Lei.
Parágrafo único. O descumprimento do disposto nos arts. 260-G e 260-I sujeitará os infratores a
responder por ação judicial proposta pelo Ministério Público, que poderá atuar de ofício, a requerimento
ou representação de qualquer cidadão.

Art. 260-K. A Secretaria de Direitos Humanos da Presidência da República (SDH/PR) encaminhará à


Secretaria da Receita Federal do Brasil, até 31 de outubro de cada ano, arquivo eletrônico contendo a
relação atualizada dos Fundos dos Direitos da Criança e do Adolescente nacional, distrital, estaduais e
municipais, com a indicação dos respectivos números de inscrição no CNPJ e das contas bancárias
específicas mantidas em instituições financeiras públicas, destinadas exclusivamente a gerir os recursos
dos Fundos.

Art. 260-L. A Secretaria da Receita Federal do Brasil expedirá as instruções necessárias à aplicação
do disposto nos arts. 260 a 260-K.

Art. 261. A falta dos conselhos municipais dos direitos da criança e do adolescente, os registros,
inscrições e alterações a que se referem os arts. 90, parágrafo único, e 91 desta Lei serão efetuados
perante a autoridade judiciária da comarca a que pertencer a entidade.
Parágrafo único. A União fica autorizada a repassar aos estados e municípios, e os estados aos
municípios, os recursos referentes aos programas e atividades previstos nesta Lei, tão logo estejam
criados os conselhos dos direitos da criança e do adolescente nos seus respectivos níveis.

Art. 262. Enquanto não instalados os Conselhos Tutelares, as atribuições a eles conferidas serão
exercidas pela autoridade judiciária.

Art. 263. O Decreto-Lei nº 2.848, de 7 de dezembro de 1940 (Código Penal), passa a vigorar com as
seguintes alterações:
1) Art. 121 ............................................................
§ 4º No homicídio culposo, a pena é aumentada de um terço, se o crime resulta de inobservância de
regra técnica de profissão, arte ou ofício, ou se o agente deixa de prestar imediato socorro à vítima, não
procura diminuir as consequências do seu ato, ou foge para evitar prisão em flagrante. Sendo doloso o
homicídio, a pena é aumentada de um terço, se o crime é praticado contra pessoa menor de catorze anos.
2) Art. 129 ...............................................................
§ 7ºAumenta-se a pena de um terço, se ocorrer qualquer das hipóteses do art. 121, § 4º.
§ 8º Aplica-se à lesão culposa o disposto no § 5º do art. 121.
3) Art. 136.................................................................
§ 3º Aumenta-se a pena de um terço, se o crime é praticado contra pessoa menor de catorze anos.
4) Art. 213 ..................................................................
Parágrafo único. Se a ofendida é menor de catorze anos:
Pena - reclusão de quatro a dez anos.
5) Art. 214...................................................................
Parágrafo único. Se o ofendido é menor de catorze anos:
Pena - reclusão de três a nove anos.

Art. 264. O art. 102 da Lei nº 6.015, de 31 de dezembro de 1973, fica acrescido do seguinte item:
"Art. 102 ....................................................................
6º) a perda e a suspensão do pátrio poder. "

Apostila gerada especialmente para: Betânia Mueller 023.712.580-32


. 79
Art. 265. A Imprensa Nacional e demais gráficas da União, da administração direta ou indireta, inclusive
fundações instituídas e mantidas pelo poder público federal promoverão edição popular do texto integral
deste Estatuto, que será posto à disposição das escolas e das entidades de atendimento e de defesa dos
direitos da criança e do adolescente.

Art. 265-A. O poder público fará periodicamente ampla divulgação dos direitos da criança e do
adolescente nos meios de comunicação social. (Redação dada pela Lei nº 13.257, de 2016)
Parágrafo único. A divulgação a que se refere o caput será veiculada em linguagem clara,
compreensível e adequada a crianças e adolescentes, especialmente às crianças com idade inferior a 6
(seis) anos. (Incluído dada pela Lei nº 13.257, de 2016)

Art. 266. Esta Lei entra em vigor noventa dias após sua publicação.
Parágrafo único. Durante o período de vacância deverão ser promovidas atividades e campanhas de
divulgação e esclarecimentos acerca do disposto nesta Lei.

Art. 267. Revogam-se as Leis nº 4.513, de 1964, e 6.697, de 10 de outubro de 1979 (Código de
Menores), e as demais disposições em contrário.

Brasília, 13 de julho de 1990; 169º da Independência e 102º da República.

FERNANDO COLLOR

Questões

01. (Câmara de Salvador/BA - Especialista - FGV/2018). De acordo com o Estatuto da Criança e do


Adolescente (ECA), a política de atendimento dos direitos da criança e do adolescente será realizada
mediante um conjunto articulado de ações governamentais e não governamentais.
Constituem linhas de ação da política de atendimento:
(A) as políticas sociais básicas;
(B) a colocação familiar;
(C) o acolhimento institucional;
(D) a internação;
(E) o atendimento psicossocial às famílias.

02. (Câmara de Salvador/BA - Especialista - Assistente Social). Mariana está prestes a dar à luz e
solicita ao obstetra que sua prima Luci fique com ela durante o trabalho de parto. Todavia, Fábio, o
companheiro de Mariana, não concorda, alegando ser seu direito permanecer junto a Mariana durante o
trabalho de parto, uma vez que é o pai da criança que irá nascer.
De acordo com o Estatuto da Criança e do Adolescente (ECA):
(A) a parturiente tem direito a um acompanhante de sua preferência;
(B) a equipe de saúde deverá designar quem acompanhará a parturiente;
(C) o pai da criança tem prioridade em acompanhar a parturiente;
(E) a avó materna tem permanência garantida junto à parturiente, devido à sua experiência;
(E) cabe ao obstetra decidir se e quem poderá acompanhar a parturiente.

03. (EBSERH - Assistente Social - INSTITUTO AOCP/2017). A respeito dos direitos da Criança e do
Adolescente, presentes na Lei 8069/93, assinale a alternativa correta.
(A) A permanência da criança e do adolescente em programa de acolhimento institucional não se
prolongará por mais de 1 ano, salvo comprovada necessidade que atenda ao seu superior interesse,
devidamente fundamentada pela autoridade judiciária.
(B) A colocação em família substituta farse-á mediante guarda, tutela ou adoção, independentemente
da situação jurídica da criança ou adolescente, nos termos desta Lei.
(C) O adotante há de ser, pelo menos, dezessete anos mais velho do que o adotando.
(D) A falta ou a carência de recursos materiais constitui motivo suficiente para a perda ou a suspensão
do poder familiar.
(E) É dever da família velar pela dignidade da criança e do adolescente, pondo-os a salvo de qualquer
tratamento desumano, violento, aterrorizante, vexatório ou constrangedor.

Apostila gerada especialmente para: Betânia Mueller 023.712.580-32


. 80
04. (SESAU/RO - Assistente Social - FUNRIO/2017). Em relação ao direito à vida e à saúde,
conforme estabelece o Estatuto da Criança e do Adolescente (ECA), as seguintes afirmativas estão
corretas, EXCETO:
(A) a criança e o adolescente têm direito a proteção à vida e à saúde, mediante a efetivação de políticas
sociais públicas que permitam o nascimento e o desenvolvimento sadio e harmonioso, em condições
dignas de existência.
(B) é assegurado a todas as mulheres o acesso aos programas e às políticas de saúde da mulher e
de planejamento reprodutivo e, às gestantes, nutrição adequada, atenção humanizada à gravidez, ao
parto e ao puerpério e atendimento pré-natal, perinatal e pós-natal integral no âmbito do Sistema Único
de Saúde
(C) o atendimento pré-natal será realizado por profissionais da atenção primária.
(D) os profissionais de saúde de referência da gestante garantirão sua vinculação, desde o primeiro
trimestre da gestação, ao estabelecimento em que será realizado o parto, garantido o direito de opção da
mulher
(E) os serviços de saúde onde o parto for realizado assegurarão às mulheres e aos seus filhos recém-
nascidos alta hospitalar responsável e contrarreferência na atenção primária, bem como o acesso a outros
serviços e a grupos de apoio à amamentação.

05. (EBSERH - Assistente Social - INSTITUTO AOCP/2017). Um adolescente de 17 anos, com dores
abdominais e mal-estar, foi até um Hospital Universitário acompanhado de sua genitora, onde, após ser
avaliado por médico, foi indicado internação por se tratar de suspeita de uma virose. A genitora relatou
que não poderia acompanhar a internação do filho, pois teria que retornar ao seu trabalho, e disse ainda
que por ele já ter 17 anos de idade, teria total condição de ficar sozinho. De acordo com a Lei 8069/90,
qual é o procedimento que o assistente social do hospital deve adotar?
(A) Por se tratar de adolescente com idade superior a 12 anos, informar a genitora que ele ficará sob
os cuidados da equipe de enfermagem.
(B) Orientar à genitora que é obrigatória a permanência em tempo integral de um dos pais ou
responsável, nos casos de internação de criança ou adolescente.
(C) Orientar à genitora que retorne apenas no período noturno, após seu horário de expediente no
trabalho.
(D) Orientar à genitora que seu outro filho de 16 anos poderá acompanhar a internação do adolescente.
(E) Orientar à genitora que o serviço social do Hospital ficará responsável pela internação do
adolescente.

06. (UFPA - Assistente Social - UFPA/2017). Com base na Lei nº 8.069/1990 (Estatuto da Criança e
do Adolescente), é CORRETO afirmar que
(A) a adoção será precedida de estágio de convivência com a criança ou o adolescente, pelo prazo
que a autoridade judiciária fixar, salvo se o adotando já estiver sob a tutela, guarda legal ou guarda de
fato do adotante durante tempo suficiente para que seja possível avaliar a conveniência da constituição
do vínculo.
(B) o vínculo da adoção é constituído por sentença judicial, que será inscrita no registro civil mediante
mandado cuja inscrição consignará o nome dos adotantes como pais e o de seus ascendentes, bem como
uma observação sobre a origem do ato, cancelando o registro original do adotado.
(C) o adotado tem direito de conhecer sua origem biológica, bem como de obter acesso irrestrito ao
processo no qual a medida foi aplicada e seus eventuais incidentes, após completar 18 (dezoito) anos
ou, se menor de 18 (dezoito) anos, a seu pedido, assegurada orientação e assistência jurídica e
psicológica.
(D) a adoção, que depende do consentimento dos pais ou do representante legal, produz seus efeitos
a partir do trânsito em julgado da sentença constitutiva, exceto na hipótese de o adotante falecer no curso
do procedimento, caso em que terá força retroativa à data do ajuizamento do pedido de adoção.
(E) os divorciados e os ex-companheiros podem adotar conjuntamente, contanto que acordem sobre
a guarda e o regime de visitas e desde que iniciem o estágio de convivência em até seis meses após a
separação e comprovem que existe vínculo de afinidade e afetividade entre o adotante e o não detentor
da guarda.

07. (SEJUDH/MT - Assistente Social - IBADE/2017). O Estatuto da Criança e do Adolescente


estabelece o estágio de convivência prévio à adoção. Considerando a adoção de uma criança de 6 (seis)
anos de idade por um casal residente fora do país, considerando a lei supracitada, o estágio de
convivência será cumprido:

Apostila gerada especialmente para: Betânia Mueller 023.712.580-32


. 81
(A) no local de preferência dos adotantes por no mínimo 120 dias.
(B) no pais de origem do adotante por no máximo 90 dias.
(C) em território nacional por no mínimo 30 dias.
(D) no país de origem do adotante por 60 dias
(E) em território nacional por no máximo 60 dias.

Gabarito

01.A / 02.A / 03.B / 04.D/ 05.B / 06. C / 07.C

Comentários
01. Resposta: A
Lei 8.069/1990
Art. 87. São linhas de ação da política de atendimento:
I - políticas sociais básicas;

02. Resposta: A
Lei 8.069/1990
Art. 8º É assegurado a todas as mulheres o acesso aos programas e às políticas de saúde da mulher
e de planejamento reprodutivo e, às gestantes, nutrição adequada, atenção humanizada à gravidez, ao
parto e ao puerpério e atendimento pré-natal, perinatal e pós-natal integral no âmbito do Sistema Único
de Saúde. (Redação dada pela Lei nº 13.257, de 2016)
( )
§ 6º A gestante e a parturiente têm direito a 1 (um) acompanhante de sua preferência durante o período
do pré-natal, do trabalho de parto e do pós-parto imediato. (Incluído pela Lei nº 13.257, de 2016)

03. Resposta: B
Lei 8.069/1990
Art. 28. A colocação em família substituta far-se-á mediante guarda, tutela ou adoção,
independentemente da situação jurídica da criança ou adolescente, nos termos desta Lei.

04. Resposta: D
Lei 8.069/1990
Art. 8º É assegurado a todas as mulheres o acesso aos programas e às políticas de saúde da mulher
e de planejamento reprodutivo e, às gestantes, nutrição adequada, atenção humanizada à gravidez, ao
parto e ao puerpério e atendimento pré-natal, perinatal e pós-natal integral no âmbito do Sistema Único
de Saúde. (Redação dada pela Lei nº 13.257, de 2016)
( )
§ 2º Os profissionais de saúde de referência da gestante garantirão sua vinculação, no último trimestre
da gestação, ao estabelecimento em que será realizado o parto, garantido o direito de opção da mulher.
(Redação dada pela Lei nº 13.257, de 2016).

05. Resposta: B
Lei 8.069/1990
Art. 12. Os estabelecimentos de atendimento à saúde, inclusive as unidades neonatais, de terapia
intensiva e de cuidados intermediários, deverão proporcionar condições para a permanência em tempo
integral de um dos pais ou responsável, nos casos de internação de criança ou adolescente. (Redação
dada pela Lei nº 13.257, de 2016)

06. Resposta: C
Lei 8.069/1990
Art. 48. O adotado tem direito de conhecer sua origem biológica, bem como de obter acesso irrestrito
ao processo no qual a medida foi aplicada e seus eventuais incidentes, após completar 18 (dezoito) anos.

07. Resposta: C
Lei 8.069/1990
Art. 46. A adoção será precedida de estágio de convivência com a criança ou adolescente, pelo prazo
máximo de 90 (noventa) dias, observadas a idade da criança ou adolescente e as peculiaridades do caso.
(Redação dada pela Lei nº 13.509, de 2017)

Apostila gerada especialmente para: Betânia Mueller 023.712.580-32


. 82
( )
§ 3º Em caso de adoção por pessoa ou casal residente ou domiciliado fora do País, o estágio de
convivência será de, no mínimo, 30 (trinta) dias e, no máximo, 45 (quarenta e cinco) dias, prorrogável por
até igual período, uma única vez, mediante decisão fundamentada da autoridade judiciária. (Redação
dada pela Lei nº 13.509, de 2017).

Crianças e adolescentes em situação de acolhimento institucional e familiar.

1 INTRODUÇÃO

No Brasil, as origens do atendimento a crianças e adolescentes em serviços de acolhimento remontam


ao período colonial. Mas foi apenas com a promulgação do Estatuto da Criança e do Adolescente (ECA)
que crianças e adolescentes passaram a ser concebidos como sujeitos de direito, em peculiar condição
de desenvolvimento e que o encaminhamento para serviço de acolhimento passou a ser concebido como
medida protetiva, de caráter excepcional e provisório (Art.101). O ECA assegurou, ainda, o direito de
crianças e adolescentes à convivência familiar e comunitária, prioritariamente na família de origem e,
excepcionalmente, em família substituta (Art. 19). 6
Em conformidade com as disposições do ECA, deve-se recorrer ao encaminhamento da criança e do
adolescente a serviços de acolhimento apenas quando esgotados todos os recursos para sua
manutenção na família de origem, extensa ou comunidade. A história brasileira revela, todavia, que, frente
à situação de pobreza, vulnerabilidade ou risco, a primeira resposta à qual durante muitos anos se
recorreu foi o afastamento da criança e do adolescente do convívio familiar. A promulgação do ECA veio
romper com essa cultura, ao garantir a excepcionalidade da medida, estabelecendo, ainda, que a situação
de pobreza da família não constitui motivo suficiente para o afastamento da criança e do adolescente do
convívio familiar (Art. 23).
Para garantir a excepcionalidade do afastamento do convívio familiar, o Art.130 do ECA estabelece
que, nos casos de violência praticada por familiar ou responsável com o qual a criança ou adolescente
resida, a autoridade judiciária poderá determinar o afastamento do agressor da moradia comum. De forma
a promover a qualidade dos serviços de acolhimento, o ECA prevê, ainda, ações de fiscalização e controle
social, ao exigir a inscrição das entidades que ofertam “programas de abrigo” no Conselho Municipal dos
Direitos da Criança e do Adolescente (Art. 90) e estabelecer princípios para sua organização (Art. 92). Do
mesmo modo, como constituem serviços que compõem a rede socioassistencial, os serviços de
acolhimento devem também possuir registro no Conselho Municipal de Assistência Social e submeter-se
também à sua fiscalização.
A despeito dos direitos assegurados no ECA, o Levantamento Nacional de Abrigos para Crianças e
Adolescentes, realizado pelo Instituto de Pesquisa Econômica Aplicada (IPEA), em 2003, identificou o
descompasso existente entre a legislação e a realidade dos serviços de acolhimento para crianças e
adolescentes no Brasil. A pesquisa foi realizada em 584 serviços de acolhimento para crianças e
adolescentes co-financiadas com recursos do MDS e representou um marco na mobilização nacional
para a discussão acerca do direito de crianças e adolescentes à convivência familiar e comunitária. Esse
processo iniciado em 2002, com a Caravana da Comissão de Direitos Humanos da Câmara dos
Deputados , ganhou impulso, em 2004, após a publicação da pesquisa, quando o Ministro Chefe da SEDH
e o Ministro de Estado do MDS, com o apoio do UNICEF, convocaram outros Ministérios e atores para a
formação de uma Comissão Intersetorial que apresentou os subsídios ao CONANDA e ao CNAS para
elaboração do Plano Nacional de Promoção, Proteção e Defesa do Direito de Crianças e Adolescentes à
Convivência Familiar e Comunitária. Os Conselhos analisaram e aprimoraram o documento, aprovado
em assembleia conjunta em 2006, após incorporação das sugestões apresentadas na consulta pública.
Ressalte-se que, além dos dois Conselhos e do Poder Público, a sociedade civil teve também uma
importante participação na construção coletiva do Plano Nacional.
Paralelo a esse processo, em 2004, foi aprovada, pelo CNAS, a Política Nacional de Assistência Social
(PNAS), com o objetivo de concretizar direitos assegurados na Constituição Federal (1988) e na Lei
Orgânica de Assistência Social (1993). A PNAS organiza a matriz de funcionamento do Sistema Único de
Assistência Social (SUAS), inaugurando no país um novo paradigma de defesa dos direitos
socioassistenciais. Na sequência, a aprovação da NOB/SUAS estabeleceu parâmetros para a
operacionalização do Sistema Único em todo o território nacional. Em 2006, foi aprovada a NOB-RH do
6
http://www.mds.gov.br/cnas/noticias/orientacoes_tecnicas_final.pdf

Apostila gerada especialmente para: Betânia Mueller 023.712.580-32


. 83
SUAS que, dentre outros aspectos, estabeleceu parâmetros nacionais para a composição das equipes
que devem atuar nos serviços de acolhimento.
A organização do SUAS como um sistema pressupõe a articulação da rede socioassistencial com as
demais políticas públicas e com o Sistema de Garantia de Direitos (SGD) e elege a família como foco
central de atenção. A previsão de serviços de caráter preventivo e de fortalecimento de vínculos familiares
e comunitários10, de atendimento especializado a indivíduos e famílias em situação de ameaça ou
violação de direitos e de serviços de acolhimento para crianças e adolescentes tem importância basilar
no que diz respeito à concretização do direito à convivência familiar e comunitária.
Um grande desafio que é colocado para a implementação do SUAS no país é o de reordenar os
serviços de acolhimento e romper com práticas incompatíveis com os marcos regulatórios vigentes. De
acordo com a PNAS (2004),
A ênfase da proteção social especial deve priorizar a reestruturação dos serviços de abrigamento - dos
indivíduos que, por uma série de fatores, não contam mais com a proteção e o cuidado de suas famílias
- para as novas modalidades de atendimento. A história dos abrigos e asilos é antiga no Brasil. A
colocação de crianças, adolescentes, pessoas com deficiências e idosos em instituições para protegê-los
ou afastá-los do convívio social e familiar foi, durante muito tempo, materializada em grandes instituições
de longa permanência, ou seja, espaços que atendiam a um grande número de pessoas, que lá
permaneciam por longo período – às vezes a vida toda. São os chamados, popularmente, como orfanatos,
internatos, educandários, asilos, entre outros.
Nesse sentido, o presente documento visa estabelecer parâmetros de funcionamento e oferecer
orientações metodológicas para que os serviços de acolhimento de crianças e adolescentes possam
cumprir sua função protetiva e de restabelecimento de direitos, compondo uma rede de proteção que
favoreça o fortalecimento dos vínculos familiares e comunitários, o desenvolvimento de potencialidades
das crianças e adolescentes atendidos e o empoderamento de suas famílias.
A elaboração dos parâmetros apresentados neste documento dá continuidade ao compromisso do
MDS, da SEDH, do CONANDA, do CNAS e da sociedade civil organizada com a afirmação, no estado
brasileiro, do direito de crianças e adolescentes à convivência familiar e comunitária.

2 PRINCÍPIOS

Os serviços de acolhimento para crianças e adolescentes deverão estruturar seu atendimento de


acordo com os seguintes princípios:

2.1 Excepcionalidade do Afastamento do Convívio Familiar

Todos os esforços devem ser empreendidos no sentido de manter o convívio com a família (nuclear
ou extensa, em seus diversos arranjos), a fim de garantir que o afastamento da criança ou do adolescente
do contexto familiar seja uma medida excepcional, aplicada apenas nas situações de grave risco à sua
integridade física e/ou psíquica.
Como este afastamento traz profundas implicações, tanto para a criança e o adolescente, quanto para
a família, deve-se recorrer a esta medida apenas quando representar o melhor interesse da criança ou
do adolescente e o menor prejuízo ao seu processo de desenvolvimento. Destaca-se que tal medida deve
ser aplicada apenas nos casos em que não for possível realizar uma intervenção mantendo a criança ou
adolescente no convívio com sua família (nuclear ou extensa).
Para que este princípio possa ser aplicado, é importante que se promova o fortalecimento, a
emancipação e a inclusão social das famílias, por meio do acesso às políticas públicas e às ações
comunitárias. Dessa forma, antes de se considerar a hipótese do afastamento, é necessário assegurar à
família o acesso à rede de serviços públicos que possam potencializar as condições de oferecer à criança
ou ao adolescente um ambiente seguro de convivência.
Destaca-se que, em conformidade com o Art. 23 do ECA, a falta de recursos materiais por si só não
constitui motivo suficiente para afastar a criança ou o adolescente do convívio familiar, encaminhá-los
para serviços de acolhimento ou, ainda, para inviabilizar sua reintegração. Nessas situações o convívio
familiar deve ser preservado e a família, obrigatoriamente, incluída em programas oficiais ou comunitários
de apoio, e demais medidas previstas no artigo 101 do ECA.
Nos casos em que o motivo que ensejaria a aplicação da medida de abrigo referir-se à falta ou
precariedade de condições de habitação da família, deve-se recorrer a medidas que preservem o convívio
familiar e mantenham a família, a criança e o adolescente em condições de segurança e proteção, como
a inclusão imediata de todos seus membros conjuntamente em serviços de acolhimento para adultos com
crianças ou adolescentes e acesso à moradia subsidiada, dentre outras. Paralelamente, deve ser

Apostila gerada especialmente para: Betânia Mueller 023.712.580-32


. 84
providenciado, junto às políticas de habitação e trabalho, e outras que o caso indicar, os
encaminhamentos necessários para alcançar soluções mais definitivas para a situação.
Do mesmo modo, caso haja criança, adolescente ou algum outro membro da família com deficiência,
doenças infectocontagiosas, transtorno mental ou outros agravos, isso não deve por si só motivar o
afastamento do convívio familiar ou a permanência em serviços de acolhimento. Nessas situações deve-
se proceder a encaminhamentos para atendimentos prestados em serviços da rede ou até mesmo no
próprio domicílio, os quais possam contribuir para a prevenção do afastamento ou para a reintegração
familiar. Se identificado que a criança, o adolescente ou outro membro da família preenche os critérios
para inclusão no Benefício de Prestação Continuada (BPC), deve-se viabilizar seu encaminhamento para
o Centro de Referência de Assistência Social (CRAS), ou órgão responsável para realizar sua inserção.
Destaca-se, ainda, a importância da inclusão e do acompanhamento da criança, do adolescente e da
família nos serviços de saúde adequados a demanda apresentada. Tais medidas são fundamentais para
evitar que a situação de pobreza, associada à presença de deficiência, resulte em afastamentos
motivados predominantemente por estes aspectos. De modo a viabilizar o acesso aos serviços das
diversas políticas públicas, orienta-se que sejam formalizados, entre os órgãos responsáveis por tais
políticas, protocolos de ação que assegurem a prioridade de acesso e o encaminhamento imediato das
famílias nessa situação a tais serviços, programas, projetos, benefícios e ações.

2.2 Provisoriedade do Afastamento do Convívio Familiar

Quando o afastamento do convívio familiar for medida mais adequada para se garantir a proteção da
criança e do adolescente em determinado momento, esforços devem ser empreendidos para viabilizar,
no menor tempo possível, o retorno seguro ao convívio familiar, prioritariamente na família de origem e,
excepcionalmente, em família substituta (adoção, guarda e tutela), conforme Capítulo III, Seção III do
ECA.
Todos os esforços devem ser empreendidos para que, em um período inferior a dois anos, seja
viabilizada a reintegração familiar – para família nuclear ou extensa, em seus diversos arranjos – ou, na
sua impossibilidade, o encaminhamento para família substituta. A permanência de crianças e
adolescentes em serviço de acolhimento por período superior a dois anos deverá ter caráter
extremamente excepcional, e estar fundamentada em uma avaliação criteriosa acerca de sua
necessidade pelos diversos órgãos que acompanham o caso.
Quando o prognóstico de permanência da criança e do adolescente no serviço de acolhimento for de
mais de dois anos, deve ser encaminhado à Justiça da Infância e da Juventude relatório baseado no
acompanhamento da situação pelo serviço de acolhimento e em outros serviços da rede que também
prestem atendimento à criança, ao adolescente e sua família. Tal relatório será fundamental para
subsidiar a avaliação, por parte da Justiça, quanto à melhor alternativa para a criança e o adolescente,
seja a continuidade dos esforços para o retorno ao convívio familiar ou o encaminhamento para família
substituta.
Nas situações em que se mostrar particularmente difícil garantir o direito à convivência familiar, como,
por exemplo, no caso encaminhamento para adoção de crianças e adolescentes com perfil de difícil
colocação em família substituta , faz-se especialmente necessário o esforço conjunto dos atores
envolvidos no sentido de buscar o fortalecimento da autonomia e das redes sociais de apoio das crianças
e adolescentes que aguardam adoção, e perseverar no desenvolvimento de estratégias para a busca
ativa de famílias para seu acolhimento.
Em nenhuma hipótese a perspectiva de um acolhimento de longa permanência deve acarretar a
desistência pela busca de alternativas para se garantir à criança e ao adolescente seu direito ao convívio
familiar, prioritariamente com a família de origem e, excepcionalmente, a substituta.

2.3 Preservação e Fortalecimento dos Vínculos Familiares e Comunitários

Todos os esforços devem ser empreendidos para preservar e fortalecer vínculos familiares e
comunitários das crianças e dos adolescentes atendidos em serviços de acolhimento. Esses vínculos são
fundamentais, nessa etapa do desenvolvimento humano, para oferecer-lhes condições para um
desenvolvimento saudável, que favoreça a formação de sua identidade e sua constituição como sujeito e
cidadão. Nesse sentido, é importante que esse fortalecimento ocorra nas ações cotidianas dos serviços
de acolhimento - visitas e encontros com as famílias e com as pessoas de referências da comunidade da
criança e do adolescente, por exemplo.

Apostila gerada especialmente para: Betânia Mueller 023.712.580-32


. 85
Crianças e adolescentes com vínculos de parentesco, não devem ser separados ao serem
encaminhados para serviço de acolhimento, salvo se isso for contrário ao seu desejo ou interesses ou se
houver claro risco de violência.

2.4 Garantia de Acesso e Respeito à Diversidade e Não-discriminação

A organização dos serviços deverá garantir proteção e defesa a toda a criança e adolescente que
precise de acolhimento. Devem ser combatidas quaisquer formas de discriminação às crianças e aos
adolescentes atendidos em serviços de acolhimento e às famílias de origem, baseadas em condição
socioeconômica, arranjo familiar, etnia, religião, gênero, orientação sexual, ou, ainda, por serem pessoas
com necessidades especiais em decorrência de deficiência física ou mental, que vivem com HIV/AIDS ou
outras necessidades específicas de saúde.
De modo a possibilitar a oferta de um atendimento inclusivo e de qualidade nos serviços de
acolhimento a crianças e adolescentes, o Projeto Político Pedagógico do serviço deve prever estratégias
diferenciadas para o atendimento a demandas específicas, mediante acompanhamento de profissional
especializado. Além disso, a articulação com a política de saúde, de educação, esporte e cultura deve
garantir o atendimento na rede local a estas crianças e adolescentes (serviços especializados, tratamento
e medicamentos, dentre outros) e a capacitação e apoio necessário aos educadores/cuidadores e demais
profissionais do serviço de acolhimento. Tal aspecto é importante para garantir, de fato, um atendimento
individualizado e personalizado, com estratégias metodológicas condizentes com as necessidades da
criança e do adolescente. Todos os equipamentos da rede socioassistencial devem, ainda, respeitar as
normas de acessibilidade, de maneira a possibilitar o atendimento integrado a usuários com deficiência.
Em atenção ao princípio da não-discriminação, os serviços de acolhimento devem buscar o crescente
aprimoramento de estratégias voltadas à preservação da diversidade cultural, oportunizando acesso e
valorização das raízes e cultura de origem das crianças e dos adolescentes atendidos, bem como de suas
famílias e comunidades de origem.
Em atenção, ainda, ao princípio da não discriminação, destaca-se que a presença de deficiência ou
de necessidades específicas de saúde não deve motivar o encaminhamento para serviço de acolhimento
ou, ainda, o prolongamento da permanência da criança ou adolescente nestes serviços.

2.5 Oferta de Atendimento Personalizado e Individualizado

Toda criança e adolescente tem direito a viver num ambiente que favoreça seu processo de
desenvolvimento, que lhe ofereça segurança, apoio, proteção e cuidado. Nesse sentido, quando o
afastamento for necessário e enquanto soluções para a retomada do convívio familiar forem buscadas,
os serviços de acolhimento deverão prestar cuidados de qualidade, condizentes com os direitos e as
necessidades físicas, psicológicas e sociais da criança e do adolescente. Para tanto, o atendimento
deverá ser oferecido para um pequeno grupo e garantir espaços privados, objetos pessoais e registros,
inclusive fotográficos, sobre a história de vida e desenvolvimento de cada criança e adolescente.
A organização de condições que favoreçam a formação da identidade da criança e do adolescente
implica o respeito à sua individualidade e história de vida. O planejamento do atendimento no serviço
deve possibilitar, portanto, espaços que preservem a intimidade e a privacidade, inclusive, o uso de
objetos que possibilitem à criança e ao adolescente diferenciar “o meu, o seu e o nosso”.
Os antecedentes religiosos de crianças e adolescentes devem ser respeitados tanto pelo serviço de
acolhimento quanto por aqueles com os quais venha a manter contato em razão de seu acolhimento.
“Nenhuma criança ou adolescente deverá ser incentivado ou persuadido a mudar sua orientação religiosa
enquanto estiver sob cuidados” em serviço de acolhimento.
Visando a garantia do direito à liberdade de crença e culto religioso, assegurado no Art. 16 do ECA,
os serviços de acolhimento devem propiciar, ainda, que a criança e o adolescente possam satisfazer suas
necessidades de vida religiosa e espiritual. Nesse sentido, deve ser viabilizado o acesso às atividades de
sua religião, bem como o direito de “não participar de atos religiosos e recusar instrução ou orientação
religiosa que não lhe seja significativa”.

2.7 Respeito à Autonomia da Criança, do Adolescente e do Jovem

As decisões acerca de crianças e adolescentes atendidos em serviços de acolhimento devem garantir-


lhes o direito de ter sua opinião considerada. O direito à escuta, viabilizada por meio de métodos
condizentes com seu grau de desenvolvimento, deve ser garantido nas diversas decisões que possam
repercutir sobre seu desenvolvimento e trajetória de vida, envolvendo desde a identificação de seu

Apostila gerada especialmente para: Betânia Mueller 023.712.580-32


. 86
interesse pela participação em atividades na comunidade, até mudanças relativas à sua situação familiar
ou desligamento do serviço de acolhimento.
A organização do ambiente de acolhimento também deverá proporcionar o fortalecimento gradativo da
autonomia, de modo condizente com o processo de desenvolvimento e a aquisição de habilidades nas
diferentes faixas etárias. O desenvolvimento da autonomia nos serviços de acolhimento não deve ser
confundido, todavia, com falta de autoridade e limites. A liberdade deve ser vista como parceira da
responsabilidade, considerando que uma não pode ser adquirida sem a outra.
Crianças e adolescentes devem ter a oportunidade de participar da organização do cotidiano do serviço
de acolhimento, por meio do desenvolvimento de atividades como, por exemplo, a organização dos
espaços de moradia, limpeza, programação das atividades recreativas, culturais e sociais. No caso de
serviços de acolhimento institucional, esta participação pode ser viabilizada, inclusive, por meio da
realização sistemática de assembleias, nas quais crianças e adolescentes possam se colocar de modo
protagonista.
Devem ser planejadas ações que favoreçam a interação das crianças e dos adolescentes entre si e
com os contextos nos quais frequentam, como a escola, a comunidade, e as instituições religiosas. O
desenvolvimento da autonomia deve levar em consideração, ainda, a cultura de origem da criança e do
adolescente e fortalecer a elaboração de projetos de vida individuais e o desenvolvimento saudável,
inclusive após o desligamento e a entrada na vida adulta.

3 ORIENTAÇÕES METODOLÓGICAS

Enquanto o acolhimento for necessário, é fundamental ofertar à criança e ao adolescente um ambiente


e cuidados facilitadores do desenvolvimento, de modo a favorecer, dentre outros aspectos: i. Seu
desenvolvimento integral; ii. A superação de vivências de separação e violência; iii. A apropriação e
ressignificação de sua história de vida; e iv. O fortalecimento da cidadania, autonomia e a inserção social.
Em consonância com diretrizes nacionais e internacionais de cuidados a crianças e adolescentes em
serviços de acolhimento, as orientações abordadas neste capítulo visam contribuir para a melhoria dos
atendimentos prestados nestes serviços.

3.1 Estudo Diagnóstico

O estudo diagnóstico tem como objetivo subsidiar a decisão acerca do afastamento da criança ou
adolescente do convívio familiar. Salvo em situações de caráter emergencial e/ou de urgência, esta
medida deve ser aplicada por autoridade competente (Conselho Tutelar ou Justiça da Infância e da
Juventude), com base em uma recomendação técnica, a partir de um estudo diagnóstico, caso a caso,
realizado por equipe interprofissional do órgão aplicador da medida ou por equipe formalmente designada
para este fim. Em todos os casos, a realização deste estudo diagnóstico deve ser realizada sob
supervisão e estreita articulação com Conselho Tutelar, Justiça da Infância e da Juventude e equipe de
referência do órgão gestor da Assistência Social. Sempre que necessário, o órgão aplicador da medida
poderá requisitar, ainda, avaliação da situação por parte de outros serviços da rede como, por exemplo,
da Delegacia de Proteção da Criança e do Adolescente e de serviços de saúde.
Os fluxos e responsabilidades referentes à realização do estudo diagnóstico deverão ser definidos a
partir de acordos formais firmados entre os órgãos envolvidos, considerando a realidade, os recursos
existentes e o respeito às competências legais de cada órgão da rede de atendimento e do Sistema de
Garantia de Direitos.
O estudo diagnóstico deve incluir uma criteriosa avaliação dos riscos a que estão submetidos à criança
ou o adolescente e as condições da família para superação das violações de direitos observadas e o
provimento de proteção e cuidados. Com a devida fundamentação teórica, o estudo deve levar em conta
a proteção e a segurança imediata da criança e do adolescente, bem como seu cuidado e
desenvolvimento em longo prazo.
Sem a pretensão de abarcar todas as questões relevantes que deverão ser levantadas em um
diagnóstico, sugere-se que o mesmo possibilite identificar: composição familiar, história e dinâmica de
relacionamento entre seus membros; valores e crenças da família; demandas e estratégias desenvolvidas
para o enfrentamento de situações adversas; e situações de vulnerabilidade e risco às quais estão
expostos os integrantes do grupo familiar. Nessa perspectiva, recomenda-se que o estudo diagnóstico
contemple, dentre outros, os seguintes aspectos:
Composição familiar e contexto socioeconômico e cultural no qual a família está inserida;
mapeamento dos vínculos significativos na família extensa e análise da rede social de apoio da criança
ou adolescente e de sua família (família extensa, amigos, vizinhos, padrinhos, instituições, etc.); valores

Apostila gerada especialmente para: Betânia Mueller 023.712.580-32


. 87
e costumes da comunidade da qual a família faça parte, especialmente, no caso de minorias étnicas ou
comunidades tradicionais; condições de acesso da família a serviços, programas e projetos das diversas
políticas públicas que possam responder às suas necessidades; situações de vulnerabilidade e risco
vivenciadas pela família que repercutam sobre sua capacidade de prover cuidados; situação atual da
criança ou adolescente e de sua família, inclusive motivação, potencial e dificuldades da família para
exercer seu papel de cuidado e proteção; história familiar e se há padrões transgeracionais de
relacionamento com violação de direitos; situações atuais e pregressas de violência intrafamiliar contra a
criança e o adolescente, gravidade e postura de cada membro da família em relação à mesma; nos casos
de violência intrafamiliar, se há consciência da inadequação e das consequências negativas destas
práticas para a criança e o adolescente e se há movimento em direção à mudança e à construção de
novas possibilidades de relacionamento; análise da intensidade e qualidade dos vínculos entre os
membros da família (vinculação afetiva, interação, interesse e participação na vida da criança e do
adolescente); percepção da criança ou adolescente em relação à possibilidade de afastamento do
convívio familiar – se demonstra, por exemplo, medo de permanecer na família ou tristeza por afastar-se
da mesma; possibilidade de intervenção profissional e encaminhamentos que visem à superação da
situação de violação de direitos, sem a necessidade de afastamento da criança e do adolescente da
família; nos casos de violência intrafamiliar, se há possibilidade de afastamento do agressor da moradia
comum para a manutenção da criança ou adolescente na moradia em condições de proteção e segurança
(Art. 130 do ECA); grau de risco e desproteção ao qual a criança ou adolescente estará exposto se não
for afastada do ambiente familiar; verificação da existência de pessoas significativas da comunidade para
a criança ou adolescente que possam vir a acolhê-los, de forma segura, no caso de necessidade de
afastamento da família de origem.
O processo de avaliação diagnóstica deve incluir uma escuta qualificada de todos os envolvidos:
integrantes da família, inclusive a criança e o adolescente, pessoas da comunidade com vínculos
significativos com a família, a criança e o adolescente e profissionais que estejam prestando-lhes
atendimento, dentre outros.
Além da avaliação dos riscos aos quais porventura a criança ou adolescente estejam expostos no
ambiente familiar, antes de considerar o encaminhamento para serviço de acolhimento como uma
alternativa para garantir sua proteção, é preciso observar se na família extensa ou comunidade há
pessoas significativas que possam e aceitem se responsabilizar por seus cuidados.
No estudo diagnóstico deve-se buscar identificar, ainda, se a situação de risco à qual está exposta a
criança ou adolescente decorre exclusivamente do contexto social, histórico e econômico de vida da
família e se a garantia de apoio, orientação e acesso às diversas políticas públicas seria suficiente para
reduzir os riscos e possibilitar a manutenção do convívio familiar.
Além de avaliar se há necessidade ou não de afastamento do convívio familiar, o estudo diagnóstico
deve analisar o perfil e as demandas específicas da criança ou adolescente, de forma a subsidiar a
decisão pelo encaminhamento para o serviço de acolhimento que melhor atenda às suas peculiaridades.
Para a identificação do serviço mais adequado, deve-se considerar, ainda, sua estrutura física, recursos
humanos e Projeto Político-Pedagógico, além da possibilidade de manutenção de vínculos comunitários
da criança ou adolescente e a continuidade da frequência à mesma escola e aos equipamentos
comunitários aos quais estejam vinculados. Salvo nos casos em que o afastamento de sua comunidade
de origem for essencial para garantir-lhes segurança, deve-se evitar que a inclusão em um serviço de
acolhimento resulte no rompimento ou na fragilização dos vínculos comunitários e de pertencimento
preexistentes.

3.2 Plano de Atendimento Individual e Familiar

Assim que a criança ou adolescente chegar ao serviço de acolhimento, a equipe técnica do serviço,
que, onde houver, poderá contar com a contribuição da equipe responsável pela supervisão dos serviços
de acolhimento (ligada ao órgão gestor da Assistência Social) para elaborar um Plano de Atendimento
Individual e Familiar, no qual constem objetivos, estratégias e ações a serem desenvolvidos tendo em
vista a superação dos motivos que levaram ao afastamento do convívio e o atendimento das
necessidades específicas de cada situação. A elaboração deste Plano de Atendimento deve ser realizada
em parceria com o Conselho Tutelar e, sempre que possível, com a equipe interprofissional da Justiça da
Infância e da Juventude. Tal Plano deverá partir das situações identificadas no estudo diagnóstico inicial
que embasou o afastamento do convívio familiar.
Quando o acolhimento tiver sido realizado em caráter emergencial e/ou de urgência, sem estudo
diagnóstico prévio, recomenda-se que este estudo seja realizado em até vinte dias após o acolhimento,
a fim de avaliar a real necessidade da medida ou a possibilidade imediata de retorno da criança ou

Apostila gerada especialmente para: Betânia Mueller 023.712.580-32


. 88
adolescente ao convívio familiar. Se o acolhimento emergencial tiver sido realizado sem prévia
determinação da autoridade competente, esta deverá ser comunicada até o 2º dia útil imediato, conforme
o Art. 93 do ECA.
No caso de crianças e adolescentes acolhidos sem referência familiar, o fato deve ser comunicado à
Delegacia de Proteção à Criança e ao Adolescente ou, quando não houver, à delegacia mais próxima. O
serviço de acolhimento, em parceria com a referida delegacia, deve consultar o Cadastro Nacional de
Crianças e Adolescentes Desaparecidos e outros Cadastros similares existentes nos Estados, a fim de
verificar se não se trata de criança ou adolescente desaparecido. Nestes casos, tendo em vista a
localização de pais ou responsáveis, deve-se, ainda, por meio de métodos adequados, buscar
informações com a própria criança ou adolescente como, por exemplo: seu nome completo, idade, nome
de pessoas da família, escola onde estudava, bairro ou pontos de referência de sua moradia, município
de procedência, etc. Recomenda-se que estas informações sejam levantadas de modo gradativo no
período inicial do acolhimento, pois, dependendo da idade, a criança pode não se lembrar de tais
informações após certo período.
Os cadastros de crianças e adolescentes desaparecidos devem também ser consultados no caso de
crianças e adolescentes acolhidos que estejam em processo de saída da situação de rua. Nestas
situações deve-se buscar analisar junto à criança ou ao adolescente sua situação familiar, os motivos que
levaram à saída de casa e o desejo de retomada do contato com familiares, sobretudo com aqueles que
a estiverem procurando.
A situação de todas aquelas crianças e adolescentes já acolhidos deve também ser revista, de modo
a garantir que todos estejam em acompanhamento. Para estas situações deve-se também, na elaboração
do Plano de Atendimento considerar os motivos do afastamento e as intervenções realizadas até o
momento, a partir dos quais devem ser delineadas outras intervenções necessárias tendo em vista o
alcance de soluções de caráter mais definitivo para a criança e o adolescente.
O Plano de Atendimento tem como objetivo orientar o trabalho de intervenção durante o período de
acolhimento, visando à superação das situações que ensejaram a aplicação da medida. Deve basear-se
em um levantamento das particularidades, potencialidades e necessidades específicas de cada caso e
delinear estratégias para o seu atendimento. Tal levantamento constitui um estudo da situação que deve
contemplar, dentre outros aspectos:
Motivos que levaram ao acolhimento e se já esteve acolhido neste ou em outro serviço anteriormente,
dentre outros;
Configuração e dinâmica familiar, relacionamentos afetivos na família nuclear e extensa, período do
ciclo de vida familiar, dificuldades e potencialidades da família no exercício de seu papel.
Condições socioeconômicas, acesso a recursos, informações e serviços das diversas políticas
públicas;
Demandas específicas da criança, do adolescente e de sua família que requeiram encaminhamentos
imediatos para a rede (sofrimento psíquico, abuso ou dependência de álcool e outras drogas, etc.), bem
como potencialidades que possam ser estimuladas e desenvolvidas;
Rede de relacionamentos sociais e vínculos institucionais da criança, do adolescente e da família,
composta por pessoas significativas na comunidade, colegas, grupos de pertencimento, atividades
coletivas que frequentam na comunidade, escola, instituições religiosas, etc.;
Violência e outras formas de violação de direitos na família, seus significados e possível
transgeracionalidade;
Significado do afastamento do convívio e do serviço de acolhimento para a criança, o adolescente e
a família;
A partir deste levantamento inicial devem ser definidas estratégias de atuação que contribuam para a
superação dos motivos que levaram ao acolhimento. Tais estratégias devem primar pelo fortalecimento
dos recursos e das potencialidades da família (nuclear ou extensa), da criança, do adolescente, da
comunidade e da rede local, a fim de possibilitar o desenvolvimento de um trabalho que possa conduzir
a soluções de caráter mais definitivo, como a reintegração familiar, a colocação sob cuidados de pessoa
significativa da comunidade ou, quando essa se mostrar a alternativa que melhor atenda ao superior
interesse da criança e do adolescente, o encaminhamento para adoção. Quando se tratar de adolescente
com idade próxima à maioridade com remotas perspectivas de colocação em família substituta devem
ser viabilizadas ações destinadas à preparação para a vida autônoma.
O Plano de Atendimento Individual e Familiar deve orientar as intervenções a serem desenvolvidas
para o acompanhamento de cada caso, devendo contemplar, dentre outras, estratégias para:
- desenvolvimento saudável da criança e do adolescente durante o período de acolhimento:
encaminhamentos necessários para serviços da rede (saúde, educação, assistência social, esporte,
cultura e outros); atividades para o desenvolvimento da autonomia; acompanhamento da situação

Apostila gerada especialmente para: Betânia Mueller 023.712.580-32


. 89
escolar; preservação e fortalecimento da convivência comunitária e das redes sociais de apoio;
construção de projetos de vida; relacionamentos e interação no serviço de acolhimento –
educadores/cuidadores, demais profissionais e colegas; preparação para ingresso no mundo do trabalho,
etc; investimento nas possibilidades de reintegração familiar: fortalecimento dos vínculos familiares e
das redes sociais de apoio; acompanhamento da família, em parceria com a rede, visando à superação
dos motivos que levaram ao acolhimento; potencialização de sua capacidade para o desempenho do
papel de cuidado e proteção; gradativa participação nas atividades que envolvam a criança e o
adolescente; etc. Nos casos de crianças e adolescentes em processo de saída da rua deve-se, ainda,
buscar a identificação dos familiares, dos motivos que conduziram à situação de rua e se há motivação e
possibilidades para a retomada da convivência familiar; acesso da família, da criança ou adolescente a
serviços, programas e ações das diversas políticas públicas e do terceiro setor que contribuam para o
alcance de condições favoráveis ao retorno ao convívio familiar;
- investimento nos vínculos afetivos com a família extensa e de pessoas significativas da comunidade:
fortalecimento das vinculações afetivas e do papel na vida da criança e do adolescente; apoio aos
cuidados com a criança ou adolescente no caso de reintegração familiar ou até mesmo
- responsabilização por seu acolhimento;
- encaminhamento para adoção quando esgotadas as possibilidades de retorno ao convívio familiar:
articulação com o Poder Judiciário e o Ministério Público para viabilizar, nestes casos, o cadastramento
para adoção. Desde que haja supervisão do Poder Judiciário, uma estratégia que pode ser empreendida
também pelos serviços de acolhimento, em parceria com Grupos de Apoio à Adoção ou similares, diz
respeito à busca ativa de famílias para a adoção de crianças e adolescentes com perfil de difícil colocação
familiar.
A elaboração do Plano de Atendimento Individual e Familiar deve envolver uma escuta qualificada da
criança, do adolescente e de sua família, bem como de pessoas que lhes sejam significativas em seu
convívio, de modo a compreender a dinâmica familiar e as relações estabelecidas com o contexto. É
necessário que a criança, o adolescente e as famílias tenham papel ativo nesse processo e possam, junto
aos técnicos e demais integrantes da rede, pensar nos caminhos possíveis para a superação das
situações de risco e de violação de direitos, participando da definição dos encaminhamentos,
intervenções e procedimentos que possam contribuir para o atendimento de suas demandas. Também
devem ser ouvidos outros profissionais que porventura estejam atendendo ou tenham atendido a criança,
o adolescente ou a família, como nos casos de acompanhamento por equipes de saúde mental, de outros
serviços da rede socioassistencial e da escola, dentre outros.
Os Planos de Atendimento Individual e Familiar deverão ser encaminhados para conhecimento do
Sistema de Justiça e do Conselho Tutelar, em prazo previamente acordado. Tais órgãos devem
acompanhar as intervenções realizadas com a família, sendo acionados quando necessária a aplicação
de outras medidas protetivas para assegurar o acesso da criança, do adolescente ou da família aos
serviços disponíveis na rede (ECA, Art. 101, Inciso I a VI).
O desenvolvimento das ações do Plano de Atendimento deve ser realizado de modo articulado com
os demais órgãos e serviços que estejam acompanhando a família, a criança ou o adolescente (escola,
Unidade Básica de Saúde, Estratégia de Saúde da Família, CAPS, CREAS, CRAS, programas de
geração de trabalho e renda, etc.), a fim de que o trabalho conduza, no menor tempo necessário, a uma
resposta definitiva para a criança e o adolescente, que não seja re-vitimizadora ou precipitada. Para tanto,
deverão ser realizadas reuniões periódicas para estudo de cada caso pelos profissionais envolvidos, para
acompanhamento da evolução do atendimento, verificação do alcance dos objetivos acordados, avaliação
da necessidade de revisão do Plano de Atendimento e elaboração de estratégias de ação que possam
responder às novas situações surgidas durante o atendimento. As conclusões resultantes de tais
reuniões servirão, inclusive, de subsídio para a elaboração de relatórios semestrais a serem enviados à
autoridade judiciária e ao Ministério Público.
Os serviços de acolhimento devem construir uma sistemática de atendimento que possibilite o início
da elaboração do Plano de Atendimento Individual e Familiar imediatamente após o acolhimento da
criança ou adolescente, para que se alcance, no menor tempo necessário, soluções de caráter mais
definitivo. Cabe ressaltar, finalmente, que a ênfase do Plano de Atendimento deve ser na construção de
estratégias para o atendimento, de modo a não transformá-lo em mera formalidade. Um registro sintético
do Plano de Atendimento não deve, ainda, significar sua limitação às estratégias inicialmente elaboradas,
devendo-se garantir que seja sempre dinâmico e aberto a mudanças, reformulações e aprimoramento,
baseado nas intervenções realizadas e em seus resultados.

Apostila gerada especialmente para: Betânia Mueller 023.712.580-32


. 90
3.3 Acompanhamento da Família de Origem

A implementação de uma sistemática de acompanhamento da situação familiar, iniciada


imediatamente após o acolhimento, é fundamental, pois, com o passar do tempo, tanto as possibilidades
de reintegração familiar, quanto de adoção podem tornar-se mais difíceis. O prolongamento do
afastamento da criança ou adolescente pode provocar enfraquecimento dos vínculos com a família, perda
de referências do contexto e de valores familiares e comunitários, exigindo preparação ainda mais
cuidadosa no caso de reintegração familiar. Do mesmo modo, o prolongamento desnecessário do
encaminhamento para adoção, nas situações em que esta constitui a melhor medida para a criança ou
adolescente, pode reduzir consideravelmente as possibilidades de colocação familiar, em virtude das
dificuldades observadas no contexto brasileiro para a realização de adoções de crianças maiores e de
adolescentes.
Dar início ao acompanhamento da situação familiar imediatamente após a chegada da criança ou
adolescente é importante, ainda, para que a equipe técnica possa, no menor tempo possível, fazer sua
análise quanto a real necessidade do acolhimento. Caso conclua que a manutenção do afastamento da
criança ou adolescente do convívio familiar não é necessária, a equipe técnica responsável pelo
acompanhamento deve proceder aos encaminhamentos para viabilizar a imediata reintegração. Nestas
situações a família também deverá ser informada do seu direito a questionar o afastamento e requerer,
junto à Justiça, por intermédio de advogado nomeado ou Defensor Público, a reintegração da criança ou
adolescente (ECA, Art. 141).
Nos serviços onde porventura haja crianças e adolescentes já acolhidos cuja situação familiar não
esteja sendo acompanhada, a equipe técnica deve também iniciar estes acompanhamentos, buscando
soluções que contribuam para assegurar a excepcionalidade e a provisoriedade do acolhimento. O
acompanhamento da situação familiar, nestas situações, é fundamental para se identificar:
- crianças e adolescentes que permanecem acolhidos unicamente pela situação de pobreza de suas
famílias: situação que exige o investimento imediato em estratégias para seu retorno ao convívio familiar;
- crianças e adolescente, ou membros da família, com direito, mas sem acesso ao BPC. Nestes casos
deve-se proceder ao encaminhamento para o CRAS - ou órgão gestor responsável - para seu
cadastramento no Benefício, devendo tal medida ser acompanhada do investimento nas possibilidades
de reintegração;
- crianças e adolescentes sem possibilidades de reintegração ao convívio familiar (família nuclear ou
extensa): situação que exige imediatos encaminhamentos para o Sistema de Justiça, para o
cadastramento para adoção e a Destituição do Poder Familiar.
A intervenção profissional na etapa inicial do acompanhamento deve proporcionar, de modo
construtivo, a conscientização por parte da família de origem dos motivos que levaram ao afastamento
da criança e/ou do adolescente e das consequências que podem advir do fato. Esta conscientização é
fundamental para que as próximas etapas possam ser planejadas, com acordos firmados entre serviço e
família, com vistas ao desenvolvimento de ações proativas que contribuam para a superação de situações
adversas ou padrões violadores que possam ter levado ao afastamento. A equipe técnica do serviço de
acolhimento deve, ainda, acompanhar o trabalho desenvolvido com a família na rede local, mantendo-a
informada, inclusive, a respeito de possíveis decisões por parte da Justiça.
Devem ser firmados acordos entre o serviço de acolhimento, a equipe de supervisão e apoio aos
serviços de acolhimento - ligada ao órgão gestor da Assistência Social - a equipe técnica do Poder
Judiciário e os demais serviços da rede das diversas políticas públicas, incluindo os não-governamentais,
a fim de promover a articulação das ações de acompanhamento à família, além de reuniões periódicas
para discussão e acompanhamento dos casos.
Durante o período de acolhimento, o serviço deverá encaminhar relatórios para a Justiça da Infância
e da Juventude com periodicidade mínima semestral, de modo a subsidiar o acompanhamento da
situação jurídico-familiar de cada criança/adolescente e a avaliação por parte da Justiça da possibilidade
de reintegração familiar ou necessidade de encaminhamento para família substituta, sobretudo nos casos
em que o prognóstico de permanência da criança e do adolescente no serviço de acolhimento for de mais
de dois anos.
No trabalho com a família devem ser consideradas tanto as questões objetivas, quanto subjetivas.
Considerar essas duas dimensões, objetiva e subjetiva, é importante para apoiar a família no processo
de reintegração e prevenir novos afastamentos. Ressalte-se que lidar com questões objetivas é
fundamental para prevenir situações que possam gerar uma tensão excessiva na família, em função do
processo de exclusão social ou das dificuldades reais para conciliar o cuidado com a criança e o
adolescente e a sobrevivência do núcleo familiar. Tais situações podem dificultar, inclusive, o

Apostila gerada especialmente para: Betânia Mueller 023.712.580-32


. 91
desenvolvimento de relações afetivas que contribuam para o exercício de seu papel de proteção e
cuidados.

Diversas técnicas podem ser utilizadas no acompanhamento às famílias, como, por exemplo:
Estudo de caso: reflexão coletiva que deve partir das informações disponíveis sobre a família e incluir
resultados das intervenções realizadas. Na medida do possível deve ser realizado com a participação dos
profissionais do serviço de acolhimento, da equipe de supervisão do órgão gestor, da Justiça da Infância
e da Juventude e de outros serviços da rede que acompanhem a família;
Entrevista individual e familiar: estratégia importante, particularmente nos primeiros contatos com a
família e seus membros, que permite avaliar a expectativa da família quanto à reintegração familiar e
elaborar conjuntamente o Plano de Atendimento. Esse instrumento também pode ser utilizado para
abordar outras questões específicas, para aprofundar o conhecimento sobre a família e para fortalecer a
relação de confiança com o serviço. Nas entrevistas podem ser realizados, ainda, o genograma, o mapa
de rede social, dentre outras técnicas.
Grupo com famílias: dentre outros aspectos, favorece a comunicação com a família, a troca de
experiências entre famílias e a aprendizagem e o apoio mútuos. Possibilita a reflexão sobre as relações
familiares e responsabilidades da família na garantia dos direitos de seus membros e sobre os aspectos
concernentes ao acolhimento. Constitui importante estratégia para potencialização dos recursos da
família para o engajamento nas ações necessárias para retomada do convívio familiar com a criança ou
adolescente; Grupo Multifamiliar: espaço importante para trocas de experiências, reflexões e discussão
com as famílias, incluindo a participação de crianças e adolescentes acolhidos. O Grupo Multifamiliar
permite a compreensão de diferentes pontos de vista dos relacionamento familiares e das diferenças
entre gerações.
Visita Domiciliar: importante recurso para conhecer o contexto e a dinâmica familiar e identificar
demandas, necessidades, vulnerabilidades e riscos. Referenciada no princípio do respeito à privacidade,
a visita possibilita uma aproximação com a família e a construção de um vínculo de confiança, necessário
para o desenvolvimento do trabalho.
Orientação individual, grupal e familiar: intervenções que têm como objetivo informar, esclarecer e
orientar pais e responsáveis sobre diversos aspectos, como a medida de proteção aplicada e os
procedimentos dela decorrentes. Deve pautar-se em uma metodologia participativa que possibilite a
participação ativa da família;
Encaminhamento e acompanhamento de integrantes da família à rede local, de acordo com demandas
identificadas: psicoterapia, tratamento de uso, abuso ou dependência de álcool e outras drogas, outros
tratamentos na área de saúde, geração de trabalho e renda, educação de jovens e adultos, etc.
Como resultado dessas atividades, de forma geral o acompanhamento familiar deve contribuir para:
- A acolhida da família, a compreensão de sua dinâmica de funcionamento, valores e cultura;
A conscientização por parte da família de sua importância para a criança e o adolescente e das
decisões definitivas que podem vir a ser tomadas por parte da Justiça, baseadas no fato da criança e do
adolescente serem destinatários de direitos;
- A compreensão das estratégias de sobrevivência adotadas pela família e das dificuldades
encontradas para prestar cuidados à criança e ao adolescente e para ter acesso às políticas públicas;
- A reflexão por parte da família acerca de suas responsabilidades, de sua dinâmica de relacionamento
intrafamiliar e de padrões de relacionamentos que violem direitos;
- O desenvolvimento de novas estratégias para a resolução de conflitos;
- O fortalecimento da autoestima e das competências da família, de modo a estimular sua resiliência,
ou seja, o aprendizado com a experiência e a possibilidade de superação dos desafios;
- O fortalecimento da autonomia, tanto do ponto de vista socioeconômico, quanto do ponto de vista
emocional, para a construção de possibilidades que viabilizem a retomada do convívio com a criança e o
adolescente.
- O fortalecimento das redes sociais de apoio da família;
- O fortalecimento das alternativas para gerar renda e para garantir a sobrevivência da família.

Além das questões da metodologia do trabalho, as crenças dos profissionais acerca das famílias e o
modo como se relacionam com as mesmas, também influenciam os resultados das intervenções. Assim,
é preciso estar atento à:
Postura de respeito à diversidade, aos diferentes arranjos familiares e às mais distintas estratégias às
quais as famílias podem recorrer para lidar com situações adversas. Diversas experiências têm
demonstrado que o trabalho bem-sucedido de reintegração familiar está fortemente associado à
possibilidade de construção de um vínculo de referência significativo da família com profissionais que a

Apostila gerada especialmente para: Betânia Mueller 023.712.580-32


. 92
acompanhem, aos quais possa recorrer, inclusive, em momentos de crise no período pós-reintegração.
O fortalecimento dos recursos da família para cuidar da criança e do adolescente está fortemente
associado às possibilidades de sentir-se também acolhida e cuidada.
A crença por parte dos profissionais nas possibilidades de reconstrução das relações, também
representa aspecto que pode contribuir para o fortalecimento da confiança da família em sua capacidade
de se responsabilizar novamente pelos cuidados à criança e ao adolescente e superar os motivos que
conduziram ao afastamento.
O acompanhamento da família deve ser sistemático para que, em um prazo de até dois anos, seja
possível viabilizar o retorno da criança ou adolescente ao convívio com sua família ou, na sua
impossibilidade, o encaminhamento para família substituta. A questão do tempo de permanência no
serviço de acolhimento deve ser um dos eixos principais da avaliação acerca da medida que melhor
atende ao interesse da criança e do adolescente. Nesse sentido, há que se considerar os prejuízos ao
desenvolvimento da criança e do adolescente que possam advir tanto da permanência prolongada quanto
de um rompimento definitivo dos vínculos com a família de origem. A questão do tempo deve levar em
consideração, sobretudo, a idade da criança e do adolescente e o significado da privação do convívio
familiar na etapa do ciclo de vida na qual se encontram.
Embora o tempo deva ser um eixo central, as decisões não podem ser prematuras ou tomadas de
modo precipitado. Quando isso ocorre sem uma preparação adequada da criança, do adolescente e da
família, a reintegração familiar pode ser conflituosa e acabar resultando em um novo afastamento do lar
de origem. Nos casos de colocação em família substituta (adoção, guarda ou tutela), a falta de uma
preparação adequada de todos os envolvidos pode conduzir também a situações que dificultem a
construção da vinculação afetiva, resultando em retorno ao serviço de acolhimento. É importante,
portanto, que um acompanhamento sistemático possibilite a inserção familiar no menor tempo necessário,
mas com preparação adequada de todos os envolvidos.
Quando o desenvolvimento do trabalho de acompanhamento familiar indicar a possibilidade de
reintegração da criança ou adolescente à sua família, nuclear ou extensa, a equipe técnica do serviço de
acolhimento, em parceria com a equipe responsável pela supervisão dos serviços de acolhimento (ligada
ao órgão gestor da Assistência Social) e, sempre que possível, a equipe interprofissional da Justiça da
Infância e da Juventude, deverá elaborar estratégias para a reintegração familiar que contemplem os
encaminhamentos necessários para viabilizar o retorno seguro da criança ou adolescente à sua família e
o fortalecimento desta para o exercício de seu papel de cuidado e proteção.
Para decidir se a reintegração é a melhor medida também devem ser ouvidos, a criança e ao
adolescente, por meios adequados, bem como suas famílias e os educadores/cuidadores ou famílias
acolhedoras. Para o trabalho com a reintegração familiar e decisão acerca da melhor medida para a
criança ou adolescente, é importante considerar, ainda, dentre outros, os seguintes aspectos:
- as reações da criança, do adolescente e da família ao afastamento e ao acolhimento no serviço;
- a vinculação afetiva e a motivação mútua da família, da criança e do adolescente pela continuidade
da relação afetiva e retomada do convívio;
- se os encaminhamentos realizados foram viabilizados e qual tem sido a resposta da família, da
criança e do adolescente aos mesmos;
- se há, por parte da família, conscientização dos motivos que levaram ao afastamento da criança ou
adolescente e motivação para superá-los;
- se há movimento de mudança nos padrões de relacionamento entre os membros da família e desta
com a comunidade e instituições sociais;
- se existem membros da família (nuclear ou extensa) que possam se responsabilizar e compartilhar
os cuidados com a criança e o adolescente;
- se a família possui redes sociais de apoio da família: vinculações significativas da família com
pessoas da comunidade e serviços que possam apoiar os cuidados à criança e ao adolescente.
Quando a reintegração familiar for considerada a melhor medida, a preparação para o retorno deverá
incluir uma crescente participação da família na vida da criança e do adolescente, inclusive no
cumprimento das responsabilidades parentais. Nesse sentido, deve ser propiciada a inserção da família
em atividades que envolvam a criança e o adolescente como, por exemplo, reuniões escolares, consultas
de saúde, comemoração do aniversário, atividades na comunidade, escola, etc. Do mesmo modo, a
criança e o adolescente devem ter a oportunidade de uma reinserção gradual no contexto de origem,
passando finais de semana ou datas comemorativas na casa da família, por exemplo. Nesse momento é
importante, ainda, que sejam fortalecidas as redes sociais de apoio da família, fundamentais para o
exercício de seu papel de cuidadora.
É importante destacar que a reintegração com familiares com os quais a criança e o adolescente não
possuam vínculo afetivo deve ser cuidadosamente avaliada, não devendo ser conduzida meramente com

Apostila gerada especialmente para: Betânia Mueller 023.712.580-32


. 93
base em uma “supervalorização dos laços consanguíneos”. Nesses casos, deve-se avaliar a possibilidade
de construção de vinculações significativas e de aceitação mútua do convívio, para se decidir quanto à
melhor alternativa a ser recomendada à autoridade judiciária: reintegração com família extensa,
colocação com pessoa significativa da comunidade ou adoção. Quando a colocação com familiar sem
vinculação prévia representar a melhor medida, deve-se preparar previamente todos os envolvidos, por
meio de uma gradativa aproximação que possibilite a construção da vinculação afetiva, fundamental para
prevenir situações futuras de rejeição e, até mesmo, de retorno da criança ou adolescente ao serviço de
acolhimento.
Após a reintegração familiar é importante que o período de adaptação mútua entre criança/adolescente
e família seja acompanhado por pelo menos seis meses, após os quais deverá avaliar-se a necessidade
de sua continuidade. Nesse momento, devido às mudanças ocorridas em cada um dos membros durante
o período do afastamento e o descompasso entre expectativas construídas e realidade, podem ocorrer
entre os integrantes da família insegurança e conflitos na relação devido à necessidade de readaptação
da rotina e regas familiares. Por isso é preciso que sejam desenvolvidas estratégias para conciliar os
cuidados com a criança e o adolescente com as demais responsabilidades da família.
O acompanhamento psicossocial nesse momento é fundamental para auxiliar a família, a criança e o
adolescente a construírem novas possibilidades para estarem juntos, apesar da separação vivida. O apoio
profissional será fundamental, ainda, para que a família se aproprie de novos padrões de relacionamento
mais saudáveis e favorecedores do desenvolvimento. A definição quanto ao órgão responsável pelo
acompanhamento no período após a reintegração familiar deverá ser objeto de acordo formal entre os
serviços de acolhimento, o órgão gestor da Assistência Social e a Justiça da Infância e da Juventude. Tal
definição deve levar em consideração a estrutura e a capacidade técnica dos serviços da rede local,
podendo ser designada para esse fim a equipe técnica dos serviços de acolhimento, a equipe responsável
pela supervisão dos serviços de acolhimento, o CREAS, ou até mesmo o CRAS ou outro serviço de
atendimento sociofamiliar existente no Município.
Nos casos em que forem esgotadas as possibilidades de reintegração familiar (família nuclear ou
extensa), a equipe técnica do serviço de acolhimento deverá elaborar e enviar à autoridade judiciária
relatório circunstanciado onde sejam relatadas a situação familiar da criança ou adolescente, as
intervenções realizadas com vistas à reintegração familiar e os resultados obtidos, sugerindo a Destituição
do Poder Familiar e a inserção da criança ou adolescente no cadastro para adoção.
Nos casos de encaminhamento para adoção, deve ser realizado um planejamento por parte da equipe
do serviço de acolhimento, da Justiça da Infância e da Juventude e, onde houver, do Grupo de Apoio à
Adoção, com vistas à preparação prévia de todos os envolvidos e a aproximação gradativa dos adotantes
e da criança/adolescente. Além da preparação dos adotantes e da criança/adolescente, o
educador/cuidador ou a família acolhedora deverá também ser incluído no processo, sendo, inclusive,
orientado quanto à preparação da criança/adolescente para a adoção.
Particularmente em relação ao encaminhamento para família substituta, constitui papel, ainda, a ser
desenvolvido em parceria entre o serviço de acolhimento, Poder Judiciário e Grupos de Apoio à Adoção,
a busca ativa de famílias para aquelas crianças e adolescentes com perfil de difícil colocação em adoção.
É de fundamental importância que o órgão gestor da Política de Assistência Social, o Poder Judiciário
e o Ministério Público estabeleçam, de forma pactuada com os demais operadores da rede de acolhida,
fluxos, prazos e procedimentos que viabilizem no menor tempo possível, tão logo haja recomendação
técnica, a reintegração familiar ou, na sua impossibilidade, o encaminhamento para família substituta.
É importante que sejam implementadas as medidas necessárias em cada localidade para o
fortalecimento das condições para o acompanhamento sistemático da situação familiar, de modo a
prevenir que a fragilidade na rede local conduza à frequente insegurança por parte dos profissionais para
tomar decisões definitivas sobre o futuro da criança e do adolescente. Devem-se criar condições e
conscientização por parte dos envolvidos acerca da importância destas decisões delicadas, mas
necessárias. A criança e o adolescente não podem permanecer por tempo indefinido no serviço de
acolhimento em razão da dificuldade de se construir condições locais favoráveis para essa tomada de
decisão.

3.4 Articulação Intersetorial

Os Serviços de Acolhimento integram o Sistema Único de Assistência Social (SUAS), tendo interface
com outros serviços da rede socioassistencial, quanto com demais órgãos do Sistema de Garantia de
Direitos. Sua atuação deve basear-se no princípio da incompletude institucional, não devendo ofertar em
seu interior atividades que sejam da competência de outros serviços. A proteção integral a que têm direito

Apostila gerada especialmente para: Betânia Mueller 023.712.580-32


. 94
as crianças e os adolescentes acolhidos deve ser viabilizada por meio da utilização de equipamentos
comunitários e da rede de serviços local.
Dessa forma, para que as intervenções realizadas junto às crianças e aos adolescentes acolhidos e
suas famílias sejam efetivas, é necessário que haja uma estreita articulação entre os diversos órgãos
envolvidos no seu atendimento. Assim, para fortalecer a complementaridade das ações e evitar
sobreposições, é importante que esta articulação proporcione o planejamento e o desenvolvimento
conjunto de estratégias de intervenção, sendo definido o papel de cada instância que compõe a rede de
serviços local e o Sistema de Garantia de Direitos, na busca de um objetivo comum.

3.4.1 Articulação no âmbito do Sistema Único de Assistência Social - SUAS

Para a garantia de um atendimento de qualidade às crianças e aos adolescentes acolhidos e às suas


famílias, os serviços de acolhimento devem funcionar de forma articulada com os demais serviços da
rede socioassistencial local. Tal articulação possibilitará a inserção dos usuários nos demais serviços,
programas e ações que integram o SUAS, que se fizerem necessários ao atendimento às demandas
específicas de cada caso, favorecendo a integração comunitária e social dos usuários.
Como exemplo, pode-se citar a inserção de crianças e adolescentes acolhidos – ou reintegrados ao
convívio familiar - em atividades socioeducativas e de convivência ofertadas pela rede de proteção social
básica, que pode ser-lhes particularmente benéfica. Nessa direção, cabe destacar também que os
adolescentes em serviços de acolhimento ou egressos destes serviços constituem um dos públicos
prioritários de diversos programas como, por exemplo, o Projovem Adolescente. O encaminhamento para
este programa pode, inclusive, favorecer o processo de fortalecimento da autonomia, socialização e
preparação dos adolescentes para o desligamento do serviço de acolhimento.
Da mesma forma, a participação em programas de inclusão produtiva, dos adolescentes maiores de
16 anos e dos familiares das crianças e adolescentes acolhidos ou reintegrados, pode contribuir
significativamente para o desenvolvimento da autonomia e acesso à renda. A inclusão das famílias em
atividades de apoio sociofamiliar pode contribuir tanto para a construção de condições favoráveis à
reintegração familiar quanto para o desenvolvimento de relações saudáveis no período pós-reintegração.
Ressalte-se, ainda, a necessidade de encaminhamento aos órgãos responsáveis pela viabilização da
inserção em programas de transferência de renda – como o PBF e o BPC - daqueles que preencherem
os critérios de elegibilidade de tais programas e benefícios.
Destaca-se a seguir, alguns dos principais equipamentos de referência do Sistema Único de
Assistência Social, bem como a equipe de supervisão e apoio aos serviços de acolhimento, ligada ao
órgão gestor:
CRAS: sempre que se identificar a necessidade de ações de proteção social básica para criança e
adolescente atendido em serviços de acolhimento ou para suas famílias, deverá ser articulada sua
inclusão em tais atividades por meio da equipe do CRAS do território de moradia da família. Para dar
agilidade a tais procedimentos, recomenda-se que sejam definidos, de forma conjunta, fluxos de
encaminhamento e canais de comunicação entre os serviços de acolhimento e o(s) CRAS, além de
encontros periódicos, que possibilitem o acompanhamento das ações.
O CRAS de referência do território de moradia da família, sempre que necessário, deverá ser acionado
para participar do processo de reintegração familiar de crianças e adolescentes atendidas em serviços de
acolhimento. Sua atuação se faz necessária para a inclusão da criança ou do adolescente que estiver
sendo reintegrado à família, e de seus familiares ou responsáveis, em serviços, programas e ações de
fortalecimento dos vínculos familiares e comunitários, bem como para fazer os encaminhamentos que se
mostrarem necessários com a retomada do convívio familiar, de modo a facilitar sua inclusão social e
comunitária nesse período de vulnerabilidade.
CREAS: Nos municípios que possuam CREAS e naqueles atendidos por CREAS regionais, quando
o motivo do afastamento do convívio familiar envolver violência intrafamiliar (física, psicológica, sexual,
negligência grave), exploração sexual ou outras situações de violação de direitos que estejam sob o
escopo de ação dos serviços desenvolvidos no CREAS, as crianças e adolescentes acolhidos e seus
familiares devem ser inseridos em seus serviços. Nesse caso, é de suma importância que as equipes
técnicas do serviço de acolhimento e do CREAS atuem de forma articulada – com planejamento conjunto
de estratégias de ação e reuniões periódicas para o acompanhamento dos casos – de modo a garantir
uma atuação complementar e sinérgica, evitando sobreposições e ações contraditórias.
Equipe de Supervisão e Apoio aos Serviços de Acolhimento: Em municípios de médio e grande porte
e nas metrópoles - e nos demais quando a demanda justificar - o órgão gestor da Assistência Social
deverá manter equipe profissional especializada de referência, para supervisão e apoio aos serviços de
Acolhimento. De acordo com a realidade e as definições locais, tal equipe poderá compor um serviço

Apostila gerada especialmente para: Betânia Mueller 023.712.580-32


. 95
especificamente voltado a esta função ou, ainda, estar vinculada ao CREAS ou diretamente ao órgão
gestor. Em todos os casos, terá como atribuições mínimas:
I. mapear a rede existente e fortalecer a articulação dos serviços de acolhimento com os demais
serviços da rede socioassistencial, das demais políticas públicas e do SGD;
II. monitorar as vagas na rede de acolhimento, indicando o serviço que melhor atenda às necessidades
específicas de cada caso encaminhado;
III. prestar supervisão e suporte técnico aos serviços de acolhimento;
IV. apoiar as equipes técnicas dos serviços de acolhimento no acompanhamento psicossocial das
famílias de origem das crianças e adolescentes acolhidos;
V. efetivar os encaminhamentos necessários, em articulação com os demais serviços da Rede
Socioassistencial, das demais Políticas Públicas e do SGD, monitorando, posteriormente, seus
desdobramentos;
VI. Monitorar a situação de todas as crianças e adolescentes que estejam em serviços de acolhimento
no município, e de suas famílias, organizando, inclusive, cadastro permanentemente atualizado contendo
o registro de todas as crianças e adolescentes atendidos nesses serviços;

3.4.2 Articulação com o Sistema Único de Saúde - SUS

O atendimento humanizado de crianças e adolescentes em serviços de acolhimento requer uma


estreita articulação entre o Sistema Único de Saúde - SUS e o Sistema Único de Assistência Social -
SUAS. Desse modo, orienta-se que os órgãos gestores dessas duas políticas desenvolvam estratégias
conjuntas e elaborem protocolos de atenção integral à saúde de crianças e adolescentes que se
encontram em Serviços de Acolhimento, bem como de seus familiares.
As ações de promoção da saúde, ações educativas e de prevenção de agravos devem ser articuladas
com a Rede de Atenção Básica/Primária, que é composta por Unidades Básica de Saúde da Família e
Postos de Saúde. Esses serviços também devem ser procurados caso haja necessidade de solicitar
requisição de exames, medicamentos básicos e acompanhamento do quadro de situação de saúde de
crianças e adolescentes em situação de acolhimento. Caso haja necessidade, a equipe desses serviços
fará também encaminhamento para unidades de atenção especializada - que inclui Hospitais,
Maternidades, Unidades de Urgências e/ou Emergências e Serviços/Unidade de Referências -
capacitados para atenderem casos que demandem atenção especializada, como adolescentes grávidas,
crianças e adolescentes com deficiência, com distúrbios de crescimento, com doenças infectocontagiosas
ou imunodepressoras, dentre outros.
Nos casos de crianças e adolescentes com transtornos mentais e/ou que apresentam problemas
devido ao uso abusivo ou dependência de álcool e outras drogas, deve ser acionada a rede de saúde
mental, por meio das ações de saúde mental na Atenção Básica, do Centro de Atenção Psicossocial
(CAPS) ou, onde houver, o Centro de Atenção Psicossocial Infanto-juvenil (CAPSi) , especializado no
atendimento de crianças e adolescentes com transtornos mentais graves (autismo, psicoses, neuroses
graves, abuso ou dependência de álcool e outras drogas).
Na articulação com o órgão gestor da saúde, devem ainda estar previstas, ainda, ações de capacitação
e acompanhamento dos educadores/cuidadores, além de profissionais dos serviços de acolhimento, bem
como das famílias acolhedoras, em relação a: cuidados diferenciados que crianças e adolescentes com
deficiência, transtorno mental ou outras necessidades específicas de saúde; amamentação, vacinação,
crescimento e desenvolvimento de crianças e adolescentes; saúde sexual e saúde reprodutiva e
orientação quanto aos direitos sexuais e direitos reprodutivos de adolescentes e prevenção do uso de
álcool e outras drogas.
Especial atenção deve ser dada no sentido de garantir à criança e ao adolescente com deficiência ou
necessidades específicas de saúde, acolhidos no serviço, reintegrados à família de origem ou
encaminhados à família substituta, o acesso a tratamentos, medicamentos, serviços especializados e
equipamentos de saúde, bem como o apoio necessário à família para o atendimento a suas necessidades
específicas.

3.4.3 Articulação com o Sistema Educacional

A articulação dos serviços de acolhimento com o sistema educacional é fundamental, pois a escola
constitui importante instrumento para assegurar o direito à convivência comunitária de crianças e
adolescentes. Essa articulação pode ser feita por meio da elaboração conjunta de protocolo de ação
entre o órgão gestor da assistência social e da educação, garantindo a permanente comunicação entre

Apostila gerada especialmente para: Betânia Mueller 023.712.580-32


. 96
os serviços, e o acesso das crianças, adolescentes acolhidos e seus familiares à rede de local de
Educação.
Assim, os serviços de acolhimento devem manter canais de comunicação permanentes com as
escolas onde estejam matriculadas as crianças e os adolescentes acolhidos, de modo a possibilitar o
acompanhamento de seu desempenho escolar. Sempre que possível e recomendável, deve-se favorecer,
ainda, o envolvimento da família de origem ou extensa no acompanhamento escolar das crianças e
adolescentes acolhidos, incentivando, inclusive, sua participação ativa nas reuniões e comemorações
escolares.
Sempre que possível, deve-se procurar manter a criança ou adolescente na mesma escola em que
estudava antes da aplicação da medida protetiva, de modo a evitar rompimentos desnecessários de
vínculos de amizade e de pertencimento e modificações radicais em sua rotina, além de prejuízos
acadêmicos. Constituem exceções a tal recomendação as situações com determinação judicial em
contrário ou recomendação técnica de mudança de escola por questões relativas à preservação da
segurança e proteção da criança ou adolescente.
É Importante, ainda, promover a inclusão de crianças e adolescentes que estão em serviços de
acolhimento nas atividades propostas pelo Programa Mais Educação, em drogas e todos aqueles que,
por sua condição psíquica, estão impossibilitados de manter ou estabelecer laços sociais. A experiência
acumulada em serviços que já funcionavam segundo a lógica da atenção diária indica que se ampliam as
possibilidades do tratamento para crianças e adolescentes quando o atendimento tem início o mais cedo
possível, devendo, portanto, os CAPSi estabelecerem as parcerias necessárias com a rede de saúde,
educação e assistência social ligadas ao cuidado da população infanto-juvenil.
A articulação com o sistema educacional permite, ainda, desenvolver ações de conscientização e
sensibilização de professores e demais profissionais da escola, de modo a que estes atuem como agentes
facilitadores da integração das crianças e adolescentes no ambiente escolar, evitando ou superando
possíveis situações de preconceito ou discriminação. Essas ações de capacitação podem ser fortalecidas
por meio do Projeto Escola que Protege, em cuja proposta de formação continuada de profissionais da
educação básica e da Rede de Proteção Integral, pode ser inserida a temática da criança e do
adolescente afastados do convívio familiar.
Articulação com outras políticas públicas e demais órgãos do Sistema de Além da articulação com os
serviços socioassistenciais, da saúde e da educação, é necessária a articulação com equipamentos
comunitários, organizações não-governamentais e serviços públicos responsáveis pela execução de
programas, projetos, serviços e ações nas áreas de cultura, esporte, lazer, geração de trabalho e renda,
habitação, transporte e capacitação profissional, garantindo o acesso de crianças e adolescentes
acolhidos e de suas famílias. Recomenda-se que, sempre que for viável, procure-se inserir as crianças e
adolescentes acolhidas em atividades localizadas nas proximidades de sua comunidade de origem, de
forma a fortalecer sua inserção comunitária, o que contribuirá para o processo de reintegração familiar ao
evitar a futura interrupção de suas atividades e dos vínculos de amizade construídos nesses espaços.
Finalmente, destaca-se a importância da articulação e da construção de fluxos locais entre os Serviços
de Acolhimento, o Serviço de Acolhimento em Família Acolhedora e os órgãos abaixo elencados, a fim
de facilitar a comunicação, o planejamento e o desenvolvimento de ações coordenadas. Tais órgãos
desempenham funções fundamentais para a garantia da excepcionalidade e provisoriedade do
afastamento do convívio familiar, bem como da reparação de possíveis violações de direito vivenciadas.
Abaixo são mencionados os principais aspectos que exigem uma articulação eficiente entre os serviços
de acolhimento e os órgãos elencados:
- Sistema de Justiça (Poder Judiciário, Ministério Público, Defensoria Pública): apoio na implementação
do Plano de Atendimento Individual e Familiar, por meio da aplicação de outras medidas protetivas
quando necessário; acompanhamento do processo de reintegração familiar; investigação e
responsabilização dos agressores nos casos de violência contra a criança ou adolescente; investigação
de paternidade e pensão alimentícia, quando for o caso; destituição do Poder Familiar e cadastramento
de crianças e adolescentes para adoção, nos casos em que não for possível a reintegração familiar;
preparação de todos os envolvidos para colocação em família substituta e deferimento da guarda, tutela
ou adoção; fiscalização do atendimento prestado nos serviços de acolhimento; acesso gratuito a serviços
advocatícios para defesa de direitos, dentre outros;
- Conselho Tutelar: apoio na implementação do Plano de Atendimento Individual e Familiar;
acompanhamento da situação familiar de crianças e adolescentes acolhidos; aplicação de outras medidas
protetivas quando necessário; apoio na reintegração familiar; dentre outros;
- Segurança Pública: investigação e responsabilização nos casos de violência contra a criança ou
adolescente; localização de familiares; acompanhamento da situação de pais ou responsáveis que

Apostila gerada especialmente para: Betânia Mueller 023.712.580-32


. 97
estejam no sistema prisional, inclusive para viabilizar a manutenção de contato destes com as crianças e
adolescentes acolhidos; dentre outros.
- Conselhos de Direitos: elaboração, aprovação e acompanhamento das ações do Plano de Nacional
de Promoção, Proteção e Defesa de Direito de Crianças e Adolescentes à Convivência Familiar e
Comunitária, no âmbito nacional, estadual e municipal; elaboração e aprovação de resoluções estaduais
e municipais; inscrição de programas governamentais e não-governamentais; registro de entidades que
executam serviços de acolhimento conforme Art. 90 do ECA; deliberação de políticas de atendimento
para atender os direitos humanos de crianças e adolescentes que se encontram atendidos nos serviços
de acolhimento.

3.5 Projeto Político-Pedagógico

Para garantir a oferta de atendimento adequado às crianças e aos adolescentes, os serviços de


acolhimento deverão elaborar um Projeto Político-Pedagógico (PPP), que deve orientar a proposta de
funcionamento do serviço como um todo, tanto no que se refere ao seu funcionamento interno, quanto
seu relacionamento com a rede local, as famílias e a comunidade. Sua elaboração é uma tarefa que deve
ser realizada coletivamente, de modo a envolver toda a equipe do serviço, as crianças, adolescentes e
suas famílias. Após a elaboração, o Projeto deve ser implantado, sendo avaliado e aprimorado a partir da
prática do dia- a dia.
Sem a intenção de propor um modelo de PPP, destacam-se abaixo alguns tópicos a serem
considerados para elaboração do mesmo:
- Apresentação (histórico, atual composição da diretoria, os principais momentos do serviço, as
principais mudanças e melhorias realizadas, em especial se sua instalação for anterior ao ECA,);
- Valores do serviço de acolhimento (valores que permeiam o trabalho e ação de todos os que
trabalham e encontram-se acolhidos no serviço);
- Justificativa (razão de ser do serviço de acolhimento dentro do contexto social); Objetivos do Serviço
de Acolhimento;
- Organização do serviço de acolhimento (espaço físico, atividades, responsabilidades, etc.);
- Organograma e quadro de pessoal (recursos humanos, cargos, funções, turnos, funcionários,
competências e habilidades necessárias para o exercício da função; modo de contratação; estratégias
para capacitação e supervisão);
- Atividades psicossociais (com as crianças e adolescentes, visando trabalhar questões pedagógicas
complementares, autoestima, resiliência, autonomia; com as famílias de origem, visando a preservação
e fortalecimento de vínculos e reintegração familiar);
- Fluxo de atendimento e articulação com outros serviços que compõe o Sistema de Garantia de
Direitos;
- Fortalecimento da autonomia da criança, do adolescente e do jovem e preparação para desligamento
do serviço;
- Monitoramento e avaliação do atendimento (métodos de monitoramento e avaliação do serviço que
incluam a participação de funcionários, voluntários, famílias e atendidos durante o acolhimento e após o
desligamento);
- Regras de convivência (direitos, deveres e sanções);

3.5.1 Atitude receptiva e acolhedora no momento da chegada da criança/adolescente e durante o


período de acolhimento

Muitas crianças e adolescentes desconhecem ou não compreendem o motivo pelo qual foram
afastadas do convívio familiar, o que pode levá-los a encarar a medida como uma espécie de punição e
despertar sentimentos de insegurança, rejeição, agressividade, revolta, abandono e outros. Diante disso,
deve-se dar especial atenção ao momento de acolhida inicial da criança/adolescente, no qual deve ser
dado tratamento respeitoso e afetuoso, apresenta-lhes, inclusive, o espaço físico, as crianças e os
adolescentes que lá se encontram, seu educador/cuidador de referência - ou membros da família
acolhedora - e seu espaço privado (cama, armário, etc.). Tanto no acolhimento institucional quanto no
acolhimento familiar, é importante que as regras de convívio no novo ambiente sejam explicadas para a
criança ou adolescente acolhido. Não é necessário que isso ocorra num primeiro momento do
acolhimento, podendo estas regras ser gradativamente explicitadas. Tais normas têm como objetivo
organizar um ambiente seguro e previsível, porém com flexibilidade e espaço para o lúdico, o coletivo e
para a construção ou reconstrução de regras que incluam a participação das crianças e adolescentes, de
modo a facilitar seu desenvolvimento. As famílias acolhedoras devem ser particularmente orientadas

Apostila gerada especialmente para: Betânia Mueller 023.712.580-32


. 98
quanto ao acolhimento inicial da criança/adolescente e sobre a importância de facilitar o processo de
conhecimento mútuo e integração da criança/adolescente ao ambiente familiar e comunitário de
acolhimento.
Nos serviços de acolhimento, para que a acolhida inicial seja afetuosa e não represente uma re-
vitimização de crianças e adolescentes é importante que o serviço disponha de:
- equipe técnica, educadores/cuidadores ou famílias acolhedoras disponíveis e capacitados para a
realização de acolhida afetuosa e segura, capazes de compreender as manifestações da criança ou
adolescente no momento de chegada que envolve ruptura, incerteza, insegurança e transição (silêncio,
choro ou agressividade, por exemplo);
- espaço físico destinado à acolhida inicial daqueles que estão chegando, adequado, inclusive, para a
acomodação daqueles que chegarem durante o período noturno;
- fluxos de comunicação eficiente e ágil dos órgãos encaminhadores (Conselho Tutelar, Justiça da
Infância e da Juventude ou outros, no caso de acolhida emergencial) com os serviços de acolhimento.
Estes fluxos são fundamentais para que os profissionais do serviço de acolhimento sejam comunicados
previamente acerca de cada novo acolhimento e, em tempo hábil, possam preparar o ambiente e aqueles
que já se encontram acolhidos para a chegada do novo colega.
No caso específico de crianças e adolescentes que estejam em situação de rua, a acolhida inicial deve
fazer parte de uma estratégia de sensibilização para o acolhimento no serviço e construção de vínculo de
confiança com o mesmo. Ao longo do processo de trabalho pela saída da rua, além dos aspectos aqui
mencionados, deve-se trabalhar também o significado do “estar e não-estar na rua”, expectativas, desejos
e temores quanto à retomada do convívio familiar e social, dentre outros aspectos.
Durante o período de acolhimento deve-se favorecer a construção da vinculação de afeto e confiança
com a equipe técnica, educador/cuidador ou família acolhedora e colegas. É importante, ainda, que ao
longo do acolhimento a criança e o adolescente tenham a possibilidade de dialogar com a equipe técnica
e com educador/cuidador de referência (ou família acolhedora) sobre suas impressões e sentimentos
relacionados ao fato de estar afastado do convívio com a família. Nessas conversas é importante que o
interlocutor possibilite uma expressão livre da criança ou do adolescente, oportunizando-lhes espaço no
qual possam falar sobre sua história de vida, sentimentos, desejos, angústias e dúvidas quanto às
vivências pregressas, ao afastamento da família de origem e sua situação familiar.
Essas conversas não precisam ocorrer imediatamente após a chegada da criança ou do adolescente,
mas em momentos de contato afetivo, nos quais a criança/adolescente possa se expressar e ser ouvido
de uma forma sensível e acolhedora. O interlocutor deve mostrar-se disponível e manter postura
acolhedora e respeitosa, sem culpabilizar ou julgar a família de origem ou a criança/adolescente. Nessas
oportunidades, deve-se esclarecer também que o serviço de acolhimento é organizado para a sua
proteção e constitui um direito seu.

3.5.2 Não-desmembramento de grupos de crianças/adolescentes com vínculos de parentesco e


fortalecimento de sua vinculação afetiva

Crianças e adolescentes com vínculos de parentesco (irmãos, primos, etc.), não devem ser separados
ao serem encaminhados para serviço de acolhimento, salvo se isso for contrário a seu desejo ou
interesses ou se houver claro risco de abuso, tendo em vista o melhor interesse da criança e do
adolescente. Para estas crianças e adolescentes que já se encontram afastados do convívio familiar, é
particularmente importante preservar e fortalecer seus vínculos fraternos e de parentesco, o que pode
contribuir para a formação de suas identidades, preservação da história de vida e referência familiar. Por
esse motivo, é importante que os serviços de acolhimento estejam organizados de modo a possibilitar
atendimento conjunto a grupos de irmãos ou de crianças e adolescentes com outros vínculos de
parentesco, que podem ter faixas etárias distintas e ambos os sexos. O PPP deve, portanto, contemplar
estratégias para a preservação do convívio e o fortalecimento da vinculação afetiva de tais crianças e
adolescentes.
No caso de adolescentes acolhidos que possuam filhos, o atendimento deve fortalecer a vinculação
afetiva, contribuir para o desenvolvimento de habilidades para o cuidado, a construção de um projeto de
vida e o desenvolvimento da autonomia, de modo a garantir a proteção à(ao) adolescente e a seu(s)
filho(s). Esse cuidado pode contribuir para prevenir a perpetuação de ciclos transgeracionais de ruptura
de vínculos, abandono, negligência ou violência, representando importante recurso para garantir o direito
à convivência familiar da(o) adolescente e da criança. Nesses casos é importante que sejam viabilizadas
condições para assegurar às mães e aos pais adolescentes os direitos inerentes aos dois momentos de
vida: maternidade/paternidade e adolescência. O PPP dos serviços de acolhimento deve também prever
estratégias para o atendimento a estas situações.

Apostila gerada especialmente para: Betânia Mueller 023.712.580-32


. 99
3.5.3 Organização de registros sobre a história de vida e desenvolvimento de cada criança e
adolescente

A equipe técnica do serviço de acolhimento deverá organizar prontuários individuais com registros
sistemáticos que incluam: histórico de vida, motivo do acolhimento, data de entrada e desligamento,
documentação pessoal, informações sobre o desenvolvimento (físico, psicológico e intelectual),
condições de saúde, informações sobre a vida escolar, etc. Crianças e adolescentes com deficiência,
transtornos mentais e necessidades específicas de saúde devem ter registros e informações que
favoreçam a prestação de cuidados adequados, inclusive, relativos à sua saúde. Devem ser organizados
registros semanais de cada criança e adolescente, nos quais conste relato sintético sobre a rotina,
progressos observados no desenvolvimento, vida escolar, socialização, necessidades emergenciais,
mudanças, encontro com familiares, dados de saúde, etc.
Tais registros devem conter, ainda, informações sobre a família de origem, o trabalho desenvolvido
com vistas à reintegração familiar (visitas, encaminhamentos, acompanhamento em grupo, encontros da
família com a criança ou adolescente, preparação para a reintegração, etc.) e o acompanhamento da
família acolhedora, se for o caso. Esses registros devem ser consultados apenas por profissionais
devidamente autorizados, devendo os serviços de acolhimento ter uma política clara de confidencialidade
desses dados, observada por todos os profissionais. A transmissão pelos técnicos aos
educadores/cuidadores ou família acolhedora de informações necessárias ao atendimento das crianças
e adolescentes deve estar pautada em princípios éticos, os quais também devem pautar a postura dos
educadores/cuidadores. Os registros devem ser acessíveis à equipe, caso a criança ou adolescente seja
novamente acolhida.
Sempre que possível, a fim de promover um sentido de identidade própria, a criança e o adolescente
- com o apoio de um educador/cuidador, família acolhedora ou pessoa previamente preparada - devem
ter a oportunidade de organizar um livro de sua história de vida que reúna informações, fotografias e
lembranças referentes a cada fase de sua vida, ao qual poderão ter acesso ao longo do ciclo vital. Este
livro deve ser uma produção da própria criança ou adolescente, com fotos e outras criações de sua
autoria. No momento do desligamento esse registro deve fazer parte dos objetos pessoais que a criança
ou adolescente levará consigo.

3.5.4 Definição do papel e valorização dos educadores/cuidadores e da família acolhedora

A postura dos educadores/cuidadores e das famílias acolhedoras e a qualidade da interação


estabelecida com a criança e do adolescente representam importantes referenciais para seu
desenvolvimento. Para tanto, o PPP deve prever estratégias para sua seleção, capacitação e
acompanhamento/supervisão.
Em função de sua importância, o educador/cuidador e a família acolhedora devem ter clareza quanto
a seu papel: vincular-se afetivamente às crianças/adolescentes atendidos e contribuir para a construção
de um ambiente familiar, evitando, porém, “se apossar” da criança ou do adolescente e competir ou
desvalorizar a família de origem ou substituta. O serviço de acolhimento, não deve ter a pretensão de
ocupar o lugar da família da criança ou adolescente, mas contribuir para o fortalecimento dos vínculos
familiares, favorecendo o processo de reintegração familiar ou o encaminhamento para família substituta,
quando for o caso.
Para exercer sua função o educador/cuidador ou a família acolhedora deve ter capacitação adequada
para desempenhar seu papel com autonomia e ser reconhecido como figura de autoridade para a criança
e o adolescente e, como tal, não ser desautorizado pelos outros profissionais do serviço (técnicos,
coordenadores), sobretudo na presença da criança e do adolescente. Além disso, devem de apoio e
orientação permanente por parte da equipe técnica do serviço, bem como de espaço para trocas, nos
quais possam compartilhar entre si experiências e angústias decorrentes da atuação, buscando a
construção coletiva de estratégias para o enfrentamento de desafios.
Visando o constante aprimoramento do cuidado prestado, devem ser realizados, periodicamente,
estudos de caso com a participação da equipe técnica e educadores/cuidadores, nos quais se possa
refletir sobre o trabalho desenvolvido com cada criança/adolescente e as dificuldades encontradas. Esses
estudos devem propiciar também planejamentos de intervenções que tenham como objetivo a melhoria
do atendimento no serviço e da relação entre educador/cuidador e criança/adolescente, bem como a
potencialização de aspectos favorecedores de seu processo de desenvolvimento, autoestima e
autonomia. Tais aspectos devem ser igualmente contemplados no acompanhamento às famílias
acolhedoras.

Apostila gerada especialmente para: Betânia Mueller 023.712.580-32


. 100
É importante que a equipe técnica do serviço de acolhimento auxilie os educadores/ cuidadores ou as
famílias acolhedoras na oferta de um cuidado individualizado para cada criança e adolescente, baseado
na avaliação de suas condições emocionais, história de vida, impacto da violência ou do afastamento do
convívio familiar, situação familiar, vinculações significativas e interações estabelecidas. Estes
profissionais devem apoiar os educadores/cuidadores ou as famílias acolhedoras no exercício de seu
papel, contribuindo para uma construção conjunta de estratégias que colaborem para o desenvolvimento
de um ambiente estruturante para a criança e o adolescente.
Finalmente, o educador/cuidador ou a família acolhedora devem participar e ter sua opinião ouvida
pela equipe técnica do serviço na tomada de decisões sobre a vida da criança e do adolescente, como,
por exemplo, nas ocasiões em que se mostrar necessária a elaboração de relatório para a Autoridade
Judiciária com recomendação de reintegração familiar ou adoção. Nesses casos, deve ser priorizada a
participação da família acolhedora ou daquele educador/cuidador com o qual a criança/adolescente
mantenha vinculação afetiva mais significativa e que conheça seus desejos e interesses.

3.5.5 Relação do Serviço com a família de origem

Trabalhar com as famílias das crianças e dos adolescentes acolhidos em abrigos ou nas famílias
acolhedoras implica compreender sua configuração, buscar suas competências e entender sua inserção
na comunidade. O trabalho com essas famílias precisa favorecer a superação das questões, por vezes
bastante complexas, que contribuíram para o afastamento da criança ou adolescente do convívio familiar.
É importante compreender como as famílias estão vivenciando a situação de afastamento de seus filhos
e potencializá-las para a retomada do convívio e exercício de seu papel de proteção e cuidados.
As crenças e significados construídos pelas famílias acolhedoras e pelos profissionais do serviço de
acolhimento acerca das famílias de origem e vice-versa influenciam a relação entre estas e o serviço de
acolhimento. Nesse sentido, é importante atentar para a forma como as famílias das crianças e
adolescentes são significadas pelos profissionais dos serviços de acolhimento e pelas famílias
acolhedoras. Se são consideradas “capazes” ou “incapazes”, “estruturadas” ou “desestruturadas”, “parte
do problema” ou “agente transformador”. Por outro lado, os serviços de acolhimento ou a família
acolhedora também podem ser percebidos pela família como “aliados” ou “raptores de seus filhos”.
O educador/cuidador ou a família acolhedora e todos os profissionais do serviço de acolhimento devem
receber orientações para, nos momentos de visitas da família ao serviço e contato com a criança e o
adolescente, atuarem, se necessário como mediadores dessa relação, proporcionando, ainda, momentos
nos quais a família possa estar a sós com a criança e adolescente. Os profissionais do serviço de
acolhimento, famílias acolhedoras e pessoas com as quais a criança ou o adolescente venham a ter
contato em razão do acolhimento não devem se referir de modo pejorativo à família de origem. Ainda que
o afastamento tenha ocorrido por motivos graves, a criança e o adolescente devem ter sua origem –
família, comunidade, cultura - tratada com respeito.
- O PPP deve prever no funcionamento do serviço e no trabalho com as famílias de origem ações que
promovam o fortalecimento dos vínculos das crianças e adolescentes com suas famílias:
- Preparação dos serviços de acolhimento institucional e da família acolhedora para aceitação e
acolhimento dos familiares;
- Flexibilidade nos horários de visitas. Devem ser acordados com a família de origem horários e
periodicidade das visitas à criança e ao adolescente. O esquema de visitação deve ser flexível e baseado
na observação da realidade familiar e das dificuldades de acesso da família ao serviço (horários de
trabalho, distância, transporte, etc.). Podem ser organizadas, ainda, atividades que incluam a participação
da família, como almoço dominical com e para os familiares;
- Participação da família na organização e comemoração de aniversários e outras datas
comemorativas, sempre que possível realizadas no domicílio da família;
- Saídas das crianças e adolescentes para finais de semana com os familiares;
- O serviço deve também apoiar as visitas da criança e do adolescente à família;
- Telefonemas para a família de origem e destas para as crianças e adolescentes que se encontrem
acolhidos;
- Realizações de atividades recreativas e culturais com as famílias, crianças, adolescentes e
profissionais do serviço;
- Realização de “Oficinas de talentos” nas quais as famílias de origem, a criança ou adolescente
difundam seus saberes e habilidades específicas (artesanato, brincadeiras, pequenos consertos,
aproveitamento de alimentos e materiais, etc.);
- Rodas de conversas para pais e filhos, abordando temas levantados pela família, crianças e
adolescentes;

Apostila gerada especialmente para: Betânia Mueller 023.712.580-32


. 101
Participação dos familiares nas reuniões da escola do filho e consultas de saúde.
Além de favorecer a aproximação entre as famílias, crianças e adolescentes, estas atividades podem
favorecer, ainda, a aproximação das famílias entre si, de modo a construir uma rede de apoio mútuo,
identificação e trocas de experiência.
No caso de acolhimento em Famílias Acolhedoras, é importante que estas possam contar com a
orientação da equipe técnica acerca do relacionamento com a família de origem, na perspectiva do
fortalecimento de vínculos com a criança e o adolescente. Nestes casos é igualmente importante que o
papel das famílias acolhedoras fique claro tanto para estas, quanto para as famílias de origem, de modo
a evitar rivalidades. Para favorecer uma interação positiva entre família acolhedora e família de origem e
a aproximação crescente desta com a criança/adolescente, ao longo do acompanhamento podem ser
promovidas pelo serviço atividades que reúnam grupos com famílias de origem, acolhedoras e crianças
e adolescentes acolhidos. Outras atividades, espontâneas ou programadas, podem, ainda, ser realizadas
por iniciativas da família acolhedora, da família de origem ou da própria criança/adolescente acolhido. As
famílias acolhedoras devem, ainda, ser orientadas pela equipe técnica nas situações em que o contato
da criança/adolescente com a família de origem tiver impedimento judicial.
O PPP deve prever também a elaboração do Plano de Atendimento Individual e Familiar, de forma a
assegurar o acompanhamento da criança e/ou adolescente no período em que estiver acolhido e ações
necessárias para que o retorno à família de origem, ou, na sua impossibilidade, a colocação em família
substituta, seja realizado da melhor maneira possível.

3.5.6 Preservação e fortalecimento da convivência comunitária

Os serviços de acolhimento devem estar localizados em áreas residenciais, sem distanciar-se


excessivamente, do ponto de vista geográfico e socioeconômico, do contexto de origem das crianças e
adolescentes. Salvo determinação judicial em contrário, quando necessário afastamento do convívio
familiar e encaminhamento para serviço de acolhimento, esforços devem ser empreendidos para manter
a criança e o adolescente o mais próximo possível de seu contexto de origem, a fim de facilitar o contato
com a família e o trabalho pela reintegração familiar. A proximidade com o contexto de origem tem como
objetivo, ainda, preservar os vínculos comunitários já existentes e evitar que, além do afastamento da
família, o acolhimento implique o afastamento da criança e do adolescente de seus colegas, vizinhos,
escola, atividades realizadas na comunidade, etc.
Sempre que possível a criança e o adolescente devem frequentar a mesma escola em que estudavam
antes do acolhimento, de modo a preservar vínculos pré-existentes – salvo mudança necessária para sua
proteção. As crianças e os adolescentes devem ter, ainda, a possibilidade de continuar a frequentar
atividades que realizavam antes do acolhimento (atividades esportivas, culturais, religiosas, entre outras).
O acolhimento não deve significar, ainda, privação do direito à convivência comunitária. Nesse sentido,
o serviço de acolhimento, em parceria com a rede local e a comunidade, deverá empreender esforços
para favorecer a construção de vínculos significativos entre crianças, adolescentes e comunidade. Para
evitar prejuízo ao convívio com a comunidade, espaços públicos e instituições, os serviços de acolhimento
não deverão concentrar em suas dependências equipamentos destinados à oferta de serviços de outra
natureza, como, por exemplo, de atendimento médico, odontológico, educação infantil (“creche”), etc. Do
mesmo modo, não deverão concentrar espaços de lazer geralmente não disponibilizados em unidades
residenciais, como quadras poliesportivas, etc.
A criança e o adolescente devem participar da vida diária da comunidade e ter a oportunidade de
construir laços de afetividade significativos com a mesma. Deve-se propiciar sua participação nas
festividades e demais eventos da comunidade, além da utilização da rede socioassistencial, de educação,
saúde, cultura, esporte e lazer disponíveis na rede pública ou comunitária. No acesso a atividades
culturais, esportivas e de lazer deve-se observar o interesse, as habilidades e grau de desenvolvimento
da criança e do adolescente. Sendo possível, deve-se propiciar que esse acesso não seja realizado
sempre de modo coletivo, ou seja, com várias crianças e adolescentes do serviço frequentando as
mesmas atividades nos mesmos horários, a fim de favorecer também a interação com outras
crianças/adolescentes da comunidade.
Além de oportunizar o contato de crianças e adolescentes acolhidos com crianças e adolescentes da
comunidade, essas medidas têm como objetivo propiciar o desenvolvimento da autonomia e da
socialização dos mesmos. O acesso aos serviços na rede local tem como objetivo, ainda, inserir a criança
e o adolescente em atividades que possam continuar a frequentar após a reintegração familiar.
Esforços também devem ser empreendidos no sentido de evitar a estigmatização da criança e do
adolescente durante e após o acolhimento. Nesse sentido, não devem ser utilizados uniformes e, sempre
que possível, o transporte em veículo com identificação. Garantidas restrições essenciais à sua

Apostila gerada especialmente para: Betânia Mueller 023.712.580-32


. 102
segurança, crianças e adolescentes devem circular pela comunidade de modo semelhante àqueles de
sua mesma faixa etária – caminhando, usando o transporte público ou bicicletas – contando com a
companhia de educadores/cuidadores ou outros responsáveis quando o seu grau de desenvolvimento ou
a situação assim exigir. No convívio com a comunidade deve ser oportunizado que crianças e
adolescentes possam tanto receber seus colegas nas dependências do serviço como participar, por
exemplo, de festas de aniversário de colegas da escola. Em síntese, no convívio comunitário devem ser
proporcionadas também experiências individualizadas.
O contato direto de pessoas da comunidade com crianças e adolescentes em serviços de acolhimento,
nas dependências do mesmo, deverá ser precedido de preparação, visando assegurar que este contato
será benéfico às crianças e aos adolescentes. Nesse sentido, é importante destacar que visitas
esporádicas daqueles que não mantêm vínculo significativo e frequentemente sequer retornam uma
segunda vez ao serviço de acolhimento, expõem as crianças e os adolescentes à permanência de
vínculos superficiais. Estes podem, inclusive, contribuir para que não aprendam a diferenciar conhecidos
de desconhecidos e tenham dificuldades para construir vínculos estáveis e duradouros, essenciais para
seu desenvolvimento. Por esse motivo, Programas de Apadrinhamento Afetivo ou similares devem ser
estabelecidos apenas quando dispuserem de metodologia com previsão de cadastramento, seleção,
preparação e acompanhamento de padrinhos e afilhados por uma equipe interprofissional, em parceria
com a Justiça da Infância e Juventude e Ministério Público.
Nos Programas de Apadrinhamento Afetivo devem ser incluídos, prioritariamente, crianças e
adolescentes com previsão de longa permanência no serviço de acolhimento, com remotas perspectivas
de retorno ao convívio familiar ou adoção, para os quais vínculos significativos com pessoas da
comunidade serão essenciais, sobretudo, no desligamento do serviço de acolhimento. Para estes casos,
a construção de vínculos afetivos significativos na comunidade pode ser particularmente favorecedora,
devendo ser estimulada, observando os critérios anteriormente citados.

3.5.7 Fortalecimento da autonomia da criança, do adolescente e do jovem

Todas as decisões a respeito de crianças e adolescentes atendidos em serviços de acolhimento devem


garantir o direito de ter sua opinião considerada. Pode meio de métodos condizentes com o grau de
desenvolvimento da criança/adolescente, deve-se assegurar o direito à escuta nas diversas decisões que
puderem repercutir sobre o desenvolvimento e a trajetória de vida da criança e do adolescente,
envolvendo desde a identificação de seu interesse pela participação em atividades na comunidade, até
mudanças relativas à sua situação familiar ou desligamento do serviço de acolhimento ou família
acolhedora.
Além de participar da elaboração de projetos que versem sobre sua trajetória futura, as crianças e os
adolescentes devem ter acesso a informações sobre sua história de vida, situação familiar e motivos do
acolhimento. A comunicação dessas informações deverá pautar-se na consideração do seu grau de
desenvolvimento e na avaliação dos benefícios ou prejuízos que poderão resultar deste conhecimento. O
acesso a essas informações deverá respeitar o processo individual de apropriação da história de vida –
devendo ser conduzido por profissionais orientados e preparados, com os quais a criança e o adolescente
mantenham vinculação afetiva significativa.
Ações devem ser desenvolvidas visando o fortalecimento de habilidades, aptidões, capacidades e
competências das crianças e adolescentes, de modo a fortalecer gradativamente sua autonomia. Assim,
a própria organização do ambiente de acolhimento deverá proporcionar o fortalecimento gradativo da
autonomia, de modo condizente com o processo de desenvolvimento e aquisição de habilidades nas
diferentes faixas etárias. Nessa direção deve-se considerar, por exemplo, desde as condições para
estimular a exploração do ambiente e desenvolvimento psicomotor das crianças nos primeiros anos de
vida, até a aquisição da autonomia para o autocuidado, preservação de objetos pessoais e cumprimento
de responsabilidades decorrentes de atividades desenvolvidas na comunidade – lazer, esporte, cultura,
saúde, educação, qualificação profissional, trabalho, etc.
Os serviços de acolhimento devem propiciar a organização de espaços de escuta e construção de
soluções coletivas com a participação das crianças e adolescentes. Nesse sentido, podem ser
organizados, por exemplo, espaço para a realização das chamadas “assembleias” nas quais crianças e
adolescentes sob cuidados em serviços de acolhimento possam desempenhar um papel participativo,
discutindo e construindo alternativas para a melhoria do serviço, para a ampliação das estratégias para
viabilizar o contato com a família de origem, etc. No caso de crianças/adolescentes acolhidos em Famílias
Acolhedoras, às regras e à rotina diária da família é importante que sejam incorporados elementos
significativos para a criança e o adolescente, propiciando a estes a oportunidade de sugerir alterações
que promovam um senso de familiaridade com o novo ambiente.

Apostila gerada especialmente para: Betânia Mueller 023.712.580-32


. 103
As crianças e os adolescentes devem ter a oportunidade de realizar pequenas mudanças nos espaços
privativos, fazer escolhas e participar da organização do ambiente de acolhimento, segundo seu grau de
desenvolvimento e capacidades. Assim, de modo gradativo e estritamente com função pedagógica,
devem participar da organização da rotina diária da instituição e assumir responsabilidade pelo cuidado
com seus objetos pessoais, com seu autocuidado e cumprimento de compromissos (escola, atividades
na comunidade, trabalho, etc). É importante ressaltar que a participação das crianças e adolescentes nas
tarefas relacionadas aos cuidados domésticos devem ser estimuladas, sem detrimento, todavia, de
qualquer outra atividade lúdica ou educativa.
Na frequência a atividades realizadas na comunidade - escola, rede de saúde, atividades culturais,
esportivas, de lazer e outras - considerar-se-á o estímulo gradativo à autonomia. Nesse sentido, não
devem ser impostas restrições injustificáveis à liberdade e conduta, em comparação com crianças e
adolescentes da mesma idade e comunidade. Tais restrições devem ser condizentes com o grau de
desenvolvimento e capacidade da criança e do adolescente e restritas apenas àquelas necessárias para
viabilizar sua segurança e proteção.
Para ampliar a iniciativa, autonomia e o senso de responsabilidade é importante que as crianças e
adolescentes acolhidos possam participar, ainda, de atividades rotineiras como ir à padaria ou ao
supermercado, recebendo instruções sobre como lidar com o dinheiro. Geralmente os serviços de
acolhimento pouco propiciam o contato de crianças e adolescentes com esse tipo de conhecimento,
aspecto que será fundamental para a construção de projetos de vida ligados ao trabalho e aquisição futura
de autonomia financeira.
Atenção especial deve ser dada aos adolescentes atendidos em serviços de acolhimento, sobretudo
àqueles cujas possibilidades de reintegração à família de origem foram esgotadas e têm reduzidas
possibilidades de colocação em família substituta, face às dificuldades de se encontrar famílias para os
mesmos na realidade brasileira. Para estes casos, o PPP deve prever metodologia voltada à construção
e fortalecimento de vínculos comunitários significativos, à ampliação do acesso à educação, à qualificação
profissional e à progressiva autonomia do adolescente para o cuidado consigo mesmo e o cumprimento
de suas responsabilidades. O atendimento deve favorecer a construção de projetos de vida e o
fortalecimento do protagonismo, desenvolvendo gradativamente a capacidade do adolescente
responsabilizar-se por suas ações e escolhas. Visando apoiar os adolescentes acolhidos após o alcance
da maioridade, devem ser organizados serviços de acolhimento em Repúblicas66, como uma forma de
transição entre o serviço de acolhimento para crianças e adolescentes e a aquisição da autonomia.
Sempre que possível, deverá também ser oportunizada a participação das crianças, adolescentes e
jovens em serviços de acolhimento - ou adultos com histórico de atendimento nesses serviços durante a
infância, adolescência ou juventude - nas instâncias de formulação de políticas públicas, que constituem
importantes espaços para estimular a participação social e o protagonismo, como as Conferências da
Assistência Social, do Direito de Criança e do Adolescente, etc.

3.5.8 Desligamento gradativo

Tanto nos casos de reintegração à família de origem quanto nos de encaminhamento para família
substituta o serviço de acolhimento deve promover um processo de desligamento gradativo, com o
preparo da criança/adolescente, oportunizando-lhe a despedida necessária do ambiente, dos colegas,
dos educadores/cuidadores e dos demais profissionais. Além da criança e do adolescente, devem ser
previamente preparados também os educadores/cuidadores e demais crianças/adolescentes com as
quais tenham mantido contato em razão do acolhimento, assim com todos os membros da famílias
acolhedoras. Nesse sentido, podem ser viabilizados rituais de despedida, atividades em grupo com as
crianças e os adolescentes para tratar do desligamento, etc. É importante que a família de origem (natural
ou extensa) ou a família adotiva sejam acompanhadas após a saída da criança/adolescente do serviço.
A criança e o adolescente em processo de desligamento devem ter a oportunidade de conversar,
ainda, sobre suas expectativas e inseguranças quanto ao retorno ao convívio familiar, bem como sobre o
sentimento de saudade do ambiente de acolhimento, da família acolhedora, dos profissionais do serviço
e dos colegas. Os educadores/cuidadores ou famílias acolhedoras, particularmente aqueles que mantêm
vinculação afetiva mais significativa com a criança e o adolescente, devem ser preparados e receber
especial apoio nesse momento. Nesse sentido, é importante que no serviço de acolhimento seja
viabilizado um espaço de acompanhamento contínuo, no qual possam expressar, inclusive, a dor pela
separação da criança ou do adolescente.
Atenção especial deve ser dada à preparação nos casos de desligamento de crianças/adolescentes
que permaneceram no serviço de acolhimento por um longo período. Uma articulação permanente com
a Justiça deve garantir um planejamento conjunto do processo de desligamento, de modo a prevenir

Apostila gerada especialmente para: Betânia Mueller 023.712.580-32


. 104
separações abruptas e permitir a avaliação do momento mais adequado para a ocorrência do
desligamento. Sempre que possível e positivo para a criança e o adolescente, devem ser viabilizados
contatos posteriores ao desligamento com colegas, educadores/cuidadores, famílias acolhedoras e
outros profissionais do serviço. Em casos de encaminhamento para adoção, é importante planejar o
encontro da criança ou adolescente com a família substituta, com formas adequadas de aproximação e
estratégias de apresentação que considerem as características específicas do caso. Nesse trabalho, é
fundamental a parceria efetiva entre a equipe interprofissional da Justiça da Infância e da Juventude e do
serviço de acolhimento, que inclua também educadores/cuidadores e demais profissionais, famílias
acolhedoras e Grupos de Apoio à Adoção, onde houver. Finalmente, é importante destacar que, em
conformidade com o ECA, decisão de quais crianças e adolescentes serão colocados em quais famílias
substitutas compete, exclusivamente, à autoridade judiciária, ouvido o Ministério Público.
É importante que as crianças/adolescentes sejam informados sobre sua real situação, sendo-lhes
explicada a possibilidade de adoção e oportunizando-lhes espaço para expressarem o que pensam e
sentem a respeito dessa possibilidade. Além disso, nos casos de encaminhamento para adoção é preciso
assegurar tempo suficiente para as crianças e adolescentes se desligarem e se despedirem gradualmente
das pessoas com as quais construíram vinculações afetivas ao longo do período de acolhimento. O
desligamento não deve ser visto como um momento apenas, mas como resultado de um processo
contínuo de desenvolvimento da autonomia e como resultado de um investimento no acompanhamento
da situação de cada criança e adolescente. Particularmente no que diz respeito aos adolescentes, a
preparação para o desligamento deve incluir o acesso a programas de qualificação profissional e inserção
no mercado de trabalho, como aprendiz ou trabalhador – observadas as devidas limitações e
determinações da Lei nesse sentido, visando sua preparação para uma vida autônoma. Sempre que
possível, ainda, o serviço manterá parceria com Repúblicas, utilizáveis como uma forma de transição
entre o abrigo e a aquisição de autonomia e independência.
No caso de desligamento pela maioridade da adolescente grávida ou com filhos pequenos, deve ser
viabilizado, sempre que possível e necessário, seu encaminhamento para serviços destinados ao
atendimento a mulheres acompanhadas de seus filhos.

3.6 Gestão do Trabalho e Educação Permanente

Na história do nosso país, os serviços de acolhimentos foram geridos e tinham o quadro de pessoal
composto principalmente por pessoas voluntárias, religiosos ou leigos. Aos poucos essa realidade tem
se modificado, mas ainda hoje há a prevalência da concepção de que “basta o bom coração” para se
trabalhar nesses serviços. O reconhecimento de que todos os profissionais que atuam em serviços de
acolhimento desempenham o papel de educador, impõe a necessidade de seleção, capacitação e
acompanhamento de todos aqueles responsáveis pelo cuidado direto e cotidiano das crianças e
adolescentes acolhidos.
Para isso, em consonância com o que já está disposto na Norma Operacional Básica de Recursos
Humanos do SUAS (NOB-RH/SUAS), seguem algumas orientações para gestão do trabalho e educação
permanente, que devem ser adequadas às necessidades de cada município, considerando suas
particularidades.

3.6.1 Seleção

Um processo de seleção criterioso dos profissionais que atuarão nos Serviços de Acolhimento é
essencial para a garantia de contratação de pessoal qualificado e com perfil adequado ao
desenvolvimento de suas funções, possibilitando a oferta de um serviço de qualidade aos usuários. Para
tanto, deve-se prever, minimamente, os seguintes passos:
Ampla divulgação, com informações claras sobre o serviço, o perfil dos usuários, as atribuições e
exigências do cargo a ser ocupado, salário e carga horária, dentre outros;
Processo seletivo, com atenção à exigência da formação mínima para cada função e experiência
profissional;
Avaliação de documentação mínima a ser exigida: documentos pessoais, certidão negativa de
antecedentes criminais, atestado de saúde física e mental;
Avaliação psicológica e social: análise da vida pregressa, entrevista individual e atividade de grupo;
Constituem características desejáveis aos candidatos(a): motivação para a função; aptidão para o
cuidado com crianças e adolescentes; capacidade de lidar com frustração e separação; habilidade para
trabalhar em grupo; disponibilidade afetiva; empatia; capacidade de lidar com conflitos; criatividade;

Apostila gerada especialmente para: Betânia Mueller 023.712.580-32


. 105
flexibilidade; tolerância; proatividade; capacidade de escuta; estabilidade emocional, dentre outras. Para
os coordenadores, é ainda desejável capacidade de liderança e gestão de equipes.
No caso de educadores/cuidadores residentes, para atendimento em casas-lares, também deverá ser
verificado disponibilidade para residir, grau de independência pessoal e familiar que permita dedicação
afetiva e profissional e capacidade para administrar a rotina doméstica.
No caso do coordenador, equipe técnica e educador/cuidador, constituem habilidades e
conhecimentos técnicos desejáveis:
Coordenador: gestão; trabalho em rede; crianças e adolescentes em situação de risco; conhecimentos
sobre seleção e desenvolvimento de Recursos Humanos; conhecimento aprofundado do ECA, SUAS,
Sistema de Justiça e PNCFC.
Equipe Técnica: violência e exclusão social, crianças e adolescentes em situação de risco,
separações, vinculações, dependência química; desenvolvimento infanto-juvenil; seleção e
desenvolvimento de Recursos Humanos; atendimento a criança, adolescente e família; atendimento em
grupo; trabalho em rede; acesso a serviços, programas e benefícios; ECA; SUAS; Sistema de Justiça e
PNCFC.
Educador/Cuidador: cuidados com crianças e adolescentes; noções sobre desenvolvimento infanto-
juvenil; noções sobre ECA; SUAS; Sistema de Justiça e PNCFC.
Quando se tratar de serviços de acolhimento governamentais, particular atenção deverá ser dada à
elaboração de editais de concursos públicos para o provimento dos cargos. Além da previsão de
formação mínima exigida para a função72, da exigência de conteúdos específicos para as provas de
seleção73 (vide sugestões no item acima, “Habilidades e conhecimentos técnicos desejáveis”) e da
previsão de provas de títulos que pontuem tanto titulação acadêmica em áreas relacionadas ao
desempenho da função, quanto experiência profissional no atendimento a crianças, adolescentes e
famílias. Recomenda-se que também sejam previstos nos editais, como etapas eliminatórias do concurso,
a avaliação psicológica; a análise de vida pregressa e curso de formação.

3.6.2 Capacitação

Investir na capacitação e acompanhamento dos educadores/cuidadores, assim como de toda a equipe


que atua nos serviços de acolhimento – incluindo coordenador, equipe técnica e equipe de apoio - é
indispensável para se alcançar qualidade no atendimento, visto se tratar de uma tarefa complexa, que
exige não apenas “espírito de solidariedade”, “afeto” e “boa vontade”, mas uma equipe com conhecimento
técnico adequado. Para tanto, é importante que seja oferecida capacitação inicial de qualidade, e
formação continuada a tais profissionais, especialmente aqueles que têm contato direto com as crianças
e adolescentes e suas famílias.
A seguir, serão apresentados temas gerais a serem abordados na capacitação dos profissionais que
atuarão nos serviços de acolhimento, os quais devem ser adaptados às necessidades e demandas
específicas de cada serviço e ser adequados de acordo com o nível de aprofundamento necessário ao
desenvolvimento da função específica de cada profissional.
Destaca-se que a realização dessa capacitação deve contar com o apoio e parceria de profissionais
que detenham conhecimento reconhecido no assunto, oriundos de possam falar da atividade e da
experiência e, tenha retorno do facilitador sobre sua participação. Muitas vezes o próprio candidato se
depara com dificuldades em determinados aspectos enfatizados na atividade de grupo que são
fundamentais para a função e declina do processo de seleção.

Capacitação Introdutória

A capacitação introdutória tem como objetivo inserir o profissional no serviço e na equipe já existente,
permitindo ainda que acompanhe, como observador, os diferentes momentos da rotina e a possibilidade
de posterior discussão sobre as observações realizadas. O nível de experiência do profissional norteará
o repasse e o conteúdo das informações na etapa inicial de adaptação à rotina do serviço.
Abaixo são elencados temas relevantes a serem trabalhados em uma capacitação inicial:
- Apresentação do serviço, suas especificidades e regras de funcionamento;
- Apresentação e discussão do Projeto Político-Pedagógico do serviço;
- Legislação pertinente (SUAS, PNCFC, ECA, dentre outros, além do presente documento);
- SGD e rede de políticas públicas - com o intuito de que o profissional compreenda as medidas
protetivas, competências e limites de atuação de cada órgão / entidade e articulação entre as instâncias
envolvidas;

Apostila gerada especialmente para: Betânia Mueller 023.712.580-32


. 106
- Etapas do desenvolvimento da criança e do adolescente (características, desafios, comportamentos
típicos, fortalecimento da autonomia, desenvolvimento da sexualidade); brincadeiras e jogos adequados
para cada faixa etária, exploração do ambiente, formas de lidar com conflitos, colocação de limites, etc.;
Comportamentos frequentemente observados entre crianças/adolescentes separados da família de
origem, que sofreram abandono, violência, etc.;
- Práticas educativas como ajudar a criança/adolescente a conhecer e a lidar com sentimentos,
fortalecer a autoestima e contribuir para a construção da identidade;
- Cuidados específicos com crianças e adolescentes com deficiência ou necessidades específicas de
saúde (doença infectocontagiosa ou imunodepressora; transtorno mental; dependência química; etc);
- Novas configurações familiares e realidade das famílias em situação de vulnerabilidade e risco;
- Metodologia de trabalho com famílias;
- Diversidade cultural e sexual, étnicas e religiosas;
- Trabalho em rede.

Capacitação Prática

Antes de assumir suas funções, é importante que todos os profissionais acompanhem como auxiliar a
rotina da instituição, para poder gradativamente se apropriar da função que lhe é devida.
O educador/cuidador deverá passar por um período mínimo de 80 horas acompanhando, como
auxiliar, os diferentes momentos da rotina institucional, sempre sob supervisão de um educador/cuidador
experiente e da equipe técnica.
No caso de educador/cuidador residente, este período deverá ser de, no mínimo, 30 dias de
acompanhamento, como auxiliar, dos diferentes momentos da rotina da casalar, sempre sob supervisão
de um(a) educador/cuidador residente experiente e da equipe técnica.

3.6.3 Formação continuada

Para garantir qualidade ao Projeto Político-Pedagógico dos serviços de acolhimento, os horários para
que os educadores/cuidadores, equipe técnica e demais funcionários possam participar de cursos,
reuniões de formação, seminários e leituras devem ter lugar no planejamento da organização e das
escalas de trabalho.
Depois da contratação, para adaptação à rotina institucional é fundamental o acompanhamento
sistemático do profissional, incrementado com capacitações continuadas. As demandas de um serviço de
acolhimento exigem resolutividade, rapidez e mobilidade, pois, com o passar do tempo, pode-se gerar
um automatismo de respostas dos profissionais. Ou seja, há grande probabilidade de se cair na rotina,
agindo sem refletir sobre o atendimento que está sendo realizado. Além disso, os casos atendidos nesses
serviços acabam afetando de alguma forma emocionalmente os profissionais. Por toda esta realidade,
algumas atividades de acompanhamento são extremamente importantes no sentido de melhorar o
desempenho do profissional, a qualidade do atendimento institucional e o bem-estar das crianças e dos
adolescentes acolhidos. São elas:
Reuniões periódicas de equipe (discussão e fechamento de casos; reavaliação de Planos de
atendimento individual e familiar, construção de consensos, revisão e melhoria da metodologia)
Formação continuada sobre temas recorrentes do cotidiano, assim como sobre temas já trabalhados
na fase de capacitação inicial, orientada pelas necessidades institucionais (promovida pela própria
instituição e/ou cursos externos):
- Estudos de caso
- Supervisão institucional com profissional externo
- Encontros diários de 15-20 minutos entre os profissionais dos diferentes turnos para troca de
informações
- Grupo de escuta mútua
- Espaço de escuta individual
- Avaliação, orientação e apoio periódicos pela equipe técnica
A seguir, serão apresentados parâmetros de funcionamento para os serviços de acolhimento, no qual
serão detalhadas, dentre outros aspectos, a formação da equipe mínima para cada serviço: i. Abrigos
Institucionais; ii. Casas-Lares; iii. Famílias Acolhedoras; iv. Repúblicas.

Apostila gerada especialmente para: Betânia Mueller 023.712.580-32


. 107
4 PARÂMETROS DE FUNCIONAMENTO

As orientações contidas nesse capítulo têm como objetivo estabelecer parâmetros para a organização
dos serviços de acolhimento para crianças e adolescentes, visando sua adequação gradativa ao Estatuto
da Criança e do Adolescente, ao Plano Nacional de Convivência Familiar e Comunitária, à Política
Nacional de Assistência Social e ao Projeto de Diretrizes das Nações Unidas sobre Emprego e Condições
Adequadas de Cuidados Alternativos com Crianças. Esses parâmetros devem ser ajustados à realidade
e cultura local, sem, todavia, acarretar perda da qualidade dos serviços de acolhimento já prestados.
Quando, para a proteção de sua integridade física e psicológica, for detectada a necessidade do
afastamento da criança e do adolescente da família de origem pela autoridade competente, os mesmos
deverão ser atendidos em serviços que ofereçam cuidados e condições favoráveis ao seu
desenvolvimento saudável, devendo-se trabalhar no sentido de viabilizar a reintegração à família de
origem ou, na sua impossibilidade, o encaminhamento para família substituta. Tais serviços podem ser
ofertados em diferentes serviços de acolhimento:
I. Abrigos Institucionais;
II. Casas Lares;
III. Famílias Acolhedoras; e
IV. Repúblicas.

A organização dos diferentes serviços de acolhimento tem como objetivo responder de forma mais
adequada às demandas da população infanto-juvenil. A partir da análise da situação familiar, do perfil de
cada criança ou adolescente e de seu processo de desenvolvimento, deve-se indicar qual serviço poderá
responder de forma mais efetiva às suas necessidades. Deve-se considerar, ainda: sua idade; histórico
de vida; aspectos socioculturais; motivos do acolhimento; situação familiar; previsão do menor tempo
necessário para viabilizar soluções de caráter permanente (reintegração familiar ou adoção); condições
emocionais e de desenvolvimento, bem como condições específicas que precisem ser observadas
(crianças e adolescentes com vínculos de parentesco – irmãos, primos, crianças e adolescentes com
diferentes deficiências, que estejam em processo de saída da rua, com histórico de uso, abuso ou
dependência de álcool ou outras drogas, etc), dentre outras.
O órgão gestor da Política de Assistência Social, em parceria com demais atores da rede local e do
Sistema de Garantia de Direitos, deve desenvolver estratégias para o aprimoramento constante da oferta
do atendimento a crianças e adolescentes, visando a melhor adequação às características das demandas
locais. A implantação de serviços de acolhimento deve basear-se em um diagnóstico local que busque
identificar a existência ou não de demanda por tais serviços no município e quais serviços são mais
adequados para seu atendimento. Particularmente nos municípios de grande porte e metrópoles deve
haver diversificação na oferta de diferentes modalidades de atendimento.
Cabe ressaltar que, conforme parâmetros do Plano Nacional de Promoção, Proteção e Defesa do
Direito de Crianças e Adolescentes à Convivência Familiar e comunitária, o financiamento dos serviços
de acolhimento deve basear-se na manutenção de sua capacidade de atendimento e não no número de
vagas ocupadas.
Destaca-se que nenhum novo serviço de acolhimento para crianças e adolescentes deverá ser criado
sem atender aos parâmetros aqui apresentados e que, gradativamente, a infraestrutura dos serviços já
existentes deverá ser adequada para o cumprimento dessas exigências.
A seguir, serão apresentados os parâmetros que deverão orientar a organização dos serviços de
acolhimento para crianças e adolescentes no País.

4.1 Abrigo Institucional

4.1.1 Definição

Serviço que oferece acolhimento provisório para crianças e adolescentes afastados do convívio familiar
por meio de medida protetiva de abrigo (ECA, Art. 101), em função de abandono ou cujas famílias ou
responsáveis encontrem-se temporariamente impossibilitados de cumprir sua função de cuidado e
proteção, até que seja viabilizado o retorno ao convívio com a família de origem ou, na sua
impossibilidade, encaminhamento para família substituta.
O serviço deve ter aspecto semelhante ao de uma residência e estar inserido na comunidade, em
áreas residenciais, oferecendo ambiente acolhedor e condições institucionais para o atendimento com
padrões de dignidade. Deve ofertar atendimento personalizado e em pequenos grupos e favorecer o

Apostila gerada especialmente para: Betânia Mueller 023.712.580-32


. 108
convívio familiar e comunitário das crianças e adolescentes atendidos, bem como a utilização dos
equipamentos e serviços disponíveis na comunidade local.

4.1.2 Público alvo

- Geral
- Crianças e adolescentes de 0 a 18 anos sob medida protetiva de abrigo.
- Especificidades
Devem ser evitadas especializações e atendimentos exclusivos - tais como adotar faixas etárias muito
estreitas, direcionar o atendimento apenas a determinado sexo, atender exclusivamente ou não atender
crianças e adolescentes com deficiência ou que vivam com HIV/AIDS. A atenção especializada, quando
necessária, deverá ser assegurada por meio da articulação com a rede de serviços, a qual poderá
contribuir, inclusive, para capacitação específica dos cuidadores.
O atendimento especializado, quando houver e se justificar pela possibilidade de atenção diferenciada
a vulnerabilidades específicas, não deve prejudicar a convivência de crianças e adolescentes com
vínculos de parentesco (irmãos, primos, etc), nem constituir-se motivo de discriminação ou segregação.
Desta forma, a organização da rede local de serviços de acolhimento deverá garantir que toda criança
ou adolescente que necessite de acolhimento receberá atendimento e que haverá diversificação dos
serviços ofertados, bem como articulação entre as políticas públicas, de modo a proporcionar respostas
efetivas às diferentes demandas dos usuários.
- Número Máximo de Usuários por Equipamento
- 20 crianças e adolescentes

4.1.3 Aspectos físicos

Localização
Áreas residenciais, sem distanciar-se excessivamente, do ponto de vista geográfico e socioeconômico,
da realidade de origem das crianças e adolescentes acolhidos.
- Fachada e aspectos gerais da construção
- Deverá manter aspecto semelhante ao de uma residência, seguindo o padrão arquitetônico das
demais residências da comunidade na qual estiver inserida.
- Não devem ser instaladas placas indicativas da natureza institucional do equipamento, também
devendo ser evitadas nomenclaturas que remetam à aspectos negativos, estigmatizando e
despotencializando os usuários.

4.1.4 Recursos humanos

Para que o atendimento em serviços de abrigo institucional possibilite à criança e ao adolescente


constância e estabilidade na prestação dos cuidados, vinculação com o educador/cuidador de referência
e previsibilidade da organização da rotina diária, os educadores/cuidadores deverão trabalhar,
preferencialmente, em turnos fixos diários, de modo a que o mesmo educador/cuidador desenvolva
sempre determinadas tarefas da rotina diária (p.ex.: preparar café da manhã, almoço, jantar, dar banho,
preparar para a escola, apoiar as tarefas escolares, colocar para dormir, etc.), sendo desaconselhável
esquemas de plantão , caracterizados pela grande alternância na prestação de tais cuidados.
Em se tratando de serviços de acolhimento desenvolvidos por organizações não governamentais, a
equipe técnica deverá pertencer ao quadro de pessoal da entidade ou, excepcionalmente, estar vinculada
ao órgão gestor da Assistência Social ou a outro órgão público ou privado, sendo exclusivamente
destinada para esse fim. Em ambos os casos, deverá ser respeitado o número mínimo de profissionais
necessários, a carga horária mínima e o cumprimento das atribuições elencadas neste documento.

4.2 Casa-Lar

4.2.1 Definição
O Serviço de Acolhimento provisório oferecido em unidades residenciais, nas quais pelo menos uma
pessoa ou casal trabalha como educador/cuidador residente – em uma casa que não é a sua – prestando
cuidados a um grupo de crianças e adolescentes afastados do convívio familiar por meio de medida
protetiva de abrigo (ECA, Art. 101), em função de abandono ou cujas famílias ou responsáveis encontrem-
se temporariamente impossibilitados de cumprir sua função de cuidado e proteção, até que seja

Apostila gerada especialmente para: Betânia Mueller 023.712.580-32


. 109
viabilizado o retorno ao convívio com a família de origem ou, na sua impossibilidade, encaminhamento
para família substituta.
Esse tipo de serviço visa estimular o desenvolvimento de relações mais próximas do ambiente familiar,
promover hábitos e atitudes de autonomia e de interação social com as pessoas da comunidade. Com
estrutura de uma residência privada, deve receber supervisão técnica, localizar-se em áreas residenciais
da cidade e seguir o padrão sócio econômico da comunidade onde estiverem inseridas.
O serviço deve organizar ambiente próximo de uma rotina familiar, proporcionar vínculo estável entre
o educador/cuidador residente e as crianças e adolescentes atendidos, além de favorecer o convívio
familiar e comunitário dos mesmos, bem como a utilização dos equipamentos e serviços disponíveis na
comunidade local, devendo atender a todas as premissas do Estatuto da Criança e do Adolescente,
especialmente no que diz respeito ao fortalecimento dos vínculos familiares e sociais, e oferecimento de
oportunidades para a (re) inserção na família de origem ou substituta.

4.2.2 Público alvo

Geral

Crianças e adolescentes de 0 a 18 anos sob medida protetiva de abrigo.

Especificidades

Devem ser evitadas especializações e atendimentos exclusivos - tais como adotar faixas etárias muito
estreitas, direcionar o atendimento apenas a determinado sexo, atender exclusivamente ou não atender
crianças e adolescentes com deficiência ou que vivam com HIV/AIDS. A atenção especializada, quando
necessária, deverá ser assegurada por meio da articulação com a rede de serviços, a qual poderá
contribuir, inclusive, na capacitação específica dos cuidadores.
Este equipamento é particularmente adequado ao atendimento a grupos de irmãos e a crianças e
adolescentes com perspectiva de acolhimento de média ou longa duração.

Número Máximo de Usuários por Equipamento

10 crianças e adolescentes

4.2.3 Características

A principal diferença entre este serviço e o Abrigo Institucional, além do menor número de crianças e
adolescentes atendidos por equipamento, está na presença do educador/cuidador residente – pessoa ou
casal que reside na casa-lar juntamente com as crianças/adolescentes atendidos, sendo responsável
pelos cuidados e pela organização da rotina da casa.
Tal profissional deve participar ativamente das decisões relacionadas à casa-lar, sendo recomendável
que o mesmo tenha autonomia para gerir a rotina “doméstica”, inclusive as despesas da casa.
Recomenda-se que também as crianças e adolescentes tomem parte nas decisões acerca da rotina da
casa, de modo que os(as) mesmos(as) reconheçam-se como parte integrante do grupo, com direitos e
deveres.
A presença do educador/cuidador residente visa proporcionar:
- estabelecimento de uma relação estável no ambiente institucional, uma vez que o educador/cuidador
residente ocupa um lugar de referência afetiva constante, facilitando o acompanhamento da vida
diária/comunitária das crianças/ adolescentes (reuniões escolares, festas de colegas, etc.),
diferentemente do que ocorre no Abrigo Institucional, onde há maior rotatividade diária de
educadores/cuidadores.
- uma rotina mais flexível na casa, menos institucional e próxima a uma rotina familiar, adaptando-se
às necessidades da criança/adolescente.
Ressalta-se que tal tarefa demanda muito deste educador/cuidador residente, por se tratar de uma
função com elevada exigência psíquica e emocional, o que torna necessária uma atenção especial na
seleção, capacitação e acompanhamento deste profissional. Além disso, é de fundamental importância a
existência de equipe técnica especializada, para acompanhamento constante das casas lares
(apoio/orientação aos cuidadores/educadores, atendimento às crianças/adolescentes e suas famílias,
articulação com o SGD, etc), o que não significa que esta equipe deva estar sediada na casa. Assim, para
que o educador/cuidador consiga cumprir bem sua função é necessário que disponha de apoio e

Apostila gerada especialmente para: Betânia Mueller 023.712.580-32


. 110
orientação por parte da equipe técnica do serviço, bem como de espaço para trocas, onde possa reunir-
se com outros educadores para compartilhar as experiências e desafios decorrentes da atuação e
encontrar soluções conjuntas.
É importante, também, que os/as cuidadores(as)/educadores(as) residentes possuam condições
dignas de trabalho e remuneração condizente, incluindo preservação da privacidade do trabalhador e
possibilidade de construção de projetos pessoais. Para tanto, deve-se prever períodos livres diários e um
esquema de folgas semanais que possibilite sua participação em atividades outras que não as da casa,
além de férias anuais fora do ambiente da Casa-lar.
Especial atenção deve ser dada à clarificação do papel a ser exercido por esse profissional, de modo
a que não se pretenda substituir o lugar e a função dos pais ou da família de origem. O educador/cuidador
residente não deve ocupar o lugar da mãe ou da família de origem, mas contribuir para o fortalecimento
dos vínculos familiares, favorecendo o processo de reintegração familiar ou o encaminhamento para
família substituta, quando for o caso. Assim, recomenda-se a substituição do termo largamente utilizado
“mãe/pai social” por educador/cuidador residente, de modo a evitar ambiguidade de papéis, disputa com
a família de origem, ou fortalecimento da ideia de permanência indefinida da criança/adolescente no
serviço e o investimento insuficiente na reintegração familiar.
Nessa forma de serviço deve-se dar especial atenção ao processo de desligamento das crianças e
adolescentes acolhidos por longos períodos, o qual deverá ocorrer de forma gradual e incluir a
participação de todos os envolvidos.

4.2.4 Aspectos físicos

Localização

Áreas residenciais, sem distanciar-se excessivamente, do ponto de vista geográfico e socioeconômico,


da realidade de origem das crianças e adolescentes acolhidos.
Deve funcionar em uma edificação residencial inserida no território de forma análoga às demais
residências locais, devendo ser evitadas estruturas que agreguem diversas casas-lares em um terreno
comum, visto que tais estruturas acabam por se tornar ambientes “fechados” que dificultam a integração
das crianças/adolescentes ali acolhidos à vizinhança.

Fachada e aspectos gerais da construção

Similar a uma residência unifamiliar, seguindo o padrão arquitetônico das demais residências da
comunidade na qual estiver inserida.
Não devem ser instaladas placas indicativas da natureza institucional do equipamento, também
devendo ser revistas nomenclaturas do serviço que remetam à aspectos negativos, estigmatizando e
despotencializando os usuários.

4.2.5 Recursos humanos

Em se tratando de serviços de acolhimento desenvolvidos por organizações não governamentais, a


equipe técnica deverá pertencer ao quadro de pessoal da entidade ou ser cedida pelo órgão gestor da
Assistência Social ou por outro órgão público ou privado, exclusivamente para esse fim. Em ambos os
casos, deverá ser respeitado o número mínimo de profissionais necessários, a carga horária mínima e o
cumprimento das atribuições elencadas neste documento.

4.3 Serviço de Acolhimento em Família Acolhedora

4.3.1 Definição
que organiza o acolhimento, em residências de famílias acolhedoras cadastradas, de crianças e
adolescentes afastados do convívio familiar por meio de medida protetiva (ECA, Art. 101), em função de
abandono ou cujas famílias ou responsáveis encontrem-se temporariamente impossibilitados de cumprir
sua função de cuidado e proteção, até que seja viabilizado o retorno ao convívio com a família de origem
ou, na sua impossibilidade, encaminhamento para adoção. Propicia o atendimento em ambiente familiar,
garantindo atenção individualizada e convivência comunitária, permitindo a continuidade da socialização
da criança/adolescente.

Apostila gerada especialmente para: Betânia Mueller 023.712.580-32


. 111
Embora ainda pouco difundida no País, esse serviço encontra-se consolidado em outros países,
especialmente nos europeus e da América do Norte, além de contar com experiências exitosas no Brasil
e América Latina. Tal serviço encontra-se contemplado, expressamente, na Política Nacional de
Assistência Social (2004), como um dos serviços de proteção social especial de alta complexidade e no
Plano Nacional de Promoção, Proteção e Defesa de Direitos de Crianças e Adolescentes à Convivência
Familiar e Comunitária (2006).
Do ponto de vista legal, assim como os serviços de acolhimento institucional, o Serviços de
Acolhimento em Família Acolhedora deve organizar-se segundo os princípios e diretrizes do Estatuto da
Criança e do Adolescente, especialmente no que se refere à excepcionalidade e à provisoriedade do
acolhimento; ao investimento na reintegração à família de origem, nuclear ou extensa; à preservação da
convivência e do vínculo afetivo entre grupos de irmãos; a permanente articulação com a Justiça da
Infância e da Juventude e a rede de serviços.
Trata-se de um serviço de acolhimento provisório, até que seja viabilizada uma solução de caráter
permanente para a criança ou adolescente – reintegração familiar ou, excepcionalmente, adoção. É uma
modalidade de acolhimento diferenciada, que não se enquadra no conceito de abrigo em entidade, nem
no de colocação em família substituta, no sentido estrito, porém podendo ser entendido como regime de
colocação familiar preconizado no artigo 90 do Estatuto da Criança e do Adolescente.

4.3.2 Público alvo

Geral

Crianças e adolescentes de 0 a 18 anos, que estão em medida protetiva.

Especificidades
Este serviço de acolhimento é particularmente adequado ao atendimento de crianças e adolescentes
cuja avaliação da equipe técnica do programa e dos serviços da rede de atendimento indique
possibilidade de retorno à família de origem, ampliada ou extensa, salvo casos emergenciais, nos quais
inexistam alternativas de acolhimento e proteção.
Para as crianças pequenas que vivenciam situações de violação de direitos, o acolhimento familiar tem
se mostrado uma forma de atendimento adequada a suas especificidades.

Número Máximo de Crianças e Adolescentes Acolhidos

Cada família acolhedora deverá acolher uma criança/adolescente por vez, exceto quando se tratar de
grupo de irmãos, quando esse número poderá ser ampliado. Neste último caso, em se tratando de grupo
de mais de dois irmãos, deverá haver uma avaliação técnica para verificar se o acolhimento em família
acolhedora é a melhor alternativa para o caso, ou se seria mais adequado o acolhimento em outra
modalidade de serviço, como Casa–lar, por exemplo. A decisão fica a critério da avaliação da equipe
técnica do programa, como também da disponibilidade da família em acolher.

4.3.3 Aspectos jurídico-administrativos

As famílias acolhedoras são selecionadas, capacitadas e acompanhadas pela equipe técnica do


Serviço de Acolhimento para que possam acolher crianças ou adolescentes em medida de proteção
aplicada por autoridade competente, a qual encaminha a criança/adolescente para inclusão nesse
serviço, competindo ao mesmo a indicação da família que esteja disponível e em condições para acolhê-
lo.
Dentro da sistemática jurídica, este tipo de acolhimento é feito por meio de um termo de guarda
provisória, solicitado pelo serviço de acolhimento e emitido pela autoridade judiciária para a família
acolhedora previamente cadastrada. A guarda será deferida para a família acolhedora indicada pelo
serviço, terá sempre o caráter provisório e sua manutenção deve estar vinculada à permanência da família
acolhedora no serviço. O termo de guarda deve ser expedido imediatamente à aplicação da medida
protetiva e início do acolhimento.

4.3.4 Funcionamento do serviço de acolhimento em família acolhedora

Divulgação, Seleção, Preparação e Acompanhamento das Famílias Acolhedoras

Apostila gerada especialmente para: Betânia Mueller 023.712.580-32


. 112
Um processo de seleção e capacitação criterioso é essencial para a obtenção de famílias acolhedoras
com perfil adequado ao desenvolvimento de suas funções, possibilitando a oferta de um serviço de
qualidade aos usuários. Para tanto, deve-se prever, minimamente, os seguintes passos:
- Ampla Divulgação: com informações precisas sobre os objetivos e a operacionalização do Serviço,
perfil dos usuários, critérios mínimos para se tornar família acolhedora, dentre outros. A sensibilização de
famílias para a participação do serviço como famílias acolhedoras requer uma estratégia de divulgação
permanente, realizada, em conjunto pelo executor e pelo órgão do Governo Municipal competente, que
privilegie a clareza dos objetivos dessa modalidade de atendimento, que não deve ser confundida com
adoção. O processo de divulgação também envolve a sensibilização de outros atores do Sistema de
Garantia de Direitos para que possam se estabelecer parcerias de trabalho.
- Acolhida e avaliação inicial: Deve ser realizada por equipe técnica multidisciplinar, qualificada e
disponível para prestar os esclarecimentos necessários às famílias interessadas, de modo individual e/ou
em grupos de familiares. Este primeiro momento de interlocução possibilita, inclusive, a identificação de
possíveis motivações equivocadas – como interesse em adoção. Esse é o momento em que as
informações devem ser claras e objetivas, de modo a evitar mal-entendidos e poupar tempo e
envolvimento emocional da equipe e dos pretendentes ao acolhimento. Deve também ser verificado se
as famílias atendem aos critérios mínimos exigidos para a função, inclusive em relação ao desejo,
disponibilidade e concordância de todos os membros do núcleo familiar em acolher e participar dos
encontros de seleção, capacitação e acompanhamento.
- Avaliação Documental: Documentação mínima a ser exigida constitui em documentos pessoais (RG,
CPF), comprovante de residência, comprovante de rendimentos, certidão negativa de antecedentes
criminais, atestado de saúde física e mental. Os documentos devem ser solicitados a todos os membros
maiores de idade do núcleo familiar. Em se tratando de casal, é indicado que o termo de guarda seja
expedido em nome de ambos. Os responsáveis pelo acolhimento não devem ter qualquer problema em
sua documentação. Quanto aos outros membros da família, a equipe técnica do programa deverá avaliar
cada situação.
- Seleção: Após a avaliação inicial, as famílias inscritas como potenciais acolhedoras deverão passar
por um estudo psicossocial, com o objetivo de identificar os aspectos subjetivos que qualificam ou não a
família para sua participação. Essa etapa deverá envolver entrevistas individuais e coletivas, dinâmicas
de grupo e visitas domiciliares, sempre utilizando metodologias que privilegiem a coparticipação das
famílias, em um processo que inclua a reflexão e autoavaliação das mesmas. É essencial que todo o
grupo familiar participe do processo de avaliação e seleção, uma vez que todos os componentes do
núcleo familiar devem estar de acordo e serem compatíveis com a proposta. Algumas características a
serem observadas são:
- disponibilidade afetiva e emocional;
- padrão saudável das relações de apego e desapego;
- relações familiares e comunitárias;
- rotina familiar;
- não envolvimento de nenhum membro da família com dependência química; espaço e condições
gerais da residência; motivação para a função; aptidão para o cuidado com crianças e adolescentes;
capacidade de lidar com separação;
- flexibilidade; tolerância; proatividade; capacidade de escuta; estabilidade emocional;
- capacidade de pedir ajuda e de colaborar com a equipe técnica, dentre outras.
Além da avaliação quanto à compatibilidade com a função de acolhimento, o estudo psicossocial
realizado pela equipe técnica deverá indicar, também, o perfil de criança e/ou adolescente que cada
família está habilitada a acolher. É importante nesse processo, ouvir a opinião da família quanto a este
aspecto, ainda que durante o processo de capacitação essa avaliação possa modificar-se.
- Capacitação: as famílias selecionadas deverão participar de processo de capacitação. Tal processo
deve ser desenvolvido com metodologia participativa, de modo dinâmico, por meio de oficinas e
seminários, que podem ser conduzidos pelos profissionais da equipe do Serviço e por especialistas
convidados (outros profissionais da rede, do Sistema de Justiça, etc). Também é bastante recomendável
que, durante o processo de capacitação, sejam feitas apresentações de experiências de famílias
acolhedoras que já vivenciaram o acolhimento, assim como de famílias de origem cujas
crianças/adolescentes foram acolhidos pelo serviço e já retornaram ao lar, de modo a dar concretude à
proposta.
Alguns temas relevantes a serem trabalhados em uma capacitação inicial são:
- Operacionalização jurídico-administrativa do serviço e particularidades do mesmo;
- Direitos da criança e do adolescente;
- Novas configurações familiares e realidade das famílias em situação de vulnerabilidade social;

Apostila gerada especialmente para: Betânia Mueller 023.712.580-32


. 113
- Etapas do desenvolvimento da criança e do adolescente (características, desafios, comportamentos
típicos, fortalecimento da autonomia, desenvolvimento da sexualidade); brincadeiras e jogos adequados
para cada faixa etária, exploração do ambiente, formas de lidar com conflitos, colocação de limites, etc.);
- Comportamentos frequentemente observados entre crianças/ adolescentes separados da família de
origem, que sofreram abandono, violência, etc;
- Práticas educativas; como ajudar a criança/adolescente a conhecer e a lidar com sentimentos,
fortalecer a autoestima e contribuir para a construção da identidade;
- Políticas públicas, direitos humanos e de cidadania;
- Papel da família acolhedora, da equipe técnica do programa e da família de origem.
Cadastramento: As famílias que forem consideradas aptas a serem acolhedoras deverão formalizar
sua inscrição no Serviço, com o preenchimento da ficha de cadastro, onde constam os documentos
necessários (já citados no item Avaliação Documental), informações sobre toda a família e indicação
quanto ao perfil de criança/ adolescente que se julga capaz de acolher. A documentação necessária
deverá ser encaminhada pela coordenação do Serviço à Justiça da Infância e Juventude, para que possa
ser emitido, com presteza, o termo de guarda e responsabilidade quando ocorrer o acolhimento de uma
criança/adolescente pela família cadastrada.
Acompanhamento: Os serviços da rede de proteção à infância e juventude, especialmente o
requerente do ingresso da criança no programa (Justiça da Infância e Juventude, Conselho Tutelar,
Equipe de Supervisão e Apoio aos Serviços de Acolhimento, etc.), devem iniciar discussão do caso com
a equipe técnica, antes que a modalidade acolhimento familiar seja a opção de proteção decidida.
Objetiva-se com isso traçar um trabalho em rede e de continuidade ao atendimento à criança e sua família.

Preparação para o Acolhimento e Acompanhamento

A partir do momento em que uma criança/adolescente for encaminhada para o serviço, a equipe
técnica deve iniciar a preparação e acompanhamento psicossocial da criança/adolescente, da família
acolhedora, da família de origem e da rede social de apoio. Isso poderá ocorrer por meio de ações
específicas tais como:

Com a criança/adolescente:
Preparação da criança/adolescente para a entrada no programa, buscando-se estabelecer um vínculo
de confiança, fornecendo explicação da situação e esclarecimentos quanto ao acolhimento familiar. Essa
ação deve ser partilhada com o órgão que encaminhou a criança ou adolescente.
Aproximação supervisionada entre a criança/adolescente e a família acolhedora.
Escuta individual da criança/adolescente, com foco na adaptação à família acolhedora.
Acompanhamento do desempenho escolar da criança e sua situação de saúde.
Viabilização de encontro semanal entre a família de origem e a criança e/ou adolescente, o qual deverá
ser acompanhado pela equipe técnica.

Com a família acolhedora:


Preparação da família acolhedora para a recepção da criança/adolescente, inclusive informando a
situação sociojurídica do caso e, quando possível, previsão inicial do tempo de acolhimento.
Aproximação supervisionada entre a criança/adolescente e a família acolhedora.
Construção de um plano de acompanhamento da família acolhedora, em conformidade com as
necessidades do acolhimento de cada criança/adolescente, respeitando-se as características das famílias
e do acolhido.
Acompanhamento da família acolhedora, com entrevistas e visitas domiciliares com foco na adaptação
e desenvolvimento do acolhimento, com frequência mínima quinzenal ou de acordo com a avaliação do
caso.
Construção de espaço para troca de experiências entre famílias acolhedoras (Ex.: grupos de apoio, de
escuta mútua).

Com a família de origem:


Contato inicial com a família de origem (salvo em situações de restrição judicial) para esclarecimento
do que é o acolhimento familiar, seus termos e regras, assim como para convidá-la a participar do
processo de adaptação da criança/adolescente na família acolhedora, fornecendo informações sobre
seus hábitos e costumes. Se possível, possibilitar o encontro da família de origem com seu filho(a).
Acompanhamento da família de origem, com entrevistas e visitas domiciliares periódicas, articuladas
com o planejamento realizado para superação das vulnerabilidades da família.

Apostila gerada especialmente para: Betânia Mueller 023.712.580-32


. 114
Construção de espaço para troca de experiências entre famílias de origem (Ex.: grupos de apoio, de
escuta mútua).

Outras atribuições da equipe técnica do programa:

Construir com a participação da família de origem e serviços da rede de proteção um plano de


acompanhamento da família de origem, nuclear ou extensa, que objetive a superação dos motivos que
levaram à necessidade do afastamento da criança/adolescente e conseqüente reintegração familiar.
Providenciar encaminhamentos jurídico-administrativos e junto à rede de educação, saúde, dentre
outros que se fizerem necessários.
Possibilitar situações de escuta individual, ao longo de todo o tempo de acolhimento, de qualquer dos
envolvidos (família de origem, família acolhedora e acolhido).

Atribuições das Famílias Acolhedoras:


Preservar o vínculo e convivência entre irmãos e parentes (primos, sobrinhos) quando o acolhimento
for realizado por famílias diferentes.
Responsabilizar-se pelas atividades cotidianas e rotineiras dos acolhidos (levar à escola,
atendimentos de saúde etc), cabendo à equipe técnica auxiliar as famílias acolhedoras na obtenção
destes atendimentos, preferencialmente na rede pública.
Comunicação à equipe do serviço todas as situações de enfrentamento de dificuldades que observem
durante o acolhimento, seja sobre a criança, seja sobre a própria família acolhedora e a família de origem.

Desligamento da criança/adolescente

O desligamento do programa ocorrerá quando for avaliado pela equipe de profissionais do serviço, em
diálogo com a Justiça da Infância e Juventude, com o Ministério Público, Conselho Tutelar e rede
envolvida - a possibilidade de retorno familiar (à família de origem, nuclear ou extensa); a necessidade
de acolhimento em outro espaço de proteção, ou o encaminhamento para adoção. A esta avaliação deve
suceder a preparação e o apoio específico por parte da equipe técnica, com ações:

Com a criança/adolescente:
Escuta individual e apoio emocional à criança/adolescente, com foco no retorno à família de origem e
separação da família acolhedora.

Com a família de origem:


Intensificar e ampliar, de forma progressiva, os encontros entre a criança/adolescente e sua família -
que gradativamente deverão deixar de ser acompanhados pela equipe, a permanência com a família nos
finais de semana e, por fim, o retorno definitivo.
Dar continuidade ao acompanhamento à família de origem após a reintegração da
criança/adolescente, por um período mínimo de seis meses, de forma a lhe dar suporte para o
cumprimento de suas funções de cuidado e proteção, buscando sua autonomia e visando evitar a
reincidência da necessidade de acolhimento. Conforme a estrutura local, tal acompanhamento poderá
ser feito pela equipe técnica do serviço de famílias acolhedoras que acompanhou o acolhimento ou por
outro serviço socioassistencial (CRAS, CREAS) em articulação com a rede local.

Com a família acolhedora:


Orientar a família acolhedora para intensificar a preparação da criança/adolescente para o retorno à
família de origem.
Realizar encontros com a família acolhedora (entrevistas individuais e com o grupo familiar), com foco
na saída da criança/adolescente e na experiência de separação, oferecendo apoio psicossocial após a
saída do(a) acolhido(a) manutenção das atividades em grupo com outras famílias acolhedoras e do
contato regular com a equipe técnica.
Intermediar e orientar a família acolhedora com relação à manutenção de vínculos com a
criança/adolescente e sua família após a reintegração familiar, o que também amplia a proteção da
criança/adolescente acolhido. Entretanto, deve ser respeitado o desejo de todos os envolvidos, além de
serem consideradas as características de cada caso, avaliando-se a pertinência ou não da manutenção
desde contato.
No caso em que forem esgotadas todas as possibilidades de reintegração familiar e a
criança/adolescente for encaminhada para adoção, a família acolhedora deverá contribuir para essa

Apostila gerada especialmente para: Betânia Mueller 023.712.580-32


. 115
transição e, em conjunto com a equipe técnica do serviço, preparar esta criança para a colocação em
uma família definitiva.
O desligamento do programa deve ocorrer mediante conhecimento e autorização da Justiça da Infância
e Juventude, que deve estar devidamente informado das ações do serviço e atuar em conjunto com estas.

4.3.5 Recursos humanos

Em se tratando de serviços de acolhimento desenvolvidos por organizações não governamentais, a


equipe técnica deverá pertencer ao quadro de pessoal da entidade ou ser cedida pelo órgão gestor da
Assistência Social ou por outro órgão público ou privado, exclusivamente para esse fim. Em ambos os
casos, deverá ser respeitado o número mínimo de profissionais necessários, a carga horária mínima e o
cumprimento das atribuições abaixo elencadas.

4.4 República

4.4.1 Definição
Serviço de acolhimento que oferece apoio e moradia subsidiada a grupos de jovens em situação de
vulnerabilidade e risco pessoal e social; com vínculos familiares rompidos ou extremamente fragilizados;
em processo de desligamento de instituições de acolhimento, que não tenham possibilidade de retorno à
família de origem ou de colocação em família substituta e que não possuam meios para autossustentação.
Com a estrutura de uma residência privada, deve receber supervisão técnica e localizar-se em áreas
residenciais da cidade, seguindo o padrão socioeconômico da comunidade onde estiverem inseridas, sem
distanciar-se excessivamente, do ponto de vista socioeconômico, da comunidade de origem dos usuários.
A república oferece atendimento durante o processo de construção de autonomia pessoal e possibilita
o desenvolvimento de auto-gestão, autossustentação e independência. Possui tempo de permanência
limitado, podendo ser reavaliado e prorrogado em função do projeto individual formulado em conjunto
com o profissional de referência.

4.4.2 Público alvo

Geral

Jovens entre 18 e 21 anos em situação de vulnerabilidade e risco pessoal e social, com vínculos
familiares rompidos ou extremamente fragilizados e que não possuam meios para autossustentação. Tal
serviço é particularmente indicado para o acolhimento de jovens em processo de desligamento de
serviços de acolhimento para crianças e adolescentes por terem completado a maioridade, porém que
ainda não tenham conquistado a autonomia, podendo também destinar-se a outros jovens que
necessitem do serviço.

Especificidades

As repúblicas devem ser organizadas em unidades femininas e unidades masculinas, garantindo-se,


na rede, o atendimento a ambos os sexos, conforme demanda local, devendo ser dada a devida atenção
à perspectiva de gênero no planejamento políticopedagógico do serviço, inclusive no que tange aos
direitos sexuais e reprodutivos e à proteção à maternidade.
Especial atenção deve ser dada à escolha dos componentes de cada república, a qual deverá ser feita
por equipe técnica capacitada, devendo ser levados em consideração aspectos como perfil, demandas
específicas e grau de autonomia de cada usuário, bem como o grau de afinidade entre os mesmos.
Quando um novo jovem vier a integrar uma república, a equipe técnica do serviço deverá prepará-lo e
aos demais jovens da república, de modo a facilitar sua inserção e integração ao ambiente.
Sempre que possível e recomendável, os jovens deverão ter participação ativa na escolha dos colegas
de república, de modo a que, na composição dos grupos, sejam respeitadas afinidades e vínculos
previamente construídos.
Deverá ser disponibilizado na rede número suficiente de repúblicas localizadas em edificações que
respeitem as normas de acessibilidade, de maneira a possibilitar o atendimento integrado a jovens com
deficiência.

Apostila gerada especialmente para: Betânia Mueller 023.712.580-32


. 116
Número Máximo de Usuários por Equipamento

6 (seis) jovens

4.4.3 Características

Os custos da locação do imóvel - no caso de imóvel alugado - e tarifas podem ser subsidiados e
gradativamente assumidos pelos jovens. As demais despesas podem ser cotizadas entre os moradores,
com subsídio quando necessário.
O grupo deve contar com supervisão técnico-profissional para a gestão coletiva da moradia (regras de
convívio, atividades domésticas cotidianas, gerenciamento de despesas, etc.), orientação e
encaminhamento para outros serviços, programas ou benefícios da rede socioassistencial e das demais
políticas públicas, em especial programas de profissionalização, inserção no mercado de trabalho,
habitação e inclusão produtiva. O apoio técnico também é essencial na organização de espaços de escuta
e construção de soluções coletivas por parte dos(as) jovens para as questões que lhes são próprias, na
construção de projetos de vida, no incentivo ao estabelecimento de vínculos comunitários fortes e na
participação nas instâncias de controle social e espaços de participação social.
Caso desejem, os(as) jovens devem ter acesso a todas as informações que lhes digam respeito que
estiverem disponíveis nas instituições que lhes prestaram atendimento durante a infância e adolescência
(Ex.: prontuários e documentos contendo informações sobre sua história de vida, possíveis familiares,
situação familiar e motivos do abrigamento). O acesso a essas informações deverá respeitar o processo
individual de apropriação da história de vida, devendo ser conduzido por profissionais orientados e
preparados.
O processo de transição do jovem do serviço de acolhimento para crianças e adolescentes para o
serviço de acolhimento em república deve desenvolver-se de modo gradativo, com a participação ativa
do mesmo no planejamento das fases subsequentes. Transições dessa natureza devem sempre ser
planejadas o mais cedo possível.
Atenção especial deve ser dada a adolescentes atendidos em serviços de acolhimento, sobretudo
àqueles cujas possibilidades de reintegração à família de origem foram esgotadas e têm reduzidas
possibilidades de colocação em família substituta. O atendimento, nesses casos, deve perseverar no
apoio ao fortalecimento dos vínculos comunitários, na qualificação profissional e na construção do projeto
de vida, bem como estar fundamentado em metodologia participativa que favoreça o exercício de seu
protagonismo.
Ações devem ser desenvolvidas visando o fortalecimento de habilidades, aptidões, capacidades e
competências dos adolescentes, que promovam gradativamente sua autonomia, de forma a que,
preferencialmente, já estejam exercendo alguma atividade remunerada quando da sua transferência para
uma república.
Para tanto, deve-se viabilizar o acesso a:
I. Programas, projetos e serviços nos quais possam desenvolver atividades culturais, artísticas e
esportivas que propiciem a vivência de experiências positivas e favorecedoras de sua autoestima;
II. Programas de aceleração da aprendizagem, para os casos de grande distorção série – idade; e
III. Cursos profissionalizantes e programas de inserção gradativa no mercado de trabalho (como
estágios, programas de adolescente aprendiz, etc.), sempre se respeitando seus interesses e habilidades.

4.4.4 Aspectos físicos

Localização

Áreas residenciais, sem distanciar-se excessivamente, do ponto de vista socioeconômico, da realidade


de origem dos usuários.
Fachada e aspectos gerais da construção

O serviço deve funcionar em construção destinada ao uso residencial, seguindo o padrão arquitetônico
das demais residências da comunidade na qual estiver inserida. Deve-se evitar a instalação de placas
indicativas da natureza institucional do equipamento.

4.4.5 Recursos humanos


Em se tratando de serviços de acolhimento desenvolvidos por organizações não governamentais, a
equipe técnica deverá pertencer ao quadro de pessoal da entidade ou ser cedida pelo órgão gestor da

Apostila gerada especialmente para: Betânia Mueller 023.712.580-32


. 117
Assistência Social ou por outro órgão público ou privado, exclusivamente para esse fim. Em ambos os
casos, deverá ser respeitado o número mínimo de profissionais necessários, a carga horária mínima e o
cumprimento das atribuições elencadas neste documento.

4.5 Regionalização do Atendimento nos Serviços de Acolhimento

Excepcionalmente pode ser necessária e justificável a regionalização do atendimento nos serviços de


acolhimento de crianças e adolescentes afastados do convívio familiar ou de Repúblicas para jovens.
Esta situação pode ser observada tanto no caso de municípios de pequeno porte - cuja demanda e
condições de gestão dificultem a implementação de serviços locais - quanto no caso do atendimento a
crianças e adolescentes ameaçados de morte, situação na qual o atendimento em serviço de acolhimento
localizado próximo ao contexto familiar e comunitário de origem pode representar risco à segurança da
criança ou adolescente ameaçado.
Serão abordados neste item estratégias e orientações para a organização de serviços de acolhimento
nas situações acima mencionadas.

4.5.1 Serviços de acolhimento para municípios de pequeno porte com compartilhamento de equipe

No caso de municípios de pequeno porte, as alternativas para a implementação especificamente de


serviços de acolhimento para crianças e adolescentes devem ser consideradas na seguinte ordem:

4.5.1.1 Implementação de serviço local

O município deve avaliar inicialmente as possibilidades de implementar, na localidade, um Serviço de


Acolhimento em Família Acolhedora ou uma Casa-lar, em conformidade com os parâmetros contidos nos
Parâmetros de Funcionamento deste documento. Nestes casos, pode-se, inclusive, avaliar a necessidade
de redução da carga horária mínima indicada à jornada diária de trabalho da equipe técnica e,
excepcionalmente, a redução da equipe técnica para um profissional, desde que este possa contar com
apoio regular de outro profissional da rede local, inclusive com papel e atribuições definidas em relação
ao serviço.

4.5.1.2 Implementação de serviço com compartilhamento de equipe

Esgotadas as possibilidades de implementação de serviço de acolhimento conforme recomendação


do item acima, deve-se considerar, nesta ordem, a implementação de Serviço de Acolhimento em Família
Acolhedora ou Casa-lar, com compartilhamento de equipe (coordenador e equipe técnica).
No caso de municípios de pequeno porte que, apesar da necessidade, apresentem dificuldades para
implementar e manter serviços de acolhimento para crianças e adolescentes - em virtude da pequena
demanda e das condições de gestão - pode-se recorrer ao compartilhamento dos seguintes profissionais:
coordenador e equipe técnica. O compartilhamento dessa equipe constitui estratégia para assegurar o
atendimento da criança e do adolescente próximo à sua comunidade de origem, de modo a evitar seu
acolhimento em serviços localizados nas capitais dos estados ou em municípios muito distantes de seu
contexto de moradia e de sua família. Em hipótese alguma a utilização de equipes compartilhadas poderá
implicar a precarização do serviço oferecido, o qual deverá atender aos demais parâmetros contidos neste
documento.
Os serviços de acolhimento com compartilhamento de equipe podem ser implementados sob gestão
estadual ou como consórcios entre municípios, desde que disponham de coordenação e equipe técnica
suficiente para o atendimento a mais de um município. Destaca-se que, a despeito da possibilidade de
compartilhamento da equipe entre municípios, o ambiente de acolhimento (casa-lar ou residência da
família acolhedora) deverá estar localizado em cada um deles. Exceção a esta exigência poderá ser feita,
apenas, no caso de Repúblicas que, por atenderem jovens com mais de 18 anos e, consequentemente,
com maior autonomia de locomoção, poderão estar localizadas no município-sede e atender jovens dos
municípios circunvizinhos.
Cabe ressaltar que a implementação de serviços de acolhimento com compartilhamento de equipe,
deve ser acompanhada do investimento efetivo, em cada um dos municípios, em estratégias preventivas
ao afastamento do convívio familiar, fortalecimento dos serviços necessários para o acompanhamento
das famílias de origem e para a reintegração familiar ou, na sua impossibilidade, encaminhamento para
família substituta.

Apostila gerada especialmente para: Betânia Mueller 023.712.580-32


. 118
Destaca-se que, nos casos de compartilhamento de equipe, tanto a coordenação quanto a equipe
técnica deverá ser destinada exclusivamente para esta finalidade, devendo-se atender aos parâmetros
deste documento no que diz respeito ao quantitativo de profissionais em relação ao número de
crianças/adolescentes ou jovens atendidos, perfil, carga horária mínima recomendada e ao cumprimento
das atribuições elencadas neste documento.
A estratégia de compartilhamento de equipe exigirá a previsão de veículos e combustível suficientes,
de modo a permitir o deslocamento da equipe técnica do município-sede para os demais os municípios
atendidos, com periodicidade mínima semanal, de modo a possibilitar o desenvolvimento de suas ações
no que diz respeito ao apoio, capacitação e acompanhamento dos educadores/cuidadores e/ou famílias
acolhedoras; acompanhamento psicossocial das crianças e adolescentes atendidos e de suas famílias
de origem; articulação com a rede se serviços e o SGD; e o exercício das demais atribuições que lhe
sejam próprias. Também devem ser previstos meios suficientes ao deslocamento eventual dos
educadores/cuidadores e/ou famílias acolhedoras, para eventos de capacitação e formação continuada,
que porventura sejam desenvolvidos fora do seu município.
De acordo com os aspectos acima mencionados, devem ser observadas, ainda, as seguintes
orientações na implementação de serviços de acolhimento com equipes compartilhadas (coordenador e
equipe técnica):
Serviço de Família Acolhedora: Neste caso, o serviço deve ter famílias cadastradas em todos os
municípios atendidos, de modo a viabilizar o acolhimento da criança ou adolescente no seu próprio
município de origem. A equipe compartilhada entre os municípios – coordenador e equipe técnica -
deverá atender a todas as atribuições que lhe são próprias, destacando-se a articulação intersetorial
necessária à implementação do serviço, a divulgação do mesmo, a mobilização, seleção, capacitação e
acompanhamento das famílias acolhedoras, a elaboração dos Planos de Atendimento Individual e
Familiar e dos relatórios a serem enviados à autoridade judicial, o acompanhamento das crianças e
adolescentes acolhidos e de suas famílias e os encaminhamentos necessários para possibilitar a
reintegração familiar ou, na sua impossibilidade, o encaminhamento para família substituta.
Casa-Lar: Neste caso, pode-se compartilhar a equipe compartilhada entre os municípios –
coordenador e equipe técnica - a qual deverá atender a todas as atribuições que lhe são próprias,
destacando-se a articulação intersetorial, a seleção, a capacitação e acompanhamento dos
educadores/cuidadores residentes e seus auxiliares, o acompanhamento das crianças e adolescentes
acolhidos e de suas famílias, os encaminhamentos necessários para viabilizar a reintegração familiar ou,
na sua impossibilidade, o encaminhamento para família substituta. Ressalte-se que cada um dos
municípios atendidos deverá dispor de uma casa-lar implantada em seu território, de modo a garantir o
acolhimento das crianças e adolescentes em seu próprio município. O número de educadores/cuidadores
por criança e adolescente, assim como os demais aspectos do serviço, deverão obedecer aos parâmetros
deste documento que trata dessa modalidade de serviço. No caso do município possuir uma demanda
por acolhimento muito pequena, permite-se a implementação de uma casalar com reduzida capacidade
de atendimento – de até 6 crianças e adolescentes. Excepcionalmente neste caso, o serviço poderá
contar com a presença de apenas um educador/cuidador residente, dispensando-se a presença
permanente do auxiliar de educador/cuidador.
Repúblicas: Neste caso, a equipe compartilhada entre os municípios – coordenador e equipe técnica
- deverá atender a todas as atribuições que lhe são próprias, destacando-se a articulação intersetorial, o
preparo dos(as) jovens para a inserção no serviço e seu posterior acompanhamento psicossocial, o apoio
à gestão coletiva da moradia (regras de convívio, atividades domésticas cotidianas, gerenciamento de
despesas, etc.) e o encaminhamento dos(as) jovens para os serviços da rede. De acordo com a avaliação
técnica que indicar a melhor opção, levando-se em consideração a realidade, demandas e recursos
locais, as unidades de República poderão estar localizadas em cada um dos municípios que
compartilharem a equipe, ou no município-sede do serviço. Neste último caso, deverão ser assegurados
meios necessários à locomoção do(a) jovem ao seu município de origem, periodicamente, de modo a
manter laços de convivência ali estabelecidos.

4.5.1.3 Implementação de casa-lar regionalizada

Deve-se, excepcionalmente, recorrer a esta alternativa quando nenhuma das anteriores for de possível
implementação. Neste caso, devem ser observados, obrigatoriamente, os critérios estabelecidos para o
compartilhamento de equipe (coordenador e equipe técnica), além de asseguradas as condições para o
deslocamento semanal, tanto das famílias para o município onde se localizar a Casalar, quanto das
crianças e adolescentes para o município de residência da família de origem, de modo a favorecer o
processo de reintegração familiar.

Apostila gerada especialmente para: Betânia Mueller 023.712.580-32


. 119
4.5.2 Serviços de acolhimento para crianças e adolescentes ameaçados de morte

Nos casos de crianças e adolescentes ameaçados de morte, sua manutenção no contexto familiar e
comunitário de origem pode representar sério risco a sua segurança. Trata-se de uma situação
particularmente delicada, na qual pode ser necessário o encaminhamento para serviço de acolhimento
em localidade distinta do município de residência habitual. Nestes casos, é preciso considerar que a
proximidade do serviço de acolhimento com a comunidade de origem, a manutenção das atividades
rotineiramente desenvolvidas e o convívio com sua rede social local – parâmetros que devem orientar os
serviços de acolhimento em geral – não são aconselháveis, por colocarem em risco a segurança da
criança ou adolescente ameaçado podendo também representar risco para as demais crianças e
adolescentes atendidos no mesmo serviço.
Dessa forma, sugere-se, para a operacionalização destes serviços, as seguintes estratégias:
Em estados / regiões onde houver número significativo de crianças e adolescentes ameaçados de
morte: podem ser organizados abrigos regionais ou estaduais destinados especificamente a esta
finalidade, os quais, em virtude das ameaças vividas pelas crianças e adolescentes atendidos, devem ser
implementados em locais que garantam o sigilo quanto à sua existência, necessário à proteção dos
acolhidos. Tais serviços deverão atender os parâmetros referentes a recursos humanos e infraestrutura
física constantes neste documento, devendo ser avaliada, ainda, de acordo com a realidade local, a
necessidade de disponibilização de profissional específico, que possa se responsabilizar por uma
articulação mais próxima do serviço com o Sistema de Justiça, o Sistema de Segurança Pública e
programas de proteção aos quais as crianças e adolescentes atendidos estejam vinculados. Em virtude
das particularidades desta situação recomenda-se o atendimento de crianças e adolescentes ameaçados
de morte em serviços regionalizados, organizados especificamente para esta finalidade e com maiores
condições de garantir proteção nestes casos.
Em estados / regiões onde o número de casos de crianças e adolescentes ameaçados de morte que
necessitem de acolhimento não justificar a implementação de serviços estaduais / regionais: podem ser
firmados acordos formais entre municípios de diferentes regiões, a fim de viabilizar a transferência da
crianças ou adolescente ameaçado para outro município, de modo a possibilitar seu acolhimento em
serviços distantes de sua comunidade de origem e, assim, facilitar a sua proteção. Nestas situações o
serviço deve também manter articulação com programas de proteção aos quais as crianças e
adolescentes atendidos estejam vinculados, além do Sistema de Justiça e do Sistema de Segurança
Pública, de modo a propiciar condições de segurança tanto para a criança ou adolescente ameaçado
quanto para os demais ali acolhidos.
Em todos os casos, recomenda-se que os serviços de acolhimento que atendam crianças e
adolescentes ameaçados de morte atuem em articulação com programas específicos de proteção, como
o Programa de Proteção à Criança e ao Adolescente Ameaçado de Morte – PPCAM.
Finalmente, ressalta-se que o encaminhamento da criança ou adolescente ameaçado de morte para
serviço de acolhimento deve ser considerado apenas quando esgotadas outras alternativas que
preservem seus vínculos familiares, como, por exemplo, a mudança de contexto ou cidade acompanhado
da família, de familiar ou responsável.

Desacolhimento7

O desacolhimento ocorrerá quando houver segurança de que a criança não mais será exposta à risco
e a vulnerabilidades. Isso se dá nas seguintes hipóteses: retorno à família de origem, mediante guarda
ou tutela a terceiros com vista à referida família, mediante guarda ou tutela ou em colocação em família
substituta com vistas à adoção - nessas situações, não se caracterizando como reintegração familiar
(Cintra et al., 2007). Em todos os casos, deverá ser expedida uma Guia de Desacolhimento pela
autoridade judiciária competente. Nessa constará, dentre outros, a data do desacolhimento, identificação
da criança e a qualificação do responsável que receberá sua guarda e o endereço de residência deste.
De acordo com o ECA (1990), a guarda destina-se a regularizar a posse de fato, podendo ser deferida,
liminar ou incidentalmente, nos procedimentos de tutela e de adoção, da seguinte forma: a guarda (artigos
33 a 35) obriga a prestação de assistência material, moral e educacional à criança ou ao adolescente,
conferindo a seu detentor o direito de opor-se a terceiros, inclusive aos pais – e isso confere à criança ou
adolescente a condição de dependente, para todos os fins e efeitos de direito; já a tutela (artigos 36 a 38)
pressupõe a prévia decretação da perda ou suspensão do poder familiar e implica necessariamente o
dever de guarda.

7
SILVA, F. L. Como ocorre a reintegração familiar? Investigando esse processo em uma amostra de crianças acolhidas. 2012.

Apostila gerada especialmente para: Betânia Mueller 023.712.580-32


. 120
Para finalizar, cabe refletir sobre o termo “desacolhimento” trazido por esta nova legislação. Ele é
empregado ao documento expedido pela autoridade judiciária para determinar que a criança deixe a
instituição de acolhimento e, para definir o processo como um todo, o “desacolhimento institucional” da
criança.

Questões

01. (SEDUC/AM - Assistente Social - FGV). As crianças e adolescentes inseridos em Serviços de


Acolhimento possuem o direito à continuidade de sua escolarização.
Para tanto,
(A) a Direção da escola pode optar por receber ou não a criança ou adolescente, dependendo da
opinião do conjunto dos alunos.
(B) é obrigatório que a criança ou adolescente seja transferido para outra instituição escolar, a fim de
ausentar-se do ambiente que provocou a medida protetiva.
(C) a manutenção da criança ou adolescente deve ser de escolha dos pais ou responsáveis, pois deve
facilitar o seu deslocamento.
(D) sempre que possível, deve-se procurar manter a criança ou adolescente na mesma escola em que
estudava antes, resguardada determinação judicial em contrário.
(E) compulsoriamente a criança ou adolescente deve permanecer na escola de origem, a fim de que
as transformações em sua vida sejam mínimas.

02. (EBSERH - Enfermeiro - INSTITUTO AOCP). Em relação ao acolhimento, assinale a alternativa


INCORRETA.
(A) O acolhimento da gestante implica a responsabilização pela integralidade do cuidado a partir da
recepção da usuária com escuta qualificada, vínculo e da avaliação de vulnerabilidades.
(B) O profissional deve permitir que a gestante expresse suas preocupações e suas angústias,
garantindo a atenção resolutiva e a articulação com os outros serviços de saúde.
(C) Cabe à equipe de saúde buscar compreender os múltiplos significados da gestação para aquela
mulher e sua família, notadamente se ela for adolescente.
(D) A participação do pai deverá ocorrer somente no pré-natal, devendo sua presença ser vetada
durante as atividades em grupo, para não atrapalhar as demais participantes.
(E) É importante acolher o(a) acompanhante de escolha da mulher no pré-natal, no trabalho de parto,
no parto e no pós-parto. O(a) acompanhante pode ser alguém da família, amigo(a) ou a doula.

Gabarito

01.D / 02.D

Comentários

01. Resposta: D
Sempre que possível, deve-se procurar manter a criança ou adolescente na mesma escola em que
estudava antes da aplicação da medida protetiva, de modo a evitar rompimentos desnecessários de
vínculos de amizade e de pertencimento e modificações radicais em sua rotina, além de prejuízos
acadêmicos. Constituem exceções a tal recomendação as situações com determinação judicial em
contrário ou recomendação técnica de mudança de escola por questões relativas à preservação da
segurança e proteção da criança ou adolescente.

02. Resposta: D

Apostila gerada especialmente para: Betânia Mueller 023.712.580-32


. 121
Orientações Técnicas: Serviços de Acolhimento para Crianças e Adolescentes.
Ministério do Desenvolvimento Social e Combate à Fome, Conselho Nacional de
Assistência Social e Conselho Nacional dos Direitos da Criança e do
Adolescente. Adolescentes em conflito com a lei e as medidas socioeducativas.

Quanto aos serviços de acolhimento para crianças e adolescentes, este assunto já foi estudado
no tópico anterior.

MINISTÉRIO DO DESENVOLVIMENTO SOCIAL E COMBATE À FOME

O Ministério do Desenvolvimento Social e Combate à Fome (MDS) foi criado em janeiro de 2004 com
a missão de promover a inclusão social, a segurança alimentar, a assistência integral e uma renda mínima
de cidadania às famílias que vivem em situação de pobreza. Para isso, o órgão implementa inúmeros
programas e políticas públicas de desenvolvimento social nas três esferas de Governo e em parceria com
instituições da sociedade civil, organismos internacionais e instituições de financiamento. Essa articulação
estabelece uma sólida rede de proteção e promoção social que quebra o ciclo de pobreza e promove a
conquista da cidadania nas comunidades brasileiras.8
A Secretaria Nacional de Segurança Alimentar e Nutricional (SESAN/MDS) adota políticas públicas de
ampliação do acesso aos alimentos, combinando programas e ações de apoio à agricultura familiar, além
da implantação de uma ampla Rede de Segurança Alimentar e Nutricional.
A Secretaria de Avaliação e Gestão da Informação (SAGI/MDS) tem por missão desenvolver e
implementar instrumentos de avaliação e de monitoramento das políticas e programas sob
responsabilidade do Ministério. Promove a gestão do conhecimento, o diálogo de políticas e a cooperação
técnica em gestão pública, de forma articulada com órgãos, entidades, poderes e esferas federativas,
além de organismos internacionais e outros países.
Os cursos de capacitação na área de segurança alimentar desenvolvidos pelo MDS normalmente
contam com parcerias de outras instituições como, por exemplo, o Conselho Nacional de Segurança
Alimentar e Nutricional (CONSEA) e a Ação Brasileira pela Nutrição e Direitos Humanos (ABRANDH).

CONSELHOS DE ASSISTÊNCIA SOCIAL9

O Conselho Nacional de Assistência Social, um dos órgãos protagonistas na construção e nos avanços
da política de assistência social, pauta o tema Controle Social para a realização da VII Conferência
Nacional de 2009. Com isso, pretende-se que a dimensão da participação e controle social seja tratada,
examinada e refletida pelos mais diversos ângulos e atores, ao que incluo os aspectos políticos, técnicos
e éticos desse processo. Decorridos quinze anos de regulamentação da Lei Orgânica da Assistência
Social, é chegado o momento de realizar um profundo debate sobre os caminhos do controle social nessa
política.
A propósito da reflexão que envolve a temática Participação e Controle Social no Sistema Único de
Assistência Social (Suas), sistema este instituído por meio da Resolução CNAS de nº.145, de 15 outubro
de 20041, importante se faz tecer algumas considerações que antecedem a aprovação do recente
Sistema, pois é sabido que a reorganização jurídica, política e técnica no tratamento da assistência social
como direito do cidadão e dever do estado inaugura tempo novo com a promulgação da Constituição
Federal de 1988, que no ano passado complementou duas décadas de existência. Porém, ainda para o
alcance de grande parte de seus princípios e fundamentos da democracia, é preciso a permanente vigília
e mobilização da sociedade nesse processo, especialmente daqueles que defendem os compromissos
com a justiça social e a democracia.
Cabe registrar que na conjuntura atual, com a edição da Medida Provisória n˚ 446/082, que dispõe
sobre as novas regras para certificação de Entidades Beneficentes de Assistência Social, o CNAS não
mais será o órgão responsável pelo processo de certificação de entidades. E que, diante dessa decisão,
é chegado o momento de o Conselho voltar-se efetivamente para sua função, aprofundar, rever e propor
novas medidas que busquem de fato o exercício do controle social no Suas. O cenário favorece o debate,
haja vista o conjunto das regulamentações aprovadas nesses últimos anos: a Política Nacional, a
NOB/SUAS e a NOB/RH, bem como o Plano Decenal, entre tantas e importantes deliberações do CNAS
e regulamentações do MDS.

8
http://www.abc.gov.br/Training/informacoes/InstituicaoMDS.aspx
9
Valdete de Barros MARTINS, V. de B. Presidente do Conselho Nacional de Assistência Social gestão 2008/2010; Representante do MDS; Assistente social;
Mestre em Serviço Social; Professora de graduação e pós graduação do curso de Serviço Social na Universidade Católica Dom Bosco.

Apostila gerada especialmente para: Betânia Mueller 023.712.580-32


. 122
O Conselho Nacional de Assistência Social deve se constituir de fato como um efetivo espaço público
de controle democrático e pautar os grandes temas que envolvem a proteção social de assistência social.
Para isso, é preciso construir uma agenda de compromissos que priorize temas e questões que envolvem
os grandes desafios da sociedade brasileira no tocante aos direitos devidos por essa política aos seus
usuários. Inscrevem-se nesse processo a luta pela alteração da Lei n˚ 8.742/93, a regulação das novas
regras para acompanhar e fiscalizar a certificação das entidades, a disputa para ampliar o orçamento da
área em todos os níveis de governo, os pactos e articulações com outros conselhos de políticas públicas
e direitos na direção do firmamento da qualidade e alcance dos benefícios e serviços das diversas áreas
de políticas públicas. Enfim, o Conselho deve ser o espaço de disputa e de defesa intransigente,
transparente e permanente da política pública e não contributiva de seguridade social.
De início, pode-se dizer que, do ponto de vista da gestão, há tensa e forte regulamentação. A dinâmica
introduzida pelas comissões intergestores desde a primeira Norma Operacional Básica tem gerado
importantes transformações na interlocução entre os três níveis de governo, o que, forçosamente, impacta
as ações da política no seu desenvolvimento. Na mesma direção se inscreve a questão do financiamento
da política ao aprovar a NOB/SUAS, em 2005, e o compromisso do CNAS com novas regras para a
gestão financeira do sistema. Quanto ao controle social, o desenho para os conselhos e conferências
segue princípios constitucionais, assim como o estabelecido na Lei Orgânica de Assistência Social,
acrescido de novas exigências legais e técnicas que hoje consolidam o Suas. Cabe destacar que a
dimensão política do controle social está relacionada à dinâmica e ao compromisso que os conselhos de
assistência social alcançam. Nesse contexto é que se estabelece a necessidade de aprofundar o debate,
fazer uma leitura local, estadual e, também, da instância nacional. Se queremos fortalecer o controle
sobre a política e recriar uma cultura que dê conta das demandas e necessidades no âmbito da
assistência social para o alcance da universalidade das atenções de proteção social, temos pela frente
desafios de grande monta para os conselhos de assistência social.
É preciso reconhecer que os recentes avanços no campo da política pública de assistência social têm
produzido consideráveis alterações no modo de ver e tratar a política. Porém, são ainda insuficientes para
romper com os processos de subalternização dos usuários no acesso aos serviços e benefícios, assim
como para a sua participação política nos processos de construção de sua autonomia. Ainda hoje persiste
o uso clientelista da assistência social assim como o deficitário investimento nos serviços de modo
especial. Nessa direção, a mobilização para a implementação de diversas estratégias que (re) desenhem
o controle social deve ser intensa e urgente.
O objetivo deste texto é apresentar uma breve reflexão sobre a concepção de controle social,
posteriormente remetendo-a ao modelo adotado pelo Suas e, nesse contexto, resgatar a regulamentação
existente no âmbito da Política, buscando proceder uma avaliação da capacidade que os conselhos
detêm no processo de acompanhamento e deliberação da Política, assim como o grau de autonomia
dessas instâncias na formulação e fiscalização da Política de Assistência Social.

Uma breve aproximação do debate sobre a categoria controle social

A intenção aqui não é remeter o debate sobre a categoria controle social sob o balizamento das
diversas teorias. Assim, valemo-nos das teses de alguns estudiosos do tema como Raichelis, Correia,
Campos e Calvi. Nesse breve levantamento bibliográfico sobre o tema, o que se observa é uma
convergência dos (as) autores (as) no entendimento da categoria controle social. Raichelis apresenta
uma primeira aproximação com a questão ao eleger o CNAS como objeto de estudo de sua tese de
doutorado. No seu entendimento sobre a concepção de esfera pública, a autora propõe como elementos
constitutivos e que devem estar articulados dinamicamente: visibilidade social, representação de
interesses coletivos, democratização, cultura política e controle social, definido como:
Significa acesso aos processos que informam as decisões no âmbito da sociedade política. Permite
participação da sociedade civil organizada na formulação e na revisão das regras que conduzem as
negociações e a arbitragem sobre os interesses em jogo, além do acompanhamento da implementação
daquelas decisões, segundo critérios pactuados.
Nesse sentido, pode-se afirmar que a categoria controle social está associada a uma dada concepção
de Estado. Correia destaca duas concepções que são básicas para este debate. A primeira é aquela que
entende o controle social como controle do Estado sobre a sociedade. Esse controle favorece os
interesses da classe dominante e as políticas sociais são meios utilizados para abrandar conflitos sociais.
Essa perspectiva garante o consenso social para aceitação da ordem do capital sobre os membros da
sociedade e classifica esse estado de Estado Restrito, apoiando-se na teoria de Marx sobre o Estado. A
segunda concepção apresentada refere-se ao controle social como controles da sociedade: “Nesta, a
sociedade tem possibilidades de controlar as ações do Estado em favor dos interesses das classes

Apostila gerada especialmente para: Betânia Mueller 023.712.580-32


. 123
subalternas”. Essa afirmação baseia-se na teoria de Gramsci que apresenta a concepção de Estado
ampliado. Neste modelo de Estado perpassa interesses de classes, ou seja, como um espaço conflitante
que, apesar de conceber hegemonicamente os interesses da classe dominante, assumem demandas das
classes subalternas. É nesse campo colidente das políticas sociais que a sociedade capitalista constrói
os consensos para legitimar os interesses da classe dominante. Nesse contexto, os movimentos
progressistas organizados conseguem introduzir na Constituição Federal de 1988 a participação da
sociedade nas diversas questões que se relacionam à esfera pública e ao controle das políticas públicas
de modo particular.
Na mesma direção, Campos assevera que o controle social pode ser abordado sob diferentes
perspectivas, seja nas sociedades modernas ou contemporâneas, e se apresenta sob dois modelos que
se polarizam. A primeira inclui o controle que o Estado exerce sobre os cidadãos e a segunda está
relacionada ao controle que os cidadãos exercem sobre o Estado. O autor desenvolve o pensamento,
num primeiro momento, chamando atenção para a perspectiva do Estado como instrumento de controle.
Bobbio, apresenta duas principais formas de exercício do controle social que estão relacionadas à área
dos controles externos e aos mecanismos de controle interno:
A primeira forma de controle está relacionada aos mecanismos de representação destinados à
manutenção da ordem. São exemplos desta modalidade as legislações restritivas, os tribunais, as
políticas etc. Trata-se do controle exercido verticalmente, de cima para baixo, de forma centralizada e
quase sempre autoritária. A segunda forma de controle social é mais sutil, porém avassaladora. E o
controle, horizontalmente construído, ocorre como produto de uma socialização que naturaliza as
desigualdades sociais e individuais, que justifica privilégios e que consente discriminações. São valores
e crenças que conformam nossa formação sociocultural e política por meio da educação, da cultura
transmitida, da força da religião e da ideologia.
Com base nas considerações acima, percebe-se que muitas podem ser as formas e arranjos de
controle social nas sociedades capitalistas. No Brasil, no bojo da redemocratização, da luta pela
descentralização das ações e decisões e da necessidade do envolvimento da sociedade civil nos
processos relacionados aos conteúdos das políticas públicas, a Constituição Federal apresenta inovações
e prevê de forma explícita, por meio de princípios e diretrizes, o reordenamento da relação entre Estado
e Sociedade. As principais inovações relacionam-se à descentralização político-administrativa e à
participação social. Evidencia-se o controle social do Estado exercido pelo cidadão, no entendimento de
um Estado a serviço do interesse público. E nessa direção faz-se oportuno lembrar que a Carta de 1988
apresenta outros mecanismos de controle pela população no que se refere ao bem público, como por
exemplo, os plebiscitos (art.14-I), a Auditoria Cidadã da Dívida externa (art.26 do Ato das Disposições
Transitórias da CF), a Lei de Iniciativa Popular (art.27, inciso IV e art.29, inciso IX), e outros como a
audiência pública, o controle do orçamento participativo, a ação civil pública.
Na trilha dos acontecimentos que envolvem a busca pela participação social, a sociedade civil tem
organizado diversos outros instrumentos que impulsionam o movimento pela consolidação da assistência
social, como a criação do Fórum Nacional de Assistência Social, em 19 de agosto de 1999, instância de
articulação e mobilização da Política5. Seus objetivos e princípios constam da Carta de sua criação.
A criação do Fórum significa a possibilidade de ampliar o espaço público da assistência social para
arenas coletivas de debate e de alianças em torno de uma agenda de prioridade a serem enfrentadas.
Trata-se de uma instância ampliada de participação e representação política, não institucionalizada, e por
isso mesmo mais permeável à participação de segmentos sociais com menor acesso aos espaços
tradicionais de deliberações políticas.
Essa iniciativa deflagrou um movimento no Brasil e fez com que muitos estados e municípios também
instituíssem fóruns da sociedade civil, o que contribuiu substancialmente para qualificar as ações da
Política, destacando-se a criação dos conselhos de assistência social.
Para a Política de Assistência Social, esses avanços imprimiram um novo paradigma. Foi o início de
um processo que significou ruptura com as velhas práticas de cunho assistencialista, clientelista e de
primeiro damismo no trato com a assistência social.
Como perspectiva de deslocamento de um modelo meritocrático particularista para um novo padrão
de atenção, muitos desafios foram enfrentados, desde a lenta e gradual aprovação da Lei Orgânica de
Assistência Social, em 1993, até a aprovação de seus instrumentos regulatórios, e do baixo investimento
público para o alcance do direito aos processos de controle social estabelecidos. Diante dessa realidade,
torna-se oportuno revisitar as bases legais e políticas para o alcance do controle social no Suas.

Apostila gerada especialmente para: Betânia Mueller 023.712.580-32


. 124
Os conselhos como espaços (institucionais) de controle social na política de assistência social

A assistência social, ao compor o tripé da seguridade social brasileira, inscreve-se como direito social
e humano e incorpora objetivos como a universalidade e o caráter democrático (art.194 da C.F/88). A
partir desse reconhecimento legal, a assistência social passa a compor o sistema de seguridade social
não contributivo. E é por esse ângulo que se pretende debater o controle social na política de assistência
social. Que controle social tem movido as decisões da Política? Como vem sendo exercido esse controle?
Quais têm sido os instrumentos de efetivo controle? Os conselhos de assistência social como mediadores
institucionais têm cumprido seu papel nesse processo?
Outra questão a ser respondida: Com a aprovação da Política Nacional de Assistência Social, em
2004, e da Norma Operacional Básica, em 2005, que alterações foram introduzidas no campo do controle
social?
Inicialmente, cabe resgatar os incisos I e II do artigo 204 da Constituição Federal, pois esses impõem
o novo modelo para a condução da assistência social como direito do cidadão e como política de
seguridade social. Fica assegurada uma gestão descentralizada e participativa da política, garantindo a
possibilidade de democratizar os processos necessários ao cumprimento da lei maior. Com a
regulamentação da Lei 8.742/93 (Loas), o desenho ganha contornos mais explícitos no sentido de
imprimir a participação da sociedade na formulação e controle das ações em todos os níveis de governo.
O art.5º, que trata das diretrizes da Política, reafirma o já mencionado na Constituição, destacando-se o
inciso III do referido artigo, que revela a urgente e necessária atuação do Estado no que tange ao
desenvolvimento das ações da Política, ao instituir diretrizes para a organização da assistência social: “III
- primazia da responsabilidade do Estado na condução da política de assistência social em cada esfera
de governo”.
Na direção de precisar o reordenamento da área encontra-se o Capitulo III da Loas, que trata da
organização e gestão:
As ações na área de assistência social são organizadas em sistema descentralizado e participativo
constituído pelas entidades e organizações de assistência social abrangidos por esta Lei, que articule
meios, esforços e recursos, e por um conjunto de instâncias deliberativas compostas pelos diversos
setores envolvidos (art. 6º LOAS).
No artigo 16, a Lei explicita qual o formato dos conselhos de assistência social:
As instâncias deliberativas do sistema descentralizado e participativo de assistência social, de caráter
permanente e composição paritária entre governo e sociedade civil, são:
I – o Conselho Nacional de Assistência Social;
II - os Conselhos Estaduais de Assistência Social;
III – o Conselho de Assistência Social do Distrito Federal;
IV – os Conselhos Municipais de Assistência Social.

Parâmetros para a criação de uma nova cultura na condução da Política Pública de Assistência Social
são estabelecidos e os conselhos ganham expressão no processo de controle social.
Os conselhos, nos moldes definidos pela Constituição Federal de 1988, são espaços públicos com
força legal para atuar nas políticas públicas, na definição de suas prioridades, de seus conteúdos e
recursos orçamentários, de segmentos sociais a serem atendidos e na avaliação dos resultados. A
composição plural e heterogênea, com representação da sociedade civil e do governo em diferentes
formatos, caracteriza os conselhos como instâncias de negociação de conflitos entre diferentes grupos e
interesses, portanto, como campo de disputas políticas, de conceitos e processos, de significados e
resultados políticos.
Por força legal e política, desde a promulgação da Loas os conselhos de assistência social vêm sendo
implantados em todo o país. Nesse período que compreende mais de uma década, não só o CNAS como
os demais conselhos de assistência social têm enfrentado algumas dificuldades para realizarem
efetivamente o controle social, seja pela conjuntura adversa relacionada à política econômica, pelas duras
resistências do aparato governamental para se instalar e obter o reconhecimento como espaço
institucional e legítimo para o exercício do controle social, seja pela recusa dos executivos em partilhar o
poder, conforme afirma Raichelis, (2006).
Contudo, é importante reconhecer que avanços são registrados. Um primeiro balanço, que foi realizado
por ocasião da IV Conferência Nacional da Assistência Social, em 2003, quando o CNAS encomendou
uma pesquisa para subsidiar um debate sobre o tema da referida Conferência, conduzida pelo Grupo de
Estudos e Pesquisa sobre Seguridade Social e Trabalho (GESST/SER/UnB), sob a responsabilidade da
profa. Dra. Ivanete Boschetti, objetivou avaliar a implantação do sistema descentralizado e participativo
previsto na Loas, sobretudo a atuação dos conselhos estaduais e municipais no acompanhamento e

Apostila gerada especialmente para: Betânia Mueller 023.712.580-32


. 125
controle da política de assistência social. Vários aspectos foram abordados, destacando três conjuntos
de temas referentes ao controle social. O primeiro refere-se ao processo e período de criação, estrutura
e funcionamento dos conselhos estaduais e municipais, enfocando também questões relativas à paridade,
representatividade e processo de eleição e indicação dos conselheiros da sociedade civil. O segundo
conjunto de temas tratado na pesquisa reporta-se à atuação dos Conselhos de Assistência Social na
inscrição das entidades, no acompanhamento e avaliação da execução das ações do Plano de
Assistência Social e de sua autonomia em relação ao órgão gestor. O terceiro grupo de temas abordou
questões relativas à atuação dos conselhos na esfera da mobilização e participação social e organização
de conferências. O resultado da pesquisa nos chama atenção em alguns aspectos que passamos a citar.
Criação e instalação dos conselhos: processo retardatário em relação à Loas.
A primeira grande mensagem da pesquisa em relação a este quesito indica que o processo de criação
dos Conselhos foi bastante lento em relação à data de aprovação da Loas, o que retardou a instalação
do Sistema Descentralizado e Participativo. Outro importante resultado mostra que os Conselhos
Municipais foram criados, majoritariamente, após a instituição dos Conselhos Estaduais.
Os dados apontam, quanto a esse aspecto, que dezoito conselhos estaduais, ou seja, a maioria, foram
criados entre 1994 e 1995, e nove só foram criados entre 1996 e 1997, o que indica atraso de mais de
três anos na implantação da Loas nos Estados. Outra questão levantada pela pesquisa é que além da
lentidão para a criação, foi constatado maior atraso no funcionamento efetivo dos conselhos, sendo que
sete só começaram a funcionar seis meses após sua criação e dois após um ano de criação. Essa
tendência também se registrou nos conselhos municipais.
A morosidade quanto a criação dos conselhos foi justificada pelo desconhecimento da sociedade civil
e do próprio governo sobre a importância desta instância deliberativa na implementação dos preceitos da
Loas, ausência de cultura organizativa e participativa da população, bem como do não reconhecimento
da assistência social como direito e de sua organização em um sistema descentralizado e participativo,
inexistência de fóruns anterior à criação do conselho, temeridade do governo em publicizar informações
com a sociedade civil.
Outro aspecto mencionado no texto da pesquisa: “em algumas situações a criação do conselho foi
realizada apenas para cumprir um dos critérios para o recebimento dos recursos de outras esferas de
governo”.
Fiz questão de destacar esses trechos da pesquisa para chamar atenção do cenário em que se
encontrava a questão do controle social após dez anos de Loas. Ainda merece destaque os comentários
relativos às dificuldades para analisar e discutir a política de assistência social.
Há precariedade nos fluxos informativos do órgão gestor para com o conselho, descontinuidade das
ações de assistência social em períodos de troca de mandato governamental, falta de capacitação dos
conselheiros, não publicização das ações assistenciais, inclusive em algumas localidades foi apontada a
inexistência formal da política de assistência social.
A intenção com as chamadas do resultado dessa pesquisa, que obviamente fora conduzida anterior à
instituição do Suas e de outros regulamentos aprovados posteriormente, é trazer para reflexão as
fragilidades e precariedades que os conselhos vivenciaram e que ainda podem ser encontradas nos dias
de hoje. Por isso, é imprescindível e urgente um olhar sobre a responsabilidade dos conselhos no que se
refere ao controle da política. Uma leitura mais atenta dessa pesquisa denota a desarticulação dos vários
atores que compõem os segmentos da sociedade civil (trabalhadores, entidades de assistência social e
usuários) no espaço dos conselhos.
Esse modo de conceber e encaminhar o controle social na política de assistência social traz
consequências políticas significativas para a área, pois retarda a noção de dever do Estado para com a
assistência social, dificulta a operacionalização da política na perspectiva do direito e retarda a
universalização da proteção social devida aos cidadãos usuários. Será que houve alterações nesse
processo? Quais foram? Como os conselhos têm se organizado e pautado o debate sobre as questões
apontadas?
Outra pesquisa que se torna oportuna referenciar é o Suplemento de Assistência Social da Pesquisa
de Informações Básicas Municipais, realizada pelo IBGE em parceria com o Ministério de
Desenvolvimento Social e Combate à Fome - MUNIC-20057. As informações levantadas buscaram traçar
um perfil dos conselhos municipais, destacando o caráter do conselho (deliberativo ou não), a paridade
entre poder público e a sociedade civil, a composição e a frequência das reuniões. Dos 5.564 municípios,
5.497 possuem conselhos. Destes, 5.211 (94,8%) têm caráter deliberativo. Segundo os dados, a presença
do caráter deliberativo encontra-se nas cidades de maior população. Na Região Norte, os conselhos têm
caráter deliberativo em 89,0% dos municípios, e nas outras regiões em 93,5% deles. Quanto à paridade,
5.407 (98,4%) têm composição paritária. Ressalta-se que nos municípios de até 5.000 habitantes, 97,3%
dos conselhos existentes têm composição paritária. A Região Centro-Oeste é a que apresenta o menor

Apostila gerada especialmente para: Betânia Mueller 023.712.580-32


. 126
percentual de municípios com conselho paritário. Já entre as unidades da federação, o Acre, tendo
conselho em 100% dos seus municípios, com paridade em 95,5% deles, é o que apresenta a menor
proporção de conselhos com composição paritária (IBGE, Perfil dos Municípios Brasileiros, Assistência
Social, 2005).
Seguindo na avaliação sobre a composição dos conselhos, a pesquisa apresenta um dado que fere
frontalmente a Loas e a própria Constituição Federal, já que sabemos o que dispõe as referidas
legislações quanto à autonomia da sociedade civil na escolha de seus representantes. Os dados indicam
que em 2.928 (53,3%) dos municípios com conselhos, os representantes da sociedade civil são
escolhidos por meio de eleições em fóruns locais, nos municípios com até 5.000 habitantes.
Segundo a pesquisa, essa modalidade de escolha acontece em apenas 41,0%, enquanto nos
municípios com mais de 500.000 habitantes essa forma de escolha ocorre em 91,4% dos mesmos. Os
levantamentos indicam que em 20,2% dos municípios brasileiros os representantes da sociedade civil
são indicados pelo poder público, ressaltando-se que nos municípios maiores, com mais de 500.000
habitantes, ocorre a indicação do poder público em 2,9%, sendo que essa proporção aumenta na medida
em que cresce o número de habitantes.
Um destaque dado pela pesquisa, em relação a alguns municípios que afirmam ter composição
paritária no Conselho Municipal, é a indicação dos representantes da sociedade civil pelo poder público,
o que, de certa forma, compromete a autonomia do conselho nas suas deliberações. Essa constatação
nos remete à seguinte reflexão:
Como processo político, o controle social investe os atores da sociedade civil de prerrogativas para
intervir propositadamente na agenda do governo, assim como pressupõe, por um lado, a existência de
governantes democráticos, estimulados ao partilhamento do poder com a sociedade civil e instalados à
construção de canais de participação e de instrumentos institucionais que favoreçam o protagonismo da
sociedade nos assuntos de interesse público.
A constatação dos dados da pesquisa confirma que no campo da política pública de assistência social
predominam posturas da relação caso a caso, submetendo os conselhos a uma situação de
subordinação, como se esse espaço fosse um departamento da prefeitura ou secretaria, caracterizando,
na maioria das vezes, gestões centralizadoras.
Quanto à composição dos conselhos, a pesquisa indicou dificuldades de representação que envolvem
os três segmentos (trabalhadores, entidades de assistência social e usuários) de forma equilibrada. Em
77,3% dos municípios brasileiros, representantes de entidades e organizações de assistência social
fazem parte da composição dos conselhos. A representação de trabalhadores da área de assistência
social está em 66,0% do total dos municípios pesquisados. Já a representação de organizações de
usuários da assistência social está em apenas 65,8% dos conselhos (IBGE, Perfil dos Municípios,
Assistência Social, 2005). Essa constatação vem ao encontro da afirmação de Raichelis.

[...] no entanto, há uma significativa unanimidade nos estudos e avaliações da experiência dos
conselhos quanto à frágil presença dos segmentos populares em tais espaços, por meio de suas
associações e representações coletivas. Se a pobreza, para além de sua expressão monetária, é um tipo
de relação social que define sociabilidades e lugares dos indivíduos na sociedade, sua contraface é a
carência de representação política nos espaços públicos.

Nessa direção, prossegue a professora, “a luta pelo alargamento dos direitos sociais está diretamente
relacionada à possibilidade de ampliação da participação nos espaços públicos, o que poderá contribuir
com a formação de novos atores políticos legitimamente representados e que possam expressar suas
reais demandas e necessidades.”
Pelas questões levantadas nesse primeiro momento de reflexão, é imprescindível que se aprofunde o
debate sobre o desenho organizacional dos conselhos como instâncias democratizadoras das políticas
públicas, sendo organizados como espaços que precisam pautar ou revisitar suas agendas na direção e
perspectiva da universalização dos direitos sociais e da cidadania. Os diversos atores políticos que
compõem esse espaço democrático devem buscar estabelecer uma relação simétrica que produza e gere
uma dinâmica na construção de uma esfera pública transparente, qualificada, competente e
comprometida com os princípios e diretrizes do Sistema Público Brasileiro de Assistência Social.
Faz-se oportuno, nesse espaço de debate, lembrar que mesmo num cenário de contradições registra-
se a realização de seis Conferências Nacionais, que mobilizaram e envolveram milhares de cidadãos nas
disputas e conquistas de um número considerável de deliberações que deveriam ser observadas e
implementadas pelos governos na elaboração de seus planos e orçamentos anuais e plurianuais. Essa
também é uma responsabilidade legal, política e ética dos conselhos de assistência social. Os avanços
na regulamentação da política serão efetivos e consolidados à medida que os conselhos se tornem

Apostila gerada especialmente para: Betânia Mueller 023.712.580-32


. 127
protagonistas no acompanhamento e defesa das deliberações das conferências e dos pactos assumidos
para a construção efetiva de uma política de Estado. Romper com personificação no âmbito dessa política
é um desafio que se coloca para o conjunto dos conselhos e dos outros espaços sócio-políticos da
assistência social
O exercício do controle social não depende apenas da criação de instâncias institucionais como os
conselhos, mas da capacidade dos movimentos, organizações, fóruns, comissões, grupos e outras
formas de articulação, por meio dos quais os atores da sociedade civil possam debater, alterar e gerar
uma cultura de participação e de construção de direitos.

O Controle social no sistema único de assistência social

Com a aprovação da Política Nacional de Assistência Social, em 2004, a categoria controle social
adquire maior visibilidade e expressão e o princípio da descentralização e participação constitui-se em
eixo fundante no processo de Implantação do Sistema Único de Assistência Social em todo o território
nacional. Vale ressaltar a importância dos fóruns de participação popular, específicos e/ou de articulação
da política em todos os níveis de governo, bem como a união dos conselhos e/ou congêneres no
fortalecimento da sociedade civil organizada na consolidação da Política Nacional de Assistência Social.
O texto ainda explicita a importância da organização de outros espaços que devem gerar o controle
social:
Na conformação do Sistema Único de Assistência Social, os espaços privilegiados onde se efetivará
essa participação são os conselhos e as conferências, não sendo, no entanto, os únicos, já que outras
instâncias somam força a esse processo.

Atribuições e articulação dos conselhos de assistência social

Ainda no artigo 18, a Loas dispõe que “os conselhos têm como principais atribuições a deliberação e
a fiscalização da execução da política e de seu financiamento, em consonância com as diretrizes
propostas pela conferência; a aprovação do plano; a apreciação e aprovação da proposta orçamentária
para a área e do plano de aplicação do fundo, com a definição dos critérios de partilha dos recursos,
exercidas em cada instância em que estão estabelecidos. Os conselhos, ainda, normatizam, disciplinam,
acompanham, avaliam e fiscalizam os serviços de assistência social prestados pela rede
socioassistencial, definindo os padrões de qualidade de atendimento, e estabelecendo os critérios para o
repasse de recursos financeiros”.
Para o avanço pretendido, a política aponta para a construção de uma nova agenda para os conselhos
de assistência social. Uma primeira vertente é a aglutinação do CNAS com os conselhos nacionais das
políticas sociais integrando um novo movimento neste país. Outra é a construção de uma agenda comum
dos conselhos nacional, estaduais e municipais de assistência social. Esta última tem como objetivo
organizar pontos comuns e ações convergentes, resguardando as peculiaridades regionais.

Norma Operacional Básica da Assistência Social

Os conselhos nacional, estaduais, do Distrito Federal e municipais são instâncias deliberativas do


Sistema Descentralizado e Participativo da Assistência Social, regulamentados na PNAS/2004 na forma
do SUAS. O CNAS, instituído pela LOAS, e os Conselhos das demais esferas, tratados na referida Lei e
instituídos por legislação especifica, têm caráter permanente e composição paritária entre governo e
sociedade civil. São vinculados ao Poder Executivo e a sua estrutura pertencente ao órgão da
Administração Pública responsável pela coordenação da Política de Assistência Social, que lhes dá apoio
administrativo, assegurando dotação orçamentária para seu funcionamento.

Conferência de Assistência Social

De acordo com o artigo 18, inciso VI, da LOAS, “as conferências têm o papel de avaliar a situação da
assistência social, definir diretrizes para a política, verificar os avanços ocorridos num espaço de tempo
determinado”.
As conferências de assistência social são instâncias deliberativas com atribuição de avaliar a Política
de Assistência Social e propor diretrizes para o aperfeiçoamento do Sistema Único da Assistência Social.
A convocação, pelos respectivos conselhos, para realização das Conferências de Assistência Social,
obedecerá a periodicidade estabelecida na Loas para a Conferência Nacional e legislação específica para
conferências estaduais, do Distrito Federal e municipais.

Apostila gerada especialmente para: Betânia Mueller 023.712.580-32


. 128
Plano Decenal de Assistência Social
As metas sobre o controle social tiveram como fomento o seguinte macro objetivo:
Fortalecer o controle social por meio da atuação dos Conselhos de Assistência Social para o
desenvolvimento de suas funções, da realização de conferências e do incentivo aos fóruns de debate
público sobre a política.

Desafios para avançar o controle social no Suas

Para finalizar, entende-se que as dificuldades destacadas nesta reflexão devem servir de parâmetros
para iniciar um processo avaliativo na questão do controle social. Por outro lado, do ponto de vista político,
há um enorme subsídio legal, teórico e ético que nos capacitam a promover um debate fecundo e
importante nesse campo. Assim, o conjunto de desafios elencados a seguir deve ser considerado para
iniciar o grande debate sobre o controle social.
- Ampliar o debate sobre a questão do controle social, buscando identificar estratégias que possam
criar novos mecanismos e instrumentos de intervenção nos espaços públicos.
- Investir na capacidade de articulação entre os níveis de governo, na direção de firmar a perspectiva
do Suas como Sistema Público democrático e participativo.
- Analisar profundamente o modelo de funcionamento dos conselhos de assistência social, suas
competências, capacidade de deliberação, grau de autonomia.
- Buscar parceria com o Ministério Público para fazer valer as decisões dos conselhos de assistência
social.
- Observar as orientações do Tribunal de Contas quanto ao papel, responsabilidade e função social
dos conselhos no processo de acompanhamento e avaliação da gestão dos recursos do fundo da
assistência social, buscando certificar se os mesmos estão sendo aplicados conforme finalidade prevista
nos Planos de Assistência Social.
- Investir na articulação entre os Conselhos de Assistência Social (CNAS, CEAS e CMAS), de modo
que as deliberações no âmbito desses espaços possam conduzir ao fortalecimento do controle social no
Suas.
- Monitorar as deliberações das Conferências, especialmente o Plano Decenal, (metas e estratégias)
em cada nível de gestão.
- Dotar os conselhos de infraestrutura (material, humana e financeira), agregando a eles, dessa forma,
condições de trabalho para que viabilizem suas ações de controle social.
- Investir na capacitação dos conselheiros e secretaria executiva, de forma que a dimensão técnica
ganhe as condições necessárias para o avanço na construção de metodologias e processos que
qualifiquem a fiscalização e avaliação das ações.
- Apoiar e incentivar novas iniciativas para a criação de espaços de controle social, de forma que
contemplem com prioridade a participação dos usuários dos serviços e benefícios da política.
- Promover ações em parceria com o Ministério Público de forma a vigiar o controle social sobre as
decisões da política.
- Estimular a instalação de Frentes Parlamentares em defesa da política de assistência social.
- Estabelecer e fortalecer a articulação da sociedade civil e Estado, na perspectiva de criar iniciativas
que valorizem processos democráticos, estabeleça pactos e favoreçam as alianças, dando uma nova
direção à institucionalização do controle social.
- Atuar na direção do comando único, da ruptura com o primeiro damismo, denunciar formas de
clientelismo e de favorecimento partidário e¬/ou de grupos e outros processos que desqualificam a política
e o direito dos usuários.
- Imprimir prioridade na luta pelo orçamento público em todas as esferas de governo.
- Rever e estabelecer regulamentações que fortaleçam os princípios e diretrizes do Suas como sistema
público, descentralizado e participativo.

O que é o Conselho dos Direitos da Criança e do Adolescente?

É um órgão ou instância colegiada de caráter deliberativo, formulador e normalizador das políticas


públicas, controlador das ações, gestor do Fundo, legítimo, de composição paritária e articulador das
iniciativas de proteção e defesa dos direitos da criança e do adolescente (ECA – artigo 88, 214 e 260).
Integra a estrutura básica do poder executivo, da secretaria ou órgão da área social e tem composição
e organização fixadas em lei.

Apostila gerada especialmente para: Betânia Mueller 023.712.580-32


. 129
Tem competência para promover e controlar todos os direitos da criança e do adolescente?
Sim. Crianças e adolescentes não são uma área – são um público que deve ter prioridade absoluta
em todas as áreas (saúde, educação, assistência social, cultura, esportes...). Por isso que se diz que é
um Conselho de público e de política, inter e multi setorial. Deve exercer o controle das ações de todos
os direitos, de forma global. Não deve ser um “Conselho do Menor”.

Quais são suas competências administrativas?


Entre outras podemos destacar as seguintes: coordenação da eleição do Conselho Tutelar; gestão do
Fundo através de uma Junta; Secretaria do Governo ou Administrador; registro das entidades inscritas
dos programas de atendimento de crianças e de adolescentes; elaboração do plano de ação e do plano
de aplicação; montagem da proposta orçamentária do Fundo; constituição de comissões; edição de
resoluções e constituição da Secretaria Executiva.

Como deve ser estruturado o Conselho?


O Conselho deve ser composto por um plenário integrado por todos os conselheiros e por uma
Secretaria Executiva. A Secretaria deve ter suas atribuições definidas em seu regimento interno e
acompanhar a execução? das deliberações do Conselho, além de servir de apoio administrativo às suas
atividades.

Quem pode encaminhar projetos de lei para a criação do Conselho?


É atribuição do Executivo Municipal elaborar o projeto de lei e encaminhá-lo ao Legislativo Municipal
para aprovação. A sociedade civil tem o papel de provocar e sensibilizar o poder executivo para esta
iniciativa legislativa. No caso de omissão do Executivo Municipal, o Ministério Público poderá instaurar
inquérito civil.

Quais são os pressupostos para a composição do Conselho?


Ser paritário – sua composição deve respeitar o princípio da paridade, ou seja, ser composto por igual
número de representantes do poder público e da sociedade civil.
Ser representativo – os representantes que compõem este Conselho devem ter plenas condições para
serem os legítimos defensores dos segmentos que representam.

Existe limite para o número de membros do Conselho?


Não. Entretanto, recomenda-se que este número não seja excessivamente grande para evitar-se a
dispersão e problemas na operacionalização e funcionamento.

Representantes de diferentes esferas de governos e poderes podem participar do Conselho?


Recomenda-se que os representantes sejam, em sua maioria, da esfera municipal de governo. Os
órgãos da esfera estadual, sediados nos municípios poderão compor o Conselho Municipal desde que,
atuem direta ou indiretamente na promoção de direitos relacionados ao segmento criança/adolescente.

Quem são os representantes da sociedade civil no Conselho?


São os representantes de organismos ou entidades privadas, ou de movimentos comunitários,
organizados como pessoas jurídicas, com atuação expressiva na defesa dos direitos de crianças e
adolescentes.

Como são escolhidos os representantes da sociedade civil?


Devem ser indicados pelos sindicatos, associações e movimentos? comunitários, devendo estes
serem escolhidos em foro próprio.

Quem deve indicar os membros do Conselho?


A indicação dos membros do Conselho é privativa das respectivas bases, entidades ou segmentos
sociais. Assim, cabe ao Prefeito apenas escolher os representantes do Executivo Municipal.

Os conselheiros podem ser substituídos antes do término de seu mandato?


Qualquer das entidades que compõe o Conselho pode substituir o seu representante, por motivo que
não cabe aos demais conselheiros discutir.
O próprio Conselho, contudo, pela lei ou pelo regimento interno, pode fixar motivos para a perda de
mandato dos seus membros.

Apostila gerada especialmente para: Betânia Mueller 023.712.580-32


. 130
E se a Constituição do Estado, a Lei Orgânica do Município ou o Decreto transitório tiverem
organizado o Conselho de maneira diferente da prevista no Estatuto da Criança e do Adolescente?
As normas gerais emanadas da União não podem ser modificadas ou descumpridas por norma
legislativa estadual ou municipal, muito menos por ato normativo do Poder Público Executivo.
Assim, se alguma legislação local contraria o Estatuto da Criança e do Adolescente, encontram-se três
alternativas ao alcance de todos:
- Mudar a Lei Estadual, a Lei Municipal ou o Decreto mediante mobilização da comunidade e dos
parlamentares interessados no autêntico e legítimo controle social;
- Denunciar junto ao Ministério Público, provocando, assim, a sua atuação ou;
- Promover ação judicial.

Qual é a relação do Conselho com o orçamento?


Os recursos são fundamentais para a realização das competências do Conselho. Formular políticas
sem o suporte financeiro pode se transformar em exercício de ficção. Daí a importância do Conselho
integrar suas diretrizes e propostas tanto no Plano Plurianual (PPA) como na Lei de Diretrizes
Orçamentárias (LDO) e seu Plano de aplicação dos recursos do Fundo, na proposta orçamentária a ser
enviada ao Poder Legislativo, gestionando para que os valores representados sejam aprovados.

Quem deve fazer o Regimento Interno?


O Regimento Interno deve ser elaborado pelo próprio Conselho. A prática tem ensinado que quanto
antes se der a sua elaboração, melhores são os resultados, uma vez que para muitas questões surgidas
no dia-a-dia, o Regimento Interno é o melhor instrumento para encontrar a solução.

Quais são os limites do Regimento Interno?


O Regimento Interno, como todo ato administrativo, não pode exceder os limites da lei. Deve
contemplar os mecanismos que garantem o pleno funcionamento do Conselho. Sua publicação deve
observar a regra adotada para a publicação dos demais atos normativos do Executivo Municipal.

O que diferencia o Conselho dos Direitos em relação ao Conselho Tutelar?


A diferença entre esses dois Conselhos está principalmente nas suas atribuições. Enquanto os
Conselhos Municipais dos Direitos são os órgãos que devem atuar na formulação e no controle da
execução das políticas sociais que asseguram os direitos de crianças e adolescentes; o Conselho Tutelar
atua no atendimento de casos concretos, de ameaça ou de violação desses direitos, sendo
exclusivamente de âmbito municipal.

Qual a relação do Conselho dos Direitos e o Fundo?


O papel fundamental do Conselho em relação ao Fundo é o de fixar critérios para a aplicação dos
recursos. Cabe ao Conselho gerir o Fundo, isto é, deliberar, gestionar, exercer o controle. A administração
do Fundo poderá ser feita por uma Junta Administrativa, por um gestor ou pela Secretaria à qual o
Conselho está vinculado.

Adolescentes em Conflito com a Lei e as Medidas Socioeducativas10

O papel desempenhado pelos movimentos sociais organizados no Brasil, a partir de meados da


década de 1970, foi essencial no sentido da luta pela redemocratização do país, que vivia os efeitos do
Regime autocrático-burguês instalado a partir de 1964. Um dos setores da sociedade que emergiu
naquele contexto, se fazendo ouvir por intermédio de contundentes denúncias e de um sem número de
propostas, foi o da militância em prol de crianças e adolescentes. Reivindicava, principalmente, o status
de sujeitos de direitos e, consequentemente, mudanças na concepção do atendimento a eles dirigido.
Inicialmente contempladas na Carta Constitucional de 1988 (art. 227), as demandas daquele setor foram
consolidadas em 1990, na forma de uma legislação específica sobre o tema, qual seja, o Estatuto da
Criança e do Adolescente –ECA.
A Constituição de 1988 foi denominada “cidadã”, tendo em vista atender significativo número de
reivindicações provenientes dos mais diversos setores da sociedade civil. O projeto de construção de
uma sociedade democrática, participativa e inclusiva que emergiu naquele contexto foi, no entanto,
desvirtuado em face da perspectiva neoliberal adotada pelos governos brasileiros a partir da década de
1990. No campo social, a frustração deveu-se ao não atendimento dos direitos a determinados grupos,

10
http://revistaseletronicas.pucrs.br/ojs/index.php/revistapsico/article/viewFile/1397/1097

Apostila gerada especialmente para: Betânia Mueller 023.712.580-32


. 131
em situação de vulnerabilidade e risco, como é o caso da parcela de crianças e adolescentes dos setores
empobrecidos da sociedade. (Oliveira, 2000).
Naquele contexto de euforia pela conquista de uma legislação que definia crianças e adolescentes
como sujeitos de direitos, preocupação específica era destinada ao adolescente em conflito com a lei,
suas possibilidades e condições de reinserção social, bem como das ações a serem desenvolvidas pelas
instituições nessa perspectiva. É para a intersecção desses aspectos que se volta o presente estudo,
buscando discutir o atribuído caráter “educativo” ao tipo de trabalho que deve ser desenvolvido pelas
instituições, visando a reinserção do adolescente.

O Estatuto da Criança e do Adolescente

A euforia que acompanhou a promulgação do ECA era justificada, visto que ele expressava a quebra
de um padrão nas políticas públicas voltadas para a infância e à adolescência brasileiras que tinha, no
mínimo, um século de duração. Ao adotar a Doutrina da Proteção Integral, também chamada Doutrina
das Nações Unidas Para a Proteção dos Direitos da Infância, o Estatuto rompeu com a tradição do
“menor”, expressa no Código de Menores de 1927, e com a Doutrina da Situação Irregular,
consubstanciada no Código de 1979 e na Política Nacional do Bem-Estar do Menor.
Em sintonia com a concepção assumida pelo ECA, de que crianças e adolescentes são sujeitos de
direitos, redirecionaram-se as atribuições do Estado e o papel da família e da sociedade em relação a
eles. Nesta perspectiva, lê-se no seu artigo 4º que, É dever da família, da sociedade e do Estado
assegurar à criança e ao adolescente, com absoluta prioridade, o direito à vida, à saúde, à alimentação,
à educação, ao lazer, à profissionalização, à cultura, à dignidade, ao respeito, à liberdade, à convivência
familiar e comunitária, além de deixá-los a salvo de toda forma de negligência, discriminação, exploração,
violência, crueldade e opressão (Brasil, 1990, p. 23).
Dentre as transformações legais contidas no Estatuto, destacam-se algumas que melhor caracterizam
o espírito da nova lei. São elas:
a) Municipalização da política de atenção direta;
b) Eliminação de formas coercitivas de internação, por motivos relativos ao desamparo social, na
medida em que suprime a figura da situação irregular. Neste sentido, a privação de liberdade só é aceita
nos casos de flagrante de ato infracional ou por ordem escrita e fundamentada da autoridade judicial
competente;
c) Participação paritária e deliberativa do governo-sociedade civil, assegurada pela existência de
Conselhos de Direitos da Criança e do Adolescente, nos três níveis da organização política e
administrativa do país: federal, estadual e municipal;
d) Hierarquização da função judicial, transferindo aos conselhos tutelares, de atuação exclusiva no
âmbito municipal, tudo o que for relativo à atenção de casos não vinculados ao âmbito da infração penal,
nem a decisões relevantes passíveis de produzir alterações importantes na condição jurídica da criança
ou do adolescente.
Vale destacar que a perspectiva do controle, vigente durante a Doutrina da Situação Irregular, foi
substituída pela da convivência, constituindo, sob o ECA, a ideia básica para assegurar a paz social e a
preservação dos direitos do conjunto da sociedade (Mendez, 1993; Costa, 1997; Volpi, 2001).

É nesta perspectiva que se observam os itens específicos do Estatuto que tratam do adolescente
infrator. A esse respeito, inicialmente, ressalte-se que somente os adolescentes – pessoas entre 12 e 18
anos de idade – são passíveis de cometerem o ato infracional, entendido como a transgressão das
normas estabelecidas, do dever jurídico, que em face das peculiaridades que os cercam, não pode se
caracterizar enquanto crime. Logo, ainda que os adolescentes se encontrem sujeitos a todas as
consequências dos seus atos infracionais, não são passíveis de responsabilização penal. Cabelhes,
nesses casos, medidas socioeducativas, cujo objetivo é menos a punição e mais a tentativa de reinserção
social, de fortalecimento dos vínculos familiares e comunitários.
Mesmo considerando o adolescente como pessoa na condição peculiar de desenvolvimento (Brasil,
1990), ao adotar medidas socioeducativas enquanto sanções – fruto da transgressão do dever jurídico –
, o ECA foge às armadilhas das concepções retribucionista e paternalista. No retribucionismo encontra-
se a defesa do aumento da repressão na proporção da gravidade das infrações praticadas, na expectativa
da prevenção do cometimento delas; o paternalismo, por seu turno, tende a isentar de culpa os
adolescentes que as cometerem, naturalizando a prática do ato infracional.
O reconhecimento de que a obediência a regras mínimas é essencial para o convívio social requer a
responsabilização do adolescente, quando ele desenvolve condutas transgressoras desses padrões.
Considerá-los pessoas em desenvolvimento expressa tão somente a tutela especial a que têm direito, por

Apostila gerada especialmente para: Betânia Mueller 023.712.580-32


. 132
lei, assim como a identidade peculiar desses sujeitos, não implicando a supressão da sua sujeição ao
ordenamento jurídico. Neste sentido, reitera-se a concepção de Leonardo Barbosa, quando defende que
“o processo de desenvolvimento do adolescente passa pela aprendizagem de um posicionamento crítico
e responsável em relação às suas condutas” (Barbosa, 2002, p. 10).
Muito embora não se questione a necessidade da obediência a um certo ordenamento jurídico,
também não se perde de vista que tais regras são definidas por e a partir de um grupo social específico,
aqui denominado dominante, o qual constrói toda uma teia de relações sociais e uma subjetividade
(ideológica) que, ao mesmo tempo em que é expressão dessa teia, é também sua fonte de
retroalimentação. Logo, há um padrão de referência de relação social, que serve para delimitar as
fronteiras do que se considera transgressão. Consequentemente, segue-se um padrão de referência
quando se pensa em ressocialização, o daquele grupo social específico.
Algumas das dúvidas que ressaltam da afirmação de que as medidas socioeducativas buscam a
reinserção social, o fortalecimento dos vínculos familiares e comunitários, dadas as considerações acima,
são: é possível essa ressocialização? Possível para quem? Do que se reveste a ideia desses vínculos
para grupos cujo padrão de sociabilidade é marcado pela violência? Acredita-se que, dada a
complexidade da abordagem dessas questões, não há possibilidade de fazê-lo no escopo do presente
escrito. Sugere-se, não obstante, além de Passetti (2002) e Fraga (2002), a leitura de Baierl e Almendra
(2002), para quem:
É certo que não se ignora que, como fórmula maior de arrefecimento, quiçá eliminação da
criminalidade, violenta ou não, está, entre outras coisas, a educação das gentes, a superação das
desigualdades sociais, a satisfação mínima de necessidades econômicas, as pulverizações de
preconceitos, de idiossincrasias, de vaidades, de ambições. Bem, mas isso não é para nós homens,
frágeis, imperfeitos, egoístas, pobres arremedos do divino modelo (p. 79).
Do ponto de vista do tratamento emprestado pelo ECA à questão do adolescente em conflito com a lei
faz-se necessário esclarecer que, enquanto sanção, a medida não é pena. Ou seja, muito embora se
assemelhe à pena ao considerar o princípio da personalidade na sua aplicação – apenas o autor do crime
responde por ele –, ser decorrência de lei e visar à ordem pública, a medida difere daquela em aspectos
essenciais. Primeiro, se a aplicação da pena, do castigo, busca estabelecer uma relação entre o ato
cometido e o rigor da punição, a aplicação da medida deve buscar uma maior individualização, no sentido
da sua adequação à história de cada adolescente em particular, ao invés de adequar-se apenas à infração
cometida.
Em segundo lugar, de caráter essencial é a diferença que remete à finalidade imediata de uma e de
outra. Com a pena, busca-se causar sofrimento ao transgressor, puni-lo por meio da privação de direitos.
Com a medida socioeducativa, por outro lado, é a ação pedagógica sistematizada que é visada, mesmo
quando se trata de medida de privação de liberdade. A respeito da relação entre ação pedagógica e
privação de liberdade, é importante considerar o debate que é suscitado a partir da compreensão de que
há uma contradição entre essas duas ações. Conforme apontado por Bazílio, “como é possível pensar
em processo educacional em estabelecimentos cujo objetivo é precisamente a tutela, o controle dos
tempos e corpos?” (Bazílio, 2003, p. 46). Poderiam ser, esses estabelecimentos, adaptados em sua
estrutura física e em relação à capacitação do pessoal técnico-administrativo de forma que, mesmo com
a “privação da liberdade”, eles pudessem cumprir o caráter educativo das medidas?
Longe de considerar a situação como simples, a nova lei admite a complexidade do problema,
sujeitando-se aos princípios da brevidade, excepcionalidade em relação à condição de pessoa em
desenvolvimento e incorporando a noção de privação de liberdade como último recurso dentre as medidas
socioeducativas. Estas podem ser, de acordo com o artigo 112 do ECA, a advertência, a obrigação de
reparar o dano causado, a prestação de serviços à comunidade, a liberdade assistida, a internação em
regime de semiliberdade e a internação em estabelecimento educacional (Brasil, 1990).
Como fica evidenciado nas disposições acima, grande foi o avanço legal no que diz respeito ao
atendimento dos adolescentes em conflito com a lei. Contudo, a observação da realidade atual das
instituições voltadas para tal clientela deixa margem para muitas interrogações sobre a distância entre o
que é preconizado pela lei e o que é efetivamente desenvolvido nessas instituições. De acordo com
Adorno, pouco depois da promulgação do Estatuto, “alterações na filosofia e nos programas de trabalho”,
adequação das “rotinas técnicas e administrativas” e “coordenação e conexão entre serviços e políticas
de diferentes ordens” (Adorno, 1993, pp. 110-111), eram identificados como desafios a serem transpostos
com vistas à efetivação, na prática, dos novos conceitos legais.
A esse respeito, avaliações recentes (Martini e Brancalhão, 2000; Bazílio e Kramer, 2003; Barbosa,
2002; Volpi, 2001) têm indicado que, catorze anos depois da promulgação do ECA, persiste um modelo
de atendimento que, ao mesmo tempo em que remete às ações desenvolvidas no contexto dos Códigos
de Menores, expressa a atualidade das preocupações de Adorno, acima referidas. De acordo com Bazílio,

Apostila gerada especialmente para: Betânia Mueller 023.712.580-32


. 133
após todos esses anos, cabe perguntar por que “a prática social com relação à infância continua sendo
marcada por violência, negligência e incompetência na esfera pública?” (Bazílio, 2003, p. 29-30). No que
concerne ao adolescente em conflito com a lei, por um lado há que questionar sobre o que existirá por
trás da dificuldade dos governos procederem ao reordenamento jurídico-institucional, que garantiria o
aperfeiçoamento do atendimento e, por outro, por que é tão difícil ao adolescente não reincidir, ou seja,
inserir-se socialmente.
Muito embora seja importante problematizar a adequação do judiciário e das instituições ao Estatuto,
no presente estudo enfatiza-se a discussão a respeito das instituições, articulando-a com reflexões sobre
formação, como base da referida inserção social. Se, em face da sua condição especial de
desenvolvimento, o adolescente infrator está sujeito às medidas do Estatuto, e não ao Código Penal,
decorre que as instituições cumpridoras da política pública de execução de medida não podem
disponibilizar a estrutura que existe nas prisões comuns. É necessário que o diferencial sócioeducativo
das medidas se manifeste, inclusive, na estrutura física das instituições para adolescentes, na expectativa
de se possibilitar a ressocialização, entendida como integração familiar, participação no sistema de
ensino, ocupação de um lugar na comunidade e, se for o caso, exercício de uma atividade laboral.
Esta possibilidade, a ser alcançada via o cumprimento das medidas, não pode ser entendida em
sentido funcionalista, que remete a uma estática do sujeito social, fadado a voltar a ser. Ao contrário,
requer considerar o sujeito ativo, que é influenciado pela dinâmica social mas que a influencia com sua
maneira de ser e de fazer. A responsabilidade que cabe ao sujeito na construção de um projeto de vida
é relativizada pela sua peculiar inserção nos grupos e classes sociais. Logo, muito embora seja de fato
na e pela ação educativa que se dá a feitura do projeto individual, toda ação educativa revela-se parcial,
condicionada.
Acredita-se que investigar o caráter educativo das medidas aplicadas ao adolescente infrator requer
ter claro que o ato educativo, de maneira mais ou menos explícita para quem com ele lida, pressupõe um
conceito, uma ideia, uma expectativa em relação ao perfil que assumirá o educando que deseja formar
através de um processo qualquer; em relação à sociedade na qual esse processo formativo se dá e essa
pessoa irá viver; e à forma como essa pessoa irá se relacionar com os demais nessa sociedade, muito
em face de tal processo. Logo, a pergunta que ressalta é: o que dá o caráter “educativo” das medidas?
Em outras palavras, educar para o quê, para o exercício de uma profissão, de uma vida em família, de
continuidade e/ou (re)integração ao sistema educativo formal? O que o adolescente que se encontra em
conflito com a lei demanda das instituições formadoras?
O fato do envolvimento com o trabalho/consumo, a família, a educação etc. comporem as expectativas
em relação ao adolesceste infrator, depois de cumprida à medida que lhe coube, significa que há, por
pressuposto, um padrão/concepção de sujeito, de organização e de convivência sociais, que as medidas
contribuiriam, de alguma maneira, para atingir.
Assim, o duplo caráter das medidas – punição (reparo) e criação de condições para a não reincidência
– em princípio, teria por finalidade operar um reordenamento dos valores e padrões de conduta do sujeito
transgressor. Possibilitar uma ressignificação dos seus padrões de socialização, de modo que os “novos
modelos” primem pela consideração da integridade da vida e da preservação do patrimônio. Neste
sentido, em última instância, denominar de socioeducativa uma medida que foi aplicada em face da
transgressão do ordenamento jurídico significa atribuir-lhe – princípio e condição – a possibilidade de
operar, no sujeito, mudanças que necessariamente impliquem na consciência de que a integridade da
vida deve ser mantida, assim como preservado o patrimônio.
A esse respeito, de acordo com Alice Itani, É fato que a educação surge como uma necessidade da
civilização para o processo de socialização. Por esse processo, os indivíduos são instituídos com a
imposição da lei social à psique, tornando-se humanos. Ou com um conteúdo que podemos considerar
de natureza humana. Essa ação educativa socializa, impõe regras de vivência coletiva, é realizada pelas
diversas instâncias da sociedade, desde a família até a escola, imprimindo-se o valor da vida como um
valor constitutivo do ser humano. Mas por essa ação pode-se também reproduzir a desigualdade e se
instalar o rompimento com uma ética da vida. (Itani, 1998, p. 38). Como já referido, dada a orgânica
relação entre educação e sociedade, que torna o processo educativo parcial e relativo, não há certeza de
que tal processo cumpra as expectativas em termos do adolescente que se espera, uma vez que seu
resultado é um sujeito fruto de uma dinâmica que, em muito, extrapola esse processo. O mais que se
pode esperar é que, dadas certas condições de trabalho, de relações, de vida, enfim, no interior das
instituições, o educando assuma uma específica postura de respeito frente à sua vida e à vida dos demais.
Referir-se às condições que as instituições de cumprimento de medidas socioeducativas dispõem no
sentido de oportunizar a reinserção social implica considerar, ao menos, os aspectos relacionados à
estrutura física, aos recursos humanos e às ações a serem desenvolvidas.

Apostila gerada especialmente para: Betânia Mueller 023.712.580-32


. 134
No tocante ao primeiro aspecto, como indicado anteriormente, é necessário ultrapassar as atuais
condições, em quase tudo semelhantes às das prisões comuns. Se, em grande parte das instituições
voltadas para os adolescentes, alcançou-se o objetivo de extinguir a coerção física e o uso da força como
estratégia educativa, se os mesmos têm direito a visitas semanais, inclusive visitas íntimas para aqueles
que têm parceiras fixas, por outro lado eles se encontram alojados em celas e não em dormitórios que,
além de não serem o espaço adequado para pessoas em processo de desenvolvimento, ainda fogem às
exigências mínimas de salubridade e dignidade para qualquer ser humano. A esse respeito, inclusive,
acrescente-se que a grande maioria das instituições não dispõe de espaço específico para possibilitar a
participação de familiares na dinâmica do atendimento aos jovens.
A respeito dos recursos humanos disponíveis, além dos educadores há que se considerar os técnicos,
como psicólogos, assistentes sociais, o corpo administrativo e o setor de segurança. Particularmente em
relação aos educadores, que fazem parte do cotidiano dos internos, é recorrente que seu papel se resuma
à guarda das celas em que se encontram os adolescentes, levando a que alguns se refiram a si mesmos
enquanto “abre e fecha cadeados”. Logo, quase nada do que se espera de um educador é desenvolvido,
no sentido da realização de ações que, efetivamente, considerem o caráter de sujeitos em condição de
desenvolvimento – em que se encontram os jovens sob seu cuidado – e tenham como perspectiva a sua
reinserção social.
Ainda em relação aos educadores, ressalte-se a inexistência de uma história de vida que contemple
processos de formação por meio do qual poderiam, realmente, serem denominados educadores. O que
se observa é a manutenção dos mesmos procedimentos de composição de equipes empregados na
época das doutrinas do Direito do Menor e da Situação Irregular, em que a preocupação básica era o
encarceramento e a repressão dos internos. A alocação de pessoal para as referidas instituições não
obedece à exigência de critérios mínimos para o trabalho educativo com jovens na condição específica
de em conflito com a lei, identificando-se atualmente nas unidades de atendimento pessoas, por exemplo,
cuja função anterior era de vigia; outras que desempenhavam atividades em unidades como creche,
abrigo etc., e que foram realocadas contra sua vontade; e mesmo outras que, apesar de concursadas
especificamente para o cargo em questão, não demonstram interesse em atuar com jovens nas condições
referidas.
Por outro lado, a ciência Psicológica, produzindo, aderindo a, e mesmo fortalecendo, um discurso que,
com raras exceções, desconsidera questões da ordem do socio-histórico-cultural, cria um conjunto de
dispositivos (teorias, instrumentos de medida e avaliação psicológicas, dentre outros) que, por um lado,
não ultrapassa os limites de um sujeito individualizado e, por outro, fica circunscrito ao reduto “psi”. Em
relação ao primeiro aspecto, assim se posicionando, a prática do psicólogo no interior da instituição de
cumprimento das medidas socioeducativas restringe-se, não raro, à utilização das referidas técnicas de
medida e avaliação, com o objetivo de emitir laudo psicológico, que via de regra funciona como
instrumento de discriminação e opressão. No entanto, é cada vez mais evidente a necessidade de um
profissional que considere o campo da subjetividade no enfrentamento da problemática da violência,
investigando sua constituição, seu desenvolvimento.
Sobre a redução da compreensão dos processos de subjetivação ao campo “psi”, é necessário
observar que essa compreensão, conforme apontado por Miranda (2000), requer uma concepção
transversalista que, necessariamente, considere os mais diversos campos do saber que se (pre)ocupam
com a constituição do sujeito. Assim, a definição das ações educativas institucionalizadas com o objetivo
de produzir novos padrões de socialização, portanto, novos processos de subjetivação, deve pautar-se
no entrelaçamento dos saberes produzidos pela Psicologia, Sociologia, Pedagogia, Arte, História, dentre
outros.
Ainda, no tocante às ações a serem desenvolvidas no interior das instituições, com vistas à
ressocialização, se esta pressupõe convívio com a família e a comunidade, (re)ingresso no sistema
escolar e o exercício de uma profissão, então se espera que tais ações efetivamente reflitam tal
perspectiva socializadora. Contudo, a literatura da área tem revelado uma quase total ausência de ações
que contemplem os aspectos esperados. A falta de um planejamento que integre o conjunto das
atividades reflete-se em projetos isolados, que geralmente resumem-se na oferta de oficinas, como
serigrafia, panificação, artesanato etc., as quais carecem de uma proposta pedagógica que oportunize,
além da atividade física, ocupacional, a formação do adolescente para a cidadania, como preconiza o
ECA (Costa, 1997; Volpi, 1997).
Em relação às oficinas, que aqui requerem atenção particular por ocuparem lugar de destaque nas
ações institucionais, observe-se que nem mesmo a habilitação profissional dos adolescentes é garantida
por seu intermédio. Esta situação decorre, ao menos, de dois aspectos essenciais: primeiro, da restrição
de opção dos jovens quanto a que trabalho desenvolver, pois nem sempre se interessam pelo tipo de
atividade que é proposto. E segundo, porque as atividades não consideram as especificidades dos

Apostila gerada especialmente para: Betânia Mueller 023.712.580-32


. 135
sujeitos a quem se destinam: a peculiar condição de pessoas em desenvolvimento, o particular momento
da adolescência e a circunstância de se encontrarem em conflito com a lei. Ou seja, constata-se a
ausência de uma integração entre a atividade manual e a perspectiva pedagógica que remete à ética da
vida.
Neste contexto de análise, uma reflexão sobre a instituição escolar é reiterativa da precariedade do
sistema de cumprimento de medidas e da ausência de um projeto pedagógico amplo para suas unidades.
De maneira geral, quando existe uma escola no interior das mesmas, ela funciona precariamente, sem
pessoal qualificado, sem vínculo com o sistema formal e sem qualquer tipo de regulamentação. No
tocante ao trabalho com pessoas do convívio dos adolescentes, como referido anteriormente, observa-
se a quase absoluta falta de estrutura para lidar com as famílias e com outros membros da comunidade
de origem, como amigos, vizinhos etc. Ou seja, tanto se lida com uma arquitetura inadequada quanto
com a falta de formação dos educadores das unidades de atendimento para desenvolverem ações dessa
natureza (Poglia et al; 2001). Ressalte-se que as condições de acompanhamento dos jovens egressos,
trabalho importantíssimo a ser desenvolvido, não deixa nada a dever ao precário quadro antes descrito.
Para ilustrar esse quadro de carências, inadequações, despreparo e insensibilidade, recorre-se ao
depoimento de adolescentes privados de liberdade. Com base em tais depoimentos, tudo leva a crer que,
se as desigualdades sociais são relativamente deixadas de lado, outras tomam seu lugar e oprimem da
mesma forma. Dessa maneira, não parece que há um contexto adequado para se trabalhar em prol de
uma ética do respeito à vida.
Dizem eles que:
Aqui a gente passa o dia na cela, sem ter o que fazer a não ser escutar rádio e ver televisão. (...) Aqui
dentro é um mundo, lá fora é outro. Tenho muita vontade de mudar. A pessoa sai com vontade de mudar,
mas a sociedade não deixa. A minha recuperação depende de mim e também da sociedade (A. T., 18
anos), (Parcelle, 2002, p. 41).
O CEDUC é uma prisão. Era para ter alguma coisa para recuperar a gente. Era para ser tipo uma
escola para a gente aprender e quando sair ter um trabalho. Ocupo meu tempo só pensando. Fico
pensando no meu futuro, como é que vai ser. Vou ver se tento reconstruir minha vida. Acho que não estou
tendo possibilidade. A gente não pode construir um caminho sozinho, tem de ter ajuda (G. C., 16 anos),
(Parcelle, 2002, p. 47).
Para concluir, concorda-se com Constantino, quando afirma que “A instituição pretende ajustar o
indivíduo à sociedade, mas acaba produzindo o efeito contrário, o de reafirmação de sua marginalidade”
(Constantino, 2000, p. 28). Como expressão dessa condição, é possível observar o progressivo aumento
do número de jovens que reincidem nas suas transgressões, comprometendo cada vez mais as já
pequenas possibilidades de reinserção.

Questões

01. (Pref. São Luís/MA -Técnico Municipal Nível Superior - CESPE/2017). O Conselho Nacional de
Assistência Social
(A) é um órgão de caráter permanente e de deliberação colegiada.
(B) é presidido por um representante dos usuários indicado pelo Ministério Público.
(C) tem a competência de fixar normas, conceder registro e avaliar as entidades prestadoras de
serviços de assistência social.
(D) é responsável pela execução da política nacional de assistência social.
(E) permite a recondução de seus membros, cujo mandato é de três anos.

02. (UFF - Assistente Social – COSEAC). O Conselho Nacional de Assistência Social está ligado
diretamente ao(à):
(A) Ministério de Integração Regional e Interior.
(B) Secretaria Geral da Presidência da República.
(C) Ministério do Trabalho, Emprego e Renda.
(D) Ministério de Desenvolvimento Social e Combate à Fome.
(E) Secretaria de Direitos Humanos e Cidadania.

Gabarito

01.A / 02.D

Apostila gerada especialmente para: Betânia Mueller 023.712.580-32


. 136
Comentários
01. Resposta: A
O CNAS, instituído pela LOAS, e os Conselhos das demais esferas, tratados na referida Lei e
instituídos por legislação especifica, têm caráter permanente e composição paritária entre governo e
sociedade civil. São vinculados ao Poder Executivo e a sua estrutura pertencente ao órgão da
Administração Pública responsável pela coordenação da Política de Assistência Social, que lhes dá apoio
administrativo, assegurando dotação orçamentária para seu funcionamento.

02. Resposta: D
O Suplemento de Assistência Social da Pesquisa de Informações Básicas Municipais, realizada pelo
IBGE em parceria com o Ministério de Desenvolvimento Social e Combate à Fome - MUNIC-20057.

Lei nº 12.594/12 – SINASE.

SINASE – SISTEMA NACIONAL DE ATENDIMENTO SOCIOEDUCATIVO

Considerações Iniciais:

A Lei Federal nº 12.594/2012 institui o Sistema Nacional de Atendimento Socioeducativo – SINASE, e


regulamenta a execução das medidas destinadas a adolescentes que pratiquem ato infracional.
Os legisladores assim definiram o SINASE: o conjunto ordenado de princípios, regras e critérios que
envolvem a execução de medidas socioeducativas, incluindo-se nele, os sistemas estaduais, distrital e
municipais, bem como todos os planos, políticas e programas específicos de atendimento a adolescente
em conflito com a lei.
Importante esclarecer que o SINASE é coordenado pela União, com integração com os sistemas
estaduais, distrital e municipais, responsáveis pela implementação de seus programas de atendimento
aos adolescente em cumprimento de medida socioeducativa, com autonomia de organização e
funcionamento, desde que respeitados os termos da Lei.

OBJETIVO DAS MEDIDAS SOCIOEDUCATIVAS:

O Estatuto da Criança e Adolescente, em seu artigo 112, prevê as medidas socioeducativas a serem
aplicadas aos adolescentes que cometam ato infracional.
A aplicação das medidas socioeducativas possuem os seguintes objetivos principais:
- a responsabilização do adolescente quanto às consequências lesivas do ato infracional, sempre que
possível incentivando a sua reparação;
- a integração social do adolescente e a garantia de seus direitos individuais e sociais, por meio do
cumprimento de seu plano individual de atendimento; e
- a desaprovação da conduta infracional, efetivando as disposições da sentença como parâmetro
máximo de privação de liberdade ou restrição de direitos, observados os limites previstos em lei.

DAS COMPETÊNCIAS:

Diante da instituição do SINASE em âmbito nacional, a Lei 12.594/12 estabelece quais são as
competências de atuação entre os entes da Federação (União, Estados, Municípios e Distrito Federal),
que assim se subdivide:

a) Compete à União:

- formular e coordenar a execução da política nacional de atendimento socioeducativo;


- elaborar o Plano Nacional de Atendimento Socioeducativo, em parceria com os Estados, o Distrito
Federal e os Municípios;
- prestar assistência técnica e suplementação financeira aos Estados, ao Distrito Federal e aos
Municípios para o desenvolvimento de seus sistemas;
- instituir e manter o Sistema Nacional de Informações sobre o Atendimento Socioeducativo, seu
funcionamento, entidades, programas, incluindo dados relativos a financiamento e população atendida;

Apostila gerada especialmente para: Betânia Mueller 023.712.580-32


. 137
- contribuir para a qualificação e ação em rede dos Sistemas de Atendimento Socioeducativo;
- estabelecer diretrizes sobre a organização e funcionamento das unidades e programas de
atendimento e as normas de referência destinadas ao cumprimento das medidas socioeducativas de
internação e semiliberdade;
- instituir e manter processo de avaliação dos Sistemas de Atendimento Socioeducativo, seus planos,
entidades e programas;
- financiar, com os demais entes federados, a execução de programas e serviços do Sinase; e
- garantir a publicidade de informações sobre repasses de recursos aos gestores estaduais, distrital e
municipais, para financiamento de programas de atendimento socioeducativo.

b) Compete aos Estados:

- formular, instituir, coordenar e manter Sistema Estadual de Atendimento Socioeducativo, respeitadas


as diretrizes fixadas pela União;
- elaborar o Plano Estadual de Atendimento Socioeducativo em conformidade com o Plano Nacional;
- criar, desenvolver e manter programas para a execução das medidas socioeducativas de
semiliberdade e internação;
- editar normas complementares para a organização e funcionamento do seu sistema de atendimento
e dos sistemas municipais;
- estabelecer com os Municípios formas de colaboração para o atendimento socioeducativo em meio
aberto;
- prestar assessoria técnica e suplementação financeira aos Municípios para a oferta regular de
programas de meio aberto;
- garantir o pleno funcionamento do plantão interinstitucional, nos termos previstos no inciso V do art.
88 do Estatuto da Criança e do Adolescente;
- garantir defesa técnica do adolescente a quem se atribua prática de ato infracional;
- cadastrar-se no Sistema Nacional de Informações sobre o Atendimento Socioeducativo e fornecer
regularmente os dados necessários ao povoamento e à atualização do Sistema; e
- cofinanciar, com os demais entes federados, a execução de programas e ações destinados ao
atendimento inicial de adolescente apreendido para apuração de ato infracional, bem como aqueles
destinados a adolescente a quem foi aplicada medida socioeducativa privativa de liberdade.

c) Compete aos Municípios:

- formular, instituir, coordenar e manter o Sistema Municipal de Atendimento Socioeducativo,


respeitadas as diretrizes fixadas pela União e pelo respectivo Estado;
- elaborar o Plano Municipal de Atendimento Socioeducativo, em conformidade com o Plano Nacional
e o respectivo Plano Estadual;
- criar e manter programas de atendimento para a execução das medidas socioeducativas em meio
aberto;
- editar normas complementares para a organização e funcionamento dos programas do seu Sistema
de Atendimento Socioeducativo;
- cadastrar-se no Sistema Nacional de Informações sobre o Atendimento Socioeducativo e fornecer
regularmente os dados necessários ao povoamento e à atualização do Sistema; e
- cofinanciar, conjuntamente com os demais entes federados, a execução de programas e ações
destinados ao atendimento inicial de adolescente apreendido para apuração de ato infracional, bem como
aqueles destinados a adolescente a quem foi aplicada medida socioeducativa em meio aberto.

d) Compete ao Distrito Federal:

Ao Distrito Federal cabem, cumulativamente, as competências dos Estados e dos Municípios.

DOS PROGRAMAS DE ATENDIMENTO:

Dos Programas de Meio Aberto: Nos programas de atendimento de adolescentes infratores cujo
cumprimento das medidas socioeducativas devem ocorrer em meio aberto, a Lei 12.594/12, estabelece
as competências e atribuições à direção do programa de prestação de serviço à comunidade ou liberdade
assistida, e assim determina:

Apostila gerada especialmente para: Betânia Mueller 023.712.580-32


. 138
- selecionar e credenciar orientadores, designando-os, caso a caso, para acompanhar e avaliar o
cumprimento da medida;
- receber o adolescente e seus pais ou responsável e orientá-los sobre a finalidade da medida e a
organização e funcionamento do programa;
- encaminhar o adolescente para o orientador credenciado;
- supervisionar o desenvolvimento da medida; e
- avaliar, com o orientador, a evolução do cumprimento da medida e, se necessário, propor à
autoridade judiciária sua substituição, suspensão ou extinção.
Será ainda atribuição da direção do programa de medida aplicada ao adolescente em regime aberto
de prestação de serviço à comunidade selecionar e credenciar entidades assistenciais, hospitais, escolas
ou outros estabelecimentos semelhantes, bem como os programas comunitários ou governamentais, de
acordo com o perfil do socioeducando e o ambiente no qual a medida será cumprida.

Dos Programas de Privação de Liberdade: Quando, pela natureza da infração, o adolescente infrator
necessitar de medida socioeducativa em programas de privação de sua liberdade, a Lei 12.594/12,
estabeleceu alguns requisitos específicos para a inscrição de entidades que suportarão os regimes de
semiliberdade ou internação, quais sejam:
- a comprovação da existência de estabelecimento educacional com instalações adequadas e em
conformidade com as normas de referência;
- a previsão do processo e dos requisitos para a escolha do dirigente;
- a apresentação das atividades de natureza coletiva;
- a definição das estratégias para a gestão de conflitos, vedada a previsão de isolamento cautelar,
exceto nos casos previstos em lei; e
- a previsão de regime disciplinar nos termos legais.
Ademais, é proibida a construção e edificação de unidades socioeducacionais em espaços contíguos,
anexos, ou de qualquer outra forma integrados a estabelecimentos penais.
Para o preenchimento do cargo de dirigente de instituição apta a receber programa de atendimento
em regime de semiliberdade ou de internação, além dos requisitos específicos previstos no respectivo
programa de atendimento, é necessário:
A) formação de nível superior compatível com a natureza da função;
B) comprovada experiência no trabalho com adolescentes de, no mínimo, 2 (dois) anos; e
C) reputação ilibada.
Os Estados e o Distrito Federal inscreverão seus programas de atendimento e alterações no Conselho
Estadual ou Distrital dos Direitos da Criança e do Adolescente, conforme o caso, enquanto os Municípios
inscreverão seus programas e alterações, bem como as entidades de atendimento executoras, no
Conselho Municipal dos Direitos da Criança e do Adolescente, tais regras se aplicam tanto para os
programas de cumprimento em regime aberto, como também para os programas que necessitem da
privação da liberdade do adolescente infrator.
A composição da equipe técnica do programa de atendimento deverá ser interdisciplinar,
compreendendo, no mínimo, profissionais das áreas de saúde, educação e assistência social, de acordo
com as normas de referência.

DA RESPONSABILIZAÇÃO DOS GESTORES, OPERADORES E ENTIDADES DE ATENDIMENTO:

A Lei 12.594/12 estabelece as diretrizes e determinações a serem cumpridas pelas Entidades de


Atendimento a Adolescentes, bem como regulamenta a conduta dos Gestores e Operadores.
Dessa forma, no caso de desrespeito, mesmo que parcial, de tais normas, seja em que esfera for,
estão sujeitos:
- gestores, operadores e seus prepostos e entidades governamentais às medidas previstas no inciso I
e no § 1º do art. 97 do Estatuto da Criança e do Adolescente
- entidades não governamentais, seus gestores, operadores e prepostos às medidas previstas no
inciso II e no § 1º do art. 97 do Estatuto da Criança e do Adolescente

DA EXECUÇÃO DAS MEDIDAS SOCIOEDUCATIVAS: PRINCÍPIOS GERAIS:

Durante a execução das medidas sócio educativas, deverá ser observados os seguintes princípio:
- legalidade, não podendo o adolescente receber tratamento mais gravoso do que o conferido ao
adulto;

Apostila gerada especialmente para: Betânia Mueller 023.712.580-32


. 139
- excepcionalidade da intervenção judicial e da imposição de medidas, favorecendo-se meios de auto
composição de conflitos;
- prioridade a práticas ou medidas que sejam restaurativas e, sempre que possível, atendam às
necessidades das vítimas;
- proporcionalidade da medida aplicada em relação à ofensa cometida;
- brevidade da medida em resposta ao ato cometido, em especial o respeito ao que dispõe o art. 122
do Estatuto da Criança e do Adolescente;
- individualização, considerando-se a idade, capacidades e circunstâncias pessoais do adolescente;
- mínima intervenção, restrita ao necessário para a realização dos objetivos da medida;
- não discriminação do adolescente, notadamente em razão de etnia, gênero, nacionalidade, classe
social, orientação religiosa, política ou sexual, ou associação ou pertencimento a qualquer minoria ou
status; e
- fortalecimento dos vínculos familiares e comunitários no processo socioeducativo.

DOS PROCEDIMENTOS:

Em todo e qualquer procedimento de apuração de infração cometida por adolescente deverá obedecer
aos preceitos do Estatuto da Criança e do Adolescente (ECA) sendo certo que a defesa e o Ministério
Público intervirão, sob pena de nulidade, no procedimento judicial de execução de medida socioeducativa,
podendo requerer as providências necessárias para adequar a execução aos ditames legais e
regulamentares.
Para aplicação das medidas socioeducativas de prestação de serviços à comunidade, liberdade
assistida, semiliberdade ou internação, será constituído um processo de execução para cada
adolescente, respeitado o disposto nos arts. 143 e 144 do Estatuto da Criança e do Adolescente, e com
autuação das seguintes peças:
- documentos de caráter pessoal do adolescente existentes no processo de conhecimento,
especialmente os que comprovem sua idade; e
- as indicadas pela autoridade judiciária, sempre que houver necessidade e, obrigatoriamente:
a) cópia da representação;
b) cópia da certidão de antecedentes;
c) cópia da sentença ou acórdão; e
d) cópia de estudos técnicos realizados durante a fase de conhecimento.
Das Reavaliações: As medidas socioeducativas de liberdade assistida, de semiliberdade e de
internação deverão ser reavaliadas no máximo a cada 6 (seis) meses, podendo a autoridade judiciária,
se necessário, designar audiência, no prazo máximo de 10 (dez) dias, cientificando o defensor, o
Ministério Público, a direção do programa de atendimento, o adolescente e seus pais ou responsável.
Entretanto, a reavaliação da manutenção, da substituição ou da suspensão das medidas de meio
aberto ou de privação da liberdade e do respectivo plano individual pode ser solicitada a qualquer tempo,
a pedido da direção do programa de atendimento, do defensor, do Ministério Público, do adolescente, de
seus pais ou responsável, devendo conter no pedido de reavaliação, dentre outros motivos:
- o desempenho adequado do adolescente com base no seu plano de atendimento individual, antes
do prazo da reavaliação obrigatória;
- a inadaptação do adolescente ao programa e o reiterado descumprimento das atividades do plano
individual; e
- a necessidade de modificação das atividades do plano individual que importem em maior restrição
da liberdade do adolescente.

Extinção da execução da medida socioeducativa:


A medida socioeducativa será declarada extinta quanto ocorrer qualquer das seguintes hipóteses:
- pela morte do adolescente;
- pela realização e conclusão de sua finalidade;
- pela aplicação de pena privativa de liberdade, a ser cumprida em regime fechado ou semiaberto, em
execução provisória ou definitiva;
- pela condição de doença grave, que torne o adolescente incapaz de submeter-se ao cumprimento
da medida; e
- nas demais hipóteses previstas em lei.

Apostila gerada especialmente para: Betânia Mueller 023.712.580-32


. 140
DOS DIREITOS INDIVIDUAIS:

São direitos invioláveis do adolescente submetido ao cumprimento de medida socioeducativa, sem


prejuízo de outros previstos em lei:
- ser acompanhado por seus pais ou responsável e por seu defensor, em qualquer fase do
procedimento administrativo ou judicial;
- ser incluído em programa de meio aberto quando inexistir vaga para o cumprimento de medida de
privação da liberdade, exceto nos casos de ato infracional cometido mediante grave ameaça ou violência
à pessoa, quando o adolescente deverá ser internado em Unidade mais próxima de seu local de
residência;
- ser respeitado em sua personalidade, intimidade, liberdade de pensamento e religião e em todos os
direitos não expressamente limitados na sentença;
- peticionar, por escrito ou verbalmente, diretamente a qualquer autoridade ou órgão público, devendo,
obrigatoriamente, ser respondido em até 15 (quinze) dias;
- ser informado, inclusive por escrito, das normas de organização e funcionamento do programa de
atendimento e também das previsões de natureza disciplinar;
- receber, sempre que solicitar, informações sobre a evolução de seu plano individual, participando,
obrigatoriamente, de sua elaboração e, se for o caso, reavaliação;
- receber assistência integral à sua saúde, conforme o disposto no art. 60 desta Lei; e
- ter atendimento garantido em creche e pré-escola aos filhos de 0 (zero) a 5 (cinco) anos.

PIA – PLANO INDIVIDUAL DE ATENDIMENTO

O cumprimento e execução das medidas socioeducativas, em regime de prestação de serviços à


comunidade, liberdade assistida, semiliberdade ou internação, dependerá da realização de Plano
Individual de Atendimento (PIA).
O Plano Individual de Atendimento é o instrumento de previsão, registro e gestão das atividades a
serem desenvolvidas com o adolescente, ao qual deverá contemplas a participação dos pais ou
responsáveis, os quais têm o dever de contribuir com o processo ressocializador do adolescente, sendo
esses passíveis de responsabilização administrativa, nos termos do Estatuto da Criança e Adolescente.
É de responsabilidade da equipe técnica do respectivo estabelecimento do programa de atendimento
a elaboração do PIA, com a participação efetiva do adolescente e de sua família, representada por seus
pais ou responsável.
Deverão constar no Plano Individual de Atendimento no mínimo:
- os resultados da avaliação interdisciplinar;
- os objetivos declarados pelo adolescente;
- a previsão de suas atividades de integração social e/ou capacitação profissional;
- atividades de integração e apoio à família;
- formas de participação da família para efetivo cumprimento do plano individual; e
- as medidas específicas de atenção à sua saúde.
Para o cumprimento das medidas de semiliberdade ou de internação, o Plano Individual de
Atendimento deverá conter ainda:
- a designação do programa de atendimento mais adequado para o cumprimento da medida;
- a definição das atividades internas e externas, individuais ou coletivas, das quais o adolescente
poderá participar; e
- a fixação das metas para o alcance de desenvolvimento de atividades externas.
Importante esclarecer que o acesso ao plano individual será restrito aos servidores do respectivo
programa de atendimento, ao adolescente e a seus pais ou responsável, ao Ministério Público e ao
defensor, exceto expressa autorização judicial, objetivando assim resguardar o sigilo das informações do
adolescente, nos termos do ECA.

DAS VISITAS AOS ADOLESCENTES EM CUMPRIMENTO DE MEDIDA DE INTERNAÇÃO:

É permitido e assegurado ao adolescente a quem foi aplicado a medida socioeducativa de internação


a visita do cônjuge, companheiro, pais ou responsáveis, parentes e amigos, observado e respeitado os
dias e horários próprios, definidos pela direção da entidade de internação.
É assegurado ainda ao adolescente casado ou que viva em união estável, desde que
comprovadamente, o direito à visita íntima, sendo que o visitante será identificado e registrado pela

Apostila gerada especialmente para: Betânia Mueller 023.712.580-32


. 141
direção do programa de atendimento, que emitirá documento de identificação, pessoal e intransferível,
específico para a realização da visita íntima.
Caso o adolescente em cumprimento de medida socioeducativa de internação tenha filhos, é garantido
o direito de receber visitas de seus filhos independente da idade desses.

DOS REGIMES DISCIPLINARES:

Nos termos da Lei 12.594/12, todas as entidades de atendimento socioeducativo deverão, em seus
respectivos regimentos, realizar a previsão de regime disciplinar.
O Regime Disciplinar das entidades de atendimento socioeducativo deverá obedecer os seguintes
princípios:
- tipificação explícita das infrações como leves, médias e graves e determinação das correspondentes
sanções;
- exigência da instauração formal de processo disciplinar para a aplicação de qualquer sanção,
garantidos a ampla defesa e o contraditório;
- obrigatoriedade de audiência do socioeducando nos casos em que seja necessária a instauração de
processo disciplinar;
- sanção de duração determinada;
- enumeração das causas ou circunstâncias que eximam, atenuem ou agravem a sanção a ser imposta
ao socioeducando, bem como os requisitos para a extinção dessa;
- enumeração explícita das garantias de defesa;

- garantia de solicitação e rito de apreciação dos recursos cabíveis; e


- apuração da falta disciplinar por comissão composta por, no mínimo, 3 (três) integrantes, sendo 1
(um), obrigatoriamente, oriundo da equipe técnica.
Não será aplicada sanção disciplinar sem expressa e anterior previsão legal ou regulamentar e o
devido processo administrativo.
E ainda não será aplicada sanção disciplinar ao socioeducando que tenha praticado a falta:
- por coação irresistível ou por motivo de força maior;
- em legítima defesa, própria ou de outrem.

Acompanhe o que traz a Lei 12.594/2012

LEI Nº 12.594, DE 18 DE JANEIRO DE 2012.

Institui o Sistema Nacional de Atendimento Socioeducativo (Sinase), regulamenta a execução das


medidas socioeducativas destinadas a adolescente que pratique ato infracional; e altera as Leis nºs 8.069,
de 13 de julho de 1990 (Estatuto da Criança e do Adolescente); 7.560, de 19 de dezembro de 1986, 7.998,
de 11 de janeiro de 1990, 5.537, de 21 de novembro de 1968, 8.315, de 23 de dezembro de 1991, 8.706,
de 14 de setembro de 1993, os Decretos-Leis nos 4.048, de 22 de janeiro de 1942, 8.621, de 10 de janeiro
de 1946, e a Consolidação das Leis do Trabalho (CLT), aprovada pelo Decreto-Lei no 5.452, de 1o de
maio de 1943.

A PRESIDENTA DA REPÚBLICA Faço saber que o Congresso Nacional decreta e eu sanciono a


seguinte Lei:

TÍTULO I
DO SISTEMA NACIONAL DE ATENDIMENTO SOCIOEDUCATIVO (Sinase)
CAPÍTULO I
DISPOSIÇÕES GERAIS

Art. 1º Esta Lei institui o Sistema Nacional de Atendimento Socioeducativo (Sinase) e regulamenta a
execução das medidas destinadas a adolescente que pratique ato infracional.
§ 1º Entende-se por Sinase o conjunto ordenado de princípios, regras e critérios que envolvem a
execução de medidas socioeducativas, incluindo-se nele, por adesão, os sistemas estaduais, distrital e
municipais, bem como todos os planos, políticas e programas específicos de atendimento a adolescente
em conflito com a lei.
§ 2º Entendem-se por medidas socioeducativas as previstas no art. 112 da Lei nº 8.069, de 13 de julho
de 1990 (Estatuto da Criança e do Adolescente), as quais têm por objetivos:

Apostila gerada especialmente para: Betânia Mueller 023.712.580-32


. 142
I - a responsabilização do adolescente quanto às consequências lesivas do ato infracional, sempre que
possível incentivando a sua reparação;
II - a integração social do adolescente e a garantia de seus direitos individuais e sociais, por meio do
cumprimento de seu plano individual de atendimento; e
III - a desaprovação da conduta infracional, efetivando as disposições da sentença como parâmetro
máximo de privação de liberdade ou restrição de direitos, observados os limites previstos em lei.
§ 3º Entendem-se por programa de atendimento a organização e o funcionamento, por unidade, das
condições necessárias para o cumprimento das medidas socioeducativas.
§ 4º Entende-se por unidade a base física necessária para a organização e o funcionamento de
programa de atendimento.
§ 5º Entendem-se por entidade de atendimento a pessoa jurídica de direito público ou privado que
instala e mantém a unidade e os recursos humanos e materiais necessários ao desenvolvimento de
programas de atendimento.

Art. 2º O Sinase será coordenado pela União e integrado pelos sistemas estaduais, distrital e
municipais responsáveis pela implementação dos seus respectivos programas de atendimento a
adolescente ao qual seja aplicada medida socioeducativa, com liberdade de organização e
funcionamento, respeitados os termos desta Lei.

CAPÍTULO II
DAS COMPETÊNCIAS

Art. 3º Compete à União:


I - formular e coordenar a execução da política nacional de atendimento socioeducativo;
II - elaborar o Plano Nacional de Atendimento Socioeducativo, em parceria com os Estados, o Distrito
Federal e os Municípios;
III - prestar assistência técnica e suplementação financeira aos Estados, ao Distrito Federal e aos
Municípios para o desenvolvimento de seus sistemas;
IV - instituir e manter o Sistema Nacional de Informações sobre o Atendimento Socioeducativo, seu
funcionamento, entidades, programas, incluindo dados relativos a financiamento e população atendida;
V - contribuir para a qualificação e ação em rede dos Sistemas de Atendimento Socioeducativo;
VI - estabelecer diretrizes sobre a organização e funcionamento das unidades e programas de
atendimento e as normas de referência destinadas ao cumprimento das medidas socioeducativas de
internação e semiliberdade;
VII - instituir e manter processo de avaliação dos Sistemas de Atendimento Socioeducativo, seus
planos, entidades e programas;
VIII - financiar, com os demais entes federados, a execução de programas e serviços do Sinase; e
IX - garantir a publicidade de informações sobre repasses de recursos aos gestores estaduais, distrital
e municipais, para financiamento de programas de atendimento socioeducativo.
§ 1º São vedados à União o desenvolvimento e a oferta de programas próprios de atendimento.
§ 2º Ao Conselho Nacional dos Direitos da Criança e do Adolescente (Conanda) competem as funções
normativa, deliberativa, de avaliação e de fiscalização do Sinase, nos termos previstos na Lei no 8.242,
de 12 de outubro de 1991, que cria o referido Conselho.
§ 3º O Plano de que trata o inciso II do caput deste artigo será submetido à deliberação do Conanda.
§ 4º À Secretaria de Direitos Humanos da Presidência da República (SDH/PR) competem as funções
executiva e de gestão do Sinase.

Art. 4º Compete aos Estados:


I - formular, instituir, coordenar e manter Sistema Estadual de Atendimento Socioeducativo,
respeitadas as diretrizes fixadas pela União;
II - elaborar o Plano Estadual de Atendimento Socioeducativo em conformidade com o Plano Nacional;
III - criar, desenvolver e manter programas para a execução das medidas socioeducativas de
semiliberdade e internação;
IV - editar normas complementares para a organização e funcionamento do seu sistema de
atendimento e dos sistemas municipais;
V - estabelecer com os Municípios formas de colaboração para o atendimento socioeducativo em meio
aberto;
VI - prestar assessoria técnica e suplementação financeira aos Municípios para a oferta regular de
programas de meio aberto;

Apostila gerada especialmente para: Betânia Mueller 023.712.580-32


. 143
VII - garantir o pleno funcionamento do plantão interinstitucional, nos termos previstos no inciso V do
art. 88 da Lei no 8.069, de 13 de julho de 1990 (Estatuto da Criança e do Adolescente);
VIII - garantir defesa técnica do adolescente a quem se atribua prática de ato infracional;
IX - cadastrar-se no Sistema Nacional de Informações sobre o Atendimento Socioeducativo e fornecer
regularmente os dados necessários ao povoamento e à atualização do Sistema; e
X - cofinanciar, com os demais entes federados, a execução de programas e ações destinados ao
atendimento inicial de adolescente apreendido para apuração de ato infracional, bem como aqueles
destinados a adolescente a quem foi aplicada medida socioeducativa privativa de liberdade.
§ 1º Ao Conselho Estadual dos Direitos da Criança e do Adolescente competem as funções
deliberativas e de controle do Sistema Estadual de Atendimento Socioeducativo, nos termos previstos no
inciso II do art. 88 da Lei nº 8.069, de 13 de julho de 1990 (Estatuto da Criança e do Adolescente), bem
como outras definidas na legislação estadual ou distrital.
§ 2º O Plano de que trata o inciso II do caput deste artigo será submetido à deliberação do Conselho
Estadual dos Direitos da Criança e do Adolescente.
§ 3º Competem ao órgão a ser designado no Plano de que trata o inciso II do caput deste artigo as
funções executiva e de gestão do Sistema Estadual de Atendimento Socioeducativo.

Art. 5º Compete aos Municípios:


I - formular, instituir, coordenar e manter o Sistema Municipal de Atendimento Socioeducativo,
respeitadas as diretrizes fixadas pela União e pelo respectivo Estado;
II - elaborar o Plano Municipal de Atendimento Socioeducativo, em conformidade com o Plano Nacional
e o respectivo Plano Estadual;
III - criar e manter programas de atendimento para a execução das medidas socioeducativas em meio
aberto;
IV - editar normas complementares para a organização e funcionamento dos programas do seu
Sistema de Atendimento Socioeducativo;
V - cadastrar-se no Sistema Nacional de Informações sobre o Atendimento Socioeducativo e fornecer
regularmente os dados necessários ao povoamento e à atualização do Sistema; e
VI - cofinanciar, conjuntamente com os demais entes federados, a execução de programas e ações
destinados ao atendimento inicial de adolescente apreendido para apuração de ato infracional, bem como
aqueles destinados a adolescente a quem foi aplicada medida socioeducativa em meio aberto.
§ 1º Para garantir a oferta de programa de atendimento socioeducativo de meio aberto, os Municípios
podem instituir os consórcios dos quais trata a Lei no 11.107, de 6 de abril de 2005, que dispõe sobre
normas gerais de contratação de consórcios públicos e dá outras providências, ou qualquer outro
instrumento jurídico adequado, como forma de compartilhar responsabilidades.
§ 2º Ao Conselho Municipal dos Direitos da Criança e do Adolescente competem as funções
deliberativas e de controle do Sistema Municipal de Atendimento Socioeducativo, nos termos previstos
no inciso II do art. 88 da Lei nº 8.069, de 13 de julho de 1990 (Estatuto da Criança e do Adolescente),
bem como outras definidas na legislação municipal.
§ 3º O Plano de que trata o inciso II do caput deste artigo será submetido à deliberação do Conselho
Municipal dos Direitos da Criança e do Adolescente.
§ 4º Competem ao órgão a ser designado no Plano de que trata o inciso II do caput deste artigo as
funções executiva e de gestão do Sistema Municipal de Atendimento Socioeducativo.

Art. 6º Ao Distrito Federal cabem, cumulativamente, as competências dos Estados e dos Municípios.

CAPÍTULO III
DOS PLANOS DE ATENDIMENTO SOCIOEDUCATIVO

Art. 7º O Plano de que trata o inciso II do art. 3o desta Lei deverá incluir um diagnóstico da situação
do Sinase, as diretrizes, os objetivos, as metas, as prioridades e as formas de financiamento e gestão
das ações de atendimento para os 10 (dez) anos seguintes, em sintonia com os princípios elencados na
Lei nº 8.069, de 13 de julho de 1990 (Estatuto da Criança e do Adolescente).
§ 1º As normas nacionais de referência para o atendimento socioeducativo devem constituir anexo ao
Plano de que trata o inciso II do art. 3o desta Lei.
§ 2º Os Estados, o Distrito Federal e os Municípios deverão, com base no Plano Nacional de
Atendimento Socioeducativo, elaborar seus planos decenais correspondentes, em até 360 (trezentos e
sessenta) dias a partir da aprovação do Plano Nacional.

Apostila gerada especialmente para: Betânia Mueller 023.712.580-32


. 144
Art. 8º Os Planos de Atendimento Socioeducativo deverão, obrigatoriamente, prever ações articuladas
nas áreas de educação, saúde, assistência social, cultura, capacitação para o trabalho e esporte, para os
adolescentes atendidos, em conformidade com os princípios elencados na Lei nº 8.069, de 13 de julho
de 1990 (Estatuto da Criança e do Adolescente).
Parágrafo único. Os Poderes Legislativos federal, estaduais, distrital e municipais, por meio de suas
comissões temáticas pertinentes, acompanharão a execução dos Planos de Atendimento Socioeducativo
dos respectivos entes federados.

CAPÍTULO IV
DOS PROGRAMAS DE ATENDIMENTO

Seção I
Disposições Gerais

Art. 9º Os Estados e o Distrito Federal inscreverão seus programas de atendimento e alterações no


Conselho Estadual ou Distrital dos Direitos da Criança e do Adolescente, conforme o caso.

Art. 10. Os Municípios inscreverão seus programas e alterações, bem como as entidades de
atendimento executoras, no Conselho Municipal dos Direitos da Criança e do Adolescente.

Art. 11. Além da especificação do regime, são requisitos obrigatórios para a inscrição de programa de
atendimento:
I - a exposição das linhas gerais dos métodos e técnicas pedagógicas, com a especificação das
atividades de natureza coletiva;
II - a indicação da estrutura material, dos recursos humanos e das estratégias de segurança
compatíveis com as necessidades da respectiva unidade;
III - regimento interno que regule o funcionamento da entidade, no qual deverá constar, no mínimo:
a) o detalhamento das atribuições e responsabilidades do dirigente, de seus prepostos, dos membros
da equipe técnica e dos demais educadores;
b) a previsão das condições do exercício da disciplina e concessão de benefícios e o respectivo
procedimento de aplicação; e
c) a previsão da concessão de benefícios extraordinários e enaltecimento, tendo em vista tornar público
o reconhecimento ao adolescente pelo esforço realizado na consecução dos objetivos do plano individual;
IV - a política de formação dos recursos humanos;
V - a previsão das ações de acompanhamento do adolescente após o cumprimento de medida
socioeducativa;
VI - a indicação da equipe técnica, cuja quantidade e formação devem estar em conformidade com as
normas de referência do sistema e dos conselhos profissionais e com o atendimento socioeducativo a ser
realizado; e
VII - a adesão ao Sistema de Informações sobre o Atendimento Socioeducativo, bem como sua
operação efetiva.
Parágrafo único. O não cumprimento do previsto neste artigo sujeita as entidades de atendimento, os
órgãos gestores, seus dirigentes ou prepostos à aplicação das medidas previstas no art. 97 da Lei nº
8.069, de 13 de julho de 1990 (Estatuto da Criança e do Adolescente).

Art. 12. A composição da equipe técnica do programa de atendimento deverá ser interdisciplinar,
compreendendo, no mínimo, profissionais das áreas de saúde, educação e assistência social, de acordo
com as normas de referência.
§ 1º Outros profissionais podem ser acrescentados às equipes para atender necessidades específicas
do programa.
§ 2º Regimento interno deve discriminar as atribuições de cada profissional, sendo proibida a
sobreposição dessas atribuições na entidade de atendimento.
§ 3º O não cumprimento do previsto neste artigo sujeita as entidades de atendimento, seus dirigentes
ou prepostos à aplicação das medidas previstas no art. 97 da Lei nº 8.069, de 13 de julho de 1990
(Estatuto da Criança e do Adolescente).

Apostila gerada especialmente para: Betânia Mueller 023.712.580-32


. 145
Seção II
Dos Programas de Meio Aberto

Art. 13. Compete à direção do programa de prestação de serviços à comunidade ou de liberdade


assistida:
I - selecionar e credenciar orientadores, designando-os, caso a caso, para acompanhar e avaliar o
cumprimento da medida;
II - receber o adolescente e seus pais ou responsável e orientá-los sobre a finalidade da medida e a
organização e funcionamento do programa;
III - encaminhar o adolescente para o orientador credenciado;
IV - supervisionar o desenvolvimento da medida; e
V - avaliar, com o orientador, a evolução do cumprimento da medida e, se necessário, propor à
autoridade judiciária sua substituição, suspensão ou extinção.
Parágrafo único. O rol de orientadores credenciados deverá ser comunicado, semestralmente, à
autoridade judiciária e ao Ministério Público.

Art. 14. Incumbe ainda à direção do programa de medida de prestação de serviços à comunidade
selecionar e credenciar entidades assistenciais, hospitais, escolas ou outros estabelecimentos
congêneres, bem como os programas comunitários ou governamentais, de acordo com o perfil do
socioeducando e o ambiente no qual a medida será cumprida.
Parágrafo único. Se o Ministério Público impugnar o credenciamento, ou a autoridade judiciária
considerá-lo inadequado, instaurará incidente de impugnação, com a aplicação subsidiária do
procedimento de apuração de irregularidade em entidade de atendimento regulamentado na Lei no 8.069,
de 13 de julho de 1990 (Estatuto da Criança e do Adolescente), devendo citar o dirigente do programa e
a direção da entidade ou órgão credenciado.

Seção III
Dos Programas de Privação da Liberdade

Art. 15. São requisitos específicos para a inscrição de programas de regime de semiliberdade ou
internação:
I - a comprovação da existência de estabelecimento educacional com instalações adequadas e em
conformidade com as normas de referência;
II - a previsão do processo e dos requisitos para a escolha do dirigente;
III - a apresentação das atividades de natureza coletiva;
IV - a definição das estratégias para a gestão de conflitos, vedada a previsão de isolamento cautelar,
exceto nos casos previstos no § 2o do art. 49 desta Lei; e
V - a previsão de regime disciplinar nos termos do art. 72 desta Lei.

Art. 16. A estrutura física da unidade deverá ser compatível com as normas de referência do Sinase.
§ 1º É vedada a edificação de unidades socioeducacionais em espaços contíguos, anexos, ou de
qualquer outra forma integrados a estabelecimentos penais.
§ 2º A direção da unidade adotará, em caráter excepcional, medidas para proteção do interno em
casos de risco à sua integridade física, à sua vida, ou à de outrem, comunicando, de imediato, seu
defensor e o Ministério Público.

Art. 17. Para o exercício da função de dirigente de programa de atendimento em regime de


semiliberdade ou de internação, além dos requisitos específicos previstos no respectivo programa de
atendimento, é necessário:
I - formação de nível superior compatível com a natureza da função;
II - comprovada experiência no trabalho com adolescentes de, no mínimo, 2 (dois) anos; e
III - reputação ilibada.

CAPÍTULO V
DA AVALIAÇÃO E ACOMPANHAMENTO DA GESTÃO DO ATENDIMENTO SOCIOEDUCATIVO

Art. 18. A União, em articulação com os Estados, o Distrito Federal e os Municípios, realizará
avaliações periódicas da implementação dos Planos de Atendimento Socioeducativo em intervalos não
superiores a 3 (três) anos.

Apostila gerada especialmente para: Betânia Mueller 023.712.580-32


. 146
§ 1º O objetivo da avaliação é verificar o cumprimento das metas estabelecidas e elaborar
recomendações aos gestores e operadores dos Sistemas.
§ 2º O processo de avaliação deverá contar com a participação de representantes do Poder Judiciário,
do Ministério Público, da Defensoria Pública e dos Conselhos Tutelares, na forma a ser definida em
regulamento.
§ 3º A primeira avaliação do Plano Nacional de Atendimento Socioeducativo realizar-se-á no terceiro
ano de vigência desta Lei, cabendo ao Poder Legislativo federal acompanhar o trabalho por meio de suas
comissões temáticas pertinentes.

Art. 19. É instituído o Sistema Nacional de Avaliação e Acompanhamento do Atendimento


Socioeducativo, com os seguintes objetivos:
I - contribuir para a organização da rede de atendimento socioeducativo;
II - assegurar conhecimento rigoroso sobre as ações do atendimento socioeducativo e seus resultados;
III - promover a melhora da qualidade da gestão e do atendimento socioeducativo; e
IV - disponibilizar informações sobre o atendimento socioeducativo.
§ 1º A avaliação abrangerá, no mínimo, a gestão, as entidades de atendimento, os programas e os
resultados da execução das medidas socioeducativas.
§ 2º Ao final da avaliação, será elaborado relatório contendo histórico e diagnóstico da situação, as
recomendações e os prazos para que essas sejam cumpridas, além de outros elementos a serem
definidos em regulamento.
§ 3º O relatório da avaliação deverá ser encaminhado aos respectivos Conselhos de Direitos,
Conselhos Tutelares e ao Ministério Público.
§ 4º Os gestores e entidades têm o dever de colaborar com o processo de avaliação, facilitando o
acesso às suas instalações, à documentação e a todos os elementos necessários ao seu efetivo
cumprimento.
§ 5º O acompanhamento tem por objetivo verificar o cumprimento das metas dos Planos de
Atendimento Socioeducativo.

Art. 20. O Sistema Nacional de Avaliação e Acompanhamento da Gestão do Atendimento


Socioeducativo assegurará, na metodologia a ser empregada:
I - a realização da auto avaliação dos gestores e das instituições de atendimento;
II - a avaliação institucional externa, contemplando a análise global e integrada das instalações físicas,
relações institucionais, compromisso social, atividades e finalidades das instituições de atendimento e
seus programas;
III - o respeito à identidade e à diversidade de entidades e programas;
IV - a participação do corpo de funcionários das entidades de atendimento e dos Conselhos Tutelares
da área de atuação da entidade avaliada; e
V - o caráter público de todos os procedimentos, dados e resultados dos processos avaliativos.

Art. 21. A avaliação será coordenada por uma comissão permanente e realizada por comissões
temporárias, essas compostas, no mínimo, por 3 (três) especialistas com reconhecida atuação na área
temática e definidas na forma do regulamento.
Parágrafo único. É vedado à comissão permanente designar avaliadores:
I - que sejam titulares ou servidores dos órgãos gestores avaliados ou funcionários das entidades
avaliadas;
II - que tenham relação de parentesco até o 3o grau com titulares ou servidores dos órgãos gestores
avaliados e/ou funcionários das entidades avaliadas; e
III - que estejam respondendo a processos criminais.

Art. 22. A avaliação da gestão terá por objetivo:


I - verificar se o planejamento orçamentário e sua execução se processam de forma compatível com
as necessidades do respectivo Sistema de Atendimento Socioeducativo;
II - verificar a manutenção do fluxo financeiro, considerando as necessidades operacionais do
atendimento socioeducativo, as normas de referência e as condições previstas nos instrumentos jurídicos
celebrados entre os órgãos gestores e as entidades de atendimento;
III - verificar a implementação de todos os demais compromissos assumidos por ocasião da celebração
dos instrumentos jurídicos relativos ao atendimento socioeducativo; e
IV - a articulação interinstitucional e intersetorial das políticas.

Apostila gerada especialmente para: Betânia Mueller 023.712.580-32


. 147
Art. 23. A avaliação das entidades terá por objetivo identificar o perfil e o impacto de sua atuação, por
meio de suas atividades, programas e projetos, considerando as diferentes dimensões institucionais e,
entre elas, obrigatoriamente, as seguintes:
I - o plano de desenvolvimento institucional;
II - a responsabilidade social, considerada especialmente sua contribuição para a inclusão social e o
desenvolvimento socioeconômico do adolescente e de sua família;
III - a comunicação e o intercâmbio com a sociedade;
IV - as políticas de pessoal quanto à qualificação, aperfeiçoamento, desenvolvimento profissional e
condições de trabalho;
V - a adequação da infraestrutura física às normas de referência;
VI - o planejamento e a autoavaliação quanto aos processos, resultados, eficiência e eficácia do projeto
pedagógico e da proposta socioeducativa;
VII - as políticas de atendimento para os adolescentes e suas famílias;
VIII - a atenção integral à saúde dos adolescentes em conformidade com as diretrizes do art. 60 desta
Lei; e
IX - a sustentabilidade financeira.

Art. 24. A avaliação dos programas terá por objetivo verificar, no mínimo, o atendimento ao que
determinam os arts. 94, 100, 117, 119, 120, 123 e 124 da Lei no 8.069, de 13 de julho de 1990 (Estatuto
da Criança e do Adolescente).

Art. 25. A avaliação dos resultados da execução de medida socioeducativa terá por objetivo, no
mínimo:
I - verificar a situação do adolescente após cumprimento da medida socioeducativa, tomando por base
suas perspectivas educacionais, sociais, profissionais e familiares; e
II - verificar reincidência de prática de ato infracional.

Art. 26. Os resultados da avaliação serão utilizados para:


I - planejamento de metas e eleição de prioridades do Sistema de Atendimento Socioeducativo e seu
financiamento;
II - reestruturação e/ou ampliação da rede de atendimento socioeducativo, de acordo com as
necessidades diagnosticadas;
III - adequação dos objetivos e da natureza do atendimento socioeducativo prestado pelas entidades
avaliadas;
IV - celebração de instrumentos de cooperação com vistas à correção de problemas diagnosticados
na avaliação;
V - reforço de financiamento para fortalecer a rede de atendimento socioeducativo;
VI - melhorar e ampliar a capacitação dos operadores do Sistema de Atendimento Socioeducativo; e
VII - os efeitos do art. 95 da Lei no 8.069, de 13 de julho de 1990 (Estatuto da Criança e do
Adolescente).
Parágrafo único. As recomendações originadas da avaliação deverão indicar prazo para seu
cumprimento por parte das entidades de atendimento e dos gestores avaliados, ao fim do qual estarão
sujeitos às medidas previstas no art. 28 desta Lei.

Art. 27. As informações produzidas a partir do Sistema Nacional de Informações sobre Atendimento
Socioeducativo serão utilizadas para subsidiar a avaliação, o acompanhamento, a gestão e o
financiamento dos Sistemas Nacional, Distrital, Estaduais e Municipais de Atendimento Socioeducativo.

CAPÍTULO VI
DA RESPONSABILIZAÇÃO DOS GESTORES, OPERADORES E ENTIDADES DE ATENDIMENTO

Art. 28. No caso do desrespeito, mesmo que parcial, ou do não cumprimento integral às diretrizes e
determinações desta Lei, em todas as esferas, são sujeitos:
I - gestores, operadores e seus prepostos e entidades governamentais às medidas previstas no inciso
I e no § 1o do art. 97 da Lei no 8.069, de 13 de julho de 1990 (Estatuto da Criança e do Adolescente); e
II - entidades não governamentais, seus gestores, operadores e prepostos às medidas previstas no
inciso II e no § 1o do art. 97 da Lei no 8.069, de 13 de julho de 1990 (Estatuto da Criança e do
Adolescente).

Apostila gerada especialmente para: Betânia Mueller 023.712.580-32


. 148
Parágrafo único. A aplicação das medidas previstas neste artigo dar-se-á a partir da análise de
relatório circunstanciado elaborado após as avaliações, sem prejuízo do que determinam os arts. 191 a
197, 225 a 227, 230 a236, 243 e 245 a 247 da Lei no 8.069, de 13 de julho de 1990 (Estatuto da Criança
e do Adolescente).

Art. 29. Àqueles que, mesmo não sendo agentes públicos, induzam ou concorram, sob qualquer forma,
direta ou indireta, para o não cumprimento desta Lei, aplicam-se, no que couber, as penalidades dispostas
na Lei no8.429, de 2 de junho de 1992, que dispõe sobre as sanções aplicáveis aos agentes públicos nos
casos de enriquecimento ilícito no exercício de mandato, cargo, emprego ou função na administração
pública direta, indireta ou fundacional e dá outras providências (Lei de Improbidade Administrativa).

CAPÍTULO VII
DO FINANCIAMENTO E DAS PRIORIDADES

Art. 30. O Sinase será cofinanciado com recursos dos orçamentos fiscal e da seguridade social, além
de outras fontes.
§ 1º (VETADO).
§ 2º Os entes federados que tenham instituído seus sistemas de atendimento socioeducativo terão
acesso aos recursos na forma de transferência adotada pelos órgãos integrantes do Sinase.
§ 3º Os entes federados beneficiados com recursos dos orçamentos dos órgãos responsáveis pelas
políticas integrantes do Sinase, ou de outras fontes, estão sujeitos às normas e procedimentos de
monitoramento estabelecidos pelas instâncias dos órgãos das políticas setoriais envolvidas, sem prejuízo
do disposto nos incisos IX e X do art. 4o, nos incisos V e VI do art. 5o e no art. 6o desta Lei.

Art. 31. Os Conselhos de Direitos, nas 3 (três) esferas de governo, definirão, anualmente, o percentual
de recursos dos Fundos dos Direitos da Criança e do Adolescente a serem aplicados no financiamento
das ações previstas nesta Lei, em especial para capacitação, sistemas de informação e de avaliação.
Parágrafo único. Os entes federados beneficiados com recursos do Fundo dos Direitos da Criança e
do Adolescente para ações de atendimento socioeducativo prestarão informações sobre o desempenho
dessas ações por meio do Sistema de Informações sobre Atendimento Socioeducativo.

Art. 32. A Lei no 7.560, de 19 de dezembro de 1986, passa a vigorar com as seguintes alterações:
“Art. 5º Os recursos do Funad serão destinados:
.............................................................................................
X - às entidades governamentais e não governamentais integrantes do Sistema Nacional de
Atendimento Socioeducativo (Sinase).
...................................................................................” (NR)
“Art. 5º-A. A Secretaria Nacional de Políticas sobre Drogas (Senad), órgão gestor do Fundo Nacional
Antidrogas (Funad), poderá financiar projetos das entidades do Sinase desde que:
I - o ente federado de vinculação da entidade que solicita o recurso possua o respectivo Plano de
Atendimento Socioeducativo aprovado;
II - as entidades governamentais e não governamentais integrantes do Sinase que solicitem recursos
tenham participado da avaliação nacional do atendimento socioeducativo;
III - o projeto apresentado esteja de acordo com os pressupostos da Política Nacional sobre Drogas e
legislação específica.”

Art. 33. A Lei no 7.998, de 11 de janeiro de 1990, passa a vigorar acrescida do seguinte art. 19-A:
“Art. 19-A. O Codefat poderá priorizar projetos das entidades integrantes do Sistema Nacional de
Atendimento Socioeducativo (Sinase) desde que:
I - o ente federado de vinculação da entidade que solicita o recurso possua o respectivo Plano de
Atendimento Socioeducativo aprovado;
II - as entidades governamentais e não governamentais integrantes do Sinase que solicitem recursos
tenham se submetido à avaliação nacional do atendimento socioeducativo.”

Art. 34. O art. 2o da Lei no 5.537, de 21 de novembro de 1968, passa a vigorar acrescido do seguinte
§ 3o:
“Art. 2º.......................................................................
.............................................................................................

Apostila gerada especialmente para: Betânia Mueller 023.712.580-32


. 149
§ 3º O fundo de que trata o art. 1o poderá financiar, na forma das resoluções de seu conselho
deliberativo, programas e projetos de educação básica relativos ao Sistema Nacional de Atendimento
Socioeducativo (Sinase) desde que:
I - o ente federado que solicitar o recurso possua o respectivo Plano de Atendimento Socioeducativo
aprovado;
II - as entidades de atendimento vinculadas ao ente federado que solicitar o recurso tenham se
submetido à avaliação nacional do atendimento socioeducativo; e
III - o ente federado tenha assinado o Plano de Metas Compromisso Todos pela Educação e elaborado
o respectivo Plano de Ações Articuladas (PAR).” (NR)

TÍTULO II
DA EXECUÇÃO DAS MEDIDAS SOCIOEDUCATIVAS
CAPÍTULO I
DISPOSIÇÕES GERAIS

Art. 35. A execução das medidas socioeducativas reger-se-á pelos seguintes princípios:
I - legalidade, não podendo o adolescente receber tratamento mais gravoso do que o conferido ao
adulto;
II - excepcionalidade da intervenção judicial e da imposição de medidas, favorecendo-se meios de auto
composição de conflitos;
III - prioridade a práticas ou medidas que sejam restaurativas e, sempre que possível, atendam às
necessidades das vítimas;
IV - proporcionalidade em relação à ofensa cometida;
V - brevidade da medida em resposta ao ato cometido, em especial o respeito ao que dispõe o art. 122
da Lei no 8.069, de 13 de julho de 1990 (Estatuto da Criança e do Adolescente);
VI - individualização, considerando-se a idade, capacidades e circunstâncias pessoais do adolescente;
VII - mínima intervenção, restrita ao necessário para a realização dos objetivos da medida;
VIII - não discriminação do adolescente, notadamente em razão de etnia, gênero, nacionalidade, classe
social, orientação religiosa, política ou sexual, ou associação ou pertencimento a qualquer minoria ou
status; e
IX - fortalecimento dos vínculos familiares e comunitários no processo socioeducativo.

CAPÍTULO II
DOS PROCEDIMENTOS

Art. 36. A competência para jurisdicionar a execução das medidas socioeducativas segue o
determinado pelo art. 146 da Lei no 8.069, de 13 de julho de 1990 (Estatuto da Criança e do Adolescente).

Art. 37. A defesa e o Ministério Público intervirão, sob pena de nulidade, no procedimento judicial de
execução de medida socioeducativa, asseguradas aos seus membros as prerrogativas previstas na Lei
no 8.069, de 13 de julho de 1990 (Estatuto da Criança e do Adolescente), podendo requerer as
providências necessárias para adequar a execução aos ditames legais e regulamentares.

Art. 38. As medidas de proteção, de advertência e de reparação do dano, quando aplicadas de forma
isolada, serão executadas nos próprios autos do processo de conhecimento, respeitado o disposto nos
arts. 143 e 144 da Lei no 8.069, de 13 de julho de 1990 (Estatuto da Criança e do Adolescente).

Art. 39. Para aplicação das medidas socioeducativas de prestação de serviços à comunidade,
liberdade assistida, semiliberdade ou internação, será constituído processo de execução para cada
adolescente, respeitado o disposto nos arts. 143 e 144 da Lei nº 8.069, de 13 de julho de 1990 (Estatuto
da Criança e do Adolescente), e com autuação das seguintes peças:
I - documentos de caráter pessoal do adolescente existentes no processo de conhecimento,
especialmente os que comprovem sua idade; e
II - as indicadas pela autoridade judiciária, sempre que houver necessidade e, obrigatoriamente:
a) cópia da representação;
b) cópia da certidão de antecedentes;
c) cópia da sentença ou acórdão; e
d) cópia de estudos técnicos realizados durante a fase de conhecimento.

Apostila gerada especialmente para: Betânia Mueller 023.712.580-32


. 150
Parágrafo único. Procedimento idêntico será observado na hipótese de medida aplicada em sede de
remissão, como forma de suspensão do processo.

Art. 40. Autuadas as peças, a autoridade judiciária encaminhará, imediatamente, cópia integral do
expediente ao órgão gestor do atendimento socioeducativo, solicitando designação do programa ou da
unidade de cumprimento da medida.

Art. 41. A autoridade judiciária dará vistas da proposta de plano individual de que trata o art. 53 desta
Lei ao defensor e ao Ministério Público pelo prazo sucessivo de 3 (três) dias, contados do recebimento
da proposta encaminhada pela direção do programa de atendimento.
§ 1º O defensor e o Ministério Público poderão requerer, e o Juiz da Execução poderá determinar, de
ofício, a realização de qualquer avaliação ou perícia que entenderem necessárias para complementação
do plano individual.
§ 2º A impugnação ou complementação do plano individual, requerida pelo defensor ou pelo Ministério
Público, deverá ser fundamentada, podendo a autoridade judiciária indeferi-la, se entender insuficiente a
motivação.
§ 3º Admitida a impugnação, ou se entender que o plano é inadequado, a autoridade judiciária
designará, se necessário, audiência da qual cientificará o defensor, o Ministério Público, a direção do
programa de atendimento, o adolescente e seus pais ou responsável.
§ 4º A impugnação não suspenderá a execução do plano individual, salvo determinação judicial em
contrário.
§ 5º Findo o prazo sem impugnação, considerar-se-á o plano individual homologado.

Art. 42. As medidas socioeducativas de liberdade assistida, de semiliberdade e de internação deverão


ser reavaliadas no máximo a cada 6 (seis) meses, podendo a autoridade judiciária, se necessário,
designar audiência, no prazo máximo de 10 (dez) dias, cientificando o defensor, o Ministério Público, a
direção do programa de atendimento, o adolescente e seus pais ou responsável.
§ 1º A audiência será instruída com o relatório da equipe técnica do programa de atendimento sobre a
evolução do plano de que trata o art. 52 desta Lei e com qualquer outro parecer técnico requerido pelas
partes e deferido pela autoridade judiciária.
§ 2º A gravidade do ato infracional, os antecedentes e o tempo de duração da medida não são fatores
que, por si, justifiquem a não substituição da medida por outra menos grave.
§ 3º Considera-se mais grave a internação, em relação a todas as demais medidas, e mais grave a
semiliberdade, em relação às medidas de meio aberto.

Art. 43. A reavaliação da manutenção, da substituição ou da suspensão das medidas de meio aberto
ou de privação da liberdade e do respectivo plano individual pode ser solicitada a qualquer tempo, a
pedido da direção do programa de atendimento, do defensor, do Ministério Público, do adolescente, de
seus pais ou responsável.
§ 1º Justifica o pedido de reavaliação, entre outros motivos:
I - o desempenho adequado do adolescente com base no seu plano de atendimento individual, antes
do prazo da reavaliação obrigatória;
II - a inadaptação do adolescente ao programa e o reiterado descumprimento das atividades do plano
individual; e
III - a necessidade de modificação das atividades do plano individual que importem em maior restrição
da liberdade do adolescente.
§ 2º A autoridade judiciária poderá indeferir o pedido, de pronto, se entender insuficiente a motivação.
§ 3º Admitido o processamento do pedido, a autoridade judiciária, se necessário, designará audiência,
observando o princípio do § 1o do art. 42 desta Lei.
§ 4º A substituição por medida mais gravosa somente ocorrerá em situações excepcionais, após o
devido processo legal, inclusive na hipótese do inciso III do art. 122 da Lei no 8.069, de 13 de julho de
1990 (Estatuto da Criança e do Adolescente), e deve ser:
I - fundamentada em parecer técnico;
II - precedida de prévia audiência, e nos termos do § 1o do art. 42 desta Lei.

Art. 44. Na hipótese de substituição da medida ou modificação das atividades do plano individual, a
autoridade judiciária remeterá o inteiro teor da decisão à direção do programa de atendimento, assim
como as peças que entender relevantes à nova situação jurídica do adolescente.

Apostila gerada especialmente para: Betânia Mueller 023.712.580-32


. 151
Parágrafo único. No caso de a substituição da medida importar em vinculação do adolescente a outro
programa de atendimento, o plano individual e o histórico do cumprimento da medida deverão
acompanhar a transferência.

Art. 45. Se, no transcurso da execução, sobrevier sentença de aplicação de nova medida, a autoridade
judiciária procederá à unificação, ouvidos, previamente, o Ministério Público e o defensor, no prazo de 3
(três) dias sucessivos, decidindo-se em igual prazo.
§ 1º É vedado à autoridade judiciária determinar reinício de cumprimento de medida socioeducativa,
ou deixar de considerar os prazos máximos, e de liberação compulsória previstos na Lei no 8.069, de 13
de julho de 1990 (Estatuto da Criança e do Adolescente), excetuada a hipótese de medida aplicada por
ato infracional praticado durante a execução.
§ 2º É vedado à autoridade judiciária aplicar nova medida de internação, por atos infracionais
praticados anteriormente, a adolescente que já tenha concluído cumprimento de medida socioeducativa
dessa natureza, ou que tenha sido transferido para cumprimento de medida menos rigorosa, sendo tais
atos absorvidos por aqueles aos quais se impôs a medida socioeducativa extrema.

Art. 46. A medida socioeducativa será declarada extinta:


I - pela morte do adolescente;
II - pela realização de sua finalidade;
III - pela aplicação de pena privativa de liberdade, a ser cumprida em regime fechado ou semiaberto,
em execução provisória ou definitiva;
IV - pela condição de doença grave, que torne o adolescente incapaz de submeter-se ao cumprimento
da medida; e
V - nas demais hipóteses previstas em lei.
§ 1º No caso de o maior de 18 (dezoito) anos, em cumprimento de medida socioeducativa, responder
a processo-crime, caberá à autoridade judiciária decidir sobre eventual extinção da execução,
cientificando da decisão o juízo criminal competente.
§ 2º Em qualquer caso, o tempo de prisão cautelar não convertida em pena privativa de liberdade deve
ser descontado do prazo de cumprimento da medida socioeducativa.

Art. 47. O mandado de busca e apreensão do adolescente terá vigência máxima de 6 (seis) meses, a
contar da data da expedição, podendo, se necessário, ser renovado, fundamentadamente.

Art. 48. O defensor, o Ministério Público, o adolescente e seus pais ou responsável poderão postular
revisão judicial de qualquer sanção disciplinar aplicada, podendo a autoridade judiciária suspender a
execução da sanção até decisão final do incidente.

§ 1º Postulada a revisão após ouvida a autoridade colegiada que aplicou a sanção e havendo provas
a produzir em audiência, procederá o magistrado na forma do § 1º do art. 42 desta Lei.
§ 2º É vedada a aplicação de sanção disciplinar de isolamento a adolescente interno, exceto seja essa
imprescindível para garantia da segurança de outros internos ou do próprio adolescente a quem seja
imposta a sanção, sendo necessária ainda comunicação ao defensor, ao Ministério Público e à autoridade
judiciária em até 24 (vinte e quatro) horas.

CAPÍTULO III
DOS DIREITOS INDIVIDUAIS

Art. 49. São direitos do adolescente submetido ao cumprimento de medida socioeducativa, sem
prejuízo de outros previstos em lei:
I - ser acompanhado por seus pais ou responsável e por seu defensor, em qualquer fase do
procedimento administrativo ou judicial;
II - ser incluído em programa de meio aberto quando inexistir vaga para o cumprimento de medida de
privação da liberdade, exceto nos casos de ato infracional cometido mediante grave ameaça ou violência
à pessoa, quando o adolescente deverá ser internado em Unidade mais próxima de seu local de
residência;
III - ser respeitado em sua personalidade, intimidade, liberdade de pensamento e religião e em todos
os direitos não expressamente limitados na sentença;
IV - peticionar, por escrito ou verbalmente, diretamente a qualquer autoridade ou órgão público,
devendo, obrigatoriamente, ser respondido em até 15 (quinze) dias;

Apostila gerada especialmente para: Betânia Mueller 023.712.580-32


. 152
V - ser informado, inclusive por escrito, das normas de organização e funcionamento do programa de
atendimento e também das previsões de natureza disciplinar;
VI - receber, sempre que solicitar, informações sobre a evolução de seu plano individual, participando,
obrigatoriamente, de sua elaboração e, se for o caso, reavaliação;
VII - receber assistência integral à sua saúde, conforme o disposto no art. 60 desta Lei; e
VIII - ter atendimento garantido em creche e pré-escola aos filhos de 0 (zero) a 5 (cinco) anos.
§ 1º As garantias processuais destinadas a adolescente autor de ato infracional previstas na Lei no
8.069, de 13 de julho de 1990 (Estatuto da Criança e do Adolescente), aplicam-se integralmente na
execução das medidas socioeducativas, inclusive no âmbito administrativo.
§ 2º A oferta irregular de programas de atendimento socioeducativo em meio aberto não poderá ser
invocada como motivo para aplicação ou manutenção de medida de privação da liberdade.

Art. 50. Sem prejuízo do disposto no § 1o do art. 121 da Lei no 8.069, de 13 de julho de 1990 (Estatuto
da Criança e do Adolescente), a direção do programa de execução de medida de privação da liberdade
poderá autorizar a saída, monitorada, do adolescente nos casos de tratamento médico, doença grave ou
falecimento, devidamente comprovados, de pai, mãe, filho, cônjuge, companheiro ou irmão, com imediata
comunicação ao juízo competente.

Art. 51. A decisão judicial relativa à execução de medida socioeducativa será proferida após
manifestação do defensor e do Ministério Público.

CAPÍTULO IV
DO PLANO INDIVIDUAL DE ATENDIMENTO (PIA)

Art. 52. O cumprimento das medidas socioeducativas, em regime de prestação de serviços à


comunidade, liberdade assistida, semiliberdade ou internação, dependerá de Plano Individual de
Atendimento (PIA), instrumento de previsão, registro e gestão das atividades a serem desenvolvidas com
o adolescente.
Parágrafo único. O PIA deverá contemplar a participação dos pais ou responsáveis, os quais têm o
dever de contribuir com o processo ressocializador do adolescente, sendo esses passíveis de
responsabilização administrativa, nos termos do art. 249 da Lei no 8.069, de 13 de julho de 1990 (Estatuto
da Criança e do Adolescente), civil e criminal.

Art. 53. O PIA será elaborado sob a responsabilidade da equipe técnica do respectivo programa de
atendimento, com a participação efetiva do adolescente e de sua família, representada por seus pais ou
responsável.

Art. 54. Constarão do plano individual, no mínimo:


I - os resultados da avaliação interdisciplinar;
II - os objetivos declarados pelo adolescente;
III - a previsão de suas atividades de integração social e/ou capacitação profissional;
IV - atividades de integração e apoio à família;
V - formas de participação da família para efetivo cumprimento do plano individual; e
VI - as medidas específicas de atenção à sua saúde.

Art. 55. Para o cumprimento das medidas de semiliberdade ou de internação, o plano individual
conterá, ainda:
I - a designação do programa de atendimento mais adequado para o cumprimento da medida;
II - a definição das atividades internas e externas, individuais ou coletivas, das quais o adolescente
poderá participar; e
III - a fixação das metas para o alcance de desenvolvimento de atividades externas.
Parágrafo único. O PIA será elaborado no prazo de até 45 (quarenta e cinco) dias da data do ingresso
do adolescente no programa de atendimento.

Art. 56. Para o cumprimento das medidas de prestação de serviços à comunidade e de liberdade
assistida, o PIA será elaborado no prazo de até 15 (quinze) dias do ingresso do adolescente no programa
de atendimento.

Apostila gerada especialmente para: Betânia Mueller 023.712.580-32


. 153
Art. 57. Para a elaboração do PIA, a direção do respectivo programa de atendimento, pessoalmente
ou por meio de membro da equipe técnica, terá acesso aos autos do procedimento de apuração do ato
infracional e aos dos procedimentos de apuração de outros atos infracionais atribuídos ao mesmo
adolescente.
§ 1º O acesso aos documentos de que trata o caput deverá ser realizado por funcionário da entidade
de atendimento, devidamente credenciado para tal atividade, ou por membro da direção, em
conformidade com as normas a serem definidas pelo Poder Judiciário, de forma a preservar o que
determinam os arts. 143 e 144 da Lei no 8.069, de 13 de julho de 1990 (Estatuto da Criança e do
Adolescente).
§ 2º A direção poderá requisitar, ainda:
I - ao estabelecimento de ensino, o histórico escolar do adolescente e as anotações sobre o seu
aproveitamento;
II - os dados sobre o resultado de medida anteriormente aplicada e cumprida em outro programa de
atendimento; e
III - os resultados de acompanhamento especializado anterior.

Art. 58. Por ocasião da reavaliação da medida, é obrigatória a apresentação pela direção do programa
de atendimento de relatório da equipe técnica sobre a evolução do adolescente no cumprimento do plano
individual.

Art. 59. O acesso ao plano individual será restrito aos servidores do respectivo programa de
atendimento, ao adolescente e a seus pais ou responsável, ao Ministério Público e ao defensor, exceto
expressa autorização judicial.

CAPÍTULO V
DA ATENÇÃO INTEGRAL À SAÚDE DE ADOLESCENTE EM CUMPRIMENTO DE MEDIDA
SOCIOEDUCATIVA
Seção I
Disposições Gerais

Art. 60. A atenção integral à saúde do adolescente no Sistema de Atendimento Socioeducativo seguirá
as seguintes diretrizes:
I - previsão, nos planos de atendimento socioeducativo, em todas as esferas, da implantação de ações
de promoção da saúde, com o objetivo de integrar as ações socioeducativas, estimulando a autonomia,
a melhoria das relações interpessoais e o fortalecimento de redes de apoio aos adolescentes e suas
famílias;
II - inclusão de ações e serviços para a promoção, proteção, prevenção de agravos e doenças e
recuperação da saúde;
III - cuidados especiais em saúde mental, incluindo os relacionados ao uso de álcool e outras
substâncias psicoativas, e atenção aos adolescentes com deficiências;
IV - disponibilização de ações de atenção à saúde sexual e reprodutiva e à prevenção de doenças
sexualmente transmissíveis;
V - garantia de acesso a todos os níveis de atenção à saúde, por meio de referência e
contrarreferência, de acordo com as normas do Sistema Único de Saúde (SUS);
VI - capacitação das equipes de saúde e dos profissionais das entidades de atendimento, bem como
daqueles que atuam nas unidades de saúde de referência voltadas às especificidades de saúde dessa
população e de suas famílias;
VII - inclusão, nos Sistemas de Informação de Saúde do SUS, bem como no Sistema de Informações
sobre Atendimento Socioeducativo, de dados e indicadores de saúde da população de adolescentes em
atendimento socioeducativo; e
VIII - estruturação das unidades de internação conforme as normas de referência do SUS e do Sinase,
visando ao atendimento das necessidades de Atenção Básica.

Art. 61. As entidades que ofereçam programas de atendimento socioeducativo em meio aberto e de
semiliberdade deverão prestar orientações aos socioeducandos sobre o acesso aos serviços e às
unidades do SUS.

Apostila gerada especialmente para: Betânia Mueller 023.712.580-32


. 154
Art. 62. As entidades que ofereçam programas de privação de liberdade deverão contar com uma
equipe mínima de profissionais de saúde cuja composição esteja em conformidade com as normas de
referência do SUS.

Art. 63. (VETADO).


§ 1º O filho de adolescente nascido nos estabelecimentos referidos no caput deste artigo não terá tal
informação lançada em seu registro de nascimento.
§ 2º Serão asseguradas as condições necessárias para que a adolescente submetida à execução de
medida socioeducativa de privação de liberdade permaneça com o seu filho durante o período de
amamentação.

Seção II
Do Atendimento a Adolescente com Transtorno Mental e com Dependência de Álcool e de
Substância Psicoativa

Art 64. O adolescente em cumprimento de medida socioeducativa que apresente indícios de transtorno
mental, de deficiência mental, ou associadas, deverá ser avaliado por equipe técnica multidisciplinar e
Multisetorial.
§ 1º As competências, a composição e a atuação da equipe técnica de que trata o caput deverão
seguir, conjuntamente, as normas de referência do SUS e do Sinase, na forma do regulamento.
§ 2º A avaliação de que trata o caput subsidiará a elaboração e execução da terapêutica a ser adotada,
a qual será incluída no PIA do adolescente, prevendo, se necessário, ações voltadas para a família.
§ 3º As informações produzidas na avaliação de que trata o caput são consideradas sigilosas.
§ 4º Excepcionalmente, o juiz poderá suspender a execução da medida socioeducativa, ouvidos o
defensor e o Ministério Público, com vistas a incluir o adolescente em programa de atenção integral à
saúde mental que melhor atenda aos objetivos terapêuticos estabelecidos para o seu caso específico.
§ 5º Suspensa a execução da medida socioeducativa, o juiz designará o responsável por acompanhar
e informar sobre a evolução do atendimento ao adolescente.
§ 6º A suspensão da execução da medida socioeducativa será avaliada, no mínimo, a cada 6 (seis)
meses.
§ 7º O tratamento a que se submeterá o adolescente deverá observar o previsto na Lei no 10.216, de
6 de abril de 2001, que dispõe sobre a proteção e os direitos das pessoas portadoras de transtornos
mentais e redireciona o modelo assistencial em saúde mental.
§ 8º (VETADO).

Art. 65. Enquanto não cessada a jurisdição da Infância e Juventude, a autoridade judiciária, nas
hipóteses tratadas no art. 64, poderá remeter cópia dos autos ao Ministério Público para eventual
propositura de interdição e outras providências pertinentes.

Art. 66. (VETADO).

CAPÍTULO VI
DAS VISITAS A ADOLESCENTE EM CUMPRIMENTO DE MEDIDA DE
INTERNAÇÃO

Art. 67. A visita do cônjuge, companheiro, pais ou responsáveis, parentes e amigos a adolescente a
quem foi aplicada medida socioeducativa de internação observará dias e horários próprios definidos pela
direção do programa de atendimento.

Art. 68. É assegurado ao adolescente casado ou que viva, comprovadamente, em união estável o
direito à visita íntima.
Parágrafo único. O visitante será identificado e registrado pela direção do programa de atendimento,
que emitirá documento de identificação, pessoal e intransferível, específico para a realização da visita
íntima.

Art. 69. É garantido aos adolescentes em cumprimento de medida socioeducativa de internação o


direito de receber visita dos filhos, independentemente da idade desses.

Apostila gerada especialmente para: Betânia Mueller 023.712.580-32


. 155
Art. 70. O regulamento interno estabelecerá as hipóteses de proibição da entrada de objetos na
unidade de internação, vedando o acesso aos seus portadores.

CAPÍTULO VII
DOS REGIMES DISCIPLINARES

Art. 71. Todas as entidades de atendimento socioeducativo deverão, em seus respectivos regimentos,
realizar a previsão de regime disciplinar que obedeça aos seguintes princípios:
I - tipificação explícita das infrações como leves, médias e graves e determinação das correspondentes
sanções;
II - exigência da instauração formal de processo disciplinar para a aplicação de qualquer sanção,
garantidos a ampla defesa e o contraditório;
III - obrigatoriedade de audiência do socioeducando nos casos em que seja necessária a instauração
de processo disciplinar;
IV - sanção de duração determinada;
V - enumeração das causas ou circunstâncias que eximam, atenuem ou agravem a sanção a ser
imposta ao socioeducando, bem como os requisitos para a extinção dessa;
VI - enumeração explícita das garantias de defesa;
VII - garantia de solicitação e rito de apreciação dos recursos cabíveis; e
VIII - apuração da falta disciplinar por comissão composta por, no mínimo, 3 (três) integrantes, sendo
1 (um), obrigatoriamente, oriundo da equipe técnica.

Art. 72. O regime disciplinar é independente da responsabilidade civil ou penal que advenha do ato
cometido.

Art. 73. Nenhum socioeducando poderá desempenhar função ou tarefa de apuração disciplinar ou
aplicação de sanção nas entidades de atendimento socioeducativo.

Art. 74. Não será aplicada sanção disciplinar sem expressa e anterior previsão legal ou regulamentar
e o devido processo administrativo.

Art. 75. Não será aplicada sanção disciplinar ao socioeducando que tenha praticado a falta:
I - por coação irresistível ou por motivo de força maior;
II - em legítima defesa, própria ou de outrem.

CAPÍTULO VIII
DA CAPACITAÇÃO PARA O TRABALHO

Art. 76. O art. 2o do Decreto-Lei no 4.048, de 22 de janeiro de 1942, passa a vigorar acrescido do
seguinte § 1º, renumerando-se o atual parágrafo único para § 2º:
“Art. 2º .........................................................................
§ 1º As escolas do Senai poderão ofertar vagas aos usuários do Sistema Nacional de Atendimento
Socioeducativo (Sinase) nas condições a serem dispostas em instrumentos de cooperação celebrados
entre os operadores do Senai e os gestores dos Sistemas de Atendimento Socioeducativo locais.
§ 2º ...................................................................... ” (NR)

Art. 77. O art. 3o do Decreto-Lei no 8.621, de 10 de janeiro de 1946, passa a vigorar acrescido do
seguinte § 1o, renumerando-se o atual parágrafo único para § 2º:
“Art. 3º .........................................................................
§ 1º As escolas do Senac poderão ofertar vagas aos usuários do Sistema Nacional de Atendimento
Socioeducativo (Sinase) nas condições a serem dispostas em instrumentos de cooperação celebrados
entre os operadores do Senac e os gestores dos Sistemas de Atendimento Socioeducativo locais.
§ 2º. ..................................................................... ” (NR)

Art. 78. O art. 1º da Lei no 8.315, de 23 de dezembro de 1991, passa a vigorar acrescido do seguinte
parágrafo único:
“Art. 1º .........................................................................
Parágrafo único. Os programas de formação profissional rural do Senar poderão ofertar vagas aos
usuários do Sistema Nacional de Atendimento Socioeducativo (Sinase) nas condições a serem dispostas

Apostila gerada especialmente para: Betânia Mueller 023.712.580-32


. 156
em instrumentos de cooperação celebrados entre os operadores do Senar e os gestores dos Sistemas
de Atendimento Socioeducativo locais.” (NR)

Art. 79. O art. 3º da Lei no 8.706, de 14 de setembro de 1993, passa a vigorar acrescido do seguinte
parágrafo único:
“Art. 3º .........................................................................
Parágrafo único. Os programas de formação profissional do Senat poderão ofertar vagas aos usuários
do Sistema Nacional de Atendimento Socioeducativo (Sinase) nas condições a serem dispostas em
instrumentos de cooperação celebrados entre os operadores do Senat e os gestores dos Sistemas de
Atendimento Socioeducativo locais.” (NR)

Art. 80. O art. 429 do Decreto-Lei no 5.452, de 1o de maio de 1943, passa a vigorar acrescido do
seguinte § 2o:
“Art. 429. .....................................................................
.............................................................................................
§ 2º estabelecimentos de que trata o caput ofertarão vagas de aprendizes a adolescentes usuários do
Sistema Nacional de Atendimento Socioeducativo (Sinase) nas condições a serem dispostas em
instrumentos de cooperação celebrados entre os estabelecimentos e os gestores dos Sistemas de
Atendimento Socioeducativo locais.” (NR)

TÍTULO III
DISPOSIÇÕES FINAIS E TRANSITÓRIAS

Art. 81. As entidades que mantenham programas de atendimento têm o prazo de até 6 (seis) meses
após a publicação desta Lei para encaminhar ao respectivo Conselho Estadual ou Municipal dos Direitos
da Criança e do Adolescente proposta de adequação da sua inscrição, sob pena de interdição.

Art. 82. Os Conselhos dos Direitos da Criança e do Adolescente, em todos os níveis federados, com
os órgãos responsáveis pelo sistema de educação pública e as entidades de atendimento, deverão, no
prazo de 1 (um) ano a partir da publicação desta Lei, garantir a inserção de adolescentes em cumprimento
de medida socioeducativa na rede pública de educação, em qualquer fase do período letivo,
contemplando as diversas faixas etárias e níveis de instrução.

Art. 83. Os programas de atendimento socioeducativo sob a responsabilidade do Poder Judiciário


serão, obrigatoriamente, transferidos ao Poder Executivo no prazo máximo de 1 (um) ano a partir da
publicação desta Lei e de acordo com a política de oferta dos programas aqui definidos.

Art. 84. Os programas de internação e semiliberdade sob a responsabilidade dos Municípios serão,
obrigatoriamente, transferidos para o Poder Executivo do respectivo Estado no prazo máximo de 1 (um)
ano a partir da publicação desta Lei e de acordo com a política de oferta dos programas aqui definidos.

Art. 85. A não transferência de programas de atendimento para os devidos entes responsáveis, no
prazo determinado nesta Lei, importará na interdição do programa e caracterizará ato de improbidade
administrativa do agente responsável, vedada, ademais, ao Poder Judiciário e ao Poder Executivo
municipal, ao final do referido prazo, a realização de despesas para a sua manutenção.

Art. 86. Os arts. 90, 97, 121, 122, 198 e 208 da Lei no 8.069, de 13 de julho de 1990 (Estatuto da
Criança e do Adolescente), passam a vigorar com a seguinte redação:
“Art. 90. ......................................................................
.............................................................................................
V - prestação de serviços à comunidade;
VI - liberdade assistida;
VII - semiliberdade; e
VIII - internação.
....................................................................................” (NR)
“Art. 97. (VETADO)”
“Art. 121. .................................…………………............
.............................................................................................

Apostila gerada especialmente para: Betânia Mueller 023.712.580-32


. 157
§ 7º A determinação judicial mencionada no § 1º poderá ser revista a qualquer tempo pela autoridade
judiciária.” (NR)
“Art. 122. .....................................................................
.............................................................................................
§ 1º O prazo de internação na hipótese do inciso III deste artigo não poderá ser superior a 3 (três)
meses, devendo ser decretada judicialmente após o devido processo legal.
...................................................................................” (NR)
“Art. 198. Nos procedimentos afetos à Justiça da Infância e da Juventude, inclusive os relativos à
execução das medidas socioeducativas, adotar-se-á o sistema recursal da Lei no 5.869, de 11 de janeiro
de 1973 (Código de Processo Civil), com as seguintes adaptações:
.............................................................................................
II - em todos os recursos, salvo nos embargos de declaração, o prazo para o Ministério Público e para
a defesa será sempre de 10 (dez) dias;
...................................................................................” (NR)
“Art. 208. .....................................................................
.............................................................................................
X - de programas de atendimento para a execução das medidas socioeducativas e aplicação de
medidas de proteção.
...................................................................................” (NR)

Art. 87. A Lei nº 8.069, de 13 de julho de 1990 (Estatuto da Criança e do Adolescente), passa a vigorar
com as seguintes alterações:
“Art. 260. Os contribuintes poderão efetuar doações aos Fundos dos Direitos da Criança e do
Adolescente nacional, distrital, estaduais ou municipais, devidamente comprovadas, sendo essas
integralmente deduzidas do imposto de renda, obedecidos os seguintes limites:
I - 1% (um por cento) do imposto sobre a renda devido apurado pelas pessoas jurídicas tributadas com
base no lucro real; e
II - 6% (seis por cento) do imposto sobre a renda apurado pelas pessoas físicas na Declaração de
Ajuste Anual, observado o disposto no art. 22 da Lei no 9.532, de 10 de dezembro de 1997.
.............................................................................................
§ 5o Observado o disposto no § 4o do art. 3o da Lei no 9.249, de 26 de dezembro de 1995, a dedução
de que trata o inciso I do caput:
I - será considerada isoladamente, não se submetendo a limite em conjunto com outras deduções do
imposto; e
II - não poderá ser computada como despesa operacional na apuração do lucro real.” (NR)
“Art. 260-A. A partir do exercício de 2010, ano-calendário de 2009, a pessoa física poderá optar pela
doação de que trata o inciso II do caput do art. 260 diretamente em sua Declaração de Ajuste Anual.
§ 1º A doação de que trata o caput poderá ser deduzida até os seguintes percentuais aplicados sobre
o imposto apurado na declaração:
I - (VETADO);
II - (VETADO);
III - 3% (três por cento) a partir do exercício de 2012.
§ 2º A dedução de que trata o caput:
I - está sujeita ao limite de 6% (seis por cento) do imposto sobre a renda apurado na declaração de
que trata o inciso II do caput do art. 260;
II - não se aplica à pessoa física que:
a) utilizar o desconto simplificado;
b) apresentar declaração em formulário; ou
c) entregar a declaração fora do prazo;
III - só se aplica às doações em espécie; e
IV - não exclui ou reduz outros benefícios ou deduções em vigor.
§ 3º O pagamento da doação deve ser efetuado até a data de vencimento da primeira quota ou quota
única do imposto, observadas instruções específicas da Secretaria da Receita Federal do Brasil.
§ 4º O não pagamento da doação no prazo estabelecido no § 3o implica a glosa definitiva desta parcela
de dedução, ficando a pessoa física obrigada ao recolhimento da diferença de imposto devido apurado
na Declaração de Ajuste Anual com os acréscimos legais previstos na legislação.
§ 5º A pessoa física poderá deduzir do imposto apurado na Declaração de Ajuste Anual as doações
feitas, no respectivo ano-calendário, aos fundos controlados pelos Conselhos dos Direitos da Criança e

Apostila gerada especialmente para: Betânia Mueller 023.712.580-32


. 158
do Adolescente municipais, distrital, estaduais e nacional concomitantemente com a opção de que trata
o caput, respeitado o limite previsto no inciso II do art. 260.”
“Art. 260-B. A doação de que trata o inciso I do art. 260 poderá ser deduzida:
I - do imposto devido no trimestre, para as pessoas jurídicas que apuram o imposto trimestralmente; e
II - do imposto devido mensalmente e no ajuste anual, para as pessoas jurídicas que apuram o imposto
anualmente.
Parágrafo único. A doação deverá ser efetuada dentro do período a que se refere a apuração do
imposto.”
“Art. 260-C. As doações de que trata o art. 260 desta Lei podem ser efetuadas em espécie ou em
bens.
Parágrafo único. As doações efetuadas em espécie devem ser depositadas em conta específica, em
instituição financeira pública, vinculadas aos respectivos fundos de que trata o art. 260.”
“Art. 260-D. Os órgãos responsáveis pela administração das contas dos Fundos dos Direitos da
Criança e do Adolescente nacional, estaduais, distrital e municipais devem emitir recibo em favor do
doador, assinado por pessoa competente e pelo presidente do Conselho correspondente, especificando:
I - número de ordem;
II - nome, Cadastro Nacional da Pessoa Jurídica (CNPJ) e endereço do emitente;
III - nome, CNPJ ou Cadastro de Pessoas Físicas (CPF) do doador;
IV - data da doação e valor efetivamente recebido; e
V - ano-calendário a que se refere a doação.
§ 1º O comprovante de que trata o caput deste artigo pode ser emitido anualmente, desde que
discrimine os valores doados mês a mês.
§ 2º No caso de doação em bens, o comprovante deve conter a identificação dos bens, mediante
descrição em campo próprio ou em relação anexa ao comprovante, informando também se houve
avaliação, o nome, CPF ou CNPJ e endereço dos avaliadores.”
“Art. 260-E. Na hipótese da doação em bens, o doador deverá:
I - comprovar a propriedade dos bens, mediante documentação hábil;
II - baixar os bens doados na declaração de bens e direitos, quando se tratar de pessoa física, e na
escrituração, no caso de pessoa jurídica; e
III - considerar como valor dos bens doados:
a) para as pessoas físicas, o valor constante da última declaração do imposto de renda, desde que
não exceda o valor de mercado;
b) para as pessoas jurídicas, o valor contábil dos bens.
Parágrafo único. O preço obtido em caso de leilão não será considerado na determinação do valor
dos bens doados, exceto se o leilão for determinado por autoridade judiciária.”
“Art. 260-F. Os documentos a que se referem os arts. 260-D e 260-E devem ser mantidos pelo
contribuinte por um prazo de 5 (cinco) anos para fins de comprovação da dedução perante a Receita
Federal do Brasil.”
“Art. 260-G. Os órgãos responsáveis pela administração das contas dos Fundos dos Direitos da
Criança e do Adolescente nacional, estaduais, distrital e municipais devem:
I - manter conta bancária específica destinada exclusivamente a gerir os recursos do Fundo;
II - manter controle das doações recebidas; e
III - informar anualmente à Secretaria da Receita Federal do Brasil as doações recebidas mês a mês,
identificando os seguintes dados por doador:
a) nome, CNPJ ou CPF;
b) valor doado, especificando se a doação foi em espécie ou em bens.”
“Art. 260-H. Em caso de descumprimento das obrigações previstas no art. 260-G, a Secretaria da
Receita Federal do Brasil dará conhecimento do fato ao Ministério Público.”
“Art. 260-I. Os Conselhos dos Direitos da Criança e do Adolescente nacional, estaduais, distrital e
municipais divulgarão amplamente à comunidade:
I - o calendário de suas reuniões;
II - as ações prioritárias para aplicação das políticas de atendimento à criança e ao adolescente;
III - os requisitos para a apresentação de projetos a serem beneficiados com recursos dos Fundos dos
Direitos da Criança e do Adolescente nacional, estaduais, distrital ou municipais;
IV - a relação dos projetos aprovados em cada ano-calendário e o valor dos recursos previstos para
implementação das ações, por projeto;
V - o total dos recursos recebidos e a respectiva destinação, por projeto atendido, inclusive com
cadastramento na base de dados do Sistema de Informações sobre a Infância e a Adolescência; e

Apostila gerada especialmente para: Betânia Mueller 023.712.580-32


. 159
VI - a avaliação dos resultados dos projetos beneficiados com recursos dos Fundos dos Direitos da
Criança e do Adolescente nacional, estaduais, distrital e municipais.”
“Art. 260-J. O Ministério Público determinará, em cada Comarca, a forma de fiscalização da aplicação
dos incentivos fiscais referidos no art. 260 desta Lei.
Parágrafo único. O descumprimento do disposto nos arts. 260-G e 260-I sujeitará os infratores a
responder por ação judicial proposta pelo Ministério Público, que poderá atuar de ofício, a requerimento
ou representação de qualquer cidadão.”
“Art. 260-K. A Secretaria de Direitos Humanos da Presidência da República (SDH/PR) encaminhará
à Secretaria da Receita Federal do Brasil, até 31 de outubro de cada ano, arquivo eletrônico contendo a
relação atualizada dos Fundos dos Direitos da Criança e do Adolescente nacional, distrital, estaduais e
municipais, com a indicação dos respectivos números de inscrição no CNPJ e das contas bancárias
específicas mantidas em instituições financeiras públicas, destinadas exclusivamente a gerir os recursos
dos Fundos.”
“Art. 260-L. A Secretaria da Receita Federal do Brasil expedirá as instruções necessárias à aplicação
do disposto nos arts. 260 a 260-K.”

Art. 88. O parágrafo único do art. 3o da Lei no 12.213, de 20 de janeiro de 2010, passa a vigorar com
a seguinte redação:
“Art. 3º ..........................................................................
Parágrafo único. A dedução a que se refere o caput deste artigo não poderá ultrapassar 1% (um por
cento) do imposto devido.” (NR)

Art. 89. (VETADO).

Art. 90. Esta Lei entra em vigor após decorridos 90 (noventa) dias de sua publicação oficial.

Brasília, 18 de janeiro de 2012; 191º da Independência e 124º da República.

DILMA ROUSSEFF

Questões

01. (DPE/AM - Defensor Público - FCC/2018). Dentre aqueles previstos na Lei n° 12.594/2012 (Lei
do Sinase), é princípio que rege a execução das medidas socioeducativas
(A) a prioridade a práticas ou medidas que sejam ressocializadoras em detrimento daquelas que
atendam às necessidades das vítimas.
(B) proporcionalidade em relação à ofensa cometida.
(C) legalidade e especialidade, vedada a aplicação, aos adolescentes, de quaisquer dispositivos da
legislação penal ou processual dos adultos.
(D) definição do grau de controle e vigilância conforme avaliação de periculosidade do adolescente.
(E) atendimento das necessidades socioassistenciais do adolescente e sua família conforme
parâmetros fixados na sentença.

02. (DPE/RS - Defensor Público - FCC/2018). Com relação aos princípios que regem a execução das
medidas socioeducativas previstos na Lei n.º 12.594/12, que instituiu o Sistema Nacional de Atendimento
Socioeducativo (Sinase), pelo princípio da
(A) não discriminação, não pode o adolescente receber tratamento mais gravoso do que o conferido
ao adulto.
(B) brevidade na imposição de medidas, devem-se favorecer os meios de autocomposição de conflitos.
(C) legalidade, não pode o adolescente receber tratamento mais gravoso do que o conferido ao adulto.
(D) individualização, a medida deve ser sempre proporcional à ofensa cometida.
(E) legalidade, não há ato infracional que justifique a imposição de medida socioeducativa sem prévia
lei que o defina.

03. (Pref. de Andradina/SP - Assistente Jurídico e Procurador Jurídico - VUNESP/2017). Nos


termos do que estabelece a Lei nº 12.594/2012 – SINASE, é correto afirmar que
(A) se entende por programa de atendimento a base física necessária para a organização e o
funcionamento de programa de atendimento ao adolescente.

Apostila gerada especialmente para: Betânia Mueller 023.712.580-32


. 160
(B) o SINASE será coordenado pelos Municípios e integrado pelos sistemas estaduais por meio dos
programas de atendimento ao adolescente infrator.
(C) compete privativamente à União elaborar o Plano Nacional de Atendimento Socioeducativo.
(D) compete aos Municípios criar e manter programas de atendimento para a execução das medidas
socioeducativas em meio aberto.
(E) ao Município e ao Estado é facultativa a edificação de unidades socioeducacionais em espaços
contíguos ou anexos a estabelecimentos penais.

04. (TJ/SC - Juiz Substituto - FCC/2017). A Lei Federal n° 12.594/12, que instituiu o SINASE −
Sistema Nacional Socioeducativo, previu como direitos dos adolescentes em cumprimento de medida
socioeducativa, expressamente,
(A) direito a creche e pré-escola de filhos de zero a cinco anos de idade e o direito de ser inserido em
medida em meio aberto quando o ato infracional praticado não estiver carregado de violência ou grave
ameaça e não houver vaga para internação no local de sua residência.
(B) possibilidade de saída monitorada sem prévia autorização judicial nos casos de falecimento de
irmão e de peticionar por escrito a qualquer autoridade ou órgão público, devendo ser respondido em até
10 (dez) dias.
(C) direito de receber visita, mesmo que de egresso do sistema socioeducativo e de ter acesso à leitura
em seu alojamento, mesmo que em quarto coletivo.
(D direito a creche e pré-escola de filhos de zero a cinco anos de idade e de ter acesso à leitura em
seu alojamento, mesmo que em quarto coletivo.
(E) direito de peticionar por escrito a qualquer autoridade ou órgão público, devendo ser respondido
em até 10 (dez) dias e de receber visita, mesmo que de egresso do sistema socioeducativo.

05. (MPE/PR - Promotor Substituto - 2017). Nos termos da Lei do Sistema Nacional de Atendimento
Socioeducativo (Lei nº 12.594/12), assinale a alternativa incorreta:
(A) A execução das medidas socioeducativas reger-se-á, entre outros, pelo princípio da prioridade a
práticas ou medidas que sejam restaurativas e, sempre que possível, atendam às necessidades das
vítimas.
(B) A visita do cônjuge, companheiro, pais ou responsáveis, parentes e amigos a adolescente a quem
foi aplicada medida socioeducativa de internação observará dias e horários próprios definidos pela
direção do programa de atendimento.
(C) É vedado à autoridade judiciária aplicar nova medida de internação, por atos infracionais praticados
anteriormente, a adolescente que já tenha concluído cumprimento de medida socioeducativa dessa
natureza, ou que tenha sido transferido para cumprimento de medida menos rigorosa, sendo tais atos
absorvidos por aqueles aos quais se impôs a medida socioeducativa extrema.
(D) A medida socioeducativa será declarada extinta, entre outras hipóteses, pela condição de doença
grave, que torne o adolescente incapaz de submeter-se ao cumprimento da medida.
(E) É direito do adolescente submetido ao cumprimento de medida socioeducativa,
independentemente da gravidade do ato infracional praticado, ser incluído em programa de meio aberto
quando inexistir vaga para o cumprimento de medida de privação da liberdade.

06. (TJ/SP - Assistente Social Judiciário - VUNESP/2017). De acordo com os procedimentos


estabelecidos pelo Sistema Nacional de Atendimento Socioeducativo – Lei n° 12.594/2012, as medidas
socioeducativas de liberdade assistida, de semiliberdade e de internação deverão ser reavaliadas no
máximo a cada 6 meses. O artigo 43 da referida lei define que, a pedido da direção do programa de
atendimento, do defensor, do Ministério Público, do adolescente, de seus pais ou responsável, a
reavaliação da manutenção da substituição ou da suspensão das medidas acima referidas e do respectivo
plano individual pode ser solicitada
(A) três meses após seu início.
(B) em prazo concomitante à revisão semestral.
(C) em intervalos de quatro meses.
(D) a qualquer tempo.
(E) após homologação do plano individual.

07. (DPE/BA - Defensor Público - FCC/2016). Sobre o SINASE – Sistema Nacional de Atendimento
Socioeducativo − é correto afirmar que

Apostila gerada especialmente para: Betânia Mueller 023.712.580-32


. 161
(A) corresponde ao conjunto ordenado de princípios, regras e critérios que envolvem a aplicação e
execução de medidas socioeducativas, incluindo-se, nele, todos os planos, políticas e programas, gerais
e específicos, de atendimento ao adolescente em conflito com a lei e a seus familiares.
(B) se trata de um subsistema do Sistema Único de Assistência Social − SUAS, por meio do qual são
regulamentados e geridos os programas socioassistenciais, socioeducativos e socioprotetivos destinados
aos adolescentes autores de atos infracionais.
(C) mesmo previsto na Constituição Federal desde 1988, foi efetivamente implantado no país somente
a partir de 2010, quando, por força de lei federal, a adesão a esse Sistema tornou-se obrigatória pelos
estados, municípios e Distrito Federal.
(D) é coordenado por uma comissão triparte de gestores representantes dos sistemas estaduais,
distrital e municipais responsáveis pela implementação dos seus respectivos programas de atendimento
ao adolescente ao qual seja aplicada medida socioeducativa.
(E) ao Conselho Nacional dos Direitos da Criança e do Adolescente (Conanda) competem as funções
normativa, deliberativa, de avaliação e de fiscalização do Sinase.

Gabarito

01.B / 02.C / 03.D / 04.A / 05.D / 06.D / 07.E

Comentários

01. Resposta: B
Lei 12.594/2012
Art. 35. A execução das medidas socioeducativas reger-se-á pelos seguintes princípios:
[ ]
IV - proporcionalidade em relação à ofensa cometida.

02. Resposta: C
Lei 12.594/2012
Art. 35. A execução das medidas socioeducativas reger-se-á pelos seguintes princípios:
I - legalidade, não podendo o adolescente receber tratamento mais gravoso do que o conferido ao
adulto;

03. Resposta: D
Lei 12.594/2012
Art. 5º Compete aos Municípios:
I - formular, instituir, coordenar e manter o Sistema Municipal de Atendimento Socioeducativo,
respeitadas as diretrizes fixadas pela União e pelo respectivo Estado;
II - elaborar o Plano Municipal de Atendimento Socioeducativo, em conformidade com o Plano Nacional
e o respectivo Plano Estadual;
III - criar e manter programas de atendimento para a execução das medidas socioeducativas em meio
aberto.

04. Resposta: A
Lei 12.594/2012
Art. 49. São direitos do adolescente submetido ao cumprimento de medida socioeducativa, sem
prejuízo de outros previstos em lei:
( )
II - ser incluído em programa de meio aberto quando inexistir vaga para o cumprimento de medida de
privação da liberdade, exceto nos casos de ato infracional cometido mediante grave ameaça ou violência
à pessoa, quando o adolescente deverá ser internado em Unidade mais próxima de seu local de
residência;
( )
VIII - ter atendimento garantido em creche e pré-escola aos filhos de 0 (zero) a 5 (cinco) anos.

05. Resposta: D
Lei 12.594/2012
Art. 49. São direitos do adolescente submetido ao cumprimento de medida socioeducativa, sem
prejuízo de outros previstos em lei:

Apostila gerada especialmente para: Betânia Mueller 023.712.580-32


. 162
( )
II - ser incluído em programa de meio aberto quando inexistir vaga para o cumprimento de medida de
privação da liberdade, exceto nos casos de ato infracional cometido mediante grave ameaça ou violência
à pessoa, quando o adolescente deverá ser internado em Unidade mais próxima de seu local de
residência;

06. Resposta: D
Lei 12.594/2012
Art. 43. A reavaliação da manutenção, da substituição ou da suspensão das medidas de meio aberto
ou de privação da liberdade e do respectivo plano individual pode ser solicitada a qualquer tempo, a
pedido da direção do programa de atendimento, do defensor, do Ministério Público, do adolescente, de
seus pais ou responsável.

07. Resposta: E
Lei 12.594/2012
Art. 3º Compete à União:
( )
§ 2º Ao Conselho Nacional dos Direitos da Criança e do Adolescente (Conanda) competem as funções
normativa, deliberativa, de avaliação e de fiscalização do Sinase, nos termos previstos na Lei no 8.242,
de 12 de outubro de 1991, que cria o referido Conselho.

O Estatuto do Idoso (Lei nº 10.741, de 1º outubro de 2003) e Política Nacional do


Idoso (Lei nº 8.842, de 4 de janeiro de 1994).

LEI Nº 10.741, DE 1º DE OUTUBRO DE 200311

Dispõe sobre o Estatuto do Idoso e dá outras providências.

O PRESIDENTE DA REPÚBLICA Faço saber que o Congresso Nacional decreta e eu sanciono a


seguinte Lei:

TÍTULO I
DISPOSIÇÕES PRELIMINARES

Art. 1º É instituído o Estatuto do Idoso, destinado a regular os direitos assegurados às pessoas com
idade igual ou superior a 60 (sessenta) anos.

Art. 2º O idoso goza de todos os direitos fundamentais inerentes à pessoa humana, sem prejuízo da
proteção integral de que trata esta Lei, assegurando-se lhe, por lei ou por outros meios, todas as
oportunidades e facilidades, para preservação de sua saúde física e mental e seu aperfeiçoamento moral,
intelectual, espiritual e social, em condições de liberdade e dignidade.

Art. 3º É obrigação da família, da comunidade, da sociedade e do Poder Público assegurar ao idoso,


com absoluta prioridade, a efetivação do direito à vida, à saúde, à alimentação, à educação, à cultura, ao
esporte, ao lazer, ao trabalho, à cidadania, à liberdade, à dignidade, ao respeito e à convivência familiar
e comunitária.
§ 1º A garantia de prioridade compreende: (Redação dada pela Lei nº 13.466, de 2017)
I – atendimento preferencial imediato e individualizado junto aos órgãos públicos e privados
prestadores de serviços à população;
II – preferência na formulação e na execução de políticas sociais públicas específicas;
III – destinação privilegiada de recursos públicos nas áreas relacionadas com a proteção ao idoso;
IV – viabilização de formas alternativas de participação, ocupação e convívio do idoso com as demais
gerações;
V – priorização do atendimento do idoso por sua própria família, em detrimento do atendimento asilar,
exceto dos que não a possuam ou careçam de condições de manutenção da própria sobrevivência;

11
Disponível em: http://www.planalto.gov.br/ccivil_03/LEIS/2003/L10.741compilado.htm - acesso em 15/10/2018 às 11h

Apostila gerada especialmente para: Betânia Mueller 023.712.580-32


. 163
VI – capacitação e reciclagem dos recursos humanos nas áreas de geriatria e gerontologia e na
prestação de serviços aos idosos;
VII – estabelecimento de mecanismos que favoreçam a divulgação de informações de caráter
educativo sobre os aspectos biopsicossociais de envelhecimento;
VIII – garantia de acesso à rede de serviços de saúde e de assistência social locais.
IX – prioridade no recebimento da restituição do Imposto de Renda.
§ 2º Dentre os idosos, é assegurada prioridade especial aos maiores de oitenta anos, atendendo-se
suas necessidades sempre preferencialmente em relação aos demais idosos. (Incluído pela Lei nº 13.466,
de 2017)

Art. 4º Nenhum idoso será objeto de qualquer tipo de negligência, discriminação, violência, crueldade
ou opressão, e todo atentado aos seus direitos, por ação ou omissão, será punido na forma da lei.
§ 1º É dever de todos prevenir a ameaça ou violação aos direitos do idoso.
§ 2º As obrigações previstas nesta Lei não excluem da prevenção outras decorrentes dos princípios
por ela adotados.

Art. 5º A inobservância das normas de prevenção importará em responsabilidade à pessoa física ou


jurídica nos termos da lei.

Art. 6º Todo cidadão tem o dever de comunicar à autoridade competente qualquer forma de violação
a esta Lei que tenha testemunhado ou de que tenha conhecimento.

Art. 7º Os Conselhos Nacional, Estaduais, do Distrito Federal e Municipais do Idoso, previstos na Lei
no 8.842, de 4 de janeiro de 1994, zelarão pelo cumprimento dos direitos do idoso, definidos nesta Lei.

TÍTULO II
DOS DIREITOS FUNDAMENTAIS
CAPÍTULO I
DO DIREITO À VIDA

Art. 8º O envelhecimento é um direito personalíssimo e a sua proteção um direito social, nos termos
desta Lei e da legislação vigente.

Art. 9º É obrigação do Estado, garantir à pessoa idosa a proteção à vida e à saúde, mediante efetivação
de políticas sociais públicas que permitam um envelhecimento saudável e em condições de dignidade.

CAPÍTULO II
DO DIREITO À LIBERDADE, AO RESPEITO E À DIGNIDADE

Art. 10. É obrigação do Estado e da sociedade, assegurar à pessoa idosa a liberdade, o respeito e a
dignidade, como pessoa humana e sujeito de direitos civis, políticos, individuais e sociais, garantidos na
Constituição e nas leis.
§ 1º O direito à liberdade compreende, entre outros, os seguintes aspectos:
I – faculdade de ir, vir e estar nos logradouros públicos e espaços comunitários, ressalvadas as
restrições legais;
II – opinião e expressão;
III – crença e culto religioso;
IV – prática de esportes e de diversões;
V – participação na vida familiar e comunitária;
VI – participação na vida política, na forma da lei;
VII – faculdade de buscar refúgio, auxílio e orientação.
§ 2º O direito ao respeito consiste na inviolabilidade da integridade física, psíquica e moral, abrangendo
a preservação da imagem, da identidade, da autonomia, de valores, ideias e crenças, dos espaços e dos
objetos pessoais.
§ 3º É dever de todos zelar pela dignidade do idoso, colocando-o a salvo de qualquer tratamento
desumano, violento, aterrorizante, vexatório ou constrangedor.

Apostila gerada especialmente para: Betânia Mueller 023.712.580-32


. 164
CAPÍTULO III
DOS ALIMENTOS

Art. 11. Os alimentos serão prestados ao idoso na forma da lei civil.

Art. 12. A obrigação alimentar é solidária, podendo o idoso optar entre os prestadores.

Art. 13. As transações relativas a alimentos poderão ser celebradas perante o Promotor de Justiça ou
Defensor Público, que as referendará, e passarão a ter efeito de título executivo extrajudicial nos termos
da lei processual civil.

Art. 14. Se o idoso ou seus familiares não possuírem condições econômicas de prover o seu sustento,
impõe-se ao Poder Público esse provimento, no âmbito da assistência social.

CAPÍTULO IV
DO DIREITO À SAÚDE

Art. 15. É assegurada a atenção integral à saúde do idoso, por intermédio do Sistema Único de Saúde
– SUS, garantindo-lhe o acesso universal e igualitário, em conjunto articulado e contínuo das ações e
serviços, para a prevenção, promoção, proteção e recuperação da saúde, incluindo a atenção especial
às doenças que afetam preferencialmente os idosos.
§ 1º A prevenção e a manutenção da saúde do idoso serão efetivadas por meio de:
I – cadastramento da população idosa em base territorial;
II – atendimento geriátrico e gerontológico em ambulatórios;
III – unidades geriátricas de referência, com pessoal especializado nas áreas de geriatria e
gerontologia social;
IV – atendimento domiciliar, incluindo a internação, para a população que dele necessitar e esteja
impossibilitada de se locomover, inclusive para idosos abrigados e acolhidos por instituições públicas,
filantrópicas ou sem fins lucrativos e eventualmente conveniadas com o Poder Público, nos meios urbano
e rural;
V – reabilitação orientada pela geriatria e gerontologia, para redução das sequelas decorrentes do
agravo da saúde.
§ 2º Incumbe ao Poder Público fornecer aos idosos, gratuitamente, medicamentos, especialmente os
de uso continuado, assim como próteses, órteses e outros recursos relativos ao tratamento, habilitação
ou reabilitação.
§ 3º É vedada a discriminação do idoso nos planos de saúde pela cobrança de valores diferenciados
em razão da idade.
§ 4º Os idosos portadores de deficiência ou com limitação incapacitante terão atendimento
especializado, nos termos da lei.
§ 5º É vedado exigir o comparecimento do idoso enfermo perante os órgãos públicos, hipótese na qual
será admitido o seguinte procedimento:
I - quando de interesse do poder público, o agente promoverá o contato necessário com o idoso em
sua residência; ou
II - quando de interesse do próprio idoso, este se fará representar por procurador legalmente
constituído.
§ 6º É assegurado ao idoso enfermo o atendimento domiciliar pela perícia médica do Instituto Nacional
do Seguro Social - INSS, pelo serviço público de saúde ou pelo serviço privado de saúde, contratado ou
conveniado, que integre o Sistema Único de Saúde - SUS, para expedição do laudo de saúde necessário
ao exercício de seus direitos sociais e de isenção tributária.
§ 7º Em todo atendimento de saúde, os maiores de oitenta anos terão preferência especial sobre os
demais idosos, exceto em caso de emergência. (Incluído pela Lei nº 13.466, de 2017).

Art. 16. Ao idoso internado ou em observação é assegurado o direito a acompanhante, devendo o


órgão de saúde proporcionar as condições adequadas para a sua permanência em tempo integral,
segundo o critério médico.
Parágrafo único. Caberá ao profissional de saúde responsável pelo tratamento conceder autorização
para o acompanhamento do idoso ou, no caso de impossibilidade, justificá-la por escrito.

Apostila gerada especialmente para: Betânia Mueller 023.712.580-32


. 165
Art. 17. Ao idoso que esteja no domínio de suas faculdades mentais é assegurado o direito de optar
pelo tratamento de saúde que lhe for reputado mais favorável.
Parágrafo único. Não estando o idoso em condições de proceder à opção, esta será feita:
I – pelo curador, quando o idoso for interditado;
II – pelos familiares, quando o idoso não tiver curador ou este não puder ser contatado em tempo hábil;
III – pelo médico, quando ocorrer iminente risco de vida e não houver tempo hábil para consulta a
curador ou familiar;
IV – pelo próprio médico, quando não houver curador ou familiar conhecido, caso em que deverá
comunicar o fato ao Ministério Público.

Art. 18. As instituições de saúde devem atender aos critérios mínimos para o atendimento às
necessidades do idoso, promovendo o treinamento e a capacitação dos profissionais, assim como
orientação a cuidadores familiares e grupos de autoajuda.

Art. 19. Os casos de suspeita ou confirmação de violência praticada contra idosos serão objeto de
notificação compulsória pelos serviços de saúde públicos e privados à autoridade sanitária, bem como
serão obrigatoriamente comunicados por eles a quaisquer dos seguintes órgãos:
I – autoridade policial;
II – Ministério Público;
III – Conselho Municipal do Idoso;
IV – Conselho Estadual do Idoso;
V – Conselho Nacional do Idoso.
§ 1º Para os efeitos desta Lei, considera-se violência contra o idoso qualquer ação ou omissão
praticada em local público ou privado que lhe cause morte, dano ou sofrimento físico ou psicológico.
§ 2º Aplica-se, no que couber, à notificação compulsória prevista no caput deste artigo, o disposto na
Lei nº 6.259, de 30 de outubro de 1975.

CAPÍTULO V
DA EDUCAÇÃO, CULTURA, ESPORTE E LAZER

Art. 20. O idoso tem direito a educação, cultura, esporte, lazer, diversões, espetáculos, produtos e
serviços que respeitem sua peculiar condição de idade.

Art. 21. O Poder Público criará oportunidades de acesso do idoso à educação, adequando currículos,
metodologias e material didático aos programas educacionais a ele destinados.
§ 1º Os cursos especiais para idosos incluirão conteúdo relativo às técnicas de comunicação,
computação e demais avanços tecnológicos, para sua integração à vida moderna.
§ 2º Os idosos participarão das comemorações de caráter cívico ou cultural, para transmissão de
conhecimentos e vivências às demais gerações, no sentido da preservação da memória e da identidade
culturais.

Art. 22. Nos currículos mínimos dos diversos níveis de ensino formal serão inseridos conteúdos
voltados ao processo de envelhecimento, ao respeito e à valorização do idoso, de forma a eliminar o
preconceito e a produzir conhecimentos sobre a matéria.

Art. 23. A participação dos idosos em atividades culturais e de lazer será proporcionada mediante
descontos de pelo menos 50% (cinquenta por cento) nos ingressos para eventos artísticos, culturais,
esportivos e de lazer, bem como o acesso preferencial aos respectivos locais.

Art. 24. Os meios de comunicação manterão espaços ou horários especiais voltados aos idosos, com
finalidade informativa, educativa, artística e cultural, e ao público sobre o processo de envelhecimento.

Art. 25. As instituições de educação superior ofertarão às pessoas idosas, na perspectiva da educação
ao longo da vida, cursos e programas de extensão, presenciais ou a distância, constituídos por atividades
formais e não formais. (Redação dada pela lei nº 13.535, de 2017)
Parágrafo único. O poder público apoiará a criação de universidade aberta para as pessoas idosas e
incentivará a publicação de livros e periódicos, de conteúdo e padrão editorial adequados ao idoso, que
facilitem a leitura, considerada a natural redução da capacidade visual. (Incluído pela lei nº 13.535, de
2017)

Apostila gerada especialmente para: Betânia Mueller 023.712.580-32


. 166
CAPÍTULO VI
DA PROFISSIONALIZAÇÃO E DO TRABALHO

Art. 26. O idoso tem direito ao exercício de atividade profissional, respeitadas suas condições físicas,
intelectuais e psíquicas.

Art. 27. Na admissão do idoso em qualquer trabalho ou emprego, é vedada a discriminação e a fixação
de limite máximo de idade, inclusive para concursos, ressalvados os casos em que a natureza do cargo
o exigir.
Parágrafo único. O primeiro critério de desempate em concurso público será a idade, dando-se
preferência ao de idade mais elevada.

Art. 28. O Poder Público criará e estimulará programas de:


I – profissionalização especializada para os idosos, aproveitando seus potenciais e habilidades para
atividades regulares e remuneradas;
II – preparação dos trabalhadores para a aposentadoria, com antecedência mínima de 1 (um) ano, por
meio de estímulo a novos projetos sociais, conforme seus interesses, e de esclarecimento sobre os
direitos sociais e de cidadania;
III – estímulo às empresas privadas para admissão de idosos ao trabalho.

CAPÍTULO VII
DA PREVIDÊNCIA SOCIAL

Art. 29. Os benefícios de aposentadoria e pensão do Regime Geral da Previdência Social observarão,
na sua concessão, critérios de cálculo que preservem o valor real dos salários sobre os quais incidiram
contribuição, nos termos da legislação vigente.
Parágrafo único. Os valores dos benefícios em manutenção serão reajustados na mesma data de
reajuste do salário-mínimo, pro rata, de acordo com suas respectivas datas de início ou do seu último
reajustamento, com base em percentual definido em regulamento, observados os critérios estabelecidos
pela Lei nº 8.213, de 24 de julho de 1991.
Art. 30. A perda da condição de segurado não será considerada para a concessão da aposentadoria
por idade, desde que a pessoa conte com, no mínimo, o tempo de contribuição correspondente ao exigido
para efeito de carência na data de requerimento do benefício.
Parágrafo único. O cálculo do valor do benefício previsto no caput observará o disposto no caput e §
2º do art. 3º da Lei nº 9.876, de 26 de novembro de 1999, ou, não havendo salários-de-contribuição
recolhidos a partir da competência de julho de 1994, o disposto no art. 35 da Lei nº 8.213, de 1991.

Art. 31. O pagamento de parcelas relativas a benefícios, efetuado com atraso por responsabilidade da
Previdência Social, será atualizado pelo mesmo índice utilizado para os reajustamentos dos benefícios
do Regime Geral de Previdência Social, verificado no período compreendido entre o mês que deveria ter
sido pago e o mês do efetivo pagamento.

Art. 32. O Dia Mundial do Trabalho, 1º de Maio, é a data-base dos aposentados e pensionistas.

CAPÍTULO VIII
DA ASSISTÊNCIA SOCIAL

Art. 33. A assistência social aos idosos será prestada, de forma articulada, conforme os princípios e
diretrizes previstos na Lei Orgânica da Assistência Social, na Política Nacional do Idoso, no Sistema Único
de Saúde e demais normas pertinentes.

Art. 34. Aos idosos, a partir de 65 (sessenta e cinco) anos, que não possuam meios para prover sua
subsistência, nem de tê-la provida por sua família, é assegurado o benefício mensal de 1 (um) salário-
mínimo, nos termos da Lei Orgânica da Assistência Social – Loas.
Parágrafo único. O benefício já concedido a qualquer membro da família nos termos do caput não será
computado para os fins do cálculo da renda familiar per capita a que se refere a Loas.

Art. 35. Todas as entidades de longa permanência, ou casa-lar, são obrigadas a firmar contrato de
prestação de serviços com a pessoa idosa abrigada.

Apostila gerada especialmente para: Betânia Mueller 023.712.580-32


. 167
§ 1º No caso de entidades filantrópicas, ou casa-lar, é facultada a cobrança de participação do idoso
no custeio da entidade.
§ 2º O Conselho Municipal do Idoso ou o Conselho Municipal da Assistência Social estabelecerá a
forma de participação prevista no § 1º, que não poderá exceder a 70% (setenta por cento) de qualquer
benefício previdenciário ou de assistência social percebido pelo idoso.
§ 3º Se a pessoa idosa for incapaz, caberá a seu representante legal firmar o contrato a que se refere
o caput deste artigo.

Art. 36. O acolhimento de idosos em situação de risco social, por adulto ou núcleo familiar, caracteriza
a dependência econômica, para os efeitos legais.

CAPÍTULO IX
DA HABITAÇÃO

Art. 37. O idoso tem direito a moradia digna, no seio da família natural ou substituta, ou
desacompanhado de seus familiares, quando assim o desejar, ou, ainda, em instituição pública ou
privada.
§ 1º A assistência integral na modalidade de entidade de longa permanência será prestada quando
verificada inexistência de grupo familiar, casa-lar, abandono ou carência de recursos financeiros próprios
ou da família.
§ 2º Toda instituição dedicada ao atendimento ao idoso fica obrigada a manter identificação externa
visível, sob pena de interdição, além de atender toda a legislação pertinente.
§ 3º As instituições que abrigarem idosos são obrigadas a manter padrões de habitação compatíveis
com as necessidades deles, bem como provê-los com alimentação regular e higiene indispensáveis às
normas sanitárias e com estas condizentes, sob as penas da lei.

Art. 38. Nos programas habitacionais, públicos ou subsidiados com recursos públicos, o idoso goza de
prioridade na aquisição de imóvel para moradia própria, observado o seguinte:
I - reserva de pelo menos 3% (três por cento) das unidades habitacionais residenciais para atendimento
aos idosos;
II – implantação de equipamentos urbanos comunitários voltados ao idoso;
III – eliminação de barreiras arquitetônicas e urbanísticas, para garantia de acessibilidade ao idoso;
IV – critérios de financiamento compatíveis com os rendimentos de aposentadoria e pensão.
Parágrafo único. As unidades residenciais reservadas para atendimento a idosos devem situar-se,
preferencialmente, no pavimento térreo.

CAPÍTULO X
DO TRANSPORTE

Art. 39. Aos maiores de 65 (sessenta e cinco) anos fica assegurada a gratuidade dos transportes
coletivos públicos urbanos e semiurbanos, exceto nos serviços seletivos e especiais, quando prestados
paralelamente aos serviços regulares.
§ 1º Para ter acesso à gratuidade, basta que o idoso apresente qualquer documento pessoal que faça
prova de sua idade.
§ 2º Nos veículos de transporte coletivo de que trata este artigo, serão reservados 10% (dez por cento)
dos assentos para os idosos, devidamente identificados com a placa de reservado preferencialmente para
idosos.
§ 3º No caso das pessoas compreendidas na faixa etária entre 60 (sessenta) e 65 (sessenta e cinco)
anos, ficará a critério da legislação local dispor sobre as condições para exercício da gratuidade nos
meios de transporte previstos no caput deste artigo.

Art. 40. No sistema de transporte coletivo interestadual observar-se-á, nos termos da legislação
específica:
I – a reserva de 2 (duas) vagas gratuitas por veículo para idosos com renda igual ou inferior a 2 (dois)
salários-mínimos;
II – desconto de 50% (cinquenta por cento), no mínimo, no valor das passagens, para os idosos que
excederem as vagas gratuitas, com renda igual ou inferior a 2 (dois) salários-mínimos.
Parágrafo único. Caberá aos órgãos competentes definir os mecanismos e os critérios para o exercício
dos direitos previstos nos incisos I e II.

Apostila gerada especialmente para: Betânia Mueller 023.712.580-32


. 168
Art. 41. É assegurada a reserva, para os idosos, nos termos da lei local, de 5% (cinco por cento) das
vagas nos estacionamentos públicos e privados, as quais deverão ser posicionadas de forma a garantir
a melhor comodidade ao idoso.

Art. 42. São asseguradas a prioridade e a segurança do idoso nos procedimentos de embarque e
desembarque nos veículos do sistema de transporte coletivo.

TÍTULO III
DAS MEDIDAS DE PROTEÇÃO
CAPÍTULO I
DAS DISPOSIÇÕES GERAIS

Art. 43. As medidas de proteção ao idoso são aplicáveis sempre que os direitos reconhecidos nesta
Lei forem ameaçados ou violados:
I – por ação ou omissão da sociedade ou do Estado;
II – por falta, omissão ou abuso da família, curador ou entidade de atendimento;
III – em razão de sua condição pessoal.

CAPÍTULO II
DAS MEDIDAS ESPECÍFICAS DE PROTEÇÃO

Art. 44. As medidas de proteção ao idoso previstas nesta Lei poderão ser aplicadas, isolada ou
cumulativamente, e levarão em conta os fins sociais a que se destinam e o fortalecimento dos vínculos
familiares e comunitários.

Art. 45. Verificada qualquer das hipóteses previstas no art. 43, o Ministério Público ou o Poder
Judiciário, a requerimento daquele, poderá determinar, dentre outras, as seguintes medidas:
I – encaminhamento à família ou curador, mediante termo de responsabilidade;
II – orientação, apoio e acompanhamento temporários;
III – requisição para tratamento de sua saúde, em regime ambulatorial, hospitalar ou domiciliar;
IV – inclusão em programa oficial ou comunitário de auxílio, orientação e tratamento a usuários
dependentes de drogas lícitas ou ilícitas, ao próprio idoso ou à pessoa de sua convivência que lhe cause
perturbação;
V – abrigo em entidade;
VI – abrigo temporário.

TÍTULO IV
DA POLÍTICA DE ATENDIMENTO AO IDOSO
CAPÍTULO I
DISPOSIÇÕES GERAIS

Art. 46. A política de atendimento ao idoso far-se-á por meio do conjunto articulado de ações
governamentais e não-governamentais da União, dos Estados, do Distrito Federal e dos Municípios.

Art. 47. São linhas de ação da política de atendimento:


I – políticas sociais básicas, previstas na Lei no 8.842, de 4 de janeiro de 1994;
II – políticas e programas de assistência social, em caráter supletivo, para aqueles que necessitarem;
III – serviços especiais de prevenção e atendimento às vítimas de negligência, maus-tratos,
exploração, abuso, crueldade e opressão;
IV – serviço de identificação e localização de parentes ou responsáveis por idosos abandonados em
hospitais e instituições de longa permanência;
V – proteção jurídico-social por entidades de defesa dos direitos dos idosos;
VI – mobilização da opinião pública no sentido da participação dos diversos segmentos da sociedade
no atendimento do idoso.

Apostila gerada especialmente para: Betânia Mueller 023.712.580-32


. 169
CAPÍTULO II
DAS ENTIDADES DE ATENDIMENTO AO IDOSO

Art. 48. As entidades de atendimento são responsáveis pela manutenção das próprias unidades,
observadas as normas de planejamento e execução emanadas do órgão competente da Política Nacional
do Idoso, conforme a Lei no 8.842, de 1994.
Parágrafo único. As entidades governamentais e não-governamentais de assistência ao idoso ficam
sujeitas à inscrição de seus programas, junto ao órgão competente da Vigilância Sanitária e Conselho
Municipal da Pessoa Idosa, e em sua falta, junto ao Conselho Estadual ou Nacional da Pessoa Idosa,
especificando os regimes de atendimento, observados os seguintes requisitos:
I – oferecer instalações físicas em condições adequadas de habitabilidade, higiene, salubridade e
segurança;
II – apresentar objetivos estatutários e plano de trabalho compatíveis com os princípios desta Lei;
III – estar regularmente constituída;
IV – demonstrar a idoneidade de seus dirigentes.

Art. 49. As entidades que desenvolvam programas de institucionalização de longa permanência


adotarão os seguintes princípios:
I – preservação dos vínculos familiares;
II – atendimento personalizado e em pequenos grupos;
III – manutenção do idoso na mesma instituição, salvo em caso de força maior;
IV – participação do idoso nas atividades comunitárias, de caráter interno e externo;
V – observância dos direitos e garantias dos idosos;
VI – preservação da identidade do idoso e oferecimento de ambiente de respeito e dignidade.
Parágrafo único. O dirigente de instituição prestadora de atendimento ao idoso responderá civil e
criminalmente pelos atos que praticar em detrimento do idoso, sem prejuízo das sanções administrativas.

Art. 50. Constituem obrigações das entidades de atendimento:


I – celebrar contrato escrito de prestação de serviço com o idoso, especificando o tipo de atendimento,
as obrigações da entidade e prestações decorrentes do contrato, com os respectivos preços, se for o
caso;
II – observar os direitos e as garantias de que são titulares os idosos;
III – fornecer vestuário adequado, se for pública, e alimentação suficiente;
IV – oferecer instalações físicas em condições adequadas de habitabilidade;
V – oferecer atendimento personalizado;
VI – diligenciar no sentido da preservação dos vínculos familiares;
VII – oferecer acomodações apropriadas para recebimento de visitas;
VIII – proporcionar cuidados à saúde, conforme a necessidade do idoso;
IX – promover atividades educacionais, esportivas, culturais e de lazer;
X – propiciar assistência religiosa àqueles que desejarem, de acordo com suas crenças;
XI – proceder a estudo social e pessoal de cada caso;
XII – comunicar à autoridade competente de saúde toda ocorrência de idoso portador de doenças
infectocontagiosas;
XIII – providenciar ou solicitar que o Ministério Público requisite os documentos necessários ao
exercício da cidadania àqueles que não os tiverem, na forma da lei;
XIV – fornecer comprovante de depósito dos bens móveis que receberem dos idosos;
XV – manter arquivo de anotações onde constem data e circunstâncias do atendimento, nome do
idoso, responsável, parentes, endereços, cidade, relação de seus pertences, bem como o valor de
contribuições, e suas alterações, se houver, e demais dados que possibilitem sua identificação e a
individualização do atendimento;
XVI – comunicar ao Ministério Público, para as providências cabíveis, a situação de abandono moral
ou material por parte dos familiares;
XVII – manter no quadro de pessoal profissionais com formação específica.

Art. 51. As instituições filantrópicas ou sem fins lucrativos prestadoras de serviço ao idoso terão direito
à assistência judiciária gratuita.

Apostila gerada especialmente para: Betânia Mueller 023.712.580-32


. 170
CAPÍTULO III
DA FISCALIZAÇÃO DAS ENTIDADES DE ATENDIMENTO

Art. 52. As entidades governamentais e não-governamentais de atendimento ao idoso serão


fiscalizadas pelos Conselhos do Idoso, Ministério Público, Vigilância Sanitária e outros previstos em lei.

Art. 53. O art. 7º da Lei nº 8.842, de 1994, passa a vigorar com a seguinte redação:

"Art. 7º Compete aos Conselhos de que trata o art. 6º desta Lei a supervisão, o acompanhamento, a
fiscalização e a avaliação da política nacional do idoso, no âmbito das respectivas instâncias político-
administrativas." (NR)

Art. 54. Será dada publicidade das prestações de contas dos recursos públicos e privados recebidos
pelas entidades de atendimento.

Art. 55. As entidades de atendimento que descumprirem as determinações desta Lei ficarão sujeitas,
sem prejuízo da responsabilidade civil e criminal de seus dirigentes ou prepostos, às seguintes
penalidades, observado o devido processo legal:
I – as entidades governamentais:
a) advertência;
b) afastamento provisório de seus dirigentes;
c) afastamento definitivo de seus dirigentes;
d) fechamento de unidade ou interdição de programa;
II – as entidades não-governamentais:
a) advertência;
b) multa;
c) suspensão parcial ou total do repasse de verbas públicas;
d) interdição de unidade ou suspensão de programa;
e) proibição de atendimento a idosos a bem do interesse público.
§ 1º Havendo danos aos idosos abrigados ou qualquer tipo de fraude em relação ao programa, caberá
o afastamento provisório dos dirigentes ou a interdição da unidade e a suspensão do programa.
§ 2º A suspensão parcial ou total do repasse de verbas públicas ocorrerá quando verificada a má
aplicação ou desvio de finalidade dos recursos.
§ 3º Na ocorrência de infração por entidade de atendimento, que coloque em risco os direitos
assegurados nesta Lei, será o fato comunicado ao Ministério Público, para as providências cabíveis,
inclusive para promover a suspensão das atividades ou dissolução da entidade, com a proibição de
atendimento a idosos a bem do interesse público, sem prejuízo das providências a serem tomadas pela
Vigilância Sanitária.
§ 4º Na aplicação das penalidades, serão consideradas a natureza e a gravidade da infração cometida,
os danos que dela provierem para o idoso, as circunstâncias agravantes ou atenuantes e os antecedentes
da entidade.

CAPÍTULO IV
DAS INFRAÇÕES ADMINISTRATIVAS

Art. 56. Deixar a entidade de atendimento de cumprir as determinações do art. 50 desta Lei:
Pena – multa de R$ 500,00 (quinhentos reais) a R$ 3.000,00 (três mil reais), se o fato não for
caracterizado como crime, podendo haver a interdição do estabelecimento até que sejam cumpridas as
exigências legais.
Parágrafo único. No caso de interdição do estabelecimento de longa permanência, os idosos abrigados
serão transferidos para outra instituição, a expensas do estabelecimento interditado, enquanto durar a
interdição.

Art. 57. Deixar o profissional de saúde ou o responsável por estabelecimento de saúde ou instituição
de longa permanência de comunicar à autoridade competente os casos de crimes contra idoso de que
tiver conhecimento:
Pena – multa de R$ 500,00 (quinhentos reais) a R$ 3.000,00 (três mil reais), aplicada em dobro no
caso de reincidência.

Apostila gerada especialmente para: Betânia Mueller 023.712.580-32


. 171
Art. 58. Deixar de cumprir as determinações desta Lei sobre a prioridade no atendimento ao idoso:
Pena – multa de R$ 500,00 (quinhentos reais) a R$ 1.000,00 (um mil reais) e multa civil a ser estipulada
pelo juiz, conforme o dano sofrido pelo idoso.

CAPÍTULO V
DA APURAÇÃO ADMINISTRATIVA DE INFRAÇÃO ÀS
NORMAS DE PROTEÇÃO AO IDOSO

Art. 59. Os valores monetários expressos no Capítulo IV serão atualizados anualmente, na forma da
lei.

Art. 60. O procedimento para a imposição de penalidade administrativa por infração às normas de
proteção ao idoso terá início com requisição do Ministério Público ou auto de infração elaborado por
servidor efetivo e assinado, se possível, por duas testemunhas.
§ 1º No procedimento iniciado com o auto de infração poderão ser usadas fórmulas impressas,
especificando-se a natureza e as circunstâncias da infração.
§ 2º Sempre que possível, à verificação da infração seguir-se-á a lavratura do auto, ou este será
lavrado dentro de 24 (vinte e quatro) horas, por motivo justificado.

Art. 61. O autuado terá prazo de 10 (dez) dias para a apresentação da defesa, contado da data da
intimação, que será feita:
I – pelo autuante, no instrumento de autuação, quando for lavrado na presença do infrator;
II – por via postal, com aviso de recebimento.

Art. 62. Havendo risco para a vida ou à saúde do idoso, a autoridade competente aplicará à entidade
de atendimento as sanções regulamentares, sem prejuízo da iniciativa e das providências que vierem a
ser adotadas pelo Ministério Público ou pelas demais instituições legitimadas para a fiscalização.

Art. 63. Nos casos em que não houver risco para a vida ou a saúde da pessoa idosa abrigada, a
autoridade competente aplicará à entidade de atendimento as sanções regulamentares, sem prejuízo da
iniciativa e das providências que vierem a ser adotadas pelo Ministério Público ou pelas demais
instituições legitimadas para a fiscalização.

CAPÍTULO VI
DA APURAÇÃO JUDICIAL DE IRREGULARIDADES EM ENTIDADE DE ATENDIMENTO

Art. 64. Aplicam-se, subsidiariamente, ao procedimento administrativo de que trata este Capítulo as
disposições das Leis nos 6.437, de 20 de agosto de 1977, e 9.784, de 29 de janeiro de 1999.

Art. 65. O procedimento de apuração de irregularidade em entidade governamental e não-


governamental de atendimento ao idoso terá início mediante petição fundamentada de pessoa
interessada ou iniciativa do Ministério Público.

Art. 66. Havendo motivo grave, poderá a autoridade judiciária, ouvido o Ministério Público, decretar
liminarmente o afastamento provisório do dirigente da entidade ou outras medidas que julgar adequadas,
para evitar lesão aos direitos do idoso, mediante decisão fundamentada.

Art. 67. O dirigente da entidade será citado para, no prazo de 10 (dez) dias, oferecer resposta escrita,
podendo juntar documentos e indicar as provas a produzir.

Art. 68. Apresentada a defesa, o juiz procederá na conformidade do art. 69 ou, se necessário,
designará audiência de instrução e julgamento, deliberando sobre a necessidade de produção de outras
provas.
§ 1º Salvo manifestação em audiência, as partes e o Ministério Público terão 5 (cinco) dias para
oferecer alegações finais, decidindo a autoridade judiciária em igual prazo.
§ 2º Em se tratando de afastamento provisório ou definitivo de dirigente de entidade governamental, a
autoridade judiciária oficiará a autoridade administrativa imediatamente superior ao afastado, fixando-lhe
prazo de 24 (vinte e quatro) horas para proceder à substituição.

Apostila gerada especialmente para: Betânia Mueller 023.712.580-32


. 172
§ 3º Antes de aplicar qualquer das medidas, a autoridade judiciária poderá fixar prazo para a remoção
das irregularidades verificadas. Satisfeitas as exigências, o processo será extinto, sem julgamento do
mérito.
§ 4º A multa e a advertência serão impostas ao dirigente da entidade ou ao responsável pelo programa
de atendimento.

TÍTULO V
DO ACESSO À JUSTIÇA
CAPÍTULO I
DISPOSIÇÕES GERAIS

Art. 69. Aplica-se, subsidiariamente, às disposições deste Capítulo, o procedimento sumário previsto
no Código de Processo Civil, naquilo que não contrarie os prazos previstos nesta Lei.

Art. 70. O Poder Público poderá criar varas especializadas e exclusivas do idoso.

Art. 71. É assegurada prioridade na tramitação dos processos e procedimentos e na execução dos
atos e diligências judiciais em que figure como parte ou interveniente pessoa com idade igual ou superior
a 60 (sessenta) anos, em qualquer instância.
§ 1º O interessado na obtenção da prioridade a que alude este artigo, fazendo prova de sua idade,
requererá o benefício à autoridade judiciária competente para decidir o feito, que determinará as
providências a serem cumpridas, anotando-se essa circunstância em local visível nos autos do processo.
§ 2º A prioridade não cessará com a morte do beneficiado, estendendo-se em favor do cônjuge
supérstite, companheiro ou companheira, com união estável, maior de 60 (sessenta) anos.
§ 3º A prioridade se estende aos processos e procedimentos na Administração Pública, empresas
prestadoras de serviços públicos e instituições financeiras, ao atendimento preferencial junto à Defensoria
Pública da União, dos Estados e do Distrito Federal em relação aos Serviços de Assistência Judiciária.
§ 4º Para o atendimento prioritário será garantido ao idoso o fácil acesso aos assentos e caixas,
identificados com a destinação a idosos em local visível e caracteres legíveis.
§ 5º Dentre os processos de idosos, dar-se-á prioridade especial aos maiores de oitenta anos.
(Incluído pela Lei nº 13.466, de 2017).

CAPÍTULO II
DO MINISTÉRIO PÚBLICO

Art. 72. (VETADO)

Art. 73. As funções do Ministério Público, previstas nesta Lei, serão exercidas nos termos da respectiva
Lei Orgânica.

Art. 74. Compete ao Ministério Público:


I – instaurar o inquérito civil e a ação civil pública para a proteção dos direitos e interesses difusos ou
coletivos, individuais indisponíveis e individuais homogêneos do idoso;
II – promover e acompanhar as ações de alimentos, de interdição total ou parcial, de designação de
curador especial, em circunstâncias que justifiquem a medida e oficiar em todos os feitos em que se
discutam os direitos de idosos em condições de risco;
III – atuar como substituto processual do idoso em situação de risco, conforme o disposto no art. 43
desta Lei;
IV – promover a revogação de instrumento procuratório do idoso, nas hipóteses previstas no art. 43
desta Lei, quando necessário ou o interesse público justificar;
V – instaurar procedimento administrativo e, para instruí-lo:
a) expedir notificações, colher depoimentos ou esclarecimentos e, em caso de não comparecimento
injustificado da pessoa notificada, requisitar condução coercitiva, inclusive pela Polícia Civil ou Militar;
b) requisitar informações, exames, perícias e documentos de autoridades municipais, estaduais e
federais, da administração direta e indireta, bem como promover inspeções e diligências investigatórias;
c) requisitar informações e documentos particulares de instituições privadas;
VI – instaurar sindicâncias, requisitar diligências investigatórias e a instauração de inquérito policial,
para a apuração de ilícitos ou infrações às normas de proteção ao idoso;

Apostila gerada especialmente para: Betânia Mueller 023.712.580-32


. 173
VII – zelar pelo efetivo respeito aos direitos e garantias legais assegurados ao idoso, promovendo as
medidas judiciais e extrajudiciais cabíveis;
VIII – inspecionar as entidades públicas e particulares de atendimento e os programas de que trata
esta Lei, adotando de pronto as medidas administrativas ou judiciais necessárias à remoção de
irregularidades porventura verificadas;
IX – requisitar força policial, bem como a colaboração dos serviços de saúde, educacionais e de
assistência social, públicos, para o desempenho de suas atribuições;
X – referendar transações envolvendo interesses e direitos dos idosos previstos nesta Lei.
§ 1º A legitimação do Ministério Público para as ações cíveis previstas neste artigo não impede a de
terceiros, nas mesmas hipóteses, segundo dispuser a lei.
§ 2º As atribuições constantes deste artigo não excluem outras, desde que compatíveis com a
finalidade e atribuições do Ministério Público.
§ 3º O representante do Ministério Público, no exercício de suas funções, terá livre acesso a toda
entidade de atendimento ao idoso.

Art. 75. Nos processos e procedimentos em que não for parte, atuará obrigatoriamente o Ministério
Público na defesa dos direitos e interesses de que cuida esta Lei, hipóteses em que terá vista dos autos
depois das partes, podendo juntar documentos, requerer diligências e produção de outras provas, usando
os recursos cabíveis.

Art. 76. A intimação do Ministério Público, em qualquer caso, será feita pessoalmente.

Art. 77. A falta de intervenção do Ministério Público acarreta a nulidade do feito, que será declarada
de ofício pelo juiz ou a requerimento de qualquer interessado.

CAPÍTULO III
DA PROTEÇÃO JUDICIAL DOS INTERESSES DIFUSOS, COLETIVOS E INDIVIDUAIS
INDISPONÍVEIS OU HOMOGÊNEOS

Art. 78. As manifestações processuais do representante do Ministério Público deverão ser


fundamentadas.

Art. 79. Regem-se pelas disposições desta Lei as ações de responsabilidade por ofensa aos direitos
assegurados ao idoso, referentes à omissão ou ao oferecimento insatisfatório de:
I – acesso às ações e serviços de saúde;
II – atendimento especializado ao idoso portador de deficiência ou com limitação incapacitante;
III – atendimento especializado ao idoso portador de doença infectocontagiosa;
IV – serviço de assistência social visando ao amparo do idoso.
Parágrafo único. As hipóteses previstas neste artigo não excluem da proteção judicial outros interesses
difusos, coletivos, individuais indisponíveis ou homogêneos, próprios do idoso, protegidos em lei.

Art. 80. As ações previstas neste Capítulo serão propostas no foro do domicílio do idoso, cujo juízo
terá competência absoluta para processar a causa, ressalvadas as competências da Justiça Federal e a
competência originária dos Tribunais Superiores.

Art. 81. Para as ações cíveis fundadas em interesses difusos, coletivos, individuais indisponíveis ou
homogêneos, consideram-se legitimados, concorrentemente:
I – o Ministério Público;
II – a União, os Estados, o Distrito Federal e os Municípios;
III – a Ordem dos Advogados do Brasil;
IV – as associações legalmente constituídas há pelo menos 1 (um) ano e que incluam entre os fins
institucionais a defesa dos interesses e direitos da pessoa idosa, dispensada a autorização da
assembleia, se houver prévia autorização estatutária.
§ 1º Admitir-se-á litisconsórcio facultativo entre os Ministérios Públicos da União e dos Estados na
defesa dos interesses e direitos de que cuida esta Lei.
§ 2º Em caso de desistência ou abandono da ação por associação legitimada, o Ministério Público ou
outro legitimado deverá assumir a titularidade ativa.

Apostila gerada especialmente para: Betânia Mueller 023.712.580-32


. 174
Art. 82. Para defesa dos interesses e direitos protegidos por esta Lei, são admissíveis todas as
espécies de ação pertinentes.
Parágrafo único. Contra atos ilegais ou abusivos de autoridade pública ou agente de pessoa jurídica
no exercício de atribuições de Poder Público, que lesem direito líquido e certo previsto nesta Lei, caberá
ação mandamental, que se regerá pelas normas da lei do mandado de segurança.

Art. 83. Na ação que tenha por objeto o cumprimento de obrigação de fazer ou não-fazer, o juiz
concederá a tutela específica da obrigação ou determinará providências que assegurem o resultado
prático equivalente ao adimplemento.
§ 1º Sendo relevante o fundamento da demanda e havendo justificado receio de ineficácia do
provimento final, é lícito ao juiz conceder a tutela liminarmente ou após justificação prévia, na forma do
art. 273 do Código de Processo Civil.
§ 2º O juiz poderá, na hipótese do § 1º ou na sentença, impor multa diária ao réu, independentemente
do pedido do autor, se for suficiente ou compatível com a obrigação, fixando prazo razoável para o
cumprimento do preceito.
§ 3º A multa só será exigível do réu após o trânsito em julgado da sentença favorável ao autor, mas
será devida desde o dia em que se houver configurado.

Art. 84. Os valores das multas previstas nesta Lei reverterão ao Fundo do Idoso, onde houver, ou na
falta deste, ao Fundo Municipal de Assistência Social, ficando vinculados ao atendimento ao idoso.
Parágrafo único. As multas não recolhidas até 30 (trinta) dias após o trânsito em julgado da decisão
serão exigidas por meio de execução promovida pelo Ministério Público, nos mesmos autos, facultada
igual iniciativa aos demais legitimados em caso de inércia daquele.

Art. 85. O juiz poderá conferir efeito suspensivo aos recursos, para evitar dano irreparável à parte.

Art. 86. Transitada em julgado a sentença que impuser condenação ao Poder Público, o juiz
determinará a remessa de peças à autoridade competente, para apuração da responsabilidade civil e
administrativa do agente a que se atribua a ação ou omissão.

Art. 87. Decorridos 60 (sessenta) dias do trânsito em julgado da sentença condenatória favorável ao
idoso sem que o autor lhe promova a execução, deverá fazê-lo o Ministério Público, facultada, igual
iniciativa aos demais legitimados, como assistentes ou assumindo o polo ativo, em caso de inércia desse
órgão.

Art. 88. Nas ações de que trata este Capítulo, não haverá adiantamento de custas, emolumentos,
honorários periciais e quaisquer outras despesas.
Parágrafo único. Não se imporá sucumbência ao Ministério Público.

Art. 89. Qualquer pessoa poderá, e o servidor deverá, provocar a iniciativa do Ministério Público,
prestando-lhe informações sobre os fatos que constituam objeto de ação civil e indicando-lhe os
elementos de convicção.

Art. 90. Os agentes públicos em geral, os juízes e tribunais, no exercício de suas funções, quando
tiverem conhecimento de fatos que possam configurar crime de ação pública contra idoso ou ensejar a
propositura de ação para sua defesa, devem encaminhar as peças pertinentes ao Ministério Público, para
as providências cabíveis.

Art. 91. Para instruir a petição inicial, o interessado poderá requerer às autoridades competentes as
certidões e informações que julgar necessárias, que serão fornecidas no prazo de 10 (dez) dias.

Art. 92. O Ministério Público poderá instaurar sob sua presidência, inquérito civil, ou requisitar, de
qualquer pessoa, organismo público ou particular, certidões, informações, exames ou perícias, no prazo
que assinalar, o qual não poderá ser inferior a 10 (dez) dias.
§ 1º Se o órgão do Ministério Público, esgotadas todas as diligências, se convencer da inexistência de
fundamento para a propositura da ação civil ou de peças informativas, determinará o seu arquivamento,
fazendo-o fundamentadamente.

Apostila gerada especialmente para: Betânia Mueller 023.712.580-32


. 175
§ 2º Os autos do inquérito civil ou as peças de informação arquivados serão remetidos, sob pena de
se incorrer em falta grave, no prazo de 3 (três) dias, ao Conselho Superior do Ministério Público ou à
Câmara de Coordenação e Revisão do Ministério Público.
§ 3º Até que seja homologado ou rejeitado o arquivamento, pelo Conselho Superior do Ministério
Público ou por Câmara de Coordenação e Revisão do Ministério Público, as associações legitimadas
poderão apresentar razões escritas ou documentos, que serão juntados ou anexados às peças de
informação.
§ 4º Deixando o Conselho Superior ou a Câmara de Coordenação e Revisão do Ministério Público de
homologar a promoção de arquivamento, será designado outro membro do Ministério Público para o
ajuizamento da ação.
TÍTULO VI
DOS CRIMES
CAPÍTULO I
DISPOSIÇÕES GERAIS

Art. 93. Aplicam-se subsidiariamente, no que couber, as disposições da Lei nº 7.347, de 24 de julho de
1985.

Art. 94. Aos crimes previstos nesta Lei, cuja pena máxima privativa de liberdade não ultrapasse 4
(quatro) anos, aplica-se o procedimento previsto na Lei nº 9.099, de 26 de setembro de 1995, e,
subsidiariamente, no que couber, as disposições do Código Penal e do Código de Processo Penal.

CAPÍTULO II
DOS CRIMES EM ESPÉCIE

Art. 95. Os crimes definidos nesta Lei são de ação penal pública incondicionada, não se lhes aplicando
os arts. 181 e 182 do Código Penal.

Art. 96. Discriminar pessoa idosa, impedindo ou dificultando seu acesso a operações bancárias, aos
meios de transporte, ao direito de contratar ou por qualquer outro meio ou instrumento necessário ao
exercício da cidadania, por motivo de idade:
Pena – reclusão de 6 (seis) meses a 1 (um) ano e multa.
§ 1º Na mesma pena incorre quem desdenhar, humilhar, menosprezar ou discriminar pessoa idosa,
por qualquer motivo.
§ 2º A pena será aumentada de 1/3 (um terço) se a vítima se encontrar sob os cuidados ou
responsabilidade do agente.

Art. 97. Deixar de prestar assistência ao idoso, quando possível fazê-lo sem risco pessoal, em situação
de iminente perigo, ou recusar, retardar ou dificultar sua assistência à saúde, sem justa causa, ou não
pedir, nesses casos, o socorro de autoridade pública:
Pena – detenção de 6 (seis) meses a 1 (um) ano e multa.
Parágrafo único. A pena é aumentada de metade, se da omissão resulta lesão corporal de natureza
grave, e triplicada, se resulta a morte.

Art. 98. Abandonar o idoso em hospitais, casas de saúde, entidades de longa permanência, ou
congêneres, ou não prover suas necessidades básicas, quando obrigado por lei ou mandado:
Pena – detenção de 6 (seis) meses a 3 (três) anos e multa.

Art. 99. Expor a perigo a integridade e a saúde, física ou psíquica, do idoso, submetendo-o a condições
desumanas ou degradantes ou privando-o de alimentos e cuidados indispensáveis, quando obrigado a
fazê-lo, ou sujeitando-o a trabalho excessivo ou inadequado:
Pena – detenção de 2 (dois) meses a 1 (um) ano e multa.
§ 1º Se do fato resulta lesão corporal de natureza grave:
Pena – reclusão de 1 (um) a 4 (quatro) anos.
§ 2º Se resulta a morte:
Pena – reclusão de 4 (quatro) a 12 (doze) anos.

Art. 100. Constitui crime punível com reclusão de 6 (seis) meses a 1 (um) ano e multa:
I – obstar o acesso de alguém a qualquer cargo público por motivo de idade;

Apostila gerada especialmente para: Betânia Mueller 023.712.580-32


. 176
II – negar a alguém, por motivo de idade, emprego ou trabalho;
III – recusar, retardar ou dificultar atendimento ou deixar de prestar assistência à saúde, sem justa
causa, a pessoa idosa;
IV – deixar de cumprir, retardar ou frustrar, sem justo motivo, a execução de ordem judicial expedida
na ação civil a que alude esta Lei;
V – recusar, retardar ou omitir dados técnicos indispensáveis à propositura da ação civil objeto desta
Lei, quando requisitados pelo Ministério Público.

Art. 101. Deixar de cumprir, retardar ou frustrar, sem justo motivo, a execução de ordem judicial
expedida nas ações em que for parte ou interveniente o idoso:
Pena – detenção de 6 (seis) meses a 1 (um) ano e multa.

Art. 102. Apropriar-se de ou desviar bens, proventos, pensão ou qualquer outro rendimento do idoso,
dando-lhes aplicação diversa da de sua finalidade:
Pena – reclusão de 1 (um) a 4 (quatro) anos e multa.

Art. 103. Negar o acolhimento ou a permanência do idoso, como abrigado, por recusa deste em
outorgar procuração à entidade de atendimento:
Pena – detenção de 6 (seis) meses a 1 (um) ano e multa.

Art. 104. Reter o cartão magnético de conta bancária relativa a benefícios, proventos ou pensão do
idoso, bem como qualquer outro documento com objetivo de assegurar recebimento ou ressarcimento de
dívida:
Pena – detenção de 6 (seis) meses a 2 (dois) anos e multa.

Art. 105. Exibir ou veicular, por qualquer meio de comunicação, informações ou imagens depreciativas
ou injuriosas à pessoa do idoso:
Pena – detenção de 1 (um) a 3 (três) anos e multa.

Art. 106. Induzir pessoa idosa sem discernimento de seus atos a outorgar procuração para fins de
administração de bens ou deles dispor livremente:
Pena – reclusão de 2 (dois) a 4 (quatro) anos.
Art. 107. Coagir, de qualquer modo, o idoso a doar, contratar, testar ou outorgar procuração:
Pena – reclusão de 2 (dois) a 5 (cinco) anos.

Art. 108. Lavrar ato notarial que envolva pessoa idosa sem discernimento de seus atos, sem a devida
representação legal:
Pena – reclusão de 2 (dois) a 4 (quatro) anos.
TÍTULO VII
DISPOSIÇÕES FINAIS E TRANSITÓRIAS

Art. 109. Impedir ou embaraçar ato do representante do Ministério Público ou de qualquer outro agente
fiscalizador:
Pena – reclusão de 6 (seis) meses a 1 (um) ano e multa.

Art. 110. O Decreto-Lei no 2.848, de 7 de dezembro de 1940, Código Penal, passa a vigorar com as
seguintes alterações:
"Art. 61. ............................................................................
II - ............................................................................
h) contra criança, maior de 60 (sessenta) anos, enfermo ou mulher grávida;
............................................................................." (NR)

"Art. 121. ............................................................................


§ 4º No homicídio culposo, a pena é aumentada de 1/3 (um terço), se o crime resulta de inobservância
de regra técnica de profissão, arte ou ofício, ou se o agente deixa de prestar imediato socorro à vítima,
não procura diminuir as consequências do seu ato, ou foge para evitar prisão em flagrante. Sendo doloso
o homicídio, a pena é aumentada de 1/3 (um terço) se o crime é praticado contra pessoa menor de 14
(quatorze) ou maior de 60 (sessenta) anos.
............................................................................." (NR)

Apostila gerada especialmente para: Betânia Mueller 023.712.580-32


. 177
"Art. 133. ............................................................................
§ 3º ............................................................................
III – se a vítima é maior de 60 (sessenta) anos." (NR)

"Art. 140. ............................................................................


§ 3º Se a injúria consiste na utilização de elementos referentes a raça, cor, etnia, religião, origem ou a
condição de pessoa idosa ou portadora de deficiência:
............................................................................ (NR)

"Art. 141. ............................................................................


IV – contra pessoa maior de 60 (sessenta) anos ou portadora de deficiência, exceto no caso de injúria.
............................................................................." (NR)

"Art. 148. ............................................................................


§ 1º............................................................................
I – se a vítima é ascendente, descendente, cônjuge do agente ou maior de 60 (sessenta) anos.
............................................................................" (NR)

"Art. 159............................................................................
§ 1º Se o sequestro dura mais de 24 (vinte e quatro) horas, se o sequestrado é menor de 18 (dezoito)
ou maior de 60 (sessenta) anos, ou se o crime é cometido por bando ou quadrilha.
............................................................................" (NR)

"Art. 183............................................................................
III – se o crime é praticado contra pessoa com idade igual ou superior a 60 (sessenta) anos." (NR)

"Art. 244. Deixar, sem justa causa, de prover a subsistência do cônjuge, ou de filho menor de 18
(dezoito) anos ou inapto para o trabalho, ou de ascendente inválido ou maior de 60 (sessenta) anos, não
lhes proporcionando os recursos necessários ou faltando ao pagamento de pensão alimentícia
judicialmente acordada, fixada ou majorada; deixar, sem justa causa, de socorrer descendente ou
ascendente, gravemente enfermo:
............................................................................" (NR)
Art. 111. O art. 21 do Decreto-Lei no 3.688, de 3 de outubro de 1941, Lei das Contravenções Penais,
passa a vigorar acrescido do seguinte parágrafo único:

"Art. 21............................................................................
Parágrafo único. Aumenta-se a pena de 1/3 (um terço) até a metade se a vítima é maior de 60
(sessenta) anos." (NR)

Art. 112. O inciso II do § 4º do art. 1º da Lei no 9.455, de 7 de abril de 1997, passa a vigorar com a
seguinte redação:

"Art. 1º ............................................................................
§ 4º ............................................................................
II – se o crime é cometido contra criança, gestante, portador de deficiência, adolescente ou maior de
60 (sessenta) anos;
............................................................................" (NR)

Art. 113. O inciso III do art. 18 da Lei no 6.368, de 21 de outubro de 1976, passa a vigorar com a
seguinte redação:

"Art. 18............................................................................
III – se qualquer deles decorrer de associação ou visar a menores de 21 (vinte e um) anos ou a pessoa
com idade igual ou superior a 60 (sessenta) anos ou a quem tenha, por qualquer causa, diminuída ou
suprimida a capacidade de discernimento ou de autodeterminação:
............................................................................" (NR)

Art. 114. O art. 1º da Lei no 10.048, de 8 de novembro de 2000, passa a vigorar com a seguinte
redação:

Apostila gerada especialmente para: Betânia Mueller 023.712.580-32


. 178
"Art. 1º As pessoas portadoras de deficiência, os idosos com idade igual ou superior a 60 (sessenta)
anos, as gestantes, as lactantes e as pessoas acompanhadas por crianças de colo terão atendimento
prioritário, nos termos desta Lei." (NR)

Art. 115. O Orçamento da Seguridade Social destinará ao Fundo Nacional de Assistência Social, até
que o Fundo Nacional do Idoso seja criado, os recursos necessários, em cada exercício financeiro, para
aplicação em programas e ações relativos ao idoso.

Art. 116. Serão incluídos nos censos demográficos dados relativos à população idosa do País.

Art. 117. O Poder Executivo encaminhará ao Congresso Nacional projeto de lei revendo os critérios de
concessão do Benefício de Prestação Continuada previsto na Lei Orgânica da Assistência Social, de
forma a garantir que o acesso ao direito seja condizente com o estágio de desenvolvimento
socioeconômico alcançado pelo País.

Art. 118. Esta Lei entra em vigor decorridos 90 (noventa) dias da sua publicação, ressalvado o disposto
no caput do art. 36, que vigorará a partir de 1º de janeiro de 2004.

Brasília, 1º de outubro de 2003; 182º da Independência e 115º da República.

LUIZ INÁCIO LULA DA SILVA

Questões

01. (Prefeitura de Sul Brasil/SC - Educador Social – ALTERNATIVE CONCURSOS/2017) De


acordo com o Estatuto do Idoso, Lei nº 10.741/2003, art. 23, a participação dos idosos em atividades
culturais e de lazer será proporcionada mediante descontos de pelo menos _____________________
nos ingressos para eventos artísticos, culturais, esportivos e de lazer, bem como o acesso preferencial
aos respectivos locais.
(A) 5% (cinco por cento)
(B) 15% (quinze por cento)
(C) 25% (vinte e cinco por cento)
(D) 50% (cinquenta por cento)
(E) 75% (setenta e cinco por cento)

02. (TRF 2ªREGIÃO - Analista Judiciário - CONSUPLAN/2017) O Estatuto do Idoso regula os direitos
assegurados às pessoas com idade igual ou superior a sessenta anos. Sobre estes direitos previstos na
Lei Federal nº 10.741, de 1º de outubro de 2003, é INCORRETO afirmar que:
(A) Ao idoso que esteja no domínio de suas faculdades mentais é assegurado o direito de optar pelo
tratamento de saúde que lhe for reputado mais favorável.
(B) O Poder Público criará e estimulará programas de profissionalização especializada para os idosos,
aproveitando seus potenciais e habilidades para atividades regulares e remuneradas.
(C) Os benefícios de aposentadoria e pensão do Regime Geral da Previdência Social observarão, na
sua concessão, critérios de cálculo que preservem o valor real dos salários sobre os quais incidiram
contribuição, nos termos da legislação vigente.
(D) Aos idosos, a partir de sessenta e cinco anos, que não possuam meios para prover sua
subsistência, nem tê-la provida por sua família, é assegurado o benefício mensal de um salário-mínimo,
nos termos da Lei Orgânica da Assistência Social – Loas. O benefício já concedido a qualquer membro
da família nos termos exposto anteriormente será computado para os fins do cálculo da renda familiar per
capita a que se refere a LOAS.

03. (Prefeitura de Sul Brasil – SC - Educador Social – Alternative Concursos/2017) De acordo


com o Estatuto do Idoso, Lei nº 10.741/2003, art. 28, o Poder Público criará e estimulará programas de:
I. Profissionalização especializada para os idosos, aproveitando seus potenciais e habilidades para
atividades regulares e remuneradas.
II. Preparação dos trabalhadores para a aposentadoria, com antecedência mínima de 5 (cinco) anos,
por meio de estímulo a novos projetos sociais, conforme seus interesses, e de esclarecimento sobre os
direitos sociais e de cidadania.
III. Estímulo às empresas públicas para admissão de idosos ao trabalho.

Apostila gerada especialmente para: Betânia Mueller 023.712.580-32


. 179
(A) Somente I está incorreta.
(B) Somente II e III estão incorretas.
(C) Somente III está incorreta.
(D) Somente I e III estão incorretas.
(E) Todas estão corretas.

04. (Prefeitura de Natal/RN - Advogado – IDECAN/2016). Estabelece o Estatuto do Idoso que NÃO
constitui obrigação das entidades de atendimento ao idoso:
(A) Oferecer instalações físicas em condições adequadas de habitabilidade.
(B) Comunicar ao Ministério Público, para as providências cabíveis, a situação de abandono moral ou
material por parte dos familiares.
(C) Fornecer vestuário adequado, se for privada, e assistência religiosa mesmo àqueles que não
desejarem e de acordo com suas crenças.
(D) Celebrar contrato escrito de prestação de serviço com o idoso, especificando o tipo de atendimento,
as obrigações da entidade e prestações decorrentes do contrato, com os respectivos preços, se for o
caso.

05. (MPE/GO – Promotor de Justiça- MPE/GO/2016) De acordo com o Estatuto do Idoso (Lei n.
10.471/03):
(A) O Ministério Público tem legitimidade para a promoção da tutela coletiva dos direitos de pessoas
com idade igual ou superior a sessenta anos, mas não poderá atuar na esfera individual de direitos dessa
parcela da população, uma vez que a senilidade não induz incapacidade para os atos da vida civil.
(B) O idoso, que necessite de alimentos, deverá acionar simultaneamente os filhos, cobrando de cada
qual, na medida de suas possibilidades.
(C) O Poder Judiciário, a requerimento do Ministério Público, poderá determinar medidas protetivas
em favor de idoso em situação de risco, tais como: requisição de tratamento de saúde, em regime
ambulatorial, hospitalar ou domiciliar; encaminhamento à família ou curador, mediante termo de
responsabilidade; abrigamento em entidade.
(D) O Poder Público tem responsabilidade residual e, no âmbito da assistência social, estará obrigado
a assegurar os direitos fundamentais de pessoa idosa, em caso de inexistência de parentes na linha reta
ou colateral até o 3º grau.

06. (Prefeitura de Xaxim/SC – Assistente Social – ASSCONPP/2015) Assinale a alternativa correta


a respeito do estatuto do idoso.
(A) Priorização do atendimento do idoso por sua própria família, em detrimento do atendimento asilar,
exceto dos que não a possuam ou careçam de condições de manutenção da própria sobrevivência.
(B) Atendimento preferencial imediato e individualizado junto aos órgãos públicos e privados
prestadores de serviços à população.
(C) Garantia de acesso à rede de serviços de saúde e de assistência social locais.
(D) Todas as alternativas estão corretas

07. (PC-SC - Delegado de Polícia - ACAFE) Analise as afirmações a seguir, identifique o que constitui
crime praticado contra o idoso e assinale a alternativa correta.
l Obstar o acesso de alguém a qualquer cargo público por motivo de idade.
ll Recusar, retardar ou dificultar atendimento ou deixar de prestar assistência à saúde, sem justa causa,
a pessoa com mais de 55 anos.
lll Deixar de cumprir, retardar ou frustrar, sem justo motivo, a execução de ordem judicial expedida nas
ações em que for parte ou interveniente pessoa com mais de 65 anos.
lV Reter o cartão magnético de conta bancária relativa a benefícios, proventos ou pensão de pessoa
com mais de 70 anos, bem como qualquer outro documento, com objetivo de assegurar recebimento ou
ressarcimento de dívida.
(A) Todas as afirmações estão corretas.
(B) Apenas II e III estão corretas.
(C) Apenas I, II e III estão corretas
(D) Apenas I, III e IV estão corretas.
(E) Apenas III e IV estão corretas.

08. (MPE/SC - Promotor de Justiça - MPE-SC) Analise os enunciados das questões abaixo e assinale
se ele é Certo ou Errado.

Apostila gerada especialmente para: Betânia Mueller 023.712.580-32


. 180
A Lei n. 10.741/03 (Estatuto do Idoso) possui tipo penal específico para punir tabelião que lavrar ato
notarial que envolva pessoa idosa sem discernimento de seus atos e sem a devida representação legal.
(A) Certo
(B) Errado

09. (PC-SP - Investigador de Polícia - VUNESP) Minerva, 45 anos de idade, é filha de Pomona, 62
anos de idade. Ambas vivem juntas. Quando Pomona veio a adoecer gravemente, Minerva a levou para
um hospital público e lá a abandonou sob os cuidados médicos do estabelecimento, não mais retornando
para buscá-la. Essa conduta de Minerva.
(A) é considerada um crime de preconceito punível pelo Estatuto do Idoso
(B) não é considerada como crime, uma vez que Pomona, embora abandonada, foi deixada sob
cuidados médicos.
(C) não é considerada crime, por se tratar de hospital público, que tem a obrigação legal de cuidar de
Pomona.
(D) seria considerada crime pelo Estatuto do Idoso apenas se Pomona fosse maior de 65 anos de
idade.
(E) é considerada um crime pelo Estatuto do Idoso.

10. (DPE/PB - Defensor Público – FCC) O Estatuto do Idoso define o idoso como aquele com idade
igual ou superior a
(A) 60 (sessenta) anos, garantindo a ele todos os direitos previstos no respectivo diploma legal.
(B) 65 (sessenta e cinco) anos, garantindo a ele todos os direitos previstos no respectivo diploma legal.
(C) 70 (setenta) anos, garantindo a ele todos os direitos previstos no respectivo diploma legal.
(D) 60 (sessenta) anos, mas estabelecendo idades e circunstâncias diferenciadas para o exercício
pleno de todos os direitos previstos no respectivo diploma legal.
(E) 65 (sessenta e cinco) anos, mas estabelecendo idades e circunstâncias diferenciadas para o
exercício pleno de todos os direitos previstos no respectivo diploma legal.

11. (DPE/AM - Defensor Público – FCC) O Estatuto do Idoso define a violência contra o idoso como
sendo
(A) o atentado contra a pessoa do idoso, nos termos da lei penal.
(B) a prática dos crimes contra a vida, de lesões corporais, de periclitação da vida e da saúde e contra
a liberdade individual do idoso.
(C) o crime que envolver violência doméstica e familiar contra o idoso.
(D) o atentado contra os direitos fundamentais do idoso.
(E) a ação ou omissão praticada em local público ou privado que lhe cause morte, dano ou sofrimento
físico ou psicológico.

12. (DPE/SC - Técnico Administrativo – FEPESE) Assinale a alternativa correta de acordo com o
Estatuto do Idoso.
(A) É dever exclusivo dos familiares prevenir as ameaças ou violação aos direitos do idoso.
(B) A proteção ao idoso deve ser restrita à preservação de sua saúde física e mental.
(C) Somente as pessoas físicas poderão ser responsabilizadas pela inobservância das normas de
prevenção e proteção ao idoso.
(D) O Estatuto do Idoso, por ser norma especial em razão da matéria, impede a aplicação de qualquer
outra legislação.
(E) Todo cidadão tem o dever de comunicar à autoridade competente qualquer forma de violação a
esta Lei que tenha testemunhado ou de que tenha conhecimento.

13. (TJ/RJ - Comissário da Infância e da Juventude – FCC) Segundo prevê o Estatuto do Idoso, é
obrigação da entidade de atendimento ao idoso
(A) comunicar ao juiz as situações de abandono moral ou material por parte dos familiares.
(B) celebrar contrato escrito ou verbal de prestação de serviço com o idoso.
(C) elaborar e remeter ao Ministério Público plano individual de atendimento para cada caso com vistas
à reintegração familiar.
(D) administrar os rendimentos financeiros de seus usuários.
(E) proporcionar cuidados à saúde, conforme a necessidade do idoso.

Apostila gerada especialmente para: Betânia Mueller 023.712.580-32


. 181
14. (MPE/TO: Promotor de Justiça – CESPE) Assinale a opção correta com referência ao Estatuto
do Idoso e ao que ele dispõe.
(A) Entre os direitos reconhecidos legalmente ao idoso no domínio de suas faculdades mentais inclui-
se o de ele optar pelo tratamento de saúde que julgar mais favorável.
(B) Apesar de exercer uma função protetiva em relação ao idoso, o referido estatuto não define um
sistema claro de defesa dos interesses da pessoa idosa na condição de pessoa humana.
(C) Todos os direitos reconhecidos ao idoso, incluída a gratuidade nos transportes coletivos urbanos
e semiurbanos, aplicam-se aos indivíduos que alcancem sessenta anos de idade ou mais.
(D) No tocante à defesa dos direitos dos idosos em juízo, guardam competência subsidiária em relação
ao MP e à OAB a União, os estados, o DF e os municípios.
(E) Associações particulares podem figurar em juízo na defesa dos interesses dos idosos, desde que
autorizadas por assembleia convocada para tal finalidade.

15. (Pref. Angra dos Reis/RJ - Enfermeiro – Diarista - FEC/UFF) De acordo com o Estatuto do Idoso,
é obrigação do Estado e da sociedade, assegurar à pessoa idosa a liberdade, o respeito e a dignidade,
como pessoa humana e sujeito de direitos civis, políticos, individuais e sociais, garantidos na Constituição
e nas leis. Nesse sentido, Assinale a alternativa que caracteriza o direito ao respeito:
(A) faculdade de buscar.
(B) preservação da autonomia.
(C) faculdade de buscar auxílio.
(D) a liberdade de crença e culto religioso.
(E) faculdade de buscar refúgio.

Respostas

01. Resposta: D
Art. 23. A participação dos idosos em atividades culturais e de lazer será proporcionada mediante
descontos de pelo menos 50% (cinquenta por cento) nos ingressos para eventos artísticos, culturais,
esportivos e de lazer, bem como o acesso preferencial aos respectivos locais.

02. Resposta: D
Art. 34. Aos idosos, a partir de 65 (sessenta e cinco) anos, que não possuam meios para prover sua
subsistência, nem de tê-la provida por sua família, é assegurado o benefício mensal de 1 (um) salário-
mínimo, nos termos da Lei Orgânica da Assistência Social – Loas.)
Parágrafo único. O benefício já concedido a qualquer membro da família nos termos do caput não será
computado para os fins do cálculo da renda familiar per capita a que se refere a Loas.

03. Resposta: B
Art. 28. O Poder Público criará e estimulará programas de:
I – profissionalização especializada para os idosos, aproveitando seus potenciais e habilidades para
atividades regulares e remuneradas;
II – preparação dos trabalhadores para a aposentadoria, com antecedência mínima de 1 (um) ano, por
meio de estímulo a novos projetos sociais, conforme seus interesses, e de esclarecimento sobre os
direitos sociais e de cidadania;
III – estímulo às empresas privadas para admissão de idosos ao trabalho.

04. Resposta: C
Lei 10741/2013
Art. 50. Constituem obrigações das entidades de atendimento:
( )
III – fornecer vestuário adequado, se for pública, e alimentação suficiente;

05. Resposta: C.
Art. 45. Verificada qualquer das hipóteses previstas no art. 43, o Ministério Público ou o Poder
Judiciário, a requerimento daquele, poderá determinar, dentre outras, as seguintes medidas:
I – encaminhamento à família ou curador, mediante termo de responsabilidade;
II – orientação, apoio e acompanhamento temporários;
III – requisição para tratamento de sua saúde, em regime ambulatorial, hospitalar ou domiciliar;

Apostila gerada especialmente para: Betânia Mueller 023.712.580-32


. 182
IV – inclusão em programa oficial ou comunitário de auxílio, orientação e tratamento a usuários
dependentes de drogas lícitas ou ilícitas, ao próprio idoso ou à pessoa de sua convivência que lhe cause
perturbação;
V – abrigo em entidade;
VI – abrigo temporário.

06. Resposta: D
a) Art. 3º, V – priorização do atendimento do idoso por sua própria família, em detrimento do
atendimento asilar, exceto dos que não a possuam ou careçam de condições de manutenção da própria
sobrevivência; b) Art. 3º, I – atendimento preferencial imediato e individualizado junto aos órgãos públicos
e privados prestadores de serviços à população; c) Art. 3º, VIII – garantia de acesso à rede de serviços
de saúde e de assistência social locais.

07. Resposta: D
Está questão cobra para sua resolução o conhecimento dos limites de idade previstos no Estatuto do
Idoso. Assim, a afirmativa I está correta, haja vista que de acordo com o tipificado no artigo 100, I, da Lei.
A alternativa III está correta, pois se enquadra no previsto pelo artigo 100, IV, da norma; importante aqui
destacar que embora o dispositivo legal mencionado não apresente expressamente idade, sabemos que
quando aparece a palavra “idoso” podemos presumir que se fala de pessoa com idade igual ou superior
a 60 (sessenta) anos. A afirmativa IV está correta, posto que se enquadra no previsto no art.104, da
norma. Por fim, apenas a afirmativa II está incorreta, tendo em vista que nem todos que possuem mais
de 55 (cinquenta e cinco) anos, possuem 60 (sessenta) anos ou mais e, por essa razão, nem todos serão
idosos, sendo incorreto afirmar que a proteção do Estatuto igualmente se aplica estes.

08. Resposta: A
O enunciado da questão está correto, uma vez que de acordo com o que dispõe o artigo 108 do
Estatuto do idoso, vejamos: “Lavrar ato notarial que envolva pessoa idosa sem discernimento de seus
atos, sem a devida representação legal: Pena – reclusão de 2 (dois) a 4 (quatro) anos”.

09. Resposta: E
É o que dispõe o artigo 98 da norma: “Abandonar o idoso em hospitais, casas de saúde, entidades de
longa permanência, ou congêneres, ou não prover suas necessidades básicas, quando obrigado por lei
ou mandado: Pena – detenção de 6 (seis) meses a 3 (três) anos e multa”.

10. Resposta: D
A resposta correta é a alternativa “D”, pois está de acordo com o artigo 1º, do Estatuto do Idoso:
Art. 1º É instituído o Estatuto do Idoso, destinado a regular os direitos assegurados às pessoas com
idade igual ou superior a 60 (sessenta) anos.
O Estatuto do Idoso é destinado a regular os direitos assegurados às pessoas, considerando-se a
idade cronológica igual ou superior a 60 anos e de dispor de seus direitos fundamentais e
de cidadania, bem como a assistência judiciária. Além de preocupar-se com a execução dos direitos pelas
entidades de atendimento que o promovem, também volta-se para sua vigilância e defesa, por meio de
instituições públicas. Tendo como seu principal objetivo, assegurar os direitos da pessoa idosa.

11. Resposta: E
De acordo com o artigo 19, §1º do Estatuto do Idoso, considera-se violência contra o idoso:
Art. 19. (...)
§ 1º Para os efeitos desta Lei, considera-se violência contra o idoso qualquer ação ou omissão
praticada em local público ou privado que lhe cause morte, dano ou sofrimento físico ou psicológico.
(...)

12. Resposta: E
De acordo com o artigo 6º do Estatuto do Idoso:
Art. 6º Todo cidadão tem o dever de comunicar à autoridade competente qualquer forma de violação
a esta Lei que tenha testemunhado ou de que tenha conhecimento.

13. Resposta: E
De acordo com o artigo 50, inciso VIII, do Estatuto do Idoso:
Art. 50. Constituem obrigações das entidades de atendimento:

Apostila gerada especialmente para: Betânia Mueller 023.712.580-32


. 183
(...)
VIII – proporcionar cuidados à saúde, conforme a necessidade do idoso;
(...)

14. Resposta: A
De acordo com o artigo 17, “caput”, do Estatuto do Idoso:
Art. 17. Ao idoso que esteja no domínio de suas faculdades mentais é assegurado o direito de optar
pelo tratamento de saúde que lhe for reputado mais favorável.
(...)

15. Resposta: B.
Art. 10. É obrigação do Estado e da sociedade, assegurar à pessoa idosa a liberdade, o respeito e a
dignidade, como pessoa humana e sujeito de direitos civis, políticos, individuais e sociais, garantidos na
Constituição e nas leis.
§ 2º O direito ao respeito consiste na inviolabilidade da integridade física, psíquica e moral, abrangendo
a preservação da imagem, da identidade, da autonomia, de valores, ideias e crenças, dos espaços e dos
objetos pessoais.

LEI Nº 8.842, DE 4 DE JANEIRO DE 199412.

Dispõe sobre a política nacional do idoso, cria o Conselho Nacional do Idoso e dá outras providências.

O PRESIDENTE DA REPÚBLICA Faço saber que o Congresso Nacional decreta e eu sanciono a


seguinte lei:

CAPÍTULO I
Da Finalidade

Art. 1º A política nacional do idoso tem por objetivo assegurar os direitos sociais do idoso, criando
condições para promover sua autonomia, integração e participação efetiva na sociedade.

Art. 2º Considera-se idoso, para os efeitos desta lei, a pessoa maior de sessenta anos de idade.

CAPÍTULO II
Dos Princípios e das Diretrizes
SEÇÃO I
Dos Princípios

Art. 3° A política nacional do idoso reger-se-á pelos seguintes princípios:


I - a família, a sociedade e o estado têm o dever de assegurar ao idoso todos os direitos da cidadania,
garantindo sua participação na comunidade, defendendo sua dignidade, bem-estar e o direito à vida;
II - o processo de envelhecimento diz respeito à sociedade em geral, devendo ser objeto de
conhecimento e informação para todos;
III - o idoso não deve sofrer discriminação de qualquer natureza;
IV - o idoso deve ser o principal agente e o destinatário das transformações a serem efetivadas através
desta política;
V - as diferenças econômicas, sociais, regionais e, particularmente, as contradições entre o meio rural
e o urbano do Brasil deverão ser observadas pelos poderes públicos e pela sociedade em geral, na
aplicação desta lei.
SEÇÃO II
Das Diretrizes

Art. 4º Constituem diretrizes da política nacional do idoso:


I - viabilização de formas alternativas de participação, ocupação e convívio do idoso, que proporcionem
sua integração às demais gerações;
II - participação do idoso, através de suas organizações representativas, na formulação,
implementação e avaliação das políticas, planos, programas e projetos a serem desenvolvidos;

12
http://www.planalto.gov.br/ccivil_03/leis/L8842.htm. Acesso em 11/10/2018 às 10:43

Apostila gerada especialmente para: Betânia Mueller 023.712.580-32


. 184
III - priorização do atendimento ao idoso através de suas próprias famílias, em detrimento do
atendimento asilar, à exceção dos idosos que não possuam condições que garantam sua própria
sobrevivência;
IV - descentralização político-administrativa;
V - capacitação e reciclagem dos recursos humanos nas áreas de geriatria e gerontologia e na
prestação de serviços;
VI - implementação de sistema de informações que permita a divulgação da política, dos serviços
oferecidos, dos planos, programas e projetos em cada nível de governo;
VII - estabelecimento de mecanismos que favoreçam a divulgação de informações de caráter educativo
sobre os aspectos biopsicossociais do envelhecimento;
VIII - priorização do atendimento ao idoso em órgãos públicos e privados prestadores de serviços,
quando desabrigados e sem família;
IX - apoio a estudos e pesquisas sobre as questões relativas ao envelhecimento.
Parágrafo único. É vedada a permanência de portadores de doenças que necessitem de assistência
médica ou de enfermagem permanente em instituições asilares de caráter social.

CAPÍTULO III
Da Organização e Gestão

Art. 5º Competirá ao órgão ministerial responsável pela assistência e promoção social a coordenação
geral da política nacional do idoso, com a participação dos conselhos nacionais, estaduais, do Distrito
Federal e municipais do idoso.

Art. 6º Os conselhos nacional, estaduais, do Distrito Federal e municipais do idoso serão órgãos
permanentes, paritários e deliberativos, compostos por igual número de representantes dos órgãos e
entidades públicas e de organizações representativas da sociedade civil ligadas à área.

Art. 7o Compete aos Conselhos de que trata o art. 6o desta Lei a supervisão, o acompanhamento, a
fiscalização e a avaliação da política nacional do idoso, no âmbito das respectivas instâncias político-
administrativas. (Redação dada pelo Lei nº 10.741, de 2003)

Art. 8º À União, por intermédio do ministério responsável pela assistência e promoção social, compete:
I - coordenar as ações relativas à política nacional do idoso;
II - participar na formulação, acompanhamento e avaliação da política nacional do idoso;
III - promover as articulações intraministeriais e interministeriais necessárias à implementação da
política nacional do idoso;
IV - (Vetado;)
V - elaborar a proposta orçamentária no âmbito da promoção e assistência social e submetê-la ao
Conselho Nacional do Idoso.
Parágrafo único. Os ministérios das áreas de saúde, educação, trabalho, previdência social, cultura,
esporte e lazer devem elaborar proposta orçamentária, no âmbito de suas competências, visando ao
financiamento de programas nacionais compatíveis com a política nacional do idoso.

Art. 9º (Vetado.)
Parágrafo único. (Vetado.)

CAPÍTULO IV
Das Ações Governamentais

Art. 10. Na implementação da política nacional do idoso, são competências dos órgãos e entidades
públicos:
I - na área de promoção e assistência social:
a) prestar serviços e desenvolver ações voltadas para o atendimento das necessidades básicas do
idoso, mediante a participação das famílias, da sociedade e de entidades governamentais e não-
governamentais.
b) estimular a criação de incentivos e de alternativas de atendimento ao idoso, como centros de
convivência, centros de cuidados diurnos, casas-lares, oficinas abrigadas de trabalho, atendimentos
domiciliares e outros;
c) promover simpósios, seminários e encontros específicos;

Apostila gerada especialmente para: Betânia Mueller 023.712.580-32


. 185
d) planejar, coordenar, supervisionar e financiar estudos, levantamentos, pesquisas e publicações
sobre a situação social do idoso;
e) promover a capacitação de recursos para atendimento ao idoso;
II - na área de saúde:
a) garantir ao idoso a assistência à saúde, nos diversos níveis de atendimento do Sistema Único de
Saúde;
b) prevenir, promover, proteger e recuperar a saúde do idoso, mediante programas e medidas
profiláticas;
c) adotar e aplicar normas de funcionamento às instituições geriátricas e similares, com fiscalização
pelos gestores do Sistema Único de Saúde;
d) elaborar normas de serviços geriátricos hospitalares;
e) desenvolver formas de cooperação entre as Secretarias de Saúde dos Estados, do Distrito Federal,
e dos Municípios e entre os Centros de Referência em Geriatria e Gerontologia para treinamento de
equipes interprofissionais;
f) incluir a Geriatria como especialidade clínica, para efeito de concursos públicos federais, estaduais,
do Distrito Federal e municipais;
g) realizar estudos para detectar o caráter epidemiológico de determinadas doenças do idoso, com
vistas a prevenção, tratamento e reabilitação; e
h) criar serviços alternativos de saúde para o idoso;
III - na área de educação:
a) adequar currículos, metodologias e material didático aos programas educacionais destinados ao
idoso;
b) inserir nos currículos mínimos, nos diversos níveis do ensino formal, conteúdos voltados para o
processo de envelhecimento, de forma a eliminar preconceitos e a produzir conhecimentos sobre o
assunto;
c) incluir a Gerontologia e a Geriatria como disciplinas curriculares nos cursos superiores;
d) desenvolver programas educativos, especialmente nos meios de comunicação, a fim de informar a
população sobre o processo de envelhecimento;
e) desenvolver programas que adotem modalidades de ensino à distância, adequados às condições
do idoso;
f) apoiar a criação de universidade aberta para a terceira idade, como meio de universalizar o acesso
às diferentes formas do saber;
IV - na área de trabalho e previdência social:
a) garantir mecanismos que impeçam a discriminação do idoso quanto a sua participação no mercado
de trabalho, no setor público e privado;
b) priorizar o atendimento do idoso nos benefícios previdenciários;
c) criar e estimular a manutenção de programas de preparação para aposentadoria nos setores
público e privado com antecedência mínima de dois anos antes do afastamento;
V - na área de habitação e urbanismo:
a) destinar, nos programas habitacionais, unidades em regime de comodato ao idoso, na modalidade
de casas-lares;
b) incluir nos programas de assistência ao idoso formas de melhoria de condições de habitabilidade e
adaptação de moradia, considerando seu estado físico e sua independência de locomoção;
c) elaborar critérios que garantam o acesso da pessoa idosa à habitação popular;
d) diminuir barreiras arquitetônicas e urbanas;
VI - na área de justiça:
a) promover e defender os direitos da pessoa idosa;
b) zelar pela aplicação das normas sobre o idoso determinando ações para evitar abusos e lesões a
seus direitos;
VII - na área de cultura, esporte e lazer:
a) garantir ao idoso a participação no processo de produção, reelaboração e fruição dos bens culturais;
b) propiciar ao idoso o acesso aos locais e eventos culturais, mediante preços reduzidos, em âmbito
nacional;
c) incentivar os movimentos de idosos a desenvolver atividades culturais;
d) valorizar o registro da memória e a transmissão de informações e habilidades do idoso aos mais
jovens, como meio de garantir a continuidade e a identidade cultural;
e) incentivar e criar programas de lazer, esporte e atividades físicas que proporcionem a melhoria da
qualidade de vida do idoso e estimulem sua participação na comunidade.

Apostila gerada especialmente para: Betânia Mueller 023.712.580-32


. 186
§ 1º É assegurado ao idoso o direito de dispor de seus bens, proventos, pensões e benefícios, salvo
nos casos de incapacidade judicialmente comprovada.
§ 2º Nos casos de comprovada incapacidade do idoso para gerir seus bens, ser-lhe-á nomeado
Curador especial em juízo.
§ 3º Todo cidadão tem o dever de denunciar à autoridade competente qualquer forma de negligência
ou desrespeito ao idoso.

CAPÍTULO V
Do Conselho Nacional

Art. 11. (Vetado.)


Art. 12. (Vetado.)
Art. 13. (Vetado.)
Art. 14. (Vetado.)
Art. 15. (Vetado.)
Art. 16. (Vetado.)
Art. 17. (Vetado.)
Art. 18. (Vetado.)

CAPÍTULO VI
Das Disposições Gerais

Art. 19. Os recursos financeiros necessários à implantação das ações afetas às áreas de competência
dos governos federal, estaduais, do Distrito Federal e municipais serão consignados em seus respectivos
orçamentos.

Art. 20. O Poder Executivo regulamentará esta lei no prazo de sessenta dias, a partir da data de sua
publicação.

Art. 21. Esta lei entra em vigor na data de sua publicação.

Art. 22. Revogam-se as disposições em contrário.

Brasília, 4 de janeiro de 1994, 173º da Independência e 106º da República.

ITAMAR FRANCO

Questões

01. (Pref. de Fortaleza/CE - Diversas Especialidades - 2018) De acordo com a Lei nº 8.842/1994,
constitui diretriz da Política Nacional do Idoso:
(A) priorização do atendimento asilar, em detrimento do atendimento ao idoso através de suas famílias.
(B) permanência de portadores de doenças que necessitem de assistência médica permanente em
instituições asilares de caráter social.
(C) igualdade na aplicação da política nacional do idoso pelos poderes públicos, independentemente
das contradições entre o meio urbano e rural.
(D) capacitação e reciclagem dos recursos humanos nas áreas de geriatria e gerontologia e na
prestação de serviços.

02. (UFRPE - Assistente Social - SUGEP/2018) Conforme disposto na Lei nº 8.842/94, faz parte dos
princípios da política nacional do idoso:
(A) o idoso não deve sofrer discriminação de qualquer natureza.
(B) o idoso carente deve ter a garantia de um salário mínimo para assegurar sua sobrevivência.
(C) o idoso deve participar através de suas organizações representativas, na formulação,
implementação e avaliação das políticas, planos, programas e projetos a serem desenvolvidos.
(D) a família deve ser capacitada para os cuidados necessários com os idosos portadores de doenças
degenerativas e incapacitantes.
(E) o apoio a estudos e pesquisas sobre as questões relativas ao envelhecimento.

Apostila gerada especialmente para: Betânia Mueller 023.712.580-32


. 187
03. (CNMP - Analista do CNMP - FCC). A política nacional do idoso reger-se-á pelos seguintes
princípios:
(A) O processo de envelhecimento diz respeito à sociedade em geral, devendo ser objeto de
conhecimento e informação para todos.
(B) Viabilização de formas alternativas de participação, ocupação e convívio do idoso, que
proporcionem sua integração às demais gerações.
(C) Participação do idoso, através de suas organizações representativas, na formulação,
implementação e avaliação de políticas, planos, programas e projetos a serem desenvolvidos.
(D) Priorização do atendimento ao idoso através de suas próprias famílias, em detrimento do
atendimento asilar, à exceção dos idosos que não possuam condições que garantam sua própria
sobrevivência.
(E) Descentralização político-administrativa.

04. (MPE/AP - Analista Ministerial - FCC). A Política Nacional do Idoso instituída trouxe um arcabouço
legal para
(A) estabelecer que a porta de entrada do atendimento ao idoso, em geral, é a política pública de
assistência social por sua situação de vulnerabilidade em decorrência do ciclo de vida.
(B) responsabilizar unicamente a União pelo financiamento dos programas destinados aos idosos,
considerando que tais programas são de alta complexidade e estão sob a responsabilidade, execução e
supervisão das outras esferas de governo.
(C) assegurar os direitos sociais do idoso criando condições para promover sua autonomia, integração
e participação efetiva na sociedade.
(D) estimular a criação de alternativas de atendimento ao idoso que necessite ser afastado da
convivência, nas entidades não governamentais de longa permanência, em detrimento do serviço de
acolhimento institucional público.
(E) definir que a saúde e a previdência social são as políticas públicas prioritárias que devem prestar
atendimento a esse segmento populacional.

Gabarito

01.D / 02.A / 03.A / 04.C

Comentários

01. Resposta: D
Lei 8.842/1994
Art. 4º Constituem diretrizes da política nacional do idoso:
[ ]
V - capacitação e reciclagem dos recursos humanos nas áreas de geriatria e gerontologia e na
prestação de serviços;

02. Resposta: A
Art. 3° A política nacional do idoso reger-se-á pelos seguintes princípios:
[ ]
III - o idoso não deve sofrer discriminação de qualquer natureza;

03. Resposta: A
Lei 8.842/1994
Art. 3° A política nacional do idoso reger-se-á pelos seguintes princípios:
( )
II - o processo de envelhecimento diz respeito à sociedade em geral, devendo ser objeto de
conhecimento e informação para todos.
04. Resposta: C
Lei 8.842/1994
Art. 1º A política nacional do idoso tem por objetivo assegurar os direitos sociais do idoso, criando
condições para promover sua autonomia, integração e participação efetiva na sociedade.

Apostila gerada especialmente para: Betânia Mueller 023.712.580-32


. 188
Violência intrafamiliar: conceito, diagnóstico e intervenção.

CONSTRUÇÃO SOCIAL DA VIOLÊNCIA E A NEGAÇÃO DA CIVILIDADE13

A violência assim como a política, institui seus sentidos através da relação entre os homens. Entretanto
ela é o avesso da política, pois em sua instituição ela nega o diálogo e a palavra. Paradoxalmente, a
violência é construída na sociedade e se processa de forma a romper com os próprios códigos de ordens
produzidos por ela. A violência emerge como uma agressão ao indivíduo e ao coletivo.

A violência, e as representações no contexto contemporâneo, constituem-se num dos maiores


problemas sociais que afeta a vida em sociedade, e exerce efeitos negativos sobre a convivência social
e a funcionalidade das instituições. “A violência ocupa um lugar central na luta pela posse do presente,
pela compreensão da experiência contemporânea, com seus mundos marginais, a sua dimensão
episódica e fragmentária, um tempo marcado pela diferença, pela falta de finalidade das formações
sociais”. O crescimento da violência tem mobilizado discussões no meio acadêmico e nos diferentes
âmbitos da sociedade civil, colocando o tema na prioridade da agenda pública das políticas
governamentais e no âmbito das pesquisas sociais. A manifestação do fenômeno no contexto
contemporâneo é vista como um mal que atinge a vida social e que deve ser combatido, pois agride a
condição humana e os direitos civis e sociais, não se restringindo apenas às relações de agressividade e
incivilidade entre grupos e indivíduos, mas também a conflitos nas formas de controle que as instituições
sociais exercem ou deviam exercer para manter a autoridade e a norma social. Como fenômeno
pluricausal, a compreensão da violência demanda interfaces com outros temas complexos, como os
direitos da cidadania, o exercício dos direitos humanos e as condições de exercício da justiça social, da
segurança pública e do Estado.

A definição de violência é ampla e polissêmica. A palavra violentia é oriunda do século XIII, mesmo
sabendo que ela enquanto ação existe antes disto, foi derivada do latim vis, designando a “força” ou o
“vigor”, e seu uso pode definir uma relação de força, submissão e constrangimento a outrem, se
expressando de forma material ou imaterial.

Violência vem do latim violentia, que significa violência, caráter violento ou bravio, força. O verbo
violare significa tratar com violência, profanar, transgredir. Tais termos devem ser referidos a vis, que quer
dizer força, vigor, potência violência, emprego de força física, mas também quantidade, abundância,
essência ou caráter essencial de alguma coisa.

Portanto, parte da complexidade da análise da violência se origina da própria dificuldade de


compreender o comportamento humano. As primeiras tentativas conceituais, como analisa Robert
Muchembled, afirmavam a violência como elemento intrínseco à própria existência humana, entendendo
que todos os seres vivos são movidos por comportamentos predatórios e instintivos de defesa, quando
são ameaçados. Neste sentido, o uso da violência seria uma forma não de aniquilar a vida, mas de
garantir a conservação da existência, e uma resposta ao medo, à frustração, à inveja, vingança ou perda
de esperança. Para Erich Fromm, há também uma forma de violência definida como compensatória,
praticada por indivíduos acometidos de impotência, e que desejam reverter a sua fraqueza em força
através da ação violenta. Segundo este autor, o homem é o único primata capaz de matar e torturar
membros de sua espécie sem nenhuma razão, por puro prazer, e tal comportamento faz da violência um
fenômeno intrínseco à condição humana.
Para Hannah Arendt, a condição humana se refere às formas de vida que o homem impõe a si mesmo
para sobreviver, exercendo ações que buscam suprir as necessidades de sua existência e que variam no
espaço e no tempo. As ações e comportamentos humanos estão condicionados por contextos sócio-
históricos. Segundo Arendt, “até mesmo aqueles que condicionam o comportamento de outros homens
tornam-se condicionados pelo próprio movimento de condicionar”.
Na condição humana, instituída no estado de natureza, “o homem era lobo do homem”, ele já nascia
com o instinto de promover o mal que era acelerado pelo desejo de competição, pela vontade da posse,
domínio e poder. Assim, violência era uma ação de preservação da existência. Segundo Thomas Hobbes,

13
Soares, A. M de C. Construção social da violência e a negação da civilidade. Latitude, Vol. 8, 2014.

Apostila gerada especialmente para: Betânia Mueller 023.712.580-32


. 189
neste estágio da natureza o homem seria naturalmente egoísta e predisposto à violência, em um ambiente
marcado pela luta de todos contra todos, pois no “estado de natureza” não havia qualquer direção ou
mediação de governo, estando os indivíduos incitados à violência e à selvageria.
A natureza deu a cada um o direito a tudo; isso quer dizer que, num estado puramente natural, ou
seja, antes que os homens se comprometessem por meio de convenções ou obrigações, era lícito cada
um fazer o que quisesse, e contra quem julgasse cabível e portanto possuir, usar e desfrutar tudo que
quisesse ou pudesse obter. Ora, como basta um homem querer uma coisa qualquer para que ela já lhe
pareça boa, e o fato dele a desejar já indica que ela contribui, ou pelo menos lhe parece contribuir, para
sua conservação [...] De tudo isso então decorre que, no estado de natureza, para todos é legal ter tudo
e tudo cometer. E é este o significado daquele dito comum, ‘a natureza deu tudo a todos’, do qual,
portanto, entendemos que, no estado de natureza, a medida do direito está na vantagem que for obtida.
Discordando de Hobbes, Jean Jacques Rousseau afirma que “o homem é bom e a sociedade o
corrompe”. Para este autor, no estado da natureza o homem era bom, vivia em harmonia com os demais,
não havia estado de guerra. Foi, portanto, o surgimento da propriedade privada que instituiu os conflitos
de interesses distintos, gerando a necessidade do estabelecimento de um contrato entre os homens. Para
John Locke, o homem não era bom nem mal, seu temperamento oscilava entre a racionalidade e a
ponderação e a irracionalidade e a agressividade. Para Locke, o que faltava, no estado da natureza, eram
princípios de racionalidade para julgar as controvérsias advindas da irracionalidade do homem.
As dialéticas apontadas pelos teóricos do contratualismo trazem reflexões sobre o comportamento do
homem no estado da natureza, caracterizado pela ausência da lei soberana e tendo a ação violenta como
medida de autopreservação do homem e de sua propriedade instituída como barbárie. A condição
humana estaria assegurada dentro de princípios em que a ação de uma pessoa seria limitada pela força
da outra, em um movimento legítimo de autopreservação, em que não existem regras, normas, direitos e
deveres estabelecidos, constituindo um ambiente em que, na ausência de processos e valores
socializadores, o uso da força violenta expressava processos de incivilidade manifestos através de
desrespeitos, desordens, agressões físicas e verbais, homicídios, entre outros tipos de comportamento
que fragilizam as relações humanas.
Neste contexto de conceber a violência como uma ação intrínseca à condição humana, este artigo
afirma que dialeticamente a violência emana do homem e atinge o próprio homem, tanto objetivamente
como subjetivamente. No esboço traçado para esta discussão, subdividimos este artigo em três vertentes:
o primeiro item dimensiona os efeitos que a violência produz para a condição humana e suas associações
com a noção de civilidade e incivilidade; o segundo como já referenciando demonstra as dialéticas e
polissemias em torno da violência como categoria social dinâmica e complexa; e, por fim, o terceiro item
recupera a amplitude das convergências e divergências entre os conceitos de violência e crime, suas
classificações e representações com base em literatura referenciada.

Violência e as intercalações entre civilidade e incivilidade

A criação de mediações que limitassem o ímpeto do homem no estado da natureza induziu à formação
de um contrato social como instrumento regulatório dos interesses distintos entre indivíduos, que, para
Rousseau, representa um pacto de direito político em que prevaleceria a soberania da sociedade e a
vontade coletiva representada pelo Estado Civil, distante do interesse singular e organizado para garantir
o interesse de todos. Tal evolução levaria o comportamento humano a substituir a conduta do “instinto”
por condutas de justiça, que recusa a violência no convívio humano. A formação desse pacto social
implicou a organização da sociedade civil como uma arena de ações coletivas, organizadas por
instituições criadas em torno de interesses e propósitos coletivos. A sociedade civil passaria a sistematizar
e fiscalizar as formas contratuais de governo, baseada na regra da lei, através da demarcação e de
regulamentações para a organização social, em que os direitos e deveres se estabeleceriam como
diretrizes para o controle e o ordenamento social.
O controle do Estado e a organização da sociedade civil exerceriam um pacto de sujeição do indivíduo
a soberania estatal, não permitindo a violência “natural”. O Estado Civil seria a instituição responsável
pelo controle da violência, e só ele, em nome do bem-estar coletivo, poderia exercer a violência legítima.
No pensamento liberal, o contrato estatal implicaria o surgimento simultâneo da sociedade política e da
sociedade civil. Para Gramsci, o conceito de sociedade civil é inseparável da luta entre as classes sociais,
e sua construção integra sua mais densa reflexão sobre o “Estado ampliado”. Na concepção gramsciana,
não há oposição entre sociedade civil e Estado; há uma relação de extensão e ajustamentos.
Conforme Gramsci, a sociedade política se elabora no seu sentido estrito. Através da coerção estatal
ela é produzida por um conjunto de aparelhos nos quais a classe dominante detém e exerce o “monopólio
legal” – aparelhos coercitivos ligados às forças armadas e à polícia. Corresponde a uma dimensão

Apostila gerada especialmente para: Betânia Mueller 023.712.580-32


. 190
marcada pela coerção, pela imposição, pelo uso da força e da violência, quando necessária. Os governos
modernos que ocupam a esfera estatal e se sustentam no sufrágio universal usam dessa hegemonia para
impor o “monopólio legal” frente à sociedade. Segundo a concepção gramsciana, a sociedade civil é
formada por um conjunto de instituições responsáveis pela representação de diferentes tipos de
interesses (privados, de grupos e ou segmentos particulares comprometidos com a manutenção ou a
ruptura da ordem). Esses segmentos e grupos sociais, apoiados em interesses específicos de classe, se
encontram comprometidos com a difusão de suas ideologias (partidos, igrejas, escolas, entre outras
instituições).
A noção de “Estado ampliado”, em Gramsci, é composta pela relação entre a sociedade política e a
sociedade civil, ou seja, é formada pela hegemonia conquistada – ao mesmo tempo – pela coerção e pela
conformidade. É um composto de correlações de forças, que pode servir para manter ou transformar as
relações sociais de acordo com os interesses da classe hegemônica. As classes sociais lutam por
hegemonia, na conquista de aliados para seus projetos, almejando o consenso e uma direção comum.
Esse movimento se expressa nas esferas do Estado, na forma como os três poderes: Executivo,
Legislativo e Judiciário se comportam face às lutas sociais que ocorrem fora da esfera estatal –
movimentos e manifestações sociais, partidos políticos, sindicatos etc.
A organização do Estado e a complexidade da sociedade civil denotou um novo estágio, em que a
(des)civilidade, a força violenta, o comportamento e os impulsos humanos passam a ser disciplinados por
quadros normativos e contratos sociais. A organização civil estaria presente no governo, manifestaria
novas formas de controle guiadas por instituições e pela presença de um Estado, que arbitraria sobre a
sociedade. Para Norberto Bobbio, a formação da sociedade civil constitui o momento intermediário entre
o indivíduo e o Estado, as transformações sociais mobilizadas pela constituição da sociedade civil
indicariam novos horizontes para a vida social, no que se refere aos direitos individuais, às regras de
conduta, ao respeito à lei e às formas de controle de condutas desviantes.
É nesse sentido que os elementos e as instituições da sociedade civil assumem um papel tanto
representativo quanto ético, integrando e educando os indivíduos para a vida cívica, instituída por direitos
e deveres comuns a todos, aproximando cada vez mais os indivíduos do sentido da sociedade civil,
requerendo do indivíduo: [...] regularidade de comportamento, regras de conduta, respeito pela lei, e
controle da violência. Assim, uma sociedade civil era sinônimo de sociedade cortês, sociedade na qual
estranhos agem de uma maneira civilizada com relação aos outros, tratando cada um com respeito mútuo,
tolerância e confiança, uma sociedade na qual o debate racional e a discussão se tornam possíveis.
A organização da sociedade civil é orientada pelo respeito às normas de convívio, obediência às regras
de urbanidade e comportamentos marcados por formas de agir civilizadas e obedientes às
regulamentações de um Estado. Este se responsabiliza pelo controle da violência, sem, contudo, extingui-
la, tendo em vista que o controle social como uma força coercitiva constitui-se em violência legítima,
exercida pelo Estado e protegida pela lei. Ao instituir-se como agente do processo civilizatório, o Estado
civil apresentaria o reverso da civilidade, que é o processo de “descivilização”, compreendido como um
fenômeno que engloba o primeiro, em que o Estado, buscando exercer o seu controle, se utiliza da ação
violenta para manter a sua autoridade legítima.
O processo de civilização do Estado, a Constituição, a educação e, por conseguinte, os segmentos
mais numerosos da população, a eliminação de tudo o que era ainda bárbaro ou irracional nas condições
vigentes, fossem as penalidades legais, as restrições de classe à burguesia ou as barreiras que impediam
o desenvolvimento do comércio – este processo civilizador devia seguir-se ao refinamento de maneiras
e à pacificação interna do país pelos reis.
Para Robert Muchembled, “o poder destrutivo da violência pode ser inibido pelas civilizações que se
organizarem para impor a ordem”. Tanto Elias como Muchembled coincidem na tese do “processo
civilizatório”, ao afirmarem que o adestramento cultural e o controle social são indispensáveis para o
desenvolvimento de modos padronizados de conduta. A ordem social é vista por esses autores como
desmobilizadoras de conflitos entre os indivíduos, pois disciplina o comportamento e controla os impulsos
e desejos antissociais com base na moral e valores, promovendo o surgimento de espaços pacificadores
e civilizados, o que viria inibir os comportamentos desviantes e violentos. Para Elias (1997), grande parte
dos cidadãos não tolera os atos violentos como forma de resolver os conflitos sociais, entendendo que a
civilidade possibilita um maior esclarecimento dos cidadãos sobre as noções de direito, respeito e justiça,
assim como em relação à responsabilidade do Estado no controle da violência.
A passagem do “estado de natureza” para o “Estado civil”, mediada por regras de convívio e direitos,
foi determinante para alterações na vida social: evolução política, desenvolvimento das instituições
modernas, emergência dos direitos, enquanto detentor de valor jurídico universal. Esta passagem foi
marcada por processos distintos que evoluem desde a completa ausência de cortesia e solidariedade, na
qual inexistem questionamentos sobre a conduta brutalizada, até a civilidade (instituição de regras de

Apostila gerada especialmente para: Betânia Mueller 023.712.580-32


. 191
convívio e polidez nas relações sociais, com evolução dos códigos de ética e de respeito) e descivilização
(transgressão da norma instituída, ruptura dos direitos humanos, retorno à condição de violência para
combater a própria violência). Processos que alteraram os significados do ato violento, que deixam de
ser apreendidos como uma condição necessária à autopreservação do indivíduo em seu estado de
liberdade natural, como afirma Hobbes (2002), para expressar ações desumanizadoras, agressivas e com
múltiplas formas de representação social e política.

Violência: uma noção polissêmica

A violência, ancorada nas relações sociais e nas tramas intersubjetivas, é um conceito amplo, transitivo
e recorrente das circunstâncias individuais e sociais. Como reflexo do ambiente cultural e histórico, a
análise da violência está também associada a outros conceitos sociais da ordem, como o poder, a força
e a potência, que convergem para o entendimento de ações e de comportamentos humanos que desafiam
o institucionalizado, o moral e o ético. A associação destes conceitos com a finalidade de problematizar
o entendimento da violência. Nesta direção, é fundamental para a compreensão de poder, força e potência
o uso das noções conceituais referenciadas principalmente nos trabalhos de Arendt, Foucault e Agamben,
no que se refere ao conceito de violência e seus desdobramentos. As tentativas de interpretação
conceitual dessa noção são sempre marcadas pela diversidade de práticas e ações caracterizadas por
contextos específicos. Portanto, a violência é um fenômeno social e culturalmente constituído ao longo
do processo civilizatório de uma sociedade, não é recente e sua incidência progressiva na
contemporaneidade induz a questionamentos que buscam explicitar as formas concretas como se
manifesta o fenômeno.
A noção de violência é, portanto, relativa, plural e contingente às circunstâncias históricas de sua
manifestação; e é muitas vezes difusa, o que dificulta a sua definição. Manifestada historicamente em
contextos de incivilidade, descivilidade, agressividade e em tantas outras situações de constrangimento,
a violência atualmente aparece naturalizada na própria sociedade civil, quando os atos violentos não são
mais “exceções”, mas manifestações práticas do cotidiano das pessoas, que normalizam gradativamente
a ação violenta. Foucault analisa o processo de normalização da violência, não por ser oficial ou legal,
mas por ser utilizada como meio de manutenção de relações de poder na dinâmica social.
O processo de normalização incorpora distorcidamente a violência no âmbito das instituições sociais
como uma conduta corriqueira e “aceitável”, o que supõe que a apreensão do sentido da norma implícita
“às situações de violência” resulte das formas concretas como os indivíduos agem, em situações
específicas, explicitando regras de poder (implícitas ou explícitas), de forma a compreender o fenômeno
do ponto de vida das mudanças da ordem social e suas formas de reprodução.
Como uma prática contraditoriamente humana, a violência revela-se desqualificadora das formas de
racionalização da vida social. A violência resulta, consciente ou inconscientemente, de dispositivos sociais
e convenções sociais construídas, que criam necessidades materiais e imateriais, as quais, quando não
alcançadas, geram conflitos, tensões e frustrações que potencializam a reprodução da violência,
manifestando-se na forma de condutas desviantes que dilaceram a moral, os valores e corroem as
relações de sociabilidade.
Como manifestação e objetivação contraditória de construção/ desconstrução de relações sociais, o
fenômeno da violência é influenciado pela mediação de aparelhos culturais, econômicos e políticos que
atuam sobre a conduta humana e sobre a ordem social expressa em um campo de disputa, marcado por
“relações de poder que influenciam comportamentos e permeiam as interações entre os grupos e as
classes”. Ainda segundo este autor, numa sociedade caracterizada por campos de disputa, a violência é
percebida como um meio de adulterar a ordem vigente e produzir mecanismos de controle alternativos
para impor sua força que desagrega e agrega, desconstrói e constrói novas ordens e representações
sociais. E nesse jogo social, empodera ou vulnerabiliza indivíduos, fortalece ou dissolve grupos.
A violência, na perspectiva de Foucault, se produz na dinâmica dos jogos de poder e se mostra cada
vez mais difusa na sociedade, envolvendo cada vez mais um número maior de pessoas, estruturando-se
cada vez mais em terrenos incertos da vida social e transitando entre fronteiras do legal e ilegal, lícito e
ilícito, formal e informal, ampliando a gravidade de dramas sociais como a exclusão social, a segregação
urbana, a pobreza e a vulnerabilidade social.
Associada ao funcionamento das redes de poder, a violência mostra-se presente na construção da
própria sociedade e na construção das relações interpessoais, institucionais, entre as individuais e as
coletivas. A sociedade se estrutura com base em relações econômicas e relações de reprodução cultural
que, por sua vez, expressam relações de poder. Conforme indica Foucault, as relações de poder são
constitutivas da sociedade e estão presentes nas relações humanas, nas contradições de classes, de
sexo, de gênero, de etnia, nas interações simbólicas, culturais, institucionais, profissionais e afetivas. O

Apostila gerada especialmente para: Betânia Mueller 023.712.580-32


. 192
exercício do poder, para Foucault, consiste em “conduzir condutas”, e essas são, ao mesmo tempo, um
ato de “conduzir” os outros e a maneira de se comportar num campo social mais ou menos aberto, de
possibilidades.
O poder inerente às relações sociais é geralmente compreendido e utilizado como potência
mobilizadora da força e substrato da violência, quando se direciona a potencializar as diferenças que
hierarquizam e discriminam os homens, fortalecem os estigmas e ampliam as desigualdades sociais, e
dilui o espaço público como lugar do diálogo, aniquilando o entendimento entre os homens pelo uso da
palavra. O verdadeiro sentido do poder deve ser compreendido como possibilidade de reverso dessas
situações, quando concebido como instrumento político de empoderamento das minorias sociais,
ressignificador de práticas e restituidor da ação coletiva. Desse modo, a depender do uso e da
compreensão que se faz do poder, contém ao mesmo tempo a violência e a antiviolência, a construção e
a desconstrução de padrões de convívio na sociedade.
As relações que definem e separam poder, força e violência se estabelecem na própria execução do
comportamento humano e na interpretação que realizamos deste comportamento. Para Arendt, o que
define e separa a violência do poder é a dimensão da política, que está ausente em situações de violência
e presente em relações de poder. Para essa autora, assim como para Foucault (1998), a violência
distingue-se do poder, pois ela é utilizada em relações sociais desiguais e hierarquizadas, em que não há
liberdade, sendo compreendida como dispositivo de controle e subjugação.
O poder só é efetivado enquanto a palavra e o ato não se divorciam, quando as palavras não são
vazias e os atos não são brutais, quando as palavras não são empregadas para velar intenções, mas
para revelar realidades e os atos não são usados para violar e destruir, mas para criar relações e novas
realidades. (ARENDT, 2006, p. 212).
Na concepção arendtiana, o poder não subjuga, e quando é utilizado para este fim ele deixa de ser
poder e se transforma em potência, que não pode ser confundida com poder. Potência é a força de um
homem ou de um coletivo que pode se voltar contra o poder instituído. O poder pertence ao mundo
cotidiano das relações entre os indivíduos. Para Foucault (1998), só há relação de poder entre pessoas
livres. Entretanto, o autor reconhece que o poder pode ser confundido como potência e se transformar
em extensão da violência, quando é utilizado para obter a sujeição e a domesticação do outro, quando o
poder se transformar em biopoder (poder sobre a vida), em que as pessoas passam a ser governadas
para que se possa obter o máximo de suas energias vitais.
A violência é a instrumentalização da potência e da força com vista à sua ampliação, e a força é a
energia liberada, que pode ser utilizada para fortalecer ou não as relações sociais. A violência é conduzida
e imposta pelo uso da força, não só a força física, mas também a simbólica, sendo uma ação destrutiva
que ameaça a autoridade e o poder. Ao compreendermos a violência como uma transgressão dos
padrões de civilidade, mediada pela força, quanto maior a concentração e direcionamento do uso da
força, maior será a intensidade agressiva da conduta violenta. Neste sentido, Ives Michaud , em seu livro
La Violence, após explorar os mais diversos sentidos da violência, de forma similar a Hannah Arendt,
afirma que a violência pode ser definida como um emprego de “força desmedida”, algo que ultrapassa e
transgride a ordem estabelecida, instituindo a “força não qualificada”, que subverte a ordem e dissemina
o clamor através de múltiplas formas de agressão.
A violência, na perspectiva de Arendt, expressa a deterioração da política, com a ausência da palavra
e do discurso, expressões efetivas do exercício do poder como constitutivos sociais. A humanização das
pessoas opera através da “palavra e do discurso”, pois é através deles que o homem presume a sua
verdade. Segundo essa autora, o discurso se institui no âmbito da ação coletiva; sem esta ação o discurso
vira pura conversa que manipula, mas não comunica; sem a comunicação pelo discurso não há
transitividade. A violência aniquila a “palavra e o discurso”, ela não permite o falar, o ouvir, pautando-se
apenas no ato agressivo. Como movimento de mudança, a ação coletiva mediadora do diálogo destrava
o homem, rompe as amarras e possibilita a liberdade do pensamento, o desenvolvimento das
subjetividades e a mediação da pluralidade humana, em seu duplo aspecto de igualdade e diferença.
Para Arendt:
Se não fossem iguais, os homens seriam incapazes de compreender-se entre si, ou de fazer planos
para o futuro e prever as necessidades das gerações vindouras. Se não fossem diferentes, os homens
não precisariam do discurso ou ação para se fazer compreender.
O discurso e a ação coletiva estão imbricados e são manifestações singulares da vida humana, e
fazem do homem um ser político e constitutivo de um mundo comum e sem violência. Em diálogo com
alguns conceitos de Hannah Arendt, Michel Foucault e Giorgio Agamben, consideramos que o poder é
um elemento fundamental na vida do indivíduo e possui relação de aproximação e distanciamento com a
violência, ou seja, ele está presente na prática do ato violento como mobilizador da potência e da força,
e pode se fazer ausente quando associado à prática política, tendo em vista que a violência institui-se

Apostila gerada especialmente para: Betânia Mueller 023.712.580-32


. 193
através de vidas apolíticas. Na concepção de Agamben, o indivíduo que tem negada a sua existência
política encontra-se encerrado na existência biológica, diferenciado, excluído e violentando pelo não
acesso a uma vida digna. Neste contexto de ausência da política, o autor incorpora, na discussão da
violência, compreensões sobre o Estado, o poder soberano, este e sua força deliberativa no desenho dos
limites da inclusão e da exclusão do indivíduo na vida política. Agamben, ao analisar a estrutura da
soberania no mundo atual e sua correlação com a condição violenta do Estado, resgatou a noção de
Estado de exceção, que se define como um regime da lei no qual a norma vale, mas não se aplica (porque
não tem força), e atos que não possuem o valor de lei adquirem sua força. A força de lei flutua como um
elemento indeterminado que pode ser reivindicado ora pela autoridade do Estado, ora pela autoridade de
uma organização revolucionária.
Nos nexos estabelecidos entre Estado, violência e organizações revolucionárias, há espaços para se
refletir sobre as instituições formativas que necessitam assumir responsabilidades pelo mundo, na
perspectiva de Arendt, reafirmando sua vinculação social transformadora através de práticas que
estimulem a liberdade do pensamento e inibam ações desumanizadoras, assegurando a circulação da
palavra e sua autoridade enquanto organização revolucionária que deve promover a inclusão da vida na
política.
Ao refletir sobre o “Estado de exceção” e o poder soberano, Agamben apresenta considerações
relevantes na discussão sobre violência, tendo em vista que o “Estado de exceção” é desenhado como
uma relação presente na origem da política; tal relação trata da delimitação entre o que está incluso e
excluso da política, do direito e da normalidade. Assim, a exclusão da vida na política corresponde à
inclusão da vida incivilizada e violenta. O “Estado de exceção” mobiliza o poder de dar e retirar a condição
de ser político, não se configurando em um ambiente ditatorial, mas em um espaço vazio de direitos.
O poder soberano, no contexto de um “Estado de exceção”, tem o arbítrio de conduzir sobre a vida e
a morte, produz zonas de indiferença, confundindo violência e direito, cria espaços de indistinção dentro
do próprio Estado. Esses espaços, segundo Agamben, [...] dissolvem as determinações do direito, que
passam a ser suspensas, e nas lacunas deixadas pela ausência da lei e da regra, se produz a vida nua,
compreendida como a vida restrita à sua dimensão biológica, marcada como condição apolítica e
destituída de quaisquer direitos.
Na constatação de Agamben, a “vida nua” insurge em espaços de exceção, onde existe a plena
ausência de ação política. A “vida nua” se constitui como uma categoria criada para o livre exercício do
poder soberano sobre aqueles que subsistem em um mundo onde o direito não se efetiva. Podemos nos
referir aos milhares de seres viventes, num estágio de “vida nua” que habitam os cortiços, albergues,
Febens, “carandirus”, favelas, conjuntos habitacionais populares etc. Lugares onde a exceção, além de
regra, é paradoxalmente a única forma de inclusão. Os espaços sociais em que vidas são desqualificadas,
em que os corpos são violados e as pessoas são convertidas em corpos “matáveis”, teriam o “Estado de
exceção” como referência e paradigma.
O “Estado de exceção” não é, portanto, o caos que precede a ordem, mas a situação que resulta da
sua suspensão. Ele se faz presente em diversos espaços institucionais da sociedade contemporânea.
Nesse sentido, a exceção é, verdadeiramente, segundo o étimo, capturada fora (excapere) e não
simplesmente excluída. Para Agamben a exceção se encontra em uma extrema relação com algo que é
incluído através de sua exclusão, condição que se reproduz nas compreensões da violência e da política,
que são colocadas em contextos ambíguos de negação e limite, onde a condição de existir é a ausência
do outro. Entretanto, no jogo de inclusão e exclusão da violência ou da política, há situações em que o
poder soberano precisa exercer a violência para combater a violência e instituir a vida política.
Segundo Agamben, a violência não pode ser totalmente incluída ou excluída do poder soberano. O
Estado não pode abrir mão do uso da violência como instrumento de pacificação, tendo em vista a sua
condição de instituidor da civilidade, mediante a necessidade de destituir a descivilidade como forma de
manter o controle social. Desse modo, o poder soberano manifesta a necessidade de definir vidas
politicamente desqualificadas, “vida nua” (despolitizada), em que a violência precisa ser exercida para
reforçar o domínio, o poder e a força do Estado.
A violência cometida pelo Estado integra o processo civilizatório de manter o controle social e promover
a justiça e o bem-estar coletivo através de regulamentações que devem ser obedecidas mesmo a
contragosto de alguns. Toda a responsabilidade alocada no Estado, no que se refere à promoção da
coesão social, faz de sua violência uma ação legítima, e é funcionalmente diferenciada da violência
cometida por outras instituições e pelos indivíduos. Mesmo com função social clara e institucionalizada,
a violência promovida pelo Estado não perde a conotação de expressão de força que agride a condição
humana e desqualifica a política.

Apostila gerada especialmente para: Betânia Mueller 023.712.580-32


. 194
Classificar e distinguir - violências e crimes

Como categorias complexas que indicam fenômenos manifestos na sociedade, a “violência” e o “crime”
não podem ser assimilados em uma mesma matriz conceitual, nem compreendidos em definições
estanques. Como explicita Maria Stela Grossi Porto, com as constantes definições e redefinições do
espaço sociocultural, a própria nomenclatura da violência e do crime segue os contornos de uma
sociedade flexível e mutante. Para ser compreendida é necessário a percebermos como uma ação difusa,
como sugere José Vicente Tavares dos Santos, que propõem o esboço do que ele chama de “sociologia
da conflitualidade”, um paradigma explicativo que busca compreender as práticas sociais consideradas
violentas próprias da sociedade contemporânea – violência política, violência costumeira, violência de
gênero, e um conjunto de outras violências que para este autor, revelam dilemas da sedimentação de um
controle social, informal e formal que tem como fator desencadeante a violência difusa na sociedade
contemporânea.
O debate sobre a violência e a criminalidade mobilizou nos últimos anos uma série de estudos e
pesquisas nacionais e internacionais que visam buscar o entendimento deste fenômeno social, que
instaura um campo de análise problemático e conflitivo, envolvendo diferentes categorias sociológicas,
atores sociais e responsabilidades públicas. Condição que coloca o tema da violência como um problema
da ordem pública, no caso particular do Brasil, a violência encontra-se situada na história particular da
formação política do País. Sérgio Adorno defende que a violência no Brasil constituiu-se como um
fenômeno determinado sócio-historicamente e está associado à crise do sistema de segurança pública,
ao crescimento das injustiças sociais e ao esvaziamento dos direitos. Em uma perspectiva similar à de
Arendt, os trabalhos de autores brasileiros como Alba Zaluar e Vera Telles apontam que a violência
danifica o projeto humano, pois nega o uso da argumentação como possibilidade de entendimento e de
exercício político de sociabilidade.
Segundo Misse, a temática da “violência” e as referências ao “mundo do crime” emergiram, portanto,
numa série de estudos dos últimos anos, como questões vinculadas estreitamente aos deslocamentos
recentes operados nas esferas estruturantes da vida popular que, antes de mais nada, colocavam em
questão a promessa de contrapartida social do assalariamento industrial, “potencializados pela
globalização”. A violência e o crime como fenômenos sociais instituem uma “sociabilidade violenta” na
forma como formulada por Machado da Silva e Misse, que destacam que ela se manifesta na sociedade
brasileira e se insere nos diversos espaços da vida cotidiana como práticas agressivas e hostis que
anunciam novas formas de sociabilidades, marcadas pela insegurança e pelo medo. Para Ruth
Vasconcelos e Elaine Pimentel, a compreensão da violência se articula também com dimensões sócio
históricas, nos Estados do nordeste brasileiro, por exemplo, não podendo ser descartado do estudo da
violência, os ranços ainda existentes do autoritarismo e do coronelismo, condições que põem a violência
e a criminalidade como fenômeno social e cultural, que tem expressão difusa e fragmentada como um
mosaico.
O fato é que violência e crime não são sinônimos, uma vez que os atos violentos e criminais podem
estar contidos na mesma ação ou não. Há atos de violência que não se configuram em crimes, mas todos
os crimes se configuram em violência, tendo em vista que o crime em si é uma violência enquadrada e
reprimida pela lei. Enquanto a violência se refere ao constrangimento físico ou moral, a criminalidade é a
expressão dada ao conjunto de infrações que são produzidas em um tempo e lugar determinado,
constituindo-se em uma transgressão da lei e da norma, uma desobediência à imposição da regra jurídica.
O crime é o resultado da violência, e esta é um fenômeno histórico-social, podendo estar presente em
todas as práticas sociais, enquanto o crime é um fenômeno jurídico. Conforme Feltran (2008), a noção
de criminalidade violenta indica o conjunto de atos ilegais e ilícitos nos quais se utiliza da força de coerção
(violenta) ou da ameaça de sua utilização.
A representação da violência e do crime em uma sociedade não se situa apenas no âmbito jurídico-
legal, mas é também social e se associa diretamente às formas de controle exercidas pelo Estado em
relação ao ato e o que ele representa para a sociedade. Cada sociedade define nas convenções legais
de seu país o que é considerado crime. Para Émile Durkheim, a existência do crime é um fato social
normal, embora sempre abominável, e logo punível à incidência do ato criminal.
O crime não se produz só na maior parte das sociedades desta ou daquela espécie, mas em todas as
sociedades, qualquer que seja o tipo destas. Não há nenhuma em que não haja criminalidade. Muda de
forma, os atos assim classificados não são os mesmos em todo o lado; mas em todo o lado e em todos
os tempos existiram homens que se conduziram de tal modo que a repressão penal se abateu sobre eles.
Os atos criminosos e violentos estão presentes nas relações humanas nos mais remotos e
diferenciados momentos históricos. Entretanto, os sentidos desses atos se encontram presentes nas
interpretações dos efeitos que eles causam no indivíduo e na sociedade, já que os atos estão

Apostila gerada especialmente para: Betânia Mueller 023.712.580-32


. 195
subordinados às convenções culturais, sociais e jurídicas que regem o contexto espacial, temporal e
relacional em que eles se processaram. Salientando que o ato criminoso se pauta pela regulamentação,
e não há crime sem lei que antes o defina, o ato violento se pauta na sua intensidade agressiva,
explicitando-se através de diversas linguagens e representações, que transgridem o convencionado como
certo, legal e ético, e estabelece, com o uso da força, deliberações que instauram novas formas de
controle à ordem social.
O crime, como categoria sociológica é menos complexo do que a violência, considerando que o crime
é objeto de regulamentação jurídica, seu entendimento é objetivado pela circunscrição penal, e a violência
é um fenômeno difuso em que suas representações sociais dependem dos “valores e das crenças que
estruturam e presidem a vida social”.
A difusão da violência na sociedade contemporânea é de difícil classificação, tendo em vista que a
intensidade da ação agressiva se altera de acordo aos contornos subjetivos instituídos à representação
dada pelo sujeito a determinadas ações que podem ser consideradas e enquadradas como violência para
determinado sujeito e para outro não.
Na sociedade contemporânea, em que a violência se dissemina de forma banalizada, a classificação
das formas de violência varia de acordo com os sentidos que os indivíduos atribuem a essa, podendo ser
categorizada em macro e microviolências. A macroviolência se relaciona às lógicas do capitalismo que,
obediente aos determinantes internacionais do mercado financeiro global, produz desigualdades sociais
e econômicas entre os continentes e os países, aumentando a pobreza e diluindo os direitos humanos. A
macroviolência é diluída e invisibilizada nas condições de vidas, possuindo efeitos nefastos na vida das
pessoas, entretanto não conseguimos percebê-la de forma materializada em nosso cotidiano, pois ela se
revela através de microviolências presentes nos conflitos e tensões recorrentes em nosso cotidiano,
manifestando-se através de conflitos intersubjetivos, prática de atos incivilizados que muitas vezes se
banalizam, até assaltos e agressões graves.
Conforme autores como Alba Zaluar e Michel Misse, os fatores que operam o crescimento das macro
e microviolências são diversos e interconectados, podendo agir de forma direta ou indireta na mobilização
do ato violento. Esses fatores podem ser organizados em dois eixos, que mobilizam os principais
elementos geradores do crescimento da violência na contemporaneidade – o fator econômico-social e o
fator cultural-ético-político. O fator econômico-social se refere ao modo de produção e às lógicas
associadas à sua materialidade econômica, formas de acumulação e estruturação de classe, relação com
o Estado e seus efeitos no gerenciamento das desigualdades sociais, na política de distribuição de renda
e acesso aos direitos civis, elevação de qualidade de vida, e seus desdobramentos nas dimensões do
emprego, salário, moradia, transporte, segurança, estudo, saúde; tudo que diz respeito à vida do indivíduo
relacionada a esses aspectos. O fator cultural-ético-político se refere a um conjunto de ideologias, vícios
e comportamentos reproduzidos socialmente e internalizados nas práticas sociais, a exemplo da
impunidade e descrença com a Justiça; corrupção e crise das instituições formais de poder do Estado;
desvalorização e descrença com a educação; crise da autoridade familiar; sensação de desprezo em
relação aos direitos e os deveres; e sentimento de “desvinculação social”.

Conceito de violência intrafamiliar

O conceito de violência intrafamiliar inclui toda e qualquer forma de agressão causada entre pessoas
que tenham vínculos familiares ou afetivos entre si e, também, vínculos decorrentes da convivência
próxima. A violência intrafamiliar pode ser praticada contra o gênero feminino e masculino. É um tipo de
violência que ocorre dentro de casa, nas relações entre as pessoas da família, entre homens e mulheres,
pais, mães e filhos, entre jovens e idosos, além de eventuais cuidadores que podem estabelecer, mesmo
que momentaneamente, um vínculo doméstico.
Exemplo de definição para este conceito, dado por Machado e Gonçalves (2003): qualquer ato,
conduta ou omissão que sirva para infligir, deliberadamente e com intensidade, sofrimentos físicos,
sexuais, mentais ou econômicos, de modo direto ou indireto (por meio de ameaças, enganos, coação ou
qualquer outro meio) a qualquer pessoa que habite no mesmo agregado doméstico privado (crianças,
jovens, mulheres, homens ou idosos) ou que, não habitando no mesmo agregado doméstico privado que
o agente da violência seja cônjuge ou companheiro marital ou ex-cônjuge ou ex-companheiro marital.

Esta definição implica a referência a vários crimes: violência doméstica, ameaça, coação, difamação,
injúria, subtração de menor, violação de obrigação de alimentos, abuso sexual, homicídio e outros.

Apostila gerada especialmente para: Betânia Mueller 023.712.580-32


. 196
Tipos de violência

- Violência física
Violência física é entendida como qualquer conduta que ofenda a integridade ou saúde corporal, como
por exemplo, lesões corporais e tortura. É o uso da força com o objetivo de ferir, deixando ou não marcas
evidentes. São comuns “murros” e “tapas”, agressões com diversos objetos e queimaduras por objetos
ou líquidos quentes.

- Violência psicológica
Violência psicológica consiste na conduta que cause dano emocional, diminuição da autoestima,
prejudique e perturbe o pleno desenvolvimento ou que vise degradar ou controlar ações,
comportamentos, crenças e decisões. São exemplos de violência psicológica: ameaça, constrangimento,
humilhação, manipulação, isolamento, vigilância constante, perseguição contumaz, insulto, chantagem,
ridicularização, exploração e limitação do direito de ir e vir ou qualquer outro meio que lhe cause prejuízo
à saúde psicológica e à autodeterminação.
A violência psicológica ou “agressão emocional”, às vezes tão ou mais prejudicial que a física, é
caracterizada por rejeição, depreciação, discriminação, humilhação, desrespeito e punições exageradas.
Trata-se de uma agressão que não deixa marcas corporais visíveis, mas emocionalmente pode causar
cicatrizes permanentes para toda a vida.

- Violência verbal
A violência verbal normalmente se dá concomitante à violência psicológica, sendo considerada um
subtítulo daquela. Alguns agressores verbais dirigem sua “artilharia” (xingamentos, palavrões, ofensas)
contra outros membros da família, inclusive nos momentos em que estes estão na presença de outras
pessoas estranhas ao lar.

- Violência sexual
A violência sexual é entendida como qualquer conduta que constranja a vítima a presenciar, a manter
ou a participar de relação sexual não desejada, mediante intimidação, ameaça, coação ou uso da força;
que induza a comercializar ou a utilizar, de qualquer modo, a sexualidade; que impeça de usar qualquer
método contraceptivo ou que force ao matrimônio, à gravidez, ao aborto ou à prostituição, mediante
coação, chantagem, suborno ou manipulação; ou que limite ou anule o exercício de seus direitos sexuais
e reprodutivos. Exemplos: estupro e exploração sexual comercial.

- Violência patrimonial
A violência patrimonial é entendida como qualquer conduta que configure retenção, subtração,
destruição parcial ou total de objetos, instrumentos de trabalho, documentos pessoais, bens, valores e
direitos ou recursos econômicos da vítima, incluindo os destinados a satisfazer suas necessidades.

- Violência moral
A violência moral é aquela entendida como qualquer conduta que configure calúnia, difamação ou
injúria. Também se dá de forma verbal, porém, o conteúdo das ofensas se dá prioritariamente em falsas
acusações contra a vítima.

- Negligência
A negligência configura-se quando os pais ou responsáveis falham em prover cuidados de saúde,
nutrição, higiene pessoal, vestimenta, educação, habitação e sustentação emocional e, quando tal falha
não é o resultado das condições de vida além do seu controle.
Os diferentes tipos de violência intrafamiliar, em sua maioria não acontecem separadamente, estão
intrinsecamente relacionados e podem ser concretizados em um único ato de violência. Alguns tipos são
mais facilmente detectados do que outros. Contudo, as consequências de todos eles deixam marcas
muitas vezes indeléveis na vida de suas vítimas e familiares.

Violência intrafamiliar contra a mulher

A violência contra a mulher é estrutural e está presente em nossa sociedade desde os primórdios da
civilização. Este tipo de violência é inerente aos sistemas patriarcal e capitalista, sendo usado como uma
ferramenta de controle da vida, corpo e sexualidade de mulheres.

Apostila gerada especialmente para: Betânia Mueller 023.712.580-32


. 197
As mulheres sempre resistiram, e seguem resistindo, a este tipo de violência, tanto no âmbito individual
quanto no coletivo. Sempre que uma mulher age dessa forma, desafiando ou denunciando a violência
contra ela mesma ou contra mulheres de sua comunidade, está rompendo com o paradigma machista
dominante e com o ciclo de violência contra a mulher.
A defesa da igualdade de gênero tem o amparo legal na Constituição Brasileira, promulgada em 1988,
que estabelece no seu artigo 5º que “todos são iguais perante a lei, sem distinção de qualquer natureza”.
A alta incidência da violência contra a mulher cometida no espaço doméstico e familiar e a luta das
mulheres alcançou um marco jurídico fundamental que é a Lei 11.340/2006 - Lei Maria da Penha.
Através desse instrumento, configura-se como Violência Doméstica e Familiar Contra a Mulher toda e
qualquer ação ou omissão que cause morte, lesão, sofrimento físico, sexual ou psicológico e dano moral
ou patrimonial, praticado por pessoa (homem ou mulher) com que a ofendida (somente a mulher) conviva
no âmbito doméstico, ou que faça parte do seu âmbito familiar ou de qualquer relacionamento íntimo de
afeto atual ou já encerrado.
A mulher em situação de violência doméstica deve ser orientada para comparecer à DDM (Delegacia
de Defesa da Mulher) ou à delegacia comum para registrar o boletim de ocorrência e para efetuar a
representação contra o agressor. A representação é a confirmação do interesse em processar
criminalmente o agressor e é exigida em alguns crimes. O prazo para efetuar a representação é de seis
meses a partir da data da agressão. Lavrado o boletim de ocorrência e feita a representação perante a
autoridade policial, a ação judicial somente será arquivada a pedido da mulher perante o(a) juiz(a), após
ouvido o(a) promotor(a) de justiça.

Violência intrafamiliar contra a pessoa com deficiência

A segregação das pessoas com deficiência fez parte da cultura das sociedades registradas
historicamente, o que já caracteriza a presença de cenários graves de preconceito e violência contra esse
segmento da população. Nos dias atuais ainda é muito difícil lidar com as minorias e com as diferenças
existentes, fazendo parte dos movimentos sociais a luta pela equidade social (garantir a igualdade de
oportunidades e direitos, mas considerando positivamente a existência das diferenças).
A falta de debate público e políticas sociais efetivas contribuem para que as pessoas com deficiência
estejam potencialmente mais expostas a situações de violência - inclusive no âmbito intrafamiliar - tendo
maior dificuldade em denunciar os abusos sofridos. Outro fator importante que devemos considerar é que,
ainda nos dias de hoje, a falta de acessibilidade provoca a perda de autonomia das pessoas com
deficiência, dificultando o rompimento com a situação de violência.
Podemos dizer que alguns fatores potencializam a vitimização da pessoa com deficiência, como a
infância, o gênero feminino e a condição de pessoa idosa, que são fatores que por si só também
caracterizam as possíveis vítimas, levando em consideração que o agressor procura estar em posição de
poder em relação ao outro (poder de autoridade, social, econômico, etc.).
A informação contribui substantivamente para o empoderamento da população, por isso, faz-se
necessário que as pessoas com deficiência e seus familiares conheçam os seus direitos e que a
Defensoria Pública participe desse processo, garantindo tanto com ações judiciais quanto
extrajudicialmente que esse público tenha direito à acessibilidade e demais direitos que compõem sua
cidadania e os protegem da violência.

Violência intrafamiliar contra crianças e adolescentes

A violência intrafamiliar ou doméstica contra crianças e adolescentes é uma das várias expressões da
questão social, considerada pelo Ministério da Saúde como um sério problema de saúde pública. Suas
raízes estão associadas ao contexto histórico, social, cultural, econômico e político em que se inserem
vítimas e agressores. Assim, este tipo de violência não pode ser compreendido somente como uma
questão de conflitos interpessoais entre pais e filhos.
Há alguns anos, pensava-se que a violência intrafamiliar contra crianças e adolescentes era
consequência de transtornos individuais, alcoolismo, toxicomania, fragilidades socioeconômicas ou
educacionais, dentre outras de caráter individual. Contudo, segundo a literatura recente, passa a ser
compreendida como uma questão multifacetada, caracterizada principalmente por aspectos sociais e
culturais.
Tanto no Brasil como em várias partes do mundo, em diferentes culturas e classes sociais,
independente de sexo ou etnia, crianças e adolescentes são vítimas cotidianas da violência doméstica.
Os casos registrados em delegacias, conselhos tutelares, hospitais e institutos médico-legais são apenas

Apostila gerada especialmente para: Betânia Mueller 023.712.580-32


. 198
um alerta; não revelam a verdadeira dimensão do problema. A cultura do silêncio e da omissão ainda é
muito forte em nossa sociedade.
De acordo com as professoras Maria Amélia Azevedo e Viviane Guerra (USP/SP, 1989) a questão da
violência doméstica contra crianças e adolescentes pode ser considerada

[...] todo ato ou omissão praticado por pais, parentes ou responsáveis contra crianças e/ou
adolescentes que - sendo capaz de causar dano físico, sexual e/ou psicológico a vítima - implica de um
lado, numa transgressão do direito que crianças e adolescentes têm de ser tratados como sujeitos e
pessoas em condição peculiar de desenvolvimento. (AZEVEDO e GUERRA, 1989).

Segundo Day et. al. (2003) a Constituição Federal e o Estatuto da Criança e do Adolescente passam
a ser os novos paradigmas para o sistema de Justiça, para a sociedade e para o Brasil como um todo. A
nova legislação, signatária da Doutrina da Proteção Integral, reconhece direitos à criança e ao
adolescente, respeitando seu estágio de desenvolvimento. É uma das legislações mais avançadas do
mundo, que pouco a pouco começa a ser implementada.
A violência contra crianças e adolescentes apresenta-se sob diversas formas, tanto que um sintoma
ou sinal isolado não permite afirmar sua existência. Por isso, é fundamental o olhar atento e crítico dos
profissionais e da sociedade frente aos problemas identificados, tanto de ordem física, sexual como
emocional - procurando a sua correlação com o relato da possível vítima, dos familiares ou pessoas de
sua convivência.

Violência intrafamiliar contra idosos

O número de idosos no Brasil está cada vez maior, mas algumas instituições e sujeitos ainda estão
despreparados para lidar com questões próprias do envelhecimento, assim como condições psicológicas
e sociais, o que contribui para o crescimento da violência contra idosos.
Fatores sociais como isolamento, casas de longa permanência, empobrecimento da população,
dificuldades dos cuidadores ou dependência financeira dos membros da família, ampliam a possibilidade
de ocorrência de violência, uma vez que torna o idoso mais vulnerável.
Em relação aos aspectos legais, a Constituição Federal impede qualquer forma de discriminação por
idade e atribui à família, a sociedade e ao Estado o dever de amparar o idoso, assegurar sua participação
na comunidade, defender sua cidadania e bem-estar e garantir seu direito à vida. Além disso, o crime
cometido contra idosos é agravante da pena nos delitos previstos pelo Código Penal.
O Estatuto do Idoso transforma em lei o que deveria ser obrigação da sociedade. Estabelece que, ao
chegar aos 60 anos, todos têm o direito de envelhecer com dignidade, e tanto o Estado, como a sociedade
e a família têm obrigação de assegurar aos idosos do país liberdade e respeito, como pessoas humanas
e sujeitos de direitos civis, políticos, individuais e sociais garantidos pela Constituição.
A maioria dos agressores de idosos é um familiar ou algum conhecido. Uma das queixas mais
registradas é a violência patrimonial, que decorre da apropriação indevida dos bens do idoso através do
uso da violência. Os idosos acabam permitindo que os agressores se apropriem de seus bens, pelo medo
da solidão, da vida muitas vezes vulnerável em casas de longa permanência e clínicas geriátricas.
Os idosos maltratados, sendo mais vulneráveis que as demais faixas etárias, encontram-se muitas
vezes inseguros e fragilizados e, por essa razão não procuram em primeira instância os serviços
especializados. Sem o apoio de familiares e sem amizades, torna-se difícil e constrangedor procurar,
sozinho, os serviços de segurança pública especializados.
Alguns idosos são, ainda, destituídos do poder de decisão, privados de ocupar um espaço físico
próprio, do direito de escolha, da liberdade de expressão e do direito de envelhecer com saúde e plenitude
de cidadania.
A presença ativa na vida social deve ser valorizada e estimulada junto aos idosos, mediante formas
de convívio e expressão, em centros de comunidade, clubes de convivência, associações culturais, de
lazer e esportivas, atividades de recuperação da história e habilidades de uma comunidade, valorizando-
se a transmissão aos mais jovens de suas experiências e histórias de vida.
Para isso a rede de atendimento à pessoa idosa é fundamental e precisa estar articulada numa
diversidade de serviços, como a viabilização de formas alternativas de participação, ocupação e convívio
do idoso.

Apostila gerada especialmente para: Betânia Mueller 023.712.580-32


. 199
O tratamento dispensado às pessoas em situação de violência intrafamiliar

Quando recebemos uma pessoa tentando romper o ciclo de violência, cabe a nós participar desse
processo, realizando um atendimento de qualidade e comprometimento diante da sua situação.
Algumas vezes vamos perceber que uma pessoa pode dar início a um processo judicial para se
desvincular da situação de violência, mas que tempos depois, desiste de levá-lo adiante. Esta situação é
comum nos casos que envolvem violência e laços afetivos, sendo caracterizada, segundo estudos
científicos, como um momento do perverso ciclo da violência.
Nas situações de violência intrafamiliar, devemos estar atentos para não fazer julgamentos baseados
no senso comum, mas sim colaborar no processo de fortalecimento de cidadania da vítima para que se
consiga estabelecer um cotidiano sem violência.
A vítima da violência intrafamiliar muitas vezes é conduzida a acreditar que é ela mesma a responsável
pela violência sofrida, o que contribui para a construção de sentimentos de vergonha e culpa. Neste
sentido, um atendimento inadequado pode colaborar para que esses sentimentos e condições éticas se
cristalizem, colocando mais obstáculos para o rompimento do ciclo de violência. Num atendimento a uma
vítima de violência, seja ela qual for, é importante, antes de tudo, que se constitua um ambiente sigiloso
e acolhedor, que poderá indicar à vítima o respaldo à sua necessidade, sem julgamento, e sem que ela
se sinta o “assunto do dia” na instituição.
Portanto, o atendimento deve ser realizado em um local que garanta o sigilo das informações que
estão sendo reveladas, cumprindo o princípio da Psicologia e do Serviço Social no Art. 1, Item IX, da
Deliberação CSDP 187, de 12/08/2010, bem como dos respectivos códigos de ética profissionais e
resoluções dos conselhos federais de profissão.
A violência intrafamiliar é constantemente tratada como assunto da esfera privada das relações sociais,
cabendo exclusivamente à vítima se desvencilhar dessa situação. Porém, é responsabilidade também da
sociedade e do Estado trazer essa temática ao campo público para garantirmos que todo cidadão tenha
o direito a uma vida sem violência.

Serviços da Rede de Proteção que podem ser acessados

- CREAS (diversos públicos)


O que é? O Centro de Referência Especializado de Assistência Social (Creas) configura-se como uma
unidade pública e estatal, que oferta serviços especializados e continuados a famílias e indivíduos em
situação de ameaça ou violação de direitos (violência física, psicológica, sexual, tráfico de pessoas,
cumprimento de medidas socioeducativas em meio aberto, etc.).
O que oferece? Atendimento psicológico, social e jurídico especializado a pessoas com direitos
violados e suas famílias, inclusive vítimas de violência.

Violência contra mulher

- CRM – Centro de Referência da Mulher


O que é? O Centro de Referência é o espaço estratégico da política de enfrentamento à violência
contra as mulheres, exercendo o papel de articulador dos serviços, organismos governamentais e não
governamentais que integram a rede de atendimento às mulheres e garantindo o acesso a outros serviços
para as que se encontram em situação de vulnerabilidade social, em função da violência de gênero.
O que oferece? Atendimento psicológico, social e jurídico a mulheres vítimas de violência, tendo ou
não registrado boletim de ocorrência.

- Casa Abrigo
O que é? As Casas Abrigo oferecem atendimento temporário para mulheres em risco de morte. Os
endereços são sigilosos para garantir a segurança e integridade física da mulher.
O que oferece? Assistência integral à mulher que tenha necessidade de acolhimento residencial, em
virtude de risco de morte.

- Delegacia de Defesa da Mulher – DDM


O que é? Delegacia especializada no atendimento a mulheres vítimas de violência.
O que oferece? Registro de denúncias (ocorrências), investigação criminal, solicitação de medidas
protetivas ao juízo competente, encaminhamento para outros órgãos de proteção.

Apostila gerada especialmente para: Betânia Mueller 023.712.580-32


. 200
- Disque 180
O que é? Este serviço oferecido pela Secretaria Especial de Políticas para as Mulheres da Presidência
da República que esclarece as vítimas sobre seus direitos e mostra onde e como obter ajuda. Funciona
24 horas, todos dias da semana, inclusive aos finais de semana.
O que oferece? Informações gratuitas via telefone

Violência contra crianças e adolescentes

- Conselho Tutelar
O que é? Órgão encarregado de zelar pelo cumprimento dos direitos da criança e do adolescente.
O que oferece? Acolhimento e averiguação de denúncias, aplicação de medidas de proteção a
crianças e adolescentes e medidas aos pais, requisição de serviços públicos

- Delegacia Especializada
O que é? Delegacia especializada no atendimento a criança e adolescentes que sofreram violência.
O que oferece? Registro de denúncias (ocorrências), investigação criminal, encaminhamento para
outros órgãos de proteção.

- Disque 100
Violência contra pessoas com deficiência

- Delegacia de Polícia
O que é? Atendimento a cidadãos vítimas de violência.
O que oferece? Registro de denúncias (ocorrências), investigação criminal, encaminhamento para
outros órgãos de proteção.

- Violência contra idosos

- Delegacia Especializada
O que é? Delegacia especializada no atendimento a idosos vítimas de violência.
O que oferece? Registro de denúncias (ocorrências), investigação criminal, solicitação de medidas
protetivas ao juízo competente, encaminhamento para outros órgãos de proteção.

- Centro de Referência do Idoso


O que é? Serviço de referência, proteção e defesa de direitos da pessoa idosa.
O que oferece? Atendimento de modo individual e coletivo estimula a participação social. É espaço
difusor de conhecimento e intercâmbio de experiências inovadoras, objetivando fortalecer as políticas
públicas e disseminar práticas qualificadas para os demais parceiros da rede de proteção ao idoso. Além
da atenção direta aos idosos; é referência para qualificação institucional e defesa dos direitos, mantendo
estratégias de trabalho articulado com a rede de proteção social ao idoso.

Referência:
Autor: Marco Antônio de Oliveira Branco
Organização: Assessoria Técnica Psicossocial e Comissão de Estudos Interdisciplinares
Revisão: Assessorias Criminal e Cível
http://www.defensoria.sp.gov.br/dpesp/repositorio/0/documentos/cam/Violencia.pdf
Questões

01. (SGA/AC - Psicólogo - CESPE). A respeito de violência e de vulnerabilidade, assinale a opção


incorreta.
(A) A violência se define pela carência material, pela periferização e pela favelização que ocorrem no
contexto de desigualdades entre ricos e pobres.
(B) Crianças vítimas de violência são mais prováveis de se tornarem agentes de violência quando
adultos.
(C) A deficiência dos sistemas judiciais e a falta de confiança da população no cumprimento das leis
contribuem para a ocorrência de atos violentos, pois os criminosos avaliam que o risco de serem
capturados é menor que os eventuais benefícios oferecidos pelo crime.
(D) O não-acesso à educação, ao trabalho e à saúde diminui as chances de aquisição e
aperfeiçoamento desses recursos que são fundamentais para o acesso do indivíduo às oportunidades de
ascensão social.

Apostila gerada especialmente para: Betânia Mueller 023.712.580-32


. 201
02. (TJ/ES - Analista Judiciário - CESPE). Considerando que a violência é um problema estrutural
das sociedades humanas, julgue os itens que se seguem.
Para se obter a mitigação da violência social, é suficiente que haja articulação das instituições de
Estado para a formulação de políticas públicas de combate à violência e inclusão social.
( ) Certo ( ) Errado

Gabarito

01.A / 02.Errado

Comentários

01. Resposta: A
Violência vem do latim violentia, que significa violência, caráter violento ou bravio, força. O verbo
violare significa tratar com violência, profanar, transgredir. Tais termos devem ser referidos a vis, que quer
dizer força, vigor, potência violência, emprego de força física, mas também quantidade, abundância,
essência ou caráter essencial de alguma coisa.

02. Resposta: errado


Por óbvio que não podemos concentrar a redução da violência apenas nas mãos do Estado, embora
ele deva nos proteger. Por exemplo, na relação intrafamiliar é responsabilidade também da sociedade e
do Estado trazer essa temática ao campo público para garantirmos que todo cidadão tenha o direito a
uma vida sem violência.

Técnicas de Resolução de Conflitos e Comunicação Não Violenta.

A mediação é uma alternativa para a resolução de conflitos. É um procedimento voluntário e envolve


a participação de um terceiro imparcial que coordena o processo, estimulando as partes envolvidas por
intermédio do diálogo. 14
Quando alguém nos escuta com atenção, abstendo-se de julgamentos, críticas e opiniões, pode
despertar em nós algo surpreendentemente novo, capaz de transformar uma situação aparentemente
impossível numa nova possibilidade, despertando nossa disposição e coragem de negociar possíveis
interesses e necessidades.
Desse modo, o processo de mediação deve “estimular a reflexão, criar espaço para uma maior
consciência de si mesmo e ampliar a consciência a respeito das necessidades e pontos de vista do outro”.
Deve garantir que a deliberação quanto ao interesse de cada indivíduo seja protagonizada por ele mesmo,
atribuindo-lhe um sentido de compromisso e autodeterminação em relação ao próprio destino, e agindo
como sujeito de sua própria história.
Como dissemos antes, a mediação é um processo que, através da ajuda de uma pessoa neutra e
imparcial (o mediador), ajuda as pessoas a dialogarem e a cooperarem para resolver um problema. Nesse
sentido, a mediação é mais do que um método para solucionar os conflitos; também é uma forma de
impedir conflitos no futuro, pois já cria um clima de cooperação entre as pessoas.

Para Almeida (2003):


[...] Mediação facilitadora (também chamada de mediação não-diretiva) é aquela na qual o mediador
exerce tão somente a função de facilitar a negociação entre as partes, focalizando seus interesses e
auxiliando a formação de um consenso mais célere e menos oneroso. O mediador, na mediação
facilitadora, não expõe sua opinião sobre os pedidos das partes e nem mesmo sobre o acordo. A atuação
do mediador tende a aproximar as partes, conciliando interesses convergentes. Assim, passa a existir
uma parceria entre elas, compondo uma mesma relação negocial [...] (p.175-200).

Ao todo, podemos dizer que a mediação de conflitos tem quatro principais objetivos: a solução de
conflitos, a prevenção de conflitos, a inclusão social e a paz social. Vejamos abaixo com mais detalhes
cada um dos objetivos:

14
http://revistauniversitas.inf.br/index.php/UNIVERSITAS/article/viewFile/16/2

Apostila gerada especialmente para: Betânia Mueller 023.712.580-32


. 202
A solução de conflitos
O principal objetivo da mediação é a solução de maneira pacífica do conflito. O diálogo é o principal
meio para se atingir este objetivo, pois, é através de um diálogo franco que as partes chegam a um acordo
satisfatório para ambas.
Quando falamos em um acordo, vale a pena lembrar que este deve ser justo, consequência de um
diálogo honesto. É papel da mediação garantir que essa comunicação aconteça de forma livre, franca
mas tranquila, sempre buscando a solução para o conflito.

A prevenção de conflitos
Durante o processo de mediação, tanto o mediador quanto as pessoas envolvidas devem se
aprofundar no problema existente, de forma a buscar uma solução definitiva – ou seja, que não deixe o
problema “voltar” no futuro.
Através dessa colaboração, as pessoas já criam as condições para evitar futuros conflitos. A mediação,
nesse sentido, é transformadora, pois cria uma relação que vai além do acordo que resolveu o problema
original: essa relação de respeito entre partes envolvidas já é o primeiro passo para resolver qualquer
conflito de forma pacífica.

A inclusão social
Através do processo de mediação, as pessoas criam uma consciência maior dos seus direitos e
deveres, possibilitando uma reflexão profunda sobre as questões sociais.
Nesse processo, as pessoas percebem que, independente da classe social, todo mundo possui
direitos, e que todos podem colaborar para escolher o melhor caminho a ser tomado.
A mediação estimula as pessoas a resolver os seus próprios conflitos. Isso faz com que cada indivíduo
conheça melhor os seus direitos e deveres; e como o diálogo pode levar a uma solução de seus
problemas de modo que todos podem sair ganhando. Esse processo ajuda na inclusão social de todos,
ao perceberem o papel importante que têm na luta para fazer valer os seus próprios direitos.
A paz social
Ao conseguir uma solução pacífica para os conflitos, a mediação também previne a violência e pacifica
a sociedade. Através do diálogo, as partes envolvidas em um conflito evitam partir para a violência física
ou moral.
A mediação é um processo que se alicerça, principalmente, na solidariedade, ajuda as pessoas a
buscar os interesses em comum, a não prejudicar um ao outro, e tudo isso ajuda a criar uma cultura de
paz social.

Princípios da mediação de conflitos

A mediação de conflitos é geralmente definida como a interferência consentida de uma terceira parte
em uma negociação ou em um conflito instalado, com poder de decisão limitado, cujo objetivo é conduzir
o processo em direção a um acordo satisfatório, construído voluntariamente pelas partes, e, portanto
mutuamente aceitável com relação às questões em disputa.
Para mediar um conflito, é importante observar alguns princípios, ou condições. Se ignorarmos estes
princípios básicos, a mediação de conflito tem poucas chances de funcionar corretamente.

- Liberdade das partes


Para resolverem os seus conflitos através da mediação, as pessoas envolvidas devem estar livres, ou
seja, não estarem sendo ameaçadas ou até mesmo sofrer algum tipo de violência física. Como dissemos,
a mediação é um processo voluntário, ou seja, as pessoas só participam se quiserem, e a mediação só
é possível quando todas as pessoas concordam com ela.
Quando optar pela mediação, um meio de solução amigável, a pessoa deve fazê-lo de forma
consciente e por vontade própria.
Isso também significa que, além da mediação em si, as pessoas envolvidas devem concordar na
escolha do mediador.

- Não-competitividade
Como dissemos antes, a mediação não é uma competição. Por isso, o conflito deve ser tratado de
maneira positiva e colaborativa. Nessa lógica, a mediação não busca criar um “vencedor” para o conflito,
mas sim uma forma de resolver o problema de forma satisfatória para todos, de forma pacífica.

Apostila gerada especialmente para: Betânia Mueller 023.712.580-32


. 203
- Poder de decisão das partes
Apenas as pessoas envolvidas no conflito têm o poder de tomar decisões ao longo do processo de
mediação. O mediador apenas ajuda as partes, facilitando o diálogo e esclarecendo cada um dos lados.
O mediador não tem poder de decisão, já que a mediação não é um processo impositivo; ou seja, ele
não interfere na tomada de uma decisão, e esta não depende de pessoas que não estejam envolvidas no
conflito. Somente as partes envolvidas é que decidirão acerca do problema.

- Participação de terceiro imparcial


O mediador tem que ser sempre imparcial, ou seja, não pode se colocar do lado de nenhum dos lados
envolvidos no conflito. É papel do mediador facilitar o diálogo, ajudar as pessoas a reconhecer o conflito
existente, porém, sem ficar favorável a nenhuma das partes.
Vamos nos aprofundar no papel que o mediador tem para cumprir ao longo deste e do próximo módulo.

- Competência do mediador
A pessoa tem que ter capacidade para resolver a controvérsia. O mediador tem que ter qualificações
mínimas para dar andamento no processo de mediação.
A pessoa que for mediar tem que ter cuidado e prudência, assegurando sempre a qualidade do
processo para obter com sucesso a solução do conflito.
Cada mediador atua como um diretor de cinema ou um maestro de orquestra, conforme salienta Jean-
François Six, quer dizer, são responsáveis pela condução do processo, mas “não tomam o lugar dos
atores ou dos músicos, a quem compreendem, infundem confiança, insuflam ritmo, trazem uma espécie
de energia suplementar e impulsionam a dar todo o seu talento. Mas ao final das contas, diretor e maestro
são tidos como os primeiros responsáveis pela obra produzida”.

- Informalidade do processo
Na mediação, não existem regras rígidas onde o processo se baseia; não existe uma forma
predeterminada. Neste método, não são seguidas as regras do Direito: existe um código de ética que
pode ser seguido, mas isso não é obrigatório, pois no Brasil não existe uma legislação que regule o
processo da mediação. Isso não significa que o compromisso assumido no processo de mediação não
tenha validade. As pessoas que participam de um processo de mediação se comprometem umas com as
outras, e elas mesmas são responsáveis pelo cumprimento de seus compromissos.

- Confidencialidade no processo
O processo de mediação é confidencial. O mediador deve manter o sigilo do que foi discutido entre as
partes, ou seja, não pode divulgar as informações discutidas durante a mediação, nem antes, durante ou
depois do processo. A confiança dos mediados no mediador também surge quando este mostra estar
comprometido com o sigilo da mediação. A juíza Ellen Gracie Northfleet, do Supremo Tribunal Federal,
afirma que:
“O clima de informalidade e confidencialidade das sessões favorecem o esclarecimento de situações
que talvez não aflorassem na sala das audiências. O diálogo que se estabelece entre as partes é mais
verdadeiro porque envolve a inteireza de suas razões e não apenas aquelas que poderiam ser deduzidas
com forma e figura de juízo”.

- Soluções ganha-ganha
A mediação de conflitos, como já foi enfatizado aqui, não é competitiva – ou seja, não segue à lógica
de que tem que haver um vencedor, e os outros são perdedores. O processo de mediação é busca para
que todos os lados saiam ganhando. Por isso, a mediação é uma solução do tipo “ganha-ganha”. As
soluções do tipo ganha-ganha caracterizam-se por atender, ao mesmo tempo, as exigências do eu
(assertividade) e do outro (compreensão). O que pede, por um lado, um autoconhecimento e um autêntico
conhecimento e escuta do outro. Muitas vezes, NÓS é que projetamos em outros as nossas próprias
sombras, criando e inventando inimigos.

Etapas da mediação de conflitos15


Apresentamos a seguir um roteiro contendo o passo a passo do processo de mediação. Acompanhe
as etapas de como a mediação pode ser feita, mas lembre-se que este roteiro não é uma receita fixa:
você pode mudar ou dispensar alguma etapa, de acordo com o conflito que estiver enfrentando. Seja
flexível e criativo, mas sempre cuidadoso.

15
http://formacaoredefale.pbworks.com/f/SEDH_Media%C3%A7%C3%A3o+de+Conflito_M%C3%B3dulo+8.pdf

Apostila gerada especialmente para: Betânia Mueller 023.712.580-32


. 204
Pré-mediação
A pré-mediação deve ser realizada de forma separada, com o mediador dialogando com cada uma
das partes envolvidas, conforme segue:
• Apresente-se como mediador;
• Pergunte às pessoas envolvidas no conflito se gostariam que você as ajudasse a resolver o problema;
• Encontre um lugar calmo para fazer a mediação;
• Combine as regras do processo de mediação.

Recepção
Receba gentilmente as pessoas e, antes de começar, peça que elas concordem com as seguintes
regras:
• Tentar solucionar o problema de forma pacífica;
• Não ofender verbalmente o outro;
• Não interromper, cada parte terá o mesmo tempo para falar;
• Guardar segredo.

Primeira parte da mediação: a escuta ativa


É função do mediador criar um ambiente em que as pessoas fiquem à vontade para falar de maneira
franca e honesta. Para isso, é preciso deixar falar e não interromper, mas também é possível fazer
algumas perguntas para desenvolver a discussão de forma racional:
• Pergunte à primeira pessoa: O que aconteceu? Parafraseie (diga a mesma coisa usando outras
palavras e procure retirar o tom negativo das frases, especialmente se elas tiverem um tom acusatório)
• Pergunte à primeira pessoa: Como está se sentindo?
• Pergunte à segunda pessoa: O que aconteceu? Parafraseie.
• Pergunte à segunda pessoa: Como está se sentindo?
O mediador deve ajudar os envolvidos a não criar um clima de acusações. A ideia é que as pessoas
devem se concentrar na raiz do problema que enfrentam, e porque ele é importante para cada um. O
mediador deve ainda estimular a capacidade das partes em compreender o ponto de vista da outra parte
e evitar ficar procurando culpados.

Mensagens-Eu
Nesta primeira parte do processo de mediação, as “mensagens-eu” são uma forma simples de dizer o
que cada uma das pessoas envolvidas está sentindo. As “mensagens-eu” ajudam a sensibilizar as
pessoas a compreender “o outro lado”, o ponto de vista da outra pessoa, e a abrir o diálogo.
Mensagens-eu são formas de expressar os sentimentos que temos nas situações de conflito, de uma
forma não-acusatória. O contrário das mensagens-eu são as “mensagens- você”, quando nós
praticamente começamos a “acusar” injustamente as outras pessoas.
Exemplos de “Mensagens-Eu”:
• “Eu estou zangado”;
• “Estou chateado, porque você pegou meu material”;
• “Fiquei ofendido porque você me xingou”.

Exemplos de “Mensagens-Você”:
• “Você é um chato.”;
• “Você é um miserável!”;
• “Você me irrita.”
Observe que se trata, portanto, de ajudar as pessoas a se expressarem melhor, de maneira mais
construtiva. As mensagens-eu caracterizam-se, geralmente, por três fases de intervenção, a saber:
a) expressão do sentimento: eu sinto...
b) a justificativa: porque...
c) um apelo à solução: que tal se...

Procurando soluções
• Pergunte à primeira pessoa: O que você poderia ter feito de forma diferente? Parafraseie.
• Pergunte à segunda pessoa: O que você poderia ter feito de forma diferente? Parafraseie.
• Pergunte à primeira pessoa: O que você pode fazer aqui e agora para ajudar a solucionar o problema?
Parafraseie.
• Pergunte à segunda pessoa: O que você pode fazer aqui e agora para ajudar a solucionar o
problema? Parafraseie.

Apostila gerada especialmente para: Betânia Mueller 023.712.580-32


. 205
Tanto quanto possível, procure elaborar perguntas criativas para aproximar mais as pessoas de uma
eventual solução.

Alguns cuidados a serem tomados

Para mediar conflitos, devemos ter alguns cuidados e precauções em relação às atitudes que podemos
tomar. A seguir, oferecemos algumas sugestões:
- Novo olhar para a situação negativa: O conflito também pode ser uma oportunidade de
transformação. O mediador deve fazer com que as pessoas deixem de olhar apenas o lado negativo do
conflito, e evitar que a conversa tome “outros rumos” e saia do foco – que é a solução do conflito através
do diálogo e cooperação;
- Empoderamento e igualdade: O mediador deve estar sempre atento para manter uma igualdade entre
as pessoas, pois em algumas mediações uma das partes pode querer ficar mais forte do que a outra. Ao
perceber este tipo de situação, o mediador pode interromper a mediação e conversar individualmente
com as partes, não para intimidar as partes, mas para manter o equilíbrio em situações difíceis;
- Conhecimento do conflito: Na situação conflituosa, o mediador deve conhecer bem as posições e os
interesses de cada parte envolvida.
- Influência de terceiros: Às vezes, o conflito envolve não somente as pessoas envolvidas na mediação,
mas também pessoas que vivenciaram a situação e que podem de certa forma influenciar na solução do
conflito. Na mediação comunitária, é comum a presença de um terceiro que nem sempre participou de
alguma situação em torno do conflito, mas que perante as pessoas da comunidade exerce uma certa
“influência” (líder religioso, membro de associação de bairro, agente comunitário e outros), seja para
trabalhar como mediador ou conciliador.

O papel do mediador
O mediador tem papel fundamental no processo de mediação, pois ele é quem atua como o interlocutor
das partes que querem resolver os seus conflitos. A pessoa que procura mediar conflitos tem que ter
alguns requisitos tais como: ter respeito pela comunidade em que vai agir; deve conhecer bem essa
comunidade; ser uma pessoa responsável, e procurar formação permanente, estudando e pesquisando,
sempre que necessário, a respeito de novas informações. Só assim poderá aperfeiçoar a sua prática.
Listamos a seguir os princípios básicos que todo o mediador deverá perseguir em suas práticas
cotidianas:

-Sensibilidade: compreender sem tomar partido.


- Ética e conhecimento dos direitos humanos: respeito à dignidade do outro, procurando sempre
estar atento às violações de direitos humanos.
- Conhecimento básico da legislação nacional: Constituição federal, Estatuto da Criança e do
Adolescente (ECA), código civil no que se refere ao direito de família e o de vizinhança, código do
consumidor, estatuto do idoso, CLT (Consolidação das Leis do Trabalho), enfim, leis que possam
auxiliar na solução do conflito.
- Capacidade comunicativa: a comunicação é a base de uma boa mediação.
- Capacidade de escuta: escutar com atenção e respeito.
- Capacidade de manter sigilo: o sigilo só deverá ser quebrado em casos de condutas criminosas
como: homicídio, violência, abuso sexual etc.
- Criatividade: bom humor, comparações, histórias do cotidiano ajudam a “quebrar o gelo”.
- Estilo cooperativo: busca da solução do conflito de forma amistosa.

É inevitável que o mediador desempenhe a função de líder entre as partes que participam da mediação,
pois ele tem a função de facilitar o processo através do diálogo. Porém, é importante lembrar sempre que
o mediador, se não for um profissional especializado, no âmbito de suas funções pode ser uma pessoa
comum, que ajuda as pessoas envolvidas em conflito a dialogar. Ela vai ajudar estas pessoas a
procurarem uma solução para o conflito que estão enfrentando, de forma cooperativa e pacífica.

Apostila gerada especialmente para: Betânia Mueller 023.712.580-32


. 206
A Mediação como Gestão de Conflitos e a Atuação do(A) Assistente Social16.

Como vimos, a mediação é um instrumento para a resolução de conflitos. Apesar de ser uma prática
muito antiga, documentada por antropólogos como presente em todas as culturas e religiões, só
recentemente surgiu como alternativa válida no Brasil.
A mediação é um procedimento de alcance mais profundo, utilizada quando os conflitos são antigos e
crônicos e as relações têm alguma permanência no tempo ou são relações que as partes têm interesse
em manter futuramente como, por exemplo, sociedades, vizinhança, trabalhistas, religiosos, familiares.
Tem por objetivo levar as partes a uma solução do conflito que deverá resultar em um acordo que poderá
ser homologado em qualquer fase do processo judicial.
A prática da mediação se mostra mais eficaz no campo das relações familiares, de acordo com
Barbosa, “[...] é a metodologia da tática e estratégia em que a escuta entra como atividade profissional,
altamente qualificada, permitindo decodificar, imaginar e concretizar. Trata-se de uma escuta dinâmica,
que dá sentido ao silêncio, que percebe as mensagens não verbais”.
Através da prática da mediação, poderemos chegar ao estágio em que os processos de família deixem
de ser analisados pelo Judiciário como meras abstrações jurídicas. Os protagonistas das demandas
judiciais são pessoas com histórias e querem uma resposta qualitativa para seu litígio. De acordo com
Rosa:17

[...] a mediação é um processo confidencial que estabelece uma negociação conduzida pelo mediador,
que por sua vez levará os participantes da sessão de mediação a construírem um acordo, mutuamente
aceito, de forma que permita aos envolvidos no conflito dar continuidade a um tipo de relacionamento que
seja construtivo [...]

O mediador deve ser aquele capaz de gerenciar os conflitos visando à satisfação das partes
envolvidas. Por essa razão, é de grande importância a capacitação para tal exercício e a postura ética.
De acordo com Azevedo:18

[...] No exercício dessa importante função, ele [o mediador] deve agir com imparcialidade e ressaltar
às partes que ele não defenderá nenhuma delas em detrimento da outra, pois não está ali para julgá-las
e sim para auxiliá-las a melhor entender suas perspectivas, interesses e necessidades. O mediador, uma
vez adotada a confidencialidade, deve enfatizar que tudo que for dito a ele não será compartilhado com
mais ninguém [...].

O perfil do mediador, segundo Fiorelli, é o de uma pessoa que tenha formação superior, competência
interpessoal, conhecimentos mínimos de direito, sintonia cultural, imagem pública, paciência,
autoconfiança, liderança, um agente que facilita o diálogo. Durante a sessão de mediação deverá acolher,
informar, esclarecer, administrar a participação das partes e desenvolver soluções de forma cooperada.
Os princípios éticos da mediação e do Serviço Social são muito próximos, pois são pautados no
respeito à dignidade do indivíduo, e, quanto aos saberes, “[...] deve privilegiar os conhecimentos do saber
das áreas humanas”.
Com vista a essas requisições, o profissional Assistente Social tem sido chamado, assim como
psicólogos, sociólogos e outros profissionais, a participar de sessões de mediação como mediador ou
comediador na atuação social:

[...] esses espaços sócio ocupacionais, que podem ser vinculados conforme tem sido denominado no
âmbito do Serviço Social, a presença do assistente social em equipes que prestam assistência à
população vem se mostrando fundamental. Sua participação tem sido chamada para diversas ações,
desde a identificação das demandas e articulações com a rede social, até como responsável pela
organização e desenvolvimento de trabalhos voltados para a mediação, conciliação e resolução de litígios
no âmbito judicial e extrajudicial.
De acordo com Gajardoni, é fundamental a participação de assistentes sociais e psicólogos: “esses
profissionais, em razão do caráter de seu trabalho, têm extrema capacidade de lidar com assuntos
delicados como os tratados nesse circuito”.

16
Texto adaptado de BATISTA, M.; MARTINS, E. M. G.; CAMOLESI, A. B. A Atuação do Assistente Social na Mediação de Conflitos.
17
ROSA, C. P. da. A Justiça que Tarda, Falha: a Mediação como Nova Alternativa no Tratamento dos Conflitos Familiares. Revista Síntese Direito da Família.
São Paulo: Editora Síntese; 2010
18
AZEVEDO, A. G.. Manual de Mediação Judicial. Brasília/DF: Ministério da Justiça e Programa das Nações Unidas para o Desenvolvimento – PNUD - 2009,
246 p.

Apostila gerada especialmente para: Betânia Mueller 023.712.580-32


. 207
Parece que o ponto da capacidade é inquestionável, a dificuldade é problematizar como é que tal
capacidade ganha operacionalidade respeitando os compromissos com o projeto profissional.
19
Recorremos a Fávero e Mazuelos para aprofundar essa questão. As autoras pontuam que:

[...] Trabalhar na perspectiva da mediação familiar com foco no entendimento das relações sociais é
desafiante, requerendo do profissional que se encontra na linha de frente da intervenção habilidades que
vão além das técnicas de negociação.

Não é raro ouvir “um bom mediador é necessariamente um bom negociador”, o que pode ser bastante
questionável, pois mediar uma ação que envolve a guarda de filhos, por exemplo, requer capacidade de
percepção e conhecimento que vão muito além da arte de negociar.
Portanto, o profissional que faz uso do recurso da mediação estabeleça conexão com os determinantes
da questão social que estão postos na problemática, como também nas implicações do conflito no âmbito
familiar e até mesmo no território no qual vive a família. Tais fatores contribuem para uma escuta
humanizada, pois qualificam “qual território” e “que famílias” estão envolvidas no conflito.

Para elas, ao trabalhar com a mediação, a questão social pode ficar mascarada, uma vez que o
problema central pode ser entendido como restringindo-se à área psíquica, ou seja, individual. O
aprofundamento revela, no entanto, que tal problemática advém de uma situação de desemprego
agravada pela dificuldade de reinserção no mundo do trabalho.
Dessa maneira, o profissional assistente social, operando com a mediação de conflitos familiares,
articula as questões privadas trazidas pelo grupo familiar com o contexto social e econômico – esfera
macrossocial”, utilizando o espaço da mediação “para a defesa de direitos, defesa de acesso à
informação, à cidadania, à desburocratização da informação, buscando com os sujeitos possibilidades
para a efetivação de direitos”, através da compreensão e explicitação dos dilemas sociais vivenciados
pelas famílias.

Comunicação Não Violenta20

Uma Maneira de concentrar a Atenção


A CNV se baseia em habilidades de linguagem e comunicação que fortalecem a capacidade de
continuarmos humanos, mesmo em condições adversas. Ela não tem nada de novo: tudo que foi
integrado à CNV já era conhecido havia séculos. O objetivo é nos lembrar do que já sabemos - de como
nós, humanos, deveríamos nos relacionar uns com os outros - e nos ajudar a viver de modo que se
manifeste concretamente esse conhecimento.
A CNV nos ajuda a reformular a maneira pela qual nos expressamos e ouvimos os outros. Nossas
palavras, em vez de serem reações repetitivas e automáticas, tornam-se respostas conscientes,
firmemente baseadas na consciência do que estamos percebendo, sentindo e desejando. Somos levados
a nos expressar com honestidade e clareza, ao mesmo tempo que damos aos outros uma atenção
respeitosa e empática. Em toda troca, acabamos escutando nossas necessidades mais profundas e as
dos outros. A CNV nos ensina a observarmos cuidadosamente (e sermos capazes de identificar) os
comportamentos e as condições que estão nos afetando. Aprendemos a identificar e a articular
claramente o que de fato desejamos em determinada situação. A forma é simples, mas profundamente
transformadora.
À medida que a CNV substitui nossos velhos padrões de defesa, recuo ou ataque diante de
julgamentos e críticas, vamos percebendo a nós e aos outros, assim como nossas intenções e
relacionamentos, por um enfoque novo. A resistência, a postura defensiva e as reações violentas são
minimizadas. Quando nos concentramos em tornar mais claro o que o outro está observando, sentindo e
necessitando em vez de diagnosticar e ouvir nossas necessidades mais julgar, descobrimos a
profundidade de nossa própria compaixão. Pela ênfase em escutar profundamente - a nós e aos outros -
, a CNV promove o respeito, a atenção e a empatia e gera o mútuo desejo de nos entregarmos de coração.
Embora eu me refira à CNV como "processo de comunicação" ou "linguagem da compaixão", ela é
mais que processo ou linguagem. Num nível mais profundo, ela é um lembrete permanente para
mantermos nossa atenção concentrada lá onde é mais provável acharmos o que procuramos.

19
FÁVERO, T.E.; MAZUELOS, E.P.Q.. Serviço Social e acesso à Justiça – reflexões com base na prática da mediação familiar. Revista Serviço Social & Saúde
9. Campinas: Unicamp, 2010, p. 39 a 68.
20
Rosenberg. Marshall B. Comunicação não-violenta: técnicas para aprimorar relacionamentos pessoais e profissionais I Marshall B. Rosenberg ; [tradução
Mário Vilela]. -São Paulo: Ágora, 2006

Apostila gerada especialmente para: Betânia Mueller 023.712.580-32


. 208
Existe a história de um homem agachado debaixo de um poste de iluminação, procurando alguma
coisa. Um policial passa e pergunta o que ele está fazendo. "Procurando as chaves do carro", responde
o homem, que parece ligeiramente bêbado.
"Você as perdeu aqui?", pergunta o policial. "Não, perdi no beco." Vendo a expressão intrigada do
policial, o homem se apressa a explicar: "É que a luz está muito melhor aqui". Acho que meu
condicionamento cultural me leva a concentrar a atenção em lugares onde é improvável que eu consiga
o que quero. Desenvolvi a CNV como uma maneira de fazer brilhar a luz da consciência de condicionar
minha atenção a se concentrar em pontos que tenham o potencial de me dar o que procuro.
A CNV melhora a comunicação interior, ao nos ajudar a traduzir mensagens internas negativas em
sentimentos e necessidades.
Nossa capacidade de distinguir nossos próprios sentimentos e necessidades e de entrar em empatia
com eles pode nos libertar da depressão. Podemos então reconhecer o elemento de escolha em todas
as nossas ações. Ao mostrar como nos concentrarmos naquilo que realmente desejamos, em vez de
naquilo que há de errado com os outros ou com nós mesmos, a CNV nos dá as ferramentas e a
compreensão de que precisamos para criar um estado mental mais pacífico. Profissionais de
aconselhamento e psicoterapia também podem utilizar a CNV para criar relacionamentos com os
pacientes que sejam mútuos e autêntico.

Questões

01. (IFB - Assistente Social - FUNIVERSA). Com relação à mediação de conflitos, assinale a
alternativa correta.
(A) A mediação de conflitos é uma categoria ontológica.
(B) A mediação de conflitos é um processo informal em que não existe um modelo de atuação definido
com etapas, diretrizes, princípios e regras.
(C) O papel de mediador requer atributos indispensáveis como escuta e acolhimento da fala dos
mediados, imparcialidade, confidencialidade, credibilidade e diligência.
(D) Constituem regras da mediação: o respeito mútuo, a não-violência e a participação incentivada e
não-voluntária no processo de mediação.
(E) A mediação é um processo de resolução de conflitos em que uma terceira pessoa, parcial e
independente, facilita o diálogo entre as partes para que melhor entendam o conflito e procurem alcançar
soluções criativas e possíveis.

02. (UNIPAMPA - Assistente Social - CESPE). Considerando as alternativas para a resolução de


conflitos, julgue o item a seguir.
A mediação, que se caracteriza como um processo confidencial, objetiva transformar um contexto
adversarial em colaborativo.

( ) Certo ( ) Errado

03. (CONDER - Assistente Social - FGV). Com relação à política de tratamento dos conflitos de
interesses, a legislação brasileira estabelece princípios para orientar a ação dos responsáveis pela
condução dos processos de mediação e conciliação.
Assinale a afirmativa que aponta uma das características desse processo.
(A) Os conciliadores podem prestar serviços profissionais privados para uma das partes envolvida no
processo de resolução de conflitos.
(B) A manutenção do sigilo no processo de conciliação e mediação pode ser revista sempre que surjam
dúvidas quanto à veracidade dos fatos.
(C) Aos mediadores e conciliadores não se coloca a exigência de capacitação periódica e continuada
pois essa atividade não requer competências específicas.
(D) A imparcialidade dos mediadores pode acontecer quando existem diferenças sociais entre os
envolvidos nos processos de resolução de conflitos.
(E) Os conciliadores podem suspender as sessões quando acharem oportuno e identificarem algumas
dificuldades durante o processo.

Gabarito

01.C / 02.certo / 03.E

Apostila gerada especialmente para: Betânia Mueller 023.712.580-32


. 209
Comentários

01. Resposta: C
O processo de mediação é confidencial. O mediador deve manter o sigilo do que foi discutido entre as
partes, ou seja, não pode divulgar as informações discutidas durante a mediação, nem antes, durante ou
depois do processo. A confiança dos mediados no mediador também surge quando este mostra estar
comprometido com o sigilo da mediação. A juíza Ellen Gracie Northfleet, do Supremo Tribunal Federal,
afirma que:
“O clima de informalidade e confidencialidade das sessões favorecem o esclarecimento de situações
que talvez não aflorassem na sala das audiências. O diálogo que se estabelece entre as partes é mais
verdadeiro porque envolve a inteireza de suas razões e não apenas aquelas que poderiam ser deduzidas
com forma e figura de juízo”.

02. Resposta: certo


O processo de mediação é confidencial. O mediador deve manter o sigilo do que foi discutido entre as
partes, ou seja, não pode divulgar as informações discutidas durante a mediação, nem antes, durante ou
depois do processo. A confiança dos mediados no mediador também surge quando este mostra estar
comprometido com o sigilo da mediação.

03. Resposta: E
O mediador deve estar sempre atento para manter uma igualdade entre as pessoas, pois em algumas
mediações uma das partes pode querer ficar mais forte do que a outra. Ao perceber este tipo de situação,
o mediador pode interromper a mediação e conversar individualmente com as partes, não para intimidar
as partes, mas para manter o equilíbrio em situações difíceis.

Grupo social e familiar: influência da família e da sociedade no desenvolvimento


da personalidade e modalidades de intervenção. Norma, disciplina e poder.

Família ou famílias?21

Todos nós temos uma imagem ideal de família. Esta família ideal, descrita por Goldani, é formada por
um casal heterossexual (ou seja, por um homem e uma mulher), legalmente casados, com dois filhos (um
de cada sexo), e todos vivendo juntos e em uma casa própria. O marido tem emprego fixo e salário
suficiente para prover todas as necessidades da família, portanto ele é, neste modelo, o principal
provedor. A mulher é a dona de casa, responsável pela economia doméstica, pela organização da casa,
pelo cuidado com as crianças, os adolescentes, os idosos e os doentes. Neste contexto, quando a mulher
trabalha fora de casa, é em tempo parcial e para “ajudar” o marido. Outra alternativa para a
complementação da renda familiar é a mulher ocupar-se de alguma atividade de geração de renda
(costurar, fazer salgadinhos, doces, entre outras) na sua própria casa. Se ela trabalhar fora de casa em
tempo integral, ainda assim é esperado que ela realize as atividades domésticas perfeitamente bem, afinal
estas atividades são entendidas como próprias das mulheres. As crianças e os adolescentes frequentam
regularmente a escola e a exploração do trabalho infanto-juvenil não existe. Esta família está livre de todo
o conflito e, mais ainda, de qualquer violência entre o marido e a esposa, entre pais e filhos ou entre
irmãos. Este texto poderia ser o do último capítulo de uma novela, porque retrata uma família idealizada
como feliz!

As famílias brasileiras ao longo da nossa história foram se transformando: a frequência dos


casamentos legais diminuiu, o número de separações aumentou, o de recasamentos cresceu, outros
arranjos sexuais na formação dos casais são experimentados, o número de nascimentos diminuiu, assim
como o número de filhos por casal. Um grupo expressivo de adolescentes torna-se mãe, algumas
mulheres vão experimentar a maternidade pela primeira vez após os 30 anos. Há homens e mulheres
que escolhem a paternidade e a maternidade, mas não o casamento, assim como há homens e mulheres
que escolhem
- se observarmos as famílias do nosso bairro, da nossa cidade, do nosso país, vamos encontrar alguma
bem parecida com este modelo?

21
Texto adaptado de MOREIRA, M. I. C. Novos rumos para o trabalho com famílias. Secretaria de Estado de Desenvolvimento Social do Governo do Estado
de Minas Gerais. Belo Horizonte. Dezembro 2008.

Apostila gerada especialmente para: Betânia Mueller 023.712.580-32


. 210
Provavelmente, sim.
- mas todas as famílias do nosso bairro, da nossa cidade, do nosso país são iguais a este modelo?
Com certeza, não. Por quê?

O casamento, mas não a maternidade e a paternidade. Houve um aumento significativo de mulheres


que conquistaram grau de escolaridade mais alto, ingressaram no mercado de trabalho, exercendo as
mais diferentes profissões, e têm ainda de conciliar estas atividades com o trabalho doméstico. Há
homens que se voltaram para as ocupações antes consideradas próprias das mulheres e alguns que
assumiram as tarefas domésticas integralmente, enquanto suas companheiras trabalham fora de casa,
ou parcialmente, dividindo com elas o cuidado com a casa e os filhos. Muitos lares brasileiros são
chefiados e sustentados pelas mulheres; outros, nos quais os homens estão desempregados, os
responsáveis por prover a família são as mulheres, as crianças e os adolescentes. Atualmente, homens
e mulheres buscam estabelecer relações mais igualitárias entre si. As crianças, os adolescentes e os
jovens são considerados sujeitos de direitos, e não “menores”.
Tratados quase como propriedade dos pais e dos adultos. Eles buscam ser ouvidos tanto em casa, na
família, quanto no grupo social mais amplo, e suas opiniões e escolhas têm sido levadas em conta, em
graus variados. Se perguntarmos aos mais velhos, pessoas com idade superior a 70 anos, sobre a relação
que eles, quando crianças, estabeleciam com seus pais, avós e tios, provavelmente vamos escutar que
tudo era muito diferente dos dias atuais, que as crianças deveriam “obedecer”, e não “se intrometer em
conversa de adulto”.
Todas estas transformações na família estão relacionadas às mudanças na estrutura econômica e
política do nosso país, e também às mudanças de valores que foram construídas ao longo das gerações.
Todos nós conhecemos famílias formadas por pai, mãe e filhos; mas também famílias formadas pela
mãe com seus filhos; pelo pai com seus filhos; pela mãe com seus filhos, a avó e os tios; pela mãe com
seus filhos e seu companheiro; ou pelo pai com seus filhos e sua companheira, entre outros formatos.
Também quando assistimos à televisão, vemos nas novelas muitas formas de organização familiar que
refletem o que acontece na sociedade. Todos estes fatos demonstram que a família tem uma enorme
capacidade de adaptação e de mudança ao longo da história.
As multifamílias consistem em uma habitação multifamiliar, ocupada por várias pessoas.

Existem muitos modelos de família que convivem no mesmo espaço social e ao mesmo tempo:
- Nuclear simples: família em que o pai e a mãe estão presentes no domicílio; além disso, todas as
crianças e os adolescentes são filhos desse mesmo pai e dessa mesma mãe. Não há mais qualquer
adulto ou criança (que não sejam filhos) morando neste domicílio.
- Monoparental feminina simples: família em que apenas a mãe está presente no domicílio vivendo
com seus filhos e, eventualmente, com outras crianças e adolescentes sob sua responsabilidade. Não há
mais nenhuma pessoa maior de 18 anos, que não seja filho, morando no domicílio.
- Monoparental masculina simples: família em que apenas o pai está presente no domicílio vivendo
com seus filhos e, eventualmente, com outras crianças e adolescentes sob sua responsabilidade. Não há
mais nenhuma pessoa maior de 18 anos, que não seja filho, morando no domicilio.
- Monoparental feminina extensa: família em que apenas a mãe está presente no domicílio vivendo
com seus filhos e ainda com outras crianças e adolescentes sob sua responsabilidade e outros adultos
sem filhos menores de 18 anos, parentes ou não.
- Monoparental masculina extensa: família em que apenas o pai está presente no domicílio vivendo
com seus filhos e ainda com outras crianças e adolescentes sob sua responsabilidade e outros adultos
sem filhos menores de 18 anos, parentes ou não.
- Nuclear extensa: família em que o pai e a mãe estão presentes no domicilio vivendo com seus filhos
e outras crianças e adolescentes sob sua responsabilidade, além de outros adultos, parentes ou não do
pai e/ou da mãe.
- Família convivente: famílias que moram juntas no mesmo domicílio, sendo ou não parentes entre
si. Cada família pode ser constituída por “pais-mãe-filhos”, por “pai-filhos”, ou por “mãe-filhos”. Outros
adultos sem filhos, parentes ou não, podem também viver no domicílio. Nessa categoria foram também
agrupadas as famílias compostas de duas ou mais gerações, desde que, em cada geração, houvesse
pelo menos uma mãe ou um pai com filhos até 18 anos.
- Família nuclear reconstituída: família em que o pai e/ou a mãe estão vivendo uma nova união, legal
ou consensualmente, podendo também a companheira ou o companheiro ter filhos com idade até 18
anos, vivendo ou não no domicílio. Outros adultos podem viver no domicílio.
- Família de genitores ausentes: família em que nem o pai nem a mãe estão presentes, mas na qual
existem outros adultos (tais como avós, tios) que são responsáveis pelos menores de 18 anos.

Apostila gerada especialmente para: Betânia Mueller 023.712.580-32


. 211
- Família nuclear com crianças agregadas: família em que o pai e a mãe estão presentes no
domicílio com seus filhos e também com outras crianças e adolescentes sob sua responsabilidade. Não
há outro adulto morando no domicílio.
- Família colateral: aquela composta por irmãos e irmãs maiores e/ou menores de 18 anos, sem a
presença dos pais, de qualquer outro parente ou adulto não parente.
- Família homoparental: aquela composta por um casal formado de pessoas do mesmo sexo, com
os filhos nascidos de uniões heterossexuais anteriores, de um ou de ambos os parceiros, ou com filhos
adotados menores de 18 anos.
Esta lista não é definitiva, porque, como já vimos, as famílias são dinâmicas e se transformam ao longo
da história. O objetivo desta lista é mostrar a diversidade de organização e dinâmica familiar. Esta
diversidade traz enormes desafios para todos nós, especialmente para os que trabalham diretamente com
as famílias, realizando cotidianamente as políticas de assistência social que visam à promoção e ao
exercício da cidadania de homens e mulheres, à proteção dos direitos de crianças, adolescentes, jovens
e idosos.

Funções da família

- A proteção de seus membros, em especial as crianças, os adolescentes, os jovens e os idosos. Para


que esta função se cumpra, os membros da família negociam internamente a divisão de papéis e de
tarefas. Além disso, a família pode necessitar de uma série de equipamentos sociais, públicos ou
privados, que a auxiliem a desempenhar suas funções.
- A divisão do trabalho e a divisão interna de papéis que facilitem a sobrevivência do grupo e a
manutenção do domicílio.
- A aprendizagem dos papéis de homens e mulheres, adultos e crianças. Estes papéis são variados
entre as diversas sociedades e culturas.
- A vivência da sexualidade e da afetividade. As famílias são organizadas por regras que regem a
vivência da sexualidade e da afetividade, compartilhadas pela sociedade em geral. O incesto é proibido,
o abuso sexual contra crianças e jovens deve ser combatido. Quando estes fatos acontecem, são tratados
como violação das leis e dos direitos individuais, e os autores de tais atos são responsabilizados e
punidos.
- As relações entre parentes. Cada membro experimenta vários tipos de relações dentro do seu grupo
familiar. Por exemplo, a relação entre pais e filhos é diferente daquela vivida entre irmãos. Existem
expectativas distintas sobre cada membro do grupo familiar e seus parentes: o grau de obediência e
solidariedade devida, os limites da autoridade de cada um, a regulação das trocas afetivas, entre outras.
Nesse conjunto é que serão definidas as relações entre mães-filhos, pais-filhos, genros-sogros, tios-
sobrinhos, irmãos-irmãs.

Famílias em situação de risco

A palavra RISCO tem muitos sentidos: aventura, incerteza, imprudência, imprevidência, perigo, entre
outros. Nos jornais, a palavra RISCO muitas vezes vem acompanhada de outras: investimento de risco
(na economia); esporte de risco (o alpinista radical que escala a montanha mais alta do mundo);
comportamento de risco (na área da saúde).
O RISCO envolve dimensões negativas e positivas. Na época das chuvas, são divulgadas as
condições das áreas de RISCO de uma cidade que devem ser evitadas porque são perigosas e os
deslizamentos de terra podem causar mortes. Esta é uma dimensão negativa.
O RISCO-aventura está ligado a uma ideia de superar obstáculos e vencê-los. No esporte radical, há
risco, e vencê-los é o objetivo do esportista. O investidor de valores corre RISCO para ganhar dinheiro.
Há profissões de RISCO, como a dos bombeiros, que são muito valorizadas pela população.
A ideia do risco está relacionada ao planejamento do futuro e à invenção de meios para avaliar os
riscos, com o objetivo de preveni-los e controlar as situações que produzem riscos.
No campo das políticas públicas voltadas para as famílias e suas crianças e seus adolescentes,
encontramos frequentemente as expressões “famílias em situação de risco social” e “crianças e
adolescentes em situação de risco pessoal e social”. Qual o sentido deste RISCO?
As famílias em situação de risco pessoal e social têm sido descritas como famílias cujos membros
apresentam baixo grau de escolaridade e recursos econômicos e culturais limitados ou precários. Além
disso, os estudiosos e os trabalhadores sociais que lidam diretamente com as famílias em situação de
risco têm percebido que tais famílias mostram padrões de comunicação e de socialização difíceis em
relação às crianças, o que muitas vezes leva ao risco da violência ou da fragilização dos vínculos.

Apostila gerada especialmente para: Betânia Mueller 023.712.580-32


. 212
Não se trata de afirmar que somente as famílias pobres estejam em risco pessoal e social, e sim que
as condições exacerbadas de carência material e cultural são um fator que contribui para a precarização
das relações afetivas.
As famílias vivem em um contexto social mais amplo e, neste sentido, é preciso considerar que a
precarização do mundo do trabalho - quer dizer, a diminuição dos postos de trabalho em todo o mundo,
as longas jornadas, a diminuição dos salários, aliadas à baixa escolaridade e formação profissional - são
determinantes na produção da pobreza e da exclusão social.
Muitas vezes, o risco só é percebido quando um problema se manifesta.

Exemplo:
Situação: a criança fora da escola.
É um problema, uma violação do direito à educação da criança, previsto no ECA.
É um risco imediato, porque, quando não está na escola, a criança pode estar na rua exposta a toda
sorte de violência, ser explorada tendo de trabalhar, seja no espaço público ou doméstico, e também é
um risco futuro, porque a baixa escolaridade vai levá-la a postos precários de trabalho, à exclusão social
e à perpetuação da pobreza.
É preciso verificar por que aquela criança está fora da escola. Esta situação pode revelar uma série
de problemas:
- não há vagas na escola?
- há vaga em uma escola muito distante de sua casa?
- a criança não foi matriculada porque não tem Certidão de Nascimento?
- a criança não frequenta a escola porque trabalha?
- a criança não frequenta a escola porque está doente?
Examinando o problema atual de cada família, encontramos a situação de risco que o gerou, assim
como o problema atual gera outros riscos futuros. A expressão círculo vicioso dá a ideia dessas relações
entre os problemas atuais e os riscos futuros.

Enfrentando os riscos

Para enfrentar os riscos, é preciso acionar a rede, estabelecer parcerias com as instituições públicas
e comunitárias, e, mais importante, considerar a potencialidade das próprias famílias. Elas precisam ser
acolhidas, pois são pontos da rede de proteção às crianças e aos adolescentes.

Ações primárias

Elas têm um caráter preventivo. Os espaços da escola, da unidade de saúde, das igrejas, dos centros
de convivência oferecem múltiplas possibilidades para os encontros entre as famílias.
- Rodas de conversação sobre temas de interesse das famílias: educação dos filhos, saúde,
sexualidade, construção de autonomia e de limites, entre tantos outros.
-Transmissão de habilidades, ou seja, encontros intergeracionais para o compartilhamento de receitas
culinárias, formas de cultivo de hortas domésticas, confecção de brinquedos, brincadeiras, artesanato.
- Exibição de filmes, seguida de debate, de compartilhamento da apreciação do filme.
- Conferências planejadas, de modo a permitir a participação de todos.
- Programas de geração de renda: inclusão dos adultos da família em cursos de capacitação para o
trabalho e em iniciativas para a geração de renda.
Estas são algumas sugestões para que as relações comunitárias, estabelecidas pelas famílias, façam
parte do cotidiano do bairro, da vila, da comunidade.

Ações secundárias

Elas têm caráter de restauração, de reparação de um dano causado nos vínculos familiares entre pais
e filhos. Podemos entender a medida socioprotetiva de acolhimento institucional de crianças e
adolescentes como uma ação secundária que visa proteger a integridade física, emocional e moral da
criança e do adolescente e estabelecer mediações com as instituições operadoras do Direito, a própria
família e a comunidade, para que a situação de violação dos direitos seja superada. Tomada
isoladamente, a medida de acolhimento é limitada e precisa estar associada a outras ações.
- Atendimento especializado para o caso de os membros da família usarem ou abusarem de drogas
lícitas ou ilícitas; e psicoterápico para a criança e o adolescente.
- Atendimento psicoterápico para a família.

Apostila gerada especialmente para: Betânia Mueller 023.712.580-32


. 213
- Atendimento psicoterápico para a criança e o adolescente.
- Capacitação dos trabalhadores sociais envolvidos no acolhimento de crianças, adolescentes e seus
familiares.

Ações terciárias

As ações terciárias têm a conotação de reabilitação, uma vez esgotadas as possibilidades de


prevenção e restauração dos vínculos partidos com a família de origem ou extensa. É preciso construir
as possibilidades para a formação de novos vínculos, tanto com a família substituta quanto com os novos
espaços comunitários, e ainda ter cuidado com o risco de institucionalização permanente de crianças e
adolescentes.
- Possibilitar a aproximação com as famílias substitutas.
- Atendimento psicoterápico para a criança e ao adolescente.

Família contemporânea

A família passou por várias transformações na segunda metade do século XIX: ampliou-se a
participação das mulheres no mercado de trabalho e nas universidades, observando-se as diferenças
entre as classes sociais, pois as mulheres mais pobres, em sua grande maioria, continuavam com pouca
qualificação e baixa escolaridade.
O casamento sofreu transformações, sendo efetivado através da anuência do casal, com a livre
escolha do parceiro, e norteado principalmente pela afinidade entre o casal e por fatores afetivos e
emocionais com bases no amor romântico.
A tradicional família nuclear apresenta transformações. Além do pai, a esposa e os filhos inserem-se
no mercado de trabalho, auxiliando nas despesas e na manutenção da família, alterando padrões de
hierarquia, autoridade e sociabilidade.
Surge a família contemporânea que é construída através de uma somatória de experiências e
trajetórias particulares, manifestando-se através de arranjos familiares diferenciados e peculiares,
denotando a impossibilidade de identificá-la como um padrão familiar uniforme e ideal.
Dentre os vários arranjos familiares, podem-se citar as famílias:

-reconstituídas, que são aquelas em que, após a separação conjugal, o indivíduo constitui uma nova
família;
- constituídas através de uniões estáveis;
- monoparentais femininas;
- monoparentais masculinas;
- nas quais avós moram e cuidam de seus netos;
- unipessoais;
- formadas por uniões homossexuais;

O modelo idealizado da família nuclear burguesa ainda perpetua no imaginário do indivíduo - coletivo.
Famílias que não fazem parte desse tipo de arranjo familiar tentam ‘adaptar’ e aproximar a sua estrutura
e padrão de funcionamento ao do ‘modelo ideal’, acreditando, muitas vezes, que não constituem uma
família, ou que a sua família é ‘errada’, quando não conseguem reproduzi-lo.
No início do século XXI, percebem-se as metamorfoses nas famílias: a diminuição do número de filhos,
a redução de número de matrimônios realizados legalmente (casamento civil), o aumento de separações
e divórcios. A divisão sexual dos papéis, ou seja, as funções socialmente destinadas aos homens e
mulheres nas famílias são questionadas, não havendo mais a rígida separação dos papéis, demonstrando
uma estrutura mais aberta e flexível.
Dados do IBGE - Instituto Brasileiro de Geografia e Estatística - apontam outras transformações
ocorridas na família contemporânea, entre as quais podem-se citar: a redução do tamanho das famílias,
o aumento de número de famílias chefiadas por mulheres e das unidades unipessoais, entre outros.
Esses dados demonstram que a família brasileira está moldando uma nova cultura e um novo modo
de viver, que não se adequam mais aos modelos patriarcal e burguês, pois em seu tecido familiar estão
imbricados as suas próprias particularidades, peculiaridades e valores.
Outro aspecto relacionado à família é que a mesma não é estática. A família possui um ciclo vital, é
dinâmica e se apresenta de forma diferenciada de acordo com a sua evolução, ou seja: ela também
nasce, cresce, amadurece, habitualmente se reproduz em novas famílias, encerrando seu ciclo vital com

Apostila gerada especialmente para: Betânia Mueller 023.712.580-32


. 214
a morte dos membros que a originaram e a dispersão de seus descendentes para constituir novos núcleos
familiares).
Desse modo a família é influenciada pelas situações internas que lhe sucedem como: nascimento,
casamento, morte de seus membros e por fatores externos: sociais, econômicos, culturais, entre outros.

Família e Estado

A família enquanto unidade integrante da sociedade é diretamente influenciada pela situação macro
sócio-política e econômica do país. As ações estatais atingem o microssistema familiar, atuando como
fatores desagregador e propiciador de situações de vulnerabilidade, sendo que esta última está
intrinsecamente relacionada às condições econômicas das famílias, a forma de distribuição de renda no
país e ao modo de funcionamento do modelo econômico vigente - o capitalismo.
A crise do Estado-Providência, as transformações em curso no mundo do trabalho e do capitalismo,
introduzem no cenário mundial a ideologia neoliberal e a globalização, sendo esta última um processo
pelo qual se busca a união dos mercados de diversos países, através da internacionalização do capital e
da economia, objetivando o lucro rápido.
O neoliberalismo surge na década de 70, inicialmente na Inglaterra e Estados Unidos, em decorrência
das crises do petróleo e da emergência da chamada Terceira Revolução Industrial, como uma reação
conservadora à presença do Estado nas esferas social e econômica e, aos poucos, vai se estendendo
aos países de outros continentes. Na década de 80, têm-se as primeiras marcas do neoliberalismo na
América Latina: México, Argentina, Venezuela e, mais recentemente no Brasil.
Abreu afirma que o neoliberalismo é uma ideologia capitalista que defende o ajuste dos Estados
Nacionais às exigências do capital transnacionalizado, portanto contrária aos pactos que subordinam o
capital a qualquer forma de soberania popular ou instituições de interesse público.
As vertentes orientadoras do neoliberalismo são derivadas do liberalismo clássico: promoção pelo
mérito, mercado auto regulador, desconfiança à intervenção do Estado, igualdades de chance para todos
os indivíduos.
A ideologia neoliberal pressupõe que a ação espontânea do mercado deve possibilitar um equilíbrio
de condições entre os indivíduos, de tal modo que qualquer pessoa pode conseguir seus objetivos através
da livre concorrência e livre escolha.
A questão da assistência, segundo a visão neoliberal, é encarada como um dever moral, sendo
estabelecido um limite: que esta não se transforme em direito para as classes subalternas, para os
empobrecidos, miseráveis e excluídos. A política de assistência social é utilizada apenas em situações
emergenciais, visando o atendimento somente do ‘mais pobre dos pobres’, sendo que o Estado atende
apenas o que a sociedade civil, as instituições filantrópicas e o voluntarismo não atende.
Os últimos governos do Estado brasileiro, Fernando Collor de Mello e Fernando Henrique Cardoso
foram gradativamente implementando planos de ajustes nacionais que se adequavam à ideologia
neoliberal. O atual governo, também compactua com esta ideologia.
Desse modo, no cenário nacional continua sendo implementadas ações governamentais que
submetem o Estado brasileiro à dinâmica da globalização capitalista. Abreu aponta as seguintes:

- a redução de capital público destinado as áreas de saúde, educação, transporte, entre outras,
incentivando o desenvolvimento de serviços privados nestas áreas;
- a desregulação das relações de trabalho, transferindo-as do setor público e submetendo-as às
condições da iniciativa privada;
- a privatização e transnacionalização das empresas públicas;
- a liberação do comércio com a eliminação das barreiras alfandegárias;
- a desregulamentação da entrada e saída de capitais estrangeiros.

Com essas práticas, as classes dirigentes brasileiras estão cumprindo as exigências impostas pelos
personagens que ditam os rumos da ordem mundial: o capital financeiro especulador, os grandes
banqueiros credores do Estado, o capital multinacional, organismos capitalistas internacionais e os
governos do G7 (grupo dos sete Estados mais ricos do mundo comandados pelos EUA).
A adequação dos países a uma ideologia neoliberal é condição para que o Estado continue integrado
às relações políticas e econômicas internacionais.
A entrada do neoliberalismo e da globalização no cenário mundial traz como consequências o
surgimento de expressões da questão social generalizadas, que assumem uma amplitude global e
produzem efeitos comuns nas diversas partes do mundo, tais como: desemprego estrutural, aumento da

Apostila gerada especialmente para: Betânia Mueller 023.712.580-32


. 215
pobreza e da exclusão social, precarização e casualização do trabalho e desmonte de direitos sociais
edificados há mais de um século.
No Brasil, as tendências políticas em relação ao tratamento da questão social levam a políticas
setoriais e fragmentadas, que procuram atender apenas situações emergentes, visando amenizar os
impactos das demandas sociais.

A ideologia neoliberal reinante apresenta um Estado desterritorializado e sujeito a comandos dos


detentores do capital externo e, desse modo, impossibilitado de exercer o controle sobre as políticas
econômicas e sociais internas e de proteger o emprego e a renda de sua população, aliado ao predomínio
do Estado Mínimo, caracterizado pela desresponsabilizarão do governo com os setores públicos,
principalmente a assistência social e a seguridade.
As modalidades de proteção se apoiam em pilares da flexibilização das relações de trabalho, na
seletividade ou focalização das políticas sociais e na desobrigação do estabelecimento dos mínimos
sociais como direitos de todos.
Os princípios neoliberais apontam para o desmonte das políticas nacionais de garantias sociais
básicas, cujas principais implicações são: cortes de programas sociais (inclusive os voltados para as
populações empobrecidas), diminuição dos benefícios da seguridade social, criminalização da pobreza e
a valorização de velhas fórmulas de ajuda social, maquiadas pelo discurso da solidariedade e da
humanidade, que são travestidas com nova ‘roupagem’ e, por isso, veiculadas como modernas e
avançadas.
É o caso da filantropia social e empresarial, das ações voluntárias e da ajuda mútua, que
involuntariamente tendem a contribuir para a desresponsabilização do Estado perante ao provimento dos
mínimos sociais e a proteção à família, à maternidade, à infância, à adolescência e à velhice, como
expressa a Lei Orgânica de Assistência Social, de 1993.
A globalização, no Brasil, apresenta em sua face a marca da desregulamentação da força de trabalho,
o achatamento dos salários e o aumento do desemprego, contribuindo, assim, para o aumento da
exclusão social.
A exclusão não é mais vista como um fenômeno de ordem individual mas, social, cuja origem deveria
ser buscada nos princípios mesmos do funcionamento das sociedades modernas.
O fenômeno da exclusão social não envolve apenas o caráter econômico da pobreza, supõe, também,
o preconceito e a discriminação.
A exclusão social é aqui entendida como uma situação de privação coletiva que inclui pobreza,
discriminação, subalternidade, a não - equidade, a não acessibilidade, a não - representação pública
como situações multiformes.
Surge, assim, o fenômeno histórico do final do século, chamado, segundo Cristovam Buarque, de
apartação social. Trata-se da naturalização da desigualdade social, criando uma barreira entre o mundo
dos ricos e dos pobres, ocasionando a culpabilização do pobre por sua pobreza.
No início do século XXI, o que se apresenta, do ponto de vista social, é o crescimento da pobreza, do
desemprego e da exclusão, ao lado de uma enorme concentração de renda e de riqueza por parte de
uma parcela minoritária da população.

Considerando-se a pobreza não como sinônimo de ‘insuficiência de renda’, mas como a pobreza
socioeconômica, ou seja, a pobreza material, originada historicamente do modo em que a sociedade se
organiza. É a desigualdade produzida economicamente que se “manifesta de modo quantitativo, ou seja,
na falta de renda, de emprego, de habitação, de nutrição, de saúde”. Essa pobreza atinge grande parte
do contingente populacional do país.
Na pobreza não encontramos somente o traço da destituição material, mas igualmente a marca da
segregação, que torna a pobreza produto típico da sociedade, variando seu contexto na história, mas se
reproduzindo na característica de repressão do acesso às vantagens sociais.
Esse panorama denuncia o retraimento do Estado, a ausência de políticas de proteção social as
famílias pertencentes as camadas sociais de baixa renda e as consequências no mundo do trabalho.
Nos anos 70, Castel afirma que ao trabalho está vinculada uma série de garantias, direitos e proteções
sociais, sendo atribuída estabilidade e status ao trabalhador. Há uma seguridade social interligada ao
trabalho, o aspecto dos direitos vinculados ao trabalho é que fez com que o trabalho não fosse apenas a
retribuição pontual de uma tarefa, mas que a ele fossem vinculados direitos.
Com o advento da internacionalização do mercado, da globalização e dos princípios de concorrência,
eficiência e lucratividade, o trabalho passa a ser alvo de redução de custos.
Para reduzir os custos, ocorre a flexibilização do mercado de trabalho, associada à desproteção e à
desmantelação dos direitos trabalhistas. Essa flexibilização pode ser interna, ou seja, a que impõe a

Apostila gerada especialmente para: Betânia Mueller 023.712.580-32


. 216
adaptabilidade da mão-de-obra a essas situações novas e que, evidentemente, expulsa os que não são
capazes de se prestar a essas novas regras do jogo.

Ou externa que se refere à subcontratação de mão-de-obra em condições mais precárias, salários


baixos e menor proteção.
Segundo Castel, a precarização do trabalho alimenta o desemprego, pois torna-o cada vez mais
fragilizado e obriga os indivíduos a entrarem em uma situação de vulnerabilidade. Tal situação atinge de
forma diferenciada as camadas sociais e as famílias brasileiras.
As políticas sociais apresentam-se incapazes e ineficazes para atender a demanda populacional de
miseráveis e excluídos, tornando-se, desse modo, focalista, residual e seletista, ou seja, são orientadas
por uma perspectiva de se atender somente a pobreza absoluta, limitando-se a ações minimalistas,
pontuais e descontínuas, que excluem cidadãos que por direito deveriam ter acesso a recursos e
benefícios.
A família, enquanto instituição inserida na sociedade, é afetada por esse processo de desenvolvimento
socioeconômico e pelo impacto da ação do Estado através de suas políticas econômicas e sociais.
A conjuntura político-econômica brasileira, norteada pelos princípios neoliberais e pela globalização,
promove o aceleramento do empobrecimento, desemprego, minimização das políticas sociais oferecidas
às comunidades, influenciando a estrutura familiar em suas relações, estrutura, papéis e formas de
reprodução e contribuindo para a desagregação dessa instituição.

O Serviço Social e o Trabalho com a Família22

A família aparece como demanda para o Serviço Social quando ocorre algum problema ou conflito na
função social, ou seja, quando a família por um certo motivo não consegue cumprir o seu papel.
Deve-se apreender a família do ponto de vista teórico com um pensamento crítico, desvelando a
realidade, analisando as relações de totalidade e principalmente considerando as determinações
históricas, para não culpabilizá-la e nem fazer uma psicologização das questões que são sociais.
Para se ter uma visão crítica de família é preciso analisá-la como uma construção histórica, como
apontamos no segundo item deste trabalho.
A ação do Assistente Social deve ser transformadora, buscando a emancipação e o
autodesenvolvimento da família. O profissional deve atuar nas demandas, essas demandas deverão
providenciar respostas, as demandas institucionais que são demandas objetivas, imediatas, devem ser
respondidas com o desenvolvimento e a utilização de instrumentos (meios) para atingir seus objetivos,
estes instrumentos podem ser: os bens, serviços, benefícios, programas e projetos, porém o âmbito da
ação profissional deve transcender a demanda institucional, passando assim para a demanda sócio
profissional, compreender as demandas na sua totalidade, as suas contradições, a sua relação com a
sociedade e assim o Assistente Social deve articular, criar meios para que família crie condições para
cumprir a sua função social.

O Assistente Social como um profissional que tem como seu objeto de intervenção as necessidades
sociais, deve intervir nas expressões da Questão Social. Estas expressões da Questão Social rebatem
no campo de trabalho como uma consequência do sistema que fundamenta o capitalismo, aparece no
sujeito individual e/ou coletivo em situação de vulnerabilidade social e pessoal, e é no âmbito da família
que se encontram o maior número de demandas, e é nela também que deve estar a ação do Assistente
Social.

O profissional deve em sua ação desnaturalizar todas as formas de discriminação, promovendo


também a garantia dos direitos dos cidadãos e possibilitando a sua autonomia como está previsto no
projeto ético-político profissional. É necessário também que este articule junto ao Estado, as organizações
que tenham o mesmo objetivo diante desta situação, para juntos buscar a edificação as respostas políticas
que garantam direitos e que esses sejam efetivados.
Através da gestão democrática, o trabalho do Assistente Social pode contribuir com a justiça e a
equidade social a favor da universalidade das políticas sociais, posicionando seus programas, serviços e
projetos e desenvolvendo ações que venham aumentar os recursos para que se tenha uma concretização
dessas políticas de forma eficaz, o profissional deve agir juntamente ao seu usuário, fazer reuniões com
todos membros da família, as crianças, o adolescente, os pais, a mulher, o homem, o idoso, enfim

22
Texto adaptado de SILVA, J. C. M. Assistente Social.

Apostila gerada especialmente para: Betânia Mueller 023.712.580-32


. 217
trabalhar com eles as questões de gênero, questões geracionais, e outras de acordo com a necessidade
da população, sempre criando meios para que eles mesmos criem os seus valores
A tarefa do Assistente social é lutar pela participação social, emancipação, autonomia (ética, política,
moral, cultural), desenvolvimento dos sujeitos sociais, e principalmente pela ampliação dos direitos sociais
e da cidadania, investindo assim nas potencialidades dos usuários, caminhando sempre na busca da
liberdade política, econômica e cultural.

Este profissional deve agir com sua formação embasada nas diretrizes curriculares, com seu
fundamento na teoria social crítica. O profissional deve ter um perfil teórico-crítico (tem que ter capacidade
para fazer uma leitura crítica da realidade), técnico-operativo (profissional interventivo, que tem um
arsenal de técnicas e instrumentos que possibilitam a intervenção) e ético-político (o agir tem uma
intenção, tem valores do código de ética). Tendo assim um práxis transformadora que supere o
imediatismo.

Estratégias para a intervenção junto às famílias atendidas pelos diversos programas

A entrevista

As entrevistas com as famílias podem ser feitas em vários momentos do processo de trabalho. Na fase
inicial, a realização da entrevista com pais, tios, avós ou responsáveis pelas crianças e pelos adolescentes
objetiva uma aproximação com a família que possibilita conhecer os modos próprios de organização da
vida cotidiana, as dificuldades vividas e as formas de enfrentamento dessas dificuldades. Podemos
também avaliar as demandas da família, suas expectativas e disponibilidades.
A entrevista é ainda uma oportunidade de percebermos o grau de conhecimento da família sobre os
equipamentos sociais e públicos disponíveis em sua comunidade, além da inclusão e frequência de seus
membros nos programas sociais oferecidos.
A entrevista é uma situação de interação entre, no mínimo, duas pessoas. É também um ótimo recurso
para resgatar a história familiar. Mas ela não deve parecer um “interrogatório policial”, e sim uma
conversação na qual o entrevistado se sinta acolhido e escutado.
Escutar é a palavra-chave na entrevista, não apenas escutar o que seu entrevistado diz com palavras,
como também com o corpo. O corpo fala!
Para escutar, é preciso uma postura de interesse pelo outro, considerar o que está sendo dito. Não é
necessário que o entrevistador fique calado o tempo todo, é possível estabelecer um diálogo.
Ambiente: imagine uma entrevista em local sem privacidade alguma, sala barulhenta ou onde algumas
pessoas transitam ou interrompem constantemente. A situação fica tensa, o entrevistado irá sentir-se
constrangido e será difícil estabelecer um laço de confiança entre o entrevistado e o entrevistador. Por
isso, é fundamental preparar o ambiente para a realização da entrevista: uma sala silenciosa onde o
entrevistado e o entrevistador tenham privacidade e não sejam interrompidos. A preparação do ambiente
revela a disposição de acolher as pessoas.
Esclarecimento: é preciso esclarecer o entrevistado quanto aos motivos e objetivos pelos quais ele foi
convidado para a entrevista. O entrevistado, seja qual for o fato que motivou a entrevista, precisa
compreender a razão de sua presença naquele lugar e se sentir à vontade para responder o que lhe for
perguntado.
Atitudes do entrevistador: o entrevistador deve ser pontual e saber o nome do entrevistado.
Todos nós gostamos quando somos chamados pelos nossos nomes, afinal este é um sinal de
reconhecimento. A postura do entrevistador é importante, ele deve procurar olhar para o entrevistado e
desta forma comunicar que está interessado na sua narrativa.
Anotações: a anotação, durante a entrevista, pode inibir o entrevistado; se for necessário anotar
alguma informação, peça-lhe licença para fazê-lo. Terminada a entrevista, tente lembrar o que foi dito e
anotar o que poderá ser útil para o acompanhamento posterior daquela família.
É possível realizar várias sessões de entrevista com a mesma família. Em alguns momentos, podemos
começar entrevistando um membro da família, aquele que demandou a entrevista ou que foi encaminhado
para ela, e depois marcar um encontro com todo o grupo familiar.
Podemos também realizar entrevistas coletivas, ou seja, com todos os membros da família ao mesmo
tempo. No caso de haver crianças pequenas, podemos deixar, na sala da entrevista, alguns brinquedos,
papel e giz de cera para que elas desenhem. Os brinquedos e o desenho são os meios de expressão das
crianças.
Para a realização de uma entrevista, é preciso ter alguns cuidados.
- Elaborar um roteiro prévio: qual o tema da entrevista e quais dados pretende-se obter.

Apostila gerada especialmente para: Betânia Mueller 023.712.580-32


. 218
No entanto, o roteiro não deve ser uma sequência de perguntas fechadas, que interrompa a fala do
entrevistado. Ele funciona como um mapa para ajudar o entrevistador a não se perder no caminho.
- Se for necessário fazer outra entrevista com a mesma família, os dados da primeira entrevista são
muito úteis. O entrevistador pode iniciar a entrevista com estes dados.
Por exemplo: “Na semana passada, você havia dito que...”.
- Podemos também realizar uma entrevista com a família no momento do encerramento de um
processo de acompanhamento. Neste caso, o grande objetivo da família é o de avaliar o processo. Os
dados da avaliação são muito úteis para o aprimoramento do trabalho que realizamos.
Podemos sugerir à família que faça um desenho coletivo. Oferecemos uma folha de papel pardo e giz
de cera. Esta é uma oportunidade para observarmos a interação da família de maneira lúdica, ou seja,
esta atividade parece uma brincadeira, mas na verdade é um facilitador para a expressão de sentimentos,
emoções, dificuldades.
Depois de concluído o desenho, devemos conversar com a família sobre a sua produção.
Eles poderão contar uma história sobre o desenho, e o responsável pelo trabalho poderá fazer as
relações entre o desenho, sua história e a situação atual da família.

Dramatização

É um recurso para o trabalho em grupo. Serve para esclarecer melhor um tema confuso, para
sensibilizar as pessoas a fim de que compreendam o ponto de vista do outro, as dificuldades que cada
um tem para exercer cotidianamente os seus papéis de pai, mãe, marido, esposa, filho(a), professor(a),
educador social, entre outros. A dramatização pode também ser um recurso para a problematização de
um assunto novo para o grupo.
No grupo de pais ou responsáveis pela criança e pelo adolescente, vamos escolher coletivamente um
fato para ser dramatizado e, também em conjunto, decidir a distribuição dos papéis. Este fato deve ter
sido significativo para o grupo, pode ter sido vivenciado na comunidade, na igreja, na escola, no posto de
saúde, como também ser uma notícia de jornal ou uma cena de novela.
Terminada a dramatização, o grupo é convidado a discutir a experiência. É preciso tempo e espaço
para conversar sobre o que cada um pôde pensar sobre aquele fato, o que sentiu e o que aprendeu.

Grupos de reflexão

O recurso de formar grupos com famílias para discutir problemas comuns e buscar construir soluções
tem sido bastante promissor. As pessoas se sentem encorajadas quando estão entre iguais. Perceber
que há outras pessoas com problemas semelhantes encoraja a expressão e fortalece a solidariedade
grupal.
Na primeira fase do trabalho, faremos um levantamento dos temas de interesse do grupo. É importante
que o animador não leve nada pronto, pois levantar os temas de interesse com o grupo é um meio de
responsabilizá-lo pela tarefa.
Na segunda fase, discutiremos cada tema com base na percepção e nos valores dos membros do
grupo. O grupo fará um inventário do que sabe daquele assunto e de suas fontes de informação. Para
que esse conhecimento inicial possa ser transformado, é preciso, em primeiro lugar, valorizá-lo.
Na terceira fase, levantaremos dúvidas sobre os assuntos tratados, faremos uma lista de perguntas e
planejaremos coletivamente como resolver as dúvidas. Por exemplo: a decisão pode ser convidar um
palestrante; neste caso, a lista de perguntas deverá ser encaminhada a ele para preparar seu contato
com o grupo.
Na quarta fase, o grupo fará seus comentários sobre a palestra e a avaliação dos resultados: as
dúvidas daquele tema foram esclarecidas? Qual será o próximo assunto a ser discutido?
A organização do trabalho em fases dá ao grupo a noção de continuidade e, ao mesmo tempo, de uma
tarefa que tem princípio, meio e fim.
Qual o ponto comum entre as estratégias de intervenção propostas?
A família, seja qual for sua configuração, é um grupo de sujeitos ativos, que têm problemas e
dificuldades, e também possibilidades. Vamos potencializar as possibilidades, fazer uma aliança com os
aspectos positivos.
Levar tudo pronto e colocar as famílias numa posição passiva, desvalorizar suas potencialidades e
considerar que elas não têm qualquer conhecimento e possibilidade trazem alguns efeitos indesejáveis:
resistência a qualquer tipo de proposta; cristalização da baixa autoestima; indisponibilidade para a
mudança.

Apostila gerada especialmente para: Betânia Mueller 023.712.580-32


. 219
Registro das experiências

Não é uma tarefa fácil, para as equipes que trabalham nos programas destinados à família, registrar
suas experiências. As demandas do cotidiano exigem ações rápidas, contínuas e emergentes. Todos nós
temos na cabeça a imagem do “bombeiro apagando incêndio”. Ainda assim, toda intervenção realizada
deve ser registrada. O registro é um ato político capaz de dar visibilidade à situação das famílias, seus
problemas, suas necessidades, bem como, às necessidades das equipes de trabalho.
Por meio dos registros, é possível avaliar as intervenções e buscar o aprimoramento delas. A prática
cotidiana é uma maneira de produzir conhecimento e, deste modo, o registro é uma forma de fazer com
que este conhecimento possa circular entre aqueles que trabalham com famílias nos diversos programas
e aqueles que pesquisam e estudam as famílias. Este registro, então, torna-se um importante elo de
ligação entre a teoria e a prática. A teoria produz a prática, mas também a prática produz a teoria. O nome
deste movimento é práxis.

As famílias são diferentes, mas são iguais! Como?

A família, seja qual for o seu modelo, é uma referência na vida de homens e mulheres, independemente
de idade, vinculação étnico-racial ou país de origem, ou seja, em todas as sociedades humanas, existe
família. Mas cada sociedade tem normas particulares que norteiam a organização familiar, e cada família
se organiza de modo singular.
Quais são as funções universais da família?
A família é responsável pela socialização primária de suas crianças, isto quer dizer que é na família
que a criança aprende a falar, conhece as regras de sua sociedade e internaliza as noções de “certo” e
de “errado”. Além disso, a família transmite aos seus membros hábitos alimentares, tradições, valores
éticos, religiosos, modos de comemorar os aniversários e os casamentos, e de lidar com a morte.
A família também faz uma mediação entre os indivíduos e a sociedade mais ampla. É a família que
inicia a criança nas práticas religiosas, por exemplo. Também é ela que leva a criança a frequentar a
escola, que, por sua vez, demanda que a família acompanhe a trajetória escolar da criança.
A família não pode substituir nem a sociedade nem o estado na defesa e promoção dos direitos de
cidadania de suas crianças e seus adolescentes, mas pode e deve mediar e lutar por esses direitos, e
precisa ser amparada, quando necessário, para realizar seu papel. as funções universais da família não
devem ser tomadas para uniformizar todas as famílias, e sim para compreender alguns limites éticos. A
postura ética indica o que a família deve fazer e, ao mesmo tempo, o que não deve fazer. Ela deve cuidar
de suas crianças, e não abandoná-las ou maltratá-las, seja qual for seu modelo ou sua estrutura.

Serviço de Proteção e Atendimento Integral à Família - PAIF.23

O Serviço de Proteção e Atendimento Integral à Família - PAIF, consiste no trabalho social com
famílias, de caráter continuado, com a finalidade de fortalecer a função protetiva da família, prevenir a
ruptura de seus vínculos, promover seu acesso e usufruto de direitos e contribuir na melhoria de sua
qualidade de vida. Prevê o desenvolvimento de potencialidades e aquisições das famílias e o
fortalecimento de vínculos familiares e comunitários, por meio de ações de caráter preventivo, protetivo e
proativo. O serviço PAIF integra o nível de proteção social básica do SUAS. (Tipificação Nacional de
Serviços Socioassistenciais).

Surgimento
O PAIF foi concebido a partir do reconhecimento que as vulnerabilidades e riscos sociais, que atingem
as famílias, extrapolam a dimensão econômica, exigindo intervenções que trabalhem aspectos objetivos
e subjetivos relacionados a função protetiva da família e ao direito à convivência familiar.
O PAIF teve como antecedentes o Programa Núcleo de Apoio à Família (NAF - 2001), e o Plano
Nacional de Atendimento Integrado à Família (PNAIF- 2003). Em 2004, o MDS, aprimorou essa proposta
com a criação do Programa de Atenção Integral à Família (PAIF).

Em 19 de maio de 2004, com o decreto 5.085 da Presidência da República, o PAIF tornou-se “ação
continuada da Assistência Social”, passando a integrar a rede de serviços de ação continuada da
Assistência Social financiada pelo Governo Federal.

23
http://mds.gov.br/acesso-a-informacao/perguntas-frequentes/assistencia-social/psb-protecao-social-basica/projetos-psb/servico-de-protecao-e-atendimento-
integral-a-familia-2013-paif

Apostila gerada especialmente para: Betânia Mueller 023.712.580-32


. 220
Em 2009, com a aprovação da Tipificação Nacional dos Serviços Socioassistenciais, o Programa de
Atenção Integral à Família passou a ser denominado Serviço de Proteção e Atendimento Integral à
Família, mas preservou a sigla PAIF. Esta mudança de nomenclatura enfatiza o conceito de ação
continuada, estabelecida em 2004, bem como corresponde ao previsto no Art. 23 da Lei Orgânica de
Assistência Social - LOAS.

Nessa direção, o PAIF concretiza a presença e responsabilidade do poder público e reafirma a


perspectiva dos direitos sociais, constituindo-se em um dos principais serviços que compõem a rede de
proteção social de assistência social, que vem consolidando no país de modo descentralizado e
universalizado, permitindo o enfrentamento da pobreza, da fome e da desigualdade, assim como, a
redução da incidência de riscos e vulnerabilidades sociais que afetam famílias e seus membros. (Caderno
de Orientações Técnicas do PAIF - vol. 1).

Concepção de família
A família para a PNAS é o grupo de pessoas que se acham unidas por laços consanguíneos, afetivos
e, ou de solidariedade. A família, independente dos formatos ou modelos que assume, é mediadora das
relações entre os sujeitos e a coletividade. Caracteriza-se como um espaço contraditório, cuja dinâmica
cotidiana de convivência é marcada por conflitos e geralmente, também, por desigualdades, sendo a
família a base fundamental no âmbito da proteção social.

Objetivos
• Fortalecer a função protetiva da família, contribuindo na melhoria da sua qualidade de vida;
• Prevenir a ruptura dos vínculos familiares e comunitários, possibilitando a superação de situações de
fragilidade social vivenciadas;
• Promover aquisições sociais e materiais às famílias, potencializando o protagonismo e a autonomia
das famílias e comunidades;
• Promover o acesso a benefícios, programas de transferência de renda e serviços socioassistenciais,
contribuindo para a inserção das famílias na rede de proteção social de assistência social;
• Promover acesso aos demais serviços setoriais, contribuindo para o usufruto de direitos;
• Apoiar famílias que possuem, dentre seus membros, indivíduos que necessitam de cuidados, por
meio da promoção de espaços coletivos de escuta e troca de vivências familiares.

Usuários do PAIF
Constituem usuários do PAIF as famílias territorialmente referenciadas ao CRAS, em situação de
vulnerabilidade social decorrente da pobreza, do precário ou nulo acesso aos serviços públicos, da
fragilização de vínculos de pertencimento e sociabilidade e/ou qualquer outra situação de vulnerabilidade
e risco social.

São prioridades as seguintes situações consideradas de maior vulnerabilidade social:


• Famílias vivendo em territórios com nulo ou frágil acesso à saúde, à educação e aos demais direitos,
em especial famílias monoparentais chefiadas por mulheres, com filhos ou dependentes;
• Famílias provenientes de outras regiões, sem núcleo familiar e comunitário local, com restrita rede
social e sem acesso a serviços e benefícios socioassistenciais;
• Famílias recém-retiradas de seu território de origem, em função da implementação de
empreendimentos com impactos ambientais e sociais; Famílias com moradia precária (sem instalações
elétricas ou rede de esgoto, com espaço muito reduzido, em áreas com risco de deslizamento,
vivenciando situações declaradas de calamidade pública, dentre outras);
• Famílias vivendo em territórios com conflitos fundiários (indígenas, quilombolas, extrativistas, dentre
outros);
• Famílias pertencentes aos povos e comunidades tradicionais (indígenas, quilombolas, ciganos e
outros);
• Famílias ou indivíduos com vivência de discriminação (étnico-raciais e culturais, etárias, de gênero,
por orientação sexual, por deficiência e outras);
• Famílias vivendo em contextos de extrema violência (áreas com forte presença do crime organizado,
tráfico de drogas, dentre outros);
• Famílias que enfrentam o desemprego, sem renda ou renda precária com dificuldades para prover o
sustento dos seus membros;

Apostila gerada especialmente para: Betânia Mueller 023.712.580-32


. 221
• Famílias com criança(s) e/ou adolescente(s) que fica(m) sozinho(s) em casa, ou sob o cuidado de
outras crianças, ou passa(m) muito tempo na rua, na casa de vizinhos, devido à ausência de serviços
socioassistenciais, de educação, cultura, lazer e de apoio à família;
• Família que entregou criança/adolescente em adoção;
• Família com integrante que apresenta problemas de saúde que demandam do grupo familiar proteção
e/ou apoios e/ou cuidados especiais (transtornos mentais, doenças crônicas etc.).

Vale ressaltar que isso não significa que todas as famílias residentes nos territórios de abrangência
dos CRAS e que vivenciam tais situações precisam ser obrigatoriamente inseridas no PAIF. O
atendimento pelo Serviço deve ser de total interesse e concordância das famílias, precedido da análise
da equipe técnica.

Quais são as seguranças afiançadas pela Política Nacional de Assistência Social?


a) segurança de acolhida - provida por meio de ofertas públicas de espaços e serviços localizados
prioritariamente em territórios de maior vulnerabilidade, com condições de escuta profissional qualificada,
informação, referência, concessão de benefícios, de aquisições materiais, sociais e socioeducativas;
b) segurança social de renda - operada por meio de concessão de Benefícios de Prestação Continuada
da Assistência Social - BPC nos termos da lei, para cidadãos não incluídos no sistema contributivo de
proteção social que apresentem vulnerabilidades decorrentes do ciclo de vida e, ou, incapacidade para a
vida independente e para o trabalho; e concessão de auxílios financeiros sob determinadas
condicionalidades;
c) segurança de convívio familiar e comunitário - oferta pública de rede de serviços continuados que
garantam oportunidades e ação profissional para: construção, restauração e fortalecimento de laços de
pertencimento (de natureza geracional, intergeracional, familiar, de vizinhança e interesses comuns e
societários); exercício capacitador e qualificador de vínculos sociais e de projetos pessoais e sociais de
vida em sociedade;
d) segurança de desenvolvimento da autonomia individual, familiar e social - provisão estatal de ações
profissionais para o desenvolvimento de capacidades e habilidades para o exercício do protagonismo, da
cidadania; a conquista de maior grau de liberdade, respeito à dignidade humana, protagonismo e certezas
de proteção social para o cidadão, a família e a sociedade; a conquista de maior grau de independência
pessoal e qualidade nos laços sociais para os cidadãos e cidadãs sob contingências e dificuldades; e
e) segurança de sobrevivência a riscos circunstanciais - provisão de acesso estatal, em caráter
transitório, de auxílios em bens materiais e em dinheiro, denominados de benefícios eventuais para
indivíduos e famílias em risco e vulnerabilidades circunstanciais e nos casos de calamidade pública.

Quais são os elementos de vulnerabilidade segundo a NOB-SUAS?


Dentre alguns dos elementos de vulnerabilidade, a NOBSUAS aponta questões relacionadas à
precariedade de infraestrutura; presença de crianças e adolescentes, idosos e pessoas com deficiência
em famílias com renda até meio salário mínimo, responsáveis analfabetos ou com baixa escolaridade e
mulheres chefes de famílias sem cônjuge, famílias com responsáveis desempregados, família em
situação de trabalho infantil ou com presença de crianças e adolescentes em idade escolar obrigatória
fora da escola, dentre outros.

Dessa forma, há uma série de indícios possíveis de serem obtidos a partir das estatísticas nacionais
que, combinados, podem representar situações agravadas de vulnerabilidade social e de reprodução da
pobreza entre gerações.

Qual a definição de trabalho social com famílias no âmbito do PAIF?


Conjunto de procedimentos a partir de pressupostos éticos, conhecimento teórico metodológico e
técnico-operativo, com a finalidade de contribuir para a convivência, reconhecimento de direitos e
possibilidades de intervenção na vida social de um conjunto de pessoas, unidas por laços consanguíneos,
afetivos e/ou de solidariedade. (Caderno de Orientações do PAIF - Vol. 2). Reconhecer que as famílias
são protagonistas de suas histórias, mas que sofrem os impactos da realidade socioeconômica e cultural
nas quais estão inseridas, em especial as contradições do território.

Ações que compõem o PAIF


Podem ser de caráter individual ou coletivo.
• Acolhida;
• Oficinas com famílias;

Apostila gerada especialmente para: Betânia Mueller 023.712.580-32


. 222
• Ações comunitárias;
• Ações particularizadas;
• Encaminhamentos.

Acesso às ações do PAIF


São quatro as formas de acesso ao PAIF descritas pela Tipificação. Destaca-se dentre tais formas de
acesso à busca ativa, pois é por meio dela que o PAIF consegue operacionalizar de modo mais efetivo a
sua função protetiva e preventiva nos territórios, visto que é capaz de antecipar a ocorrência de situações
de vulnerabilidade e risco social e não somente reagir passivamente às demandas apresentadas pelas
famílias. (Tipificação Nacional de Serviços Socioassistenciais).
• Por procura espontânea;
• Por busca ativa;
• Por encaminhamento da rede socioassistencial;
• Por encaminhamento das demais políticas públicas.

Elementos necessários para execução do serviço PAIF


• Ambiente físico;
• Recursos materiais;
• Recursos humanos; e
• Trabalho social essencial ao serviço.

O que é a Acolhida?
É o processo de contato inicial do usuário com o PAIF e tem por objetivo instituir o vínculo necessário
entre as famílias usuárias e o PAIF para a continuidade do atendimento sócio-assistencial iniciado. A
Acolhida ocorre em grande parte na recepção do CRAS. Deve ser cuidadosamente organizada, para se
constituir referência para as famílias. A acolhida é primordial na garantia de acesso da população ao
SUAS e de compreensão da assistência social como direito de cidadania.

O que é oficinas com famílias?


Consistem na realização de encontros previamente organizados, com objetivos de curto prazo a serem
atingidos com um conjunto de famílias, por meio de seus responsáveis ou outros representantes, sob a
condução de técnicos de nível superior do CRAS.

O que são ações comunitárias?


São ações de caráter coletivo, voltadas para a dinamização das relações no território. Possuem escopo
maior que as oficinas com famílias, por mobilizar um número maior de participantes, e devem agregar
diferentes grupos do território a partir do estabelecimento de um objetivo comum.

O que são ações particularizadas?


Referem-se ao atendimento prestado pela equipe técnica do CRAS à família - algum (ns) membro(s)
ou todo o grupo familiar, após a acolhida, de modo individualizado. As ações particularizadas devem ser
realizadas por indicação do técnico responsável pela acolhida da família ou a pedido da família.

O que são Encaminhamentos?


São os processos de orientação e direcionamento das famílias, ou algum de seus membros, para
serviços e/ou benefícios socioassistenciais ou de outros setores. Têm por objetivo a promoção do acesso
aos direitos e a conquista da cidadania.

Como baseia-se a busca ativa no PAIF?


Refere-se à procura intencional, realizada pela equipe de referência, das ocorrências que influenciam
o modo de vida da população em determinado território. Tem como objetivo identificar as situações de
vulnerabilidade e risco social, ampliar o conhecimento e a compressão da realidade social, para além dos
estudos e estatísticas. Contribui para o conhecimento da dinâmica do cotidiano da população (a realidade
vivida pela família, sua cultura e valores, as relações que estabelece no território e fora dele); os apoios
e recursos existentes e, seus vínculos sociais. (Caderno de Orientações Técnicas do CRAS).

Qual a diferença entre atendimento e acompanhamento familiar no âmbito do PAIF?


O trabalho social com famílias do PAIF pode ocorrer por meio dos dois processos distintos, porém
complementares. O atendimento refere-se a uma ação imediata de prestação ou oferta de atenção, com

Apostila gerada especialmente para: Betânia Mueller 023.712.580-32


. 223
vistas a uma resposta qualificada de uma demanda da família ou do território, ou seja, a inserção em
alguma das ações do serviço. O acompanhamento familiar consiste em um conjunto de intervenções,
desenvolvidas de forma continuada, a partir do estabelecimento de compromissos entre famílias e
profissionais, que pressupõem a construção de um Plano de Acompanhamento Familiar - com objetivos
a serem alcançados, a realização de mediações periódicas, a inserção em ações do PAIF, buscando a
superação gradativa das vulnerabilidades vivenciadas.

O acompanhamento familiar é obrigatório?


O acompanhamento não é um processo que visa avaliar a(s) família(s), sua organização interna, seu
modo de vida, sua dinâmica de funcionamento. Ao contrário, é uma atuação do serviço sócio-assistencial,
com foco na garantia das seguranças afiançadas pela política de assistência social. São acompanhadas
as famílias que aceitam participar do processo de acompanhamento. O acompanhamento familiar
constitui um direito, portanto, sua participação não deve ser algo imposto pelos profissionais.

Quais as diretrizes metodológicas para o trabalho social com famílias do PAIF?


• Fortalecer a assistência social como direito social de cidadania;
• Respeitar a heterogeneidade dos arranjos familiares e sua diversidade cultural;
• Rejeitar concepções preconceituosas, que reforçam desigualdades no âmbito familiar;
• Respeitar e preservar a confidencialidade das informações repassadas pelas famílias no decorrer do
trabalho social;
• Utilizar e potencializar os recursos disponíveis das famílias no desenvolvimento do trabalho social;
• Utilizar ferramentas que contribuam para a inserção efetiva de todos os membros da família no
acompanhamento familiar.

É importante que as ações do PAIF sejam adequadas às experiências, situações, contextos vividos
pelas famílias. Portanto, ao implementá-las cabe refletir sobre o tipo de família a que a ação se destina e
se ela terá algum significado.

Qual o tempo de permanência das famílias no PAIF?


Não há um período máximo de permanência das famílias no serviço. No entanto, é necessário avaliar
os casos em que as equipes têm dificuldades para desligar as famílias, partindo do critério do
cumprimento dos objetivos das ações propostas no CRAS ou em sua rede socioassistencial. O
desligamento deve ser planejado e realizado de maneira progressiva, com acompanhamento familiar por
período determinado para verificar a permanência dos efeitos positivos das ações, tendo como referência
os resultados esperados.

Referências:
http://mds.gov.br/acesso-a-informacao/perguntas-frequentes/assistencia-social/psb-protecao-social-basica/projetos-psb/servico-de-protecao-e-atendimento-
integral-a-familia-2013-paif

Questões

01. (MPE/SE - Analista do Ministério Público - FCC) De acordo com o Serviço de Proteção e
Atendimento Integral à Família (PAIF), é correto afirmar que família é um
(A) conjunto de pessoas unidas somente pela ascendência e descendência.
(B) conjunto de pessoas unidas somente pela descendência e afinidade voluntária.
(C) grupo social unido por vínculos de ascendência, descendência, afinidade e (ou) solidariedade.
(D) grupo concebido a partir da convivência individual, com o reconhecimento de direitos e
possibilidades de intervenção.
(E) conceito abstrato, definido no PAIF de forma diferente da Política Nacional de Assistência Social
(PNAS, 2004).

02. (CIAAR - Oficial Temporário - CIAAR) Para os efeitos do disposto no da Lei nº 12.435, de 06 de
julho de 2011, a família:
(A) é definida pelos membros que compõem o núcleo residencial, restrito aos que possuam afinidade
de consanguinidade e que convivam na mesma casa.
(B) é restrita aos membros da família nuclear, pai, mãe e filhos, desde que residam sob o mesmo teto.
(C) é formada pelos arranjos familiares que incluem a família nuclear e a extensa, como também o
requerente e todos aqueles que estejam solicitando os benefícios no mesmo processo.

Apostila gerada especialmente para: Betânia Mueller 023.712.580-32


. 224
(D) é constituída por pai, mãe, filhos e avós, desde que residam na mesma casa, e enteados, caso
estes estejam sob a guarda legal de um dos requerentes.
(E) é composta pelo requerente, o cônjuge ou companheiro, os pais e, na ausência de um deles, a
madrasta ou o padrasto, os irmãos solteiros, os filhos e enteados solteiros e os menores tutelados, desde
que vivam sob o mesmo teto.

03. (TJ/RO - Analista Judiciário - CESPE) De acordo com a PNAS, o princípio da matricialidade da
família:
(A) fundamenta-se no conceito de família como unidade econômica para a realização da avaliação
social.
(B) recomenda que se adote a concepção de tutela para as famílias que se encontram em situação de
pobreza extrema, pois estão incapacitadas de desempenhar seu papel adequadamente.
(C) concebe um modelo idealizado de família, com estrutura definida.
(D) alerta para os riscos que se corre nos trabalhos ao adotar visões disciplinadoras que visam
enquadrar as famílias em normas rígidas, independentemente do universo cultural a que pertencem.
(E) reconhece que a família só exerce o seu potencial protetivo quando não está vulnerabilizada.

04. (TJ/RO - Analista Judiciário - CESPE) De acordo com o ECA, no processo de colocação de
criança ou adolescente indígena em família substituta, deve-se
(A) respeitar a identidade social da criança ou do adolescente, bem como suas instituições, desde que
não sejam incompatíveis com os direitos fundamentais reconhecidos pela Constituição Federal.
(B) dispensar a autorização judicial para os casos de transferência da criança ou adolescente a
entidades que atuam em defesa da causa indígena.
(C) dispensar a etapa de preparação, por se tratar de situação em que se aplica a excepcionalidade
em razão da preservação da cultura e das tradições da criança ou do adolescente.
(D) priorizar famílias que residem no meio rural.
(E) excluir a participação de representantes antropólogos nas equipes de acompanhamento do caso.

05. (CEFET/RJ - Assistente Social - EXATUS) “Os programas que visam o repasse direto de recursos
dos fundos de assistência social aos beneficiários como forma de acesso à renda, visando o combate à
fome, à pobreza e outras formas de privação de direitos que levem à situação de vulnerabilidade social,
criando possibilidades para a emancipação, o exercício da autonomia das famílias e indivíduos atendidos
e o desenvolvimento local”, de acordo com a NOB/SUAS são denominados de:
(A) Enfrentamento à pobreza
(B) Benefícios eventuais
(C) Transferência de Renda
(D) Salário família

06. (METRÔ/DF - Assistente Social - IADES). De acordo com o Serviço de Proteção e Atendimento
Integral à Família (PAIF), é correto afirmar que família é um
(A) conjunto de pessoas unidas somente pela ascendência e descendência.
(B) conjunto de pessoas unidas somente pela descendência e afinidade voluntária.
(C) grupo social unido por vínculos de ascendência, descendência, afinidade e (ou) solidariedade.
(D) grupo concebido a partir da convivência individual, com o reconhecimento de direitos e
possibilidades de intervenção.
(E) conceito abstrato, definido no PAIF de forma diferente da Política Nacional de Assistência Social
(PNAS, 2004).
Gabarito

01.C / 02.E /03.D / 04.A / 05.C / 06.C

Comentário
01. Resposta: C
A família para a PNAS (e também adotado no PAIF) constitui como um grupo de pessoas que se
acham unidas por laços consanguíneos, afetivos e, ou de solidariedade. A família, independente dos
formatos ou modelos que assume, é mediadora das relações entre os sujeitos e a coletividade.
Caracteriza-se como um espaço contraditório, cuja dinâmica cotidiana de convivência é marcada por
conflitos e geralmente, também, por desigualdades, sendo a família a base fundamental no âmbito da
proteção social.

Apostila gerada especialmente para: Betânia Mueller 023.712.580-32


. 225
02. Resposta: E
De acordo com a Redação dada pela Lei nº 12.435, de 2011 consta:
Art. 20 O benefício de prestação continuada é a garantia de um salário-mínimo mensal à pessoa com
deficiência e ao idoso com 65 (sessenta e cinco) anos ou mais que comprovem não possuir meios de
prover a própria manutenção nem de tê-la provida por sua família.
§ 1º Para os efeitos do disposto no caput, a família é composta pelo requerente, o cônjuge ou
companheiro, os pais e, na ausência de um deles, a madrasta ou o padrasto, os irmãos solteiros, os filhos
e enteados solteiros e os menores tutelados, desde que vivam sob o mesmo teto.

03. Resposta: D
De acordo com a PNAS, a centralidade considera que família, independente dos formatos que assume,
é mediadora do indivíduo e a coletividade. Reconhece ainda que a família não deve ser entendida como
uma casa onde vivem pai, mãe e filhos, mas devido às transformações sociais surgem novas concepções
de famílias que devem ser consideradas principalmente como um grupo de pessoas que convivem em
determinado lugar e que se acham unidas por laços de sangue, solidariedade. Por essa razão o SUAS o
não faz distinção para a forma que a família é constituída, o importante é incluir os membros do grupo
familiar em programa e projetos sociais para que saiam da situação de exclusão.

04. Resposta: A
Sobre este assunto o ECA dispõe:
Art.28. A colocação em família substituta far-se-á mediante guarda, tutela ou adoção,
independentemente da situação jurídica da criança ou adolescente, nos termos desta Lei.
§ 6º Em se tratando de criança ou adolescente indígena ou proveniente de comunidade remanescente
de quilombo, é ainda obrigatório:
I - que sejam consideradas e respeitadas sua identidade social e cultural, os seus costumes e
tradições, bem como suas instituições, desde que não sejam incompatíveis com os direitos fundamentais
reconhecidos por esta Lei e pela Constituição Federal

05. Resposta: C
Segundo o MDS, transferência de renda constitui programas que visam o repasse direto de recursos
dos fundos de Assistência Social aos beneficiários, como forma de acesso à renda, visando o combate à
fome, à pobreza e outras formas de privação de direitos, que levem à situação de vulnerabilidade social,
criando possibilidades para a emancipação, o exercício da autonomia das famílias e indivíduos atendidos
e o desenvolvimento local.

06. Resposta: C
O Serviço de Proteção e Atendimento Integral à Família - PAIF, consiste no trabalho social com
famílias, de caráter continuado, com a finalidade de fortalecer a função protetiva da família, prevenir a
ruptura de seus vínculos, promover seu acesso e usufruto de direitos e contribuir na melhoria de sua
qualidade de vida.

Psicologia, Direitos Humanos e Democracia.

O encontro entre a Psicologia e os Direitos Humanos no Brasil pode parecer algo dispensável,
prioritariamente por duas razões. A primeira delas é que, para alguns, propor a convergência entre a
Psicologia e os Direitos Humanos pode soar como uma obviedade, tendo em vista os diversos campos
de atuação da Psicologia que hoje incidem em questões relativas aos Direitos Humanos. Abordar essa
temática seria uma questão já superada, já que esse diálogo se encontra em profícuo desenvolvimento.
A segunda razão seria exatamente o contrário, ou seja, para outros, propor um diálogo entre Psicologia
e Direitos Humanos pode significar uma sobreposição de áreas, já que o campo dos Direitos Humanos
tradicionalmente sempre foi algo relacionado ao Direito.24
Entende-se que, cada vez mais, torna-se imprescindível explicitar o diálogo entre esses dois campos,
não apenas para demonstrar os pontos que possuem em comum, mas principalmente para fundamentar
teoricamente quais as conexões entre Psicologia e Direitos Humanos. Secundariamente, evidenciar aos

24
https://revistas.pucsp.br/index.php/psicorevista/article/viewFile/6790/4913

Apostila gerada especialmente para: Betânia Mueller 023.712.580-32


. 226
que ainda não vislumbram esse diálogo a possibilidade de perceber a proximidade inerente que existe
entre essas duas áreas.
Para tanto, num primeiro momento, torna-se fundamental uma breve abordagem histórica do período
que ensejou a construção e a institucionalização da Psicologia, enquanto ciência e profissão no Brasil, no
início da década de 60. Também se faz igualmente importante tecer alguns apontamentos históricos
relativos ao contexto sócio-político em que essa nova categoria profissional surgia. Além disso,
considerando as mudanças ocorridas na Psicologia desde seu surgimento, não há como deixar de
mencionar as transformações e rupturas ocorridas nesse campo do conhecimento até os dias de hoje.
Num segundo momento, entende-se como relevante trazer alguns elementos relacionados à
construção do campo dos Direitos Humanos e como isso se deu no Brasil, assim como seus objetivos e
alguns elementos presentes na sua discussão mais atual. Por fim, pretende-se iluminar aspectos que
evidenciam importantes pontos de convergências entre essas duas áreas.

Psicologias e Psicologias

Em relação à Psicologia, cabe colocar que, esta foi regulamentada como categoria profissional em
1962, período histórico que precedeu no país a instalação do golpe militar, regime ditatorial que suprimiu
a democracia. Não há como minimizar a relevância desta conjuntura sócio-política, considerando que um
dos principais objetivos da Psicologia, na época, dizia respeito à adaptação e ajustamento do indivíduo.
De acordo com a Lei n. 4.119 (1962) e com o Decreto n. 53.464 (1964) que dispõem sobre os cursos
de formação em Psicologia e a profissão de psicólogo, é possível verificar que a função precípua dessa
nova categoria profissional era produzir perfis psicológicos – realizando diagnósticos psicológicos – e
promover a adequação dos indivíduos em seus respectivos contextos, como bem explicita o art. 13, § 1º,
alínea d, ao expressar “solução de problemas de ajustamento”.
Vale dizer ainda que, nessa época, os locais de atuação da Psicologia estavam prioritariamente
circunscritos ao ambiente escolar, às indústrias – sendo o trabalho direcionado para processos de seleção
e recrutamento de pessoal – e à clínica strictu sensu, entendida aqui como o atendimento psicoterapêutico
individual em consultório.
A fim de ilustrar isso, é interessante trazer a descrição feita por Bock (2001) sobre o trabalho da
Psicologia:
Psicólogo: estuda o comportamento e mecanismo mental dos seres humanos, realiza pesquisas sobre
os problemas psicológicos que se colocam no terreno da medicina, da educação e da indústria e
recomenda o tratamento adequado:
a. Projeta e realiza experimentos e estudos em seres humanos para determinar suas características
mentais e físicas;
b. Analisa a influência de fatores hereditários, ambientais e outros mais na configuração mental e
comportamento dos indivíduos;
c. Faz diagnósticos, tratamentos e prevenção de transtornos emocionais e da personalidade, assim
como dos problemas de inadaptação ao meio social e de trabalho;
d. Cria e aplica testes psicológicos para determinar a inteligência, faculdade, aptidões, atitudes e outras
características pessoais, interpreta os dados obtidos e faz as recomendações pertinentes (p. 26-27).

Procura-se demonstrar aqui que não é à toa que exatamente num governo ditatorial se gesta uma
profissão que busca criar perfis e trabalhar para o ajustamento de pessoas que não se enquadram ao
“normal”. Fatos como esse revelam as construções históricas que são feitas, de acordo com seus
respectivos contextos e possibilidades.
Desse modo, é interessante trazer Nietzsche (1887, citado por Foucault, 2003) para se pensar a
questão do conhecimento. Intencionalmente o filósofo afirma que o conhecimento foi inventado e não
aborda sua suposta origem, fazendo a importante distinção entre origem e invenção:
Nietzche afirma que, em determinado ponto do tempo e determinado tempo do universo, animais
inteligentes inventaram o conhecimento; a palavra que emprega invenção é frequentemente retomada
em seus textos e sempre com sentido e intenção polêmicos. Quando fala em invenção, Nietzche tem
sempre em mente uma palavra que opõe a invenção, a palavra origem.
Quando diz invenção é pra não dizer origem (p. 14).
Assim, pode-se inferir que, nessa lógica, o conhecimento em si não possui origem, mas sim, foi
inventado. Em outras palavras, as verdades históricas que, em determinados períodos ganham validade,
em detrimento de outras, não passam de invenções/criações que, por razões diversas, ganham o estatuto
de legitimidade perante a sociedade. Nesse contexto de construção do conhecimento, a Psicologia
surgiria no contexto brasileiro.

Apostila gerada especialmente para: Betânia Mueller 023.712.580-32


. 227
Combinado com isso é preciso considerar também o lugar de poder/ saber trabalhado também por
Foucault (1979) que se constitui enquanto elemento-chave para entender como determinados fenômenos
adquirem força e passam a ser reconhecidos socialmente. De acordo com o referido autor:
O importante, creio eu, é que a verdade não existe fora do poder ou sem poder (...). A verdade é deste
mundo; ela é produzida nele graças a múltiplas coerções e nele produz efeitos regulamentados de poder.
Cada sociedade tem seu regime de verdade, sua ‘política geral’ de verdade: isto é, os tipos de discurso
que ela acolhe e faz funcionar como verdadeiros; os mecanismos e as instâncias que permitem distinguir
os enunciados verdadeiros dos falsos, a maneira como se sanciona uns e outros, as técnicas e os
procedimentos que são valorizados para a obtenção da verdade; o estatuto daqueles que têm o encargo
de dizer o que funciona como verdadeiro (p. 12).
Nesta linha, não há como se furtar de que a Psicologia veio como um lugar de saber/poder, muitas
vezes, a serviço da repressão e da lógica normatizadora do sujeito. Nesse sentido, em relação ao saber
psicológico produzido, eis uma caracterização que traduz a formação dessa categoria profissional que
estava surgindo no país, como bem descreveu Coimbra (1995):
(...) a formação “psi”, em geral, traz certas características modelares instituídas e tão bem marcadas;
como, em nossa formação, predomina o viés positivista, onde se tornam hegemônicos os conceitos de
neutralidade, objetividade, cientificidade e tecnicismo; onde, nos diferentes discursos e práticas, o homem
e a sociedade são apresentados como “coisas em si”, abstratos, naturais e não produzidos historicamente
(p. ix).
Ainda que a maior parte do texto legislativo que ensejou a criação da Psicologia já tenha sido
reformulado e que essa formação “psi” já tenha sido significativamente superada, o objetivo aqui é de
contextualizar em que base histórica se configurou a Psicologia no Brasil. Assim, torna-se possível
perceber que o trabalho desse novo profissional almejava a construção de perfis psicológicos de
funcionamento para os indivíduos e a criação de soluções para indivíduos que não se adequavam ao
meio em que viviam.
Por décadas, este paradigma determinista permaneceu em vigor de forma hegemônica nos espaços
em que a Psicologia se fazia presente. Além disso, ainda de acordo com Coimbra (1995), se fortalece
uma psicologia especializada em investigar a privacidade dos sujeitos, instituindo-se assim o especialista
da intimidade, como é possível observar a seguir:
Aumentam a preocupação e o investimento com as questões relativas ao “interior” e o conhecimento
de si mesmo torna-se uma finalidade, em vez de um meio para se conhecer o mundo. Esta visão intimista
é extremamente valorizada na década de 70, quando a realidade social, o domínio público são esvaziados
e desprovidos de sentido. O único sentido está no privado (...). O importante não é o que se faz, mas o
que se sente. Ou seja, há um esvaziamento político, há uma psicologização do cotidiano e da vida social
(p. 34).
A partir da (re) democratização do país, o campo psicológico se ampliou e houve uma ruptura com o
que inicialmente foi a proposta da profissão. Já não era mais possível manter uma Psicologia
individualizante, descontextualizada e a-histórica.
Esse momento político vivenciado no país apresenta-se como momento de ruptura para a Psicologia,
ou pelo menos, permite à profissão o começo de uma longa e fértil revisão de suas propostas de
intervenção. Pode-se dizer, inclusive, que esse novo contexto reforçou a necessidade de uma avaliação
da profissão e seus objetivos, enfim, de sua função pública perante a sociedade brasileira.
Nesta conjuntura de expansão da categoria, constata-se atualmente uma diversidade de práticas
profissionais que se inserem no campo da Psicologia; esse cenário não necessariamente aponta para
uma uniformidade e consenso entre os diferentes conhecimentos psicológicos, até porque, muitas vezes,
são linhas ou abordagens teóricas que implicam em diferentes concepções de sujeito.
Dessa maneira, para conhecer e compreender o campo da Psicologia torna-se de fundamental
relevância considerar as diversas correntes teóricas existentes, situando-as em seus respectivos
contextos sócio-históricos. Essa realidade pode ser caracterizada por uma imensa pluralidade e
heterogeneidade de abordagens teóricas e diferentes perspectivas epistemológicas. Assim, a Psicologia
é constituída por uma rede complexa de saberes que podem ser convergentes ou não. Nessa lógica, a
mesma situação se aplica à Psicologia Social enquanto especialidade do campo da Psicologia e
certamente a que mais se aproxima dos Direitos Humanos, do ponto de vista teórico-prático.
Segundo Spink (2007), teríamos aproximadamente 04 (quatro) diferentes “histórias” da Psicologia
Social que têm sido difundidas na atualidade. A primeira delas foca uma Psicologia Social experimental
que permitiria a mensuração de comportamentos de modo objetivo e rigoroso, seguindo a lógica das
ciências naturais. Uma segunda história, derivada da primeira, considera uma perspectiva mais flexível
que a primeira, priorizando aplicações práticas do que está colocado em teorias.

Apostila gerada especialmente para: Betânia Mueller 023.712.580-32


. 228
Já a terceira história da Psicologia Social trabalharia com contextos sociais e históricos, entendendo a
própria construção da disciplina como produto de sua época; nessa perspectiva, já estaria colocado um
viés crítico, ultrapassando o enfoque descritivo das propostas anteriores. Por fim, a quarta história
trabalharia com a compreensão da própria disciplina Psicologia Social funcionar como instrumento de
dominação social, sendo imprescindível considerar as consequências de se trabalhar com conduta
humana e subjetividade.
Essas diferentes “histórias” da Psicologia Social não estão colocadas de modo linear e cronológico,
como se estivessem sobrepostas ou ultrapassadas; pelo contrário, são perspectivas atuais e atuantes no
campo da Psicologia, tendo diferentes representantes e grupos de pesquisa, espalhados pelas diversas
universidades brasileiras.
Nesse grande leque de possibilidades e compreensões, a autora faz um resumo do que seriam essas
diferentes linhas de pesquisa em Psicologia Social e, ao final, introduz uma questão bastante significativa:
De um lado, encontramos uma Psicologia Social do biológico, do intraindividual, do interindividual e do
grupo; no meio, uma psicologia social da subjetividade, da linguagem, das representações sociais, dos
grupos e dos processos políticos; e, de outro lado, uma Psicologia Social centrada na interação social, na
reprodução, na mudança e nos movimentos coletivos. Se todos eles são textos recentes sobre Psicologia
Social, por que são tão diferentes? (p. 569).
Assim, é possível constatar que os estudos e pesquisas, no campo da Psicologia Social estão
localizados nessa rede epistemológica que se caracteriza, fundamentalmente, pela diversidade de
compreensões e pressupostos.
Dessa imensa rede de produções e possibilidades, fica a questão central acerca do conhecimento
psicológico que é produzido nos dias de hoje, e, essencialmente, o que é feito com essas produções.
Significa afirmar a necessidade de que esse saber não seja produzido para reforçar o paradigma
determinista daquela Psicologia individualizante supostamente já superado e compreender a
instrumentalização que muitas vezes é feita com as produções do conhecimento.
Dentro dessa pluralidade que envolve teoria e prática em Psicologia, há outro aspecto que merece
destaque não apenas pela sua importância, mas, principalmente, pela pouca discussão do assunto nos
meios acadêmicos e no próprio exercício profissional. Trata-se dos usos e efeitos que os diversos saberes
psicológicos produzem nos dias de hoje; ou seja, quais são os desdobramentos dessas intervenções, e
em última análise, o que é feito com esse conhecimento psicológico, compreendendo-o enquanto
instrumento de saber/poder. Significa afirmar a necessidade de entender a instrumentalização que pode
ser feita com esses saberes/poderes, partindo do pressuposto de que a intervenção psicológica é uma
ação política, portanto, que tem efeitos na sociedade como um todo.
Para tanto, torna-se fundamental ter como princípio que todo e qualquer conhecimento deve sempre
ser contextualizado de acordo com sua realidade social, já que inexiste indivíduo ou grupo apartado de
uma sociedade.
De acordo com Jacó-Vilela (1999), estas seriam questões de relevância:
(...) como o psicólogo aceitará/atuará frente a este encargo: será o estrito avaliador da intimidade,
aperfeiçoando seus métodos de exame? Ou lembrar-se-á que este sujeito também é sujeito-cidadão,
cujos direitos e deveres se constituem no espaço público, território onde perpassam outros discursos e
práticas que não o exclusivamente psicológico? (p. 17).
De forma explícita, constata-se a necessidade de retomar e também redimensionar o foco de análise
da Psicologia para não se incorrer em velhos reducionismos. Concomitantemente, esse questionamento
já sinaliza para o lugar da Psicologia mais ampliada que inclui a noção de cidadania, direitos e espaço
público.

Direitos Humanos: uma concepção contemporânea


Sinteticamente, é possível afirmar que os Direitos Humanos constituem-se como o conjunto de
conquistas resultante de uma luta coletiva de diversos atores sociais por uma cultura de respeito aos
direitos civis, políticos, econômicos, sociais e culturais. Entende-se aqui que os principais fundamentos
dos Direitos Humanos dizem respeito à igualdade e à dignidade humana, a partir de uma perspectiva
sócio-histórica, não naturalista e enquanto uma construção em determinado período da história mundial,
não sendo atributos naturais e inerentes ao ser humano.
Segundo Rabenhorst (2005):
A teoria dos Direitos Humanos é uma invenção da modernidade. Afinal, até o fim da Idade Média o
direito foi pensado praticamente em termos de deveres ou obrigações e não com pretensões ou interesse
subjetivos. Obviamente, isso não significa dizer que as culturas antigas não tenham defendido uma certa
concepção de justiça ou do respeito devido aos seres humanos. Contudo, a pressuposição

Apostila gerada especialmente para: Betânia Mueller 023.712.580-32


. 229
contemporânea de que todos os homens possuem o mesmo valor e que, por tal razão, são titulares de
um idêntico conjunto de direitos inalienáveis, era absolutamente estranha aos antigos (p. 205).
No mundo, o triste episódio da Segunda Guerra Mundial, com o saldo de aproximadamente 50 milhões
de pessoas brutalmente assassinadas, impôs às nações a necessidade de se pensar estratégias
internacionais que inibissem novos genocídios dessa proporção. Esse foi o embrião, no Ocidente, do que
se convencionou nomear de Direitos Humanos.
Fica evidente que a ideia de Direitos Humanos é moderna na história da sociedade ocidental. É
importante ressaltar que, nesse artigo, parte-se da concepção contemporânea de Direitos Humanos cuja
perspectiva principal é a da universalidade, indivisibilidade e interdependência destes direitos. De acordo
com Piovesan (2002), esta concepção deriva do pós Segunda Guerra Mundial, momento em que o mundo
se confrontou com inúmeras atrocidades oriundas dos regimes totalitários.
Este cenário aterrorizador mobilizou a comunidade internacional para a necessidade premente de
criação de um dispositivo normativo que pudesse frear estas violações, a partir do questionamento da
soberania estatal, tendo em vista que, na Segunda Guerra Mundial, o Estado foi o maior violador dos
Direitos Humanos acobertado por esta soberania nacional.
Neste sentido, houve um entendimento de que deveria existir um instrumento internacional que
garantisse a proteção aos Direitos Humanos de qualquer pessoa, independentemente de seu país de
origem. Portanto, em 1945, criou-se a mais importante organização internacional – Organização das
Nações Unidas (ONU) – com o objetivo de proteção aos Direitos Humanos, assim como manutenção da
paz e da segurança em âmbito mundial. Nesta conjuntura de construção de dispositivos normativos para
proteção internacional dos direitos, adota-se a Declaração Universal dos Direitos Humanos (DUDH) em
1948. Se este triste episódio da história mundial significou o rompimento com todos os direitos até então
conquistados, a Declaração surgia com o objetivo de reconstruir este esfacelamento, superando a
soberania dos Estados.
Nesta concepção contemporânea, a primeira característica diz respeito aos direitos dirigidos
universalmente para todas as pessoas, bastando a condição de ser humano para ser titular destes
direitos. No entanto, no contexto atual, há uma parcela significativa da população mundial que seus
direitos respeitados. Dessa forma, é imperativo reforçar uma questão anterior, apresentada por Arendt
(2000) que se refere primeiramente ao “direito a ter direitos”, ou seja, a imensa maioria da população
mundial desconhece o conteúdo dos Direitos Humanos como um conjunto de direitos conquistados
historicamente que lhe assiste ou não possui mecanismos de acesso ou reivindicação destes direitos.
Já a segunda característica sinaliza para a indivisibilidade dos Direitos Humanos, partindo do
pressuposto de inter-relação e interdependência destes direitos. Não basta que os direitos civis e políticos
estejam garantidos, faz-se necessário observar também os direitos sociais, econômicos e culturais
(DHESCs); se um direito é violado, os demais também o são. É necessário superar antigas dicotomias
que dedicavam lugares e prioridades distintas aos direitos civis e políticos e aos direitos econômicos,
sociais e culturais. Significa reafirmar que um direito não existe isolado dos demais e no caso de violação
de um direito, necessariamente, outros também estarão sendo violados.
Nesta perspectiva normatizadora, pode-se afirmar que principalmente a partir da Segunda Guerra
Mundial houve inúmeros avanços em âmbito mundial no que tange a adoção de Declarações, Tratados
e Pactos Internacionais de respeito e proteção aos Direitos Humanos, dos quais diversos países, inclusive
o Brasil, são signatários.
Após a Declaração Universal dos Direitos Humanos (DUDH), seguiram-se diversos outros dispositivos
normatizadores, valendo destacar os principais: Pacto Internacional sobre os Direitos Civis e Políticos
(1966),
Pacto Internacional sobre os Direitos Econômicos, Sociais e Culturais (1966), Convenção Internacional
sobre a Eliminação de Todas as Formas de Discriminação Racial (1968), Convenção sobre a Eliminação
de Todas as Formas de Discriminação contra a Mulher (1979), Convenção sobre os Direitos da Criança
(1989), dentre outros.
É importante salientar que a suposta contradição entre a existência de Pactos Internacionais distintos
para os Direitos Civis e Políticos e para os DHESCs, e a assertiva anterior a respeito da indivisibilidade
destes direitos, deve-se à bipolaridade da política mundial naquele período. A impossibilidade de diálogo
entre os blocos capitalista e socialista obrigou a ONU a viabilizar dois instrumentos diferentes,
contrariando a proposta inicial que visava um único instrumento detalhando os direitos previstos na
DUDH.
Assim, fica evidente que os Direitos Humanos não estão imunes ao contexto sociopolítico da sua época
e também são instrumentalizados de acordo com determinados interesses, como também ocorreu com a
Psicologia conforme já mencionado anteriormente. Na conjuntura da guerra fria, a Anistia Internacional
(2008) tece sua análise:

Apostila gerada especialmente para: Betânia Mueller 023.712.580-32


. 230
Os Direitos Humanos se transformaram em um jogo excludente entre as duas ‘superpotências’
envolvidas em uma luta ideológica e geopolítica para estabelecer sua supremacia. Enquanto um dos
lados negava os direitos civis e políticos, o outro rebaixava os direitos econômicos e sociais. Ao invés de
favorecer a dignidade e bem estar das pessoas, os Direitos Humanos eram usados como instrumento
para promover objetivos estratégicos (p. 4).
O bloco liderado pelos Estados Unidos (EUA) compreendia que os Direitos Civis e Políticos podiam e
deviam ser implementados imediatamente, enquanto que esta agilidade não se aplicaria aos DHESCs
que, necessariamente, demandariam um maior tempo para sua implementação. Em contraposição, o
bloco capitaneado pela União Soviética (URSS), em função da prioridade nas questões sociais em seus
regimes políticos, entendia que os direitos econômicos e sociais tinham primazia.
O que se observa a partir desta duplicidade ideológica de direitos – que teve origem no cenário político
mundial pós Segunda Guerra – refere-se a como os direitos são hoje compreendidos e quais os efeitos
dessa dicotomia histórica.
Considerando que os Direitos Civis e Políticos estão mais relacionados às liberdades individuais, ao
direito à vida, ao voto, à livre circulação, dentre outros, e que os DHESCs retomam o direito a uma vida
digna que inclua saúde, trabalho, educação, moradia, previdência social, assistência social, lazer, etc, é
possível reconhecer uma maior tolerância das pessoas e das próprias nações frente às violações dos
DHESCs se comparadas aos Direitos Civis e Políticos.
Esta cultura de aceitação pode estar relacionada à maior dificuldade dos países em cumprirem com
os DHESCs, posto que a conjuntura mundial atual aponta para o aumento significativo da dependência
econômica e política dos países periféricos/emergentes em relação aos seus financiadores.
No entanto, frente a essa cultura de aceitação é bastante pertinente o questionamento de Bobbio
(2004) em relação à suposta tolerância que se deve para a devida implementação dos direitos:
Um direito cujo reconhecimento e cuja efetiva proteção são adiados sine die, além de confiados à
vontade de sujeitos cuja obrigação de executar o “programa” é apenas uma obrigação moral ou, no
máximo, política, pode ainda ser chamado corretamente de “direito”? (p. 92).
De qualquer sorte, há que se resgatar a concepção contemporânea de universalidade e indivisibilidade
de direitos, não existindo nenhum tipo de valoração ou sobreposição de direitos, tendo em vista que todos
são igualmente relevantes para a promoção da igualdade e da dignidade humana enquanto principais
fundamentos dos Direitos Humanos.
Para tanto, toma-se como referência uma reflexão de Mondaini (2006) como indicativo de qual o lugar
dos Direitos Humanos na contemporaneidade:
Na atualidade, não obstante as inúmeras conquistas obtidas em torno da afirmação dos direitos
humanos, tanto no campo jurídico-legal como no plano cultural-ideal, continuam a se fazer presentes
críticas que parecem ignorar o fato de que o único instrumento capaz de medir o nível de civilidade
alcançado por uma sociedade – e seu progressivo distanciamento da barbárie – localiza-se exatamente
na capacidade que esta tem de fazer com que os seus concidadãos sejam protegidos pelo generoso
guarda-chuva dos direitos humanos (p. 12).
Ou seja, independente do país ou do contexto sócio-histórico vivido, podemos perceber hoje que os
Direitos Humanos funcionam como termômetro ou bússola norteadora que indica o grau de civilidade de
uma sociedade. Isso não se refere apenas ao respeito formal, pois importantes textos foram assinados
pelo Brasil, conforme já colocado anteriormente, sendo referências no assunto. Trata-se aqui da
construção de uma cultura que respeite os Direitos Humanos efetivamente para todas as pessoas.
Assim, além dos Direitos Humanos serem uma referência em relação ao progresso histórico de
determinada sociedade, é preciso compreender que o campo dos direitos é necessariamente uma arena
política como bem sinaliza Bobbio (2004): “O problema fundamental em relação aos direitos dos homens,
hoje, não é tanto o de justificá-los, mas o de protegê-los. Trata-se de um problema não filosófico, mas
político” (p. 43).
No cenário brasileiro, os primeiros sinais de movimentos sociais pelos Direitos Humanos datam da
década de 60 e 70, período político mais conhecido como os anos de chumbo. De acordo com Sader
(1988), frente a um Estado repressor e autoritário, novos atores sociais surgem enquanto estratégia de
resistência, principalmente a partir de movimentos ligados à Igreja, à esquerda e ao sindicalismo:
Com isso (os novos sujeitos políticos) acabaram alargando a própria noção de política, pois politizaram
múltiplas esferas do seu cotidiano. Apoiando-se nos valores de justiça contra as desigualdades
imperantes na sociedade; da solidariedade entre os dominados, os trabalhadores, os pobres; da
dignidade constituída na própria luta em que fazem reconhecer seu valor; fizeram da afirmação da própria
identidade um valor que antecede cálculos racionais para a obtenção de novos objetivos concretos (p.
312).

Apostila gerada especialmente para: Betânia Mueller 023.712.580-32


. 231
Tal digressão histórica baseia-se na necessidade de trazer elementos que colaborem para essa
proposta de investigação entre a Psicologia e os Direitos Humanos, a fim de identificar seus cursos,
percursos e principais pontos de convergência.

Psicologia e Direitos Humanos: pontos de convergência na teoria e na prática

Feito esse preâmbulo sobre a trajetória desenvolvido pela Psicologia como ciência e profissão e os
Direitos Humanos, enquanto direitos, deveres e cidadania para todos, pode parecer que não há ponto de
interseção entre esses campos. No entanto, considerando que, por excelência, a Psicologia trabalha com
seres humanos em suas mais diversas condições, situações e contextos, o mesmo se aplica aos Direitos
Humanos. Não se quer dizer aqui que se trata da mesma intervenção, mas, sim, do mesmo objeto de
trabalho, ou seja, o ser humano.
Elegendo a dignidade humana como eixo fundante dos Direitos Humanos, torna-se possível pensar
nessa meta também para a Psicologia, na medida em que esta trabalha para o desenvolvimento e a
melhoria do ser humano e suas condições de vida nas mais diversas esferas. A constatação de que
ambos os campos buscam, direta ou indiretamente, a dignidade humana reforça a similaridade entre a
Psicologia e os Direitos Humanos. Ou seja, o pensar teórico e a práxis cotidiana de ambos os campos
possibilita uma aproximação fundamental.
Nessa perspectiva, é de suma importância trazer a reflexão de Camino: “Os Direitos Humanos não
seriam uma questão externa à Psicologia, mas algo que se coloca diariamente em nossa prática
profissional e acadêmica. Nossa prática, nossa ciência tem a ver diretamente com a construção dos
Direitos Humanos”.
Nesse sentido, não só existe a interlocução entre Psicologia e Direitos Humanos, como também se
torna importante ressaltar que a prática profissional da Psicologia tem relação direta com a construção
dos Direitos
Humanos.
Isso permite avançar nessa análise e compreender que, do ponto de vista teórico-prático, uma
intervenção psicológica pode contribuir para construir ou não os Direitos Humanos de uma determinada
sociedade. Em outras palavras, significa afirmar que, o agir profissional pode incorrer em construção dos
Direitos Humanos ou no seu contrário.
Considerando ainda que o sofrimento humano tem sido um dos principais objetos de estudo e
intervenção da Psicologia, infere-se que frequentemente violações de Direitos Humanos são colocadas
aos profissionais, tendo em vista que, muitas vezes, violações vem acompanhadas de sofrimento e
adoecimento psíquico. Nessa lógica, fica o questionamento sobre o que foi e tem sido feito pelos
profissionais nessas situações de violações?
De um lado, levando em conta o paradigma individualista antes determinante na Psicologia, a resposta
pode ser bastante preocupante. Porém, de outro, avaliando as mudanças ocorridas na profissão e
fazendo uma análise contextualizada, a resposta pode ser menos inquietante e mais condizente com uma
Psicologia crítica.
Apenas a título exemplificativo dessa realidade, é possível tomar o Movimento de Luta Antimanicomial
como expressão da proximidade entre Psicologia e Direitos Humanos. Após décadas de violações de
direitos em instituições psiquiátricas, os trabalhadores dos serviços, usuários e familiares se organizaram
para enfrentar uma realidade cotidiana de violência e arbitrariedade.
Entendendo esse movimento como importante instrumento de mudança, foi através dele que se tornou
possível estabelecer diretrizes de intervenção com o objetivo de resgatar a dignidade dessas pessoas e
iniciar a Reforma Psiquiátrica. Funda-se, nesse movimento, um fecundo diálogo entre a Psicologia e os
Direitos Humanos, na medida em que se passou a questionar o tratamento dispensado nos hospitais
psiquiátricos, ensejando uma verdadeira mudança de paradigma na saúde mental brasileira.
Isso evidencia a necessidade dos Direitos Humanos, cada vez mais, integrarem a formação a ação
profissional da Psicologia, para fortalecer e qualificar o pensar teórico e a práxis psicológica. Essa nova
perspectiva de análise e compreensão do ser humano, baseada no diálogo entre a Psicologia e os Direitos
Humanos, é fundamental para se construir uma sociedade menos injusta e que tenha como princípio
norteador o respeito à dignidade humana.

Psicologia e a Democracia

Tão antiga quanto o próprio ser humano, a busca da igualdade talvez seja o propósito mais complexo
e controvertido que se impõe na vida em sociedade. Ser igual implica, por pressuposto, a existência do
plural: ao menos duas pessoas são necessárias, interagindo no mesmo espaço de vida. E essa

Apostila gerada especialmente para: Betânia Mueller 023.712.580-32


. 232
convivência interpessoal, que tem sua significação afetada por uma multiplicidade de variáveis, é objeto
de estudo fundamental na Psicologia.25
A compreensão do conceito de igualdade social conduz um inevitável exame de modos de organização
social do Estado, a uma percepção necessária das relações entre participação e emancipação e à
constatação de sua indissociabilidade em relação às ideias de liberdade e cidadania. Não sem o risco de
algum reducionismo involuntário, é impossível analisar essas vinculações conceituais à luz do papel que
a Psicologia pode desempenhar como área detentora de conhecimento promissor para a compreensão e
mesmo a militância no contexto da cidadania. Esclarecer esses mecanismos, todavia, torna-se impossível
se se ousa prescindir das contribuições dos antropólogos, dos sociólogos e dos historiadores, para dizer
o mínimo. Qualquer tentativa apenas vertical dentro da Psicologia torna-se inócua em razão da própria
natureza do fenômeno: o aprofundamento da análise está condicionado a um mínimo de horizontalidade
ditado pela multidisciplinaridade. Nessa direção, o presente ensaio incluirá, preliminarmente, uma
incursão às raízes históricas daquilo que constitui o tipo de organização sócio-política mais comum na
sociedade contemporânea, sempre buscando desvelar implicações para a compreensão da cidadania.

Evolução Histórica da Organização Social do Estado: liberalismo e democracia em questão

A literatura mostra flagrante discrepância entre os conceitos de Estado liberal e Estado democrático.
No primeiro, o particularismo do poder se define na ideia da propriedade, enquanto no segundo a
distribuição do poder se contrapõe às ideias de autocracia e oligarquia. Em cada qual, a questão da
cidadania deve ser tratada sob ótica diversa. Como no caso brasileiro o modelo de Estado, o tipo de
regime e a ideologia navegam ora em águas liberais, ora em águas democráticas, não é sem motivo que
se torna imprescindível, aqui, melhor caracterizar esses aspectos. Até porque, mais recentemente, essa
pauta de discussão se amplia pela inclusão de um novo item, qual seja a validade atual da dicotomia
direita-esquerda: o tema começou a ser veiculado no último livro Bobbio(1995) e invade os meios
acadêmicos de todo o Ocidente.
Para o mesmo Bobbio (1990), por liberalismo entende-se uma concepção de Estado possuidor de
poderes e funções limitadas, em contraposição ao Estado absoluto. Por democracia, entende-se uma
forma de governo em que o poder está nas mãos da maioria, em contraposição às formas autocráticas.
Todavia, a compatibilidade projetada entre liberalismo e democracia nem sempre tem sido real através
da história. Originalmente, o liberalismo paternalizado por Locke e presente na Revolução Francesa, está
representado pela conquista burguesa do poder político, tomado à aristocracia e implicando um modelo
de cidadania que alcança apenas o proprietário (Buffa). O conceito de homem livre se vincula à ideia de
que todos são proprietários de seu próprio corpo e, por extensão, de seu próprio trabalho: nesse sentido,
a liberdade aí implícita é assegurada pelo arbítrio pessoal quanto ao uso desse organismo auto-
apropriado. Uma liberdade evidentemente limitada, até porque coloca um sentido absolutamente ingênuo
de autodeterminação, especialmente quando se sabe, na Psicologia como em toda ciência social, que as
contingências subjacentes ao ir-e-vir não são apenas internas: normas, regras, convenções, crenças,
práticas e tantas variáveis mais entram nessa composição causal. O exemplário dessa pseudoliberdade
é imenso: para qualquer trabalhador assalariado, um contrato com o proprietário representa, quase
sempre, uma condição de subordinação e não de igualdade. Alçar à condição de cidadão, para ambos,
não tem o mesmo preço. Historicamente, na verdade, o cidadão acaba sendo tão-somente proprietário.
É por essa razão que o Zeitgeist da Revolução Francesa, acabou disseminando-se a ideia de uma
proposta educacional para os proprietários e outra para os não-proprietários ou cidadãos de "segunda
categoria", se isso é possível. A divisão parcelar do trabalho, mormente na Inglaterra de Adam Smith
(1723-1790) serve como justificativa (negativa) para que o Estado imponha uma educação básica que
inclui o trabalhador: supõe-se que o povo (limitadamente) instruído é ordeiro, de modo que o que se
propõe para a maioria da população é apenas o mínimo. E o modelo liberal para fazer do trabalhador, na
verdade, um cidadão passivo que, apesar de tudo, tem alguns poucos direitos (Buffa).
O liberalismo, enquanto concepção de Estado, apresentou-se robustecido após a Revolução
Francesa, consolidando-se como modelo teórico compatível com a realidade moderna. Todavia, por mais
que tente ser aliado, esbarra em alguns pressupostos da democracia enquanto forma de governo muito
mais antiga e que tem nuanças particulares em cada Estado onde se aplica. No Brasil, por exemplo, a
aliança liberal-democrata é reconhecível nas ações de governo, que ora são centradas na privatização
de serviços e empresas, com um consequente desmonte de um aparato estatal considerado arcaico e
ora, paradoxalmente, se dirigem à personificação do governo enquanto mantenedor do controle social do
Estado. Para sumariar, o peso histórico da democracia assegura a ela, hoje, a condição de forma de

25
Texto adaptado de Kester Carrara, disponível em http://www.scielo.br/

Apostila gerada especialmente para: Betânia Mueller 023.712.580-32


. 233
governo privilegiada mundialmente, não sem que permaneça aberta a discussão de uma verdadeira
tipologia de regimes democráticos particulares, em alguns dos quais uma aliança mais ou menos intensa
com o liberalismo é admitida. Para Bobbio (1990), ao longo de todas as transformações que os séculos
impuseram à democracia desde a Grécia antiga, jamais se alterou seu aspecto descritivo (de governo do
povo), embora tenha sido alterado o seu aspecto valorativo (que se refere à amplitude da democracia).
Dito de outro modo, o que essencialmente muda de um modelo democrático para outro é a intensidade
da abertura democrática e a composição da representatividade popular no governo. Sob o manto do
discurso democrático, as práticas político-ideológicas diferem, de modo que, se se pretende propor traços
preliminares de um modelo de contribuição da Psicologia à construção da cidadania, não se pode alienar
à análise a identificação do modelo brasileiro de governo. No mínimo, por essa razão, é fundamental ver
em que casos é consequente a necessidade de um afastamento da democracia liberal e uma
aproximação à democracia social e em que casos há pressupostos que permitem uma convivência de
critérios de ambas as tendências. Para Bobbio (1990), a igualdade na liberdade é uma das formas de
igualdade não só herdada, mas requerida pelo liberalismo: nas suas palavras, "cada um deve gozar de
tanta liberdade quanto compatível com a liberdade dos outros..."(p. 39). Essa concepção, básica a
qualquer forma democrática, inspira normas constitucionais de igualdade perante a lei e igualdade de
direitos, de onde a importância de se considerar, numa proposta de trabalho de Psicologia, os conceitos
de direitos e deveres comuns, sem privilégios em função de propriedade ou posse, como filosoficamente
implícitos em qualquer processo de conquista da cidadania.
Em última análise, o conceito e o modelo de cidadania que se veicula no presente trabalho pretende
ancorar-se sob as garantias básicas de uma forma de governo democrática: isto deve ser razão mínima
para que o leitor se familiarize, mais adiante, com facetas da proposta que necessariamente implicarão
certo desapontamento para quem se acomoda sob a manutenção de alguns privilégios. A ideia de
proteção dos direitos humanos, embora nascida sob a égide do liberalismo, é hoje condição indispensável
ao processo democrático. Bobbio (1990) assevera que ideias liberais e o que ele chama de método
democrático combinaram-se gradualmente de tal modo que, "se é verdade que os direitos de liberdade
foram desde o início a condição necessária para a aplicação direta das regras do jogo democrático, é
igualmente verdadeiro que o desenvolvimento da democracia se tornou o principal instrumento para a
defesa dos direitos de liberdade"(p. 44). A veracidade a afirmação se comprova pelo fato de que hoje
apenas os Estados resultantes de revoluções liberais são verdadeiramente democráticos e apenas os
Estados democráticos amparam seguramente os direitos humanos, o que se denota do fato de que todos
os Estados autoritários sustentam duas características: são antiliberais e são antidemocráticos. Não seria
preciso reiterar que no caso do Brasil a democracia é incipiente, a não ser para esclarecer que um dos
passos básicos para a conquista da cidadania se constitui na consolidação dessa forma básica de
governo mediante ações políticas intensas e de longa duração. As manipulações na superfície da política
monetária, sem que se invista em mudanças estruturais (a questão econômica tem sido tratada como se
fosse um problema de software, quando parece que é também de hardware), exemplificam a dificuldade
de romper com o modelo de domínio capitalista que não pode ser simplesmente importado sem ajustes
para a realidade nacional. Razões existem, de sobra, para crer no argumento de Chauí (1986), que mostra
a distância entre a democracia concebida e de discurso e a democracia praticada. Na verdade, a
caracterização capitalista do Brasil se mostra pela constituição autoritária e hierarquizada de nossa
sociedade, onde os direitos do homem e do cidadão simplesmente não existem. São dispensados pela
elite, de vez que ela não precisa de maiores direitos porque tem privilégios, estando, pois, acima deles; e
não existem para a maioria da população, porque suas tentativas de consegui-los são sempre encaradas
como problemas de política e tratadas com todo o rigor do aparato repressor do Estado (p.2). Esperar
como dádiva do Estado as condições básicas para alcançar a condição da cidadania é pouco promissor.
Além disso, é mesmo inseparável do conceito de cidadania a ideia de conquista e não de dádiva.

Os caminhos da emancipação: da cidadania oferecida à cidadania conquistada

O conservadorismo populista tão ao gosto, nos quase quinhentos anos de Brasil, à maioria dos homens
que ocuparam cargos públicos, tem feito disseminar o conceito de que direitos são matérias de
concessão. Sob o discurso da ampliação de direitos (de voto, de greve, de amparo à saúde etc.), pratica-
se o manobrismo da barganha eleitoreira, como se fosse lícito prometer à população o que já lhe pertence
e exigir dela o que não pode ser objeto de permuta. Esse imaginário político converte-se em triste
realidade que se dissemina ao longo da história brasileira, fazendo crer que a cidadania é algo que pode
acontecer, de forma mais ou menos plena, de conformidade com a boa vontade (a "vontade política") do
governante. Tais práticas, aliadas ao descaso para com a educação e a cultura, têm tornado fáceis as

Apostila gerada especialmente para: Betânia Mueller 023.712.580-32


. 234
ações desses verdadeiros usurpadores da consciência popular, constituindo-se em agentes que
atravancam o caminho mais curto e mais seguro para a conquista da cidadania: a participação.
Na lição de Demo (1988), o processo participativo define a cidadania organizada, que faz prevalecer
o coletivo sobre o individual, sendo objeto de conquista e não de consentimento. Acrescenta que o
conceito de cidadania, embora possua laivos conservadores históricos desde os gregos, que privilegiam
como cidadãos apenas um grupo de elite, precisa hoje ser entendido como condição desejável à
sociedade integral. Nesse sentido, deve escapar à tutela do Estado e emergir a partir da iniciativa dos
interessados, dos desiguais, dos excluídos (p.70). O Estado, na figura das instituições e das pessoas que
ocupam o poder, será efetivamente mobilizado na direção do aprimoramento da condição social coletiva,
a partir das ações organizadas dos membros dessa sociedade. Descaracteriza-se, portanto, o mecanismo
da concessão, que subverte a ordem do processo e funciona como engodo que cobra ao cidadão um
compromisso individual. Para Weffort (1981), é razoável supor que o processo de democratização por via
concessiva esteja próximo do esgotamento, de modo que se prioriza a perspectiva da democratização
progressiva, conquistada, capaz de privilegiar o aprimoramento de recursos institucionais de
representação e participação. Assim, a construção de qualquer proposta nova passa pela recusa ao
modelo de cidadania regulada, na medida em que esta é vertical e vai do topo às bases da pirâmide social
e na medida em que carece de representatividade popular, uma vez que oferecida pela elite e não
conquistada pela maioria social.
Privilegiar a participação e rejeitar a cidadania concedida são procedimentos estratégicos para
assegurar legitimidade ao processo, mas são igualmente instrumentos para se compreender que outra
díade, moralização-educação, embora condições necessárias, não pode ser vista como constituindo
solução única e isolada para a instituição de uma sociedade igualitária. A defesa ingênua da educação,
como mecanismo isoladamente suficiente para a transformação social, constitui raciocínio que elide a
questão do poder (Arroyo, 1988) e se reduz a um pedagogismo estéril, descolado do econômico, do
político, do ideológico. Desse modo, este trabalho pretende deixar claro que a Psicologia apenas poderá
instrumentalizar sua contribuição à construção da cidadania na medida em que privilegie um processo
educacional contextualizado sócio politicamente e formalize a participação enquanto estratégia de
trabalho.

Educação, participação e igualdade: implicações conceituais para a compreensão da cidadania


a conquistar

Weffort (1981)- na época professor adjunto do departamento de Ciências Sociais da USP- analisando
a questão dos direitos sociais e participação e diante de um Brasil em que "nem o liberalismo, nem a
classe operária e, talvez menos ainda, nem a burguesia apresentam a nitidez que podemos perceber nos
países mais modernos" (p-139), crê prudente indagar se poderia os trabalhadores contribuir para a
conquista da democracia. Estava implícita a concepção de participação, embora ainda fosse incipiente
uma efetiva forma de governar democrática, ainda hoje em processo de consolidação. Desde esse
momento da história brasileira, em que a comunidade acadêmica manifestava clara consciência da
necessidade de democratização, pouca coisa se fez quanto a instrumentalizar respostas práticas à
pergunta de Weffort, sem os riscos do paternalismo. Caminhou-se pela via artificial do consentimento
administrativo, político e legal: o mecanicismo e o artificialismo do direito consentido provêm justamente
do seu caráter de benesse da autoridade. Não se pode fazer o mesmo com a cidadania, na sua inteira
acepção: quando se passa da conquista à concessão, a legitimidade desmorona. Até mesmo quanto ao
próprio conceito de autoridade, que não se impõe (porque transforma-se em autoritarismo), mas se
conquista.
A emancipação social, por essa ótica, requer o envolvimento, a participação, a crença, a consciência
e muito trabalho de todos quantos a vejam como condição que não privilegia o particular, até porque
quando apenas alguns levam vantagem, não só a maioria perde, mas todos perdem, ainda que em
momentos e instâncias diferentes (Carrara).
A procura da igualdade implica, por tautologia, que se parta de uma condição de desigualdade,
ademais uma fonte estrutural de mudanças, tal como aponta Demo (1988): da reunião dos desiguais
nasce a conquista participativa, vinculada a uma visão de política social que implica interligação à questão
da infraestrutura econômica, na medida em que toda política efetivamente social significa mudanças na
ordem econômica. Na prática, pode-se constatar medidas econômicas que distribuam ou concentrem
renda, por exemplo, com as consequentes implicações as tensões sociais. Para Demo (1988), a
construção participativa de uma sociedade majoritariamente desejada ressalta a dimensão política, mas
não sua supervalorização desligada do processo econômico, bem como ressalta a importância do
conceito de desenvolvimento, em contrapartida a crescimento (porque para o primeiro concorre a

Apostila gerada especialmente para: Betânia Mueller 023.712.580-32


. 235
necessidade de participação e a ideia de processo qualitativo, como tal, dinâmico). Essa política social
passa pela necessidade de inserção no mercado de trabalho, pela conquista política e pela rejeição a
certos vícios como o assistencialismo (que coíbe a autopromoção, instituindo a prática da concessão, da
tutela e da manipulação). Acautela com propriedade o mesmo autor (Demo, 1988), que a política social
é apenas parcialmente política pública; na sua maior parte é iniciativa da sociedade civil, apesar do
Estado, ou mesmo à revelia ou contra o Estado. Além disso, constitui erro estratégico imaginar possível
ficar esperando aflorar toda a potencialidade para o desenvolvimento: não se pode permanecer à espera
de iniciativas mágicas, até porque as comunidades excluídas nem sempre se comprometem
imediatamente com mudanças, aparentando uma alienação enganosa, que não lhe é inerente por
natureza, mas decorrente de repressão histórica de quem manipula o poder. Não se pode deixar de
reconhecer, nesse sentido, que as comunidades pobres também se encontram sob efeitos alienantes,
não decorrentes, obviamente, da ausência de projetos e expectativas, mas porque a manipulação que
lhes impõe o poder tem as malhas fortes da legislação, do saber consensualmente constituído e do
domínio das relações de troca entre capital e trabalho.
Pelo menos por estas razões, a conquista da cidadania deve passar (ao contrário da expectativa
ingênua frequentemente veiculada de que as mudanças todas brotarão naturalmente) por um processo
construído, projetado com a utilização ótima do conhecimento disponível e com a integral participação de
todos quantos estejam conscientes de sua importância. Não é sem grande trabalho e sem projetos que
se mudam, por exemplo, os conceitos do fatalismo religioso (que concebe a divisão de classes como
inerente à natureza humana) e da prevalência do poder econômico (onde, não sem motivos, mas
desprotegida, a maioria excluída se auto-obriga à sujeição nas relações de trabalho).
Entre as características básicas da participação reitera-se a de processo coletivo planejado. É um
processo na medida em que não existem objetivos terminais na participação social: trata-se de algo
interminável, que revela novas facetas e possibilidades a cada etapa vencida. Deve ser coletiva em razão
da amplitude e generalidade dos ganhos que se esperam: embora importante, a participação individual
nem sempre traz ganhos que possam ser distribuídos equitativamente por toda a população. E deve ser
planejada, porque a participação na busca pela cidadania implica necessariamente a utilização de todos
os instrumentos práticos e conceitos teóricos acessíveis e oriundos das mais diversas fontes organizadas
de conhecimento: as diversas áreas científicas, por exemplo, terão seus efeitos diluídos, desperdiçados
ou confundidos, se não forem mobilizados de forma criteriosa e orientada.
Ao mesmo tempo, embora se possam encontrar resistências localizadas em diversos setores do
aparato estatal, não há Estado que seja completamente monolítico. Com Demo, pode-se assegurar que
uma visão histórica de Estado mostra que sempre há brechas a explorar mediante uma forma organizada
de participação.
Uma das brechas fundamentais, mas não a única, através da qual a população pode aprimorar seus
projetos de conquista da cidadania é a Educação. Não se pense, todavia, no processo educacional como
veículo único para tal conquista. Se assim fosse, qualquer projeto que mudasse certas estatísticas
brasileiras seria suficiente: embora desejável, não será o fato de que deixemos de ter trinta milhões de
analfabetos funcionais e vinte milhões de analfabetos absolutos que concederá ao país o título de Brasil-
cidadão. Claro está que a reversão desse quadro só pode resultar de um esforço de toda gente séria e
comprometida com mudanças, mas além disso existe muito trabalho a ser feito. A Educação formal, pelo
menos, não pode eliminar completamente as tentativas de "domesticação ideológica", apesar de que
funcione como espaço seguro de discussão. Conforme Arroyo, não é fácil defender, como não é fácil
afastar "as formas sinuosas e sutis através das quais a vinculação entre Educação e cidadania, como
pré-condição para a participação, vem agindo durante séculos para justificar a exclusão da cidadania, a
condenação das camadas populares à condição de incivilizados, de não-aptos como sujeitos de história
e de política, bem como a legitimação da repressão..." (p.40). De certo modo, o escudo da incivilidade,
da falta de preparo, da ignorância às letras, tem sido ruidosamente manipulado pela classe dominante. A
população, nesse sentido, não estaria "preparada" para a ampliação de direitos. No discurso político, a
necessidade de educar é ressaltada, mas na prática o suporte à Educação é removido enquanto medida
que impede a proliferação de gente que raciocina e que, por consequência, passa a questionar a realidade
vigente. Para Arroyo (1988), os trabalhadores já foram considerados como objeto de amor, caridade,
filantropia e educação, o que caracteriza uma submissão das camadas populares à condescendência e
benesses da classe dominante: manter essa condição de subserviência tem sido mais uma questão
política do que pedagógica. Lamentavelmente, por essa vertente, o ideal de educação política da criança
não seria o de prepará-la para participar do jogo do poder, mas para renunciar a ele e dedicar-se apenas
a uma convivência apolítica e fraterna. O que, convenhamos, não assegura uma sociedade igualitária.
Também não é suficiente uma Educação, tal como hoje se privilegia, centrada na transmissão de
conhecimento e/ou na socialização. Ambos caracterizam o reprodutivismo até aqui tão denunciado: no

Apostila gerada especialmente para: Betânia Mueller 023.712.580-32


. 236
primeiro caso, retira-se a intocabilidade do edifício da ciência; no segundo, a reprodução de costumes,
atitudes e expectativas da geração precedente, preservando a mesma divisão entre os privilegiados e os
desprivilegiados (Demo).
Definitivamente, a função precípua da Educação é de ordem política, como condição ao
desenvolvimento da participação e no melhor sentido que o conceito de política possa ter. Num bom
projeto de cidadania, alguns componentes básicos incluem a noção de formação e não de adestramento;
a noção de sujeito social e não de recipiente passivo do saber; a noção de conquista e não de concessão
da cidadania; a noção de direitos e deveres do cidadão; a noção de democracia como forma de governo
melhor habilitada a tornar possível a participação; a noção de liberdade, de igualdade e de comunidade,
que leva à consolidação de ideologias comprometidas com a redução de diferenças sociais. Demo aduz
ainda que a Educação deve estar comprometida com construir "gente", numa dimensão que vai para além
de simples trabalhadores treinados, pessoas bem comportadas, seres informados.
Torna-se evidente, pelo exposto, que o processo educacional é instrumento de vital importância para
a construção da cidadania apenas se se ancorar na preparação para a vida em comum, se se delimitar
enquanto espaço para a livre discussão das relações interpessoais, se contemplar a ideia de oferecer
instrumental efetivo à construção participativa da cidadania e se levar em conta integralmente o contexto
político-econômico típico da democracia vigente.
Entretanto, não se pense que brincando de democracia na escola o cidadão aprenderá a construir a
democracia (Arroyo): da simulação deve-se passar apropriadamente à prática no convívio social, único
lugar onde os efeitos da aprendizagem política se concretizam enquanto conquista da cidadania.

Caso Brasil: a relevância da Psicologia na construção da verdadeira cidadania

O rol de pressupostos para uma ação da Psicologia em qualquer projeto de construção da cidadania
pode parecer mais amplo que a dimensão da própria ação. Afortunadamente, não há excesso de zelo em
se delimitar com clareza as razões e implicações, menos estratégicas que éticas, dos principais
mecanismos em jogo no processo. Em particular no caso brasileiro, onde a Psicologia tem curta história
e, via de regra, sofre as mazelas das adaptações teóricas importadas, não é sem tempo que a consciência
ética do psicólogo seja estimulada para agir de modo consequente. A partir da concepção de que constitui
cidadania a qualidade social de uma sociedade organizada sob a forma de direitos e deveres majoritários
e não o que é engodo corriqueiro-uma série de valores do psicólogo enquanto pessoa (por mais virtuosos
que possam parecer), o profissional deve buscar suas fontes, de qualquer natureza teórica, de modo a
pensar um projeto que implique o desenvolvimento coletivo.
Respeitados os pressupostos sócio-políticos do projeto, o passo seguinte consiste na escolha da
natureza teórica do trabalho. Será mais progressista, mais "engajado", politicamente correto, trabalhar
com esta ou aquela abordagem? Há alternativas teóricas que são incompatíveis com um trabalho
comunitário? Desafortunadamente ou felizmente, não há respostas prontas para estas questões. No
entanto, seguramente não é a linha teórica que delimita a amplitude e alcance de um projeto de
construção da cidadania, mas uma demarcação clara da conquista pretendida. Ajuda muito tentar
responder com nitidez à tradicional e sempre atual questão: a quem a Psicologia estará servindo nesse
momento? Se a resposta representar avanços coletivos na direção da democracia, da igualdade e da
cidadania, qualquer obstáculo relacionado à natureza teórica poderá ser superado.
O trabalho básico do profissional estará centrado no colocar integralmente o conhecimento acumulado
em Psicologia a serviço dos setores majoritários da população que reivindicam mudança: nessa direção,
todas as vertentes teóricas têm contribuições a oferecer, sem distinção. Ainda que nenhuma delas,
isoladamente, possa responder completamente a todas as dúvidas formuladas, o conhecimento em
Psicologia possui hoje argumentos sólidos para inúmeros problemas. Além disso, cabe ao bom
profissional a necessidade de ser transparente às contribuições teóricas divergentes, desde que
assentadas em pelo menos uma de duas virtudes: bons dados e argumentação sólida. Qualquer outra
postura pode implicar ortodoxia infrutífera: já se disse que a Psicologia atual possui diversas construções
teóricas sérias, porém igualmente possui alguns modismos perigosos. Finalmente, nessa questão da
preferência teórica, a própria multiplicidade de concepções é uma questão de direitos e liberdade de
pensamento implícita na ideia de cidadania. Aliás, inserida na constituição brasileira: há psicólogos de
todas as formações e espera-se que a pluralidade das teorias adotadas possa representar uma vocação
democrática da própria área.
Outra consideração refere-se ao local de atuação do psicólogo: deve atuar no ambiente natural, na
clínica, em organizações? No caso da cidadania, onde é seu espaço de trabalho? Também neste caso a
resposta deve compatibilizar-se com os resultados esperados. Sabe-se que quanto mais distante da
realidade onde se dá o fenômeno, tanto mais demorada ou difícil a transferência de resultados. Nesse

Apostila gerada especialmente para: Betânia Mueller 023.712.580-32


. 237
sentido, a clínica seria a última alternativa a ser escolhida, sendo primeira o cotidiano social. Entretanto,
por um lado não há muito tempo para escolhas sutis e, por outro, todas as instâncias devem ser somadas
para o resultado total, porquanto a busca da cidadania deve estar representada em todos os lugares e
momentos. A escola, por exemplo, é local privilegiado para o trabalho psicopedagógico, mas trabalhar o
acadêmico sem trabalhar a família constitui ver apenas meio corpo do problema.
Um terceiro aspecto é o que responde à questão da necessidade ou não de uma subárea ou uma
disciplina especificamente preocupada com a questão da cidadania. Nem isto é necessário, nem
recomendável. Por um lado, restringiria o número de profissionais envolvidos, por outro descaracterizaria
a Psicologia como área complexa de conhecimento, porque a mutilaria denotando que constitui um
mosaico de pequenas e específicas psicologias (se essa metáfora é possível). Assim, o que já se tem é
mantido: trabalham-se momentos importantes em que a consciência social aflora quando se estuda a
Psicologia do Desenvolvimento; discutem-se os mecanismos que facilitam ou dificultam a aquisição de
conceitos quando se está no campo da Psicologia da Aprendizagem; centraliza-se a discussão nos
valores que sustentam a dinâmica grupal, quando se está na Psicologia Social; encara-se a ética do
assegurar ao cliente instrumentos para que possa lutar por seus objetivos, respeitados os direitos
daqueles com os quais convive, quando se trabalha com qualquer conjunto de teorias e técnicas
psicoterápicas na clínica; identificam-se reivindicações e oferecem-se procedimentos de contra controle
ao poder vigente, quando se opta pela Psicologia Comunitária; cuida-se para não ceder a um
comprometimento com o poder econômico da empresa ou instituição, quando se pratica a Psicologia das
Relações Humanas; aprofunda-se a dimensão da crítica social, sem deixar de lado o aprimoramento da
competência, quando se atua como psicólogo escolar. Enfim, o exemplário é tão infinito quanto as
possibilidades da própria Psicologia, tornando-se absolutamente desnecessário (embora seja possível)
delinear um campo particular para a construção da cidadania. O que se pode veicular, a título de maior
organização, são projetos nacionais ou regionais, com a possibilidade de envolvimento de uma
multiplicidade de profissionais. Nesse caso, o projeto maior desmembra-se em campos de atuação que
aproximem afinidades teóricas ou estratégicas naturais em vista da finalidade geral, que é a construção
da cidadania a partir da contribuição do psicólogo.
Neste ponto, duas observações importantes precisam ser feitas:
1. Não se pense na possibilidade de um conjunto de sugestões de trabalho enquanto um receituário;
2. Não se imagine que a atuação isolada do psicólogo pode resolver a questão da cidadania.
Respectivamente, não existe receituário em Psicologia, até porque são tão dinâmicas as relações
interpessoais que sempre há aspectos novos em jogo e como já se expôs, a busca da cidadania é
conquista e, como tal, implica participação, o que, por sua vez, é um processo coletivizante que recebe
múltiplas influências.
De qualquer maneira e embora este abreviado ensaio não diga respeito a fórmulas prontas de atuação,
aparenta clareza a ideia de que em qualquer subárea ou disciplina, um traço fundamental é necessário:
o sujeito do processo (a coletividade) precisa cientificar-se e aprender a lidar com os mecanismos sociais
que dão origem a limites, normas convenções, leis e regras, que dizem respeito ao seu próprio
comportamento no cotidiano. Ou seja, em todo trabalho da Psicologia direcionado à busca da cidadania,
um conjunto de procedimentos deve estar disponível para tornar claro, por exemplo, como se dão as
relações de trabalho, como o Executivo administra, como o Legislativo produz leis, como o Judiciário as
aplica. Mais: como e por que o mesmo Executivo deixa de administrar, por qual razão o Legislativo não
fiscaliza e o que emperra o Judiciário. Ainda mais: qual o mecanismo pelo qual todas as ações (aqui
minimamente exemplificadas) institucionais públicas e privadas afetam diretamente o cidadão. E,
sobretudo, muito mais importante: de que maneira, na busca da cidadania e, com o auxílio instrumental
da Psicologia, as pessoas podem organizar-se e atuar efetivamente para a mudança da realidade
presente. No Brasil, o caminho passa pela elaboração multidisciplinar de um grande Projeto de
Construção da Cidadania, que muito bem pode ser liderado pela Psicologia, envolvendo o Conselho
Federal, os Regionais, os cursos de Psicologia, os departamentos de Psicologia de outros cursos. Um
congresso preliminar e uma atuação forte dos conselhos pode polarizar motivações para a
operacionalização das frentes de atuação, sem que se corra o risco de uma proposição unilateral. Um
fórum inicial de publicações sobre o tema da cidadania, sugeridas algumas preocupações centrais, pode
funcionar como estratégia geradora de atividades da categoria. No caso brasileiro, estamos seguros de
que a lamentável grandeza da demanda pela cidadania, aliada à configuração político-ideológica
favorável ora instalada e à potencialidade que se reconhece na categoria dos psicólogos, podem, juntas,
catalisar grandes mudanças. Afinal, o que todos nós profissionais buscamos é, também, uma Psicologia
cidadã.

Apostila gerada especialmente para: Betânia Mueller 023.712.580-32


. 238
Psicologia social.

A psicologia é usualmente definida como ciência do comportamento humano e a psicologia social como
aquele ramo dessa ciência que lida com a interação humana. Um dos maiores propósitos da ciência é o
estabelecimento de leis gerais por meio da observação sistemática. Para o psicólogo social, tais leis
gerais são desenvolvidas a fim de descrever e explicar a interação social. 26

Implicações para uma ciência histórica do comportamento social

Sob a luz dos presentes argumentos, a tentativa contínua de construir leis gerais do comportamento
social parece mal direcionada, e a crença associada a ela de que o conhecimento da interação social
pode ser acumulado como nas ciências naturais revela-se injustificada. Em essência, o estudo em
psicologia social é fundamentalmente um empreendimento histórico. Estamos essencialmente engajados
em incontáveis questões contemporâneas. Utilizamos metodologia científica, porém os resultados não
são princípios científicos no sentido tradicional. No futuro, historiadores poderão voltar-se para tais relatos
do passado a fim de alcançar uma melhor compreensão acerca da vida nos dias atuais. Entretanto, é
provável que os psicólogos do futuro encontrem pouco valor no conhecimento contemporâneo. Esses
argumentos não são puramente acadêmicos e não se limitam a uma simples redefinição de ciência. Aqui
estão implicadas significantes alterações na atividade de campo. Cinco dessas alterações merecem
atenção.
Entre psicólogos acadêmicos encontra-se difundido um preconceito contra a pesquisa aplicada, um
preconceito que é evidenciado pelo enfoque dado à pesquisa pura pelos periódicos de prestígio e pela
dependência de promoção e manutenção de contribuições à pesquisa pura em oposição à pesquisa
aplicada. Esse preconceito baseia-se, em parte, na suposição de que a pesquisa aplicada é de valor
transitório. Enquanto esta se limitaria a resolver problemas imediatos, a pesquisa pura contribuiria para
um conhecimento básico e duradouro. Do ponto de vista atual, o solo no qual se assentam tais
preconceitos não é merecedor de respeito. O conhecimento que a pesquisa pura se dedica em
estabelecer é também transitório; generalizações nessa área de pesquisa geralmente não perduram. A
tal ponto que, quando generalizações da pesquisa pura têm grande validade transhistórica, podem estar
refletindo processos de interesse periférico ou importantes para o funcionamento da sociedade.
Psicólogos sociais são treinados para usar ferramentas de análise conceitual e metodologia científica
a fim de explicar a interação humana. No entanto, dada a esterilidade em aperfeiçoar os princípios gerais
ao longo do tempo, essas ferramentas mostram-se mais produtivas quando usadas na resolução de
problemas de importância imediata para a sociedade. Isso não implica que tais pesquisas devam ser de
alcance restrito. Um defeito fundamental de grande parte das pesquisas aplicadas é que os termos usados
para descrever e explicar são relativamente concretos e específicos para o caso em mãos. Enquanto os
comportamentos concretos estudados pelos psicólogos acadêmicos são frequentemente mais triviais, a
linguagem explicativa é altamente geral, e assim mais amplamente heurística. É assim que os argumentos
presentes sugerem uma intensa focalização em assuntos sociais contemporâneos, baseados na
aplicação de métodos científicos e ferramentas conceituais largamente generalizadas.
O objetivo central da psicologia é tradicionalmente encarado como a predição e o controle do
comportamento. Do nosso ponto de vista, esse objetivo é despropositado e oferece pouca justificativa
para a pesquisa. Princípios do comportamento humano podem ter valor preditivo temporalmente limitado,
e seu alto conhecimento pode torná-los impotentes como ferramentas de controle social. Todavia,
previsão e controle não precisam servir de pedras angulares do campo. A teoria psicológica pode
desempenhar um papel excessivamente importante enquanto dispositivo de sensibilização. Pode
esclarecer-nos acerca da gama de fatores que potencialmente influenciam o comportamento sob várias
condições. A pesquisa pode também oferecer algumas estimativas da importância desses valores num
determinado momento. Seja no caso do domínio da política pública ou dos relacionamentos pessoais, a
psicologia social pode aguçar a sensibilidade de um indivíduo para influências sutis e apontar suposições
sobre o comportamento que não se mostraram úteis no passado.
Quando se pede um conselho ao psicólogo social sobre um provável comportamento em uma situação
concreta, a reação consiste em desculpar-se. É necessário explicar que o campo ainda não se encontra
suficientemente desenvolvido a ponto de que predições confiáveis possam ser feitas. Do nosso ponto de

26
GERGEN, K. J. A psicologia social como história. Psicol. Soc.: Florianópolis, 2008.

Apostila gerada especialmente para: Betânia Mueller 023.712.580-32


. 239
vista, tais desculpas são inapropriadas. O campo pode raramente fornecer princípios para que predições
confiáveis possam ser feitas. Padrões de comportamento estão sob constante mudança. Contudo, o que
o campo pode e deve oferecer são pesquisas informando o inquiridor do número de possíveis ocorrências,
ampliando assim sua sensibilidade e preparando-o para uma acomodação mais rápida à modificação
ambiental. Pode prover ferramentas conceituais e metodológicas com as quais um número maior de
juízos de discernimento pode ser efetuado.
Psicólogos sociais evidenciam uma contínua preocupação com processos psicológicos básicos, ou
seja, processos que influenciam um vasto e variado conjunto de comportamentos sociais. Simulando a
preocupação de psicólogos experimentais com processos básicos, como visão em cores, aquisição da
linguagem, memória e assim por diante, psicólogos sociais detiveram-se em alguns processos, tais como
dissonância cognitiva, nível de aspiração e atribuição causal. Entretanto, há uma profunda diferença entre
os processos estudados nos domínios da psicologia geral experimental e no domínio da psicologia social.
No primeiro caso, os processos estão frequentemente guardados biologicamente no organismo, não
estão sujeitos a efeitos de esclarecimento e não dependem de circunstâncias culturais. Ao contrário, a
maioria dos processos de domínio social é dependente de disposições sujeitas a modificação ao longo
do tempo.
Assim sendo, é um erro considerar os processos em psicologia social como básicos no sentido das
ciências naturais. Antes, podem ser largamente considerados a contrapartida psicológica de normas
culturais. Da mesma maneira que um sociólogo preocupa-se em medir preferências parciais ou padrões
de mobilidade no decurso do tempo, o psicólogo social poderia atentar para os padrões de mudança das
disposições psicológicas e a sua relação com o comportamento social. Se a redução de dissonância é
um processo importante, então deveríamos estar aptos a medir a prevalência e a força de tal disposição
no seio da sociedade ao longo de tempo e os modos de redução de dissonância prediletos num dado
momento. Se a elevação da estima parece influenciar a interação social, os amplos estudos culturais
deveriam revelar a extensão dessa disposição, sua força em várias subculturas, e a forma do
comportamento social com a qual se encontra mais associada a um dado momento. Embora
experimentos em laboratório sejam adequados ao isolamento de disposições particulares, são pobres
indicadores da série e da significância dos processos da vida social contemporânea. São extremamente
necessárias metodologias que estabeleçam contato com a prevalência, força e forma das disposições
sociais no tempo. Com efeito, uma tecnologia dos indicadores sociais psicologicamente sensíveis (Bauer,
1969) é desejada.
O fenômeno social pode variar consideravelmente na medida em que se submete à mudança histórica.
Certos fenômenos podem ser mais estreitamente vinculados a dados fisiológicos. A pesquisa de
Schachter sobre estados emocionais parece ter uma forte base fisiológica, assim como o trabalho de
Hess sobre afeto e constrição pupilar. Embora disposições adquiridas possam vir a superar algumas
tendências fisiológicas, tais tendências deveriam se reafirmar gradualmente. Outras propensões
fisiológicas, ainda, podem ser irreversíveis. Pode haver também disposições que são suficientemente
poderosas para que nem o esclarecimento e nem mesmo as mudanças históricas venham a causar-lhe
algum impacto. Algumas pessoas geralmente evitarão estímulos físicos dolorosos, apesar de suas
sofisticações ou das normas correntes. Devemos pensar, então, em termos de um contínuo de
durabilidade histórica, com fenômenos altamente suscetíveis à influência histórica num extremo e
processos mais estáveis no outro.
Assim, métodos de pesquisa habilitando-nos a discernir a durabilidade relativa do fenômeno social são
bastante necessários. Métodos interculturais poderiam ser empregados para esse fim. Embora a
replicação intercultural seja repleta de dificuldades, similaridade numa dada função entre culturas
amplamente divergentes atestaria fortemente sua durabilidade no tempo. Técnicas de análise de
conteúdo poderiam também ser empregadas no exame de períodos históricos recentes. Até agora, tais
empreendimentos têm fornecido pouco além de citações indicando que algum grande pensador
pressentiu uma hipótese familiar. Temos ainda que travar contato com a vasta quantidade de informações
referentes aos padrões de interação nos últimos períodos. Embora a progressiva sofisticação dos padrões
de comportamento ao longo do espaço e do tempo fornecesse valiosas compreensões referentes à
durabilidade, alguns difíceis problemas apresentar-se-iam. Alguns padrões de comportamento podem
permanecer estáveis até uma observação minuciosa. Outros podem simplesmente tornar-se
disfuncionais com o passar do tempo. A confiança do homem num conceito de deidade tem uma longa
história e é encontrada em numerosas culturas. Entretanto, muitos são céticos sobre o futuro desta
crença. Taxas de durabilidade teriam assim que contribuir para a estabilidade potencial tanto quanto atual
do fenômeno.
Ainda que a pesquisa por disposições culturais mais duráveis seja extremamente valiosa, não
deveríamos daí concluir que seja mais útil ou desejável que estudar os padrões passados de

Apostila gerada especialmente para: Betânia Mueller 023.712.580-32


. 240
comportamento. Grande parte da variabilidade do comportamento social deve-se indubitavelmente a
disposições historicamente dependentes, e o desafio de capturar tais processos "em luta" e durante
períodos preciosos da história é imenso.
Sustentou-se que a pesquisa em psicologia social é fundamentalmente o estudo sistemático da história
contemporânea. Assim sendo, parece miopia manter a separação disciplinar (a) do estudo tradicional de
história e (b) de outras ciências historicamente fronteiriças (incluindo sociologia, ciência política e
economia). As particulares estratégias de pesquisa e a sensibilidade do historiador poderiam elevar a
compreensão da psicologia social, passada e presente. Particularmente útil seria a sensibilidade do
historiador às sequências causais no curso do tempo. Muitas pesquisas em psicologia social centram-se
em segmentos momentâneos de processos em andamento. Temos nos concentrado muito pouco na
função desses segmentos dentro de seu contexto histórico. Temos pouca teoria lidando com a inter-
relação entre eventos dentro de longos períodos de tempo. Da mesma feita, historiadores poderiam
beneficiar-se das mais rigorosas metodologias empregadas pelos psicólogos sociais tanto quanto de sua
sensibilidade a variáveis psicológicas. Contudo, o estudo da história, passada e presente, deveria ser
empreendido da maneira mais ampla possível. Fatores políticos, econômicos e institucionais são todos
fatores necessários à compreensão numa perspectiva integrada. A concentração em psicologia apenas
oferece uma compreensão distorcida de nossa condição presente.

Psicologia social e saúde coletiva: Reconstruindo identidades 27

A constituição da Psicologia como campo de conhecimento e profissão faz-se no entrelaçamento de


diversos saberes e acontecimentos de ordem social, política e econômica, como bem demonstra a história
da psicologia.
Cruzamentos estes que foram produzindo diversos desdobramentos, gerando multiplicidade em
termos de teorias e de práticas, na tentativa de dar conta das diferentes demandas cotidianas.

O psicólogo, como profissional, no Brasil, tem uma história muito recente.


Apesar de o ensino da Psicologia ser feito desde os anos 1930, foi somente em 1962 que a psicologia
passou a existir como profissão.
Assim, há três décadas os psicólogos garantiram um espaço institucionalizado de trabalho. Sabe-se
que com a regulamentação, o psicólogo passou a atuar em basicamente quatro áreas: clínica, escolar,
industrial e magistério, áreas que atualmente estão bastante ampliadas e que não correspondem mais ao
universo de atuação do psicólogo brasileiro. Segundo Dimenstein (1998), a própria pressão do mercado
de trabalho passou a impulsionar os profissionais para outros campos de atuação. A assistência pública,
dentre estas novas áreas, foi para onde convergiu uma considerável parcela dos profissionais. “As quase
três décadas desde que a profissão foi regulamentada foram acompanhadas de um alargamento dos
campos de atuação do psicólogo, forjado pelo próprio crescimento do contingente de profissionais assim
como pelo maior conhecimento de sua atividade e, consequentemente, pelo aumento da demanda por
seus serviços”.
Um marco importante sobre a inserção do psicólogo nos serviços de saúde ocorreu em São Paulo a
partir de 1982, com a adoção de uma política explícita, por parte da Secretaria da Saúde, da
desospitalização e da extensão dos serviços de saúde mental à rede básica. A política adotada pela
Coordenadoria de Saúde Mental levou à criação de equipes de saúde mental integradas por equipes
mínimas, das quais o psicólogo fazia parte, que passariam a atuar nos centros de saúde. “Constituía-se,
assim, uma rede de serviços teoricamente integrados com atuação nos níveis primário, secundário e
terciário”).
Yamamoto a respeito da situação profissional da psicologia no Brasil, comenta que o levantamento
feito pelo Conselho Federal de Psicologia evidencia duas tendências: “de uma parte a manutenção da
hegemonia clínica com relação às demais; de outra, uma ampliação das oportunidades profissionais,
proporcionada pela abertura de novos espaços de inserção social”. Sendo que um destes espaços é o
campo da saúde. Como diz o autor: “Um dos campos onde a psicologia tem mostrado maior inserção é o
da saúde. (...) Os psicólogos ingressam no campo da saúde através de duas formas: nas Unidades
Básicas de Saúde, articulados aos demais profissionais do campo, e nos Núcleos e Centros de Atenção
Psicossocial (NAPS/CAPS)”.
Silva afirma que: A psicologia clínica durante décadas foi pensada e planejada como disciplina
autônoma. É relativamente recente sua inserção em instituições de saúde pública, nas diferentes
instâncias de serviços. Com as modificações no sistema de saúde, a psicologia, bem como as demais

27
AGUIAR, S. G.; RONZANI, T. M. Psicologia social e saúde coletiva: reconstruindo identidades. Psicol. pesq.: Juiz de Fora, 2007.

Apostila gerada especialmente para: Betânia Mueller 023.712.580-32


. 241
profissões consideradas da área de saúde, que praticamente só eram absorvidas em instituições
ambulatoriais e hospitalares, passam a ser incorporadas às Unidades Básicas. Isto se torna possível a
partir da VIII Conferência Nacional de Saúde (...) para que se chegasse a um Sistema Único de Saúde
que possibilitasse uma atenção integral à saúde.
Spink acrescenta ainda, em acordo com os autores anteriormente citados, que, até recentemente, o
campo da atuação da psicologia se resumia a duas principais dimensões: em primeiro lugar, as atividades
exercidas em consultórios particulares. Uma atividade exercida de forma autônoma, como profissional
liberal e, de forma geral, não inserida no contexto dos serviços de saúde. A segunda vertente compreendia
as atividades exercidas em hospital e ambulatórios de saúde mental.
No campo da saúde, a autora considera que importantes transformações ocorreram entre os anos
1970 e 1990 e possibilitaram a inclusão do psicólogo nas ações de saúde. Primeiro houve uma
ressignificação da causalidade na explicação da doença, passando a ser vista como um processo e,
especialmente, como um fenômeno complexo e transdisciplinar, que precisa ser abordado de forma
integradora englobando as dimensões biopsicossocial. A nova linguagem abriu espaço para ação e
explicação de cunho psicológico. As mudanças foram lentas, mas o espaço foi sendo conquistado, por
exemplo, nos hospitais.
A psicologia, embora intimamente relacionada ao conceito de saúde (definida pela Organização
Mundial de Saúde como bem-estar físico, mental e social), como disciplina, chega tardiamente à área da
saúde. Chega tarde neste cenário e “chega miúda, tateando, buscando ainda definir seu campo de
atuação, sua contribuição teórica efetiva e as formas de incorporação do biológico e do social ao fator
psicológico, procurando abandonar os enfoques centrados em um indivíduo abstrato e a-histórico tão
frequentes na psicologia clínica tradicional.
Mais recentemente, surgiu no cenário da psicologia a psicologia da saúde, que tem se orientado mais
pelos problemas vinculados ao desenvolvimento da saúde humana do que pela doença.
O desenvolvimento da psicologia da saúde estimulou o trabalho do psicólogo no âmbito da prevenção
e da promoção da saúde, assim como sua participação em equipes interdisciplinares, tanto em
instituições de saúde quanto em sua atuação no trabalho comunitário, tornando-se um espaço importante
de prevenção e promoção de saúde. O desenvolvimento de uma psicologia comunitária orientada pelo
trabalho nas áreas de saúde, independentemente das orientações teóricas às quais se filia, na verdade
representou um questionamento aos estanques rígidos definidos nas pesquisas e nas práticas
psicológicas.
Segundo Bock, o termo psicologia da Saúde aparece a partir do Seminário Internacional da Saúde
realizado em Cuba e relatado no Jornal do Psicólogo, nº 11/84. Esta expressão ‘psicologia da Saúde’
também é usada por Spink como um novo campo do saber. Para Spink, falar da psicologia da saúde
como novo campo do saber parece ser, à primeira vista, uma temeridade. Afinal, os aspectos psicológicos
da saúde/doença vêm sendo discutidos desde longa data, e os psicólogos já há muito vêm marcando
presença na área de saúde mental. Entretanto, mudanças recentes na forma de inserção dos psicólogos
na saúde e a abertura de novos campos de atuação vêm introduzindo transformações qualitativas na
prática que requerem, por sua vez, novas perspectivas teóricas. É isto, pois, que nos permite afirmar que
estamos nos defrontando com a emergência de um novo saber.
Os fatores conjunturais (a maior aceitação da psicologia e o crescimento do número de profissionais),
associados à postura crítica de certos segmentos da profissão, levaram à definição de novas áreas de
atuação, buscando estender os serviços psicológicos às camadas mais pobres da população e, neste
afã, ampliar o referencial teórico de modo a focalizar os problemas sociais mais amplos subjacentes à
problemática individual. “Nesse processo, muitos psicólogos deslocaram suas atividades dos consultórios
particulares, inserindo-se diretamente na comunidade ou nas instituições voltadas ao atendimento das
camadas desprivilegiadas da população”. Num processo de revisão dessas práticas e a busca por
melhores formas de responder às necessidades dos diferentes locais de atuação, foi gerando novos
campos de saber e ampliou sua inserção na saúde.

A Psicologia Social como alternativa para a nova prática

Na fomentação de uma nova política pública de saúde, abrem-se espaços de trabalho para a
psicologia, que passa a problematizar a aplicação das práticas tradicionais em novo cenário de atuação.
Outras ferramentas de intervenção – mais apropriadas para a efetiva inserção na área – devem ser
construídas para o trabalho na Saúde Pública, a fim de que possam contribuir com as transformações
propostas pelo Sistema Único de Saúde (SUS). A Psicologia Social da Saúde, que compreende, em seus
pressupostos, uma intervenção mais local e coletiva, tem sido um importante campo de conhecimento e
prática para construir formas diferenciadas de intervenção na saúde.

Apostila gerada especialmente para: Betânia Mueller 023.712.580-32


. 242
A psicologia social, tendo como arena de atuação a complexa relação entre a esfera individual e a
social, tem necessariamente uma vocação interdisciplinar, sendo suas fronteiras permeáveis às
contribuições de uma variedade de outras disciplinas afins. “Cabe à psicologia social recuperar o indivíduo
na intersecção de sua história com a sociedade. Abandonar, portanto, a dicotomia indivíduo sociedade”.
A psicologia (social) comunitária utiliza-se do enquadre da psicologia social, privilegiando o trabalho
com os grupos, buscando colaborar para a formação da consciência crítica e “para a construção de uma
identidade social e individual orientadas por preceitos eticamente humanos”.
Segundo Ronzani & Rodrigues, “a psicologia comunitária constitui um importante campo teórico-
prático para o trabalho em APS, uma vez que pode possibilitar uma maior aproximação das questões de
relevância social das comunidades”. Costa e Lopez afirmam que é através da atuação da Psicologia
Comunitária que programas de saúde podem ser aplicados ao âmbito local de cada comunidade. A
Psicologia da Saúde e a Psicologia Comunitária estabeleceriam, assim, uma relação na qual esta última
se converteria em um instrumento de implementação dos programas que envolvem conceitos da primeira.
Seria através da Psicologia Comunitária, propõem Costa e López, que os programas de saúde se
tornariam ágeis e integrados ao tecido social em que os processos de saúde, adoecimento e morte se
dão, e é justamente nesse nível que a intervenção preventiva deveria ocorrer. Entretanto, o que se deve
tentar obter da comunidade é que a mesma análise e modifique seus comportamentos tentando torná-los
favoráveis à saúde. Assim, a Psicologia Comunitária seria um ponto de ligação entre o sistema de saúde
e a comunidade, numa configuração dinâmica e móvel.
A Psicologia Social da Saúde configura-se como um campo de conhecimento e prática que trata das
questões psicológicas com enfoque mais social, coletivo e comunitário voltado para a saúde. Segundo
Marín, caracteriza-se pela interlocução da Psicologia Social – com seus conhecimentos e técnicas – com
o âmbito da saúde e destaca a interação como ponto fundamental do processo saúde doença. A interação
refere-se tanto ao homem e seu ambiente quanto aos diversos atores sociais presentes no cuidado com
saúde. O autor ainda salienta que todas as atividades da Psicologia Social da Saúde centram-se mais na
busca de uma saúde integral e não somente na saúde mental.
Para Spink, a psicologia social da saúde é como um campo ampliado de atuação do psicólogo nas
instituições de saúde. Essa ampliação ocorreria, principalmente, em relação ao referencial de trabalho
utilizado e exercido, pois, segundo a autora, a intervenção deve ser contextualizada, ou seja, é importante
compreender toda a história e o contexto da instituição na qual será implementada uma ação, assim como
as pessoas que compõem essa instituição. Faz-se necessário compreender que cada organização tem
sua realidade local, sua cultura de relações e as histórias específicas das pessoas que recorrem a esses
serviços.
A autora explica o que seria a Psicologia Social da Saúde: A primeira característica é o compromisso
com os direitos sociais pensado numa ótica coletiva. Foge, portanto, das perspectivas mais tradicionais
da psicologia voltadas à compreensão e processos individuais ou intraindividuais. Dialoga com teorias e
autores que pensam as formas de vida e de organização na sociedade brasileira contemporânea. Tende
a pesquisar e atuar em serviços de atenção primária, em contextos comunitários, em problemas de saúde
em que pesam a prevenção à doença e a promoção da saúde ou onde há necessidade de
acompanhamento continuado (como as doenças crônicas e a saúde mental). Tende ainda a atuar na
esfera pública.
Ainda a mesma autora coloca que a Psicologia Social da Saúde tem como características principais a
atuação centrada em uma perspectiva coletiva e o comprometimento com os direitos sociais e com a
cidadania. Rompe, portanto, com enfoques mais tradicionais centrados no indivíduo. A atuação se dá
principalmente nos serviços de atenção primária à saúde, focaliza a prevenção da doença e a promoção
da saúde, e com extrema importância o incentivo dos atores sociais envolvidos para a geração de
propostas de transformação do ambiente em que vivem. Trata-se, portanto, de um processo de
transformação crítica e democrática que potencializa e fortalece a qualidade de vida.
A Psicologia Social da Saúde objetiva trabalhar dentro de um modelo mais integrado, reconhece a
saúde como um fenômeno multidimensional em que interagem aspectos biológicos, psicológicos e sociais
e caminha para uma compreensão mais holística do processo saúde-doença-cuidado. Dessa maneira
sua inserção na atenção primária pode ser útil para contribuir na transformação das práticas em saúde
rumo à integralidade. Traz conceitos potentes e propostas de ação que muito se aproximam aos
pressupostos de trabalho da atenção primária. Vemos que os dois discursos se organizam em torno de
eixos que apostam na construção do fazer conjunto, coletivo e valorizam a localidade e as interações dela
decorrentes.
Os pressupostos da Psicologia Social da Saúde ecoam, nesse modo de organização do trabalho, à
medida que têm como ponto fundamental também a contextualidade e a interação com ações construídas
coletivamente a partir das imprevisibilidades do cotidiano. Assim, tanto a ESF (estratégia de saúde da

Apostila gerada especialmente para: Betânia Mueller 023.712.580-32


. 243
família) como esse campo da psicologia privilegiam o processo de produção de conhecimento e a
construção das intervenções a partir das práticas sociais, dos processos interativos e da cultura. A
proposta, portanto, é a de que qualquer entendimento do processo saúde-doença-cuidado possa ser
analisado e referido a partir de seu contexto, ou seja, a partir da compreensão de uma pessoa,
pertencente à determinada família, inserida numa comunidade específica, e assim por diante.
Assim, a Psicologia Social da Saúde viria ao encontro da Atenção Primária à Saúde (APS),
especificamente do Programa de Saúde da Família (PSF), no objetivo de construir um modelo de atenção
à saúde pertinente à realidade local e gerador de interlocuções entre equipe de saúde e comunidade.
Nesse sentido, a parceria pode ser útil para pensar discursos, na saúde, que propiciem a construção de
espaços viabilizadores de acolhimento e a construção do vínculo, contribuindo para a reflexão e a
problematização dessas práticas que se propõem coletivas.

A prática do psicólogo na atenção primária

Martinez Calvo coloca que a promoção de saúde se origina nas ciências que se ocupam do
comportamento social.
Se nas propostas da atenção primária objetiva-se trabalhar com promoção, o interesse para a
psicologia é evidente. As ações promocionais, segundo Calatayud, necessitam apoiar-se em conceitos
puramente psicológicos, tais como: hábitos, atitudes, motivação, interações pessoais e familiares e
habilidades. Faz algumas recomendações para o trabalho dos psicólogos:
1) identificar os problemas que requerem atenção prioritária;
2) para esta identificação, as informações sobre a comunidade são a fonte para a tomada de decisões;
3) trabalhar em equipe com profissionais de outras disciplinas, compartilhando conhecimentos;
4) estimular a participação dos membros da comunidade, levando em conta sua opinião na definição
das prioridades e as estratégias, tornando-os multiplicadores.
Segundo Calatayud, há um conjunto de temas que geralmente aparecem como prioritários para a
psicologia na atenção primária, “e este caráter prioritário se deve ao fato de que são temas que mais
afetam o estado de saúde das pessoas, os quais se recebem a correta atenção, podem conduzir a
melhorias importantes na saúde da população”.
Como veremos adiante, cada um destes temas relaciona-se com aspectos biológicos, sociais e
psicológicos. Estes últimos nos servirão de pauta para guiar o trabalho do psicólogo na atenção
primária.

1) Saúde Reprodutiva
Alguns problemas que afetam a saúde reprodutiva e podem ser abordados pela psicologia:
- práticas sexuais que conduzem a gravidez indesejada, ou contágio de doenças sexualmente
transmissíveis;
- gravidez na adolescência;
- aborto induzido;
- comportamento de risco para o bom desenvolvimento da gravidez (álcool, drogas, etc.);
- Preparação insuficiente da gestante e da família para os cuidados físicos e emocionais do recém-
nascido;
- insuficiente conhecimento de comportamentos paternos que propiciem a satisfação das
necessidades psicológicas do bebê no primeiro ano de vida.
Segundo o autor, tais questões podem ser trabalhadas com grupos para adolescentes, gestantes,
grupos com familiares das gestantes, grupos com mães a respeito das necessidades do primeiro ano de
vida da criança, entre outros.

2) Saúde das crianças


Alguns problemas frequentes que afetam as crianças e que podem ser abordados pela psicologia:
- ambiente doméstico prejudicial;
- maus-tratos à criança;
- dificuldades de aprendizagem.
Tais dificuldades podem ser abordadas, segundo Calatayud (1999), através de grupos com as
crianças, para que elas coloquem suas dificuldades, intervenção junto aos familiares, identificação de
ambientes familiares prejudiciais, entre outros.

Apostila gerada especialmente para: Betânia Mueller 023.712.580-32


. 244
3) Adolescentes
Os adolescentes constituem um grupo que abre espaço para várias possibilidades de trabalho. Podem
ser abordados os seguintes temas, através de grupos, palestras, ou, se necessário, individualmente: início
da vida sexual, gravidez indesejada, drogas, álcool, dificuldades familiares etc.
4) Idosos
Também os idosos são citados pelo autor como um grupo potencial de trabalho. Vários aspectos
podem ser abordados. Entre eles: distância dos familiares, solidão, morte do cônjuge ou amigos, aumento
das limitações físicas, tempo ocioso, diminuição da autoestima, depressão etc.
Pelo que vimos até o momento, podemos considerar que existem diversos pontos em comum entre os
princípios norteadores do PSF e aqueles que devem também subsidiar o trabalho do psicólogo,
como, por exemplo, o atendimento da demanda de acordo com as necessidades dos indivíduos (e não
de cima para baixo); a busca do resgate da autonomia, da autoestima e da cidadania; a ênfase na criação
de vínculo entre o profissional e o cliente; a valorização dos saberes individuais e grupais; o respeito às
diferenças; a relação de compromisso e corresponsabilidade dos profissionais com os usuários; a visão
do humano como ser integral e não exclusivamente biológico; a valorização de ações de prevenção,
promoção e manutenção da saúde, e não somente cura e reabilitação; o enfoque centrado nas
potencialidades para o crescimento e não apenas na erradicação do sintoma ou da doença; a valorização
do contexto social, histórico, cultural, ambiente familiar e psicológico dos indivíduos, além da dimensão
orgânica e fisiológica.
A finalidade do PSF, como já visto, é o acompanhamento da clientela, dentro do seu contexto
sociocultural, de forma a aproximar a família, a comunidade e os profissionais, com vistas principalmente
à promoção da saúde para melhoria da qualidade de vida da população. A inserção do psicólogo na
equipe de saúde da família também deve visar à promoção da saúde da população, no que concerne à
atenção para os aspectos psicológicos, tanto em termos de prevenção quanto de promoção.
Cardoso aponta como objetivos gerais da atuação do psicólogo no PSF, independente da clientela
atendida, os de atuar junto à comunidade, fornecendo e difundindo informações sobre saúde mental;
identificar pessoas portadoras de doenças orgânicas crônicas com comprometimentos emocionais que
demandem assistência psicológica; possibilitar espaço terapêutico de trocas de experiências, com vistas
ao desenvolvimento das potencialidades das pessoas para atender às próprias necessidades,
proporcionando, além da melhora do quadro clínico, a da sua qualidade de vida; atuar junto aos
profissionais da equipe do PSF, para integrar esforços, estimular a reflexão e a troca de informações
sobre a população atendida e facilitar a avaliação e a evolução clínica.
O psicólogo pode abarcar ainda, além do atendimento individual, avaliação da demanda, estudos de
caso e o incentivo para facilitação da comunicação entre a comunidade e a equipe de saúde, já que
muitas vezes os pacientes revelam dados nem sempre acessíveis à equipe. O teatro informativo pode
ser utilizado para fornecer informações sobre o que é a psicologia e o trabalho do psicólogo, tais como
seus objetivos, a questão do sigilo, a composição dos grupos, esclarecimento sobre as concepções
errôneas a respeito da psicoterapia e do psicólogo. A visita domiciliar é outra atividade que auxilia na
divulgação do trabalho, ajuda a conhecer um pouco da realidade das pessoas atendidas e, quando
necessário, a prestar assistência psicológica a pacientes impossibilitados de sair de sua residência.
Dentro desta mesma ideia, Ronzani acrescenta que: uma das funções do psicólogo pode ser o
acolhimento dos novos pacientes, fazendo encaminhamentos, quando necessário, intervenção
psicossocial, desenvolvendo oficinas terapêuticas, atendendo a pacientes graves, fazendo visitas
domiciliares e proporcionando suporte familiar, especialmente para aqueles portadores de transtornos
mentais.
Duran-Gonzáles apontam que o profissional de APS deve estar capacitado para proporcionar
mudanças de atitudes da população; entrar em contato com indivíduos que possam influenciar
diretamente nas práticas de saúde da população. O psicólogo então se torna um ator importante na
medida em que pode promover a participação da comunidade no autocuidado e ainda ser o ponto de
intersecção entre a população e a equipe de saúde.

Da formação à ação

Pensar a atuação do psicólogo nas Unidades Básicas de Saúde não é uma tarefa fácil. O tempo de
inserção destes profissionais é relativamente pequeno; há um contingente reduzido de profissionais
atuando na área — apesar de vir aumentando gradativamente — existem poucas pesquisas mais
sistemáticas, tanto nacionais quanto locais, sobre a atuação do psicólogo neste campo específico de
trabalho.

Apostila gerada especialmente para: Betânia Mueller 023.712.580-32


. 245
Nas palavras de Dimenstein, é possível apontar que tais dificuldades encontradas pelos psicólogos
para a realização da Psicologia nas Unidades Básicas de Saúde no país advêm tanto da inadequação da
sua formação acadêmica para o trabalho no setor, quanto seu limitado modelo de atuação, bem como as
dificuldades de adaptar-se às dinâmicas condições de perfil profissional.
A inserção em instituições públicas ou na comunidade parece ser bastante problemática tanto no que
se refere às dificuldades externas, relativas à falta de recursos, quanto à ausência de modelos de atuação,
apontando aí carências de uma formação acadêmica mais voltada ao modelo clínico hegemônico.
O conhecimento e as técnicas dos quais lança mão a Psicologia da Saúde são frequentemente
oriundos da clínica e naturalmente passam por uma série de adaptações que se configuram em uma
verdadeira reconstrução da atividade e da identidade do psicólogo e da Psicologia. As implicações da
atuação ou reinserção do psicólogo no campo da saúde são de natureza complexa e envolvem desde um
treinamento específico para atuar neste campo — treinamento que não tem sido observado na grande
maioria dos cursos de graduação — até o equacionamento mais otimizado possível de uma resistência
que é exercida por outros profissionais, tradicionalmente atuantes na área, em frente da “a aparente
inutilidade da atuação dos psicólogos no campo da promoção da saúde e da alta demanda para a atuação
no modelo clínico”.
Essa resistência não é pequena e também não parte exclusivamente dos profissionais já firmemente
estabelecidos no campo da saúde, mas também do próprio psicólogo que não (re)conhece as
possibilidades de sua atividade.
As perguntas que se impõem são: Como se configura a prática deste campo? É possível uma atuação
junto à comunidade? Que papel terão as práticas usualmente utilizadas na Psicologia Clínica? Yamamoto
(1996) faz a mesma pergunta: O arsenal teórico-prático de que dispõe a Psicologia pode ser aplicado em
novos campos que a Psicologia desbrava? O autor nos assegura que as respostas a essas questões não
são simples e ressalta que não basta mudar de cliente; é importante que, além de mudar de cliente, o
psicólogo realize uma revisão daquilo que teoricamente embasa sua.
Considerando, então, que a formação básica do psicólogo privilegia a atuação clínica, centrada no
indivíduo e localizada no consultório, é comum a mera transferência do referencial teórico obtido na
graduação, para o contexto institucional.

Spink sustenta que a pratica em instituições requer uma expansão do referencial teórico utilizado em
dois sentidos:
a) expansão do referencial contextual, ou seja, a busca de dados que permitam melhor localizar o
psicólogo e outros atores envolvidos na dinâmica social e/ou institucional;
b) “expansão do referencial teórico, no sentido de conseguir trabalhar com alteridade, ou seja, com a
perspectiva de um “outro” definido culturalmente como diferente do eu”.

O que está em pauta no primeiro caso é a compreensão das determinações sociais mais amplas que
afetam a relação profissional do psicólogo e as pessoas ou grupos que recebem algum tipo de
intervenção. Já no segundo caso, o que está em pauta é a aceitação de uma realidade multiforme, cuja
definição, ou mesmo a percepção, é fruto de uma sociedade determinada e, dentro desta, de classes e
segmentos sociais específicos.
Dessa forma acreditamos que, ainda hoje, os psicólogos buscam um modelo para “fazer psicologia”
nos serviços de saúde. Reconhecemos que o trabalho é complexo e requer do psicólogo um
embasamento amplo de várias áreas do conhecimento. É um modelo que está continuamente sendo feito,
uma identidade sendo formada:
Acreditamos que podemos ser mais úteis ao campo da assistência pública à saúde a partir do momento
que nossa cultura profissional passe a fornecer modelos mais ampliados de atuação, os quais não se
revelem como barreiras à troca de saberes com outros profissionais, e que o psicólogo possa se
reconhecer como um trabalhador da saúde, preocupado com a promoção do bem-estar da população.
Discutir a formação necessária para a inserção institucional do psicólogo na área da saúde exige um
momento anterior de reflexão sobre as especificidades desta prática. Ao procurar entender e pensar a
formação do psicólogo para a prática em instituições, estamos, na verdade, buscando subsídios para a
sua inserção em uma organização. Entretanto, esta organização é sobre determinada por normas gerais,
que estão intimamente ligadas às representações coletivas que, com o passar dos anos, atingem o
estatuto de normas universais ou leis.
A compreensão do processo de institucionalização destas normas e, portanto, a compreensão do pano
de fundo que formata o cenário no qual se desenvolve nossa prática é obviamente um ingrediente
importante para um desempenho profissional consciente e conscientizador.

Apostila gerada especialmente para: Betânia Mueller 023.712.580-32


. 246
A inserção do psicólogo no campo da saúde tem se mostrado, nos diferentes estudos já realizados,
difícil, tanto por questões externas quanto por falta de modelos de atuação, o que se relaciona também
com o modelo da formação hegemônico mais voltado para a clínica.
A formação acadêmica está longe de contribuir com uma inserção inovadora e mais coerente com as
práticas sociais e com o SUS, mas há também incertezas e dúvidas diante das mudanças, da invenção.
O Departamento de Psicologia da Universidade Federal de Juiz de Fora (UFJF), em especial o Polo de
Pesquisa em Psicologia Social e Saúde Coletiva (POPS), vem há algum tempo se preocupando com a
formação crítica e inovadora na área de psicologia e saúde, proporcionando experiências junto aos
serviços de APS de Juiz de Fora e região, bem como produzindo pesquisas junto com alunos de
graduação e pós-graduação. Alguns resultados positivos já podem ser observados na formação dos
alunos, aumentando a preocupação em desenvolver ações contextualizadas e criativas junto aos
problemas da população brasileira. Mas ainda há um longo caminho a percorrer para uma consolidação
ainda maior dessa área.
Objetivamos aqui demonstrar várias frentes de trabalho que podem ser assumidas pelo psicólogo no
campo da saúde coletiva. A psicologia social da saúde é apresentada como referência que não se esgota
como ferramenta de compreensão e intervenção, mas é congruente com a demanda da atenção primária.

Questões

01. (Prefeitura de Portão/RS - Psicólogo - LEGALLE Concursos/2016) __________é um ramo da


psicologia que estuda como as pessoas pensam, influenciam e se relacionam umas com as outras. Surgiu
no século XX como uma- área de atuação da psicologia para estabelecer uma ponte entre a psicologia e
as ciências sociais (sociologia, antropologia, geografia, história, ciência política). Sua formação
acompanhou os movimentos ideológicos e conflitos do século, a ascensão do nazifascismo, as grandes
guerras, a luta do capitalismo contra o socialismo, entre outros. Quanto ao objeto de estudo, a
_______________procura explicar os sentimentos, pensamentos e comportamentos do indivíduo na
presença real ou imaginada de outras pessoas.

A partir do fragmento acima, estamos falando da:


(A) Psicologia Jurídica.
(B) Psicologia Social.
(C) Psicologia Clínica.
(D) Psicologia do esporte.
(E) Psicologia Hospitalar.

02. (SEDS/TO - Analista em Defesa Social - FUNCAB). Sobre o desenvolvimento da Psicologia


Social, é INCORRETO afirmar:
(A) No contexto científico, em uma visão cartesiana, a Psicologia Social se configurou como
individualista, em que o social não passava da soma de indivíduos
(B) Ao final da década de 50 e início da década de 60, Moscovici começou a questionar a Psicologia
Social existente na Europa, indicando que a Psicologia Social deveria ser uma ciência mais do movimento,
do que da ordem
(C) Diferentemente da Psicologia Individual, o papel da Psicologia Social seria conseguir uma
compreensão mais profunda das representações públicas e culturais.
(D) A Psicologia Social é uma ciência do comportamento na qual considera-se o social como a
expressão de comportamentos individuais encobertos e dispersos na cultura
Gabarito

01.B / 02.D

Comentários

01. Resposta: B
A psicologia social, tendo como arena de atuação a complexa relação entre a esfera individual e a
social, tem necessariamente uma vocação interdisciplinar, sendo suas fronteiras permeáveis às
contribuições de uma variedade de outras disciplinas afins. “Cabe à psicologia social recuperar o indivíduo
na intersecção de sua história com a sociedade. Abandonar, portanto, a dicotomia indivíduo sociedade”.

Apostila gerada especialmente para: Betânia Mueller 023.712.580-32


. 247
02. Resposta: D
A psicologia (social) utiliza-se do enquadre da psicologia social, privilegiando o trabalho com os grupos,
buscando colaborar para a formação da consciência crítica e “para a construção de uma identidade social
e individual orientadas por preceitos eticamente humanos”.

Sociedade e processos de exclusão e segregação.

O que é inclusão e exclusão social?

Inclusão:
De acordo com o artigo 1 da Declaração Universal dos Direitos Humanos é um conjunto de meios e
ações que combatem a exclusão, provocada pelas diferenças de classes sociais, idade, sexo, escolhas
sexuais, educação, deficiências, preconceitos raciais etc.
A Inclusão Social tem como objetivo oferecer oportunidades de acesso à tudo para todos.
O processo de inclusão vem sendo aplicado em cada sistema social, na educação, nos ambientes de
trabalho, no lazer, nos transportes etc. Todo o sistema deve ser inclusivo, educação, trabalho, lazer e
meio de transporte para todos!

'' Todos os seres humanos nascem livres e iguais em dignidade e em direitos. Dotados de razão e de
consciência, devem agir uns para com os outros em espírito de fraternidade. ''

Exclusão:
Na Declaração Universal dos Direitos Humanos, a exclusão relaciona-se à dificuldades ou problemas
sociais que levam ao isolamento e até à discriminação de um determinado grupo. O sociólogo francês
Robert Castel, definiu a exclusão social como o ponto máximo atingível no decurso da marginalização,
sendo este, um processo no qual o indivíduo se vai progressivamente afastando da sociedade através de
rupturas consecutivas com a mesma.
A pobreza pode, por exemplo, levar a uma situação de exclusão social, no entanto, não é obrigatório
que estes dois conceitos estejam intimamente ligados. Um trabalhador de uma classe social baixa, pode
ser pobre e estar integrado na sua classe e comunidade. Deste modo, fatores/estados como a pobreza,
o desemprego ou emprego precário, as minorias étnicas e ou culturais, os deficientes físicos e mentais,
os sem-abrigo, trabalhadores informais e os idosos podem originar grupos excluídos socialmente mas,
não é obrigatório que o sejam.
''Todos os seres humanos podem invocar os direitos e as liberdades proclamados na presente
Declaração, sem distinção alguma, nomeadamente de raça, de cor, de sexo, de língua, de religião, de
opinião política ou outra, de origem nacional ou social, de fortuna, de nascimento ou de qualquer outra
situação. Além disso, não será feita nenhuma distinção fundada no estatuto político, jurídico ou
internacional do país ou do território da naturalidade da pessoa, seja esse país ou território independente,
sob tutela, autônomo ou sujeito a alguma limitação de soberania. ''

Inclusão social28

Inclusão social é um conceito usado de forma bastante genérica, tanto nos livros quanto nos discursos
políticos. Para entender o que significa esse termo, é preciso compreender antes seu oposto, a exclusão
social. Nos anos 1970, a Europa sofria com as consequências de uma grande crise econômica, o que
causou o empobrecimento de uma parte de sua população. Naquele momento, a França – que antes
havia alcançado um alto patamar de qualidade de vida com emprego para praticamente toda a população
– assiste a exclusão de algumas categorias do mercado de trabalho. Nessas categorias estavam, por
exemplo, idosos, deficientes físicos e imigrantes. É nesse contexto que se usa pela primeira vez o termo
“exclusão social”, para designar setores que foram momentaneamente excluídos de uma sociedade que
já havia superado a pobreza. A expressão “inclusão social” emerge aí para designar as políticas
assistencialistas voltadas especificamente para esse público.
Nos anos 1990, organismos internacionais recuperam esse conceito, dando a ele um sentido mais
amplo. Exclusão social passa a significar o processo de privação do acesso aos direitos sociais como um

28
BETONI, C. Inclusão social, disponível em https://www.infoescola.com

Apostila gerada especialmente para: Betânia Mueller 023.712.580-32


. 248
todo – direitos políticos, a saúde, a educação, o emprego e a educação, por exemplo. O mercado de
trabalho mudou muito nesse período e o acesso a empregos estáveis passou a ser mais difícil, levando
muitos grupos sociais ao desemprego ou a ocupação de trabalhos precários (temporários, informais ou
com baixos salários). Esse movimento de exclusão do trabalho, que afetou especialmente os jovens e as
minorias étnicas, desencadeia a exclusão social mais ampla. Do ponto de vista psicológico, a exclusão
social deixa o indivíduo vulnerável, com um sentimento de inutilidade social que pode fragilizar seus laços
sociais e, as vezes, desencadear situações graves (como a depressão e a dependência química).
O processo que faz com que uma pessoa seja excluída socialmente é cumulativo. Ele acontece através
de uma cadeia de privações, incluindo origens familiares pobres, nível de escolarização baixa,
alimentação deficiente, pouco acesso a saúde, condições de trabalho precárias, falta de moradia,
dificuldade de acesso aos serviços públicos, exposição a violência, etc. Entretanto, a exclusão não
acontece somente devido a situação socioeconômica do sujeito. Condições de gênero, etnia, deficiência
física ou intelectual ou a falta de conhecimentos específicos (como os de informática) também podem
gerar uma situação de exclusão social, quando o indivíduo não consegue ter acesso aos direitos básicos
que deveriam estar à disposição de todos.
Muitas vezes, a inclusão social é colocada como a meta oficial das instituições educacionais. A escola
deve garantir que os indivíduos, independentemente de suas condições sociais ou biológicas, tenham a
chance de se inserir não só no mercado de trabalho, mas na sociedade como um todo. Isso é, que estejam
aptos de acessar também os bens culturais e os direitos políticos. Do ponto de vista sociológico, a ideia
de inclusão social remete as noções que Émile Durkheim tinha da educação e sua função de integrar
harmoniosamente o indivíduo na sociedade, evitando os conflitos e o isolamento. Algumas políticas
públicas, como a educação de jovens e adultos e a escolarização da população carcerária, buscam
claramente esse objetivo. Há, no entanto, uma perspectiva crítica que afirma que a ideia de inclusão social
é mobilizada de forma a desviar o olhar de problemas estruturais, como a desigualdade social e a pobreza,
que seriam as grandes causas de exclusão de amplos setores da sociedade. A inclusão em si só poderia
ser feita em países desenvolvidos, onde alguns direitos existem mesmo que indivíduos sejam excluídos
deles. Em países periféricos, como o Brasil, esses direitos básicos nunca chegaram a ser amplamente
realizados.
29
As diferenças representam grandes oportunidades de aprendizado. As diferenças oferecem um
recurso grátis, abundante e renovável... o que é importante nas pessoas – e nas escolas – é o que é
diferente, não o que é igual.
Em paralelo e integrado aos processos de democratização da sociedade brasileira, são inegáveis os
avanços que vêm sendo obtidos no caminho da inclusão de todas as pessoas nas escolas. Desde os
movimentos políticos e sociais que lutam para que 100% de nossas crianças ingressem e concluam a
educação básica, passando pelos movimentos inclusivos que defendem o acesso das pessoas com
deficiência na escola, até alcançar movimentos sociais que lutam pelo direito a uma educação de
qualidade para grupos étnico-raciais diferentes, o Brasil vem galgando espaços significativos na melhoria
das condições de acesso à educação.
É inegável, porém, que nossas escolas continuam sendo produto e produtoras de exclusões sociais,
dos mais diversos tipos. A resistência em mudar o paradigma que sustenta um perfil excludente de
educação, em que as “categorizações” das pessoas por suas diferenças sociais, econômicas, psíquicas,
físicas, culturais, religiosas, raciais, ideológicas e de gênero reforçam conflitos e violências físicas e
simbólicas, e tornam-se entraves para a constituição da inclusão educacional.
A escola precisa mudar e não os alunos e as alunas. Ela precisa ser ressignificada de acordo com o
paradigma de ética, cidadania e democracia que sustenta os movimentos inclusivos. Tais mudanças,
concomitantes àquelas porque passa a sociedade em geral, supõem uma abertura à pluralidade e à
diversidade das pessoas que convivem nos espaços educativos, dirigindo o foco dos conteúdos, dos
métodos e das relações humanas para a aprendizagem dos estudantes e não apenas para o ensino de
temáticas descontextualizadas da sociedade contemporânea e de suas vidas.
São as mudanças nessa direção que devem permitir um segundo grande passo nos processos de
democratização. Se o primeiro passo foi a luta pelo acesso, em que os resultados já são visíveis, o
segundo está na conquista da qualidade da educação para cada um e para todas as pessoas, de forma
a consolidar a inclusão e a igualdade de oportunidades para todos os seres humanos. Dessa maneira,
estaremos combatendo as exclusões em suas distintas formas de manifestação.
Neste segundo movimento estamos caminhando a passos lentos, e é papel dos(as) educadores(as)
comprometidos(as) com a construção de uma sociedade mais justa, solidária e feliz atuar na construção
de um novo paradigma educacional, inclusivo.

29
http://portal.mec.gov.br/

Apostila gerada especialmente para: Betânia Mueller 023.712.580-32


. 249
O Programa Ética e Cidadania pretende trazer para o centro das preocupações do Fórum Escolar de
Ética e de Cidadania as principais temáticas relacionadas à inclusão e à exclusão social, focando
principalmente suas repercussões no âmbito escolar. O objetivo é contribuir para a construção de escolas
verdadeiramente inclusivas, abertas às diferenças e voltadas para os interesses e necessidades de todos
os seres humanos.
Com este módulo que foca a inclusão social e seu reverso, a exclusão, o Programa pretende fornecer
bases conceituais e metodológicas para que profissionais da educação e membros do Fórum Escolar de
Ética e de Cidadania possam estudar e desenvolver projetos acadêmicos sustentados na pluralidade e
complexidade da vida cultural, natural e social.
O material apresentado a seguir começa com excertos do livro Inclusão Escolar: O que é? Por quê?
Como fazer? de Maria Teresa Eglér Mantoan (Moderna, 2003). Para o trabalho com outras linguagens,
propõe-se a exibição do curta-metragem de animação Ratos de rua, de Rafael de Paula Rodrigues. Para
encerrar o módulo, apresenta-se a história em quadrinhos Cris Córner. A personagem é uma adolescente
de 16 anos que sofre de epilepsia e, diante desse fato, enfrenta preconceitos e estigma social.

Inclusão social: O Mundo de Cris e o tratamento da epilepsia

A epilepsia é a condição neurológica crônica mais comum em todo o mundo e afeta todas as idades,
raças e classes sociais. Impõe, aos(às) seus(suas) portadores(as), um peso grande nas áreas
psicológica, física, social e econômica, revelando dificuldades não só individuais, mas também familiares,
escolares e sociais, especialmente devido ao desconhecimento, crenças, medo e estigma.
Dificilmente um(a) professor(a), com alguns anos de profissão, não experienciou na escola cenas de
crises epilépticas. Embora atualmente exista maior consciência por parte da comunidade sobre a
epilepsia, suas causas, as formas de tratamento e as consequências para o(a)s portadores(as) desse tipo
de enfermidade continuam devastadoras para suas vidas pessoal e social. Principalmente no caso de
crianças.
A pessoa que tem epilepsia, além de sofrer com os problemas neurológicos causados pela doença,
pode ter de enfrentar, no decorrer de sua vida, um obstáculo difícil de transpor: o de ser socialmente
estigmatizada. As eventuais convulsões ou crises de um(a) epilético(a), geralmente, assustam quem as
assiste quando elas acontecem em um ambiente social como a escola, por exemplo. E para a criança
com epilepsia, sofrer o estigma chega a ser pior que a própria doença.
A estigmatização a que são submetidos(as) epiléticos (as) e até mesmo o preconceito que sofrem
costumam marcar profundamente suas vidas. Portadores de epilepsia sofrem com o estigma, o
preconceito, a vergonha e o medo do desconhecido.
A epilepsia é uma doença cerebral caracterizada por convulsões, que vão desde as quase
imperceptíveis até aquelas graves e frequentes. A Organização Mundial da Saúde estima que cerca de
50 milhões de pessoas no mundo sejam portadoras de epilepsia, sendo que destas, 40 milhões estão em
países subdesenvolvidos. Apesar desse cenário alarmante, a organização afirma que 70% dos novos
casos diagnosticados podem ser tratados com sucesso, desde que a medicação seja usada de forma
correta.
Os tipos de epilepsia mais frequentes nos países do Terceiro Mundo são aqueles relacionados às
condições precárias de higiene, falta de saneamento básico, atendimento médico insuficiente e de baixa
qualidade e problemas nutricionais. A alta incidência, sobretudo nas áreas rurais, de portadores de
epilepsia causada por neurocisticercose é uma fato que denuncia a baixa qualidade de vida da população
dos países em desenvolvimento.
No Brasil, é estimado que existam três milhões de pessoas com epilepsia, sendo que a esse número
somam-se 300 novos casos por dia. Aproximadamente 50% dos casos de epilepsia têm início na infância
ou adolescência. Provavelmente, a maioria dessas pessoas carrega o “status de epiléptico”, mas muitos
ainda não sabem que o tratamento existe e é eficaz, podendo controlar 80% dos casos. Entretanto, em
países como o Brasil, o tratamento dado a uma parcela significativa da população não é adequado, devido
à baixa qualidade do sistema de saúde aliada ao preconceito e à estigmatização. Estes dois últimos têm
como fator perpetuante o mito decorrente da falta de informação correta.

Questões

01. (CEFET/RJ - Psicólogo - CESGRANRIO). Com a globalização e com as novas formas de contrato
de trabalho, a distinção da inclusão e da exclusão do mundo do trabalho vem sendo cada vez mais difícil
de sustentar.

Apostila gerada especialmente para: Betânia Mueller 023.712.580-32


. 250
No contexto contemporâneo, mesmo um trabalhador com vínculo estável pode experimentar a
precarização subjetiva de seu trabalho quando
(A) o equipamento utilizado é inadequado às características do corpo humano.
(B) o exercício de sua função pode expô-lo a situações em que há risco de ele ser assaltado ou
sequestrado.
(C) os recursos necessários para responder às exigências de seu trabalho não estão sendo garantidos.
(D) a possibilidade de crescimento e evolução na carreira e a adesão aos treinamentos oferecidos pela
organização estão vinculadas.
(E) as condições ambientais em que as tarefas vão ser realizadas estão aquém do limite da salubridade

02. (UFES - Assistente Social - AOCP). A institucionalização e legitimação do Serviço Social como
profissão, no Brasil, têm como fundamento processos de reprodução social da vida, e nestes,
particularmente, as diversas manifestações da questão social, como a pobreza, a subalternidade e
(A) a cidadania.
(B) a democracia.
(C) a exclusão social.
(D) o trabalho.
(E) a justiça social.

Gabarito

01. C / 02. C

Comentários

01. Resposta: C
Na Declaração Universal dos Direitos Humanos, a exclusão relaciona-se à dificuldades ou problemas
sociais que levam ao isolamento e até à discriminação de um determinado grupo. O sociólogo francês
Robert Castel, definiu a exclusão social como o ponto máximo atingível no decurso da marginalização,
sendo este, um processo no qual o indivíduo se vai progressivamente afastando da sociedade através de
rupturas consecutivas com a mesma.

02. Resposta: C
A pobreza pode, por exemplo, levar a uma situação de exclusão social, no entanto, não é obrigatório
que estes dois conceitos estejam intimamente ligados. Um trabalhador de uma classe social baixa, pode
ser pobre e estar integrado na sua classe e comunidade. Deste modo, fatores/estados como a pobreza,
o desemprego ou emprego precário, as minorias étnicas e ou culturais, os deficientes físicos e mentais,
os sem-abrigo, trabalhadores informais e os idosos podem originar grupos excluídos socialmente mas,
não é obrigatório que o sejam.

Criminalização e produção de subjetividades.

O fenômeno delitivo30

O estudo do fenômeno delitivo tem apresentado diversas classificações ao longo da história, ora
tratando-o como manifestação individual, ora social, ou, ainda, conjugando-se ambos os fatores. No
entanto, a extensa lista de classificações denota a persecução dos doutrinadores em catalogar tipos
delinquentes que não se definem no cotidiano, vez que indicar delinquência somente a partir do autor do
fato representa deixar de fora todas as vicissitudes sociais.
Apresenta-se a seguir, apenas para fins didáticos, a classificação proposta pelo Prof. Hilário Veiga de
Carvalho, citado por Maranhão (1981), que se refere ao indivíduo que comete o crime e as influências
para que o ato delitivo ocorra. Nesta classificação, associa-se a origem do comportamento criminoso a
dois tipos de fatores: as forças do meio e as forças intrapsíquicas.

30
Fiorelli, José Osmir. Rosana Cathya Ragazzoni Mangini. Psicologia Jurídica, 8ª edição, São Paulo: Atlas, 2017.

Apostila gerada especialmente para: Betânia Mueller 023.712.580-32


. 251
Tem-se, assim:
- mesocriminoso: atuação antissocial por força das injunções do meio exterior, como se o indivíduo
fosse mero agente passivo; por exemplo, o silvícola;
- mesocriminoso preponderante: maior preponderância de fatores ambientais;
- mesobiocriminoso: determinantes tanto ambientais, quanto biológicos;
- biocriminoso preponderante: portador de anomalia biológica insuficiente para levá-lo ao crime, mas
capaz de torná-lo vulnerável a uma situação exterior, respondendo a ela com facilidade;
- biocriminoso puro: atua em virtude de incitações endógenas, como ocorre em algumas perturbações
mentais.
O primeiro e o quinto tipo são considerados pseudocriminosos, por faltar ao primeiro o animus
delinquendi e ao quinto a capacidade de imputação penal. Quanto aos demais, aplica-se o seguinte
quadro:

TIPO CORREÇÃO REINCIDÊNCIA


Mesocriminoso preponderante Esperada Excepcional
Mesobiocriminoso Possível Ocasional
Biocriminoso preponderante Difícil Potencial

No capítulo dedicado ao estudo do delinquente, observou-se extensa gama de componentes de origem


intrapsíquica, identificáveis na origem do comportamento delitivo. Também foram apresentados fatores
que combinam elementos intrapsíquicos com os sociais, tais como:
- Os valores, crenças e conceitos;
- a cópia e/ou identificação com modelos (pessoas significativas);
- a influência do grupo ou equipe à qual o indivíduo pertence ou com a qual participa de ações;
- o condicionamento capaz de produzir comportamentos estereotipados inadequados;
- emoções extremas, que conduzem a momentos de descontrole em que o indivíduo comete ações
fora do domínio consciente, embora responsável por elas.
Viu-se, também, que o comportamento delitivo não pode ser analisado desconsiderando-se dois
importantes elementos presentes no campo de forças de quem o pratica:
- a vítima; e
- os mecanismos de desestímulo a esse comportamento.
A vítima participa direta ou indiretamente do comportamento delitivo por meio dos próprios
comportamentos, conforme estudado anteriormente. Não basta, em muitas situações, o desejo ou a
expectativa do que pratica o delito, mas, também, é necessária a participação da vítima para promover a
motivação imediata ou para desencadear o comportamento, seja pelo favorecimento simples, seja pelo
proporcionar a expectativa favorável de sucesso, seja pelo despertar de uma emoção.
Os mecanismos de desestímulo e controle têm influência direta e determinante sobre as perspectivas
de sucesso e sobre as expectativas quanto às consequências do comportamento. Eles afetam,
diretamente, a percepção do potencial delinquente e combinam-se com todos os elementos anteriores,
intra e extrapsíquicos.
Trata-se, pois, de um quadro de extrema sofisticação, ao qual a adoção de classificações
simplificadoras ou modelos elementares de causa-e-efeito simplesmente não se aplica.
Ademais, conforme ensina o Prof. Alvino Augusto de Sá, a abordagem que se faz da motivação criminal
é a "pedra de toque" pela qual se diferenciam os mais diversos posicionamentos científicos e ideológicos
sobre crime, criminalidade e homem criminoso (FIGUEIREDO DIAS; COSTA ANDRADE, 1997; GARCIA-
PABLOS DE MODA; GOMES, 1997).
Na psicologia, houve uma mudança, conforme assinalam Lane e Godo (1999, p. 11), ao constatar a
tradição biológica da psicologia, em que o indivíduo era considerado um organismo que interage no meio
físico, sendo que os processos psicológicos (o que ocorre "dentro" dele) são assumidos como causa, ou
uma das causas que explicam o seu comportamento. Ou seja, para compreender o indivíduo, bastaria
conhecer o que ocorre "dentro" dele, quando ele se defronta com estímulos do meio.
Ao se confrontar as teorias de cunho exclusivamente biológico, busca-se ainda em Lane e Godo (1999,
p. 12) a indicação dessa base social: o homem fala, pensa, aprende e ensina, transforma a natureza; o
homem é cultura, é história. Este homem biológico não sobrevive por si e nem constitui uma espécie que
se reproduz tal e qual com variações decorrentes de clima, alimentação etc. O seu organismo é uma
infraestrutura que permite o desenvolvimento de uma superestrutura que é social e, portanto, histórica.
Desse modo, ampliam-se o conceito e a noção de que buscar exclusivamente no indivíduo que
cometeu o crime a resposta para este ato significa restringir a um universo individual aquilo que se
encontra em constante movimento, em constante interação: o comportamento humano.
Apostila gerada especialmente para: Betânia Mueller 023.712.580-32
. 252
Na intersecção entre direito penal e psicologia, isso fica bastante evidente. Em geral, observa-se que
a conjunção da história individual com a história social produz o caldo que dá espaço para o surgimento
da maioria dos conflitos que chegam aos tribunais.
Não se trata, porém, de descartar a existência de casos em que prepondera a influência do
componente biopsíquico, em que o transtorno de caráter pode estar relacionado a um comprometimento
da saúde mental em grau elevado, embora não sejam estes os que se encontrem com mais frequência
no sistema prisional. Além disso, mesmo estes devem ser entendidos sob a ótica da saúde e não do
sistema penal, uma vez que pessoas nessas condições são classificadas como inimputáveis perante a
lei penal, devendo receber, pois, intervenção focada na saúde do indivíduo.
Há, ainda, que se considerar a extraordinária plasticidade do comportamento humano e sua inigualável
capacidade de se adaptar às mais complexas e inusitadas situações.
O que fazer, pois, em relação ao fenômeno delitivo?
Sem dúvida, aprofundar-se sobre os fatores que levam ao comportamento indesejado é imprescindível;
nesse caso, quantidade e qualidade de informações são igualmente importantes. Tamanha é a
diversidade das condições sociais que a obtenção de suficiência estatística em qualquer análise requer
uma extensa (e exasperante) segmentação do público-alvo, que leve em conta:
- faixas etárias;
- características étnicas;
- características psicológicas e comportamentais;
- microssistemas sociais;
- grupos de atuação;
- escolaridade;
- especialidade profissional;
- opções políticas, religiosas e outras etc.
Além disso, é necessário que o fenômeno seja entendido na relação com os desencadeantes dos
comportamentos, que compõem outra matriz de fatores.
Observa-se que a visão sistêmica, oferece uma gama de informações relevantes para este estudo,
considerando "sistemas" no plural: família, grupo social, grupo de funcionários da justiça (tanto na fase
processual, quanto de execução da pena) etc.

Hipóteses
Diante da abrangência do assunto, cabe ressaltar a importância de não limitar as abordagens a uma
visão reducionista, quer pelo aspecto biológico, quer social. Os estudos acerca da influência do meio vêm
ganhando maior destaque, hajam vistas as dimensões apontadas por Baratta (1990; 1999) e Zaffaroni
(1998); por outro lado, estudos sobre o funcionamento cerebral e influências endócrinas algumas vezes
recebem proeminência.
Comentam-se, a seguir, duas dessas hipóteses. a) O crime como resultado da privação
Do ponto de vista das teorias que privilegiam a percepção, demonstra-se que a privação tem antes um
efeito relativo do que absoluto. Trata-se, aqui, da relação figura e fundo.
Se, por um lado, a privação, tanto econômica, quanto afetiva, pode influenciar negativamente no
desenvolvimento do ser humano, por outro, encontram-se diversos exemplos na vida cotidiana que
indicam a possibilidade de um comportamento adaptativo e resiliente que levam indivíduos a reagir
satisfatoriamente, do ponto de vista social, mesmo quando submetidos a ela. Por exemplo, comunidades
carentes cuja adesão a comportamentos criminosos é irrelevante ou situa-se dentro dos padrões da
população.
Assim não fosse, um país com imensos desequilíbrios econômicos como o Brasil veria a população
que mais padece de privações engolida por um turbilhão de delitos, o que, absolutamente, não acontece.
A situação oposta ratifica essa conclusão; os graves delitos cometidos por indivíduos no outro extremo
da cadeia social (que se suspeita tenham incidência dentro da média desse tipo de comportamento) tanto
sinalizam para a veracidade do efeito da percepção sobre os comportamentos como para a evidência de
que a privação nem sempre é determinante para o comportamento delituoso.
Além disso, reconhece-se que são inúmeras as vias de solução da privação, que não a delinquência,
tais como:
- a elaboração psíquica; influenciada, sobremaneira, pela aquisição de novas habilidades e pelo
desenvolvimento cognitivo, para o qual a escola contribui fortemente;
- o deslocamento ou a sublimação, como acontece com o relacionamento amoroso, a dedicação a
atividades de alcance social (lucrativas ou não), a prática de atividades prazerosas;
- procedimentos obsessivos estereotipados, socialmente ajustados, como a concentração no trabalho
e sua utilização com objeto substitutivo;

Apostila gerada especialmente para: Betânia Mueller 023.712.580-32


. 253
- transtornos mentais como a ciclotimia, a distimia, a depressão;
- a drogadição, anteriormente estudada e que constitui um mecanismo eficiente (mas não eficaz) de
fuga.
A delinquência, sob esta ótica, pode ser percebida como um mecanismo de reação à privação; eficaz,
quando comparada com a drogadição ou o transtorno mental; ineficaz, quando esse mecanismo cronifica-
se e acaba por afastar o indivíduo, definitivamente, do convívio social satisfatório ou o leva a produzir
males que não têm possibilidade de reparação (como o homicídio).
b) O crime como produto do meio
Vê-se, aqui, a delinquência como o resultado inexorável do microssocial. Nega-se ao indivíduo o livre-
arbítrio e a possibilidade de percorrer um caminho diferente daquele dos seus pares. A respeito do livre-
arbítrio, indica-se o filme O advogado do diabo (Taylor Hackford, 1997), em que o leitor identificará,
também, o processo de vitimização.
Esta hipótese considera, implicitamente, que:
- os condicionamentos (comportamentos aprendidos do grupo) não podem ser alterados, ainda que o
indivíduo tenha a oportunidade de praticar novas convivências. A esse respeito, durante dez meses, um
grupo de formandos em Psicologia, da Universidade Tuiuti do Paraná, observou o comportamento de
crianças recolhidas de rua em uma instituição de acolhimento.
Verificou-se a grande dificuldade de alterar os condicionamentos (desde a disposição para dormir no
chão, a negativa em praticar a higiene pessoal etc.).
Entretanto, em diversas atividades, verificou-se uma rápida modificação dos condicionamentos; por
exemplo, a aceitação de rotinas de trabalho produtivo em que o benefício/custo mostrava-se visível e a
curto prazo.
A conclusão é que os condicionamentos estabelecidos são fortes, porém, não substituíveis e tudo
depende de uma conjugação de fatores, entre os quais, a persistência talvez seja dos mais importantes;
- os indivíduos tornam-se escravos de seus modelos. No ambiente original, modelos inadequados
orientam, comandam e gratificam. Esses modelos, de grande poder sob o ponto de vista emocional,
tornam-se pessoas extremamente significativas para os indivíduos, que com eles desenvolvem vínculos
afetivos, de fidelidade, extremamente duradouros.
A grande dificuldade encontra-se em proporcionar modelos alternativos.
Na mesma experiência relatada, o novo condicionamento era conseguido porque o modelo,
representado pelo líder da instituição, conseguia desenvolver, rapidamente, uma forte relação empática
com o público-alvo e demonstrava, por suas ações concretas (não intelectuais), que seus procedimentos
eram eficientes para combater as três maiores carências daquela população: o frio, o medo da violência
e a fome.
Inúmeras tentativas de recuperação de delinquentes esbarram na dificuldade de oferecer modelos
substitutivos que lhes acenem (e cumpram) com vantagens que lhes dispensarão o agasalho emocional
do modelo inadequado anterior;
- novos modelos surgem, também, através das redes de comunicação. Não há mais necessidade de
se experimentar o contato físico ou a proximidade daquele que será eleito como modelo.
No mundo virtual, o modelo nem mesmo precisa ter existência física: pode ser um personagem de
desenho animado, por exemplo. Os comportamentos desses modelos virtuais (em jogos, por exemplo)
estendem-se aos extremos do bem e do mal e, naturalmente, avalizam as correspondentes reproduções.
Essa questão agrava-se quando se vai além da comunicação pelos sistemas abertos ao público (redes
sociais similares ao Facebook). Ao se adentrar no pantanoso terreno da comunicação utilizando os
protocolos de acesso direto aos equipamentos, sem qualquer monitoração do ponto de vista ético ou
legal, abrem-se os portões para os "monstros do Id”: aplainam-se os terrenos para a explosão de insólitos
e iníquos desejos e desvios de conduta e de caráter.
Tamanha facilidade presta-se à ação criminosa de alcance individual, por exemplo, em que se exibe o
assassinato "ao vivo", e/ou de abrangência coletiva, quando preparam-se ações contra a coletividade,
como as de vandalismo e terrorismo.
Medram nesse ambiente os modelos estimulantes de comportamentos inimagináveis pelos padrões
comuns da sociedade. Subjaz a percepção de uma teia de comunicações protetora dos agentes,
ensejando-lhes sentimentos de impunidade e invulnerabilidade.
Vastos, dispersos e diferenciados os campos de aplicação! Não há limite para a criatividade humana:
as aplicações incluem tráfico de drogas, pornografia infantil, contratação de execuções, suicídios
assistidos, crimes de racismo — apenas para citar os mais suportáveis para o senso comum.
- os indivíduos são dominados por crenças inadequadas, perversas, antissociais.
Essas crenças, ainda que possam ter esses rótulos, são aquelas que produziram resultados mais
satisfatórios para lidar com os desafios da sobrevivência e/ou para suprir as demandas desses indivíduos.

Apostila gerada especialmente para: Betânia Mueller 023.712.580-32


. 254
Elas serão substituídas por outras, desde que estas apresentam vantagens, do ponto de vista psíquico,
em relação àquelas.
Estas vantagens devem levar em consideração o nível de pensamento do público-alvo. Não se pode
acenar com crenças cuja realização aconteça em um futuro remoto quando o indivíduo apresenta um
pensamento operatório-concreto (na concepção piagetiana já apresentada).
Um dos grandes motivos de abandono de qualquer tipo de desenvolvimento psicológico, inclusive na
escola convencional, é a apresentação de conceitos que requerem elaboração mental em níveis de
pensamento muito superiores àqueles que o indivíduo consegue elaborar (isso explica, por exemplo, a
dificuldade de crianças com matemática).
A substituição de crenças deve obedecer uma estratégia de deslocamento gradativo em quantidade e
em qualidade. Em quantidade, porque, quanto mais próximo do concreto for o nível de pensamento, mais
reduzida será a capacidade de processamento mental; em qualidade, porque os conceitos deverão
subordinar-se, em simplicidade e clareza, às possibilidades da linguagem desses indivíduos.
A análise dessas duas hipóteses sugere ao leitor uma linha de avaliação de qualquer outra hipótese
reducionista, que pode ser realizada a partir das teorias psicológicas e dos conceitos e propriedades das
funções mentais superiores vistos neste livro.

As Modalidades de Crime

A conduta humana é tipificada como crime a partir da ilicitude e materialidade do fato. Antes, porém,
da realização do delito, esta ação percorre um caminho subjetivo que vai da leve sugestão interna ou
desejo à intenção, decisão e o efetivo cometimento, o qual, não encontrando resistências internas e/ou
externas, eclode para o social.
Nesta seção, privilegia-se o enfoque psicológico sobre estas condutas, relembrando que o
comportamento humano é dinâmico e em constante interação com os demais.
O sistema de crenças coloca justificativas para os comportamentos que escapam à normalidade social.
São situações de extrema gravidade, em geral envolvendo grupo significativo de pessoas que se apoiam
mutuamente, em suas ações, em torno de motivos relacionados com suas crenças. O denominador
comum é a aderência a valores ou códigos de seus próprios grupos.
Encontra-se, por exemplo, em pessoas com forte ligação a grupos religiosos, rigorosos e polarizados.
Alguns exemplos dão a dimensão do potencial alcance desse tipo de movimento:
- a Ku-Klux-Klan nos EUA (surgida no Tennessee, em 1865; ativa até 1944, notabilizando-se pelas
torturas e linchamentos); ainda mantém simpatizantes;
- a Inquisição, movimento religioso que protagonizou dezenas de milhares de assassinatos.
A manifestação contemporânea corresponde aos grupos de skinheads (que pregam o preconceito
contra outros grupos, corno, por exemplo, os homossexuais); grupos ultrarradicais de torcidas
organizadas; equipes de luta etc.
O ponto central nesses movimentos é a inexistência de sentimento de culpa. Liderança forte,
sentimentos de afiliação intensos, simbologia marcante e de grande significado estimulam a coesão e a
perseguição dos objetivos comuns.
Avalia-se que, do ponto de vista de estrutura psíquica, esses indivíduos comungam de sentimentos
relacionados a:
- medo (de perda de benefícios, por exemplo);
- inferioridade (real ou apenas percebida; presente ou futura);

Tem especial interesse para o Direito a intenção que cerca o ato criminoso, por parte de quem o
comete. Sob essa ótica, os delitos dividem-se em dois grandes grupos:
- o delito culposo, que consiste na prática de ato voluntário, porém, com resultados involuntários;
- o delito doloso, em que ocorre ato voluntário com resultado esperado.

Delito doloso

No delito doloso, há evidência da vontade consciente; contudo, a análise, em profundidade, das


motivações que levam a pessoa à prática do delito agregará elementos para melhor ilustrar a maneira
como essa vontade se expressa.
Em primeiro lugar, é de se considerar que a consciência estabeleça um diálogo com o inconsciente,
na avaliação da vantagem de perpetrar o ato. Desse diálogo (desconhecido, obviamente, pelo indivíduo)
resultará o mecanismo de defesa que o próprio inconsciente utilizará para justificar-se pelo
comportamento.

Apostila gerada especialmente para: Betânia Mueller 023.712.580-32


. 255
Entre esses mecanismos, destacam-se:
- a projeção: atribui-se a alguém a culpa pelo próprio insucesso ou infortúnio.
- a racionalização: inventa-se uma razão para justificar o ato censurável.
O delito doloso encontra fácil justificativa no desequilíbrio emocional: ele se apresenta como a solução
que o psiquismo dispõe para dar fim à evolução de um conflito em que o estresse se acumula e precisa
de uma válvula de escape. Explode a pólvora ou explode a mente. Esta escolhe aquela.
Em boa parte das situações, o condicionamento surge como uma explicação razoável para o
comportamento. Na colisão na rotatória, na agressão no trânsito, tudo parece indicar que os agressores
são dados a comportamentos violentos, à reação impulsiva, inconsciente em sua aparência.
Vistas mais de perto, essas reações não são assim tão impulsivas, porque elas acontecem
corriqueiramente, em inúmeras situações que pontuam a vida dessas pessoas. Inúmeras vezes,
conhecidos e familiares já lhes sinalizaram a respeito dos problemas desses comportamentos que são
mantidos por esses indivíduos.
Conforme se comentou, trata-se de uma "drogadição interna". A mente acostuma-se com a adrenalina
em altos níveis e solicita-a. Ou recebe doses extras por meio do mecanismo de sublimação, ou vem a
explosão, dirigida ao alvo mais próximo. Não há como debitar essa conta ao passivo do inconsciente.

Delito culposo

O Código Penal refere-se a três situações às quais aplica-se a classificação de delito culposo:
- a imprudência;
- a negligência; e
- a imperícia.
O caso paradigmático pode ser o do cirurgião que "esquece" uma ferramenta no interior da cavidade,
provocando a morte do paciente, ou ainda o empregado que, contrariado com o que ele interpretou como
"injusta determinação do patrão", "esquece-se" de trancar a porta da loja e esta é saqueada.
Sob a ótica da psicologia, todas essas situações apresentam interpretações que roubam a
responsabilidade das mãos do acaso, para transferi-la, em variados graus, para as mãos do autor —
ainda que se reconheça o caráter inconsciente do comportamento delituoso.
Para ancorar esse raciocínio, será utilizado o caso que envolve três personagens:
a) Wilson, o jovem imprudente;
b) Ivã, o pai negligente; e
c) Neuza, a mãe dependente.
De Wilson, tudo se espera. Das drogas leves à pesada; dos pequenos delitos aos graves; no final do
túnel espera-o, não a luz, mas a escuridão do homicídio, por exemplo.
Nesse percurso, a saga de Wilson encontra-se já diagramada aguardando a arte-final dos
acontecimentos. O destino, pacientemente, ajusta o story board para inserir as vítimas involuntárias — a
senhora com a filhinha que aguarda o ônibus e receberá o efeito da derrapada do BMW; ou o aposentado
que se livrará da fila mensal pela bala perdida no assalto ao banco ou algo assim.
É preciso desenhar esse percurso. O que move Wilson? A aventura ou a busca do desfecho? Qual
desfecho?
Wilson vai colocando depósitos na caderneta de poupança de culpas que acumula (reveja-se o
conceito de dissonância cognitiva, no capítulo de teorias). Não há saques. A contabilidade emocional não
consegue fechar o ativo dos comportamentos com o passivo dos compromissos.
Se ao observador externo apresentam-se as cores da imprudência, ao analista revela-se a busca da
redenção - a "pulsão de morte" de Freud - que se realiza por meio do Outro para voltar para ele mesmo.
Não tem a coragem para se enfrentar, mas tem a vantagem de o Outro nada significar para ele. Assim
"morre" Wilson, quando sepulta os valores sociais sob as ferragens ou fere-os mortalmente pela via
simbólica da hemorragia de um desconhecido e resgata o investimento acumulado.
A defesa de Wilson será paga pelo pai, ainda que nisso ele empenhe tudo o que já angariou. Wilson,
o mesmo que roubou dele e dos irmãos as atenções de Neuza, que se tornou o "queridinho da mamãe".
Depois que ele chegou, a família transformou-se. Tudo girava em tomo dele. Nada mais se fazia sem
que ele não fosse o centro das atenções.
Ivã descuidou da escolinha. Não prestou atenção quando ele começou, mais tarde, a repetir. Não tinha
tempo. Não tinha vontade. Alguém tinha que cuidar dos mais velhos...
Afinal, ele tinha a mãe (que já não era esposa...). Mas, agora, poderia contar com o pai, que não iria
abandoná-lo, mesmo distante. Não se rasga a promissória da paternidade.
Neuza, coitada, depois que Wilson nasceu, viveu para ele. Só para ele. Conhecia-lhe os mínimos
gestos e desejos. A ele dedicava todas as preces e sofria com seus fracassos.

Apostila gerada especialmente para: Betânia Mueller 023.712.580-32


. 256
Como exigir dele o que exigiu dos irmãos? Eles eram mais fortes, mais dedicados aos estudos. Wilson
precisava dela, não era uma opção, era um dever de mãe.
O desafio de transformar Wilson em pessoa responsável, que assuma seu papel na sociedade, é
monumental.

Delinquência ocasional

Denomina-se "ocasional" o delito praticado por agente até então socialmente ajustado e obediente à
lei, que só chegou à ação antissocial respondendo a uma forte solicitação externa (MARANHÃO, 1981,
p. 52). Em geral, apresenta uma personalidade ajustada aos padrões de normalidade.
Alguns dos casos aqui apresentados referem-se a esse tipo de comportamento (casos 1.1, 1.4, 2.2,
7.1).
Já se tratou aqui da prática de pequenos delitos e de seu impacto na construção de um comportamento
rotineiro, principalmente quando a cada delito não corresponde algum tipo de consequência.
Nesse caso, surge o condicionamento para o ato delituoso, cuja remissão torna-se difícil pela
frequência com que as oportunidades se apresentarão.
A delinquência ocasional, entretanto, de grande dimensão - como o homicídio - surge como resposta
a uma forte emoção e sua repetição torna-se mais improvável pelas características do momento ou do
agente
Estas considerações têm por finalidade reafirmar ao leitor a importância de se dar atenção aos
pequenos delitos porque, pela via da delinquência ocasional, eles se tornam a porta para comportamentos
delituosos persistentes e evolutivos.

Delinquência psicótica

É a prática criminosa que se efetiva em função de um transtorno mental.


Diversas psicopatologias podem conduzir a comportamento delitivo; devem ter diagnóstico por
especialista e é indispensável que o quadro seja predominante ao tempo da ação.
Uma dificuldade, para essa comprovação, é o tempo transcorrido entre o fato e a avaliação.
A delinquência psicótica, entretanto, ocasiona grande impacto emocional ao observador porque:
- há o temor de que o comportamento se repita;
- não existe um quadro de referências que se possa considerar associado ao comportamento; ele pode
ocorrer a qualquer momento, em qualquer lugar;
- há também o temor de que o indivíduo seja considerado "curado" quando isso, de fato, não aconteceu
e ele é liberado da instituição de exclusão à qual possa ter sido recolhido, por medida de segurança e
para tratamento;
- finamente, tem-se também o temor de que a delinquência psicótica possa ser uma sofisticada
simulação.
Ressalte-se que o conceito de medida de segurança está ligado ao de periculosidade e de intervenção
por profissional de saúde.

A exposição de motivos do Código Penal assevera:


a medida de segurança, de caráter meramente preventivo e assistencial, ficará reservada aos
inimputáveis. Isso, em resumo, significa: culpabilidade — pena; periculosidade — medida de segurança
[...] duas espécies de medida de segurança consagra o projeto: a detentiva e a restritiva. A detentiva
consiste na internação em hospital de custódia e tratamento psiquiátrico, fixando-se o prazo mínimo de
internação entre um e três anos [...] a restritiva consiste na sujeição do agente a tratamento ambulatorial
cumprindo-lhe comparecer ao hospital nos dias que lhe forem determinados pelo médico, a fim de ser
submetido à modalidade terapêutica prescrita.

A espécie restritiva é uma inovação que corresponde ao cometimento de crimes menos graves e que
encontra eco na desinstitucionalização, na desinternação progressiva e na luta antimanicomial.

Delinquência neurótica

Na delinquência neurótica, a conduta delitiva é encarada corno uma manifestação dos conflitos do
sujeito com ele mesmo. O que incomoda o psiquismo reflete-se no ato, com a finalidade inconsciente
(total ou parcial) de punição. Trata-se, pois, de urna delinquência sintomática.
A punição serve para aplacar um sentimento de culpa de outra origem (o conflito primário).

Apostila gerada especialmente para: Betânia Mueller 023.712.580-32


. 257
Maranhão distingue "neurose" de "personalidade delinquente", conforme o quadro seguinte.

NEUROSE PERSONALIDADE DELINQUENTE


Conflito interno Aparentemente sem conflito interno
Agressividade voltada a si Agressividade voltada à sociedade
Gratificação por meio de fantasias Alívio de tensões internas por ações criminosas
Admissão dos próprios impulsos e reconhecimento
Atribuição de seus impulsos ao mundo exterior
dos erros
Desenvolvimento de reações emocionais positivas Desenvolvimento de defesas emocionais
Superego desenvolvido Superego desarmônico
Comportamento dissociai (desconsideração para
Comportamentos socialmente ajustados
com os códigos sociais)
Reação à passividade e dependência com Tentativa de negar a passividade e a dependência
sofrimento, mas admitindo a situação com atitudes agressivas
Caráter "normal" Caráter deformado (dissociai)
Perturbações psicossomáticas menos frequentes Perturbações psicossomáticas mais frequentes

Delinquência profilática

O agente entende que estará evitando um mal maior e não revela remorso; por exemplo, a eutanásia.
Nas situações anteriormente citadas, em que grupos atuam movidos por poderosas crenças comuns,
pode haver a interpretação de que suas ações tenham a genuína missão de profilaxia social.
Se essa compreensão (eventualmente) não passa pelos líderes, que se aproveitam dos benefícios
econômicos dos movimentos, não se estranhe de que seja percebida como real por substancial parte dos
liderados. Isso, entretanto, não significa que as fragilidades psíquicas e a personalidade antissocial não
constituam elementos predominantes nesses indivíduos.
As questões que envolvem a delinquência profilática são de grande complexidade e devem ser
analisadas com estreita observância dos aspectos sociais e culturais que sobre ela exercem influência
determinante.
Elas podem, por exemplo, no caso de violência doméstica, em que pai ou mãe inflingem severos
castigos aos filhos, resultar de um procedimento já convencional na história familiar, pelo qual os
progenitores mesmos passaram. Não praticar esse tipo de procedimento significaria, sob a ótica da
família, a não assunção de responsabilidade sobre a educação das crianças.
Contornos diferentes, porém relacionados, ganham os delitos praticados por justiceiros, presos em
cumprimento de penas, e até mesmo por representantes das forças de segurança pública. Sob a ótica da
assepsia social, manifestam-se em homicídios, sevícias e torturas. Evidencia-se um tênue limite entre o
caráter "pedagógico" que lhes imprimem seus autores, procurando evitar o "mal maior", e um caráter
meramente vingativo.
É comum, por exemplo, que em prisões determinados delitos sujeitem aqueles que os praticaram a
também experimentá-los, praticados por outros presos. Por exemplo, o abuso sexual; deve-se cuidar para
garantir a integridade física do detento, porque esse tipo de crime, em geral, desperta sentimentos de
repugnância e desejo de vingança para os demais sentenciados.
Indo além: há ações a tal ponto não toleradas pela população prisional que o castigo, não havendo
nada que o impeça, é a morte daqueles que a praticaram, retomando uma forma antiga de se fazer justiça,
a pena de Talião, o "olho por olho, dente por dente". Esse procedimento funciona como uma expiação de
culpa, com a qual os sentenciados se redimem, senão perante a sociedade, perante os próprios
psiquismos.

Drogas, assistência e cidadania.

Drogas são substâncias utilizadas para produzir alterações, mudanças nas sensações, no grau de
consciência e no estado emocional. As alterações causadas por essas substâncias variam de acordo com
as características da pessoa que as usa, qual droga é utilizada e em que quantidade, o efeito que se
espera da droga e as circunstâncias em que é consumida.
Geralmente achamos que existem apenas algumas poucas substâncias extremamente perigosas: são
essas que chamamos de drogas. Achamos também que drogas são apenas os produtos ilegais como a
Apostila gerada especialmente para: Betânia Mueller 023.712.580-32
. 258
maconha, a cocaína e o crack. Porém, do ponto de vista da saúde, muitas substâncias legalizadas podem
ser igualmente perigosas, como por exemplo o álcool, que também é considerado uma droga como as
demais.
Drogas psicotrópicas são aquelas que atuam sobre nosso cérebro, alterando de alguma maneira nosso
psiquismo, podendo deprimir, estimular ou perturbar a atividade do Sistema Nervoso Central.

Drogas psicotrópicas: trópica = tropismo (ter atração por) psico = (psiquismo).

Drogas depressoras da atividade do SNC:


- Álcool
- Soníferos
- Morfina
- Heroína
- Inalantes ou solventes

Drogas estimulantes do SNC:


- Anfetaminas
- Cocaína

Drogas perturbadoras da atividade do SNC:


- LSD
- Ecstasy

Tipos de drogas e seus efeitos

As drogas atuam no cérebro afetando a atividade mental, sendo por essa razão denominadas
psicoativas. Basicamente, elas são de três tipos:
- Drogas que diminuem a atividade mental, também chamadas de depressores. Afetam o cérebro,
fazendo com que funcione de forma mais lenta. Essas drogas diminuem a atenção, a concentração, a
tensão emocional e a capacidade intelectual. Exemplos: Ansiolíticos (tranquilizantes), álcool, inalantes
(cola), narcóticos (morfina, heroína);
- Drogas que aumentam a atividade mental, são chamadas de estimulantes. Afetam o cérebro, fazendo
com que funcione de forma mais acelerada. Exemplos: Cafeína, tabaco, anfetamina, cocaína, crack;
- Drogas que alteram a percepção, são chamadas de substâncias alucinógenas e provocam distúrbios
no funcionamento do cérebro, fazendo com que ele passe a trabalhar de forma desordenada, numa
espécie de delírio. Exemplo: Lsd, ecstasy, maconha e outras substâncias derivadas de plantas.

Álcool:

A ingestão de álcool provoca diversos efeitos, que aparecem em duas fases distintas: uma estimulante
e outra depressora.
Nos primeiros momentos após a ingestão de álcool, podem aparecer efeitos estimulantes, como
euforia, desinibição e loquacidade (maior facilidade para falar).
Com o passar do tempo, começam a surgir os efeitos depressores, como falta de coordenação motora,
descontrole e sono. Quando o consumo é muito exagerado, o efeito depressor fica exarcebado, podendo
até mesmo provocar o estado de coma. Os efeitos do álcool variam de intensidade de acordo com as
características pessoais.

Solventes ou Inalantes:

- Primeira fase: A chamada fase de excitação, que é desejada, pois a pessoa fica eufórica,
aparentemente excitada, sentindo tonturas e tendo perturbações auditivas e visuais. Também pode
aparecer tosse, espirros, muita salivação e as faces podem ficar avermelhadas.
- Segunda fase: A depressão do cérebro começa a predominar, ficando a pessoa confusa,
desorientada, com a voz meio pastosa, visão embaçada, perda do autocontrole, dor de cabeça, palidez;
ela começa a ver ou a ouvir coisas.
- Terceira fase: A depressão aprofunda-se com redução acentuada do estado de alerta, incoordenação
ocular (a pessoa não consegue mais fixar os olhos nos objetos), incoordenação motora com marcha
vacilante, fala “enrolada”, reflexos deprimidos, podendo ocorrer processos alucinatórios evidentes.

Apostila gerada especialmente para: Betânia Mueller 023.712.580-32


. 259
- Quarta fase: Depressão tardia, que pode chegar a inconsciência, queda da pressão, sonhos
estranhos, podendo ainda a pessoa apresentar surtos de convulsões (“ataques”). Essa fase ocorre com
frequência entre aqueles cheiradores que usam saco plástico e, após um certo tempo, já não conseguem
afasta-lo do nariz e, assim, a intoxicação torna-se muito perigosa, podendo levar ao coma e a morte.

Cocaína e Crack:

Os efeitos provocados pela cocaína ocorrem por todas as vias (aspirada, inalada, endovenosa). Logo
após do uso, o usuário tem uma sensação de grande prazer, intensa euforia e poder.
A tendência do usuário é aumentar a dose da droga na tentativa de sentir efeitos mais intensos. Porém,
essas quantidades maiores acabam por levar o usuário a comportamento violento, irritabilidade, tremores
e atitudes bizarras devido ao aparecimento de paranóia. No caso do usuário de crack, isso provoca um
grande medo e desconfiança e passam a vigiar o local onde usam a droga. Eventualmente, podem ter
alucinações e delírios. A esse conjunto de sintomas dá-se o nome de “psicose cocaínica”. Além dos
sintomas descritos os usuários de cocaína e crack perdem muito peso em poucas semanas e perdem o
interesse sexual.

Tabaco (cigarro):

Quando o fumante dá uma tragada, a nicotina é absorvida pelos pulmões, chegando ao cérebro
aproximadamente em nove segundos.
Os principais efeitos da nicotina no sistema nervoso central consiste em elevação do humor,
estimulação e diminuição do apetite.
A nicotina produz um pequeno aumento no batimento cardíaco, na pressão arterial, na frequência
respiratória e na atividade motora.
O uso intenso e constante de cigarros aumenta a probabilidade de ocorrência de alguma doenças
como, por exemplo, pneumonia, câncer de (pulmão, laringe, faringe, esôfago, boca, estômago etc.),
infarto de miocárdio, bronquite crônica, enfisema pulmonar, derrame cerebral, úlcera digestiva e etc.

Maconha:

Efeitos físicos agudos: os olhos ficam avermelhados, a boca fica seca, o coração dispara, de 60 a 80
batimentos por minuto ou até mesmo mais (taquicardia).
Os efeitos psíquicos agudos dependerão da qualidade da maconha fumada e da sensibilidade de quem
fuma. Para uma parte de quem fuma os efeitos são de bem-estar acompanhada de calma e relaxamento,
sentir-se menos fadigado, vontade de rir, e para outras pessoas os efeitos são mais para o lado
desagradável, pois, sentem angústia, temerosas de perder o controle mental, trêmulas, suadas. È o que
comumente chamam de “bad trip” (má viagem).

Há, ainda, perturbação na capacidade da pessoa calcular o tempo e espaço e um prejuízo de memória
de curto prazo e atenção.
Aumentando-se a dose ou dependendo da sensibilidade, os efeitos psíquicos agudos podem chegar
até a alterações mais evidentes, com predominância de delírios e alucinações.
No caso do delírio a pessoa escuta a sirene da ambulância e julga que é a polícia que vem prende-la.
Na alucinação a pessoa tem uma percepção sem o objeto, isto é, a pessoa pode ouvir a sirene da
polícia ou ver duas pessoas conversando quando não existe nem sirene nem pessoas.
Os efeitos físicos crônicos da maconha já são de maior gravidade. A maconha contém alto teor de
alcatrão e nele existe uma substância chamada benzopireno, conhecido agente cancerígeno.
Outro efeito físico adverso do uso crônico da maconha refere-se á testosterona. Consequentemente o
homem terá muita dificuldade de gerar filhos e é importante dizer que o homem não perde interesse
sexual, só fica incapacitado de engravidar sua companheira.
Sabe-se que o uso continuado interfere na capacidade de aprendizagem e memorização e pode induzir
a um estado de “amotivação”, a pessoa não tem vontade de fazer mais nada, pois tudo fica sem graça e
sem importância.

LSD:

O Lsd atua produzindo uma série de distorções no funcionamento do cérebro, trazendo como
consequência uma variada gama de alterações psíquicas.

Apostila gerada especialmente para: Betânia Mueller 023.712.580-32


. 260
A experiência subjetiva com o Lsd depende da personalidade do usuário, de suas expectativas quanto
ao uso da droga e do ambiente onde esta é ingerida.
O Lsd é capaz de produzir distorções na percepção do ambiente, cores, formas e contornos alterados,
além de estímulos olfativos e táteis parecem visíveis e cores podem ser ouvidas. Outro aspecto que
caracteriza a ação do Lsd no cérebro refere-se aos delírios. Estes são chamados os “falsos juízos da
realidade”, isto é, há uma realidade, um fato qualquer, más a pessoa delirante não é capaz de avalia-la
corretamente.
Logo após toma-lo, o pulso pode ficar mais rápido, as pupilas podem ficar dilatadas.
Ainda no campo dos efeitos tóxicos, há também descrições de pessoas que, após tomarem o Lsd,
passaram a apresentar por longos períodos de ansiedade, depressão ou mesmo acessos psicóticos.

Ecstasy:

A droga apresenta efeitos semelhantes aos estimulantes do sistema nervoso central, (agitação), bem
como efeitos perturbadores (mudança na percepção da realidade).
Seus efeitos mais marcantes são a sensação de melhora nas relações entre as pessoas, o desejo de
se comunicar, melhora na percepção musical e aumento nas percepções das cores.
O Ecstasy causa, também, diminuição do apetite, dilatação das pupilas, aceleração do batimento
cardíaco, aumento da temperatura do corpo (hipertemia), rangido de dentes e aumento na secreção do
hormônio antidiurético.
Muitos usuários relatam ter um episódio depressivo nos dias após o uso da droga, o que é chamada
de depressão de meio de semana. Fadiga e insônia também são comuns.

Dependência

Podemos definir uso como qualquer consumo de substâncias (experimental, esporádico ou episódico),
abuso ou uso nocivo como sendo um consumo de substâncias que já está associado a algum prejuízo
(quer em termos biológicos, psicológicos ou sociais) e, por fim, dependência como o consumo sem
controle, geralmente associado a problemas sérios para o usuário.
Dependência é o impulso que leva a pessoa a usar uma droga de forma contínua (sempre) ou periódica
(frequentemente) para obter prazer. Alguns indivíduos podem também fazer uso constante de uma droga
para aliviar tensões, ansiedades, medos, sensações físicas desagradáveis etc. O dependente caracteriza-
se por não conseguir controlar o consumo de drogas, agindo de forma impulsiva e repetitiva.
Para compreendermos melhor a dependência, vamos analisar as duas formas principais em que ela
se apresenta: a física e a psicológica.
A dependência física se caracteriza pela presença de sintomas e sinais físicos que aparecem quando
o indivíduo para de tomar a droga ou diminui bruscamente o seu uso: é a síndrome de abstinência. Os
sinais e sintomas de abstinência dependem do tipo de substância utilizada e aparecem algumas horas ou
dias depois que ela foi consumida pela última vez. No caso dos dependentes de álcool, por exemplo, a
abstinência pode ocasionar desde um simples tremor nas mãos a náuseas, vômitos e até um quadro de
abstinência mais grave denominado “delirium tremes”, com risco de morte, em alguns casos.
Já a dependência psicológica corresponde a um estado de mal-estar e desconforto que surge quando
o dependente interrompe o uso de uma droga. Os sintomas mais comuns são ansiedade, sensação de
vazio, dificuldade de concentração, más que podem variar de pessoa para pessoa.
Com os medicamentos existentes atualmente, a maioria dos casos relacionados a dependência física
podem ser tratados. Por outro lado, o que quase sempre faz com que uma pessoa volte a usar drogas é
a dependência psicológica, de difícil tratamento e não pode ser resolvida de forma relativamente rápida
e simples como a dependência física.

Transtornos psiquiátricos mais associados às dependências

A depressão é o transtorno que mais se associa ao abuso e a dependência de drogas. Outros


transtornos frequentemente encontrados entre os dependentes são os: transtorno de ansiedade, o
obsessivo-compulsivo, os de personalidade e, mais raramente, alguns tipos de psicoses. Mais
recentemente descobriu-se que indivíduos com transtornos neurocognitivos estão mais propensos a se
tornarem dependentes de drogas. Esses podem se manifestar através de problemas de atenção,
memória, concentração ou linguagem, entre outros.
A grande dificuldade decorre de, com muita frequência, esses sinais não serem identificados nem pelos
familiares nem pela escola, podendo estar presentes desde a mais tenra idade.

Apostila gerada especialmente para: Betânia Mueller 023.712.580-32


. 261
Exemplificando, muitos jovens considerados rebeldes, preguiçosos, desinteressados, vagabundos ou
indisciplinados, na verdade podem apresentar um transtorno específico de aprendizagem ou de atenção.
É importante ressaltar que esses transtornos prejudicam profundamente a autoestima e o
desenvolvimento das crianças e dos jovens, atrasando ou até mesmo impossibilitando o uso de suas
potencialidades.
Se fossem diagnosticados de modo correto, esses distúrbios poderiam ser facilmente tratados,
evitando assim consequências drásticas que ocorrem quando não são identificados. Como exemplo
disso, muitos dependentes de drogas que apresentavam transtorno de atenção, quando o problema foi
adequadamente tratado, pararam de consumir drogas.
Por que se tratar

A dependência de substâncias psicoativas (álcool e drogas) é uma síndrome médica bem definida
internacionalmente, cujo diagnóstico é realizado pela presença de uma variedade de sintomas que
indicam que o indivíduo consumidor apresenta uma série de prejuízos e comprometimentos devido ao
seu consumo. É considerada doença crônica quando essa acompanha o indivíduo por toda sua vida.
Como toda doença crônica, o tratamento é voltado para a redução dos sintomas, que afetam não apenas
o paciente, más toda a comunidade ao seu redor, períodos de controle da enfermidade são observados
no tratamento, más uma das características fundamentais é o retorno de toda a sintomatologia (recaídas)
em alguns períodos da vida do indivíduo. Apesar dos prejuízos que o indivíduo passa a apresentar pelo
uso de drogas, outra característica fundamental da dependência é o fato de o sujeito ainda assim manter
o consumo ou frequentemente a este retornar. Perde-se, parcial ou totalmente, a capacidade de controlar
o uso, a droga passa a controlar a rotina do indivíduo. A definição como “síndrome” implica uma série de
sintomas que necessitam estar todos presentes ao mesmo tempo para o diagnóstico ser realizado. Este
fato resulta em uma variedade de quadros clínicos que se apresentam aos diferentes serviços de
atendimento, garantindo as diferenças individuais entre os pacientes dependentes.
O tratamento deste transtorno psiquiátrico deve, portanto, incluir aspectos comuns a todos os
indivíduos acometidos (aspectos comuns da população de dependentes), bem como aspectos
individualizados de cada paciente.
Os resultados do tratamento mais frequentemente citados são a redução do consumo de substâncias,
a diminuição na utilização de sistemas de saúde e a menor participação em comportamentos ilícitos,
associados direta e indiretamente ao uso de drogas e álcool.
O tratamento consiste na elaboração de determinada estratégia para obtenção de seus objetivos.
Também é uma forma de intervenção para obtenção de cura ou alívio para o paciente. Estratégias e
intervenções lançam mão de modalidades e terapias: Modalidade: cada aspecto ou diversa feição das
coisas; Terapia: parte da medicina que se ocupa da escolha e administração dos meios para curar
doenças ou obter alívio do indivíduo acometido. Cada uma destas possui sua própria forma de atuação,
ou Método: conjunto dos meios dispostos convenientemente para obtenção de um fim, modo de proceder.

Abordagem do paciente dependente

Chamamos de “Abordagem” do paciente o período inicial de tratamento, os primeiros contatos entre


serviço assistencial e o paciente. Esta fase é crucial para o processo, pois uma avaliação cuidadosa e o
mais completa possível é o ponto inicial e essencial para que os indivíduos com problemas decorrentes
do consumo de drogas possam receber ajuda afetiva. O objetivo desta fase é construir, juntamente com
o paciente, o retrato detalhado e atual de seu envolvimento com o consumo, seu meio ambiente e os
resultados deste uso. Outro fator de fundamental importância nesta fase é o estabelecimento de um bom
vínculo entre a equipe terapêutica (médico, psicólogo, assistente social, enfermeiro etc.) e o paciente,
onde a confiança possa crescer gradativamente.
Dificilmente um indivíduo procura tratamento por estar convencido de que está usando droga ou álcool
demasiadamente. As principais razões para a procura de tratamento são geralmente problemas e
prejuízos que se acumulam ao longo da vida de consumo do paciente.
Dentre as principais causas de busca de serviço de assistência podemos relacionar as complicações
médicas, por exemplo, convulsões, perda de emprego, separação conjugal, imposição familiar, sentença
judicial, dívidas ou atrasos nos compromissos, depressão ou alucinações decorrentes ao consumo.
É de suma importância a investigação das expectativas do paciente quanto ao tratamento.
Expectativas irreais, exageradas, acarretam frustração e denotam, ás vezes, pouca disponibilidade
individual para mudança do estilo de vida, fator essencial para a manutenção dos resultados do processo
terapêutico.

Apostila gerada especialmente para: Betânia Mueller 023.712.580-32


. 262
A história clínica do paciente dependente deve, obrigatoriamente, conter algumas informações
essenciais, como as drogas já utilizadas e as que o indivíduo atualmente consome, utilização
concomitante de outras drogas e álcool, a via de uso atual e anteriores, quantidade consumida no último
ano, mês e semana, a dose diária comumente utilizada, o volume de gastos monetários dirigidos ao uso
e a frequência que o consumo da droga ocorre. Estes últimos indicam a dimensão da compulsão do
indivíduo para o consumo. As circunstâncias onde o consumo ocorre são fundamentais para a previsão
de possíveis fontes de recaída durante o seguimento. A utilização concomitante de drogas ou de álcool é
a regra entre pacientes que procuram o tratamento.
As informações sobre o consumo devem ser recolhidas paralelamente a história de vida do paciente,
sugerindo determinado contexto contributório para o uso de drogas. É de fundamental importância a
entrevista com um familiar do paciente, tanto para corroborar informações obtidas junto ao paciente, como
para investigar fatores familiares que possam estar contribuindo para o consumo. A entrevista com um
familiar auxilia ainda no estabelecimento de uma rede de suporte mínimo que possa auxiliar o paciente
em seus primeiros passos da abstinência.
O levantamento das consequências físicas e psíquicas do consumo deve também ser realizada na
abordagem inicial do paciente dependente. Todo o paciente deve ser avaliado, clínica e laboratorialmente,
quanto ao seu estado físico, bem como possuir uma avaliação psíquica completa pela possibilidade de
associação da dependência com os diversos quadros psiquiátricos. Sempre que possível, análises
toxicológicas devem ser obtidas, como forma de verificar a veracidade das informações obtidas na história
a respeito do consumo de drogas.
A abordagem do paciente, como instrumento inicial do tratamento, deve incluir o contrato terapêutico
assumido entre o serviço e o indivíduo, um conjunto de linhas básicas e atribuições de ambas as partes,
para que o tratamento não seja enfocado como punitivo ou controlador. O contrato deve incluir a
frequência das visitas e sua duração, o comprometimento com a abstinência total, métodos para evitar o
contato com drogas e com outros usuários (“colegas de uso”) ou traficantes e a eliminação de restos de
droga e objetos utilizados no consumo desta, por exemplo, (cachimbos, cinzeiros, isqueiros e etc.) O
paciente não deve nunca realizar sozinho esta última tarefa, pela enorme possibilidade de experimentar
“fissuras” e não resistir a “despedida”.

Quando se faz necessário internar o paciente

A internação geralmente está mais indicada em casos mais severos, por se constituir em refúgio mais
seguro para pacientes menos capazes de resistir por conta própria ás “fissuras” pelo consumo de drogas.
Deve-se enfatizar, porém, que a internação não é tratamento, más sim uma estratégia modalidade
terapêutica para a promoção da abstinência, que é apenas a parte inicial do tratamento. Para que algum
resultado seja obtido, a internação deve estar obrigatoriamente vinculada a seguimento ambulatorial e a
grupos de autoajuda.

Algumas indicações de tratamento hospitalar para dependentes de drogas:


- Paciente com ameaça de suicídio ou comportamento autodestrutivo;
- Paciente que ativamente ameaça a integridade física dos outros;
- Paciente com sintomas psiquiátricos graves como psicose, depressão, mania;
- Presença de complicações clínicas importantes;
- Necessidade de internação por dependência de outra substância, exemplo: desintoxicação do álcool;
- Falhas recorrentes na promoção da abstinência em nível ambulatorial;
- Não possuem suporte social algum, ou seja, seus relacionamentos são exclusivamente com outros
usuários.

A importância do papel da família no tratamento do paciente dependente químico

A terapia familiar é um aspecto sumamente importante na recuperação do dependente em tratamento.


Durante o tratamento sempre nos deparamos com familiares com conhecimentos insuficientes sobre
(drogas, álcool e suas implicações) para compreender a necessidade da participação no processo
terapêutico e poder lidar satisfatoriamente com o problema. Preconceitos e sofrimento acumulado pelos
familiares interferem com o processo terapêutico.
Atenção especial deve ser dirigida aos familiares, o núcleo vivencial mínimo do paciente, para
minimizar as chances de fracasso.
A família necessita discutir seus conceitos, melhorar a qualidade das relações interpessoais para criar
uma real estrutura de suporte ao paciente que auxilie em sua reabilitação.

Apostila gerada especialmente para: Betânia Mueller 023.712.580-32


. 263
Faz-se necessário que a família aprenda a não “sabotar” o processo de recuperação do paciente e
mesmo sua abstinência. Os profissionais já se acostumaram com perguntas de familiares do tipo “Mas
se ele é dependente de cocaína, por que não posso oferecer uma cervejinha para ele no churrasco, junto
com a família?”.
As famílias precisam lidar com negação, identificar outros possíveis casos, lidar com mal-entendidos,
defesas mal estruturadas, estigma e sua própria ignorância em relação ás diversas dimensões do
problema da dependência de substâncias psicoativas.
A mudança no estilo de vida, aspectos psicoeducacionais devem, portanto, ser incluídos em qualquer
tratamento dirigido a família, qualquer que seja a orientação psicológica, com o devido cuidado de
preservar o espaço de discussão do funcionamento e rotina familiar e suas relações interpessoais.
Tratamento farmacológico

A utilização de intervenções farmacológicas não apenas são estratégias válidas e disponíveis, como
muitas vezes salvam a vida de dependentes em determinadas condições.
Categorias dos agentes farmacológicos utilizados no tratamento das dependências:

- Medicamentos para quadros de intoxicação: utilidade, por exemplo, em superdosagens;


- Medicamentos para abstinência: revertem ou atenuam sintomas físicos e psíquicos imediatos á
abstinência;
- Medicamentos para controle da compulsão: reduzem o impulso para buscar e consumir álcool e
drogas;
- Medicamentos para controle de condições psiquiátricas.

Tratamento terapêutico

As principais formas de atuação dos profissionais podem ser agrupadas segundo as modalidades:

- Seguimento individual – aconselhamento


- Psicoterapia individual
- Psicoterapia em grupo
- Prevenção de recaída
- Treinamento da habilidades sociais;
- Tratamento familiar.

O objetivo final de todas estas técnicas é que o paciente desenvolva capacidade de evitar
comportamentos associados ao consumo, lidando melhor com relacionamentos e com fontes de estresse,
aumentando a autoestima e promovendo uma mudança significativa do estilo de vida do paciente.

Algumas Definições

Experimentador: pessoa que experimenta a droga, levada geralmente por curiosidade. Aquele que
prova a droga uma ou algumas vezes e em seguida perde i interesse em repetir a experiência.
Usuário Ocasional: utiliza uma ou várias drogas quando disponíveis ou em ambiente favorável, sem
rupturas (distúrbios) afetiva, social ou profissional.
Usuário Habitual: faz uso frequente, porém sem que haja ruptura afetiva, social ou profissional, nem
perda de controle.
Usuário Dependente: usa a droga de forma frequente e exagerada, com rupturas dos vínculos
afetivos e sociais. Não consegue parar quando quer.
Dependência: quando a pessoa não consegue largar a droga, porque o organismo acostumou-se com
a substância e sua ausência provoca sintomas físicos (quadro conhecido como síndrome da abstinência),
ou porque a pessoa acostumou-se a viver sob os efeitos da droga, sentindo um grande impulso de usá-
la com frequência (“fissura”).
Escalada: é quando a pessoa passa do uso de drogas consideradas “leves” para as mais “pesadas”,
ou quando, com uma mesma droga, passa de consumo ocasional para consumo intenso.
Tolerância: quando o organismo se acostuma com a droga e passa a exigir doses maiores para
conseguir os mesmos efeitos.
Poliusuário: pessoa que utiliza combinação de várias drogas simultaneamente, ou dentro de um curto
período de tempo, ainda que tenha predileção por determinada droga.

Apostila gerada especialmente para: Betânia Mueller 023.712.580-32


. 264
Overdose: dose excessiva de uma droga, com graves implicações físicas e psíquicas, podendo levar
à morte por parada respiratória ou cardíaca.

Redução de danos

No âmbito da psicoterapia das adições e toxicomanias a redução de danos, mais popularmente


conhecida pela sigla RD, pode ser considerada um paradigma, uma abordagem ou uma perspectiva, e é
utilizada para proporcionar uma reflexão ampliada sobre a possibilidade de diminuir danos relacionados
a alguma prática que cause ou possa causar danos. Valoriza e põe em ação estratégias de proteção,
cuidado e autocuidado, possibilitando mudança de atitude frente à situações de vulnerabilidade. De
acordo com Andrade a RD constitui uma estratégia de abordagem dos problemas com as drogas, que
não parte do princípio que deve haver uma imediata e obrigatória extinção do uso de drogas no âmbito
da sociedade, seja no caso de cada indivíduo, mas que formula práticas que diminuem os danos para
aqueles que usam drogas e para os grupos sociais com que convivem. O risco de suicídio, overdose e
evolução dos efeitos prejudiciais da substancia psicoativa tem que ser monitorados constantemente,
cogitando-se a internação involuntária e a desintoxicação
A RD é também uma política pública oficial do Ministério da Saúde do Brasil, e de diversos outros
países, para lidar de forma adequada com problemas que podem ser gerados pelo uso de álcool e outras
drogas. Portanto, está preconizada na Política de Atenção Integral a Usuários de Álcool e Outras Drogas
(2003), e respaldada pela PORTARIA Nº 1.059/GM DE 4 DE JULHO DE 2005, do Ministério da Saúde
que destina incentivo financeiro para o fomento de ações de redução de danos em Centros de Atenção
Psicossocial para o Álcool e outras Drogas - CAPSad - e dá outras providências.
Redução de danos não pode ser confundida com incentivo ao uso de drogas, embora fundamente-se
no princípio da tolerância ou respeito às escolhas individuais. A RD contribui, entre outras coisas, para
gerar informações adequadas sobre riscos, danos, práticas seguras, saúde, cidadania e direitos, para
que as pessoas que usam álcool e outras drogas possam tomar suas decisões, buscar atendimento de
saúde (se necessários) e estarem inseridas socialmente em um contexto de garantias de direitos e
cidadania.
A RD também atua em um nível macro político, fomentando discussões e ações no campo das leis de
drogas, sobre atenção em saúde mental e reforma psiquiátrica, sobre exclusão social e violência
estrutural, entre outros temas relacionados às políticas públicas e legislação.

- Os projetos em sua maioria são desenvolvidos marginalmente ao Sistema Único de Saúde, com
pouca integração formal com outras instâncias;
- Seu espectro de ação em nosso meio é limitado, não tendo na maioria dos lugares atingidos todos
os setores que necessitam de seu trabalho na comunidade.
Mas, por outro lado, mudanças vem ocorrendo. Por exemplo, nas capitais do país e cidades com certo
números de habitantes, são implementados os CAPS-AD (Centro de Atenção Psicossocial Álcool e
Drogas).
As Equipes são formadas por profissionais Multidisciplinares (entre eles Psiquiatras, Psicólogos,
Assistentes Sociais, enfermeiros, Terapeutas Ocupacionas, e Redutores de Danos).
Em muitas cidades o SUS implementa o tratamento para usuários de Drogas, e entre as capitais, cita-
se Recife, Salvador e São Paulo como modelos de gestão.
Projetos como Consultórios nas Ruas também são executados em convênio com o SUS.

Associação Brasileira de Redutoras e Redutores de Danos – ABORDA

A ABORDA foi criada em 1997, durante o 2º Congresso Brasileiro de Prevenção da Aids, em Brasília.
Os objetivos, daquela época até os dias de hoje, não mudaram muito: a implementação e o fortalecimento
da Redução de Danos como política pública, e a defesa da dignidade do redutor de danos.
Nos últimos 10 anos, houve grandes transformações nas perspectivas de Redução de Danos. Inscritos
no cotidiano do SUS a partir da necessidade de enfrentamento das formas de transmissão do HIV ligadas
ao compartilhamento de equipamentos de uso de drogas injetáveis, os redutores de danos contribuíram
significativamente na luta contra a Aids, e abriram um leque de novas possibilidades na forma de se fazer
e pensar saúde no Brasil. A Redução de Danos, hoje, constitui-se em um conjunto de políticas públicas
ligadas ao enfrentamento dos eventuais problemas relacionados ao uso de drogas, articulando distintas
realidades: prevenção ao HIV/Aids e hepatites, promoção integral de saúde às pessoas que usam drogas
e diminuição da violência. Tal articulação consiste no apoio/incentivo ao protagonismo das pessoas que
usam drogas, na busca pelo cuidado de si e manejo do seu uso de drogas.

Apostila gerada especialmente para: Betânia Mueller 023.712.580-32


. 265
Hoje, a ABORDA (o.c.) está presente em quase todos os estados brasileiros, do Rio Grande do Sul ao
Amapá, contando com a força de centenas pessoas ligadas a dezenas de programas governamentais e
não governamentais de Redução de Danos, e constituindo-se na grande roda de articulação política em
defesa da Redução de Danos e dos Direitos Humanos das pessoas que usam drogas.
A ABORDA (o.c.) busca destacar a importância de redutoras e redutores de danos, defendendo a
melhoria de suas condições de vida e trabalho, e contribuindo para sua organização e capacitação
técnica. Além disto, defendemos a ruptura com paradigmas ora instituídos no que tange aos discursos
sobre drogas e pessoas que as usam através da articulação com movimentos sociais, universidade e
Estado, contribuindo para a construção de políticas públicas diferenciadas, abertas à participação cidadã
de populações sobre as quais recaem os efeitos de dispositivos que muitas vezes impedem seu acesso
aos mais básicos direitos.
No desenvolvimento de suas atividades, a ABORDA (o.c.) observa os princípios da legalidade,
impessoalidade, moralidade, publicidade, economicidade e da eficiência, colaborando com todos os
movimentos sociais que buscam o fim de todas as formas de preconceito e discriminação, incentivando
o protagonismo social e a democracia participativa em defesa dos princípios e diretrizes do SUS.

Rede Brasileira de Redução de Danos – REDUC


A Reduc foi criada em outubro de 1998, na cidade de São Paulo, no Encontro Nacional de Redução
de Danos, uma parceria entre o Programa de Orientação e Assistência a Dependentes -
PROAD/UNIFESP, o IEPAS - Instituto de Estudos e Pesquisas em Aids de Santos, o Programa Estadual
DST/AIDS-SP, o Laboratório Biocintética e com o apoio da Coordenação Nacional de DST/AIDS e da
USAID - Agência Norte-Americana para o Desenvolvimento Internacional.

Principais propostas da Reduc

Discutir as temáticas sobre drogas, ampliando a discussão numa perspectiva cientifica e do ponto de
vista do impacto social da estratégias de redução de danos com os diversos segmentos sociais- governos,
sociedade civil organizada, universidades, profissionais de saúde, educação, mídia, iniciativa privada e
todas as áreas de interface com o fenômeno das drogas.
- Planejar novas parcerias com organizações do Brasil e do exterior para otimizar e aprimorar as
práticas de redução de danos e Direitos Humanos das pessoas que usam álcool e outras drogas e sua
rede de interação social, visando garantia de direitos.
- Elaborar propostas que visem novas políticas para as drogas no Brasil, em conjunto com todas as
esferas de interface para o fenômeno de drogas, buscando a defesa dos Direitos das pessoas que usam
álcool e outras drogas e sua rede de interação social
- Articular redes permanentes de comunicação entre as várias iniciativas já existentes, com o
objetivo de ampliar as discussões para outras instancias, ações que discutam as questões das drogas
licitas e ilícitas, assegurando qualidade de vida e defesa de direitos das pessoas que usam álcool e outras
drogas e sua rede de interação social
- Fiscalizar o cumprimento de normas constitucionais.
- Apoiar iniciativas de políticas públicas aos usuários de drogas lícitas e ilícitas.

Objetivos da Redução de Danos

No Brasil, Grã-Bretanha e em vários países do mundo, a Redução de Danos (RD) é uma política de
saúde pública e um conjunto de estratégias práticas, voltados para o campo do uso de drogas, que visam
reduzir danos desse uso, sejam biológicos, sociais, econômicos ou culturais, sem necessariamente
interrompê-lo, levando em consideração a liberdade de escolha das pessoas e a liberdade de uso do seu
próprio corpo. A Redução de Danos também pode ser compreendida como um movimento social e
político, que tem como objetivo contribuir para transformação da visão de mundo e a postura da sociedade
diante do problema das drogas, possibilitando diálogo na sociedade e expressão das pessoas que usam
drogas, sobre os usos, necessidades, desejos, direitos e deveres. Observe-se, porém, que esse modelo
ou princípio de intervenção pode ser aplicado a outras áreas da saúde pública e promoção da saúde que
não os problemas associados e decorrentes da drogadição.

Exemplos práticos de Redução de Danos


- No trânsito: não dirigir veículos automotores se usar bebidas alcoólicas, para evitar acidentes
automotivos;

Apostila gerada especialmente para: Betânia Mueller 023.712.580-32


. 266
- No uso de substâncias: não compartilhar seringas para o uso de drogas injetáveis, para não se
infectar ou infectar outras pessoas com doenças de transmissão sanguínea; reduzir a frequência de uso
de alguma droga para diminuir os danos que ela possa causar; não levar bebidas em garrafas de vidro
para eventos e espaços públicos, evitando, assim, que acidentes com vidro possam ferir as pessoas;
propor o uso de medicamentos que controlem a abstinência ou a substituição de uma droga com efeito
mais devastador por uma que tenha um potencial de dependência menor, reduzindo aos poucos esse
consumo até que o mesmo se cesse ;
- No campo da violência: conscientizar os homens sobre a necessidade de não naturalizar a violência
contra a mulher como sendo própria da “natureza masculina”, para evitar esse tipo de violência;
- Redução de danos ambientais: realizar coleta seletiva de lixo para diminuir danos ao ecossistema.
Os grupos populacionais alvo de projetos de intervenção no campo da Saúde Pública, em especial
nas ações de prevenção desenvolvidas no âmbito dos projetos e programas de prevenção as DST/AIDS,
são considerados segundo sua situação de risco e vulnerabilidade, sendo o conceito de vulnerabilidade
entendido como a pouca ou nenhuma capacidade do indivíduo ou grupo social de decidir sobre sua
situação de risco.
A vulnerabilidade está diretamente associada aos fatores culturais, sociais, políticos, econômicos e
biológicos. Assim, o grau de vulnerabilidade de uma pessoa ou um grupo à infecção pelo HIV ou ao
desenvolvimento da doença, segundo a proposição de Jonathan Mann, é avaliado a partir de três níveis,
inter-relacionados. O primeiro nível é o da vulnerabilidade individual, que está relacionado com os fatores
biológicos, comportamentais, psicológicos e cognitivos que interferem num maior ou menor poder para a
adoção de medidas preventivas. Num segundo nível, temos a vulnerabilidade social, que é constituída
pela rede de interações sociais e pelos determinantes demográficos que protegem ou expõem os grupos
e os indivíduos à infecção pelo HIV ou a desenvolver aids. Por fim, a vulnerabilidade programática, que
diz respeito aos bens, insumos e serviços disponibilizados para as populações por meio de políticas
públicas de saúde, educação e justiça.
Existe em nosso país uma falta evidente de recursos para o atendimento dos usuários de drogas,
particularmente endovenosas, assim como para outros indivíduos que sofrem pelo preconceito causado
pelo seu uso de drogas em relação à sociedade. Esta falta de aceitação, e consequente dificuldade de
acesso aos meios de saúde, é em parte gerada por um descrédito por parte dos técnicos (e também dos
próprios pacientes) na sua capacidade de adesão a qualquer tipo de intervenção da saúde. É então de
fato impressionante que, mesmo assim, alguns pacientes ainda consigam ultrapassar uma grande
sequência de barreiras e iniciar um processo de busca de auxílio e tratamento, mesmo quando as chances
lhes são absolutamente desfavoráveis.
O trabalho comunitário é um momento privilegiado para a promoção da saúde, para prevenção dos
riscos e dos danos relacionados ao uso e ao ambiente nos quais substâncias psicoativas são utilizadas.
Mais do que nunca, é estratégico termos abordagem honesta e verdadeira para com a comunidade e a
articulação de uma rede de projetos com objetivos afins, permite que instâncias de governo e da
sociedade civil organizada, compartilhem as responsabilidades pelas ações desenvolvidas em suas
regiões, legitimando e otimizando os recursos de cada, um dos projetos isoladamente, contribuindo para
o aperfeiçoamento de tecnologias para a assistência, capacitação e prevenção.
O conceito de vulnerabilidade, no entanto, não pressupõe o abandono da noção de risco. No caso da
articulação entre drogas e aids, temos que ter clareza sobre os riscos reais de infecção pelo HIV a que
os usuários de drogas, principalmente injetáveis, estão submetidos. Se, num primeiro momento da
epidemia, o conceito de risco acabou por estigmatizar homossexuais e usuários de drogas em categorias
("grupos de risco") que só faziam ratificar o preconceito social sobre as formas de prazer e satisfação
sexual que eram condenadas pela moral dominante, dificultando-lhes o acesso ao sistema de saúde,
hoje, a abordagem “transdisciplinar” do risco pode trazer novas luzes sobre a questão da prevenção da
aids e do uso de drogas.
É a partir desta perspectiva, que somos levados a priorizar as estratégias de "redução da demanda" e
"redução de danos":
- A redução da demanda, promovendo a prevenção das DST/AIDS e do uso indevido de drogas e a
articulação do binômio drogas e aids entre populações vulneráveis ao consumo de drogas, e
- A redução de danos à saúde pelo uso de drogas, priorizando o usuário de drogas injetáveis para que,
no período de vida no qual não pode ou não quer parar de usar drogas, não se infecte com o HIV.
As drogas mais usadas em nossa sociedade são justamente aquelas que se comercializam livremente,
como as bebidas alcoólicas, o cigarro e os medicamentos como os calmantes e as fórmulas para
emagrecimento. Seus usos são intensamente estimulados e estão intimamente associados aos maiores
problemas de saúde pública em nosso país. Apesar disso, a grande maioria das pessoas não as

Apostila gerada especialmente para: Betânia Mueller 023.712.580-32


. 267
reconhece como tal e não estão preparadas para lidar com a prevenção dos riscos e dos danos ligados
ao uso abusivo dessas substâncias.
Isso vem ao encontro do que já sabemos: uma pessoa alcoolizada ou sob efeito de outras drogas, tem
a capacidade de julgamento e reflexos alterados. E, que, nestas circunstâncias, pode se abster de
cuidados em relação ao sexo seguro, isto é não usar preservativo nas relações sexuais. E sabemos como
isso é comum. Em casos como esses qual a prevenção possível? O que devemos prevenir primeiro?
Inicialmente, podemos nos orientar sobre a prioridade frente a esta situação. Vejamos: uma pessoa
alterada pelo uso de alguma droga e que se encontre em situação favorável a ter uma relação sexual. O
que podemos fazer efetivamente? Certamente não será dando uma aula sobre os malefícios das drogas.
O que nos resta é tentar convencê-la a usar preservativo e, se possível, disponibilizá-lo ao par, com um
forte apelo para que o utilizem.
Neste caso identificamos dois riscos. Um mais imediato que se refere à possível infecção por uma
DST, caso se consume o sexo desprotegido. E se uma das pessoas estiver infectada pelo HIV?
A outra situação está relacionada ao uso indevido de uma droga. Neste caso, a pessoa poderá ser
abordada em momento mais oportuno, em que se tenha a oportunidade de esclarecer as motivações, as
percepções sobre o uso da droga e as reais condições para o tratamento, se for o caso. Neste contexto,
podemos valer-nos do próprio exemplo do risco do sexo desprotegido para apoiar a nossa intervenção.

Questões

01. (SESAU/RO - Psicólogo - FUNRIO/2017). Sobre a dependência química, assinale a afirmativa


INCORRETA:
(A) é caracterizada por abuso da substância e consequente desenvolvimento de tolerância.
(B) é marcada por abstinência quando da retirada da droga.
(C) dados de prevalência sugerem que o alcoolismo pode afetar até 10% da população.
(D) a perspectiva de redução de danos sugere que as consequências da dependência devem ser
minoradas, sem sugerir a necessidade de abstinência.
(E) fatores contextuais e ambientais não interferem no estabelecimento da dependência química.

02. (SUSA - Psicólogo – FGV). Em relação à Política de Redução de Danos, assinale a afirmativa
correta.
(A) A política de redução de danos tem, como objetivo principal, diminuir o consumo de álcool e drogas.
(B) O Brasil não tem uma política oficial de redução de danos.
(C) A grande preocupação das políticas de redução de danos é a formulação de práticas que diminuam
os danos para os familiares dos usuários de drogas.
(D) A política de redução de danos não inclui medidas em relação a pacientes aidéticos.
(E) O incentivo ao comportamento de não dirigir veículos, no caso do uso de bebidas alcoólicas, está
incluído na política de redução de danos.

03. (MPE/GO - Psicólogo - FUNIVERSA). A dependência química constitui hoje problema de saúde
experienciado por um contingente significativo da população. Considerando esse contexto, os municípios
precisam organizar-se para atender a pessoas acometidas por essa patologia, indo ao encontro dos
pressupostos da Reforma Psiquiátrica Brasileira.
Uma das drogas de ampla ingestão é o álcool. O consumo desse é um dos hábitos sociais mais antigos
e difundidos entre as populações, já que está correlacionado a ritos religiosos, valores sociais e culturais,
além de lhe serem conferidos efeitos como calmante, afrodisíaco, estimulante de apetite, desinibidor,
entre outros. Além do mais, "a cultura pode influenciar o padrão e o contexto, assim como a quantidade
do consumo de álcool e o padrão desse consumo pode, por sua vez, ser um determinante importante dos
problemas com bebida".
Cerca de 15% das pessoas que consomem álcool progridem para o alcoolismo. Esse é um dado
realmente preocupante, já que tal patologia pode prejudicar tanto a qualidade de vida dos sujeitos
alcoolistas, como do seu meio familiar e social.

A respeito do assunto abordado no texto IV, assinale a alternativa que apresenta o encaminhamento
a ser dado por um psicólogo, caso constate a dependência química em um paciente dele.
(A) Fazer o acompanhamento psicoterápico por algum tempo e, depois, se necessário, encaminhar o
paciente para a manutenção, cujo objetivo é reorganizar a vida do paciente.
(B) Entrar em contato com a família e sugerir a interdição temporária do dependente, bem como
encaminhá-lo para uma clínica de desintoxicação, visando à reorganização de seus hábitos.

Apostila gerada especialmente para: Betânia Mueller 023.712.580-32


. 268
(C) Encaminhar o paciente para a desintoxicação, que visa à retirada das drogas, e, em seguida, para
a manutenção, que tem por objetivo reorganizar a vida do paciente sem o uso delas.
(D) Sugerir ao paciente que se afaste de suas atividades profissionais visando deixar a dependência.
(E) Encaminhar o paciente para uma terapia comunitária e recomendar terapia familiar ao restante do
núcleo familiar do paciente.

04. (DETRAN/PE - Psicólogo - FUNCAB). Considerando o distúrbio caracterizado como alcoolismo,


tolerância e dependência ao álcool são dois eventos distintos e indissociáveis. Desta forma, podemos
dizer que:
(A) dependência é a necessidade de doses maiores de álcool para a manutenção da tolerância e
consequente aumento do efeito de embriaguez.
(B) a dependência geralmente é tanto mais intensa quanto menor for o grau de tolerância ao álcool.
(C) a dependência não é necessariamente simultânea à tolerância, sendo a primeira geralmente
posterior à segunda.
(D) efeito da bebida, mas a ausência ou presença de embriaguez.
(E) tolerância é a necessidade de doses maiores de álcool para a manutenção do efeito de embriaguez
obtido nas primeiras doses.

Gabarito

01.E / 02.E / 03.C / 04.E

Comentários

01. Resposta: E
A vulnerabilidade às drogas está diretamente associada aos fatores culturais, sociais, políticos,
econômicos e biológicos.

02. Resposta: E
No trânsito podemos citar como exemplo de redução de danos não dirigir veículos automotores se usar
bebidas alcoólicas, para evitar acidentes automotivos.

02. Resposta: C
Com relação ao atendimento aos dependentes químicos, cabe destacar que apresenta duas fases: a
desintoxicação, que visa a retirada das drogas, e a manutenção, que tem por objetivo reorganizar a vida
do sujeito sem o uso delas. Cabe assinalar que, quando o paciente procura voluntariamente o tratamento
e tem participação ativa na definição de programas terapêuticos, é maior a possibilidade de sucesso da
intervenção. Na maior parte dos serviços ambulatoriais, um dos critérios para o ingresso nesses espaços
é de que o usuário venha desintoxicado, portanto, preconizando somente a segunda fase do tratamento
aos dependentes químicos, a manutenção.31

03. Resposta: E
Na tolerância, o organismo se acostuma com a droga e passa a exigir doses maiores para conseguir
os mesmos efeitos.

Sexualidade, gênero e identidade.

A sexualidade tem grande importância no desenvolvimento e na vida psíquica das pessoas, pois
independentemente da potencialidade reprodutiva, relaciona-se com a busca do prazer, necessidade
fundamental dos seres humanos. Nesse sentido, a sexualidade é entendida como algo inerente, que se
manifesta desde o momento do nascimento até a morte, de formas diferentes a cada etapa do
desenvolvimento. Além disso, sendo a sexualidade construída ao longo da vida, encontra-se
necessariamente marcada pela história, cultura, ciência, assim como pelos afetos e sentimentos,
expressando-se então com singularidade em cada sujeito. Indissociavelmente ligado a valores, o estudo

31
http://www.revistasusp.sibi.usp.br/scielo.php?pid=S1806-69762007000200005&script=sci_arttext

Apostila gerada especialmente para: Betânia Mueller 023.712.580-32


. 269
da sexualidade reúne contribuições de diversas áreas, como Antropologia, História, Economia,
Sociologia, Biologia, Medicina, Psicologia e outras mais. Se, por um lado, sexo é expressão biológica
que define um conjunto de características anatômicas e funcionais (genitais e extragenitais), a
sexualidade é, de forma bem mais ampla, expressão cultural. Cada sociedade cria conjuntos de
regras que constituem parâmetros fundamentais para o comportamento sexual de cada indivíduo.
Nesse sentido, a proposta de Orientação Sexual considera a sexualidade nas suas dimensões
biológica, psíquica e sociocultural.

Sexualidade na infância e na adolescência32

Os contatos de uma mãe com seu filho despertam nele as primeiras vivências de prazer. Essas
primeiras experiências sensuais de vida e de prazer não são essencialmente biológicas, mas constituirão
o acervo psíquico do indivíduo, serão o embrião da vida mental no bebê. A sexualidade infantil se
desenvolve desde os primeiros dias de vida e segue se manifestando de forma diferente em cada
momento da infância. A sua vivência saudável é fundamental na medida em que é um dos aspectos
essenciais de desenvolvimento global dos seres humanos.
A sexualidade, assim como a inteligência, será construída a partir das possibilidades individuais e de
sua interação com o meio e a cultura. Os adultos reagem, de uma forma ou de outra, aos primeiros
movimentos exploratórios que a criança faz em seu corpo e aos jogos sexuais com outras crianças. As
crianças recebem então, desde muito cedo, uma qualificação ou “julgamento” do mundo adulto em que
está imersa, permeado de valores e crenças que são atribuídos à sua busca de prazer, o que comporá a
sua vida psíquica.
Nessa exploração do próprio corpo, na observação do corpo de outros, e a partir das relações
familiares é que a criança se descobre num corpo sexuado de menino ou menina. Preocupa-se então
mais intensamente com as diferenças entre os sexos, não só as anatômicas, mas também com todas as
expressões que caracterizam o homem e a mulher. A construção do que é pertencer a um ou outro sexo
se dá pelo tratamento diferenciado para meninos e meninas, inclusive nas expressões diretamente
ligadas à sexualidade e pelos padrões socialmente estabelecidos de feminino e masculino. Esses padrões
são oriundos das representações sociais e culturais construídas a partir das diferenças biológicas dos
sexos e transmitidas pela educação, o que atualmente recebe a denominação de relações de gênero.
Essas representações absorvidas são referências fundamentais para a constituição da identidade da
criança.

As formulações conceituais sobre sexualidade infantil datam do começo deste século e ainda hoje não
são conhecidas ou aceitas por parte dos profissionais que se ocupam de crianças, inclusive educadores.
Para alguns, as crianças são seres “puros” e “inocentes” que não têm sexualidade a expressar, e as
manifestações da sexualidade infantil possuem a conotação de algo feio, sujo, pecaminoso, cuja
existência se deve à má influência de adultos. Entre outros educadores, no entanto, já se encontram
bastante difundidas as noções da existência e da importância da sexualidade para o desenvolvimento de
crianças e jovens.
Em relação à puberdade, as mudanças físicas incluem alterações hormonais que, muitas vezes,
provocam estados de excitação incontroláveis, ocorre intensificação da atividade masturbatória e instala-
se a função genital. É a fase das descobertas e experimentações em relação à atração e às fantasias
sexuais. A experimentação dos vínculos tem relação com a rapidez e a intensidade da formação e da
separação de pares amorosos entre os adolescentes.

É uma questão bastante atual e presente no cotidiano de todos os profissionais da educação a postura
a ser adotada, dentro das escolas, em face das manifestações da sexualidade dos alunos.

Como dito anteriormente, sexo também é coisa de criança33. Tendo sempre em mente que cada
criança é uma criança, vamos pensar o desenvolvimento sexual da criança.
Tomando por base os modos de viver e expressar a dimensão humana, temos seis períodos distintos
– primeira infância, fase pré-escolar, segunda infância, adolescência, maturidade e terceira idade. Aqui
vamos nos ater apenas aos três primeiros: primeira infância (0 a 2 anos), fase pré-escolar (2 a 6 anos) e
segunda infância (6 a 10 anos).

32
BRASIL. Secretaria de Educação Fundamental. Parâmetros Curriculares Nacionais: Orientação Sexual. Portal MEC.
33
Colunista Portal Educação, 2013 em http://www.portaleducacao.com.br.

Apostila gerada especialmente para: Betânia Mueller 023.712.580-32


. 270
- Primeira infância (0 a 2 anos):

“A educação sexual começa a partir das atitudes dos pais, no momento em que decidem ter filhos”.
As primeiras atitudes dos pais podem proporcionar ou um ambiente afetivo e amoroso, ou um ambiente
ríspido e tumultuado. Esse ambiente será a primeira influência no desenvolvimento da criança. É “nos
primeiros anos de vida que se estabelecem as bases do comportamento erótico do adulto e se inicia a
formação de uma sexualidade saudável”.
Neste período (0 a 2 anos) a criança começa a explorar seu mundo através de seu corpo, de suas
sensações. Será através do gosto, do cheiro, do toque, do olhar e do ouvir que a criança vai experimentar
o prazer. Essa relação com seu corpo e com os sentidos formará suas atitudes sexuais mais tarde.
A relação que essa criança tem com seus cuidadores também será definidor das suas atitudes
relacionais. Esse primeiro vínculo é um primeiro passo. Ele será fortalecido, ou não, no seu
desenvolvimento.
É nessa fase que começamos a amar e sermos amados. A nossa capacidade de amar e de se
relacionar está diretamente ligada a esse aprendizado na infância.

- Fase pré-escolar (2 a 6 anos):

Essa fase tem quatro momentos importantes:

1. Formação da Identidade de gênero:

A identidade de gênero é a condição de pertencer a um sexo. Nesta fase a criança começa a definir-
se como menino ou menina. Os pais e educadores(as) devem, neste momento, favorecer o processo de
identificação da criança, através da brincadeira. Mostrar as diferenças e semelhanças entre ser menino
e ser menina (evitar ao máximo estereótipos!). Reforçar a visão de sexo da criança, sem nunca
desvalorizar o sexo oposto. A questão não é superioridade/inferioridade, mas sim diferenças.

2. Assimilação do papel sexual (social):

O papel sexual diz respeito ao comportamento que a criança terá diante sua identidade de gênero.
Importante evitar a manutenção de preconceitos de comportamentos tipicamente masculinos e/ou
femininos.

3. Aprendizagem e controle dos esfíncteres

É a primeira oportunidade da criança de aprender e exercer o autocontrole, através do treinamento do


controle dos esfíncteres.
Segundo as considerações de Figueirêdo Netto, a aprendizagem do controle dos esfíncteres, no que
se refere ao desenvolvimento da sexualidade, tem fundamental importância, pois:
a) “As áreas genitais se encontram na mesma zona do corpo que intervém na excreção. Os músculos
que participam deste ato são exatamente os mesmos que posteriormente atuarão na resposta sexual.
b) O ato de reter e expulsar os excrementos (urina e fezes) produz prazer sensual, pela tensão e alívio
ou relaxamento, que acompanham estes comportamentos.
c) O controle voluntário desses músculos, assim como as sensações prazerosas deles resultantes,
são associados à sexualidade”.
Para não adiantar nem atrasar esse processo da criança é preciso ter em mente que ele(a) poderá ter
este tipo de controle entre os dois e três anos de idade. Adiantar ou atrasar esse momento pode ser
prejudicial ao desenvolvimento da criança. Importante, ainda, salientar que pais e educadores devem
evitar relacionar questões negativas (como sujo, feio, associar a castigos e chantagens), no decorrer do
treinamento do controle dos esfíncteres.

4. Interesses e curiosidades sexuais:

É a conhecida fase dos porquês. Além das perguntas, as crianças querem ver e saber. Com tantas
perguntas, é um bom momento para ensinar às crianças os nomes corretos das partes de seu corpo.
Como parte de seu desenvolvimento a masturbação aparece como curiosidade natural da criança de
seu corpo e suas sensações. É um jogo exploratório de sensações. Não tem a mesma conotação da
masturbação na adolescência e no adulto. Assim, é um bom momento para ensinar às crianças sobre a

Apostila gerada especialmente para: Betânia Mueller 023.712.580-32


. 271
intimidade. O público e o privado. Não precisa problematizar a situação, apenas orientar. A repressão é
indesejada.
Além de se tocarem, as crianças exploram também os outros. É a fase da conhecida “brincadeira de
médico”. Se a brincadeira for entre crianças da mesma idade não há razão para se preocupar, é
conhecimento não abuso.
Nessa fase o pensamento é mágico e fantasioso, por isso devem ser evitadas conversas como a da
“cegonha” e da “sementinha”. As respostas devem ser claras e objetivas o suficiente para satisfazer a
curiosidade da criança. Ela quer saber do fato, a maldade está na cabeça do(a) adulto(a). Outro cuidado
com as histórias fantasiosas é que elas podem gerar fantasias negativas, temores e culpas.
Desnecessário.

- Segunda Infância (6 a 10 anos):

Período no qual a sexualidade entra em latência. Ou seja, entra em adormecimento para ser mais bem
elaborada. É um momento de sensualidade, pois as crianças estão aptas a experimentar as sensações.
Por isso, há muitos jogos sexuais nesta fase. O lúdico aparece na imitação de modelos. É um momento
em que pais e educadores(as) devem tomar cuidado com o que falam e com o que fazem. A criança está
em constante observação. Assim, é um bom momento para transmitir informações e valores (confiança,
respeito, amor, honestidade, responsabilidade), as crianças estão prestando atenção.
É nesse período que se fortalece a identidade de gênero e prepara a criança para o próximo período,
a puberdade.

O que são jogos sexuais?

Definição: são brincadeiras que ajudam a satisfazer a curiosidade sexual.

Alguns tipos:
- Cócegas;
- Pegar nos próprios genitais e nos dos / das coleguinhas;
- Brincadeiras de médico;
- Brincadeiras de papai e mamãe.

Atenção: essas brincadeiras devem ser feitas com crianças da mesma idade.
Ainda sobre os jogos sexuais, Suplicy afirma que “os professores constataram que em geral os jogos
sexuais são realizados na hora do recreio. As crianças escolherem um lugar protegido, fora da vista do
adulto; não tiram a roupa e brincam de médico e de papai-e-mamãe. Se esses jogos forem observados,
mas não atrapalharem nenhuma atividade, não precisam ser interrompidos, pois fazem parte do
desenvolvimento sexual da criança. O professor só deve estar atento para que não haja coação nessas
brincadeiras”.

Os jovens e a sexualidade34

Para realizar uma prática adequada de Orientação Sexual com jovens, é necessário que o profissional
conheça o público beneficiário de sua ação, ou seja, de quem e com quem falamos na condição de
educadores.

Segundo o Estatuto da Criança e do Adolescente – ECA (Lei 8.069, de 13 de julho de 1.990 – Art. 2º)
“considera-se criança, [...], a pessoa até doze anos de idade incompletos, e adolescente aquela entre
doze e dezoito anos de idade” (Brasil, 1990).

Muitos autores que se preocupam com a temática da infância e juventude afirmam que não é possível
definir o período que compreende a infância e a adolescência apenas pela faixa etária. Quando podemos
afirmar que uma criança deixou de sê-lo e passou a ser adolescente? Quais comportamentos são
considerados infantis, juvenis e/ou adultos? Estes são questionamentos complexos.
Em todos os questionamentos que formulamos a respeito dos seres humanos, devemos sempre
conceber o homem enquanto ser integral, biopsicossocial. Desta forma, precisamos considerar as

34
BRANCO, M. A. O.; PINTO, M. J. C.; VIANNA, a. M. S. A. Orientação Sexual com Jovens: Construindo um Exercício Responsável da Sexualidade. Simpósio
Internacional de Educação Sexual da UEM, 2009.

Apostila gerada especialmente para: Betânia Mueller 023.712.580-32


. 272
dimensões biológica, psicológica e social das pessoas, compreendendo que estas não são separadas,
mas integradas na existência humana.
Em relação à dimensão biológica, percebemos que uma criança começa a deixar de sê-lo quando ela
vivencia o período do desenvolvimento humano chamado de puberdade. Para esta discussão, tomaremos
como referência o trabalho de Gewandsznajder.
Na puberdade, o corpo do menino ou da menina passa por um processo de transformação, deixando
de ser um corpo infantil para se tornar um corpo adulto, ou seja, pronto para reprodução.
A faixa etária que corresponde a este período é variável. Em geral, a puberdade ocorre nos garotos
entre 11 e 13 anos e nas garotas entre 10 e 12 anos. É necessário saber que estas idades não são fixas,
podendo variar de pessoa para pessoa.
Tanto em garotos quanto em garotas ocorre o chamado “estirão”, ou seja, um crescimento do corpo
acentuado em um curto período de tempo. O “estirão” costuma iniciar mais cedo nas meninas que nos
meninos, razão pela qual as meninas por volta dos 12 anos de idade são frequentemente mais altas que
os meninos. Também tanto em garotos quanto em garotas ocorre o aparecimento de pêlos pubianos e
axilares. A pele se torna mais oleosa e o corpo, através do suor, passa a ter um cheiro característico de
pessoa adulta, diferenciando-se da criança.

Nos garotos ocorre o aparecimento da barba, e a laringe se alarga provocando a tendência da voz se
tornar mais grave. Também ocorre o aumento da massa muscular, com consequente ampliação da força
física, e o aumento do pênis e testículos.
Nas garotas ocorre o aumento dos seios, quadris, nádegas e coxas, dando ao corpo o aspecto de
mulher em fase adulta. A partir da puberdade a garota passa a menstruar, característica que sinaliza que
seu organismo está pronto para gerar filhos.
É preciso deixar claro que puberdade não é sinônimo de adolescência. Puberdade compreende as
transformações corporais que tornam o corpo humano adequado para a reprodução, deixando de ser um
corpo infantil para tornar-se um corpo adulto. A adolescência compreende um período mais extenso e
significativo que a puberdade, sendo esta etapa constituinte daquela.
O termo adolescência vem do termo latino adolescere, que significa “crescer, engrossar, tornar maior”.
Em relação à dimensão psicológica, segundo Canosa Gonçalves et. al. e Tavares, as crianças que se
tornam adolescentes também passam por transformações. A principal delas é em relação à própria
identidade. Neste momento, o adolescente necessita se reconhecer num corpo transformado, que não é
mais o corpo infantil que ele tinha, e que agora é um corpo adulto, visivelmente modificado.
Outro passo importante é a consolidação de si próprio enquanto pessoa “independente”, sob o ponto
de vista da determinação de suas escolhas pessoais e da responsabilidade que elas trazem. É neste
momento que pode haver uma divergência, e até um questionamento, com as regras determinadas pela
família e pela sociedade.
Na adolescência é comum ocorrer uma identificação muito intensa do jovem com seu grupo de “iguais”,
em geral outros jovens. Não é raro este grupo (galera, turma, etc.) compartilhar um determinado modo de
conversar, de se vestir, enfim, de se comportar. Esta identificação com o grupo é importante na construção
da própria identidade (pessoal, sexual, social) do adolescente.
Em geral, nesta fase do desenvolvimento ocorrem as primeiras manifestações da sexualidade adulta,
ou seja, o primeiro beijo, o “ficar”, o namoro, as primeiras experiências eróticas. Trata-se de uma busca
pelo outro para um relacionamento afetivo-sexual. “A adolescência é uma fase de descobertas, de
desafios e a sexualidade humana talvez seja, para a maioria dos jovens, o aspecto mais interessante
desta jornada”.
Em relação à dimensão social, precisamos considerar que a adolescência enquanto processo de
desenvolvimento humano não é universal, ou seja, não é igual para todos os jovens. Cada um vivenciará
a sua adolescência de acordo com suas condições de vida, o seu lugar de moradia, a dinâmica de sua
família de origem, as características de acesso à escola ou aos serviços de saúde, as modalidades de
lazer a que tem acesso, dentre outros condicionantes. Todas as transformações vivenciadas pelo jovem
são construídas mediante as relações sociais que eles estabelecem. Não existe um “padrão”. Cada
indivíduo, a partir de sua realidade social, vivenciará sua juventude de forma particular.
Não devemos pensar a juventude como crise, mas como um processo do ciclo vital do jovem. Isto quer
dizer que devemos compreender o jovem não enquanto um “problema” ou um “fardo”. Deve ser
compreendido sempre a partir da sua pessoa em condição peculiar de desenvolvimento inserida num
determinado contexto sociocultural.
Outro fator importante a ser abordado é o prolongamento da juventude. Atualmente vivenciamos uma
clara dificuldade em delimitar o término deste período. Não é raro encontrarmos pessoas que pretendem

Apostila gerada especialmente para: Betânia Mueller 023.712.580-32


. 273
terminar seus estudos, incluindo até cursos de mestrado e doutorado, antes de decidirem morar sozinhos
ou casaram-se, e então deixar de morar com seus pais.
Partindo da premissa de todas estas transformações contemporâneas, é interessante tomarmos a
definição do Conselho Nacional da Juventude no que diz respeito a estender até os 29 anos a faixa etária
das pessoas que são consideradas jovens.
São estes jovens que constituem o público beneficiário da prática de Orientação Sexual, no enfoque
deste trabalho.

Orientação Sexual X Educação Sexual

Os autores que se preocupam atualmente com a temática da Orientação Sexual formulam


questionamentos a respeito do termo que deve ser utilizado para definir tais práticas. Quando falamos
em Orientação Sexual e em Educação Sexual, utilizamos a mesma definição para as duas expressões?
De acordo com Ribeiro falamos em Educação Sexual quando nos referimos aos “processos culturais
contínuos [...] que direcionam os indivíduos para diferentes atitudes e comportamentos ligados à
manifestação de sua sexualidade”. Nesta definição, podemos pensar que a educação sexual tem seu
início no nascimento de cada indivíduo, sendo que o processo educacional acontece através da relação
deste indivíduo com seu meio social. Então, as “atitudes e comportamentos ligados à manifestação da
sexualidade” são construídos por cada pessoa em contato com a sociedade, ou seja, amigos, grupos
religiosos e/ou de convivência, meios de comunicação e, principalmente, a família. Portanto, a sociedade
pratica ações educativas em sexualidade em relação aos indivíduos que a constituem. Porém, em grande
parte das vezes, estas ações se tornam “deseducativas”, na medida em que reproduzem e perpetuam
tabus, desinformações e atitudes repressivas em relação à sexualidade humana.
Para Ribeiro, a Orientação Sexual pressupõe uma intervenção institucionalizada, sistematizada e
realizada por profissionais especialmente preparados para exercer esta função. Diferencia-se, portanto,
da Educação Sexual, que acontece durante toda a vida das pessoas, e que diz respeito ao processo
educacional referente às atitudes em relação à sexualidade. Desta forma, podemos pensar a Orientação
Sexual enquanto prática interventiva na vida das pessoas, prática que intervém na Educação Sexual que
todas elas receberam em contato com a sociedade em que vivem.

Citando Suplicy et. al. “Orientação Sexual é um processo de intervenção sistemática na área de
sexualidade, realizado principalmente nas escolas e envolve o desenvolvimento sexual compreendido
como: saúde reprodutiva, relações interpessoais, afetividade, imagem corporal, autoestima e relações de
gênero. Enfoca as dimensões fisiológicas, sociológicas, psicológicas e espirituais da sexualidade, através
do desenvolvimento das áreas cognitiva, afetiva e comportamental, incluindo as habilidades para a
comunicação e a tomada responsável de decisões”.

Percebemos a concordância de Suplicy et. al. com Ribeiro em afirmar que a Orientação Sexual é
uma prática interventiva sistemática na área da sexualidade. Suplicy et. al., na definição citada,
enfatiza que a Orientação Sexual deve ser pensada e executada a partir da consideração do orientando
enquanto ser integral, ou seja, devem ser consideradas suas dimensões fisiológicas, sociológicas,
psicológicas e espirituais no exercício de sua sexualidade. Além disso, a Orientação Sexual deve
contemplar diversos aspectos do desenvolvimento sexual dos indivíduos, ou seja, saúde reprodutiva,
relações interpessoais, afetividade, imagem corporal, autoestima e relações de gênero. Compreende-se
o ser humano enquanto ser sexuado inserido num meio social, que continuamente se relaciona com
outros seres humanos. Desta forma, amplia-se o enfoque da Orientação Sexual no Brasil que, no início e
meados do século XX priorizava a dimensão biológica da sexualidade. No final do século XX e nos dias
atuais, deve-se compreender a sexualidade enquanto manifestação humana, com desdobramentos além
da mera reprodução e da possibilidade de contágio de doenças sexualmente transmissíveis. Tais
aspectos não devem ser descartados, mas deve-se somar a eles outros aspectos como o prazer, as
relações afetivas e os papéis sexuais na (re)definição de gênero.
Neste contexto, Santos e Bruns apontam que um dos objetivos da Orientação Sexual é levar o
indivíduo a valorizar o prazer, o respeito mútuo, possibilitando-lhe uma vivência mais íntegra e feliz.

Breve histórico da Orientação Sexual no Brasil

No Brasil, a sexualidade tem sido um aspecto polêmico do cotidiano das pessoas, desde a época da
Colônia do século XVI.

Apostila gerada especialmente para: Betânia Mueller 023.712.580-32


. 274
O homem brasileiro branco, nos primeiros anos da colonização, mantinha relações sexuais com várias
índias, tendo com elas muitos filhos, caracterizando um comportamento sexual bastante promíscuo.
Com o advento da escravatura, os jovens homens filhos dos senhores de engenho eram incentivados
a se relacionar sexualmente com as escravas negras, para provar que eram “machos”. As mulheres
brancas eram dominadas e submetidas às regras de seus pais, inicialmente, e de seus maridos, após o
casamento. Em geral, casavam ainda adolescentes com homens bem mais velhos que elas. Era-lhes
exigido um comportamento acanhado e humilde frente à sociedade.
Tal cenário brasileiro se mantém praticamente o mesmo durante os séculos XVII, XVIII e XIX. Neste
período da História do Brasil não há registros conhecidos de Orientação Sexual enquanto intervenção
sistematizada.
A preocupação com a Orientação Sexual no Brasil, enquanto tema científico e pedagógico, data do
início do século XX. Neste momento da história brasileira registra-se a organização dos primeiros espaços
urbanos, que originaram as cidades brasileiras. Nestes locais a comunidade científica brasileira se
organizava sofrendo forte influência europeia.

Barroso e Bruschini afirmam que, no início do século XX, esta influência europeia manifesta-se no
Brasil através de algumas correntes médicas e higienistas de sucesso na Europa. Tais correntes
pregavam a necessidade de uma Educação Sexual eficaz no combate à masturbação e às doenças
venéreas (termo utilizado na época para referir-se às doenças sexualmente transmissíveis – DST´s) e
que preparasse a mulher para desempenhar adequadamente seu “nobre papel de esposa e de mãe”.
Notamos que, logo no início de suas atividades no Brasil, a Orientação Sexual carrega uma característica
de incitação do medo aos jovens (combate à masturbação e às doenças sexualmente transmissíveis –
DST´s), além de ser impregnada pela chamada ideologia de gênero machista (preparar a mulher para
desempenhar adequadamente seu papel de esposa e mãe).
Neste momento, emerge a produção de teses, livros e manuais que tratam da Orientação Sexual,
todos baseados no modelo médico higienista vigente. Referenciando este período, Chauí cita uma obra
datada de 1938, de autoria de Oswaldo Brandão da Silva, intitulada Iniciação Sexual-Educacional. Este
livro, segundo consta, tinha um conteúdo destinado somente aos “meninos de valor”. Segundo esta
autora, o autor da obra não explica o significado do termo “valor”, mas fica claro que as meninas estavam
proibidas de ler tal obra, pois deveriam manter-se inocentes e ser iniciadas na vida sexual apenas por
seus maridos. Interessante ressaltar que, do grupo de meninas excluídas do acesso ao conteúdo da obra,
não fazem parte as prostitutas. Estas eram consideradas uma tentação para os meninos enquanto
aquelas eram chamadas de meninas de “boa família”.
Entre as décadas de 1920 e 1940, mesma época em que foi publicado o manual citado por Chauí,
foram publicados vários outros livros de orientação sexual cientificamente fundamentados, escritos por
médicos, professores e até sacerdotes. Assim foi criada a sexologia enquanto campo oficial do saber
médico.

Concomitante à consolidação do conhecimento científico da época em relação à sexualidade, a Igreja


Católica imprime severa repressão às práticas sexuais da população brasileira. Desta forma, a década
de 50 é considerada pobre no sentido de não contar com nenhuma iniciativa no campo da Orientação
Sexual.
Na década de 60 surgem as primeiras experiências de Orientação Sexual nas escolas dos estados de
Minas Gerais (Belo Horizonte, em 1963, no Grupo Escolar Barão do Rio Branco), Rio de Janeiro (Rio de
Janeiro, em 1964, no Colégio Pedro Alcântara; em 1968, nos colégios Infante Dom Henrique, Orlando
Rouças, André Maurois e José Bonifácio) e São Paulo (São Paulo, de 1963 a 1968, no Colégio de
Aplicação Fidelino Figueiredo; de 1961 a 1969, nos Ginásios Vocacionais; de 1966 a 1969, no Ginásio
Estadual Pluricurricular Experimental). Estas experiências são realizadas com base na ênfase ao aspecto
biológico da sexualidade humana, tal qual era o tratamento dado a esta questão nos livros que
possibilitaram o surgimento da sexologia enquanto área do conhecimento da medicina. Além disso, estas
experiências foram fortemente carregadas com as marcas da repressão das manifestações da
sexualidade.

Na época das primeiras experiências em Orientação Sexual nas escolas brasileiras, o país vivia seu
período histórico e político chamado de ditadura militar. Em 1964, a população assiste à chegada das
forças armadas ao poder da República Federativa do Brasil, através da imposição do Golpe de Estado.
A partir daí, o regime militar reprime não só as manifestações políticas, mas também as manifestações
sexuais e as implicações nos padrões de comportamento delas decorrentes.

Apostila gerada especialmente para: Betânia Mueller 023.712.580-32


. 275
Em 1968, a deputada federal do Rio de Janeiro Júlia Steinbruk apresentou um projeto de lei que previa
a introdução obrigatória da Educação Sexual nas escolas brasileiras. Tal projeto de lei não foi
transformado em legislação porque o então Ministério da Educação e Cultura, através de sua Comissão
Moral e Civismo, rejeitou o projeto, demonstrando o severo receio por parte dos gestores da educação
brasileira da época em relação ao tratamento de questões sexuais com os estudantes.
Na década de 70, cresce a censura do governo militar e há um quase desaparecimento de projetos de
Orientação Sexual nas escolas brasileiras. Apenas em 1978, com a abertura política trazida pelo
presidente Ernesto Geisel, a Prefeitura Municipal de São Paulo implantou projetos de Orientação Sexual
em três escolas, os quais, posteriormente, foram ampliados para muitas escolas municipais, envolvendo
orientadores educacionais e professores de Ciências e Biologia. Em 1979, a rede pública estadual
paulista iniciou um trabalho de informação aos estudantes sobre os aspectos biológicos da reprodução,
por intermédio da disciplina de Ciências e Programas de Saúde da Secretaria de Educação do Estado de
São Paulo.
Ao fim da década de 70 e durante a década de 80, surgem novas ações no plano da Orientação Sexual,
como o aparecimento de serviços telefônicos, programas de rádio e de televisão, enciclopédias e
fascículos, congressos e encontros de professores. Proliferam as iniciativas na rede particular de ensino.
Nasce nessa época a SBRASH – Sociedade Brasileira de Sexualidade Humana.

De 1989 a 1992, na cidade de São Paulo, foi desenvolvido um abrangente projeto de Orientação
Sexual nas escolas municipais, com a participação do renomado GTPOS (Grupo de Trabalho e Pesquisa
em Orientação Sexual). Este projeto atingiu 30.000 alunos e foram capacitados 1.105 professores para
oferecer ações de orientação sexual nas escolas.
Nota-se que, desde as primeiras experiências de projetos de Orientação Sexual na década de 1960,
não existiram ações continuadas, sendo que estes projetos historicamente ficaram atrelados às vontades
político-partidárias de prefeitos ou governadores.
Ribeiro corrobora dizendo que, somente com a aprovação da LDB – Lei de Diretrizes e Bases em 1996
e o estabelecimento dos Parâmetros Curriculares Nacionais em 1997 como linhas a serem seguidas para
se concretizar a meta da educação para o exercício da cidadania, a Orientação Sexual teve oficialmente
reconhecida sua necessidade e importância enquanto ação educativa escolar.

Os programas de Orientação Sexual

Podemos constatar na maioria dos programas de Orientação Sexual executados no Brasil, ainda nos
dias atuais, uma tendência de mostrar apenas os problemas e possíveis más consequências da
sexualidade. Em geral, no conteúdo destes programas são enfatizadas (quando não são exclusivas) as
DST – Doenças Sexualmente Transmissíveis e as gravidezes precoces na adolescência, com
maternidade e/ou paternidade indesejadas. Este conteúdo não sensibiliza os jovens para a discussão
construtiva do tema sexualidade humana. Eles costumam não se sentir à vontade para receber uma
adequada Orientação Sexual, pois identificam claramente a repressão sexual que experimentam em seu
meio social, aqui também reproduzida pelos profissionais orientadores sexuais.
Em contato com um conteúdo de Orientação Sexual que prioriza os problemas advindos de uma
vivência inadequada da sexualidade e não os aspectos afetivos, prazerosos, e de respeito às relações
humanas, os jovens costumam não perceber uma relação coerente entre o conteúdo abordado e suas
próprias experiências reais concretas. Comenta-se que o sexo traz problemas, mas a maioria dos jovens
percebe suas experiências sexuais como prazerosas, surgindo aí um paradoxo.
Desta forma, urge a necessidade da discussão de conteúdos adequados à realidade dos jovens para
que eles possam realmente tomar atitudes responsáveis na vivência de suas sexualidades. Assim, um
programa efetivo de Orientação Sexual deve reconhecer o exercício prazeroso da sexualidade, sem
deixar de contemplar as medidas de proteção à saúde e os métodos contraceptivos para tornar possível
a emergência de maternidades e paternidades responsáveis, no momento de escolha consciente de cada
pessoa que deseje ter filhos.
Nos dias atuais, percebe-se a crescente preocupação de alguns pais e educadores diante do número
de gestações na adolescência. Segundo o Ministério da Saúde, enquanto a taxa de fecundidade de
mulheres adultas tem caído nas últimas quatro décadas, entre as mulheres jovens existe uma relação
inversamente proporcional. “Desde os anos 90, a taxa de fecundidade entre adolescentes aumentou 26%.
Tal preocupação mobiliza e estimula o avanço das ações em orientação sexual, o que pode ser
intensamente benéfico para os jovens, visto que eles poderão ter maior acesso a programas desta
natureza. No entanto, cabe questionar se pais e educadores ainda mantêm seu foco sob uma concepção
repressiva da sexualidade humana, desejando que uma Orientação Sexual possa produzir uma atitude

Apostila gerada especialmente para: Betânia Mueller 023.712.580-32


. 276
sexualmente abstinente dos jovens brasileiros, desejo que se mostra absolutamente inalcançável e
indesejável. De outro modo, a preocupação advinda dos pais e educadores quanto ao número de
gestações na adolescência pode ser um ponto de partida para propiciar espaços abertos de discussão,
onde o jovem possa refletir sobre sua própria sexualidade, no sentido de conscientemente poder efetuar
escolhas para sua vida, que incluem ter ou não filhos. Para tal escolha, o jovem, que num futuro próximo
se tornará um adulto, deve ter conhecimento e autonomia sobre o uso de métodos contraceptivos.
Outra preocupação de pais e educadores que mobiliza a execução de programas de Orientação Sexual
são as doenças sexualmente transmissíveis uma vez que, ao iniciar a vida sexual, muitos jovens, ainda
que possuam conhecimento de prevenção, não utilizam preservativo.
Infelizmente a maioria dos programas brasileiros de Orientação Sexual não é contínua. Caracterizam-
se muitas vezes pelo oferecimento de palestras pontuais sobre sexualidade. Este tipo de programa não
atinge os objetivos de propiciar elementos para uma construção adequada do exercício da sexualidade
dos jovens. Para trazer efetivos benefícios à juventude, o processo de educação precisa de continuidade,
de vínculo, de tempo, de reconhecimento.

Orientação Sexual como tema transversal

O governo federal brasileiro, através do Ministério da Educação - MEC, em seus Parâmetros


Curriculares Nacionais (1997), estabelece a Orientação Sexual no Ensino Fundamental enquanto tema
transversal, isto é, um assunto a ser trabalhado em todas as disciplinas escolares, por quaisquer
professores que se sintam mobilizados, sempre que houver espaço na grade curricular ou em horários
extraclasses.
Segundo os Parâmetros Curriculares Nacionais - PCN, “propõe-se que a Orientação Sexual oferecida
pela escola aborde com as crianças e os jovens as repercussões das mensagens transmitidas pela mídia,
pela família e pelas demais instituições da sociedade. Trata-se de preencher lacunas nas informações
que a criança e o adolescente já possuem e, principalmente, criar a possibilidade de formar opinião a
respeito do que lhes é ou foi apresentado. A escola, ao propiciar informações atualizadas do ponto de
vista científico e ao explicitar e debater os diversos valores associados à sexualidade e aos
comportamentos sexuais existentes na sociedade, possibilita ao aluno desenvolver atitudes coerentes
com os valores que ele próprio eleger como seus”.

Percebemos o complexo dever atribuído à Orientação Sexual no âmbito escolar na medida em que é
sua função a reflexão contínua sobre as informações constantes recebidas pelos jovens em suas relações
sociais. Daí decorre a necessidade de que os profissionais que executam programas de Orientação
Sexual tenham conhecimentos científicos suficientes e adequados para abordar as demandas cotidianas
da juventude em relação à sexualidade. É preciso que, pela Orientação Sexual, os jovens possam formar
suas opiniões a respeito do tema para propiciar um pleno exercício de suas sexualidades.
Apesar da clara proposição dos PCN de conceber a Orientação Sexual no âmbito escolar enquanto
tema transversal extremamente importante para a formação de valores conscientes pelos jovens em
relação à sexualidade, muitas dificuldades têm permanecido no exercício diário desta prática educacional.
Como sexo é um assunto intensamente repleto de repressões em nossa sociedade ocidental, muitos
educadores não manifestam interesse sobre o tema, deixando de buscar formação adequada para o
trabalho de Orientação Sexual com a juventude.
Além dos profissionais diretamente em contato com os jovens, há uma grande parcela de educadores
que são dirigentes de estabelecimentos educacionais e, reproduzem as mesmas repressões sociais em
relação à sexualidade, não contribuindo positivamente para a execução de bons programas de Orientação
Sexual, uma vez que não acreditam que este tema seja importante para a comunidade estudantil ou
acreditam que falar sobre sexualidade com jovens estudantes pode induzi-los à prática precoce de
relações sexuais.
A Orientação Sexual na escola ainda tem um extenso caminho a ser trilhado para que a sexualidade,
presente na vida de todas as pessoas, possa ser tratada (e aprendida) pelos profissionais da educação
e seus respectivos educandos sem os massacrantes e silenciadores tabus e com respeito e propriedade,
para inibir práticas inadequadas e produzir práticas saudáveis do exercício da sexualidade.

O Educador/Orientador Sexual

Retomando a discussão sobre a definição dos termos “educação sexual” e “orientação sexual”
presente no item “Orientação Sexual X Educação Sexual” deste trabalho, encontramos com maior
frequência na literatura especializada o termo “educador sexual” referindo-se àquele profissional que

Apostila gerada especialmente para: Betânia Mueller 023.712.580-32


. 277
exerce a prática educacional de Orientação Sexual, enquanto prática institucionalizada e sistematizada.
Desta forma, neste momento, utilizaremos o termo “educador sexual” para fazermos referência a este
profissional especializado e não aos membros da família e demais relações interpessoais dos jovens, que
contribuem para a sua educação em um sentido mais amplo, conforme Vitiello.
Segundo Canosa Gonçalves, o desenvolvimento psicossexual é um processo único e pessoal, que
sofre transformações ao longo do processo por diversos aspectos do comportamento sexual humano
sendo eles: constituição biológica do indivíduo (hereditariedade, níveis hormonais), relações familiares,
padrão econômico, características culturais, adoção da fé, entre outros.
Portanto, o educador sexual, ao realizar sua prática, está inserido neste complexo contexto do
comportamento humano e deve intervir nesta realidade. Os jovens com os quais o educador sexual
trabalhará trazem em suas histórias de vida diversas realidades, variadas construções biopsicossociais
em um mesmo grupo de jovens orientandos. Cabe ao educador sexual ter capacidade para perceber tais
diferenças e pautar suas ações de maneira a privilegiar a diversidade, num contexto de respeito às
escolhas pessoais de cada jovem. Ao educador sexual é requerida abertura intelectual, moral e afetiva
para tornar possível a realização da Orientação Sexual com jovens tão diversos.
A Orientação Sexual deve ser uma prática ofertada a todos os jovens, mas não uma prática arbitrária
e unidimensional, que reproduz os preconceitos repressivos de nossa sociedade. Assim, o educador
sexual deve ser flexível em relação às diversas orientações afetivo-sexuais, às religiosidades, enfim,
diversas concepções construídas sobre sexualidade na história pessoal de cada jovem. Orientação
Sexual “se destina à pessoa humana, com a prerrogativa de igualdade entre os seres humanos, em
primeiro lugar”.
O educador sexual deve apresentar adequação sexual, isto é, reconhecer-se enquanto pessoa
sexuada, com suas preferências e limites, e não influenciar as decisões dos jovens a partir destas
preferências. Diferenciar-se pessoalmente de quem orienta é imprescindível para que o educador sexual
possa propiciar condições para reflexão ao jovem para que este possa realizar suas próprias escolhas.
Segundo Canosa Gonçalves um bom educador sexual é “aquele que convive com os jovens no
dia-a-dia, que os conhece e é reconhecido por eles, e que tem em sua prática profissional os
pressupostos da educação”.
Desafiante para o trabalho do educador sexual com jovens é utilizar métodos e técnicas que
prendam a atenção deste público, que provoquem reflexão e que sejam capazes de fazer com que
o jovem se comprometa consigo próprio e com suas parcerias.
É imprescindível que o educador sexual possua conhecimentos científicos adequados sobre
desenvolvimento humano, constituição dos órgãos sexuais, saúde reprodutiva, métodos de prevenção às
DST´s e/ou contraceptivos, relacionamentos interpessoais e relações de gênero. Não é necessário que o
profissional detenha estes conhecimentos em nível de especialista em sexualidade humana, mas deve
continuar buscar atualizar tais saberes, afim de oferecer uma prática de qualidade em relação à
Orientação Sexual.
Nesta realidade, o desafio proposto ao orientador sexual é que, através de seu trabalho, possa
propiciar condições para que os jovens reflitam a respeito de suas sexualidades e possam exercê-las de
maneira saudável. Segundo Vitiello educar é dar ao educando condições e meios para que cresça
interiormente.

Mas afinal como é diversidade sexual/de gênero no ambiente escolar?

Gênero e sexualidade: diálogos e conflitos

Marcas epistemológicas
O modo de compreender a diferença evoluiu no sentido de pensa-la junto com o seu duplo, seu
contrário, seu avesso, ou seja, ela é sempre relacional e dificilmente bipolarizada. Esse modo de
compreensão aguça a sensibilidade humana e sua condição de experimentar, de se (auto)inventar.
A relevância do debate crítico ancorado no domínio discursivo da heterossexualidade que,
pretensiosamente hegemônica e unificada em um modo de ser, desconsidera outras formas que não
atendem às suas práticas discursivas. Pensamos que essa situação reflete-se diretamente nas práticas
curriculares, prejudicando o entendimento de diversas relações sociais e culturais presentes na escola, e
mais amplamente, na sociedade. Estamos entendendo como currículos as ações escolares, culturais e
tecnológicas (arquitetura, livros didáticos, vestimentas, músicas, conteúdos e dizeres científicos, meios
midiáticos e outros) que, significadas na cultura, ensinam e regulam o corpo, produzindo subjetividades
e arquitetando formas e configurações de viver na sociedade.

Apostila gerada especialmente para: Betânia Mueller 023.712.580-32


. 278
Os equívocos
Recorda-se que, no Brasil, a homossexualidade deixou de se configurar como doenças nos
instrumentos médicos (mais precisamente como desvio mental e transtorno sexual), em fevereiro de
1985. Essa alteração foi fruto de uma intensa campanha, liderada pelo antropólogo Luiz Mott, junto com
o Conselho Federal de Medicina (CFM) que, por resolução, retirou a homossexualidade da lista de
doença. Sendo importante lembrar que, já em 1973, a American Psychiatric Association, afirmara que a
homossexualidade não tinha ligação alguma com qualquer tipo de patologia e propusera a sua retirada
do Manual de Diagnóstico e Estatística de Transtornos Mentais (DSM-IV). Já a Organização Mundial de
Saúde (OMS), somente no dia 17 de maio de 1990, reuniu-se em Assembleia Geral e retirou a
homossexualidade de sua lista de doenças mentais, declarando que ela não constituía um distúrbio, uma
doença ou perversão. Assim, o que antes tinha sido classificado, estabelecido e difundido como desvio e
anormalidade, a partir dessa assembleia, seria considerado normal.
Se aceitarmos a sexualidade assim como a experiência estão condicionadas pela necessidade
humana de se construir nas interações sociais, culturais e históricas, aceitaremos também que não há
uma única sexualidade. A ausência de liberdade impede o movimento de busca pela completude, na qual
a sexualidade, como dimensão da humanidade, se constitui.
Existe um nexo entre a sexualidade, a vida e a curiosidade pelo saber. Esse movimento infinito em
busca de completude e em busca de conhecimento é fator que constitui o ser humano e seu desejo de
liberdade.
No entanto, ainda que pareça contraditório, não confiamos no desejo como princípio, condição e direito
de liberdade. Não cremos, em absoluto, que haja desejo anterior a um conjunto de normas ou acordos
sociais que o faça livre. Nós o pensamos como criado singularmente, mas em redes de relações.
Sem dúvidas, a compreensão da sexualidade poderá contribuir, de modo significativo, para novas
possibilidades de construção de conhecimentos e caminhos de busca do saber. Não se trata, portanto,
de aprisioná-la nos discursos sobre o ato sexual, mas de aproveitá-la em seu potencial epistemológico.
Essa análise é especialmente oportuna e necessária à escola.

A discussão na escola
Na escola, as atitudes de hostilidade às identidades sexuais dissidentes são capazes de gerar
inúmeras situações de violências homofóbicas. Algumas, que não se encontram na esfera dos números
e dados quantitativos, são vivenciadas no silêncio e ocultadas na invisibilidade.
A discriminação afirma o “direito” dos que discriminam e a subalternidade dos que são discriminados.
Nesse sentido, ela é observada nos espaços-tempos escolares. As identidades vinculadas às
expectativas de gênero e/ou sexo biológico estão no interior das hierarquizações e classificações sociais,
tanto quanto nos currículos e, mais amplamente, nas ações e relações do cotidiano escolar.
A sexualidade, infelizmente, é algo temido e capaz de gerar tantos discursos na sociedade, na ciência
e na cultura. Sua estreita relação com o conhecimento amedronta os que se nutrem da arrogância, porque
fragiliza suas verdades e certezas.

Foucault35 nos ajuda a observar que é preciso fortalecer, aprofundar e prosseguir contra a dicotomia e
lógica binária, até que as oposições binárias deixem de ter sentido e se consolidem convivências
solidárias, em contextos sem discriminações e violências. Como estratégia para fazer difuso o antigo jogo
de poder que se instala na relação entre opressor e oprimido, a proposta foucaultiana é a “proliferação”
de saberes sobre os seres humanos e as relações e de poder que os oprimem, de tal modo que o modelo
jurídico de poder como opressão e regulação deixe de ser hegemônico. Talvez, desse significado de
“proliferação” de saberes, possamos retirar as bases para “proliferar” inúmeras e ilimitadas formas de
compreender os seres humanos, sem as violências, já tantas vezes vivenciadas, e com tantas
exterminações em massa, como na Segunda Guerra, devido à não aceitação do “outro”, a quem se atribui
dessemelhança e desigualdade, potencializando os efeitos destrutivos da xenofobia que, em todas as
suas manifestações, incluindo as homofóbicas, conduz e justifica a aversão, o domínio ou a eliminação
dos “estranhos”, que ameaçam e incomodam o exercício arbitrário do poder.

Para saber mais...

A seguir alguns termos relevantes a serem considerados sobre a diversidade de gênero:


ASSIMETRIAS DE GÊNERO: desigualdades de oportunidades, condições e direitos entre homens e
mulheres, gerando hierarquias. Por exemplo: no mercado de trabalho.

35
FOUCAULT, M. História da sexualidade – A vontade de saber. Rio de Janeiro: Graal, 1988.

Apostila gerada especialmente para: Betânia Mueller 023.712.580-32


. 279
BINARISMO: forma de pensamento que separa e opõe masculino e feminino, apoiando-se numa
concepção naturalizante dos corpos biológicos.

BISSEXUAL: pessoa que tem desejos, práticas sexuais e relacionamento afetivo-sexual com pessoas
de ambos os sexos;

CORPO: inclui além das potencialidades biológicas, todas as dimensões psicológicas, sociais e
culturais do aprendizado pelo qual as pessoas desenvolvem a percepção da própria vivência. Não existe
um corpo humano universal – mas sim corpos marcados por experiências específicas de classe, de etnia,
de raça, de gênero, de idade. Visto que os corpos são significados e alterados pelas diferentes culturas,
pelos processos morais, pelos hábitos, pelas distintas opções e possibilidades de desejo, além das
diversas formas de intervenção e produção tecnológica. Por isso, o corpo é uma produção histórica.
Foucault ao analisar instituições como escolas, prisões, hospitais psiquiátricos, fábricas, fala das
maneiras como as diferentes disciplinas controlam, domesticam, normalizam os corpos. Sua preocupação
é com as práticas sociais, sendo que é no corpo que se dá o controle da sociedade sobre os indivíduos.
Os corpos apresentam as marcas do processo de passar ou não pela escola como o auto disciplinamento,
o investimento continuado e autônomo do sujeito sobre si mesmo.
Louro parte do pressuposto antropológico de que "os corpos são o que são na cultura”, isto é, que os
corpos adquirem seu significado apenas através dos discursos na cultura e na história. Essa vertente se
afasta das discussões teóricas nas quais o corpo é tido como “natural”, no qual o biológico determina o
gênero.

DESIGUALDADE: é um fenômeno social que produz uma hierarquização entre os indivíduos e/ou
grupos que não permite o tratamento igualitário (em termos de mercado de trabalho, de acesso a bens e
recursos, para todos e todas.
Essa desigualdade existe na divisão dos atributos entre homens e mulheres. Esse desnível se
evidencia em vários contextos: familiar, social, escolar, religioso, econômico, político,... Dessa forma, fica
claro que existem fronteiras que separam atitudes e comportamentos tidos como apropriados, válidas e
legítimas relacionadas ao sexo masculino e ao feminino.

DIFERENÇA: indivíduos e/ou grupos possuem várias formas de distinção e de semelhanças (cor, sexo,
idade, nacionalidade). A desigualdade pauta-se por essas diferenças e semelhanças que constituem os
indivíduos e/ou grupos.

DIREITOS SEXUAIS: direitos que asseguram aos indivíduos a liberdade e a autonomia nas escolhas
sexuais, como a de exercer a orientação sexual sem sofrer discriminações ou violência. Os direitos
sexuais englobam múltiplas expressões legítimas da sexualidade, como por exemplo, o direito à saúde –
direito de cada pessoa de ver reconhecidos e respeitados o seu corpo (autonomia), o seu desejo e o seu
direito de amar (reconhecimento da diversidade sexual).

DISCRIMINAÇÃO: ação de discriminar, tratar diferente, excluir, marginalizar.

ESTEREÓTIPO: é uma generalização de julgamentos subjetivos feitos a um grupo ou a um indivíduo.


Pode ser atribuindo valor negativo desqualificando-os e impondo-lhes um lugar inferior, ou simplesmente,
reduzindo determinado grupo ou indivíduo a algumas características e, assim, definindo lugares
específicos a serem ocupados.

FEMINILIDADE: se refere às características e comportamentos considerados por uma determinada


cultura associados ou apropriados às mulheres.
Caracterizar os comportamentos como “masculinos” ou “femininos” é basear-se nas noções
essencialistas do binarismo mulher/homem, isto quer dizer que, atributos que muitas vezes são
considerados femininos podem estar baseados no biológico e nas diferenças físicas. Dessa forma, a
feminilidade nos homens, bem como a masculinidade nas mulheres, é considerada negativa por agir
contra os papéis tradicionais da nossa cultura. Um estereótipo comum para homens homossexuais é de
que são efeminados porque utilizam ou exageram comportamentos tidos como femininos, por exemplo.

GÊNERO: conceito formulado a partir das discussões trazidas do movimento feminista para expressar
contraposição ao sexo biológico e aos termos “sexo” e “diferença sexual”, distinguindo a dimensão
biológica da dimensão sexual e, acentuando através da linguagem, “o caráter fundamentalmente social

Apostila gerada especialmente para: Betânia Mueller 023.712.580-32


. 280
das distinções baseadas no sexo”. Não com a intenção de negar totalmente a biologia dos corpos, mas
para enfatizar a construção social e histórica produzida sobre as características biológicas. Dessa forma,
gênero seria a construção social do sexo anatômico demarcando que homens e mulheres são produtos
da realidade social e não decorrência da anatomia dos seus corpos.

HETERONORMATIVIDADE: termo utilizado para expressar que existe uma norma social que está
relacionada ao comportamento heterossexual como padrão. Dessa forma, a ideia de que apenas o padrão
de conduta heterossexual é válido socialmente, colocando em desvantagem os sujeitos que possuem
uma orientação sexual diferente da heterossexual.

HETEROSSEXISMO: Se refere à ideia de que a heterossexualidade é a orientação sexual “normal” e


“natural”. Considerar a heterossexualidade como “natural”, aponta para algo inato, instintivo e que não
necessita de ser ensinado ou aprendido. Ao considerar a heterossexualidade “normal”, contrapõe-se a
ideia de que as outras orientações sexuais (homossexualidade e bissexualidade, por exemplo) são um
desvio à norma e reveladoras de perturbação, não sendo encaradas como um dos aspectos possíveis na
diversidade das expressões da sexualidade humana. O heterossexismo funciona através de um sistema
de negação e discriminação – a sociedade tende a negar a existência da homossexualidade, tornando-a
invisível (em quantos manuais escolares existem referências neutras ou positivas à homossexualidade?)
e tende a reprimir e discriminar todos aqueles que se tornam visíveis.

HETEROSSEXUAL: quem tem atração sexual por pessoas do sexo oposto ao seu, e relacionamento
afetivo-sexual com elas. Heterossexuais não precisam, necessariamente, terem vivido experiências
sexuais com pessoas do mesmo sexo ou do sexo oposto para se identificarem como tal.

HETEROSSEXUALIDADE COMPULSÓRIA: sistema que acomoda e hierarquiza as relações de


gênero, no qual o homem é o modelo para todas as relações, inclusive aquelas em que ele não está
presente.

HOMOAFETIVO: é um termo utilizado para descrever relações entre pessoas do mesmo sexo e tem
relação com os aspectos emocionais e afetivos envolvidos na relação amorosa e sexual entre essas
pessoas.

HOMOFOBIA: termo usado para descrever vários fenômenos sociais relacionados ao preconceito, a
discriminação e à violência contra os homossexuais (ter desprezo, ódio, aversão ou medo de pessoas
com orientação sexual diferente do padrão heterossexual). O termo, no entanto, não se refere ao conceito
tradicional de fobia, facilmente associável à ideia de doença e tratados com terapias e antidepressivos.
Atualmente, grupos lésbicos, bissexuais e transgêneros, com o intuito de conferir maior visibilidade
política à suas lutas e criticar normas e valores postos pela dominação masculina, propõem, também, o
uso dos termos lesbofobia, bifobia e transfobia.
Daniel Borrillo faz uma leitura epistemológica e política desse conceito, não para compreender a
origem e o funcionamento da homossexualidade, mas para “analisar a hostilidade provocada por essa
forma específica de orientação sexual”. Segundo este autor quando a homossexualidade requer
publicamente sua expressão é que se torna insuportável, pois rompe com a hierarquia da ordem sexual.
Por isso, a tarefa pedagógica deve ser questionar a heterossexualidade compulsória e mostrar que a
hierarquia de sexualidades é tão insustentável quanto a de sexos, bem como incluir a ideia de diversidade
sexual em livros e apostilas escolares.

HOMOSSEXUAL: é a pessoa que tem atração sexual e afetiva por pessoas do mesmo gênero e
relacionamento com elas.

HOMOSSEXUALIDADE: é a atração sexual e afetiva por pessoas do mesmo sexo. Cabe uma
ressalva, não é correto o uso do termo homossexualismo, porque reveste de conotação negativa,
atribuindo-lhe significado de doença e aberração. Por isso, devemos preferir a utilização dos termos
homossexualidade, lesbianidade, bissexualidade, travestilidade, transgeneridade e transexualidade.

IDENTIDADE DE GÊNERO: Expressão utilizada primeiramente no campo médico-psiquiátrico para


designar os “transtornos de identidade de gênero”, isto é, o desconforto persistente criado pela
divergência entre o sexo atribuído ao corpo e a identificação subjetiva com o sexo oposto. Entretanto,
atualmente, a identidade de gênero corresponde à experiência de cada um, que pode ou não

Apostila gerada especialmente para: Betânia Mueller 023.712.580-32


. 281
corresponder ao sexo do nascimento. Podemos dizer que a identidade de gênero é a maneira como
alguém se sente e se apresenta para si ou para os outros na condição de homem ou de mulher, ou de
ambos, sem que isso tenha necessariamente uma relação direta com o sexo biológico. É composta e
definida por relações sociais e moldadas pelas redes de poder de uma sociedade. Os sujeitos têm
identidades plurais, múltiplas, identidades que se transformam, que não são fixas ou permanentes, que
podem até ser contraditórias. Os sujeitos se identificam, social e historicamente, como masculinos e
femininos e assim constroem suas identidades de gênero.
Cabe enfatizar que a identidade de gênero trata-se da forma que nos vemos e queremos ser vistos,
reconhecidos e respeitados, como homens ou mulheres, e não pode ser confundida com a orientação
sexual (atração sexual e afetiva pelo outro sexo, pelo mesmo sexo ou por ambos).

IDENTIDADE SEXUAL: Identidades sexuais se constituem através das formas como vivemos nossa
sexualidade, e refere-se a duas questões diferenciadas:
1) é o modo como a pessoa se percebe em termos de orientação sexual;
2) é o modo como ela torna pública (ou não) essa percepção de si em determinados ambientes ou
situações. Quer dizer, corresponde ao posicionamento (nem sempre permanente) da pessoa como
homossexual, heterossexual, ou bissexual, e aos contextos em que essa orientação pode ser assumida
pela pessoa e/ou reconhecida em seu entorno.

INTERSEXUAL OU INTERSEX: a palavra intersexual é preferível ao termo hermafrodita e é um termo


usado para se referir a uma variedade de condições (genéticas e/ou somáticas) com que uma pessoa
nasce, apresentando uma anatomia reprodutiva e sexual que não se ajusta às definições de masculino e
feminino, tendo parcial ou completamente desenvolvidos ambos os órgãos sexuais, ou um predominando
sobre o outro. A intersexualidade, enquanto transgeneridade é uma condição e não uma orientação
sexual. Portanto, as pessoas que se autodenominam intersexuais podem se identificar como
homossexuais, heterossexuais ou bissexuais.

LESBOFOBIA: termo usado para descrever vários fenômenos sociais relacionados ao preconceito, a
discriminação e à violência contra as lésbicas (ter desprezo, ódio, aversão ou medo de pessoas com
orientação sexual diferente do padrão heterossexual). Ver homofobia.

MACHISMO: é a crença de que os homens são superiores às mulheres. É uma construção cultural
que definiu que as características atribuídas aos homens, tem um valor maior. Se pensarmos na educação
de meninos e meninas, veremos que há um tratamento diferenciado que reproduz as manifestações de
machismo nos meninos, e às vezes, nas próprias meninas. Ao incentivar (infidelidade, violência
doméstica, esporte, diferença de direitos).

MASCULINIDADE: Faz oposição ao termo feminilidade e diz respeito a imagem estereotipada de tudo
aquilo que seria próprio dos indivíduos homens, ou seja, às características e comportamentos
considerados por uma determinada cultura como associados ou apropriados aos homens. Ver
feminilidade, pois são conceitos relacionais que não passíveis de serem entendidos separadamente.

MASCULINIDADE HEGEMÔNICA: É um modelo construído socialmente que controla, domina e


substima as diversas formas de expressão de outras masculinidades, tornando-se um padrão de
masculinidade.

MOVIMENTO FEMINISTA: o movimento feminista surgiu para questionar a organização social,


política, econômica, sexual e cultural de uma sociedade profundamente hierárquica, autoritária,
masculina, branca e excludente. Sendo assim, o feminismo pode ser entendido como uma luta pela
transformação da condição das mulheres, que é pública e também privada. E que pode ser entendida, a
partir de três eixos:
1) como movimento social e político;
2) como política social;
3) e como ciência, ampliando os debates teóricos e conceituais (derivando a categoria gênero como
analítica de sexo).
Essas vias se entrecruzam, por diversas vezes, para desestabilizar representações, questionar a
divisão sexual da sociedade, opor-se à hierarquização dos gêneros e, por isso, as teorias nem sempre
podem dissociar-se de suas ações políticas, e vice-versa.

Apostila gerada especialmente para: Betânia Mueller 023.712.580-32


. 282
PODER/RELAÇÕES DE PODER: nossas definições, crenças, convenções, identidades e
comportamentos sexuais têm sido modeladas no interior de relações definidas de poder. Para Michel
Foucault, o poder está em toda parte; não porque englobe tudo e sim porque provém de todos os lugares.
O poder se exerce de diversas formas: poder de produzir os corpos que controla, produz sujeitos, fabrica
corpos dóceis, induz comportamentos. Foucault propõe que observemos o poder como uma rede que,
capilarmente, se distribui por toda a sociedade. Nas palavras dele: “lá onde há poder, há resistência e, no
entanto (ou melhor, por si mesmo) esta nunca se encontra em posição de exterioridade em relação ao
poder”.

PRECONCEITO: é um pré-conceito uma opinião que se emite antecipadamente alimentada pelo


estereótipo, é um juízo preconcebido, manifestado geralmente na forma de uma atitude discriminatória
perante pessoas, lugares ou tradições considerados diferentes ou "estranhos".

RACISMO: conjunto de princípios que se baseia na superioridade de uma raça sobre a outra. A atitude
racista é aquela que atribui qualidades aos indivíduos conforme seu suposto pertencimento biológico a
uma determinada raça. Não é apenas uma reação ao outro, mas é uma forma de subordinação do outro.

SEXISMO: atitude preconceituosa que difere homens de mulheres definindo características


específicas para cada um, subordinando o feminino ao masculino.

SEXO BIOLÓGICO: é o conjunto de características fisiológicas, informações cromossômicas, órgãos


genitais, potencialidade individual para o exercício de qualquer função biológica que diferencia machos e
fêmeas. Entretanto, o sexo não é simplesmente algo que lhe foi dado pela biologia. Foucault analisa o
sexo biológico como um efeito discursivo. O poder cria o corpo ao anunciá-lo sexuado, ao fazer de sua
constituição biológica um fator natural que carrega características específicas e torna indiscutível a divisão
dos humanos em dois blocos distintos (homens e mulheres). Isto não significa que o corpo não exista de
forma sexuada. O que o poder cria é outra coisa: é a importância dada a esse fator corporal (biológico).
O sexo produz, interdita, possibilita e regula o corpo limitando certos tipos de escolhas para a produção
de um corpo sexuado que seja culturalmente aceitável e inteligível. Assim, o sexo é uma norma através
da qual alguém se torna viável.

SEXUALIDADE: É aprendida, ou melhor, é construída ao longo de toda a vida, de muitos e diferentes


modos, por todos os sujeitos por isso, é entendida como um conceito dinâmico que se modifica conforme
as posições do sujeito e suas disputas políticas. A sexualidade tem a ver tanto com o corpo, como também
com os rituais, o desejo, a fantasia, as palavras, as sensações, emoções, imagens e experiências. Ela
não tem ligação somente com a questão do sexo e dos atos sexuais, mas também com os prazeres e
sua relação com o corpo e a cultura compreendendo o erotismo, o desejo e o afeto; até questões relativas
a reprodução, saúde sexual, utilização de novas tecnologias.

TRANSEXUAL: pessoa que possui uma identidade de gênero diferente do sexo designado no
nascimento. Homens e mulheres transexuais podem manifestar o desejo de se submeterem a
intervenções médico-cirúrgicas para realizarem a adequação dos seus atributos físicos de nascença
(inclusive genitais) à sua identidade de gênero constituída.

TRANSFOBIA: termo usado para descrever vários fenômenos sociais relacionados ao preconceito, a
discriminação e à violência contra transexuais (ter desprezo, ódio, aversão ou medo de pessoas com
orientação sexual diferente do padrão heterossexual). Ver homofobia.

TRANSGÊNEROS OU TRANS: são termos utilizados para reunir, numa só categoria, travestis e
transexuais como sujeitos que realizam um trânsito entre um gênero e outro.

TRAVESTI: pessoa que nasce do sexo masculino ou feminino, mas que tem sua identidade de gênero
oposta a seu sexo biológico, assumindo papéis de gênero diferentes daquele imposto pela sociedade.
Muitas travestis modificam seus corpos através de hormonioterapias, aplicações de silicone e/ou cirurgias
plásticas, porém vale ressaltar que isso não é regra para todas (Definição adotada pelo Conferência
Nacional LGBT em 2008)

Apostila gerada especialmente para: Betânia Mueller 023.712.580-32


. 283
ORIENTAÇÃO SEXUAL: refere-se ao sexo das pessoas que elegemos para nos relacionar afetiva e
sexualmente. Atualmente temos três tipos de orientação sexual: heterossexual, homossexual e bissexual.
Contrapõem a OPÇÃO SEXUAL entendida como escolha deliberada e realizada de forma autônoma.

VIOLÊNCIA DE GÊNERO: É aquela oriunda do preconceito e da desigualdade entre homens e


mulheres e apoia-se no estigma da virilidade masculina (legítima defesa da honra) e da submissão
feminina.
Quando as vítimas são crianças e adolescentes o Art. 245 do ECA, obriga os profissionais da saúde e
educadores e educadoras a comunicarem o fato aos órgãos competentes. Na escola a discriminação é
manifestada por meio de apelidos, exclusões, perseguição, agressão física.

Questões

01. (SEDUC/SP - Conhecimentos Pedagógicos - FGV) Leia o fragmento a seguir. “Além das novas
demandas e dos entraves do cenário escolar e suas próprias condições de vida e de trabalho, o professor
ainda se depara com outras dificuldades que complicam a realização das intenções dos PCNs de ênfase
em parâmetros curriculares não tradicionais, como sexualidade e gênero”. (Abramovay et al., 2004)
Assinale a alternativa que apresenta a proposta que tem como objetivo mitigar o apresentado no
fragmento.
(A) Suspender a aplicação do tema transversal orientação sexual.
(B) Deixar o tema da sexualidade e da afetividade como responsabilidade exclusiva dos professores
da área de Biologia, já que configuram o “saber competente”.
(C) Capacitar os professores para lidar com o tema sexualidade.
(D) Delegar a responsabilidade pela orientação sexual aos movimentos sociais.
(E) Delegar a responsabilidade pela orientação sexual às famílias dos alunos.

02. (SEDUC/RJ - Conhecimentos Básicos - CEPERJ) Uma das questões formativas fundamentais
da vida humana, incorporadas pelos Parâmetros Curriculares Nacionais, é a orientação sexual. Segundo
os PCNs, as questões relativas à orientação sexual devem constituir:
(A) uma nova disciplina com horário específico de aulas na escola
(B) uma nova área de conhecimento a ser desenvolvida em interface com as agências de educação
permanente da sociedade
(C) uma área de conhecimento específica do ensino médio e tratada como disciplina
(D) um tema específico a ser tratado nas aulas de Biologia e Sociologia
(E) um tema transversal que permeia as diferentes disciplinas e áreas de conhecimento

03. (IF/PE - Assistente de alunos) Leia a seguinte sentença: “Temas como gênero, sexualidade e
diversidade sexual estão pautados dentro das políticas sociais e devem ser discutidos em diferentes
instâncias da sociedade”. A expressão “gênero”, na sentença transcrita, refere-se
(A) exclusivamente às características físicas e biológicas entre o corpo do homem e da mulher, do
menino e da menina.
(B) às diferenças entre o masculino e o feminino que foram construídas a partir do século XXI.
(C) às diferenças entre o masculino e o feminino que foram construídas exclusivamente depois dos
movimentos feministas.
(D) às diferenças entre o masculino e o feminino que foram construídas no decorrer da história da
humanidade por meio dos costumes, ideias, atitudes, crenças e regras criadas pela sociedade.
(E) exclusivamente às características físicas e biológicas entre o corpo do homem e da mulher, depois
que eles atingem a maturidade sexual.

04. (IF/PE - Assistente de Alunos) Leia a seguinte sentença: “Temas como gênero, sexualidade e
diversidade sexual estão pautados dentro das políticas sociais e devem ser discutidos em diferentes
instâncias da sociedade”. A expressão “gênero”, na sentença transcrita, refere-se
(A) exclusivamente às características físicas e biológicas entre o corpo do homem e da mulher, do
menino e da menina.
(B) às diferenças entre o masculino e o feminino que foram construídas a partir do século XXI.
(C) às diferenças entre o masculino e o feminino que foram construídas exclusivamente depois dos
movimentos feministas.
(D) às diferenças entre o masculino e o feminino que foram construídas no decorrer da história da
humanidade por meio dos costumes, ideias, atitudes, crenças e regras criadas pela sociedade.

Apostila gerada especialmente para: Betânia Mueller 023.712.580-32


. 284
(E) exclusivamente às características físicas e biológicas entre o corpo do homem e da mulher, depois
que eles atingem a maturidade sexual.

Gabarito

01.C / 02.E / 03.D / 04.D

Comentários

01. Resposta: C.
A letra A trata a suspensão do tema e essa não é uma pratica proposta uma vez que a orientação
sexual esta como tema transversal nos PCN desde 1995.
A letra B trata de deixar o tema como responsabilidade do professor de biologia, está errada pois como
tema transversal não é um saber competente de uma única disciplina.
A letra C está correta.
A letra D sugere delegar a responsabilidade pela orientação sexual aos movimentos sociais, está
errada pois os movimentos sociais podem até tratar o tema mas não deve ser colocado como
responsabilidade destes.
A letra E está errada uma vez que a responsabilidade pela orientação sexual não deve ser somente
da família mas também da escola.

02. Resposta: E.
O governo federal brasileiro, através do Ministério da Educação - MEC, em seus Parâmetros
Curriculares Nacionais (1997), estabelece a Orientação Sexual no Ensino Fundamental enquanto tema
transversal, isto é, um assunto a ser trabalhado em todas as disciplinas escolares, por quaisquer
professores que se sintam mobilizados, sempre que houver espaço na grade curricular ou em horários
extraclasses.

03. Resposta: D.
Ao falar em sexo referem-se às características físicas e biológicas de cada um, às diferenças entre um
corpo de homem e de mulher, de menino e de menina. Porém ao falar em gênero, trata-se às diferenças
que foram construídas ao longo da história da humanidade por meio dos costumes, ideias, atitudes,
crenças e regras criadas pela sociedade.

04. Resposta: D.
Quando falamos em sexo nos referimos às características físicas e biológicas de cada um, às
diferenças entre um corpo de homem e de mulher, de menino e de menina.
Mas, quando falamos em gênero, nos referimos às diferenças que foram construídas ao longo da
história da humanidade por meio dos costumes, ideias, atitudes, crenças e regras criadas pela sociedade.
Relações de Gênero: “...Saber a respeito das diferenças sexuais - histórica, social e culturalmente
construída. Portanto relativa, contextual, contestável e mutável. É um saber que atravessa todas as
relações que se constituem na sociedade, organizando as relações de poder a partir do significados que
cada sociedade atribui à diferença sexual”. É mais do que a maneira que as pessoas se relacionam é o
jeito de olhar e compreender a realidade.

Qualidade de vida na velhice.

Velhice36

A velhice é considerada a última fase do desenvolvimento humano e representa o conjunto de efeitos


de fatores biológicos, psicológicos e socioculturais. Dentre as inúmeras alterações correspondentes a
esta etapa estão as perdas fisiológicas, as modificações em algumas habilidades cognitivas, tais como
na memória de trabalho, na velocidade de pensamento e em habilidades visuoespaciais. O
envelhecimento psicológico tem relação com o esforço pessoal contínuo na busca do autoconhecimento

36
Revista Kairós Gerontologia, 15(3). Online ISSN 2176-901X - Print ISSN 1516-2567. São Paulo (SP), Brasil, 2012

Apostila gerada especialmente para: Betânia Mueller 023.712.580-32


. 285
e do sentido da vida, possibilitando uma redução da vulnerabilidade nesta fase (Moraes; Moraes & Lima,
2010).
O processo de envelhecer bem depende do equilíbrio entre as limitações do indivíduo e suas
potencialidades, o que possibilita o desenvolvimento de mecanismos para lidar com as perdas referentes
a este processo e uma adaptação às desvantagens e restrições (Resende, 2006). Para Neri (2001), as
caracterizações de mudanças evolutivas como ganhos e perdas, mocidade e velhice, são também
questões relativas a critérios subjetivos que envolvem anseios e valores, e consequentemente variam
segundo o contexto sociocultural. Destarte, a descrição de padrões de envelhecimento deve ser
entendida como uma orientação geral para observação de tendência, e não como o estabelecimento
inequívoco de categorias independentes. E os critérios adotados para o estabelecimento desses padrões
combinam longevidade, presença de patologias e conservação de mecanismos adaptativos do
organismo.
Nesse contexto de análise, Neri (2001) aborda diferentes padrões de envelhecimento: normal, ótimo e
patológico. Estes são processos inerentes ao organismo, pois estão relacionados a todas as modificações
que ocorrem nesta etapa do desenvolvimento (Silva, 2009). Entretanto, tal processo é experienciado de
maneira distinta e particular, por ser altamente individualizado, além de ser influenciado por fatores
intrínsecos e extrínsecos ao indivíduo.
As ocorrências das alterações vivenciadas na velhice fizeram emergir inúmeros estereótipos e ideias
preconcebidas acerca do declínio funcional e das modificações biopsicossociais que ocorrem nesse
período. Nesse sentido, o envelhecer e ser idoso estão ligados a uma denominação depreciativa, o que
pode propiciar, a partir desta visão, o aparecimento de crenças negativas a respeito dessa fase da vida
(Neri & Resende, 2009).
Segundo Neri (2006), os estereótipos são elementos de organização do ser humano que ajudam o
indivíduo a compreender o mundo. Eles podem, porém, nos levar a uma negligência ou minimização das
diferenças individuais, como, por exemplo, a qualificação feita aos idosos como antiquados, implicantes,
dependentes ou sábios. Assim, os preconceitos podem ser uma forma de supersimplificação, isto é,
algumas características de um determinado grupo são selecionadas e realçadas, definindo seus
componentes de uma forma geral.
A formação de estereótipos negativos, atitudes e preconceitos em relação ao idoso foram influenciados
pelos cientistas, principalmente pelo modelo biomédico, que destaca os processos biofisiológicos e a
funcionalidade física e mental. Algumas teorias sociológicas também focalizam a velhice como problema,
enfatizando o afastamento recíproco entre idoso e sociedade. Desse modo, políticas designadas para os
idosos baseadas em crenças inadequadas intensificam os preconceitos (Neri, 2006).
Os estereótipos e preconceitos vigentes na sociedade são reflexos da supervalorização na cultura
ocidental da beleza, juventude, independência e a habilidade de ser produtivo, o que provém do
dominante modelo econômico capitalista. Porém, a representação de velhice associada às ideias de
deterioração e perda não é unânime. À medida que o envelhecimento é documentado em outros povos,
constata-se que ele é um processo fortemente influenciado pela cultura (Uchôa, 2003).
De acordo com Martins e Rodrigues (2004), a valorização dos estereótipos projeta sobre a velhice
uma representação social gerontofóbica e contribui para a imagem que os idosos têm de si próprios, bem
como das condições e circunstâncias que envolvem a velhice. Tal conjuntura gera uma perturbação, visto
que nega o processo de crescimento dos idosos e os impedem de reconhecer as suas potencialidades,
de procurar soluções precisas para os seus problemas e de encontrar medidas adequadas.
Assim, pode-se dizer que existe uma correspondência entre a concepção de velhice presente em uma
sociedade e as atitudes frente às pessoas que estão envelhecendo (Irigaray & Schneider, 2008). Sendo
as crenças existentes na atual sociedade prejudiciais e distorcidas, isso pode afetar a representação
desta fase do desenvolvimento e influenciar diretamente na forma que os idosos vivenciam este momento,
na sua autoestima, autoimagem, na aceitação das próprias limitações e no seu bem-estar psicológico.
Por outro lado, é possível uma ressignificação desse processo do desenvolvimento, possibilitando que a
velhice seja vista como um processo peculiar ao indivíduo, em consonância com sua história de vida e
com a representação da velhice presente na sociedade, de forma que se considerem também as
experiências adquiridas pelos mesmos, suas potencialidades, contribuições e acréscimos ao longo da
vida.

Atitudes e Crenças em relação à velhice


O conceito de atitude faz parte de um campo em que se configuram também as noções de crenças,
preconceitos, estereótipos, valores e ideologia (Todaro, 2008). Destarte, as atitudes são aprendidas no
processo de socialização, no ambiente em que estão inseridas; podem ser organizadas em sistemas e

Apostila gerada especialmente para: Betânia Mueller 023.712.580-32


. 286
sujeitas a avaliação. Conforme Rodrigues, Assmar e Jablonski (2007), as atitudes envolvem o que as
pessoas pensam, sentem, e como elas gostariam de se comportar em relação a um objeto atitudinal.
Os gerontólogos sociais sustentam que atitudes preconceituosas em relação à velhice determinam
práticas sociais que contribuem para a manutenção de ideias discriminativas, preconceituosas e
paternalistas. Tais fenômenos influenciam nas atitudes dos idosos perante a velhice, visto que as atitudes
da sociedade com relação ao idoso interferem na sua percepção, e determinam as lentes pelas quais se
vê a velhice (Neri & Resende, 2009).
Segundo Neri (2005), a vitimização do idoso acaba gerando uma superproteção que prejudica sua
autonomia e independência, como também os preconceitos inibem a sua participação de forma mais ativa
na sociedade. A mesma autora comenta que algumas políticas em relação ao idoso são inapropriadas,
pois criam generalizações extremadas de características desejáveis ou indesejáveis compartilhadas por
todos inclusive pelos próprios idosos como, por exemplo, a dependência física, depressão, doenças ou
visão dos idosos como generosos e sábios.
De acordo com Couto (2005), as atitudes de preconceito e discriminação contra o idoso ou ageísmo37
podem ser prejudiciais principalmente no âmbito das relações interpessoais. Neste sentido, os aspectos
mais atingidos são a percepção que o idoso tem de si e o sentimento de segurança na comunidade em
que está inserido. Isso indica que experiências de discriminação ocorridas face a face podem fazer com
que o idoso se isole ou, até mesmo, diminua sua vontade de viver.
Há diversas definições para atitude e elas são socialmente aprendidas devido à influência de
interações entre o indivíduo, grupos e instituições sociais (Neri, 2006). A partir das esquematizações das
diversas definições de atitudes são considerados três elementos para sua constituição: componente
cognitivo, afetivo (pró ou contra), e uma predisposição comportamental. Uma atitude social é, portanto,
segundo Rodrigues, Assmar e Jablonski (2007), uma organização de crenças e cognições em geral, com
uma carga afetiva a favor ou contra um objeto social definido, que propicia uma ação coerente com as
cognições e afetos relativos a este objeto.
Para a psicologia social, diferentemente do que se pensa no senso comum, as atitudes não são
tomadas (ação, comportamento), mas sim desenvolvidas (crenças, valores) em relação aos objetos do
meio social. Sendo assim, a análise de atitudes e preferências é indispensável para compreensão dos
comportamentos, já que, os componentes da atitude (afeto, ação, informação) tendem a ser congruentes,
promovendo certa regularidade em relação ao meio (Rodrigues, Assmar & Jablonski, 2007).
Pesquisa realizada por Neri (1996) com adultos e idosos participantes de Universidades de Terceira
Idade, mostrou que as atitudes em relação ao idoso são positivas e as expectativas em relação à própria
velhice ainda mais positivas, independentemente da idade, do gênero, da escolaridade e da região de
residência dos sujeitos. O estudo de Silva (1999) realizado com 100 idosos, participantes de um programa
educacional, verificou que 41,05% da amostra apresentava uma opinião positiva sobre o que é ser velho,
20% opiniões negativas e 38,95% opiniões neutras.
De acordo com Neri, Cachioni e Resende (2002), os adultos maduros e os idosos mostram atitudes e
crenças positivas em relação à velhice. Essas atitudes positivas contribuem para a adaptação às
incapacidades e perdas presentes na velhice, funcionando como um recurso de enfrentamento que
atenua a adversidade de fatores estressantes que, como consequência, possibilita um senso de
ajustamento pessoal ou bem-estar psicológico positivo.

Bem-estar psicológico ou ajustamento pessoal


O bem-estar psicológico pode ser entendido como a percepção da própria pessoa acerca do
ajustamento emocional e social em relação aos desafios da vida. Com base em pesquisas e inúmeros
estudos, Ryff (1989) propôs um modelo multidimensional de bem-estar psicológico composto por seis
dimensões, a saber: autoaceitação, que se refere a ter atitudes positivas em relação a si e à vida; relação
positiva com outros, definida como uma relação satisfatória com os outros; autonomia, que diz respeito à
resistência às pressões sociais; e avaliação de si com base nos próprios padrões; domínio sobre o
ambiente, que se relaciona com a habilidade de escolher ou controlar ambientes apropriados à condição
física; propósito na vida, que diz respeito ao senso de significado da vida; e crescimento pessoal, que
está associado ao senso de realização e desenvolvimento do próprio potencial (Rabelo & Neri, 2006).
Ainda de acordo com esse modelo, pessoas ajustadas têm alto senso de desenvolvimento e
crescimento contínuo; estão abertas a novas experiências; reconhecem seu potencial de realização, e
suas mudanças refletem autoconhecimento e autoeficácia. Conforme Queroz (2003, 2008), o conceito de
bem-estar psicológico corresponde a uma busca de ideal de excelência, de crescimento pessoal ou de
autorrealização e está associado ao ajustamento emocional e social, na medida em que reúne o

37
Ageísmo: discriminação relacionada à idade.

Apostila gerada especialmente para: Betânia Mueller 023.712.580-32


. 287
cumprimento de expectativas sociais e emocionais, considerando seus atributos físicos, cognitivos e
afetivos, idade e gênero.
O envelhecimento satisfatório depende do equilíbrio entre as potencialidades e as limitações do
indivíduo. As necessidades de autoatualização e de crescimento pessoal não declinam com a idade, mas
podem ser afetadas quando o indivíduo enfrenta limitações de atividades e de capacidades em múltiplos
domínios e quando enfrenta estresse psicológico. Além disso, indivíduos bem-ajustados
psicologicamente têm uma melhor habilidade para adaptar-se às demandas e dificuldades em termos
sociais e emocionais (Neri & Resende, 2009).
O senso de ajustamento psicológico é constituído a partir da interação entre as oportunidades, as
condições vividas pelo indivíduo (saúde, trabalho, moradia, educação), a maneira pessoal como cada um
organiza seu conhecimento e responde às necessidades individuais, sociais e às demandas ambientais.
Assim sendo, as experiências vividas pelo indivíduo no contexto social, profissional e emocional
influenciam no bem-estar psicológico (Queroz, 2003).
Nesse sentido, os preconceitos e estereótipos em relação à velhice influenciam nas crenças que os
idosos têm sobre o próprio processo de envelhecimento, nas suas atitudes e consequentemente pode
interferir no bem-estar dos mesmos. A questão central deste estudo é verificar se os idosos com atitudes
positivas em relação à velhice também apresentam um senso positivo de bem-estar psicológico. Supomos
que tais atitudes em relação à velhice funcionam como um recurso de enfrentamento diante das perdas
e incapacidades dessa fase, favorecendo um senso positivo de ajustamento psicológico.
Levando em conta que a pesquisa sobre atitudes ocupa-se em identificar de que maneira as crenças
e os mitos sobre um determinado objeto social influenciam a forma como ele é tratado, a investigação
das atitudes de idosos em relação à velhice se faz importante, inclusive, considerando a possibilidade de
aplicações práticas a nível educacional – visto que, a educação pode ser uma forte ferramenta na
mudança desse panorama que desprivilegia o idoso. Tais ideias que existem sobre o envelhecimento
podem tanto contribuir para a perpetuação de preconceitos e estereótipos negativos quanto para a
promoção de melhorias no que tange à saúde e qualidade de vida dos idosos.
Nesse contexto de análise, o presente trabalho busca incentivar a elaboração de políticas educativas
na comunidade que visem a abordar tais assuntos para que a desconstrução de preconceitos e
estereótipos negativos seja facilitada e os idosos passem a ter atitudes mais realistas para com essa fase
da vida, utilizando-as como um mecanismo de enfrentamento.

Desenvolvimento humano: fases, influências.

No século XXI psicólogos do desenvolvimento enfrentam novos desafios uma vez que as novas
concepções de atuação profissional que enfatizam a prevenção e a promoção de saúde fazem com que
profissionais de várias áreas busquem na psicologia do desenvolvimento subsídios teóricos e
metodológicos para sua prática profissional. O que está em questão é o desenvolvimento harmônico do
indivíduo, que integra não apenas um aspecto, mas todas as dimensões do desenvolvimento humano
sejam elas: biológicas, cognitivas, afetivas ou sociais. 38

A delimitação conceitual do campo da Psicologia do Desenvolvimento

O desenvolvimento humano envolve o estudo de variáveis afetivas, cognitivas, sociais e biológicas em


todo ciclo da vida. Desta forma faz interface com diversas áreas do conhecimento como: a biologia,
antropologia, sociologia, educação, medicina entre outras.
Tradicionalmente o estudo do desenvolvimento humano focou o estudo da criança e do adolescente,
ainda hoje muitos dos manuais de psicologia do desenvolvimento abordam apenas esta etapa da vida
dos indivíduos.
O interesse pelos anos iniciais de vida dos indivíduos tem origem na história do estudo científico do
desenvolvimento humano, que se inicia com a preocupação com os cuidados e com a educação das
crianças, e com o próprio conceito de infância como um período particular do desenvolvimento.
No entanto, este enfoque vem mudando nas últimas décadas, e hoje há um consenso de que a
psicologia do desenvolvimento humano deve focar o desenvolvimento dos indivíduos ao longo de todo o
ciclo vital. Ao ampliar o escopo de estudo do desenvolvimento humano, para além da infância e

38
Texto adaptado de Márcia Elia da Mota, disponível em http://pepsic.bvsalud.org/

Apostila gerada especialmente para: Betânia Mueller 023.712.580-32


. 288
adolescência, a psicologia do desenvolvimento acaba por fazer interface também com outras áreas da
psicologia. Só para citar algumas áreas temos: a psicologia social, personalidade, educacional, cognitiva.
Assim surge a necessidade de se delimitar esse campo de atuação, definindo o que há de específico
na psicologia do desenvolvimento humano. A necessidade de se integrar ao estudo do desenvolvimento
humano uma perspectiva interdisciplinar, que adote uma metodologia de pesquisa própria, faz com que
alguns autores sugiram que o estudo desenvolvimento humano constitua um campo de atuação
independente da Psicologia, que tem sido chamado de “Ciência do Desenvolvimento Humano”.
Pesquisadores do desenvolvimento humano concordam que um dos objetos de estudo do psicólogo
do desenvolvimento é o estudo das mudanças que ocorrem na vida dos indivíduos. Papalia e Olds, por
exemplo, definem desenvolvimento como “o estudo científico de como as pessoas mudam ou como elas
ficam iguais, desde a concepção até a morte”.
A definição destes autores salienta o fato de que psicólogos do desenvolvimento estudam as
mudanças, mas não nos oferece nenhuma informação sobre questões fundamentais ao estudo do
desenvolvimento humano. O que muda? Como muda? E quando muda? Estas são perguntas frequentes
nas pesquisas sobre o desenvolvimento, e são frequentemente abordadas de forma distintas pelas
diferentes abordagens teóricas que descrevem o desenvolvimento humano.
Dizer que ao longo do tempo mudanças ocorrem na vida dos indivíduos não nos esclarece estas
questões. O tempo é apenas uma escala, não é uma variável psicológica. Portanto, é preciso entender
como as condições internas e externas ao indivíduo afetam e promovem essas mudanças. As mudanças
no desenvolvimento são adaptativas, sistemáticas e organizadas, e refletem essas situações internas e
externas ao indivíduo que tem que se adaptar a um mundo em que as mudanças são constantes.
Variáveis internas podem ser entendidas como aquelas ligadas à maturação orgânica do indivíduo, as
bases genéticas do desenvolvimento.
Recentemente, os processos inatos que promovem o desenvolvimento humano voltam a ser discutidos
por teóricos do desenvolvimento humano.
As variáveis externas são aquelas ligadas à influência do ambiente no desenvolvimento. As
abordagens sistêmicas de investigação do desenvolvimento humano há muito chamam atenção para a
importância de se entender as diversas interações que ocorrem nos múltiplos contextos em que o
desenvolvimento se dá. Incluindo-se nesta discussão uma análise do momento histórico em que o
indivíduo se desenvolve.
Biaggio argumenta que a especificidade da psicologia do desenvolvimento humano está em estudar
as variáveis externas e internas aos indivíduos que levam as mudanças no comportamento em períodos
de transição rápida (infância, adolescência e envelhecimento). Teorias contemporâneas do
desenvolvimento aceitam que as mudanças são mais marcadas em períodos de transição rápida, mas
mudanças ocorrem ao longo de toda a vida do indivíduo, não só nestes períodos. Portanto, é preciso se
ampliar o escopo do entendimento do que é o estudo do desenvolvimento humano.
Para que se leve a termo estas considerações, as pesquisas em desenvolvimento humano utilizam
metodologia específica, entre elas a mais comumente usada são os estudos longitudinais. A “International
Society for the Study of Behavioral Development” lançou em 2005 uma edição especial intitulada
“Longitudinal Research on Human Development: Approachs, Issues and New Directions”. Nesta edição
se discute as contribuições e limitações dos estudos longitudinais para a produção do conhecimento na
psicologia do desenvolvimento.
Cillessen ressalta que estudos longitudinais se aplicam as várias áreas do conhecimento não apenas
a Psicologia do Desenvolvimento. Também não se aplicam apenas a estudos de longo prazo e com
muitos indivíduos, mas na psicologia do desenvolvimento adquirem uma importância fundamental, pois
permitem que se acompanhe o desenvolvimento dos indivíduos ao longo do tempo, ao mesmo tempo em
que, controlam-se as múltiplas variáveis que afetam o desenvolvimento.
Os teóricos que trabalham na abordagem do Curso da Vida, chamam atenção para algumas das
limitações deste tipo de abordagem, que estudam apenas uma coorte de cada vez, não permitindo
inferências sobre o comportamento entre gerações. Apontam para a necessidade de incluir outras coortes
históricas em estudos sobre o desenvolvimento humano, ressaltando a necessidade de estudos
longitudinais de coorte, mais amplos que os estudos longitudinais tradicionais.
Além da Teoria do Curso da Vida, teóricos de diversas abordagens chamam a atenção para a
necessidade de se considerar as questões metodológicas específicas ao estudo do desenvolvimento e
as limitações das metodologias tradicionais Assim, pelas questões acima citadas, consideramos que uma
melhor definição de Psicologia do Desenvolvimento seria “O estudo, através de metodologia específica e
levando em consideração o contexto sócio histórico, das múltiplas variáveis, sejam elas cognitivas,
afetivas, biológicas ou sociais, internas ou externas ao indivíduo que afetam o desenvolvimento humano
ao longo da vida”.

Apostila gerada especialmente para: Betânia Mueller 023.712.580-32


. 289
Através da identificação dos fatores que afetam o desenvolvimento humano podemos pensar sobre
trabalhos de intervenção mais eficazes, que levem a um desenvolvimento harmônico do indivíduo. Sendo
assim, os conhecimentos gerados por essa área da psicologia trazem grandes contribuições para os
trabalhos de prevenção e promoção de saúde. Aqui a concepção de saúde adquire uma perspectiva mais
ampla e engloba os diversos contextos que fazem parte da vida dos indivíduos (escola, trabalho, família).
O desenvolvimento humano39 se realiza em períodos que se distinguem entre si pelo predomínio de
estratégias e possibilidades específicas de ação, interação e aprendizagem.
Os períodos de desenvolvimento são, normalmente, referidos como infância, adolescência, maturidade
e velhice. É mais adequado, porém, pensarmos o processo de desenvolvimento humano em termos das
transformações sucessivas que o caracterizam, transformações que são marcadas pela evolução
biológica (que é constante para todos os seres humanos) e pela vivência cultural.

Plasticidade Cerebral

O cérebro humano apresenta uma grande plasticidade. Plasticidade é a possibilidade de formação de


conexões entre neurônios a partir das sinapses. A plasticidade se mantém pela vida toda, embora sua
amplitude varie segundo o período de formação humana. Assim é que, quanto mais novo o ser humano,
maior plasticidade apresenta. Certas conexões se fazem com uma rapidez muito grande na criança
pequena. É isto que possibilita o desenvolvimento da linguagem oral, a aprendizagem de uma ou mais
línguas maternas simultaneamente, o domínio de um instrumento musical, o desenvolvimento dos
movimentos complexos e a perícia de alguns deles, como aqueles envolvidos no ato de desenhar, de
correr, de nadar...
Consequentemente a infância é o período de maior plasticidade e isto atende, naturalmente, ao
processo intenso de crescimento e desenvolvimento que ocorre neste período. Assim, a plasticidade
atende às necessidades da espécie.
Que possibilidades concretas são estas de formação de conexões? O cérebro humano dispõe de
cerca de 100 bilhões de neurônios, sendo que cada um pode chegar a estabelecer cerca de 1000
sinapses, em certas circunstâncias ainda mais. Desta forma, as possibilidades são de trilhões de
conexões, o que significa que a capacidade de aprender de cada um de nós é absolutamente muito
ampla.
Enquanto espécie, o ser humano apresenta, desde o nascimento, uma plasticidade muito grande no
cérebro, podendo desenvolver várias formas de comportamento, aprender várias línguas, utilizar
diferentes recursos e estratégias para se inserir no meio, agir sobre ele, avaliar, tomar decisões, defender-
se, criar condições de sobrevivência ao longo de sua vida.
A plasticidade cerebral também permite que áreas do cérebro destinadas a uma função específica
possam assumir outras funções, como, por exemplo, o córtex visual no caso das crianças que nascem
cegas. Como esta parte do cérebro não será “chamada a funcionar”, pois o aparelho da visão apresenta
impedimentos (então não manda informação a partir da percepção visual para o cérebro), ela poderá
assumir outras funções.
Plasticidade cerebral é, também, a possibilidade de realizar a “interdisciplinaridade” do cérebro: áreas
desenvolvidas por meio de um tipo de atividade podem ser “aproveitadas” para aprender outros
conhecimentos ou desenvolver áreas relativas a outro tipo de atividade. Por exemplo, áreas
desenvolvidas pela música, como a de ritmo, são “aproveitadas” no ato da leitura da escrita ou a de
divisão do tempo na aprendizagem de matemática.

A ação da criança depende da maturação orgânica e das possibilidades que o meio lhe oferece: ela
não poderá realizar uma ação para a qual não tenha o substrato orgânico, assim como não fará muitas
delas, mesmo que biologicamente apta, se a organização do seu meio físico e social não propiciar sua
realização ou se os adultos não a ensinarem.
O ser humano aprende somente as formas de ação que existirem em seu meio, assim como ele
aprende somente a língua ou as línguas que aí forem faladas. As estratégias de ação e os padrões de
interação entre as pessoas são definidos pelas práticas culturais.
Isto significa que a cultura é constitutiva dos processos de desenvolvimento e de aprendizagem.
A criança se constitui enquanto membro do grupo por meio da formação de sua identidade cultural,
que possibilita a convivência e sua permanência no grupo. Simultaneamente ela constitui sua
personalidade que a caracterizará como indivíduo único.

39
http://portal.mec.gov.br/seb/arquivos/pdf/Ensfund/indag1.pdf

Apostila gerada especialmente para: Betânia Mueller 023.712.580-32


. 290
Os comportamentos e ações privilegiados em cada cultura são, então, determinantes no processo de
desenvolvimento da criança.
A vida no coletivo sempre envolve a cultura: as brincadeiras, o faz de conta, as festas, os rituais, as
celebrações são todas situações em que a criança se constitui como ser de cultura.

Desenvolvimento cultural

O desenvolvimento tecnológico e o processo de globalização da informação por meio da imagem


modificaram os processos de desenvolvimento cultural por introduzirem novas formas de mediação. As
novas gerações desenvolvem-se com diferenças importantes em relação às gerações precedentes, por
meio, por exemplo, da interação com a informática, com as imagens presentes por meio urbano (várias
formas de propaganda, como cartazes, outdoors móveis). O mesmo acontece com crianças nas zonas
rurais com o advento da eletricidade e da TV, ou com crianças indígenas que passaram a experienciar o
processo de escolarização e, também, em vários casos a presença de novos instrumentos culturais como
o rádio, a TV, câmeras de vídeo, fotografia, entre outros.
O desenvolvimento do cérebro é função da cultura e dos objetos culturais existentes em um
determinado período histórico. Novos instrumentos culturais levam a novos caminhos de
desenvolvimento. O computador é um bom exemplo: modificou as formas de lidar com informações,
provocando mudanças nos caminhos da memória. A presença de novos elementos imagéticos e
cinestésicos repercute no desenvolvimento de funções psicológicas como a atenção e a imaginação.
Considerando, então, que o cérebro se desenvolve do diálogo entre a biologia da espécie e a cultura,
verifica-se que, na escola, o currículo é um fator que interfere no desenvolvimento da pessoa.
Os “conteúdos” escolhidos para o currículo irão, sem dúvida, ter um papel importante na formação. As
atividades para conduzirem às aprendizagens, precisam estar adequadas às estratégias de
desenvolvimento próprias de cada idade. Em outras palavras, a realização do currículo precisa mobilizar
algumas funções centrais do desenvolvimento humano, como a função simbólica, a percepção, a
memória, a atenção e a imaginação.

Linguagem e Imagens mentais: Percepção, Memória e Imaginação Desenvolvimento da Função


Simbólica

A partir da sua ação e interação com o mundo (a natureza, as pessoas, os objetos) e das práticas
culturais, a criança constitui o que chamamos de função simbólica, ou seja, a possibilidade de representar,
mentalmente, por símbolos o que ela experiencia, sensivelmente, no real.
O desenvolvimento da função simbólica no ser humano é de extrema importância, uma vez que é por
meio do exercício desta função que o ser humano pode construir significados e acumular conhecimentos.
Todo ensino na escola, de qualquer área do conhecimento, implica na utilização da função simbólica.
As atividades que concorrem para a formação da função simbólica variam conforme o período de
desenvolvimento. Por exemplo, o desenho e a brincadeira de faz-de-conta são atividades simbólicas
próprias da criança pequena, que antecedem a escrita: na verdade, elas criam as condições internas para
que a criança aprenda a ler e a escrever.
A linguagem escrita, a matemática, a química, a física, o sistema de notação da dança, da música são
manifestações da função simbólica. As aprendizagens escolares são apropriações de conhecimentos
formais, ou seja, conhecimentos organizados em sistemas. Sistematizar é estabelecer conceitos, ordená-
los em níveis de complexidade com regras internas que regulam a relação entre os elementos que os
compõem. Todo conhecimento formal é representado, simbolicamente, pela linguagem de cada sistema.
Por exemplo:
a) a2 = b2 + c2
b) 15 + 36 = 51
c) O gato correu atrás do cachorro.
O cachorro correu atrás do gato

Em b e c temos uma regra importante que é o valor posicional: a posição dos elementos simbólicos
determina o significado (1 e 5) 15 é diferente de 51. O mesmo se aplica ao gato que corre atrás do
cachorro, em que se explicita a ação inversa do cachorro que corre atrás do gato.
A função simbólica é a atividade mais básica das ações que acontecem na escola, tanto do educador
como do educando. Quando os elementos do currículo não mobilizam adequadamente o exercício desta
função, a aprendizagem não se efetua.
Apostila gerada especialmente para: Betânia Mueller 023.712.580-32
. 291
Nesta dimensão do simbólico, as artes destacam-se, pois são elas as formas mais complexas de
atividade simbólica humana. Anteriores aos conhecimentos formais, elas propiciaram a estruturação dos
movimentos e das imagens de forma que eles pudessem evoluir culturalmente para sistemas de registros.

Percepção
A percepção é realizada pelos cinco sentidos externos. O ser humano desenvolve estes sentidos
desde que não haja impedimentos nos órgãos dos sentidos ou nas estruturas cerebrais que processam
a percepção de cada um deles. Quando isto acontece, um sentido “compensa” o outro: a pessoa
desenvolve mais o tato quando não enxerga, desenvolve mais a visão quando não ouve. Nestes casos,
também, o ser humano pode desenvolver os dois subsentidos externos que são a vibração e o calor.
Isso revela que os sentidos funcionam com interdependência, o que tem uma relevância fundamental
para os professores, pois o ensino deve mobilizar várias dimensões da percepção para que o aluno possa
“guardar” conteúdos na memória de longa duração.
Há maior empenho em perceber algo quando há algum interesse neste “algo”. Por exemplo, quando
alguém ouve uma música de um cantor de quem gosta muito, fica atento e evoca a melodia ou a letra.
Se for uma canção nova e se reconhece a voz do cantor, mobiliza os processos mentais da memória
auditiva a partir da percepção auditiva, ou seja, seleciona a canção, destacando-a das outras informações
sonoras e/ou ruídos presentes no ambiente.
Por outro lado, a percepção pode criar um interesse novo. Ao ser introduzida a um conhecimento novo,
uma pessoa pode se interessar ou não por ele, dependendo das estratégias utilizadas por quem o
introduz. Assim, em sala de aula, não é somente o conteúdo que motiva, mas, sobretudo, como o
professor trabalha com o conteúdo, seja ele da escrita, artes ou ciências.
A percepção visual é o processamento de atributos do objeto como cor, forma e tamanho. Ela acontece
em regiões do córtex cerebral e há fortes indicações de que estas regiões sejam as mesmas ou estejam
muito próximas daquelas que “guardariam” a memória dos objetos. Desta forma, percepção e memória
estão muito próximas nas aprendizagens escolares.

Memória
Toda aprendizagem envolve a memória. Todo ser humano tem memória e utiliza seus conteúdos a
todo o momento. São três os movimentos da memória: o de arquivar, o de evocar e o de esquecer. Ao
entrar em contato com algo novo, o ser humano pode criar novas memórias, ou seja, arquiva este
conhecimento, experiência ou ideia em sua memória de longa duração. As impressões gravadas na
memória de longa duração, a partir das experiências vividas, podem ser “evocadas”, trazidas à
consciência. Outras experiências, informações, vivências, imagens e ideias são esquecidas.
Sabemos que estes movimentos têm uma participação do sistema límbico no qual se originam nossas
emoções. A memória é modulada pela emoção. Isto quer dizer que os estados emocionais podem
“interferir”, facilitando ou reforçando a formação de novas memórias, assim como podem, também,
enfraquecer ou dificultar a formação de uma nova memória.
Quanto ao tempo, os tipos de memória são muito importantes para o educador, pois as aprendizagens
escolares dependem da formação de novas memórias de longa duração. Muitas vezes, no entanto, os
conteúdos ficam no nível da curta duração e desaparecem rapidamente. O desafio da pedagogia é
formular metodologias de ensino que transformem esta primeira ação da memória (curta duração) em
memórias de longa duração. É importante mencionar aqui que temos, também, a possibilidade de formar
uma memória ultrarrápida que desaparece após a sua utilização, como quando, por exemplo, gravamos
um número de telefone para discá-lo e, logo em seguida, já o esquecemos.
Quanto à natureza, temos vários tipos de memória. Temos a memória implícita, a memória explícita e
a operacional. A memória explícita pode ser semântica ou episódica.
Para as aprendizagens escolares, precisam ser mobilizadas a memória explicita semântica e a
memória operacional.
Para a formação de novas memórias dos conteúdos escolares ao aluno precisa, desde o início da
escolarização, ser ensinado o que fazer e como para aprender os conhecimentos envolvidos nas
aprendizagens escolares. O aluno precisa ser capaz de “refazer” o processo da aprendizagem. Refazer
implica tanto em recapitular o conteúdo ensinado, como em retomar as atividades (humanas) que o
levaram a “guardar” o conteúdo na memória de longa duração.

Memória explícita semântica


Também chamada de declarativa, a memória explícita semântica inclui as memórias que podem ser
explicitadas pela linguagem. Este tipo de memória engloba aquilo que pode ser lembrado por meio das

Apostila gerada especialmente para: Betânia Mueller 023.712.580-32


. 292
imagens, símbolos ou sistemas simbólicos. A capacidade da memória declarativa está ligada à
organização de informações em padrão.
Pesquisas demonstram que o ser humano se lembra “mais facilmente” daquilo que está organizado
segundo regras. Isto implica na existência de padrões internos. Todas as linguagens são organizadas por
padrões: a linguagem das ciências, das várias áreas do conhecimento, a linguagem escrita, a matemática,
a cartográfica, a linguagem da dança, da música. Toda atividade artística também depende de utilização
de elementos que se organizam em padrões, que têm regras próprias em cada forma de arte.
Na escrita, os padrões aparecem nas cinco dimensões da linguagem, embora apareçam, mais
fortemente, na sintaxe. Por isto, a sintaxe é o elemento forte, o instrumentador da língua escrita. A palavra
solta é um símbolo, a palavra na construção sintática surge como estrutura. Na linguagem oral humana,
o eixo forte do padrão é o verbo. Há maior resiliência no cérebro para os símbolos que representam a
ação humana, uma vez que o movimento é o grande recurso na espécie para o desenvolvimento cultural
e tecnológico, além de ser a matéria bruta primeira da comunicação entre humanos e de expressão das
emoções.
As pessoas tendem a memorizar, mais facilmente, aquilo em que elas conseguem aplicar padrões.
Para as aprendizagens escolares isto é fundamental: o ensino bem sucedido é aquele que
“instrumentaliza” a pessoa para construir, aplicar, reconhecer e “manipular” padrões.

Memória operacional
Como o próprio nome diz, a memória operacional se ocupa das operações, ou seja, um sistema de
ações organizadas, segundo a natureza do comportamento. Por exemplo, está na memória operacional
o comportamento de andar, de dirigir, de dançar. São comportamentos que se efetuam, muito
rapidamente, para os quais não há “tempo” para comandos do cérebro. São comportamentos que têm
uma ordem de movimentos a ser seguida e esta ordem já está “fixada” na memória.
Na memória operacional estão as conjugações verbais, isto é, os tempos futuro, presente e passado
do verbo. Assim, a organização da ação no tempo se realiza com a participação deste tipo de memória.
Este fato tem implicações para as aprendizagens escolares. Com estas descobertas somos levados a
rever o ensino da sintaxe em português: a gramática é necessária para o aluno, pois fornece estrutura
para a apropriação e organização da linguagem escrita e a organização das informações em todas as
matérias.

Imaginação - A Capacidade Imaginativa na Espécie Humana


Se considerarmos a evolução de nossa espécie, veremos que ela é pautada pela invenção, ou seja,
pela criação de objetos, de sistemas, de linguagens, tecnologia, teorias, ciência, arte, códigos etc. Toda
produção cultural é resultante de um processo cumulativo de invenções, pequenas e grandes, que dão
base para as invenções futuras.
A comunicação, atividade primordial da espécie, ganha a cada geração novos processos, novas
tecnologias. O ser humano dedica grande parte de sua criatividade a ampliar e desenvolver meios de
comunicação e meios de transporte que facilitem os processos comunicativos e que tornem mais ágeis
os deslocamentos das pessoas.
A possibilidade de criar depende, na nossa espécie, da imaginação, função psicológica pela qual nós
somos capazes de unir elementos percebidos e experiências em novas redes de conexão. O
funcionamento da imaginação e seu desenvolvimento, embora relacionados às outras funções
psicológicas superiores, têm uma grande autonomia e se manifestam tanto na ação como no ato de
aprender.
Desta forma, podemos dizer que para as aprendizagens escolares a imaginação desempenha um
papel central e deve ser considerada no planejamento, na alocação de tempo das atividades dentro e
fora da sala de aula, nas situações comuns do cotidiano escolar. Os alunos devem, também, ser
acompanhados avaliativamente na evolução de sua imaginação.

A ligação entre imaginação e memória


Vygotsky trata da diferença entre reprodução e criação: ambas atividades têm apoio na memória, mas
diferem pelo alcance temporal. Reproduzir algo, mentalmente, se apoia na experiência sensível anterior.
Por exemplo, construo uma imagem mental da casa onde moravam meus avós, pelos elementos
gravados na memória, mas crio uma imagem mental da casa dos avós de uma personagem em um
romance a partir dos elementos oferecidos pelo autor. Ou seja, no segundo caso uso a imaginação para
criar este espaço utilizando, com certeza, elementos percebidos, anteriormente, mas que se combinam
entre si, de acordo com a relação dialógica estabelecida com o texto no ato da leitura, diferentemente, do
primeiro caso em que busco a fidedignidade da imagem mental, tendo a casa concreta como referencial.

Apostila gerada especialmente para: Betânia Mueller 023.712.580-32


. 293
Vygotsky coloca que a primeira experiência se apoia na análise do passado. Ela é uma reprodução do
que se viveu, enquanto que no segundo é uma realização do presente projetada no futuro. A criação
literária dá esta possibilidade de partilhamento na criação, pois possibilita ao leitor a superação do texto
para a criação das imagens de cada personagem, que é constituída pelos dados oferecidos de sua
personalidade, de suas ações, de suas formas de pensamento, criação de imagens do contexto.
A imaginação na realidade não se “desprega” da memória, mas recria com os elementos da memória.
Imaginar implica, portanto, em se liberar das conexões que estão feitas dos elementos percebidos, para
“reutilizar” estes elementos em outras configurações.
Temos aí duas implicações importantes: primeiramente, que a imaginação não é dada na espécie, é
construída. Segundo, que ela é parte integrante do processo de aprendizagem, porque aprender significa,
exatamente, ser capaz de estabelecer conexões entre informações, construindo significado. Podemos ver
que, neste segundo caso, a imaginação é base para o estabelecimento destas novas redes, uma vez que
ela é a função psicológica que estabelece relações significativas entre elementos que não estavam
conectados entre si. A imaginação cria condições de aprendizagem.
Temos assim que a relação entre imaginação e memória tem sentido duplo: a base para o
funcionamento da imaginação são os elementos que estão contidos na memória e o próprio
funcionamento da imaginação desenvolve a memória (por meio do processo imaginativo, novas
mediações semióticas são realizadas, dando à pessoa uma maior complexidade aos sistemas contidos
na memória de longa duração).

Porque a imaginação é importante na aprendizagem?

1. Ela está na origem da construção do conhecimento que vamos ensinar.


O conhecimento científico e o conhecimento estético foram produzidos a partir do exercício
da imaginação humana nos vários períodos históricos.

2. Ela está na origem do conhecimento que será construído pelo aluno.


A imaginação motiva. Muitos educadores concordarão que a motivação é um fator
importante para o educando aprender. Motivar implica em mobilização para, interesse em,
envolvimento com o objeto de aprendizagem.
Esta disponibilidade para aprender envolve, do ponto de vista psicológico, a imaginação.

Por exemplo, podemos motivar o aluno para um fenômeno científico que será estudado com o
concurso da mobilização da imaginação: como será que a energia elétrica surge na represa? Como será
que a luz chega à lâmpada?
Que será que acontece com a semente debaixo da terra? Como será que o computador guarda tanta
informação? Porque o rio muda de cor?
Levantar hipóteses para qualquer destas questões implica em ter liberdade de pensamento. Isto é, a
capacidade imaginativa no ser humano tem como base a liberação da experiência sensível imediata,
desta forma a pessoa pode lidar, livremente, com o acervo mental que detém de imagens, informações,
sensações colhidas nas várias experiências de vida, juntamente com as emoções e sentimentos que as
acompanharam.
O desenvolvimento humano e a aprendizagem, na escola, envolvem, precisamente, esta dialética de
receber informações por meio dos sentidos e ter a possibilidade de ir além delas pelas funções mentais.

O desenvolvimento humano na teoria de Piaget

De acordo com a publicação de Marcia Regina Terra40 o estudo do desenvolvimento do ser humano
constitui uma área do conhecimento da Psicologia em que concentram-se no esforço de compreender o
homem em todos os seus aspectos, englobando fases desde o nascimento até o seu mais completo grau
de maturidade e estabilidade. Tal esforço, conforme mostra a linha evolutiva da Psicologia, tem culminado
na elaboração de várias teorias que procuram reconstituir, a partir de diferentes metodologias e pontos
de vistas, as condições de produção da representação do mundo e de suas vinculações com as visões
de mundo e de homem dominantes em cada momento histórico da sociedade.
Assim, dentre essas tantas teorias tem-se a de Jean Piaget, que, como as demais, busca compreender
o desenvolvimento do ser humano. No entanto, ela se destaca de outras pelo seu caráter inovador quando

40
http://www.unicamp.br/iel/site/alunos/publicacoes/

Apostila gerada especialmente para: Betânia Mueller 023.712.580-32


. 294
introduz uma 'terceira visão' representada pela linha interacionista que constitui uma tentativa de integrar
as posições dicotômicas de duas tendências teóricas que permeiam a Psicologia em geral - o
materialismo mecanicista e o idealismo - ambas marcadas pelo antagonismo inconciliável de seus
postulados que separam de forma estanque o físico e o psíquico.
Um outro ponto importante a ser considerado, segundo estudiosos, é o de que o modelo piagetiano
prima pelo rigor científico de sua produção, ampla e consistente ao longo de 70 anos, que trouxe
contribuições práticas importantes, principalmente, ao campo da Educação - muito embora, curiosamente
aliás, a intenção de Piaget não tenha propriamente incluído a ideia de formular uma teoria específica de
aprendizagem.
Tendo em vista o objetivo da teoria piagetiana que de acordo com Coll e Gillièron é "compreender
como o sujeito se constitui enquanto sujeito cognitivo, elaborador de conhecimentos válidos" cabe
algumas considerações sobre o método piagetiano sobre o desenvolvimento humano.

1. A visão interacionista de Piaget: a relação de interdependência entre o homem e o objeto do


conhecimento
Introduzindo uma terceira visão teórica representada pela linha interacionista, as ideias de Piaget
contrapõem-se, conforme mencionamos mais acima, às visões de duas correntes antagônicas e
inconciliáveis que permeiam a Psicologia em geral: o objetivismo e o subjetivismo. Ambas as correntes
são derivadas de duas grandes vertentes da Filosofia (o idealismo e o materialismo mecanicista) que, por
sua vez, são herdadas do dualismo radical de Descartes que propôs a separação estanque entre corpo
e alma, id est, entre físico e psíquico, ou seja, para ele havia uma ruptura radical entre o corpo e a alma
que eram distintos e independentes entre si.
Assim sendo, a Psicologia objetivista, privilegia o dado externo, afirmando que todo conhecimento
provém da experiência; e a Psicologia subjetivista, em contraste, calcada no substrato psíquico, entende
que todo conhecimento é anterior à experiência, reconhecendo, portanto, a primazia do sujeito sobre o
objeto.
Desta forma as duas teorias distintas entre si privilegiam cada uma a sua proposta ora o subjetivismo,
as experiências internas, as vivências e tudo que é inerente ao indivíduo e ora o objetivismo com tudo
que é externo ao indivíduo não havendo assim um meio termo entre ambas.
Sendo assim, considerando insuficientes essas duas posições para explicar o processo evolutivo da
filogenia humana, Piaget formula o conceito de epigênese, argumentando que "o conhecimento não
procede nem da experiência única dos objetos nem de uma programação inata pré-formada no sujeito,
mas de construções sucessivas com elaborações constantes de estruturas novas".
Quer dizer, o processo evolutivo da filogenia humana tem uma origem biológica que é ativada pela
ação e interação do organismo com o meio ambiente - físico e social - que o rodeia, significando entender
com isso que as formas primitivas da mente, biologicamente constituídas, são reorganizadas pela psique
socializada, ou seja, existe uma relação de interdependência entre o sujeito conhecedor e o objeto a
conhecer.
Esse processo, por sua vez, se efetua através de um mecanismo auto regulatório que consiste no
processo de equilibração progressiva do organismo com o meio em que o indivíduo está inserido.
Deste modo considera-se que as experiências internas, inatas do indivíduo em relação direta com o
meio externo é o que produz o conhecimento, ou seja, o social em conjunto com o individual é que forma
a estrutura completa do ser humano e, a cada novo contato com o meio existem reorganizações para que
se atinja novamente o estado de equilíbrio o indivíduo com o meio que o cerca.

Psicologia Objetivista Psicologia Subjetivista Interacionismo


Materialismo mecanicista Idealismo Piaget
Privilegia o psiquismo, para
Privilegia o dado externo,
ela o conhecimento é
assim todo o conhecimento É um “meio termo” entre o
anterior à experiência, ou
vem da experiências do objetivismo e o subjetivismo.
seja, é o indivíduo que “age”
indivíduo.
sobre o objeto.
É o meio ambiente e objetos Nessa visão o processo evolutivo
que cercam o indivíduo bem As vivências inatas e humano tem uma origem biológica
como suas experiências inerentes ao ser humano que que é ativada pela ação e interação
externas que possibilitam o possibilitam o conhecimento. do organismo com o meio ambiente -
conhecimento físico e social - que o rodeia.

Apostila gerada especialmente para: Betânia Mueller 023.712.580-32


. 295
2. O processo de equilibração: a busca pelo pensamento lógico
Pode-se dizer que o "sujeito epistêmico" protagoniza o papel central do modelo piagetiano, pois a
grande preocupação da teoria é desvendar os mecanismos processuais do pensamento do homem,
desde o início da sua vida até a idade adulta.
Nesse sentido, a compreensão dos mecanismos de constituição do conhecimento, na concepção de
Piaget, equivale à compreensão dos mecanismos envolvidos na formação do pensamento lógico,
matemático. Como lembra La Taille, "(...) a lógica representa para Piaget a forma final do equilíbrio das
ações. Ela é um sistema de operações, isto é, de ações que se tornaram reversíveis e passíveis de serem
compostas entre si'".
Com base nisso Piaget sustenta que a gênese do conhecimento está no próprio sujeito, ou seja, o
pensamento lógico não é inato ou tampouco externo ao organismo mas é fundamentalmente construído
na interação homem-objeto, assim o desenvolvimento da filogenia humana se dá através de um
mecanismo auto regulatório que tem como base condições biológicas (que são inatas) e que são ativadas
pela ação e interação do organismo com o meio ambiente tanto físico quanto social.
Está implícito nessa ótica de Piaget que o homem é possuidor de uma estrutura biológica que o
possibilita desenvolver o mental, no entanto, esse fato por si só não assegura o desencadeamento de
fatores que propiciarão o seu desenvolvimento, haja vista que este só acontecerá a partir da interação do
sujeito com o objeto a conhecer. Por sua vez, a relação com o objeto, embora essencial, da mesma forma
também não é uma condição suficiente ao desenvolvimento cognitivo humano, uma vez que para tanto é
preciso, ainda, o exercício do raciocínio. Por assim dizer, a elaboração do pensamento lógico demanda
um processo interno de reflexão. Tais aspectos deixam à mostra que, ao tentar descrever a origem da
constituição do pensamento lógico, Piaget focaliza o processo interno dessa construção.
Simplificando ao máximo, o desenvolvimento humano, no modelo piagetiano, é explicado segundo o
pressuposto de que existe uma conjuntura de relações interdependentes entre o sujeito conhecedor e o
objeto a conhecer. Esses fatores que são complementares envolvem mecanismos bastante complexos e
intrincados que englobam o entrelaçamento de fatores que são complementares, tais como: o processo
de maturação do organismo, a experiência com objetos, a vivência social e, sobretudo, a equilibração do
organismo ao meio.
O conceito de equilibração torna-se especialmente marcante na teoria de Piaget pois ele representa o
fundamento que explica todo o processo do desenvolvimento humano. Trata-se de um fenômeno que
tem, em sua essência, um caráter universal, já que é de igual ocorrência para todos os indivíduos da
espécie humana mas que pode sofrer variações em função de conteúdos culturais e do meio em que o
indivíduo está inserido. Nessa linha de raciocínio, o trabalho de Piaget leva em conta a atuação de dois
elementos básicos ao desenvolvimento humano: os fatores invariantes e os fatores variantes.
(a) Os fatores invariantes: Piaget postula que, ao nascer, o indivíduo recebe como herança uma série
de estruturas biológicas - sensoriais e neurológicas - que permanecem constantes ao longo da sua vida.
São essas estruturas biológicas que irão predispor o surgimento de certas estruturas mentais. Em vista
disso, na linha piagetiana, considera-se que o indivíduo carrega consigo duas marcas inatas que são a
tendência natural à organização e à adaptação, significando entender, portanto, que, em última instância,
o 'motor' do comportamento do homem é inerente ao ser.
(b) Os fatores variantes: são representados pelo conceito de esquema que constitui a unidade básica
de pensamento e ação estrutural do modelo piagetiano, sendo um elemento que se transforma no
processo de interação com o meio, visando à adaptação do indivíduo ao real que o circunda. Com isso,
a teoria psicogenética deixa à mostra que a inteligência não é herdada, mas sim que ela é construída no
processo interativo entre o homem e o meio ambiente (físico e social) em que ele estiver inserido.
Em síntese, pode-se dizer que, para Piaget, o equilíbrio é o norte que o organismo almeja mas que
paradoxalmente nunca alcança, haja vista que no processo de interação podem ocorrer desajustes do
meio ambiente que rompem com o estado de equilíbrio do organismo, eliciando esforços para que a
adaptação se restabeleça. Essa busca do organismo por novas formas de adaptação envolvem dois
mecanismos que apesar de distintos são indissociáveis e que se complementam: a assimilação e a
acomodação.
A assimilação consiste na tentativa do indivíduo em solucionar uma determinada situação a partir da
estrutura cognitiva que ele possui naquele momento específico da sua existência. Representa um
processo contínuo na medida em que o indivíduo está em constante atividade de interpretação da
realidade que o rodeia e, consequentemente, tendo que se adaptar a ela. Como o processo de
assimilação representa sempre uma tentativa de integração de aspectos experienciais aos esquemas
previamente estruturados, ao entrar em contato com o objeto do conhecimento o indivíduo busca retirar

Apostila gerada especialmente para: Betânia Mueller 023.712.580-32


. 296
dele as informações que lhe interessam deixando outras que não lhe são tão importantes, visando sempre
a restabelecer a equilibração do organismo.
A acomodação, por sua vez, consiste na capacidade de modificação da estrutura mental antiga para
dar conta de dominar um novo objeto do conhecimento. Quer dizer, a acomodação representa "o
momento da ação do objeto sobre o sujeito" emergindo, portanto, como o elemento complementar das
interações sujeito-objeto.
Em síntese, toda experiência é assimilada a uma estrutura de ideias já existentes (esquemas) podendo
provocar uma transformação nesses esquemas, ou seja, gerando um processo de acomodação.
Como observa Rappaport, os processos de assimilação e acomodação são complementares e acham-
se presentes durante toda a vida do indivíduo e permitem um estado de adaptação intelectual (...) É muito
difícil, se não impossível, imaginar uma situação em que possa ocorrer assimilação sem acomodação,
pois dificilmente um objeto é igual a outro já conhecido, ou uma situação é exatamente igual a outra.
Vê-se nessa ideia de "equilibração" de Piaget a marca da sua formação como biólogo que o levou a
traçar um paralelo entre a evolução biológica da espécie e as construções cognitivas.
Dessa perspectiva, o processo de equilibração pode ser definido como um mecanismo de organização
de estruturas cognitivas em um sistema coerente que visa a levar o indivíduo a construção de uma forma
de adaptação à realidade. Haja vista que o "objeto nunca se deixa compreender totalmente", o conceito
de equilibração sugere algo móvel e dinâmico, na medida em que a constituição do conhecimento coloca
o indivíduo frente a conflitos cognitivos constantes que movimentam o organismo no sentido de resolvê-
los.
Em última instância, a concepção do desenvolvimento humano, na linha piagetiana, deixa ver que é
no contato com o mundo que a matéria bruta do conhecimento é 'arrecadada', pois que é no processo de
construções sucessivas resultantes da relação sujeito-objeto que o indivíduo vai formar o pensamento
lógico.
É bom considerar, ainda, que, na medida em que toda experiência leva em graus diferentes a um
processo de assimilação e acomodação, trata-se de entender que o mundo das ideias, da cognição, é
um mundo inferencial. Para avançar no desenvolvimento é preciso que o ambiente promova condições
para transformações cognitivas, id est, é necessário que se estabeleça um conflito cognitivo que demande
um esforço do indivíduo para superá-lo a fim de que o equilíbrio do organismo seja restabelecido, e assim
sucessivamente.
No entanto, esse processo de transformação vai depender sempre de como o indivíduo vai elaborar
e assimilar as suas interações com o meio, isso porque o estado conquistado na equilibração do
organismo reflete as elaborações possibilitadas pelos níveis de desenvolvimento cognitivo que o
organismo detém nos diversos estágios da sua vida.
Deste modo, por toda a vida do indivíduo ele passa por processos de assimilação e acomodação
buscando atingir o estado de equilibração, porém a conquista desse estado está diretamente relacionada
com os níveis de desenvolvimento do indivíduo nos diversos estágios de sua vida.
A esse respeito, para Piaget, os modos de relacionamento com a realidade são divididos em 4 períodos
distintos, no processo evolutivo da espécie humana que são caracterizados "por aquilo que o indivíduo
consegue fazer melhor" no decorrer das diversas faixas etárias ao longo do seu processo de
desenvolvimento. São eles:

- 1º período: Sensório-motor (0 a 2 anos)


- 2º período: Pré-operatório (2 a 7 anos)
- 3º período: Operações concretas (7 a 11 ou 12 anos)
- 4º período: Operações formais (11 ou 12 anos em diante)

Cada uma dessas fases é caracterizada por formas diferentes de organização mental que possibilitam
as diferentes maneiras do indivíduo relacionar-se com a realidade que o rodeia. De uma forma geral,
todos os indivíduos passam por esses períodos na mesma sequência, porém o início e o término de cada
uma delas pode sofrer variações em função das características da estrutura biológica de cada indivíduo
e da riqueza (ou não) dos estímulos proporcionados pelo meio ambiente em que ele estiver inserido. Por
isso mesmo é que esta forma de divisão nessas faixas etárias é uma referência, e não uma norma rígida.

3. Os Estágios Cognitivos Segundo Piaget41


Piaget, quando descreve a aprendizagem, tem um enfoque diferente do que normalmente se atribui à
esta palavra. Piaget separa o processo cognitivo inteligente em duas palavras: aprendizagem e

41
Tafner, M. A construção do conhecimento segundo PIAGET. s/d. Em http://www.cerebromente.org.br/

Apostila gerada especialmente para: Betânia Mueller 023.712.580-32


. 297
desenvolvimento. Para Piaget, segundo Macedo, a aprendizagem refere-se à aquisição de uma resposta
particular, aprendida em função da experiência, obtida de forma sistemática ou não. Enquanto que o
desenvolvimento seria uma aprendizagem de fato, sendo este o responsável pela formação dos
conhecimentos.

Sensório-motor
Para Piaget o universo que circunda a criança é conquistado mediante a percepção e os movimentos
(como a sucção, o movimento dos olhos, por exemplo).
Neste estágio, a partir de reflexos neurológicos básicos, o bebê começa a construir esquemas de ação
para assimilar mentalmente o meio, é nesse período que a criança começa a discriminar ainda que de
forma pouco desenvolvida o meio que o cerca.
Segundo Lopes, as noções de espaço e tempo são construídas pela ação, configurando assim, uma
inteligência essencialmente prática, ou seja, é no contato direto com o objeto que o bebe começa a
construir a noção de espaço e de tempo de forma que ainda não há, neste período, uma construção
simbólica desenvolvida.
Considerando que esse período é marcado pela construção prática das noções de objeto, espaço,
causalidade e é assim que os esquemas vão "pouco a pouco, diferenciando-se e integrando-se, no
mesmo tempo em que o sujeito vai se separando dos objetos podendo, por isso mesmo, interagir com
eles de forma mais complexa." Nitzke diz que o contato com o meio é direto e imediato, sem
representação ou pensamento.

Exemplos: O bebê pega o que está em sua mão; "mama" o que é posto em sua boca; "vê" o que está
diante de si. Aprimorando esses esquemas, é capaz de ver um objeto, pegá-lo e levá-lo a boca.

Pré-operatório
Para Piaget, o que marca a passagem do período sensório-motor para o pré-operatório é o
aparecimento da função simbólica ou semiótica, ou seja, é a emergência da linguagem. Assim, conforme
demonstram as pesquisas psicogenéticas, a emergência da linguagem acarreta modificações importantes
em aspectos cognitivos, afetivos e sociais da criança, uma vez que ela possibilita as interações
interindividuais e fornece, principalmente, a capacidade de trabalhar com representações para atribuir
significados à realidade. Tanto é assim, que a aceleração do alcance do pensamento neste estágio do
desenvolvimento, é atribuída, em grande parte, às possibilidades de contatos interindividuais fornecidos
pela linguagem.
É nesta fase que surge, na criança, a capacidade de substituir um objeto ou acontecimento por uma
representação, e esta substituição é possível, conforme Piaget, graças à função simbólica. Assim este
estágio é também muito conhecido como o estágio da Inteligência Simbólica.
Contudo, Macedo lembra que a atividade sensório-motor não está esquecida ou abandonada, mas
refinada e mais sofisticada, pois verifica-se que ocorre uma crescente melhoria na sua aprendizagem,
permitindo que a mesma explore melhor o ambiente, fazendo uso de mais e mais sofisticados movimentos
e percepções intuitivas.

A criança deste estágio:


- É egocêntrica, centrada em si mesma, e não consegue se colocar, abstratamente, no lugar do outro.
- Não aceita a ideia do acaso e tudo deve ter uma explicação (é fase dos "por quês").
- Já pode agir por simulação, "como se".
- Possui percepção global sem discriminar detalhes.
- Deixa se levar pela aparência sem relacionar fatos.

Exemplos: Mostram-se para a criança, duas bolinhas de massa iguais e dá-se a uma delas a forma de
salsicha. A criança nega que a quantidade de massa continue igual, pois as formas são diferentes. Não
relaciona as situações.

Operatório-concreto
Conforme Nitzke, neste estágio a criança desenvolve noções de tempo, espaço, velocidade, ordem,
casualidade ,..., sendo então capaz de relacionar diferentes aspectos e abstrair dados da realidade.
Apesar de não se limitar mais a uma representação imediata, depende do mundo concreto para abstrair.
Um importante conceito desta fase é o desenvolvimento da reversibilidade, ou seja, a capacidade da
representação de uma ação no sentido inverso de uma anterior, anulando a transformação observada.

Apostila gerada especialmente para: Betânia Mueller 023.712.580-32


. 298
Exemplos: Despeja-se a água de dois copos em outros, de formatos diferentes, para que a criança
diga se as quantidades continuam iguais. A resposta é afirmativa uma vez que a criança já diferencia
aspectos e é capaz de "refazer" a ação.

Operatório-formal
De acordo com a tese piagetiana, ao atingir esta fase, o indivíduo adquire a sua forma final de
equilíbrio, ou seja, ele consegue alcançar o padrão intelectual que persistirá durante a idade adulta. Isso
não quer dizer que ocorra uma estagnação das funções cognitivas, a partir do ápice adquirido na
adolescência, como enfatiza Rappaport, "esta será a forma predominante de raciocínio utilizada pelo
adulto. Seu desenvolvimento posterior consistirá numa ampliação de conhecimentos tanto em extensão
como em profundidade, mas não na aquisição de novos modos de funcionamento mental".
A representação agora permite à criança uma abstração total, não se limitando mais à representação
imediata e nem às relações previamente existentes.
Agora a criança é capaz de pensar logicamente, formular hipóteses e buscar soluções, sem depender
mais só da observação da realidade. Em outras palavras, as estruturas cognitivas da criança alcançam
seu nível mais elevado de desenvolvimento e tornam-se aptas a aplicar o raciocínio lógico a todas as
classes de problemas.

Exemplos: Se lhe pedem para analisar um provérbio como "de grão em grão, a galinha enche o papo",
a criança trabalha com a lógica da ideia (metáfora) e não com a imagem de uma galinha comendo grãos.

4. As consequências do modelo piagetiano para a ação pedagógica


Conforme mencionado anteriormente, a teoria psicogenética de Piaget não tinha como objetivo
principal propor uma teoria de aprendizagem. A esse respeito, Coll faz a seguinte observação: "ao que
se sabe, ele nunca participou diretamente nem coordenou uma pesquisa com objetivos pedagógicos".
Não obstante esse fato, de forma contraditória aos interesses previstos, portanto, o modelo piagetiano,
curiosamente, veio a se tornar uma das mais importantes diretrizes no campo da aprendizagem escolar,
por exemplo, nos USA, na Europa e no Brasil, inclusive.
De acordo com Coll as tentativas de aplicação da teoria genética no campo da aprendizagem são
numerosas e variadas, no entanto os resultados práticos obtidos com tais aplicações não podem ser
considerados tão frutíferos. Uma das razões da difícil penetração da teoria genética no âmbito da escola
deve-se, principalmente, segundo o autor, "ao difícil entendimento do seu conteúdo conceitual como pelos
método de análise formalizante que utiliza e pelo estilo às vezes 'hermético' que caracteriza as
publicações de Piaget". O autor ressalta, também, que a aplicação educacional da teoria genética tem
como fatores complicadores, entre outros:
a) as dificuldades de ordem técnica, metodológicas e teóricas no uso de provas operatórias como
instrumento de diagnóstico psicopedagógico, exigindo um alto grau de especialização e de prudência
profissional, a fim de se evitar os riscos de sérios erros;
b) a predominância no "como" ensinar coloca o objetivo do "o quê" ensinar em segundo plano,
contrapondo-se, dessa forma, ao caráter fundamental de transmissão do saber acumulado culturalmente
que é uma função da instituição escolar, por ser esta de caráter preeminentemente político-metodológico
e não técnico como tradicionalmente se procurou incutir nas ideias da sociedade;
c) a parte social da escola fica prejudicada uma vez que o raciocínio por trás da argumentação de que
a criança vai atingir o estágio operatório secundariza a noção do desenvolvimento do pensamento crítico;
d) a ideia básica do construtivismo postulando que a atividade de organização e planificação da
aquisição de conhecimentos estão à cargo do aluno acaba por não dar conta de explicar o caráter da
intervenção por parte do professor;
e) a ideia de que o indivíduo apropria os conteúdos em conformidade com o desenvolvimento das suas
estruturas cognitivas estabelece o desafio da descoberta do "grau ótimo de desequilíbrio", ou seja, o
objeto a conhecer não deve estar nem além nem aquém da capacidade do aprendiz conhecedor.
Por outro lado, como contribuições contundentes da teoria psicogenética podem ser citados, por
exemplo:
a) a possibilidade de estabelecer objetivos educacionais uma vez que a teoria fornece parâmetros
importantes sobre o 'processo de pensamento da criança' relacionados aos estádios do desenvolvimento;
b) em oposição às visões de teorias behavioristas que consideravam o erro como interferências
negativas no processo de aprendizagem, dentro da concepção cognitivista da teoria psicogenética, os
erros passam a ser entendidos como estratégias usadas pelo aluno na sua tentativa de aprendizagem de
novos conhecimentos (PCN);

Apostila gerada especialmente para: Betânia Mueller 023.712.580-32


. 299
c) uma outra contribuição importante do enfoque psicogenético foi lançar luz à questão dos diferentes
estilos individuais de aprendizagem; (PCN); entre outros.

Em resumo, conforme aponta Coll, as relações entre teoria psicogenética x educação, apesar dos
complicadores decorrentes da "dicotomia entre os aspectos estruturais e os aspectos funcionais da
explicação genética" e da tendência dos projetos privilegiarem, em grande parte, um reducionismo
psicologizante em detrimento ao social, pode-se considerar assim que a teoria psicogenética trouxe
contribuições importantes ao campo da aprendizagem escolar.

Origens do pensamento e da língua e o significado das palavras e a formação de conceitos de


acordo com Vygotsky42

Assim como no reino animal, para o ser humano pensamento e linguagem têm origens diferentes.
Inicialmente o pensamento não é verbal e a linguagem não é intelectual.
Convém ressaltar porém que o desenvolvimento da linguagem e do pensamento se cruzam, assim
com cerca dos dois anos de idade as curvas de desenvolvimento do pensamento e da linguagem, até
então separadas, encontram-se para, a partir daí, dar início a uma nova forma de comportamento. É a
partir deste ponto que o pensamento começa a se tornar verbal e a linguagem racional. Inicialmente a
criança aparenta usar linguagem apenas para interação superficial em seu convívio, mas, a partir de certo
ponto, esta linguagem penetra no subconsciente para se constituir na estrutura do pensamento da
criança. Sendo assim se torna possível à criança utilizar a linguagem de forma racional, atribuindo-lhe
significados.
A partir do momento que a criança descobre que tudo tem um nome, cada novo objeto que surge
representa um problema que a criança resolve atribuindo-lhe um nome. Quando lhe falta a palavra para
nomear este novo objeto, a criança recorre ao adulto. Esses significados básicos de palavras assim
adquiridos funcionarão como embriões para a formação de novos e mais complexos conceitos.

Pensamento, linguagem e desenvolvimento intelectual


De acordo com Vygotsky, todas as atividades cognitivas básicas do indivíduo ocorrem de acordo com
sua história social e acabam se constituindo no produto do desenvolvimento histórico-social de sua
comunidade. Portanto, as habilidades cognitivas e as formas de estruturar o pensamento do indivíduo
não são determinadas por fatores congênitos. São, isto sim, resultado das atividades praticadas de acordo
com os hábitos sociais da cultura em que o indivíduo se desenvolve. Consequentemente, a história da
sociedade na qual a criança se desenvolve e a história pessoal desta criança são fatores cruciais que vão
determinar sua forma de pensar. Neste processo de desenvolvimento cognitivo, a linguagem tem papel
crucial na determinação de como a criança vai aprender a pensar, uma vez que formas avançadas de
pensamento são transmitidas à criança através de palavras.
Para Vygotsky, um claro entendimento das relações entre pensamento e língua é necessário para que
se entenda o processo de desenvolvimento intelectual. Linguagem não é apenas uma expressão do
conhecimento adquirido pela criança. Existe uma inter-relação fundamental entre pensamento e
linguagem, um proporcionando recursos ao outro. Desta forma a linguagem tem um papel essencial na
formação do pensamento e do caráter do indivíduo.

Zona de desenvolvimento próximo (ou proximal)


Um dos princípios básicos da teoria de Vygotsky é o conceito de "zona de desenvolvimento próximo".
Zona de desenvolvimento próximo representa a diferença entre a capacidade da criança de resolver
problemas por si própria e a capacidade de resolvê-los com ajuda de alguém. Em outras palavras,
teríamos uma "zona de desenvolvimento autossuficiente" que abrange todas as funções e atividades que
a criança consegue desempenhar por seus próprios meios, sem ajuda externa. Zona de desenvolvimento
próximo, por sua vez, abrange todas as funções e atividades que a criança ou o aluno consegue
desempenhar apenas se houver ajuda de alguém. Esta pessoa que intervém para orientar a criança pode
(pais, professor, responsável, instrutor de língua estrangeira) quanto um colega que já tenha desenvolvido
a habilidade requerida.
Uma analogia interessante nos vem à mente quando pensamos em zona de desenvolvimento próximo.
Em mecânica, quando regula-se o ponto de um motor a explosão, este deve ser ajustado ligeiramente à
frente do momento de máxima compressão dentro do cilindro, para maximizar a potência e o
desempenho.

42
Texto adaptado disponível em http://www.ebooksbrasil.org/eLibris/vigo.html

Apostila gerada especialmente para: Betânia Mueller 023.712.580-32


. 300
A ideia de zona de desenvolvimento próximo é de grande relevância em todas as áreas educacionais.
Uma implicação importante é a de que o aprendizado humano é de natureza social e é parte de um
processo em que a criança desenvolve seu intelecto dentro da intelectualidade daqueles que a cercam.
De acordo com Vygotsky, uma característica essencial do aprendizado é que ele desperta vários
processos de desenvolvimento internamente, os quais funcionam apenas quando a criança interage em
seu ambiente de convívio.

Teoria Vygotskiana43
Vygotsky trabalha com teses dentro de suas obras nas quais são possíveis descrever como: à relação
indivíduo/ sociedade em que afirma que as características humanas não estão presentes desde o
nascimento, nem são simplesmente resultados das pressões do meio externo, elas são resultados das
relações homem e sociedade, pois quando o homem transforma o meio na busca de atender suas
necessidades básicas, ele transforma-se a si mesmo. A criança nasce apenas com as funções
psicológicas elementares e a partir do aprendizado da cultura, estas funções transformam-se em funções
psicológicas superiores, sendo estas o controle consciente do comportamento, a ação intencional e a
liberdade do indivíduo em relação às características do momento e do espaço presente. O
desenvolvimento do psiquismo humano é sempre mediado pelo outro que indica, delimita e atribui
significados à realidade, dessa forma, membros imaturos da espécie humana vão aos poucos se
apropriando dos modos de funcionamento psicológicos, comportamento e cultura. Neste caso podemos
citar a importância da inclusão de fato, onde as crianças com alguma deficiência interajam com crianças
que estejam com desenvolvimento além, realizando a troca de saberes e experiências, onde ambos
passam a aprender junto.
Vygotsky defende a educação inclusiva e acessibilidade para todos. Devido ao processo criativo que
envolve o domínio da natureza, o emprego de ferramentas e instrumentos, o homem pode ter uma ação
indireta, planejada tendo ou não deficiência, assim, pessoas com deficiência auditiva, visuais, e outras
podem ter um alto nível de desenvolvimento, a escola deve permitir que dominem depois superem seus
saberes do cotidiano. As crianças cegas podem alcançar o mesmo desenvolvimento de uma criança
normal, só que de modo diferente, por outra via, é muito importante para o pedagogo conhecer essa
peculiaridade, é a lei da compensação, não é o limite biológico que determina o não desenvolvimento do
surdo, cego, mas sim a sociedade que vem criando estes limites para que os deficientes não se
desenvolvam totalmente.
A segunda tese refere-se à origem cultural das funções psíquicas que se originam nas relações do
indivíduo e seu contexto social e cultural, isso mostra que a cultura é parte constitutiva da natureza
humana, pois o desenvolvimento mental humano não é passivo, nem tão pouco independente do
desenvolvimento histórico e das formas sociais da vida. O desenvolvimento mental da criança é um
processo continuo de aquisições, desenvolvimento intelectual e linguístico relacionado à fala interior e
pensamento e impondo estruturas superiores, ao saber de novos conceitos evita-se que a criança tenha
que reestruturar todos os conceitos que já possui. Vygotsky tinha como objetivo constatar como as
funções psicológicas, tais como memória, a atenção, a percepção e o pensamento aparecem primeiro na
forma primária para, posteriormente, aparecerem em formas superiores, assim é possível perceber a
importante distinção realizada entre as funções elementares (comuns aos animais e aos humanos) e as
funções psicológicas superiores (especificamente vinculada aos humanos).
A terceira tese refere-se a base biológica do funcionamento psicológico o cérebro é o órgão principal
da atividade mental, sendo entendido como um sistema aberto, cuja estrutura e funcionamento são
moldados ao longo da história, podendo mudar sem que ajam transformações físicas no órgão.
A quarta tese faz referência à característica mediação presente em toda a vida humana em que
usamos técnicas e signos para fazermos mediação entre seres humanos e estes com o mundo. A
linguagem é um signo mediador por excelência por isso Vygotsky a confere um papel de destaque no
processo de pensamento. Sendo esta uma capacidade exclusiva da humanidade. Através da fala
podemos organizar as atividades práticas e das funções psicológicas. As pesquisas de Vygotsky foram
realizadas com a criança na fase em que começa a desenvolver a fala, pois se acreditava que a
verdadeira essência do comportamento se dá a partir da mesma. É na atividade pratica, ou seja, na
coletividade que a pessoa se aproveita da linguagem e dos objetos físicos disponíveis em sua cultura,
promovendo assim seu desenvolvimento, dando ênfase aos conhecimentos histórico-cultural,
conhecimentos produzidos e já existentes em seu cotidiano.

43
COELHO, L.; PISONI, S. Vygotsky: sua teoria e a influência na educação. Revista e-Ped- FACOS/ CNEC Osório. Vol 02/2012.

Apostila gerada especialmente para: Betânia Mueller 023.712.580-32


. 301
O desenvolvimento e a aprendizagem
Vygotsky dá um lugar de destaque para as relações de desenvolvimento e aprendizagem dentro de
suas obras. Para ele a criança inicia seu aprendizado muito antes de chegar à escola, mas o aprendizado
escolar vai introduzir elementos novos no seu desenvolvimento. A aprendizagem é um processo contínuo
e a educação é caracterizada por saltos qualitativos de um nível de aprendizagem a outro, daí a
importância das relações sociais, desse modo dois tipos de desenvolvimento foram identificados: o
desenvolvimento real que se refere àquelas conquistas que já são consolidadas na criança, aquelas
capacidades ou funções que realiza sozinha sem auxílio de outro indivíduo, habitualmente costuma-se
avaliar a criança somente neste nível, ou seja, somente o que ela já é capaz de realizar e o
desenvolvimento potencial que se refere àquilo que a criança pode realizar com auxílio de outro indivíduo.
Neste caso as experiências são muito importantes, pois ele aprende através do diálogo, colaboração,
imitação... A distância entre os dois níveis de desenvolvimentos chamamos de zona de desenvolvimento
potencial ou proximal, o período que a criança fica utilizando um ‘apoio’ até que seja capaz de realizar
determinada atividade sozinha. Por isso Vygotsky afirma que “aquilo que é zona de desenvolvimento
proximal hoje será o nível de desenvolvimento real amanhã – ou seja, aquilo que uma criança pode fazer
com assistência hoje, ela será capaz de fazer sozinha amanhã”. O conceito de zona de desenvolvimento
proximal é muito importante para pesquisar o desenvolvimento e o plano educacional infantil, porque este
permite avaliar o desenvolvimento individual. Aqui é possível elaborar estratégias pedagógicas para que
a criança possa evoluir no aprendizado uma vez que esta é a zona cooperativa do conhecimento, assim,
o mediador ajuda a criança a concretizar o desenvolvimento que está próximo, ou seja, ajuda a
transformar o desenvolvimento potencial em desenvolvimento real.
O desenvolvimento e a aprendizagem estão inter-relacionados desde o momento do nascimento, o
meio físico ou social influenciam no aprendizado das crianças de modo que chegam as escolas com uma
série de conhecimentos adquiridos, na escola a criança desenvolverá outro tipo de conhecimento.
Assim se divide o conhecimento em dois grupos: aqueles adquiridos da experiência pessoal, concreta
e cotidiana em que são chamados de ‘conceitos cotidianos ou espontâneos’ em que são caracterizados
por observações, manipulações e vivências diretas da criança já os ‘conceitos científicos’ adquiridos em
sala de aula se relacionam àqueles não diretamente acessíveis à observação ou ação imediata da
criança. A escola tem papel fundamental na formação dos conceitos científicos, proporcionando à criança
um conhecimento sistemático de algo que não está associado a sua vivência direta principalmente na
fase de amadurecimento.
O brinquedo é um mundo imaginário onde a criança pode realizar seus desejos, o ato de brincar é uma
importante fonte de promoção de desenvolvimento, sendo muito valorizado na zona proximal, neste caso
em especial as brincadeiras de ‘faz de conta’. Sendo estas atividades utilizadas, em geral, na Educação
Infantil fase que as crianças aprendem a falar (após os três anos de idade), e são capazes de envolver-
se numa situação imaginária. Através do imaginário a criança estabelece regras do cotidiano real.
Mesmo havendo uma significativa distância entre o comportamento na vida real e o comportamento
no brinquedo, a atuação no mundo imaginário e o estabelecimento de regras a serem seguidas criam
uma zona de desenvolvimento proximal, na medida em que impulsionam conceitos e processos em
desenvolvimento.

Vygotsky e a educação
A escola se torna importante a partir do momento que dentro dela o ensino é sistematizado sendo
atividades diferenciadas das extraescolares e lá a criança aprende a ler, escrever, obtém domínio de
cálculos, entre outras, assim expande seus conhecimentos. Também não é pelo simples fato da criança
frequentar a escola que ela estará aprendendo, isso dependerá de todo o contexto seja questão política,
econômica ou métodos de ensino. Conforme foi visto até aqui, aulas onde o aluno fica ouvindo e
memorizando conteúdos não basta para se dizer que o aprendizado ocorreu de fato, o aprendizado exige
muito mais. O trabalho pedagógico deve estar associado à capacidade de avanços no desenvolvimento
da criança, valorizando o desenvolvimento potencial e a zona de desenvolvimento proximal. A escola
deve estar atenta ao aluno, valorizar seus conhecimentos prévios, trabalhar a partir deles, estimular as
potencialidades dando a possibilidade de este aluno superar suas capacidades e ir além ao seu
desenvolvimento e aprendizado. Para que o professor possa fazer um bom trabalho ele precisa conhecer
seu aluno, suas descobertas, hipóteses, crenças, opiniões desenvolvendo diálogo criando situações onde
o aluno possa expor aquilo que sabe. Assim os registros, as observações são fundamentais tanto para o
planejamento e objetivos quanto para a avaliação.

Apostila gerada especialmente para: Betânia Mueller 023.712.580-32


. 302
Infância e Adolescência

Infância
Memória dos bebês44
Você consegue se lembrar de alguma coisa que aconteceu antes dos seus 2 anos de idade?
Provavelmente não. Os cientistas do desenvolvimento propuseram várias explicações para esse
fenômeno comum. Uma explicação, sustentada por Piaget e outros, é que eventos dessa época não são
armazenados na memória, porque o cérebro ainda não está suficientemente desenvolvido. Freud, por
outro lado, acreditava que as primeiras lembranças estão armazenadas, porém reprimidas, porque são
emocionalmente perturbadoras. Outros pesquisadores sugerem que as crianças só conseguem
armazenar eventos na memória quando podem falar sobre eles.
Pesquisas mais recentes que utilizam o condicionamento operante com tarefas não verbais e
apropriadas para a idade sugerem que o processamento da memória nos bebês pode não ser
fundamentalmente diferente do que acontece com crianças mais velhas e adultos, salvo que o tempo de
retenção dos bebês é mais curto. Esses estudos constataram-que os bebês repetirão uma ação dias ou
semanas mais tarde - se eles foram periodicamente lembrados da situação em que a aprenderam.
Em uma série de experimentos realizados por Carolyn Rovee-Collier e associados, os bebês foram
submetidos a condicionamento operante para mexer a perna e ativar um móbile preso a um dos
tornozelos por uma fita. Bebês de 2 a 6 meses, aos quais foram apresentados os mesmos móbiles dias
ou semanas depois, repetiam os chutes, mesmo quando seu tornozelo não mais estava preso ao móbile.
Quando os bebês viram esses móbiles, deram mais chutes do que antes do condicionamento, mostrando
que o reconhecimento dos móbiles acionava a lembrança de sua experiência inicial com esses objetos.
Em uma tarefa semelhante, crianças de 9 a 12 meses foram condicionadas a pressionar uma alavanca
para fazer um trem de brinquedo percorrer um circuito. A extensão de tempo que uma resposta
condicionada podia ser retida aumentou com a idade, de dois dias para crianças de 2 meses a 13
semanas para crianças de 18 meses.
A memória de bebês novos sobre um comportamento parece estar associada especificamente ao
indicativo original. Bebês entre 2 e 6 meses repetiam o comportamento aprendido somente quando viam
o móbile ou o trem original. Entretanto, crianças entre 9 e 12 meses experimentavam o comportamento
em um trem diferente se não mais que duas semanas se passassem desde o treinamento.
Um contexto familiar pode melhorar a evocação quando a lembrança de alguma coisa enfraqueceu.
Crianças de 3, 9 e 12 meses inicialmente podiam reconhecer o móbile ou o trem num ambiente diferente
daquele onde foram treinadas, mas não depois de passado muito tempo. Lembretes não verbais
periódicos por meio de uma breve exposição ao estímulo original podem manter uma lembrança desde a
primeira infância até entre 1 e 2 anos de idade.
Pelo menos um importante pesquisador da memória refuta a alegação de que as memórias
condicionadas sejam qualitativamente as mesmas das crianças mais velhas e dos adultos. De uma
perspectiva evolucionista do desenvolvimento, as habilidades se desenvolvem à medida que podem
realizar funções úteis na adaptação ao ambiente. O conhecimento procedural e perceptual demonstrado
logo cedo pelos bebês ao chutar um móbile para ativá-lo não é a mesma coisa que a memória explícita
de uma criança mais velha ou de um adulto sobre eventos específicos. A primeira infância é uma fase de
grandes transformações, e é improvável que a retenção de experiências específicas seja útil por muito
tempo. Essa pode ser uma das razões de os adultos não se lembrarem de eventos que aconteceram
quando eram bebês. Mais adiante discutiremos pesquisas sobre o cérebro que lançam alguma luz sobre
o desenvolvimento da memória na primeira infância. 45

Abordagem psicométrica: testes de desenvolvimento e de inteligência


Embora não haja um consenso científico claro sobre a definição de comportamento inteligente, a
maioria dos profissionais concorda que o comportamento inteligente é orientado para uma meta e é
adaptativo: direcionado para se adaptar às circunstâncias e condições de vida. A inteligência permite às
pessoas adquirir, lembrar e utilizar conhecimento; entender conceitos e relações; e resolver os problemas
do dia a dia.
A natureza precisa da inteligência tem sido debatida por muitos anos, e também a melhor maneira de
medi-la. O movimento moderno para testar a inteligência teve início no começo do século XX, quando
administradores de escolas em Paris pediram ao psicólogo A1fred Binet que elaborasse um modo de
identificar crianças que não pudessem acompanhar o trabalho escolar e precisassem de instruções

44
PAPALIA, D. E.; FELDMAN, R. D. Desenvolvimento Humano, 12ª edição, 2013, editor: AMGH.
45
ROVEE-COLLIER, C.; HARTSHORN, k. & DIRUBBO, M. Long-term maintenance of infant memory. Developmental Psychobiology.

Apostila gerada especialmente para: Betânia Mueller 023.712.580-32


. 303
especiais. O teste desenvolvido por Binet e seu colega Theodore Simon foi o precursor dos testes
psicométricos que avaliam a inteligência por números.
O objetivo da aplicação de testes psicométricos é medir quantitativamente os fatores que
supostamente constituem a inteligência (tais como compreensão e raciocínio) e, a partir dos resultados
dessa medida, prever o desempenho futuro (como o desempenho escolar). Os testes de 01 (quociente
de inteligência) consistem em perguntas ou tarefas que devem mostrar quanto das habilidades medidas
a pessoa possui, comparando seu desempenho com normas estabeleci das para um grupo extenso que
compôs a amostra de padronização. Para crianças em idade escolar, as pontuações no teste de
inteligência podem servir para prever o desempenho na escola com razoável precisão e confiabilidade.
Testar bebês e crianças pequenas já é outra questão. Como os bebês não podem nos dizer o que sabem
e como pensam, a maneira mais óbvia de aferir sua inteligência é avaliando o que sabem fazer. Mas se
eles não pegarem um chocalho, é difícil saber se não o fizeram porque não sabiam como, não estavam
com vontade, não perceberam o que se esperava deles ou simplesmente perderam o interesse.

Testes de desenvolvimento infantil


Embora seja praticamente impossível medir a inteligência de um bebê, é possível testar seu
desenvolvimento. Os testes de desenvolvimento comparam o desempenho do bebê numa série de tarefas
com normas estabelecidas baseadas na observação do que um grande número de bebês e crianças
pequenas sabe fazer em determinadas idades.
As Escalas Bayley de Desenvolvimento Infantil (Bayley, 1969, 1993,2005) constituem um teste de
desenvolvimento amplamente utilizado e elaborado para avaliar crianças entre 1 mês e 3 anos e meio.
Pontuações na Bayley-IIl indicam os pontos fortes e fracos e as competências de uma criança em cada
uma tías cinco áreas do desenvolvimento: cognitivo, linguagem, motor, socioemocional e comportamento
adaptativo. Uma escala opcional de classificação do comportamento pode ser preenchida pelo
examinador, em parte com base nas informações dadas pelo cuidador. Pontuações separadas, chamadas
Cle quocientes de desenvolvimento (ODs), são calculadas para cada escala. Os ODs são muito úteis
para detectar, logo no início, perturbações emocionais e déficits sensoriais, neurológicos e ambientais, e
podem ajudar pais e profissionais a planejar o atendimento das necessidades da criança.

Intervenção Precoce
A intervenção precoce é um processo sistemático de planejamento e fornecimento de serviços
terapêuticos e educacionais para famílias que precisam de ajuda para satisfazer as necessidades de
desenvolvimento de bebês e crianças em idade pré-escolar.

Fundamentos do desenvolvimento psicossocial


Embora os bebês apresentem os mesmos padrões de desenvolvimento, cada um deles, desde o início,
exibe uma personalidade distinta: a combinação relativamente coerente de emoções, temperamento,
pensamento e comportamento é que torna cada pessoa única. De maneira geral, bebês podem ser
alegres; outros se irritam com facilidade. Há crianças que gostam de brincar com as demais; outras
preferem brincar sozinhas. Esses modos característicos de sentir, pensar e agir, que refletem influências
tanto inatas quanto ambientais, afetam a maneira como a criança responde aos outros e se adapta ao
seu mundo. Da primeira infância em diante, o desenvolvimento da personalidade se entrelaça com as
relações sociais; e essa combinação chama-se desenvolvimento psicossocial.
Ao explorarmos o desenvolvimento psicossocial, primeiro focalizaremos as emoções, os blocos de
construção da personalidade; em seguida, o temperamento ou disposição; e depois as primeiras
experiências sociais da criança na família. Finalmente, discutiremos como os pais moldam as diferenças
comportamentais entre meninos e meninas.

Emoções
Emoções, como tristeza, alegria e medo, são reações subjetivas à experiência e que estão associadas
a mudanças fisiológicas e comportamentais. O medo, por exemplo, é acompanhado de aceleração dos
batimentos cardíacos e, geralmente, de ações de autoproteção. O padrão característico de reações
emocionais de uma pessoa começa a se desenvolver durante a primeira infância e constitui um elemento
básico da personalidade. As pessoas diferem na frequência e na intensidade com que sentem uma
determinada emoção, nos tipos de eventos que podem produzi-la, nas manifestações físicas que
demonstram e no modo como agem em consequência disso. A cultura também influencia o modo como
as pessoas se sentem em relação a uma situação e a maneira como expressam suas emoções.

Apostila gerada especialmente para: Betânia Mueller 023.712.580-32


. 304
Algumas culturas asiáticas, que enfatizam a harmonia social, desencorajam expressões de raiva, mas
dão muita importância à vergonha. O oposto geralmente é verdadeiro na cultura norte-americana, que
enfatiza a autoexpressão, a autoafirmação e a autoestima.

Quando aparecem as emoções?


O desenvolvimento emocional é um processo ordenado; emoções complexas desdobram-se de outras
mais simples. De acordo com um dos modelos, o bebê revela sinais de contentamento, interesse e aflição
logo após o nascimento. Trata--se de respostas difusas, reflexas, a maior parte fisiológicas, à estimulação
sensorial ou a processos internos. Aproximadamente nos próximos seis meses, esses estados
emocionais iniciais se diferenciam em verdadeiras emoções: alegria, surpresa, tristeza, repugnância, e
depois raiva e medo - reações a eventos que têm significado para o bebê. Conforme será discutido mais
adiante, a emergência dessas emoções básicas, ou primárias, está relacionada à maturação neurológica.
As emoções autoconscientes, como o constrangimento, a empatia e a inveja, surgem somente depois
que a criança desenvolveu a auto consciência: compreensão cognitiva de que ela tem uma identidade
reconhecível, separada e diferente do resto de seu mundo. Essa consciência da própria identidade parece
emergir entre 15 e 24 meses. A autoconsciência é necessária para que a criança possa estar consciente
de ser o foco da atenção, identificar-se com o que outras "identidades" estão sentindo, ou desejar o que
outra pessoa tem.
Por volta dos 3 anos, tendo adquirido autoconsciência e mais algum conhecimento sobre os padrões,
regras e metas aceitas de sua sociedade, a criança torna-se mais capacitada para avaliar seus próprios
pensamentos, planos, desejos e comportamento com relação àquilo que é considerado socialmente
apropriado. Só então ela pode demonstrar emoções auto avaliadoras como orgulho, culpa e vergonha.

Crescimento do cérebro e desenvolvimento emocional


O desenvolvimento do cérebro após o nascimento está intimamente ligado a mudanças na vida
emocional: as experiências emocionais são afetadas pelo desenvolvimento do cérebro e podem causar
efeitos duradouros na estrutura cerebral.
Quatro importantes mudanças na organização do cérebro correspondem aproximadamente a
mudanças no processamento emocional. Durante os três primeiros meses, começa a diferenciação das
emoções básicas à medida que o córtex cerebral torna-se funcional e faz emergir as percepções
cognitivas. Diminuem o sono REM e o comportamento reflexo, incluindo o sorriso neonatal espontâneo.
A segunda mudança ocorre por volta dos 9 ou 10 meses, quando os lobos frontais começam a interagir
com o sistema límbico, uma das regiões do cérebro associada às reações emocionais. Ao mesmo tempo,
estruturas límbicas como o hipocampo tornam-se maiores e mais semelhantes à estrutura adulta.
Conexões entre o córtex frontal e o hipotálamo e o sistema límbico, que processam a informação
sensorial, podem facilitar a relação entre as esferas cognitiva e emocional. À medida que essas conexões
tornam-se mais densas e mais elaboradas, o bebê poderá ao mesmo tempo experimentar e interpretar
emoções.
A terceira mudança ocorre durante o segundo ano, quando o bebê desenvolve a autoconsciência, as
emoções autoconscientes e maior capacidade para regular suas emoções e atividades. Essas mudanças,
que coincidem com maior mobilidade física e com o comportamento exploratório, podem estar
relacionadas à mielinização dos lobos frontais.
A quarta mudança ocorre por volta dos 3 anos, quando alterações hormonais no sistema nervoso
autônomo (involuntário) coincidem com a emergência das emoções avaliadoras. Subjacente ao
desenvolvimento de emoções como a vergonha pode estar um afastamento da dominância por parte do
sistema simpático, a parte do sistema autônomo que prepara o corpo para a ação, enquanto amadurece
o sistema parassimpático, a parte do sistema autônomo envolvida na excreção e na excitação sexual.

Desenvolvimento da autonomia
À medida que a criança amadurece - fisicamente, cognitivamente e emocionalmente - ela é levada a
buscar sua independência em relação aos vários adultos aos quais está apegada. "Eu fazer!" é a frase
típica da criança quando começa a usar seus músculos e sua mente para tentar fazer tudo sozinha - não
somente andar, mas alimentar-se, vestir-se e explorar o mundo.
Erikson identificou o período entre 18 meses e 3 anos como o segundo estágio no desenvolvimento
da personalidade, autonomia versus vergonha e dúvida, marcado pela passagem do controle externo
para o autocontrole. Tendo atravessado a primeira infância com um senso de confiança básica no mundo
e uma autoconsciência florescente, a criança pequena começa a substituir o julgamento dos cuidadores
pelo seu próprio. A "virtude" que emerge durante esse estágio é a vontade. O treinamento do controle

Apostila gerada especialmente para: Betânia Mueller 023.712.580-32


. 305
das necessidades fisiológicas é um passo importante em direção à autonomia e ao autocontrole; o mesmo
acontece com a linguagem.
À medida que a criança torna-se mais apta a expressar seus desejos, ela passa a ter mais poder.
Como a liberdade sem limites não é segura nem saudável, disse Erikson, vergonha e dúvida ocupam um
lugar necessário. As crianças pequenas precisam que os adultos estabeleçam limites apropriados; assim,
a vergonha e a dúvida ajudam-nas a reconhecer a necessidade desses limites.
Nos Estados Unidos, os "terríveis dois anos" assinalam um desejo de autonomia. Crianças pequenas
precisam testar as noções de que são indivíduos, têm algum controle sobre seu mundo e possuem novos
e emocionantes poderes. São levadas a experimentar suas novas ideias, exercitar suas próprias
preferências e tomar suas próprias decisões. Esse desejo se manifesta na forma de negativismo, a
tendência a gritar "Não!" só para resistir à autoridade. Quase todas as crianças ocidentais exibem algum
grau de negativismo; geralmente começa antes dos 2 anos de idade, com tendência a atingir o máximo
aos 3 anos e meio ou 4 anos e declina por volta dos 6 anos. Cuidadores que consideram as expressões
de autoafirmação da criança como um esforço normal e saudável por independência contribuem para seu
senso de competência e evitam excesso de conflitos.
Surpreendentemente, os "terríveis dois anos" não são universais. Em alguns países em
desenvolvimento, a transição da primeira para a segunda infância é relativamente suave e harmoniosa.

As raízes do desenvolvimento moral: socialização e internalização


Socialização é o processo pelo qual a criança desenvolve hábitos, habilidades, valores e motivações
que as tornam membros responsáveis e produtivos de uma sociedade. A aquiescência às expectativas
parentais pode ser vista como um primeiro passo em direção à submissão aos padrões sociais. A
socialização depende da internalização desses padrões. Crianças bem- sucedidas na socialização não
mais obedecem a regras ou comandos apenas para obter recompensas ou evitar punições; elas fazem
dos padrões da sociedade seus próprios padrões

Contato com outras crianças


Embora os pais exerçam uma grande influência sobre a vida dos filhos, o relacionamento com as
outras crianças - seja dentro de casa ou fora - também é importante já a partir da primeira infância.

Irmãos
O relacionamento entre irmãos desempenha um papel distinto na socialização. Conflitos entre irmãos
podem tornar-se um veículo para a compreensão de relações sociais. Lições e habilidades aprendidas
nas interações com os irmãos são passadas para os relacionamentos fora de casa
É comum os bebês se apegarem a seus irmãos e irmãs mais velhos. Embora a rivalidade possa estar
presente, a afeição também estará. Quanto mais o apego dos irmãos aos pais for um apego seguro,
melhor será o relacionamento entre eles
No entanto, à medida que os bebês tornam-se mais independentes e autoconfiantes, inevitavelmente
entram em conflito com os irmãos - pelo menos na cultura norte-americana. O conflito entre irmãos
aumenta dramaticamente depois que a criança mais nova atinge os 18 meses. Durante os próximos
meses, os irmãos mais novos começam a ter uma participação mais intensa nas interações familiares e
se envolvem com maior frequência nas disputas em família. À medida que isso acontece, eles tornam-se
mais conscientes das intenções e dos sentimentos dos outros. Começam a reconhecer o tipo de
comportamento que vai transtornar ou irritar os irmãos mais velhos e quais os comportamentos
considerados "feios" ou "bons".
À medida que se desenvolve a compreensão cognitiva e social, o conflito entre irmãos tende a se
tornar mais construtivo, e o irmão mais novo participa de tentativas de reconciliação. O conflito construtivo
entre irmãos ajuda as crianças a reconhecerem as necessidades, os desejos e os pontos de vista uns
dos outros, e também ajuda a aprender como brigar, discordar e chegar a um acordo no contexto de um
relacionamento seguro e estável.

Sociabilidade com outras crianças


Bebês e - mais ainda - crianças pequenas mostram interesse em pessoas de fora do círculo familiar,
principalmente pessoas de seu tamanho. Nos primeiros meses, eles olham, sorriem e arrulham para
outros bebês. Dos 6 aos 12 meses, cada vez mais querem tocá-los, além de sorrir e balbuciar para eles.
Por volta de I ano, quando os principais itens de sua agenda são aprender a andar e a manipular objetos,
os bebês prestam menos atenção às outras pessoas. Essa fase, porém, é curta. A partir de
aproximadamente I ano e meio até quase 3 anos de idade, a criança demonstra cada vez mais interesse
no que as outras crianças fazem e uma compreensão cada vez maior de como lidar com elas.

Apostila gerada especialmente para: Betânia Mueller 023.712.580-32


. 306
Crianças pequenas aprendem imitando umas às outras. Brincadeiras como a de seguir o líder ajudam
a estabelecer um vínculo com as outras crianças, preparando-as para brincadeiras mais complexas
durante os anos pré-escolares. A imitação das ações uns dos outros resulta em uma comunicação verbal
mais frequente (algo como "Entre na casinha", "Não faça isso!" ou "Olhe pra mim"), que ajuda os pares a
coordenar atividades conjuntas. A atividade cooperativa desenvolve-se durante o segundo e o terceiro
ano à medida que cresce a compreensão social. Assim como acontece com os irmãos, o conflito também
pode ter um propósito: ajuda a criança a aprender a negociar e a resolver disputas.
Evidentemente, algumas crianças são mais sociáveis que outras, refletindo traços de temperamento
como o seu humor habitual, disposição para aceitar pessoas desconhecidas e capacidade para se
adaptar à mudança. A sociabilidade também é influenciada pela experiência. Bebês que passam algum
tempo com outros bebês, como nas creches, tornam-se sociáveis mais cedo do que aqueles que passam
quase todo o tempo em casa.

A Formação da Criança – Os Laços Afetivos

Podíamos considerar há alguns tempos atrás que o primeiro vínculo de afeto se dava na relação entre
a mãe e o bebê, pelo fato dela ser a figura que gerou e mais do que isso, por ser ela a pessoa que sempre
está presente em sua vida, amamentando, cuidando.
Hoje, esse padrão de mãe acabou sendo um pouco desmistificado, devido a sociedade capitalista que
vivemos entrando em cena a figura dos avós, babás.
Entretanto, algumas atividades são exclusivas da mãe, como por exemplo o leite materno, ninguém
melhor do que ela para saber a sensação que é ter um filho em seus braços, em um contanto íntimo com
o seu corpo e seu interior.

“Todo o processo físico funciona precisamente porque a relação emocional se está desenvolvendo
naturalmente”. (WINNICOTT, 2008, p.33)

Afastar um bebê de sua mãe por qualquer motivo que seja é um dos momentos mais sacrificantes para
os dois, principalmente quando a criança vai tomar uma vacina, o adulto sabe que aquele momento é
para o bem da vida da criança, contudo esta não faz ideia do porquê retirarem-na no colo de sua mãe ou
quem estiver dela cuidando para passar por um grande momento de dor.
O bebê quando chega próximo dos 9 meses, começa a brincar, jogar objetos, segurar e geralmente
faz isso quando tem um adulto por perto, para chamar a atenção e assim pegar para ele. Esse é um bom
momento para tirar a criança do leito materno, pois já demostra a capacidade que o bebê tem em
abandonar as coisas.
O pai também precisa estar presente, vivendo junto com a mãe os momentos de início da vida do
bebê, e segundo Winnicott, se o pai não estiver presente para servir-lhe de alvo, ela detestará a mãe e
isso confundi-la-á, visto ser à mãe que a criança mais profundamente ama.
Se a mãe de repente, não estiver mais presente, seja pelo motivo que for, haverá toda uma mudança
no psicológico dessa criança, pois ela perde aquele sentimento de calma que via na figura da mãe. Se a
mãe retornar novamente ao lar, terá que construir novamente tudo isso em sua cabeça, seu coração,
levando muito tempo para ter esse sentimento.
Crianças que sofrem desilusões tendem a fazer coisas que geralmente não fariam se estivessem
felizes e satisfeitas, como por exemplo pintar as paredes, cortar flores de jardins, entre outras travessuras.
Em alguns momentos, a criança acaba retrocedendo para ver se chama a atenção e recebe novamente
atenção ou pensa que pode crescer fisicamente, sem se preocupar com seu amadurecimento interno. O
fato mais corriqueiro de retroagir está no fato da criança voltar a chupar o dedo.
Porém, o pior momento da ruptura dos laços é a separação dos pais, ainda mais se a criança já
conseguir entender um pouco do que está acontecendo ao seu redor, pensando no que poderá lhe
acontecer de ruim, na saudade de ter os pais juntos para passearem, isso tudo lhe causará fortes danos
mentais.
Infelizmente muitos pais esquecem que apenas se separam do marido/mulher e não do filho, muitos
apenas pagam uma pensão (os que pagam) e acham que isso basta, afeto, amor, não precisa existir para
constituição da felicidade.
Um dos pais ou ambos podem constituir uma nova família, fazendo com que a criança perca vínculos
com a sua família originária (avôs, tios). Além disso, um dos pais prefere morar em outra cidade ou até
mesmo outro país, fazendo com que o filho tenha que conviver com outra cultura, escola, amigos. Ou
seja, toda sua base solidificada acaba sendo destruída.

Apostila gerada especialmente para: Betânia Mueller 023.712.580-32


. 307
Os reflexos desse laço rompido começam a demonstrar sinais com o atraso na aprendizagem escolar,
que não tem mais amigos, não brinca. A criança se encontra fragmentada em todos os aspectos. É um
comportamento característico da criança abalada por completo, o mesmo ocorre quando um dos pais
adoece seriamente ou mesmo na morte de um deles.
Não diremos que a solução esteja em demonstrar para a criança que tenha idade para entender a
separação esteja em ir demonstrando o problema que seus pais estão enfrentando, mas talvez isso possa
ir preparando-a para um futuro rompimento.
Se nada for explicado para a criança, ela sempre vai ter aquela esperança que seus pais irão reatar o
casamento e ficarão “com as portas abertas” esperando pelo retorno daquele que deixou o lar.
Durante o processo de separação as crianças começam a sentir diversos sintomas, como por exemplo,
dor de cabeça, na barriga, medo, pesadelo. Isso acontece porque a criança não consegue se expressar
verbalmente e seu inconsciente repassa esses sentimentos dessa maneira.
Pensando em toda essa problemática enfrentada que temos a guarda e a guarda compartilhada, sendo
esta última, aquela em que a criança tem dois lares a sua disposição. Essas crianças podem crescer com
déficit de atenção, rebeldia, mau comportamento.
Os pais que se separaram e não buscaram outro relacionamento, por culpa exclusiva dos filhos,
acabam fazendo com que eles se tornem eternos bebês, ficando imaturos, sem atuarem no polo ativo,
tornando-se passivos para tudo, sem criatividade e opinião própria.

Como são Construídos os Vínculos Afetivos no Mundo da Criança46

Um adulto normal, mentalmente e emocionalmente saudável, é o que todos desejamos ser. A


construção deste indivíduo começa, segundo Winnicott, desde o nascimento do bebê e seu primeiro
contato com sua mãe. Refiro-me, aqui, exclusivamente, a mãe, mas sei perfeitamente que principalmente
com as grandes mudanças que ocorrem atualmente na sociedade, afetando principalmente as relações
familiares, poderíamos chamar essa “mãe” de cuidador ou cuidadora, simplesmente aquele que cuida do
bebê. Entretanto, como a boa ou a má formação dos vínculos afetivos da criança dependem da dedicação
constante e ininterrupta daquela que cuidará do bebê, Winnicott atribui à mãe quem melhor realizará essa
recompensante tarefa como veremos mais a seguir.
Contudo, acontecimentos fatais durante uma existência nos acometem e para tanto quando uma mãe
vier a faltar, não por negligência, mas por uma fatalidade, o pai ou mesmo a avó ou ainda algum outro
membro da família que se dispuser a tomar conta do bebê que ficou sem a mãe e o fizer com total
dedicação, não poderá ser acusado, inconsequentemente, de não ter cumprido sua tarefa de maneira
que a mãe, ela mesma, tivesse realizado esse feito. Nesse sentido e entendendo da mesma forma que
Winnicott afirmara, será utilizado aqui, a palavra mãe para designar aquela que cuida do bebê, provê seu
sustento físico e emocional.
A mulher está grávida. Antes mesmo que ela tome conhecimento desse novo mundo que surge em
sua vida, o bebê já começa a ganhar vida, forma e percepção das coisas que o cerca e, quando mais
desenvolvido, do mundo exterior ao ventre materno.
É desde o mais remoto início desta existência que esse pequeno ser reage às emoções pelas quais a
mãe passa. Serão nove meses em que o bebê vai se acostumando a ouvir as vozes daqueles que estarão
por perto quando chegar à hora dele nascer. As emoções da mãe não são menos percebidas. Quando a
grávida está tranquila e satisfeita, o bebê sente que seu mundo está em perfeita harmonia. São inúmeras
conexões neurais, enzimas e hormônios liberados na corrente sanguínea que realizam maravilhas na
formação do feto. Calmo, tranquilo e sem agitação, ele se desenvolverá de modo que nada o atrapalhe.
Isso determinará a maneira tranquila que ocorrerá o parto, caso a mãe não possua nenhuma doença pré-
existente; determinará, também, a tranquilidade dos intervalos das mamadas e até a qualidade do sono
do bebê. Durante os nove meses, tudo de aflitivo pode começar a ser trabalhado de modo que tudo ocorra
com tranquilidade e contribua para uma vida boa para o bebê. Um bebê que teve sua gestação na sintonia
de uma perfeita ordem será um bebê que dorme bem, sem interrupções desnecessárias ao sono. Ele
acordará sem estar ansioso ou agitado para mamar. Apenas estará com fome e, então, sem ansiedade,
pegará o seio da melhor maneira ou poderá ser guiado pela mãe, tranquilamente, para que isso ocorra
sem problemas. Será apenas fome e não ânsia por mamar. Sendo assim ele não acorda várias vezes
antes do tempo adequado para se alimentar, como se o leite da mãe não fosse suficiente para saciar sua
fome.
Com a fralda limpa e a fome saciada, a criança tranquila não tem motivos para acordar de um sono
que tem a função de prover tudo o que seu organismo precisa para crescer forte e saudável.

46
GOSS, A. F. G.; RENNES, P. Formação e rompimento dos laços afetivos. UNESP, 2010.

Apostila gerada especialmente para: Betânia Mueller 023.712.580-32


. 308
Todavia, aquele bebê que teve uma mãe agitada, ansiosa, preocupada e, até mesmo, irritada e
nervosa durante toda a sua gestação, terá todos os requisitos para um parto difícil ou mesmo com sérios
problemas. Esse bebê com uma gestação sempre agitada onde a mãe passava a ele todos os hormônios
e enzimas de sua ânsia ou estresse aprendeu a sempre estar ansioso, esperando algo eminente
acontecer. Seu sono será sempre agitado, marcado por despertares repentinos e, então,
consequentemente, o bebê não conseguirá distinguir se nesses intervalos de sono, já é hora de mamar
porque está com fome ou ansioso demais para esperar pelo tempo aproximado entre uma mamada
saudável e outra. Dotado, então, de uma ansiedade extrema, será até mesmo dificultoso o processo de
pegar o seio da mãe de maneira correta de modo a não engolir ar demais e leite de menos, causando,
desse modo, cólicas, e sem a quantidade de leite correta, o que culminará em fome e em despertares
mais frequentes para uma nova mamada.
Sabemos, é claro, que este não seria o único fator contribuinte para o surgimento das cólicas, mas,
sim, o entendemos como um grande determinante e/ ou agravante.
Vimos de maneira simples como o bebê é influenciado desde o ventre materno. Passaremos, então,
para a fase após o seu nascimento, logo nas primeiras mamadas. A mulher que prezava sua liberdade
para trabalhar fora e se divertir nas horas de lazer, vai cedo descobrir que a privação de tudo em benefício
de um pequeno e indefeso ser vai agora reger toda a sua vida. Esse pequeno ser será como escreveu
Melaine Klein “sua majestade, o bebê”. Todas as suas necessidades serão atendidas pela mãe, mesmo
cansada, de maneira pronta e inquestionável. Não importa se a mãe é ou não uma mulher inteligente,
instruída ou tão pouca experiência de vida que possuir. A verdade é que nada disso conta para o fato de
ser, ou não, uma boa mãe. Não é estranho que algo tão importante como ser mãe dependa tão pouco de
uma inteligência? Para que os bebês se convertam, finalmente, em adultos saudáveis, em indivíduos
independentes, mas socialmente preocupados, dependem totalmente de que lhes seja dado um bom
princípio, o qual está segurado, na natureza, pela existência de um vínculo entre a mãe e o seu bebê:
amor é o nome desse vínculo. Portanto, se você ama o seu filhinho, ele estará recebendo um bom
princípio.
Quanto à alimentação do bebê, muito se tem discutido durante a história, visto que nos tempos mais
remotos, acreditava-se que se, por exemplo, o bebê tinha raquitismo ou algum problema intestinal, era
devido ao leite materno não ser bom.
Hoje, com a quantidade de informações que dispomos, sabemos bem que não há nada melhor para a
alimentação do bebê que o leite materno e do ponto de vista nutricional, sabemos que aquela mãe que,
por ventura, tiver algum problema em que ela não possa amamentar o bebê, também não há motivo para
pânico se ela puder comprar um bom leite no mercado. Não há o que temer quanto à nutrição, os leites
disponíveis no mercado possuem toda a sorte de nutrientes que o bebê precisa. Mas seria a alimentação
somente o ato de nutrir?

Não só Winnicott, mas, precisamente ele, discutiu muito a importância da amamentação para a relação
entre a mãe e o bebê. Essa prática é considerada a primeira e mais significativa relação na construção
dos laços afetivos. “Todo o processo físico funciona precisamente porque a relação emocional se está
desenvolvendo naturalmente”.
Imaginem afastar um bebê do contato de sua mãe por alegações de que se precisam fazer exames
ou tomar vacinas ou, ainda, preencher informações desnecessárias naquele precioso momento em que
o bebê se desliga da mãe no nascimento e procura o reencontro de maneira desesperada. O que deveria
ser crucial neste momento?
Nada pode ser mais importante para mãe ou para o bebê que o contato entre mãe e filho. É a relação
mais profunda de amor e, portanto, um dos mais importantes alicerces para a segurança e tranquilidade
do bebê. A amamentação não é um ato mecanizado e para que produza o efeito esperado, não deve ser
mecânico o procedimento realizado, em outras palavras, a amamentação não terá o mesmo efeito se a
enfermeira der a mamadeira desinteressadamente e, após terminar, devolver o bebê para mãe ou ainda
pior: colocá-lo para dormir. Onde estaria o estabelecimento de vínculo afetivo neste ato mecanizado? Ou
mesmo nos hospitais, onde insistem em embrulhar o bebê que não pode usar as mãos para sentir o
contato com sua mãe? Na amamentação todo cenário criado em torno da mãe e seu bebê é importante.
A vivacidade com que a mãe toma o bebê nos braços, o carinho com que o abraça e conversa com ele,
o ato de deixar suas mãozinhas livres para que toque seu seio, seu próprio rosto, esse contato, essa
relação entre mãe e filho, é extremamente importante na construção dos laços afetivos iniciais.
Para que a amamentação se torne, de fato, o primeiro importante passo para a formação dos vínculos
afetivos, a amamentação só poderá ter horários estabelecidos quando essa relação que deve ser
incentivada ocorrer de maneira natural. Se o bebê quiser mamar e sua mãe não o fizer porque ainda não
é a hora, quero dizer, não chegou o próximo horário da mamada (a cada três horas, aproximadamente),

Apostila gerada especialmente para: Betânia Mueller 023.712.580-32


. 309
o bebê sentirá uma grande angústia, uma ansiedade que só é restabelecida se a mãe decidir amamentar
o bebê, quando exigido, por um certo período, voltando aos horários regulares quando possível, sem
pressão. A base de aceitação da realidade externa é o primeiro período em que a mãe obedece às
necessidades do bebê de maneira natural e a aceitação de um mundo exterior a ele começa a se formar
tranquilamente. A alimentação infantil bem sucedida é parte essencial para a educação da criança.
Por outras palavras, a única base autêntica para as relações de uma criança com a mãe e o pai, com
as outras crianças e, finalmente, com a sociedade, consiste na primeira relação bem sucedida entre a
mãe e o bebê, entre duas pessoas, sem que mesmo uma regra de alimentação regular se interponha
entre elas, nem mesmo uma sentença que dite que um bebê deve ser amamentado ao peito materno.
Nos assuntos humanos, os mais complexos só podem evoluir a partir dos mais simples.
Ao cabo dos nove meses, aproximadamente, o bebê inicia suas brincadeiras de atirar as coisas que
tem na mão no chão e espera que sempre tenha um adulto que possa apanhá-las. Essa brincadeira, em
especial, nos mostra que o bebê está apto a se desvencilhar de algumas coisas; se a amamentação teve
êxito, o bebê por si só será capaz de produzir um desmame tranquilo. É a capacidade que o bebê tem de
abandonar as coisas e nos aproveitando disto, é a hora perfeita para introduzir um desmame sem
problemas, sem que pareça obra do acaso, mas uma evolução nos vínculos já estabelecidos. Nesse caso
a amamentação com sucesso produziu, ao longo do período, experiências mais que suficientes para que
o bebê tenha bons sonhos e ótimas recordações, habilitando-o a aceitar os riscos de maneira mais
saudável. O desmame é a demolição gradual das ilusões, que é parte das tarefas que devem ser
realizadas pelos pais.
Quando observamos um bebê que adquire algo que quer, brinca um pouco e depois o abandona,
jogando no chão como é de costume um bebê fazer, concluímos que esta criança foi de uma ponta à
outra da experiência, podendo vivê-la desde o seu começo, seu meio e seu fim e iniciando uma
consciência sobre o tempo total. Quando estamos apressados e ansiosos, não permitimos ao bebê
vivenciar acontecimentos completos e o desenvolvimento e a noção de tempo deste ficam prejudicados.
É essa vivência que permite ao bebê construir a consciência de que o que está em marcha terá um fim.
O meio dos acontecimentos só poderá ser tolerado se houver a ideia de que existe começo e fim. Dessa
forma a mãe propicia a capacidade para o bebê ser capaz de desfrutar todas as experiências.
Essa riqueza de experiências sobre o que o bebê quer fazer ou o que quer segurar ou soltar, não
sabemos precisamente quando começa, visto que podemos observar um bebê de apenas três meses
tentando levar seu dedo ou sua mão ao seio da mãe, mas por volta dos seis meses, quando essas
preferências ficam mais claras, é nesta época que a mãe deve ser precisa sobre o que proibir para que a
criança não fique sem saber seus limites. Sabendo que o bebê colocará tudo à boca, deve a mãe deixar
perto dele somente coisas que ele possa levar à boca, para que toda a ação do bebê para descobrir o
mundo não se transforme numa eterna sequência de “sonoros nãos”, confundindo-o. A mãe, desta forma,
evita que o bebê fique desorientado sobre o que é bom ou mal para se tocar e aos poucos vai dizendo
pequenas falas como “é quente”, “isto corta”, até que ele aprenda o que realmente não deve fazer porque
representa perigo. Sempre que for possível, a mãe deve explicar o porquê das coisas ao seu filho de
maneira a produzir entendimento e não obediência cega às ordens.
Desfrute encontrando o que há para encontrar, à medida que aparece da pessoa que o vosso bebê é,
porque ele precisa isso de você. De modo que você esperará, sem pressa, precipitação ou impaciência,
que o bebê queira brincar. É isso, sobretudo, o que indica a existência de uma vida interior pessoal no
bebê. Se ele encontrar em você uma correspondente disposição lúdica, a riqueza íntima do bebê
desabrochará e as brincadeiras entre a mãe e o bebê tornam-se a melhor parte das relações entre ambos.
Uma evolução onde a família fornece bases para a segurança da criança, sem desvios ou entraves,
proporcionará indivíduos saudáveis emocionalmente.
Quanto ao desenvolvimento da moralidade na criança, o início se dá aos seis meses de vida quando
surge a necessidade de separação do objeto por parte da criança quando o atira ao chão. Inicia-se,
portanto, a capacidade de destruição. É essa a hora em que a mãe tem a oportunidade de integrar os
impulsos de atacar e destruir e de dar e compartilhar, humanizando a moralidade do bebê. Essa
dedicação sem pressa e de maneira humana com que a mãe integra esses sentimentos no bebê vai
formando, gradualmente, na criança a noção de responsabilidade, que a esta época é ainda o sentido de
culpa. Essa fase dura dos seis meses aos dois anos e a mãe por perto para mediar essa relação e essa
confusão de sentimentos desordenados que surgirão, irá definir a ideia, por parte da criança, que ela
pode amar e odiar um objeto ao mesmo tempo.
A criança torna-se gradativamente apta a tolerar o sentimento de angústia (culpa), a respeito dos
elementos destrutivos nas experiências instintivas, porque sabe que haverá uma oportunidade de
recompensar e reconstruir [...] O equilíbrio aí implícito acarreta um sentido de justo e de errado mais
profundo do que quaisquer normas meramente impostas pelos pais.

Apostila gerada especialmente para: Betânia Mueller 023.712.580-32


. 310
Até agora muito da construção da saúde mental de uma criança foi atribuída à mãe, mas somente para
explicar seu papel. Sabemos perfeitamente que o pai constitui um papel não menos importante na
consolidação de todo o amor e dedicação que a mãe vem construindo através de seu primeiro contato
com o bebê que é a amamentação.
Toda mãe sabe que no dia a dia com o bebê muita coisa se aprende e aquilo que possa parecer sem
sentido ou importância para quem está fora, é extremamente interessante para a mãe dividir com seu
companheiro e mesmo ouvir-lhe a opinião, mesmo que ainda não concorde. Quando o bebê cresce, a
riqueza de detalhes aumenta e então os vínculos entre mãe e pai se estreitam cada vez mais.
Quando o pai entra em cena, é esperado pela mãe que o bebê reconheça no pai sentimentos que ele
associa a mãe como carinho, ternura, prontidão, paciência, ou seja, se o pai tiver afinidade nas ações
com a mãe, rapidamente o bebê aceitará esse novo ser em sua vida, e será um grande alívio para mãe
poder compartilhar isso com seu companheiro e seu bebê. No momento em que o pai está em casa e
participa das brincadeiras das crianças, oferece com suas habilidades diferentes recursos para que a
criança possa intercalar em diferentes momentos de suas brincadeiras acrescentando sempre novos
elementos.
Essa não é a única maneira em que o pai se torna importante. Um lar onde há união entre os cônjuges
promove paz e segurança social ao desenvolvimento do bebê e a criança cresce segura e feliz. O pai é
também o ser humano que apoia e sustenta a mãe em sua autoridade com a criança promovendo
estabilidades nas relações triangulares.
A criança está constantemente predisposta a odiar alguém e se o pai não estiver presente para servir-
lhe de alvo, ela detestará a mãe e isso confundi-la-á, visto ser à mãe que a criança mais profundamente
ama.
Quando pai e mãe unem-se na criação do filho está montada a base para um bom lar e, por
conseguinte, para o desenvolvimento normal da criança. Mas o que chamaríamos de uma criança
normal? Seria aquela que nunca se zanga, não demonstra raiva ou frustração ou mesmo não tem seu
momento que os pais chamariam de rebeldia?
Mesmo a criança sadia de corpo e com intelecto excelente, não necessariamente é considerada
normal. O que precisamos saber é se sua personalidade e seu caráter se desenvolvem de maneira
adequada. Se seu desenvolvimento emocional foi comprometido ou houve algum problema, a criança
precisará ter um retrocesso e se comportará como um bebê ou criança menor. Por exemplo, quando a
criança volta a urinar na cama à noite para chamar a atenção dos pais ou afirmar seu direito como
indivíduo no caso de protesto contra a severidade que lhe foi imposta. O mesmo ocorre até mesmo na
fase adulta quando observamos que alguém se comporta como criança birrenta ou ameaça ter um ataque
do coração quando se sente frustrada ou com raiva.
Para um indivíduo normal, há outras maneiras de enfrentar uma frustração. As pessoas precisam
recuperar sentimentos que pertenceram à infância a qualquer custo, devido à intensidade com que foram
vividos, daí a necessidade de regressão à infância sempre que não se consegue resolver os conflitos. Se
esses sentimentos da infância foram bem resolvidos, o ser humano adulto sabe como lidar com os
problemas que tiver à medida que for crescendo.
A criança deve sempre lançar mão de todos os recursos que estiverem ao seu alcance para se proteger
da raiva, da angústia e da frustração. Pior será a criança que bloqueada, não usar desses recursos para
frear seus sentimentos considerados ruins. Enquanto a criança sentir repulsa, raiva e descontrole em face
de algo que a incomoda, ela estará protegida mentalmente. As crianças que os pais exigem obediência
cega, quando chegarem à adolescência, possuirão um sentimento de rebeldia que dificilmente será
controlado.
A esses recursos, normalmente empregados pelas crianças, é que chamamos sintomas, e dizemos
que uma criança normal é capaz de ter qualquer espécie de sintomas, em circunstâncias apropriadas.
Mas com uma criança doente, não são os sintomas que constituem a dificuldade; é o fato de que os
sintomas não estão cumprindo sua tarefa e constituem tanto um incômodo para a criança como para a
mãe.
Veremos, então, como se concretizam as primeiras experiências de independência da criança. O tão
simples bicho de pelúcia de que a criança se apropria e elege como preferido, que todas as pessoas que
cuidam de crianças observam, constitui uma riqueza de elementos para apreender como a criança
elabora suas relações com o mundo exterior a ela e suas relações de independência.
Esse novo objeto pode ser um bicho de pelúcia, pode ser o nó que a criança fica fazendo no paninho
ou o cobertor que ela esfrega no rosto até dormir, entre tantos os tipos que observamos, e que constitui
sua primeira possessão, algo que ela tem afeto e que não pertence a ela como seu dedo ou sua mão. É
o que chamamos de objeto transitório. É sua primeira relação com o outro, com o mundo. Quando isso

Apostila gerada especialmente para: Betânia Mueller 023.712.580-32


. 311
acontece sabemos que tudo vai bem quanto ao desenvolvimento da criança. É o desenvolvimento do
sentido de segurança e da sua relação com algo externo a ela.
É claro que existem crianças que não necessitam deste objeto porque talvez precisem da mãe em
pessoa o tempo todo, ou porque pularam essa parte de transição no desenvolvimento, fato esse que não
quer dizer que a criança não esteja se desenvolvendo de maneira saudável.
Não é o tamanho ou o tipo de objeto escolhido pela criança que importa. O que realmente tem valor
são a textura e, principalmente, o cheiro que esse objeto adquire.
Os pais logo aprendem que não se deve lavar este objeto. Ele ficará sujo e fedido para muitos, mas
há, aí, uma razão especial.
Esse é o objeto criado pela imaginação da criança, é sua primeira criação do mundo e visa supri-la de
afeto e segurança quando a mãe não está por perto, na hora de dormir, entre outras situações de angústia
pelas quais a criança passe. Por esta razão ele não deve ser simplesmente apresentado pela mãe ou por
outro membro da família. Isso rouba da criança a capacidade de criar e então o primeiro sentido do objeto
transitório se perde.
As técnicas usadas pela criança em momentos de separação ou aflição são sem fim. Pode ser o
bichinho que a agrade, pode ser o cobertor ou mesmo uma fralda macia, entendendo, assim, que sempre
esse objeto usado será de textura macia para proporcionar conforto, que é uma das suas funções. Às
vezes pode ser uma extensão da mãe como seu cabelo que a criança enrola e termina por esfregar no
rosto até adormecer ou mesmo o murmurar de alguns sons que podem acalmá-la até que durma.
É a partir do interesse da criança nos objetos transitórios que ela desenvolve a capacidade de cuidar
dos brinquedos e dos animais no futuro.
Em estado saudável, esses fenômenos transitórios evoluem para a capacidade de brincar que é um
sintoma do desenvolvimento saudável da criança. Aos poucos eles simplesmente desaparecem para dar
lugar às brincadeiras, que transitam entre o mundo exterior e o sonho.
Dentre muitos dos problemas que a criança pode manifestar no decorrer da infância como, por
exemplo, acessos de cólera, gritos noturnos, hábitos de asseio, entre outros, está o terrível hábito de
roubar que deixa a mãe extremamente preocupada e sem ação. Até onde é apenas uma fase ou algum
problema na formação dos laços afetivos ocorreu?
É de se esperar que a mãe considere apenas uma fase, toda a boa criança pega suas moedas e se
diverte com elas, ou até mesmo mexe em sua bolsa, vez ou outra, e esparrama tudo. A mãe sabe que
essa é apenas uma fase e até se diverte com isto.
Mas para aquela mãe que já tem outro filho adulto com propensão ao roubo, vê esta mesma cena com
terror nos olhos. Toda sorte de preocupação será transferida para o filho menor no sentido de impedir
que o pior aconteça e a história se repita.
Talvez, será necessário que os pais estabeleçam certas regras para manter o lar em ordem, como é
de costume ouvir que a criança não deve pegar açúcar e outras guloseimas na despensa, sem o
consentimento prévio da mãe.
Entretanto, há a criança que rouba algo e não desfruta daquilo que roubou, como há aquela que rouba
por prazer e aí sim há motivos para uma grande preocupação. Uma criança assim está doente. Ela não
está buscando o objeto roubado, mas a pessoa de quem esse objeto foi roubado.
Quanto às crianças em que o ato de roubar é apenas passageiro, deve haver tolerância por parte dos
pais que tentam passar por essa fase da maneira mais ajuizada possível, e de saber também que pode
ser uma boa hora para estabelecer uma quantia em dinheiro e ensinar a criança a usá-lo com sabedoria.
Os pais que compreendem essa questão não cairão em sermões para com a criança de modo a permitir
que ela comece a mentir criando assim um real problema.
Considerando, assim, tudo o que foi mencionado até agora, constata-se/ baseia-se no princípio de que
conhecer a criança desde o início é estreitar não só os laços de afeto, mas também os laços de
compreensão mútua tanto para entender o choro de um bebê bem como um momentâneo afastamento
do seu filho com os problemas peculiares da adolescência. Se essa relação foi bem construída, desde o
início, será mais fácil entender e ajudar seu filho.
O que é significativo é a experiência individual de desenvolvimento desde bebê a uma criança e um
adolescente, numa família que continua existindo e que se considera capaz de enfrentar os seus próprios
problemas localizados – os problemas do mundo em miniatura. Em miniatura, sim... mas não menor no
que respeita à intensidade de sentimentos e riqueza de experiência, menor apenas na acepção
relativamente secundária da quantidade e complexidade.
Sabemos que a alimentação não é a única maneira de se estabelecer uma relação boa com a criança,
entretanto é a primeira e uma das mais importantes. Sendo assim, se essa relação for bem embasada a
criança será capaz de construir sua relação com a mãe de maneira que essa relação possa superar todas
as frustrações e revezes da vida, até mesmo a perda por separação.

Apostila gerada especialmente para: Betânia Mueller 023.712.580-32


. 312
Dentre as muitas situações ideais já expostas, existem algumas outras situações a serem
consideradas a respeito da construção positiva dos laços afetivos, ou seja, quando estes se alicerçam no
lar ideal.
Uma situação que merece ser observada com muito carinho é o brincar da criança, onde ela exercita
muitas de suas fantasias e também traz o mundo real para ser experimentado e retratado por ela da
maneira que esta concebe o mundo exterior.
Um dos grandes exemplos disso é o “brincar de casinha” ou de “papai e mamãe”. Quando a criança
experimenta o brincar de casinha, ela está recriando o mundo real em que vive. Ela distribuirá as tarefas
de acordo com que são cumpridas em sua própria casa. Haverá um dos cônjuges retratado por um e
outro; outra pessoa será o filho e filha, e assim por diante. Viverão por algumas horas como se fossem
adultos com grandes responsabilidades e deveres, arrumarão a casa e até mesmo manterão uma
estrutura onde “seus filhos” poderão exercer sua própria espontaneidade. Algumas crianças irão recriar
a situação em que vivem se esta não for ideal. Poderão também retratar “uma vida de novela”, aquela
que gostariam de ter no lar se não a tiverem de fato. O importante é que depois de viverem tudo isto como
“adulto”, elas guardarão os brinquedos e irão jantar como crianças normais e gulosas, brigando por um
copo de suco a mais do que o permitido. Se a criança tem um lar adequado, pode continuar a descobrir
sua espontaneidade. Se na vida real elas possuem ótimos pais e não precisam se preocupar com isso,
podem continuar a serem pais e mães eles próprios só nas brincadeiras. “Sabemos que isso é saudável;
se as crianças podem brincar assim juntas, não precisarão mais tarde que lhes ensinem formar um lar.
Já conhecem o essencial.”

A Deficiência na Formação dos Vínculos Afetivos e/ou seu Rompimento

A criança faz-se necessário amor e dedicação para um desenvolvimento adequado. Além disso, ela
vai precisar de um lar estável, onde encontre além de amor e dedicação, um lar com atitudes consistentes,
com rotinas para que a criança saiba construir seu mundo sem surpresas grandes quando ainda não é
hora. A criança cujos pais souberem que para criarem um filho de maneira saudável não é necessário
nenhum profundo conhecimento ou inteligência além do normal, mesmo porque alguém que possuir o
conhecimento em anatomia não vai garantir que seu filho seja saudável, essa pessoa deve saber que
para tanto só é necessário amor profundo e dedicação, atenção total àquele ser que precisa de amor
para se desenvolver e que se isso não lhe faltar, com certeza teremos uma base sólida de construção da
personalidade onde ninguém nem nada no mundo poderão destruir. É exatamente dessa forma que se
formará um indivíduo com perfeita saúde mental para enfrentar as adversidades da vida.
[...] se a mãe não souber ver no filho recém-nascido um ser humano, haverá poucas probabilidades
de que a saúde mental seja alicerçada com uma solidez tal que a criança, em sua vida posterior, possa
ostentar uma personalidade rica e estável, suscetível não só de adaptar-se ao mundo, mas também de
participar de um mundo que exige adaptação.
Importante também se faz a mãe permitir que o filho viva suas experiências salvaguardando seus
direitos dentro de casa. É necessário para início que a criança tenha seu espaço, seu mesmo, onde
ninguém tenha que dizer a ela o que pode fazer ou não ou mesmo como algo deve ser feito. Um espaço
só dela, um canto onde possa brincar sem se preocupar se vai sujar ou bagunçar, um lugar, uma parede
só sua que possa pintar e rabiscar, se expressar sem preocupações. Nesse local reservado, ela se sentirá
com direito e segurança para realizar suas ideias, fantasias, brincar e crescer saudável. Já vimos como
a brincadeira, o brincar é fundamental para a criança. Garante Winnicott que é grande a compensação
quando a mãe permite que a criança tenha seus direitos assegurados, desta forma, no início e, mais
tarde, aumentando os direitos e responsabilidades das crianças, deixando, por exemplo, que a criança
planeje ou ajude a organizar seu aniversário, um passeio ou qualquer outra responsabilidade dada a ela
que a permita viver como indivíduo onde a família confia e, portanto, cresce de maneira maravilhosa.
Podemos também observar que a criança, como já dito anteriormente, precisa de um ambiente estável
e seguro, com rotinas e horários estabelecidos para sua segurança interior. As crianças não gostam de
uma confusão que não cessa, nem da total bagunça que a falta de rotina se caracteriza. Isso faz com que
se sintam inseguras para se desenvolverem, sentindo-se o tempo todo dependente dos pais. Não há
segurança para voarem sozinhas e, portanto, para que se arriscarem? A criança pode então ser lesada
se a mãe não se mostrar preocupada com os seus direitos. A mãe que não é dependente e é
suficientemente confiante de seu papel como mulher e mãe, saberá deixar seu filho ter direitos, crescer
de maneira progressiva, saudável. É claro que o outro extremo, aquela mãe que permite que os filhos
façam tudo como querem, é de igual ou pior modo prejudicial para todos, principalmente para a criança,
que perde sua maior orientação para a vida.

Apostila gerada especialmente para: Betânia Mueller 023.712.580-32


. 313
Outro assunto já mencionado anteriormente é a importância do afeto, também tema central deste
trabalho. Quanto a esse assunto temos a obrigação de levar em consideração a ausência da mãe, seja
por qual motivo for.
A criança pode ficar sem a mãe por algum tempo sem se perceber uma mudança imediata uma vez
que a criança conserva, durante algum tempo, dentro de si a imagem viva da mãe. Mas se a mãe de
quem essa criança depende se tornar ausente por um limite de tempo que excede a capacidade da
criança de conservar dentro de si a imagem viva da mãe, isso fará com que o objeto transitório que a
criança usa para acalmar-se quando a mãe não está por perto, perca seu significado e a criança torna-
se, então, incapaz de usá-lo. Aqui se perdeu toda área intermediária de contato afetivo. Acaso a mãe
retorne ao lar, a criança tem que reconstruir todo o afeto novamente e isso leva um determinado tempo,
para que depois, após confiar novamente na mãe e ter seus vínculos afetivos restabelecidos, essa criança
possa recomeçar a usar os objetos intermediários.
Os roubos por crianças privadas desse contato e que estão recuperando-o podem ser considerados
como fazendo parte da busca de um objeto transitório, que se perde através da morte ou desaparecimento
da versão interiorizada da mãe.
Entretanto, quando a fase do roubo se torna um pouco mais compulsiva, pede tato e atenção por parte
dos pais. No início devem ser tolerantes e depois devem dispensar certa dose de atenção todos os dias.
É necessário que os pais não perguntem o porquê das coisas o tempo todo, pois ao exigirem uma
explicação a qualquer custo, estarão forçando a criança que já rouba a também mentir e então o círculo
vicioso só aumenta e intensifica a ponto de tornar-se de fato um grande problema. A criança não pode
dar a explicação simplesmente porque a ignora e o resultado poderá ser que em vez de sentir uma culpa
insuportável, ela opte sem perceber por mentir compulsivamente.
De qualquer maneira toda a criança que sofreu um desilusionamento está apta a fazer coisas sem um
por que; criam confusões, roubam pequenas coisas, recusam-se a defecar no momento apropriado,
cortam as flores do jardim, pintam as paredes etc.
Falando em retroceder, e isso às vezes faz-se necessário, assistimos por diversas vezes o retorno ao
estado de bebê em crianças mais velhas. Alguma dificuldade impediu a marcha do progresso e, então, a
criança sente necessidade de regressar ao terreno protegido da infância, a fim de restabelecer os direitos
infantis e as leis de desenvolvimento natural segundo Winnicott. Uma das regressões típicas é voltar a
chupar o dedo, fato este que a criança usava como objeto transitório quando bebê.
Como sabemos, o objeto transitório é, segundo Winnicott, a base de toda a vida cultural do ser humano
adulto e faz parte do desenvolvimento emocional normal. Se quando bebê houver uma privação desta
técnica como objeto transitório, tal ocorrência poderá acarretar em inquietação e insônia. Isso faz parte
também da formação do afeto, que se perdida, revela uma criança carente e demonstrará, quando mais
velha, uma tendência a ser antissocial ou até mesmo uma propensão à delinquência.
Outro problema que pode se manifestar devido à falha na formação dos vínculos afetivos é a
deficiência no processo de holding.

O holding é descrito por Winnicott como uma fase em que a mãe ou substituta:
– Protege da agressão fisiológica.
– Leva em conta a sensibilidade cutânea do lactente... e a falta de conhecimento por parte deste da
existência de qualquer coisa que não seja ele mesmo.
– Inclui a rotina completa do cuidado dia e noite adequada a cada bebê e segue também as mudanças
instantâneas do dia-a-dia que fazem parte do crescimento e do desenvolvimento do lactente, tanto físico
quanto psicológico. O holding (segurar) inclui especialmente o holding físico do lactente...”

O desenvolvimento, em poucas palavras, é uma função da herança de um processo de maturação, e


da acumulação de experiências da vida; mas esse desenvolvimento só pode ocorrer num ambiente
propiciador. A importância deste ambiente propiciador é absoluta no início, e a seguir relativa; o processo
de desenvolvimento pode ser descrito em termos de dependência absoluta, dependência relativa e um
caminhar rumo à independência.
Outro aspecto que ainda vale mencionar é a formação do ego. Quando a relação mãe/filho é sadia, o
ego formado é um ego reforçado segundo as palavras de Winnicott. É um ego capaz de desenvolver
defesas e organizar padrões pessoais fortemente desenvolvidos e embasados em padrões hereditários.
É com esse ego muito forte que a criança logo se torna ela mesma. Se o ego da mãe é fraco, o ego da
criança torna-se reativo ao ambiente. A criança não se desenvolve no aspecto pessoal e suas reações
são mais contra os revezes ambientais do que devido a fatores e urgências internas. Como já
mencionado, é um ego reativo e não ativo. Ele simplesmente reage aos fatores ambientais e não age por
condições internas ou fatores genéticos. Os bebês que não recebem um desenvolvimento egóico, são

Apostila gerada especialmente para: Betânia Mueller 023.712.580-32


. 314
bebês propensos à inquietude, estranhamento, apatia, inibição e complacência. É com o desenvolvimento
de um ego forte que as crianças podem começar a formar suas memórias e expectativas.
Quando o bebê percebe que seu objeto impiedosamente atacado é o mesmo objeto amado, surge o
estágio da preocupação.
De acordo com a teoria de Melaine Klein, a criança é agora obrigada a lidar com dois conjuntos de
fenômenos. Uma coisa boa foi atacada e provocou um sentimento satisfatório na criança. Algo de bom
foi formado dentro dela. Mas também tem que lidar com a culpa que emerge deste fato. Surge, então,
uma saída para o problema: a criança torna-se capaz de dar em troca, reparar, consertar, devolver aquilo
que na fantasia dela foi roubado. Quando tudo vai bem, não se desenvolve um sentido exclusivamente
de culpa, mas desenvolve-se outro bem mais importante, um sentido de responsabilidade.
A culpa só deve vir à tona quando o sentido de reparação se torna insuficiente. De acordo com as
ideias de Françoise Dolto, quanto menos existe o sentimento de culpa, mais existe o da responsabilidade
entre os seres humanos. É importante salientar que dificuldades nesse campo, associadas à repressão
de conflitos dolorosos, dão origem a várias manifestações neuróticas e distúrbios de humor de qualidade
psicótico que virão a se tornar material para a loucura propriamente dita.

Problemas e Transtornos de Comportamento Decorrentes da Deficiência da Formação e/ou


Rompimento dos Laços Afetivos

Como já foi mencionado anteriormente, o primeiro e mais persistente vínculo afetivo é o da mãe e seu
filho. É talvez o único vínculo que persiste até a vida adulta, possibilitando afirmação de que a relação
entre mãe e filho, mesmo depois de separados, quando o filho se torna adulto, é o vínculo que nem
mesmo a morte dissocia.
Entretanto de uma forma um tanto paradoxal, é importante salientar que o comportamento do tipo
agressivo desempenha um papel crucial e decisivo na manutenção dos vínculos afetivos. Esse
comportamento assume duas formas distintas: primeiro ataques de afugentamento de intrusos e,
segundo, a punição de um parceiro errante, seja ele esposa, marido ou filho. Há provas de que boa parte
do comportamento agressivo de um tipo desconcertante e patológico tem origem em uma ou outra dessas
formas.
Os vínculos afetivos e os estados emocionais caminham juntos. Sendo assim, muitas das emoções
humanas surgem durante a formação, manutenção e rompimento dos vínculos afetivos.
Em termos subjetivos podemos descrever que a ameaça da perda gera ansiedade e a perda real causa
tristeza, ao passo que ambas as situações podem despertar raiva.
Finalmente, a manutenção incontestada de um vínculo é experimentada como uma fonte de segurança
e a renovação de um vínculo como uma fonte de júbilo.
Portanto, qualquer pessoa interessada em estudar os problemas na formação dos vínculos afetivos de
um indivíduo, vai efetivamente se deparar com distúrbios de personalidade que muito frequentemente
essas pessoas estão sujeitas a desenvolverem.
Para iniciarmos, é comprovadamente produtivo considerar muitos distúrbios psiconeuróticos e de
personalidade nos seres humanos como um reflexo de um distúrbio da capacidade para estabelecer
vínculos afetivos, em virtude de uma falha no desenvolvimento na infância ou de um transtorno
subsequente.
Aqueles que padecem de distúrbios psiquiátricos – psiconeuróticos sociopáticos ou psicóticos –
manifestam sempre uma deterioração da capacidade para estabelecer ou manter vínculos afetivos, uma
deterioração que, com frequência, é grave e duradoura e, em muitos casos, é primária derivando de falhas
no desenvolvimento, que terão ocorrido numa infância vivida num ambiente familiar que não foi propício
ao desenvolvimento do ser humano, permitindo-nos classificá-lo como um lar que não é ideal.
Ao examinarem as possíveis causas dos distúrbios psiquiátricos na infância, ficou constatado que o
problema encontra-se na ausência de oportunidades para estabelecer vínculos afetivos ou, ainda, as
repetidas rupturas dos vínculos que foram estabelecidos.
Foi sistematicamente apurado que duas síndromes psiquiátricas e duas espécies de sintomas
associados são precedidas por uma elevada incidência de vínculos afetivos desfeitos durante a infância.
As síndromes são a personalidade psicopática (ou sociopática) e a depressão; os sintomas persistentes,
a delinquência e o suicídio.
No psicopata, a capacidade de estabelecer e manter os vínculos afetivos são dificultosas ou até
mesmo inexistentes. É constatado que tais indivíduos foram seriamente perturbados na infância pela
morte, separação ou divórcio dos pais ou, ainda, por outros eventos que resultam na deficiência ou ruptura
dos vínculos afetivos. A incidência desses tipos de problemas são maiores nesses grupos do que em
qualquer outro.

Apostila gerada especialmente para: Betânia Mueller 023.712.580-32


. 315
Adotando como critério a ausência da mãe durante seis meses ou mais, antes dos seis anos de idade,
foi apurada uma incidência de 41% para os sociopatas e somente 5% para os restantes. Quando o critério
é ampliado, a incidência aumenta. E, ainda, quando foi adotada a ausência da mãe e do pai antes dos
dez anos como critérios para pesquisa, foi constatado que o índice sobe para 65%.
Outro grupo psiquiátrico que mostra incidência muito alta de perda na infância é a dos pacientes
suicidas. As perdas ocorrem na infância, mais precisamente até os cinco anos de idade, tendo sido
causadas não só pela morte de um dos pais como também por ilegitimidade e o divórcio como nos mostra.
Outra condição, que está associada às perdas na infância, é a depressão. Entretanto, é importante
salientar que essas perdas não se devem frequentemente por ilegitimidade ou divórcio dos pais, mas com
mais incidência por morte de um deles. A orfandade tende a ser maior dos cinco aos dez anos de idade
e em alguns casos no terceiro quinquênio da infância. Segundo as pesquisas, “as indicações são de que
a perda por um dos pais por morte ocorre com frequência duas vezes maior num grupo de depressivos
do que na população em geral”.
Assim, parece agora razoavelmente certo que, em numerosos grupos de pacientes psiquiátricos, a
incidência de rompimento de vínculos afetivos durante a infância é significativamente elevada. [...] As
maiores incidências de vínculos afetivos desfeitos incluem tanto os vínculos com os pais como com as
mães, e são observados entre os cinco e os catorze anos, tanto quanto nos primeiros cinco anos. Além
disso nas condições mais extremas – sociopatia e tendências suicidas – não só é provável que uma perda
inicial tenha ocorrido nos primeiros anos de vida mas também é provável que tenha sido uma perda
permanente, seguida da experiência de repetidas mudanças de figuras parentais.
Há também aquilo que chamamos de efeitos em curto prazo de vínculos desfeitos. Quando uma
criança pequena se vê entre estranhos e longe da figura dos pais, tal fato torna-se motivo de grande
aflição e comprometimento posterior nas relações parentais.
Nas crianças separadas dos pais foram observados dois comportamentos antagônicos. De um lado,
crianças desligadas emocionalmente e, noutro, crianças extremamente dependentes, requisitando
atenção dos pais o tempo inteiro. Em sua maioria as crianças de dois anos que permaneceram desligadas
dos pais por uma ou duas semanas, experimentaram no seu regresso uma atitude distante e desligada
da mãe. No entanto, quando a criança está longe dos pais nos primeiros dias experimenta um grande
desespero em querer a mãe e, às vezes, chora muito sua falta. Quando finalmente regressa parece não
reconhecer e até mesmo evitá-la. Todo o comportamento de busca afetiva está ausente e continuará
assim por um período de tempo, pois essa reaproximação é sempre lenta e gradual e dependerá do
tempo em que durou o desligamento. Quando o vínculo é reatado e o comportamento de ligação se
estabelece, a criança torna-se extremamente ligada a mãe demandando extensa dedicação de sua parte.
Se a mãe, por sua vez, não se demonstra disponível, a criança torna-se muito hostil e com comportamento
negativista, o que não foi notado em crianças que não sofreram separação.
Outro assunto tratado por Bowlby é que o tipo de perda ocorrido durante a infância determina o tipo
de depressão que a pessoa poderá ter na vida adulta.

[...] as mulheres que perderam a mãe por morte ou separação antes dos onze anos de idade, são mais
propensas a reagir à perda, ameaça de perda e outras dificuldades e crises na vida adulta mediante o
desenvolvimento de um distúrbio depressivo do que mulheres que não experimentaram essa perda na
infância. Em segundo lugar, se uma mulher sofreu uma ou mais perdas de membros da família por morte
ou separação antes dos 17 anos de idade, qualquer depressão que se desenvolva subsequentemente é
susceptível de ser mais grave do que uma mulher que não tenha sofrido perdas desse tipo. Em terceiro
lugar, a forma assumida pela perda na infância afeta a forma de qualquer doença depressiva que possa
desenvolver-se mais tarde.
Quando a perda na infância foi devida a separação, é provável que qualquer doença que seja
subsequentemente contraída mostre características de depressão neurótica, com sintomas de ansiedade.
Quando a perda se deve a morte, qualquer doença que se desenvolva subsequentemente poderá
apresentar características de depressão psicótica.
Ansiedade, depressão ou até mesmo suicídio são os tipos mais comuns de problemas atribuídos aos
rompimentos dos laços afetivos. Sabemos que crianças separadas das mães até os primeiros cinco anos
de idade são frequentes em pacientes mais tarde diagnosticados como psicopatas ou sociopatas.
Sabemos, também, que grandes perdas afetivas (poderão acarretar?) acarretarão, mais tarde, problemas
potencialmente perigosos. Entre essas perdas faz-se necessário citar o luto e o pesar na infância.

Apostila gerada especialmente para: Betânia Mueller 023.712.580-32


. 316
Adolescentes47

A busca da identidade A busca da identidade - que Erikson definiu como uma concepção coerente do
self, constituída de metas, valores e crenças com os quais a pessoa está solidamente comprometida -
entra em foco durante os anos da adolescência. O desenvolvimento cognitivo dos adolescentes lhes
possibilita construir uma "teoria do self". Como Erikson enfatizou, o esforço de um adolescente para
compreender o self não é "uma espécie de enfermidade do amadurecimento". Ele faz parte de um
processo saudável e vital fundamentado nas realizações das etapas anteriores - na confiança, autonomia,
iniciativa e produtividade - e lança os alicerces para lidar com os desafios da idade adulta.
Entretanto, uma crise de identidade raramente é totalmente resolvida na adolescência; questões
relativas à identidade surgem repetidamente durante toda a vida adulta.

Erikson: identidade x confusão de identidade


A principal tarefa da adolescência, dizia Erikson, é confrontar a crise de identidade versus confusão
de identidade, ou confusão de identidade versus confusão de papel, de modo a tornar-se um adulto
singular com uma percepção coerente do self e com um papel valorizado na sociedade. O conceito da
crise de identidade baseou-se em parte na experiência pessoal de Erikson. Criado na Alemanha como o
filho bastardo de uma mulher judia dinamarquesa que havia se separado do seu primeiro marido, Erikson
jamais conheceu o pai biológico. Embora tenha sido adotado aos 9 anos de idade pelo segundo marido
de sua mãe, um pediatra judeu alemão, ele se sentia confuso a respeito de quem era. Debateu-se durante
algum tempo antes de encontrar sua vocação. Quando viajou para os Estados Unidos, precisou redefinir
sua identidade como imigrante.
A identidade, segundo Erikson, forma-se quando os jovens resolvem três questões importantes: a
escolha de uma ocupação, a adoção de valores sob os quais viver e o desenvolvimento de uma identidade
sexual satisfatória.
Durante a terceira infância, as crianças adquirem as habilidades necessárias para obter sucesso em
suas respectivas culturas. Quando adolescentes, elas precisam encontrar maneiras de usar essas
habilidades. Quando os jovens têm problemas para fixar-se em uma identidade ocupacional- ou quando
suas oportunidades são artificialmente limitadas -, eles correm risco de apresentar comportamento com
consequências negativas sérias, tal como atividades criminosas.
De acordo com Erikson, a moratória psicossocial, um período de adiamento que a adolescência
proporciona, permite que os jovens busquem compromissos aos quais possam ser fiéis. Os adolescentes
que resolvem essa crise de identidade satisfatoriamente desenvolvem a virtude da fidelidade: lealdade
constante, fé ou um sentimento de integração com uma pessoa amada ou com amigos e companheiros.
Fidelidade também pode ser uma identificação com um conjunto de valores, uma ideologia, uma religião,
um movimento político, uma busca criativa ou um grupo étnico.
A fidelidade é uma extensão da confiança. Na primeira infância, é importante que a confiança nos
outros supere a desconfiança; na adolescência, torna-se importante que a própria pessoa seja confiável.
Os adolescentes estendem sua confiança a mentores e aos entes queridos. Ao compartilhar pensamentos
e sentimentos, o adolescente esclarece uma possível identidade ao vê-la refletida nos olhos do ser
amado. Entretanto, essas intimidades do adolescente diferem da intimidade madura, que envolve maior
compromisso, sacrifício e conciliação.
Erikson via como o principal perigo desse estágio a confusão de identidade ou de papel que pode
atrasar consideravelmente a maturidade psicológica. (Ele não resolveu sua crise de identidade até os 20
e poucos anos.) Algum grau de confusão de identidade é normal. De acordo com Erikson, ela é
responsável pela natureza aparentemente caótica de grande parte do comportamento dos adolescentes
e pela penosa auto consciência deles. Grupos fechados e intolerância com as diferenças, ambos marcas
registradas do cenário social adolescente, são defesas contra a confusão de identidade.
A teoria de Erikson descreve o desenvolvimento da identidade masculina como norma. De acordo com
ele, um homem não é capaz de estabelecer uma intimidade real até ter adquirido uma identidade estável,
enquanto as mulheres se definem através do casamento e da maternidade (algo que talvez fosse mais
verdadeiro na época em que Erikson desenvolveu sua teoria do que na atualidade). Desse modo, as
mulheres (ao contrário dos homens) desenvolvem a identidade por meio da intimidade, não antes dela.
Conforme veremos, essa orientação masculina da teoria de Erikson foi alvo de críticas. Ainda assim, seu
conceito de crise de identidade inspirou muitas pesquisas valiosas.

47
PAPALIA, D. E.; FELDMAN, R. D. Desenvolvimento Humano, 12ª edição, 2013, editor: AMGH.

Apostila gerada especialmente para: Betânia Mueller 023.712.580-32


. 317
Fatores étnicos na formação da identidade
Para muitos jovens de grupos minoritários, a raça ou a etnia é fundamental na formação da identidade.
Um estudo de 940 adolescentes, estudantes universitários e adultos afro-americanos encontrou
evidência de todos os quatro estados de identidade em cada faixa etária. Apenas 27 dos adolescentes
estavam no grupo de identidade realizada, comparado com 47 dos estudantes universitários e 56 dos
adultos. Em vez disso, os adolescentes eram mais propensos a estar na moratória, ainda explorando o
que significa ser afro-americano. Vinte e cinco por cento dos adolescentes estavam em execução, com
sentimentos sobre a identidade afro-americana baseados em sua educação familiar. Os três grupos
(realização, moratória e execução) relataram mais consideração positiva por serem afro americanos do
que os 6 de adolescentes que eram difusos (nem comprometidos nem em processo de exploração).
Aqueles de qualquer idade que estavam no estado realizado eram mais propensos a ver a raça como
central em sua identidade. E, alcançar este estágio de formação da identidade racial tem aplicações
práticas. Embora o efeito seja mais forte para os homens do que para as mulheres, aumentos na
identidade racial do período de um ano foram relacionados com uma diminuição no risco de sintomas
depressivos, mesmo quando fatores como auto estima são controlados.
Outro modelo focaliza-se em três aspectos da identidade racial/étnica: conexão com o próprio grupo
racial/étnico, consciência de racismo e realização incorporada, a crença de que a realização acadêmica
é uma parte da identidade do grupo. Um estudo longitudinal de jovens de grupos rninoritários de baixa
renda revelou que os três aspectos da identidade parecem estabilizar-se e até aumentar ligeiramente na
metade da adolescência. Portanto, a identidade racial/étnica pode atenuar as tendências a uma queda
nas notas e na ligação com a escola durante a transição do ensino fundamental para o ensino médio. Por
outro lado, a percepção de discriminação durante a transição para a adolescência pode interferir na
formação da identidade positiva e levar a problemas de conduta ou a depressão. Como exemplo, as
percepções de discriminação em adolescentes sino-americanos estão associadas com sintomas
depressivos, alienação e queda no desempenho acadêmico. Os fatores de proteção são pais carinhosos
e envolvidos, amigos pró-sociais e desempenho acadêmico forte.
Um estudo longitudinal de 3 anos com 420 adolescentes norte-americanos de ascendência africana,
latina e europeia examinou duas dimensões da identidade étnica: estima do grupo (sentir-se bem em
relação à própria etnia) e exploração do significado da etnia na vida da pessoa. A estima do grupo
aumentou durante a adolescência, especialmente para afro-americanos e latinos, para os quais ela era
mais baixa de início. A exploração do significado da etnia aumentou apenas na metade da adolescência,
talvez refletindo a transição de escolas fundamentais de bairros relativamente homogêneos para escolas
secundárias de etnia mais diversa. As interações com os membros de outros grupos étnicos podem
estimular a curiosidade dos jovens sobre sua própria identidade étnica. A pesquisa verificou que as
meninas parecem passar pelo processo de formação de identidade mais cedo que os meninos. Por
exemplo, um estudo com mais de 300 adolescentes mostrou que durante um período de quatro anos
meninas latinas passaram por exploração, resolução e afirmação de sentimentos positivos em relação a
suas identidades étnicas, enquanto os meninos apresentaram aumentos apenas na afirmação. Este
achado é importante porque o aumento na exploração - que os meninos não demonstraram - era o único
fator ligado a aumento na autoestima.
O termo socialização cultural refere-se a práticas que ensinam as crianças sobre sua herança racial
ou étnica, promovem costumes e tradições culturais e alimentam o orgulho racial/ étnico e cultural. Os
adolescentes que passaram por socialização cultural tendem a ter identidade étnica mais forte e mais
positiva do que aqueles que não a experimentaram.

Sexualidade
Ver-se como um ser sexual, reconhecer a própria orientação sexual, chegar a um acordo com as
primeiras manifestações da sexualidade e formar uniões afetivas ou sexuais, tudo isto faz parte da
aquisição da identidade sexual. A consciência da sexualidade é um aspecto importante da formação da
identidade que afeta profundamente a auto imagem e os relacionamentos. Embora este processo seja
impulsionado biologicamente, sua expressão é, em parte, definida culturalmente.
Durante o século XX, uma mudança importante nas atitudes e no comportamento sexual nos Estados
Unidos e em outros países industrializados trouxe uma aceitação mais generalizada do sexo antes do
casamento da homossexualidade e de outras formas de atividade sexual anteriormente desaprovadas.
Com o acesso difundido à internet, o sexo casual com conhecidos virtuais que se conectam por meio das
salas de bate-papo online ou de sites de encontro de solteiros tornou-se mais comum.
Telefones celulares, e-mail e mensagens instantâneas facilitam que adolescentes solitários arranjem
esses contatos com pessoas anônimas, sem a supervisão dos adultos.

Apostila gerada especialmente para: Betânia Mueller 023.712.580-32


. 318
Velhice48

O envelhecimento econômico de uma população que está envelhecendo depende da proporção de


pessoas saudáveis e fisicamente capazes dessa população. A tendência é encorajadora. Alguns
problemas que eram considerados inevitáveis agora são entendidos como resultantes do estilo de vida
ou doenças, e não do envelhecimento.
O envelhecimento primário é um processo gradual e inevitável de deterioração física que começa cedo
na vida e continua ao longo dos anos, não importa o que as pessoas façam para evita-lo. Assim, o
envelhecimento secundário é uma consequência inevitável de ficar velho. O envelhecimento secundário
resulta de doenças, abusos e maus hábitos, fatores que em geral podem ser comparadas ao conhecido
debate natureza experiência.
Uma classificação mais significativa é a idade funcional, que é a capacidade de uma pessoa interagir
em um ambiente físico e social em comparação com outros da mesma idade cronológica. Podemos
encontrar casos de pessoas com 90 anos de idade que estão mais jovens do que umas com 60 anos, por
estarem bem de saúde.
A senescência é um período marcado por declínios no funcionamento físico associados ao
envelhecimento.
As teorias de programação genética sustentam que o corpo da pessoa envelhece de acordo com o
relógio evolutivo normal inato dos genes. Estudos sobre gêmeos constataram que as diferenças genéticas
são responsáveis por aproximadamente um quarto da variância no tempo de vida adulto humano. Essa
influência genética é mínima antes dos 60, mas aumenta depois dessa idade.
Algumas mudanças físicas costumam estar associadas ao envelhecimento, sendo óbvias para um
observador causal, embora afetem mais algumas pessoas do que outras. A pele mais velha tende a se
tornar mais pálida e menos elástica; e assim como a gordura e os músculos encolhem, a pele fica
enrugada. São comuns varizes nas pernas. O cabelo fica mais fino, grisalho e depois branco, e os pelos
do corpo tornam-se mais ralos.
O envelhecimento no cérebro varia muito de uma pessoa para outra. O cérebro pode reorganizar os
circuitos neuronais para responder ao desafio do envelhecimento neurobiológico.
Há uma diminuição na quantidade ou densidade do neurotransmissor dopamina devido à perda de
sinapses. Os receptores de dopamina são importantes na medida em que ajudam a regular a atenção.
Nem todas as manifestações no cérebro são destrutivas. Os pesquisadores descobriram que cérebros
mais velhos podem criar novas células nervosas a partir de células-tronco – algo impensado no passado.

Principais problemas comportamentais e mentais


Muitos idosos com problemas comportamentais e mentais tendem a não procurar ajuda. Alguns desses
problemas são intoxicação medicamentosa, delírio, transtornos metabólicos, baixo funcionamento da
tireoide, pequenos ferimentos da cabeça, alcoolismo e depressão.
A depressão está associada a outros problemas de saúde. Pelo fato de a depressão estar associada
a outros problemas de saúde, um diagnóstico preciso, prevenção e tratamento adequado podem ajudar
pessoas idosas a viverem mais tempo e permanecerem mais ativas. A depressão pode ser tratada com
drogas, psicoterapia, antidepressivos.
O Mal de Alzheimer é uma das mais comuns e mais temidas doenças terminais entre as pessoas
idosas. Gradualmente, rouba dos pacientes a inteligência, a consciência e até mesmo a habilidade de
controlar as funções de seu corpo, e finalmente os mata. Os sintomas clássicos do mal de Alzheimer são
a diminuição da capacidade de memória, deterioração da linguagem e deficiências no processamento
espacial e visual. O principal sintoma é incapacidade de lembrar acontecimento recente ou absorver
novas informações.
A memória episódica é particularmente vulnerável aos efeitos do envelhecimento; efeitos que são
agrados à medida que as tarefas da memória tornam-se mais complexas ou exigentes, ou requerem a
livre recordação de informações, em oposição ao reconhecimento de material previamente visto.

Questões

01. (Pref. de Quixadá/CE - Psicólogo - ACEP). Sobre as fases do desenvolvimento humano e os


fatores psicológicos do desenvolvimento, assinale a alternativa CORRETA.

48
PAPALIA, D. E.; FELDMAN, R. D. Desenvolvimento Humano, 12ª edição, 2013, editor: AMGH.

Apostila gerada especialmente para: Betânia Mueller 023.712.580-32


. 319
(A) Freud abordou a construção das estruturas mentais do pensamento pautadas na epistemologia
genética.
(B) A interação e a socialização, segundo Piaget, são mecanismos importantes que favorecem a
autorregulação do ser humano.
(C) No Construtivismo, o período da criança entre 02 a 07 anos caracteriza-se como estágio pré-
operacional.
(D) Vygotsky associou o desenvolvimento da inteligência da criança aos processos de assimilação e
acomodação.

02. (IFB - Psicólogo - FUNIVERSA). Durante o processo de ensino e de aprendizagem, o lúdico


contribui para a construção de várias funções no desenvolvimento psicológico da criança. Em relação ao
papel do lúdico no desenvolvimento infantil, especialmente na educação infantil e no ensino fundamental,
assinale a alternativa correta.
(A) Atividades lúdicas são desaconselháveis dentro da sala de aula, pois promovem a indisciplina e o
descontrole da organização do trabalho pedagógico.
(B) São prejudiciais ao desenvolvimento mental da criança atividades que promovem fantasia e fuga
da realidade durante as brincadeiras.
(C) Para estimular, condicionar e controlar processos psicológicos complexos, as atividades lúdicas
adequadas são exclusivamente as que usam brinquedos pedagógicos.
(D) Atividades lúdicas devem fazer parte dos processos de ensino e de aprendizagem para
favorecerem a mediação simbólica entre a realidade e o desenvolvimento da subjetividade da criança.
(E) As brincadeiras de imitação, por não interferirem nos desenvolvimentos afetivo, cognitivo e
psicanalítico, são as mais importantes para a aprendizagem da criança, pois determinam como ela irá se
adaptar aos limites do mundo e dos papéis sociais das suas relações parentais.

03. (FUNTELPA - Psicólogo - IDECAN). Sobre o desenvolvimento psicológico, Vygotsky afirma que
“A internalização de formas culturais de comportamento envolve a reconstrução da atividade
psicológica...” Tal afirmativa denota que:
(A) O uso de signos externos é também radicalmente reconstruído.
(B) Os processos psicológicos permanecem tal como aparecem nos animais.
(C) As mudanças nas operações linguísticas são tímidas.
(D) A fala egocêntrica se fortalece fazendo surgir a fala externa.
(E) Deixam de ser internalizadas as atividades sociais e históricas.

04. (Pref. de Fortaleza/CE- Psicólogo - 2016) Ao se falar de zona de desenvolvimento proximal, está-
se referindo à teoria de desenvolvimento de:
(A) Henri Wallon.
(B) Lev Vygotsky.
(C) Carl Gustav Jung.
(D) Jean Piaget.

05. (CEFET/RJ- Psicólogo- CESGRANRIO) A posição de Vygotsky sobre a relação entre


desenvolvimento e aprendizagem é que o
(A) desenvolvimento é dependente da maturação e condiciona o aprendizado.
(B) desenvolvimento é definido como a substituição de respostas inatas a partir do aprendizado.
(C) aprendizado e o desenvolvimento são coincidentes e contemporâneos.
(D) aprendizado alavanca o desenvolvimento devido ao estabelecimento das relações sociais.
(E) processo de desenvolvimento da criança é independente do aprendizado.

06. (MPE/ES - Agente Técnico - VUNESP) Para Jean Piaget, o processo de assimilação
(A) é um mecanismo automático e determinado biologicamente, no qual o indivíduo é relativamente
passivo.
(B) envolve a eliminação de esquemas anteriores do indivíduo decorrente da aquisição de novas
informações.
(C) equivale ao processo de adaptação, porque este implica, necessariamente, revisão de
conceitos diante de novas situações.
(D) está diretamente relacionado ao processo de linguagem e, consequentemente, à socialização.
(E) corresponde a uma integração de novas informações a estruturas prévias do indivíduo.

Apostila gerada especialmente para: Betânia Mueller 023.712.580-32


. 320
07. Winnicott entende que um dos primeiros momentos dos laços afetivos ocorrem no momento da
amamentação
( ) Certo ( ) Errado

08. O momento certo para desmamar o bebê ocorre quando ele começa a iniciar suas brincadeiras.
Atirando seus brinquedos, pois demonstra que já está chegando o momento do desapego.
( ) Certo ( ) Errado

09. Winnicott descreve o holding como uma fase em que a mãe ou substituta protege da agressão
fisiológica, – Inclui a rotina completa do cuidado dia e noite adequada a cada bebê e segue também as
mudanças instantâneas do dia-a-dia que fazem parte do crescimento e do desenvolvimento do lactente,
tanto físico quanto psicológico, além de proteger da agressão fisiológica.
( ) Certo ( ) Errado

Gabarito

01.C / 02.D / 03.A / 04.B / 05.D / 06.E / 07. certo / 08.certo / 09.certo

Comentários

01. Resposta: C
Para Piaget, os modos de relacionamento com a realidade são divididos em 4 períodos distintos, no
processo evolutivo da espécie humana que são caracterizados "por aquilo que o indivíduo consegue fazer
melhor" no decorrer das diversas faixas etárias ao longo do seu processo de desenvolvimento. São eles:
-1º período: Sensório-motor (0 a 2 anos)
- 2º período: Pré-operatório (2 a 7 anos)
- 3º período: Operações concretas (7 a 11 ou 12 anos)
- 4º período: Operações formais (11 ou 12 anos em diante)

02. Resposta: D
O brinquedo é um mundo imaginário onde a criança pode realizar seus desejos, o ato de brincar é uma
importante fonte de promoção de desenvolvimento, sendo muito valorizado na zona proximal, neste caso
em especial as brincadeiras de ‘faz de conta’. Sendo estas atividades utilizadas, em geral, na Educação
Infantil fase que as crianças aprendem a falar (após os três anos de idade), e são capazes de envolver-
se numa situação imaginária. Através do imaginário a criança estabelece regras do cotidiano real.

03. Resposta: A
A teoria vygotskyana é instrumental, histórica e cultural. É instrumental, por se referir à natureza
mediada das funções psicológicas superiores. Diferentemente dos animais, que mantém relação direta
com a natureza, o processo de hominização surge com o trabalho, que inaugura a mediação com o uso
de signos e instrumentos, permitindo a modificação do psiquismo humano e da realidade externa,
respectivamente. Em um movimento dialético, os seres humanos criam novos cenários, que determinam
novos atores, novos papéis.

04. Resposta: B
Um dos princípios básicos da teoria de Vygotsky é o conceito de "zona de desenvolvimento próximo".
Zona de desenvolvimento próximo representa a diferença entre a capacidade da criança de resolver
problemas por si própria e a capacidade de resolvê-los com ajuda de alguém.

05. Resposta: D
(A) INCORRETA – O aprendizado se dá através da relação do sujeito com o meio e impulsiona o
desenvolvimento.
(B) INCORRETA – Para Vygotsky o desenvolvimento se estabelece a partir da aprendizagem e pelas
relações sociais, esse é o eixo fundamental de sua teoria.
(C) INCORRETA - a aprendizagem ocorre antes do desenvolvimento portanto, não são coincidentes.
(D) CORRETA - é pela mediação (relação social) que ocorre a aprendizagem e,
consequentemente, o desenvolvimento
(E) INCORRETA - O desenvolvimento só é possível através da aprendizagem

Apostila gerada especialmente para: Betânia Mueller 023.712.580-32


. 321
06. Resposta: E
Assimilação: refere-se ao processo de receber as informações e incorporar às estruturas cognitivas já
existentes do sujeito.
Acomodação: refere-se ao processo de mudança nos esquemas existentes para incluir esse novo
conhecimento.
A equilibração é a tendência em mantar as estruturas cognitivas em equilíbrio e é o aspecto que vai
determinar a mudança da assimilação para a acomodação.

07. Resposta: Certo


Não só Winnicott, mas, precisamente ele, discutiu muito a importância da amamentação para a relação
entre a mãe e o bebê. Essa prática é considerada a primeira e mais significativa relação na construção
dos laços afetivos. “Todo o processo físico funciona precisamente porque a relação emocional se está
desenvolvendo naturalmente”.

08. Resposta: Certo


Ao cabo dos nove meses, aproximadamente, o bebê inicia suas brincadeiras de atirar as coisas que
tem na mão no chão e espera que sempre tenha um adulto que possa apanhá-las. Essa brincadeira, em
especial, nos mostra que o bebê está apto a se desvencilhar de algumas coisas; se a amamentação teve
êxito, o bebê por si só será capaz de produzir um desmame tranquilo. É a capacidade que o bebê tem de
abandonar as coisas e nos aproveitando disto, é a hora perfeita para introduzir um desmame sem
problemas, sem que pareça obra do acaso, mas uma evolução nos vínculos já estabelecidos. Nesse caso
a amamentação com sucesso produziu, ao longo do período, experiências mais que suficientes para que
o bebê tenha bons sonhos e ótimas recordações, habilitando-o a aceitar os riscos de maneira mais
saudável. O desmame é a demolição gradual das ilusões, que é parte das tarefas que devem ser
realizadas pelos pais.

09. Resposta: certo


O holding é descrito por Winnicott como uma fase em que a mãe ou substituta:
– Protege da agressão fisiológica.
– Leva em conta a sensibilidade cutânea do lactente... e a falta de conhecimento por parte deste da
existência de qualquer coisa que não seja ele mesmo.
– Inclui a rotina completa do cuidado dia e noite adequada a cada bebê e segue também as mudanças
instantâneas do dia-a-dia que fazem parte do crescimento e do desenvolvimento do lactente, tanto físico
quanto psicológico. O holding (segurar) inclui especialmente o holding físico do lactente...”

O papel do psicólogo em equipe multidisciplinar.

Atuação na equipe interdisciplinar49.

O trabalho de assistentes sociais, psicólogos/as e pedagogos/as, que constituem as principais


profissões hoje atuantes no SUAS, requer interface com as políticas da saúde, Previdência, educação,
trabalho, lazer, meio ambiente, comunicação social, segurança e habitação, na perspectiva de mediar o
acesso dos(as) cidadãos(ãs) aos direitos sociais.
As abordagens das profissões podem somar-se com intuito de assegurar uma intervenção
interdisciplinar capaz de responder a demandas individuais e coletivas, com vistas a defender a
construção de uma sociedade livre de todas as formas de violência e exploração de classe, gênero, etnia
e orientação sexual. Ao integrar a equipe dos(as) trabalhadores(as) no âmbito da política de Assistência
Social, os(as) profissionais podem contribuir para criar ações coletivas de enfrentamento a essas
situações, com vistas a reafirmar um projeto ético e sócio-político de uma nova sociedade que assegure
a divisão equitativa da riqueza socialmente produzida.
Dessa forma, o trabalho interdisciplinar em equipe deve ser orientado pela perspectiva de totalidade,
com vistas a situar o indivíduo nas relações sociais que têm papel determinante nas suas condições de
vida, de modo a não responsabilizá-lo pela sua condição socioeconômica.

49
Parâmetros para a Atuação de assistentes Sociais na Política de Assistência Social.

Apostila gerada especialmente para: Betânia Mueller 023.712.580-32


. 322
A construção do trabalho interdisciplinar impõe aos(às) profissionais a realização permanente de
reuniões de planejamento e debates conjuntos a fim de estabelecer as particularidades da intervenção
profissional, bem como definir as competências e habilidades profissionais em função das demandas
sociais e das especificidades do trabalho. Balizados pelos seus Códigos de Ética, Leis de
Regulamentação e Diretrizes Curriculares de formação profissional, os(as) profissionais podem instituir
parâmetros de intervenção que se pautem pelo compartilhamento das atividades, convivência não
conflituosa das diferentes abordagens teórico metodológicas e estabelecimento do que é próprio e
específico a cada profissional na realização de estudos socioeconômicos, visitas domiciliares,
abordagens individuais, grupais e coletivas.
A atuação interdisciplinar requer construir uma prática político profissional que possa dialogar sobre
pontos de vista diferentes, aceitar confrontos de diferentes abordagens, tomar decisões que decorram de
posturas éticas e políticas pautadas nos princípios e valores estabelecidos nos Códigos de Ética
Profissional. A interdisciplinaridade, que surge no processo coletivo de trabalho, demanda uma atitude
ante a formação e conhecimento, que se evidencia no reconhecimento das competências, atribuições,
habilidades, possibilidades e limites das disciplinas, dos sujeitos, do reconhecimento da necessidade de
diálogo profissional e cooperação.

Exemplo:
Embora Serviço Social e Psicologia, principais profissionais hoje inseridos no SUAS, possuam
acúmulos teórico-políticos diferentes, o diálogo entre essas categorias profissionais aliará reflexão crítica,
participação política, compreensão dos aspectos objetivos e subjetivos inerentes ao convívio e à formação
do indivíduo, da coletividade e das circunstâncias que envolvem as diversas situações que se apresentam
ao trabalho profissional. É possível construir, a partir dessa ação interdisciplinar, um cenário de discussão
sobre responsabilidades e possibilidades na construção de uma proposta ético-política e profissional que
não fragmente o sujeito usuário da política de Assistência Social. O trabalho em equipe não pode
negligenciar a responsabilidades individuais e competências, e deve buscar identificar papéis, atribuições,
de modo a estabelecer objetivamente quem, dentro da equipe multidisciplinar, encarrega-se de
determinadas tarefas.

O conhecimento da legislação social é um pré-requisito para o exercício do trabalho. No caso do


Serviço Social, esta é uma matéria obrigatória prevista nas Diretrizes Curriculares. A atualização do
conhecimento dos marcos legais, contudo, é uma necessidade contínua de todos(as) os(as)
trabalhadores(as) e deve ser buscada conjuntamente pelas equipes do SUAS.
A consolidação do processo coletivo de trabalho de assistentes sociais na política de Assistência Social
não está desvinculada das lutas pela garantia de um Estado democrático, comprometido com os direitos
da classe trabalhadora. Isso porque a intervenção profissional não se realiza e nem pode ser tratada
como responsabilidade individual dos(as) trabalhadores(as).

Vamos diferenciar a Interdisciplinaridade e multidisciplinaridade

- Multidisciplinaridade: corresponde ao nível mais baixo e integração. Caracteriza-se como uma


justaposição de disciplinas com a intenção de esclarecer alguns de seus elementos comuns.
- Interdisciplinaridade: reúne estudos diferenciados de diversos especialistas em um contexto
coletivo de pesquisa. Implica um esforço por elaborar um contexto mais geral, no qual cada uma das
disciplinas é modificada e passa a depender cada qual das demais. A interação proporcionará um
enriquecimento recíproco, com transformações em diferentes aspectos, como, por exemplo, nas suas
metodologias de pesquisa, nos seus conceitos, na formulação dos problemas, nos instrumentos de
análise, nos modelos teóricos, etc. Os intercâmbios entre as disciplinas são mútuos. A bioquímica, a
sociolinguística, as neurociências são áreas do conhecimento resultantes de trabalhos interdisciplinares.

Atuação do psicólogo em equipe multiprofissional50

A inserção do Psicólogo em empresas tem aumentado consideravelmente nos últimos anos, seja em
empresas públicas, privadas ou de outras modalidades. Aliado a isso, em muitas dessas empresas o
psicólogo atua numa equipe multiprofissional, sendo requisitado a trabalhar de forma integrada com
profissionais de outras áreas, como médicos, enfermeiros, assistentes sociais, pedagogos, etc.

50
Conselho Regional de Psicologia 9ª Região GO. 2015.

Apostila gerada especialmente para: Betânia Mueller 023.712.580-32


. 323
Diante dessa crescente inserção do psicólogo em instituições e em equipes multiprofissionais a
Comissão Permanente de Orientação e Fiscalização do Conselho Regional de Psicologia tem verificado
junto à categoria novos desafios a serem superados. Dentre eles, destaca-se a defesa de uma prática
que priorize o respeito adequado da relação de trabalho do psicólogo com seus pares, com outros
profissionais e com o usuário do serviço de psicologia prestado nos diversos órgãos e instituições.
Neste contexto, a COF ressalta que é imprescindível que as práticas adotadas em resposta as atuais
demandas sociais atendam aos padrões éticos e técnicos definidos na legislação que regulamenta a
profissão de Psicólogo.
Para tanto, dentre as diversas responsabilidades previstas no Código de Ética Profissional do
Psicólogo (CEPP), no que se refere a atuação nas diversas modalidades de instituições, destaca-se os
seguintes artigos:

Art. 1º - São deveres fundamentais dos psicólogos:


a. Conhecer, divulgar, cumprir e fazer cumprir este Código;
b. Assumir responsabilidades profissionais somente por atividades para as quais esteja capacitado
pessoal, teórica e tecnicamente;
c. Prestar serviços psicológicos de qualidade, em condições de trabalho dignas e apropriadas à
natureza desses serviços, utilizando princípios, conhecimentos e técnicas reconhecidamente
fundamentados na ciência psicológica, na ética e na legislação profissional; (...)
j. Ter, para com o trabalho dos psicólogos e de outros profissionais, respeito, consideração e
solidariedade, e, quando solicitado, colaborar com estes, salvo impedimento por motivo relevante;(...)

Art. 2º - Ao psicólogo é vedado: (...)


a. Praticar ou ser conivente com quaisquer atos que caracterizem negligência, discriminação,
exploração, violência, crueldade ou opressão;
b. Induzir a convicções políticas, filosóficas, morais, ideológicas, religiosas, de orientação sexual ou a
qualquer tipo de preconceito, quando do exercício de suas funções profissionais;
c. Utilizar ou favorecer o uso de conhecimento e a utilização de práticas psicológicas como
instrumentos de castigo, tortura ou qualquer forma de violência;
d. Acumpliciar-se com pessoas ou organizações que exerçam ou favoreçam o exercício ilegal da
profissão de psicólogo ou de qualquer outra atividade profissional; (...)

Art. 3º - O psicólogo, para ingressar, associar-se ou permanecer em uma organização, considerará a


missão, a filosofia, as políticas, as normas e as práticas nela vigentes e sua compatibilidade com os
princípios e regras deste Código.
Parágrafo único: Existindo incompatibilidade, cabe ao psicólogo recusar-se a prestar serviços e, se
pertinente, apresentar denúncia ao órgão competente. (...)

Art. 6º - O psicólogo, no relacionamento com profissionais não psicólogos:

a. Encaminhará a profissionais ou entidades habilitados e qualificados demandas que


extrapolem seu campo de atuação;

Conforme determinado nos artigos expostos acima, destaca-se que cabe ao psicólogo que integra uma
equipe multiprofissional realizar somente atividades que estejam embasadas nos conhecimentos técnicos
reconhecidos e fundamentados na ciência psicológica, na ética e na legislação profissional. Portanto, tais
psicólogos devem ser cautelosos ao serem solicitados a colaborarem com outros profissionais, tendo
sempre em foco que não podem assumir atividades que sejam privativas de outra profissão. Assim, caso
recebam demandas que extrapolem seu campo de atuação, deverão encaminhá-las para o profissional
ou instituição competente.
A título de exemplo, no caso de instituições que possuam equipes multiprofissionais constituídas por
psicólogos, enfermeiros, assistentes sociais ou pedagogos, os psicólogos não poderão ministrar
medicamentos, que se constitui em atividade privativa de enfermeiros, técnicos e auxiliares de
enfermagem, farmacêuticos e médicos. Neste exemplo, o profissional psicólogo que administrar
medicamento estará incorrendo no exercício ilegal de profissão e poderá responder a processo ético no
Conselho de Classe, bem como ser denunciado aos órgãos competentes. Por fim, ainda neste caso
hipotético, caso o psicólogo tome conhecimento do exercício ilegal de profissão por um psicólogo ou outro
profissional, deverá apresentar denúncia ao órgão competente.

Apostila gerada especialmente para: Betânia Mueller 023.712.580-32


. 324
Questões

01. SAP-SP- Psicólogo - VUNESP) O psicólogo, no trabalho com equipes multiprofissionais,


(A) trabalha e intervém em situações relacionadas ao atendimento feito por diversos profissionais.
(B) só intervém em casos de outros profissionais da mesma área de conhecimento que a sua.
(C) atua em qualquer caso atendido pela equipe, desde que autorizado pela chefia médica dessa
equipe.
(D) concentra sua intervenção nos problemas emocionais identificados nos membros da equipe.
(E) pode substituir qualquer membro da equipe, em relação a qualquer procedimento adotado por essa
equipe.

02. (DPE-RJ- Técnico Superior Especializado- FGV) Um trabalho multidisciplinar é aquele em que
uma equipe de profissionais de diferentes áreas
(A) discute permanentemente suas práticas em busca de soluções para problemas complexos.
(B) mantém espaços de diálogo permanente que permitam a recomposição de trabalhos parcelares.
(C) elabora projetos conjuntos de ação para enfrentamento de situações, ainda que atuem
individualmente
(D) reúne-se com periodicidade sistemática para discutir casos e processos institucionais.
(E) estabelece uma ação coletiva no planejamento e na execução de suas intervenções.

Gabarito

01.A / 02.B

Comentários

01. Resposta: A
Nas equipes multiprofissionais os profissionais de diversas áreas distintas trabalham de forma
integrada.

02. Resposta: B
Trabalho multidisciplinar reúne diferentes profissionais que por seus ramos distintos podem até
fracionar o trabalho, mas que ao final se juntam para construir um todo.

Apostila gerada especialmente para: Betânia Mueller 023.712.580-32


. 325

Você também pode gostar